You are on page 1of 717

D. A.

Evans

An Introduction to Frontier Molecular Orbital Theory-1


Problems of the Day

Chem 206

http://www.courses.fas.harvard.edu/~chem206/ http://evans.harvard.edu/problems/

Chemistry 206 Advanced Organic Chemistry


Lecture Number 1

The molecule illustrated below can react through either Path A or Path B to form salt 1 or salt 2. In both instances the carbonyl oxygen functions as the nucleophile in an intramolecular alkylation. What is the preferred reaction path for the transformation in question? O O Br N H O Br
Path B Path A 1

Br

N O H Br O

N O H Br

Br

Introduction to FMO Theory


General Bonding Considerations The H2 Molecule Revisited (Again!) Donor & Acceptor Properties of Bonding & Antibonding States Hyperconjugation and "Negative" Hyperconjugation Anomeric and Related Effects

This is a "thought" question posed to me by Prof. Duilo Arigoni at the ETH in Zuerich some years ago

(First hr exam, 1999)


The three phosphites illustrated below exhibit a 750fold span in reactivity with a test electrophile (eq 1) (Gorenstein, JACS 1984, 106, 7831).
(RO)3P OMe O P O A O

Reading Assignment for week:


Kirby, Stereoelectronic Effects Carey & Sundberg: Part A; Chapter 1 Fleming, Chapter 1 & 2 Fukui,Acc. Chem. Res. 1971, 4, 57. (pdf) Curnow, J. Chem. Ed. 1998, 75, 910 (pdf) Alabugin & Zeidan, JACS 2002, 124, 3175 (pdf) D. A. Evans
1-01-Cover Page 9/15/03 8:56 AM

El(+)
O P O B

+ (RO)3PEl

(1)

O P OMe O C

Rank the phosphites from the least to the most nucleophilic and provide a concise explanation for your predicted reactivity order.

Monday, September 15, 2003

D. A. Evans

An Introduction to Frontier Molecular Orbital Theory-1


Stereoelectronic Effects

Chem 206

Universal Effects Governing Chemical Reactions There are three: Steric Effects
Nonbonding interactions (Van der Waals repulsion) between substituents within a molecule or between reacting molecules
Me S N2 Me Nu C R RO H H Me Me2CuLi RO Me H H R

Geometrical constraints placed upon ground and transition states by orbital overlap considerations. Fukui Postulate for reactions:
"During the course of chemical reactions, the interaction of the highest filled (HOMO) and lowest unfilled (antibonding) molecular orbital (LUMO) in reacting species is very important to the stabilization of the transition structure."

Nu:

C R

Br

Br:

RO H

major

General Reaction Types


O

p. p.

minor

Radical Reactions (~10%): Polar Reactions (~90%):

A + B A(:) + B(+)
Lewis Acid

A A

B B

Electronic Effects (Inductive Effects):


The effect of bond and through-space polarization by heteroatom substituents on reaction rates and selectivities Inductive Effects: Through-bond polarization Field Effects: Through-space polarization

Lewis Base

FMO concepts extend the donor-acceptor paradigm to non-obvious families of reactions Examples to consider

Me R C R Br

S N1

+
R R C Me + Br:

H2

2 Li(0) Mg(0)

2 LiH CH3MgBr

CH3I +

rate decreases as R becomes more electronegative "Organic chemists are generally unaware of the impact of electronic effects on the stereochemical outcome of reactions." "The distinction between electronic and stereoelectronic effects is not clear-cut."

1-02-Introduction-1

9/12/03 4:44 PM

D. A. Evans

Steric Versus Electronic Effects; A time to be careful!!


O R3SiO EtO O R3Si OSiR3 O TiCl4 Nu OSiR3

Chem 206

Steric Versus electronic Effects: Some Case Studies


When steric and electronic (stereoelectronic) effects lead to differing stereochemical consequences
Woerpel etal. JACS 1999, 121, 12208.

diastereoselection >94:6 OSiR3 H O diastereoselection 93:7 H OSiR3

OAc

SnBr4 Me SiMe3

O Me

Me

AlCl3

stereoselection 99:1 stereoselection >95:5 OSiR3

OAc

SnBr4 BnO

O Danishefsky et al JOC 1991, 56, 387 BnO

p. p.

BnO

O EtO2C O EtO2C (R)2CuLi Bu OTBS O OTBS diastereoselection 8:1 O Ph N O only diastereomer Bu OTBS OAc OAc O H Ph N O H N N N N O OAc OAc H H N R R O TBS Al R O Yakura et al Tetrahedron 2000, 56, 7715 O Ph 60-94% AcO AcO H H N O only diastereomer Ph

Bu3Al
R3 O Al O

EtO2C

EtO

Yakura's rationalization:

Mehta et al, Acc Chem. Res. 2000, 33, 278-286

1-03-Introduction-1a

9/15/03 8:14 AM

D. A. Evans

The H2 Molecular Orbitals & Antibonds The H2 Molecule (again!!)

Chem 206

Linear Combination of Atomic Orbitals (LCAO): Orbital Coefficients Rule Two: Each MO is constructed by taking a linear combination of the individual atomic orbitals (AO): Bonding MO Antibonding MO = C11 + C22 = C*11 C*22

Let's combine two hydrogen atoms to form the hydrogen molecule. Mathematically, linear combinations of the 2 atomic 1s states create two new orbitals, one is bonding, and one antibonding:
Rule one: A linear combination of n atomic states will create n MOs. (antibonding) E Energy H 1 p. p. E (bonding) 1s 1s H 2

The coefficients, C1 and C2, represent the contribution of each AO. Rule Three: (C1)2 + (C2)2 = 1

The squares of the C-values are a measure of the electron population in neighborhood of atoms in question Rule Four: bonding(C1)2 + antibonding(C*1)2= 1 In LCAO method, both wave functions must each contribute one net orbital Consider the pibond of a C=O function: In the ground state pi-CO is polarized toward Oxygen. Note (Rule 4) that the antibonding MO is polarized in the opposite direction.

Let's now add the two electrons to the new MO, one from each H atom:
(antibonding) E1

Energy

(antibonding)

H 1

1s E2

1s

H 2 Energy

(bonding)

Note that E1 is greater than E2. Why?


C O

(bonding)

1-04-Introduction-2 9/15/03 8:38 AM

D. A. Evans

Bonding Generalizations

Chem 206

Bond strengths (Bond dissociation energies) are composed of a covalent contribution ( Ecov) and an ionic contribution ( Eionic). Bond Energy (BDE) = Ecovalent + Eionic (Fleming, page 27)

Orbital orientation strongly affects the strength of the resulting bond.


For Bonds: A B Better than A B

When one compares bond strengths between CC and CX, where X is some other element such as O, N, F, Si, or S, keep in mind that covalent and ionic contributions vary independently. Hence, the mapping of trends is not a trivial exercise.

For Bonds:

Better than

Useful generalizations on covalent bonding


Overlap between orbitals of comparable energy is more effective than overlap between orbitals of differing energy.
p. p.

This is a simple notion with very important consequences. It surfaces in the delocalized bonding which occurs in the competing anti (favored) syn (disfavored) E2 elimination reactions. Review this situation. An anti orientation of filled and unfilled orbitals leads to better overlap. This is a corrollary to the preceding generalization. There are two common situations.

For example, consider elements in Group IV, Carbon and Silicon. We know that C-C bonds are considerably stronger by Ca. 20 kcal mol-1 than C-Si bonds.
C C C CC CSi C

better than

Si

Si

Case-1: Anti Nonbonding electron pair & CX bond


X
Si-SP3

X A
lone pair HOMO

C-SP3 CC

C-SP3

* CX LUMO

X Better than
lone pair HOMO

* CX LUMO

C-SP3 CSi

H3CCH3 BDE = 88 kcal/mol Bond length = 1.534

H3CSiH3 BDE ~ 70 kcal/mol Bond length = 1.87

Case-2: Two anti sigma bonds


X C
CY HOMO

This trend is even more dramatic with pi-bonds: CC = 65 kcal/mol CSi = 36 kcal/mol SiSi = 23 kcal/mol Weak bonds will have corresponding low-lying antibonds.
Formation of a weak bond will lead to a corresponding low-lying antibonding orbital. Such structures are reactive as both nucleophiles & electrophiles A Y

Y
* CX LUMO

X C

Better than

X C

CY HOMO

* CX LUMO

1-05-Introduction-3 9/12/03 4:36 PM

D. A. Evans

Donor-Acceptor Properties of Bonding and Antibonding States

Chem 206

Donor Acceptor Properties of C-C & C-O Bonds Consider the energy level diagrams for both bonding & antibonding orbitals for CC and CO bonds.
* C-C * C-O

Hierarchy of Donor & Acceptor States Following trends are made on the basis of comparing the bonding and antibonding states for the molecule CH3X where X = C, N, O, F, & H.
-bonding States: (CX)
CH3CH3 CH3H CH3NH2 CH3OH

C-SP

C-SP3 O-SP
3

very close!!

decreasing -donor capacity C-C C-O

CH3F

poorest donor

p. p.

The greater electronegativity of oxygen lowers both the bonding & antibonding C-O states. Hence:
CC is a better donor orbital than CO CO is a better acceptor orbital than CC

-anti-bonding States: (CX)


CH3H

For the latest views, please read Alabugin & Zeidan, JACS 2002, 124, 3175 (pdf)
CH3CH3 CH3NH2 CH3OH CH3F

Donor Acceptor Properties of CSP3-CSP3 & CSP3-CSP2 Bonds


* CC * CC better acceptor Increasing -acceptor capacity

best acceptor

C-SP3

C-SP3 C-SP2

The following are trends for the energy levels of nonbonding states of several common molecules. Trend was established by photoelectron spectroscopy.
Nonbonding States

CC

better donor

CC

H3P: H2S: H3N: H2O: HCl: decreasing donor capacity


poorest donor

The greater electronegativity of CSP2 lowers both the bonding & antibonding CC states. Hence: CSP3-CSP3 is a better donor orbital than CSP3-CSP2 CSP3-CSP2 is a better acceptor orbital than CSP3-CSP3
1-06-donor/acceptor states 9/12/03 5:16 PM

D. A. Evans

Hybridization vs Electronegativity

Chem 206

Electrons in 2S states "see" a greater effective nuclear charge than electrons in 2P states. This becomes apparent when the radial probability functions for S and P-states are examined: The radial probability functions for the hydrogen atom S & P states are shown below.
100 % 100 %

There is a linear relationship between %S character & Pauling electronegativity


5

N
4.5

SP

Radial Probability

Pauling Electronegativity

Radial Probability

1 S Orbital

N N
3.5
SP3

SP2

C 3

SP

2 S Orbital

2 S Orbital 2 P Orbital

p. p.

3 S Orbital 3 P Orbital

2.5

SP2

SP3

20

25

30

35

40

45

50

55

% S-Character

There is a direct relationship between %S character & hydrocarbon acidity


60

CH (56)

S-states have greater radial penetration due to the nodal properties of the wave function. Electrons in S-states "see" a higher nuclear charge. Above observation correctly implies that the stability of nonbonding electron pairs is directly proportional to the % of S-character in the doubly occupied orbital
Least stable Most stable
Pka of Carbon Acid

55

50

45

C H (44)
6 6

40

CSP3

CSP2

CSP

35

PhCC-H (29)
30

The above trend indicates that the greater the % of S-character at a given atom, the greater the electronegativity of that atom.
1-07-electroneg/hybrization 9/12/03 4:49 PM

25 20 25 30 35 40 45 50 55

% S-Character

D. A. Evans

Hyperconjugation: Carbocation Stabilization

Chem 206

The interaction of a vicinal bonding orbital with a p-orbital is referred to as hyperconjugation. This is a traditional vehicle for using valence bond to denote charge delocalization.
R H H C

Physical Evidence for Hyperconjugation


Bonds participating in the hyperconjugative interaction, e.g. CR, will be lengthened while the C(+)C bond will be shortened. First X-ray Structure of an Aliphatic Carbocation

+
C

R+ H H H H C C H H

The graphic illustrates the fact that the C-R bonding electrons can "delocalize" to stabilize the electron deficient carbocationic center. Note that the general rules of drawing resonance structures still hold: the positions of all atoms must not be changed.
p. p.

1.431

[F5SbFSbF5]

+
C

Stereoelectronic Requirement for Hyperconjugation: Syn-planar orientation between interacting orbitals

100.6

1.608

Me

Me Me

The Molecular Orbital Description


CR CR
T. Laube, Angew. Chem. Int. Ed. 1986, 25, 349

+
C

H H

+
C

H H

The Adamantane Reference (MM-2)


H

1.528

CR

CR

110
Me Me Me

1.530

Take a linear combination of CR and CSP2 p-orbital: "The new occupied bonding orbital is lower in energy. When you stabilize the electrons is a system you stabilize the system itself."
1-08-Hyperconj (+)-1 9/12/03 4:53 PM

D. A. Evans

"Negative" Hyperconjugation
Syn Orientation
R

Chem 206
antibonding CR R: H H C X+ H H antibonding CR R: R X+ H H C filled hybrid orbital X

Delocalization of nonbonding electron pairs into vicinal antibonding orbitals is also possible R H H C

R C X

R H H H H

H X H H H

filled hybrid orbital

This decloalization is referred to as "Negative" hyperconjugation

Anti Orientation
R

Since nonbonding electrons prefer hybrid orbitals rather that P orbitals, this orbital can adopt either a syn or anti relationship to the vicinal CR bond.

H H

H H

The Molecular Orbital Description


CR Overlap between two orbitals is better in the anti orientation as stated in "Bonding Generalizations" handout.

Nonbonding e pair

The Expected Structural Perturbations


Change in Structure
Shorter CX bond Longer CR bond

Spectroscopic Probe
X-ray crystallography X-ray crystallography Infrared Spectroscopy Infrared Spectroscopy NMR Spectroscopy NMR Spectroscopy

CR

As the antibonding CR orbital decreases in energy, the magnitude of this interaction will increase Note that CR is slightly destabilized

Stronger CX bond Weaker CR bond Greater e-density at R Less e-density at X

1-09-Neg-Hyperconj 9/12/03 4:53 PM

D. A. Evans

Lone Pair Delocalization: N2F2


The trans Isomer

Chem 206
Now carry out the same analysis with the same 2 orbitals present in the trans isomer.

The interaction of filled orbitals with adjacent antibonding orbitals can have an ordering effect on the structure which will stabilize a particular geometry. Here are several examples: Case 1: N2F2 F N N This molecule can exist as either cis or trans isomers F F N N F There are two logical reasons why the trans isomer should be more stable than the cis isomer. The nonbonding lone pair orbitals in the cis isomer will be destabilizing due to electron-electron repulsion. The individual CF dipoles are mutually repulsive (pointing in same direction) in the cis isomer. In fact the cis isomer is favored by 3 kcal/ mol at 25 C. Let's look at the interaction with the lone pairs with the adjacent CF antibonding orbitals. The cis Isomer F
filled N-SP2 F antibonding NF NF (LUMO) filled N-SP2 (HOMO)

filled N-SP2

N F

F antibonding NF filled N-SP2 (HOMO)

NF (LUMO)

In this geometry the "small lobe" of the filled N-SP2 is required to overlap with the large lobe of the antibonding CF orbital. Hence, when the new MO's are generated the new bonding orbital is not as stabilizing as for the cis isomer.

Conclusions
Lone pair delocalization appears to override electron-electron and dipole-dipole repulsion in the stabilization of the cis isomer. This HOMO-LUMO delocalization is stronger in the cis isomer due to better orbital overlap. Important Take-home Lesson Orbital orientation is important for optimal orbital overlap.

forms stronger pi-bond than

Note that by taking a linear combination of the nonbonding and antibonding orbitals you generate a more stable bonding situation. Note that two such interactions occur in the molecule even though only one has been illustrated.
1-10- N2F2 9/12/03 4:59 PM

forms stronger sigma-bond than

This is a simple notion with very important consequences. It surfaces in the delocalized bonding which occurs in the competing anti (favored) syn (disfavored) E2 elimination reactions. Review this situation.

D. A. Evans

Lone Pair Delocalization: The Gauche Effect

Chem 206

The interaction of filled orbitals with adjacent antibonding orbitals can have an ordering effect on the structure which will stabilize a particular conformation. Here are several examples of such a phenomon called the gauche effect: Hydrazine
H N H N H H
anti

The closer in energy the HOMO and LUMO the better the resulting stabilization through delocalization. Hence, N-lone pair NH delocalization better than NH NH delocalization. Hence, hydrazine will adopt the gauche conformation where both N-lone pairs will be anti to an antibonding acceptor orbital. The trend observed for hydrazine holds for oxygen derivatives as well Hydrogen peroxide
H O O H anti

Hydrazine can exist in either gauche or anti conformations (relative to lone pairs).

H N H

H H

H N H H

gauche

observed HNNH dihedral angle Ca 90

There is a logical reason why the anti isomer should be more stable than the gauche isomer. The nonbonding lone pair orbitals in the gauche isomer should be destabilizing due to electron-electron repulsion. In fact, the gauche conformation is favored. Hence we have neglected an important stabilization feature in the structure. HOMO-LUMO Interactions Orbital overlap between filled (bonding) and antibonding states is best in the anti orientation. HOMO-LUMO delocalization is possible between: (a) N-lone pair NH; (b) NH NH
H filled N-SP3 (HOMO) N N NH (LUMO) NH (HOMO) H NH (LUMO) N N H NH (LUMO)

H2O2 can exist in either gauche or anti conformations (relative to hydrogens). The gauche conformer is prefered.

H O

O H

gauche H

observed HOOH dihedral angle Ca 90

Major stabilizing interaction is the delocalization of O-lone pairs into the CH antibonding orbitals (Figure A). Note that there are no such stabilizing interactions in the anti conformation while there are 2 in the gauche conformation. Figure A Figure B
OH (LUMO) (HOMO) filled O-SP3

filled N-SP3 (HOMO)

(HOMO) filled O-SP3

Note that you achieve no net stabilization of the system by generating molecular orbitals from two filled states (Figure B). better stabilization
NH (HOMO)

Problem: Consider the structures XCH2OH where X = OCH3 and F. What is the most favorable conformation of each molecule? Illustrate the dihedral angle relationship along the CO bond.

1-11 Gauche Effect 9/11/01 11:27 PM

D. A. Evans

The Anomeric Effect: Negative Hyperconjugation


Useful LIterature Reviews

Chem 206

http://www.courses.fas.harvard.edu/~chem206/

Kirby, A. J. (1982). The Anomeric Effect and Related Stereoelectronic Effects at Oxygen. New York, Springer Verlag. Box, V. G. S. (1990). The role of lone pair interactions in the chemistry of the monosaccharides. The anomeric effect. Heterocycles 31: 1157. Box, V. G. S. (1998). The anomeric effect of monosaccharides and their derivatives. Insights from the new QVBMM molecular mechanics force field. Heterocycles 48(11): 2389-2417. Graczyk, P. P. and M. Mikolajczyk (1994). Anomeric effect: origin and consequences. Top. Stereochem. 21: 159-349. Juaristi, E. and G. Cuevas (1992). Recent studies on the anomeric effect. Tetrahedron 48: 5019. Plavec, J., C. Thibaudeau, et al. (1996). How do the Energetics of the Stereoelectronic Gauche and Anomeric Effects Modulate the Conformation of Nucleos(t)ides? Pure Appl. Chem. 68: 2137-44. Thatcher, G. R. J., Ed. (1993). The Anomeric Effect and Associated Stereoelectronic Effects. Washington DC, American Chemical Society.

Chemistry 206 Advanced Organic Chemistry


Lecture Number 2

Stereoelectronic Effects-2
Anomeric and Related Effects Electrophilic & Nucleophilic Substitution Reactions The SN2 Reaction: Stereoelectronic Effects Olefin Epoxidation: Stereoelectronic Effects Baeyer-Villiger Reaction: Stereoelectronic Effects Hard & Soft Acid and Bases (Not to be covered in class)

Problem 121

http://evans.harvard.edu/problems/

Sulfonium ions A and B exhibit remarkable differences in both reactivity and product distribution when treated with nucleophiles such as cyanide ion (eq 1, 2). Please answer the questions posed in the spaces provided below. KCN

Reading Assignment: Kirby, Chapters 1-3


S BF4 Et

rel. rate = 8000

Et

+ PhCH2CN

(1)

D. A. Evans

Wednesday, September 17, 2003

KCN S Et B rel. rate = 1 S

MeCH2CN

(2)

2-00-Cover Page 9/17/03 8:35 AM

D. A. Evans
The Anomeric Effect

The Anomeric Effect: Negative Hyperconjugation

Chem 206

It is not unexpected that the methoxyl substituent on a cyclohexane ring prefers to adopt the equatorial conformation. H OMe H

OMe Gc = +0.6 kcal/mol What is unexpected is that the closely related 2-methoxytetrahydropyran prefers the axial conformation: H O OMe Gp = 0.6 kcal/mol O H OMe

Since the antibonding CO orbital is a better acceptor orbital than the antibonding CH bond, the axial OMe conformer is better stabilized by this interaction which is worth ca. 1.2 kcal/mol. Other electronegative substituents such as Cl, SR etc also participate in anomeric stabilization. H 1.781 H O Cl H O O Cl 1.819 Cl
This conformer preferred by 1.8 kcal/mol Why is axial CCl bond longer ?

axial O lone pair CCl CCl

That effect which provides the stabilization of the axial OR conformer which overrides the inherent steric bias of the substituent is referred to as the anomeric effect. Let anomeric effect = A Gp = A = Gc + A Gp Gc

O Cl

H O HOMO

The Exo-Anomeric Effect

CCl

A = 0.6 kcal/mol 0.6 kcal/mol = 1.2 kcal/mol Principal HOMO-LUMO interaction from each conformation is illustrated below: H

There is also a rotational bias that is imposed on the exocyclic COR bond where one of the oxygen lone pairs prevers to be anti to the ring sigma CO bond H O O R O R favored R O O

OMe

H OMe

axial O lone pair CH

axial O lone pair CO

A. J. Kirby, The Anomeric and Related Stereoelectronic Effects at Oxygen, Springer-Verlag, 1983 E. Jurasti, G. Cuevas, The Anomeric Effect, CRC Press, 1995

2-01-Anomeric Effect-1 9/16/03 2:40 PM

D. A. Evans

The Anomeric Effect: Carbonyl Groups


Aldehyde CH Infrared Stretching Frequencies

Chem 206

Do the following valence bond resonance structures have meaning?


R C X O X R C O

Prediction: The IR CH stretching frequency for aldehydes is lower than the closely related olefin CH stretching frequency. For years this observation has gone unexplained. R R C H CH = 2730 cm CF3
-1

R C C R

Prediction: As X becomes more electronegative, the IR frequency should increase O Me CH3 Me O CBr3 Me O

O H

CH = 3050 cm -1

C=O (cm-1)

Sigma conjugation of the lone pair anti to the H will weaken the bond. This will result in a low frequency shift.

1720

1750

1780

Infrared evidence for lone pair delocalization into vicinal antibonding orbitals.
The NH stretching frequency of cis-methyl diazene is 200 cm-1 lower than the trans isomer. Me H N N Me
filled N-SP2 H antibonding NH

Prediction: As the indicated pi-bonding increases, the XCO bond angle should decrease. This distortion improves overlap.

R C X * CX O lone pair O

R C X O

NH = 2188 cm -1 Me N N H NH = 2317 cm -1

Me N
filled N-SP2

antibonding NH H

Evidence for this distortion has been obtained by X-ray crystallography Corey, Tetrahedron Lett. 1992, 33, 7103-7106

..

The low-frequency shift of the cis isomer is a result of NH bond weakening due to the anti lone pair on the adjacent (vicinal) nitrogen which is interacting with the NH antibonding orbital. Note that the orbital overlap is not nearly as good from the trans isomer. N. C. Craig & co-workers JACS 1979, 101, 2480.

2-02-Anomeric Effect-2 9/16/03 2:41 PM

D. A. Evans

The Anomeric Effect: Nitrogen-Based Systems


CMe3 Me3C CH N N N CMe3 Me3C N N N Me3C

Chem 206

Observation: CH bonds anti-periplanar to nitrogen lone pairs are spectroscopically distinct from their equatorial CH bond counterparts

H H H H

N H

Me3C G = 0.35kcal/mol

N HOMO CH

A. R. Katritzky et. al., J. Chemm. Soc. B 1970 135

Favored Solution Structure (NMR)


Me MeN NMe NMe Me N Me J. E. Anderson, J. D. Roberts, JACS 1967 96 4186 N N N Me MeN

Spectroscopic Evidence for Conjugation Infrared Bohlmann Bands Characteristic bands in the IR between 2700 and 2800 cm-1 for C-H4, C-H6 , & C-H10 stretch Bohlmann, Ber. 1958 91 2157 Reviews: McKean, Chem Soc. Rev. 1978 7 399 L. J. Bellamy, D. W. Mayo, J. Phys. Chem. 1976 80 1271 NMR : Shielding of H antiperiplanar to N lone pair H10 (axial): shifted furthest upfield H6, H4: = Haxial - H equatorial = -0.93 ppm Protonation on nitrogen reduces to -0.5ppm H. P. Hamlow et. al., Tet. Lett. 1964 2553 J. B. Lambert et. al., JACS 1967 89 3761
2-03-Anomeric Effect-3 9/16/03 2:43 PM

Favored Solid State Structure (X-ray crystallography)


1.484 1.453

Bn N Me
1.457

Me
1.453

N N Bn
1.459

A. R. Katrizky et. al., J. C. S. Perkin II 1980 1733

D. A. Evans

Anomeric Effects in DNA Phosphodiesters

Chem 206

Calculated Structure of ACGTGC Duplex

The Phospho-Diesters Excised from Crystal Structure

Guanine Cytosine Cytosine

1B

2B 1A Phosphate-1A p. p.
Thymine Adenine

Phosphate-1B

The Anomeric Effect


Acceptor orbital hierarchy: * POR * > * PO

R
O

R
O

P
O

O O R

O O R

O R P O O R
Phosphate-2A Phosphate-2B

Gauche-Gauche conformation
O

R P
O

O O R

Anti-Anti conformation Gauche-Gauche conformation affords a better donor-acceptor relationship

Oxygen lone pairs may establish a simultaneous hyperconjugative relationship with both acceptor orbitals only in the illustrated conformation.
Plavec, et al. (1996). How do the Energetics of the Stereoelectronic Gauche & Anomeric Effects Modulate the Conformation of Nucleos(t)ides? Pure Appl. Chem. 68: 2137-44.

2-04-DNA Duplex/Anomeric 9/17/03 9:25 AM

D. A. Evans

Carboxylic Acids (& Esters): Anomeric Effects Again?


Hyperconjugation: (Z) Conformer
O O R' O Me H O Me

Chem 206

Conformations: There are 2 planar conformations.


O O O O H R' R

Let us now focus on the oxygen lone pair in the hybrid orbital lying in the sigma framework of the C=O plane. * CO
O R R O

(Z) Conformer Specific Case: Methyl Formate

(E) Conformer

G = +4.8 kcal/mol

C R

In the (Z) conformation this lone pair is aligned to overlap with * CO.

The (E) conformation of both acids and esters is less stable by 3-5 kcal/mol. If this equilibrium were governed only by steric effects one would predict that the (E) conformation of formic acid would be more stable (H smaller than =O). Since this is not the case, there are electronic effects which must also be considered. These effects will be introduced shortly. Rotational Barriers: There is hindered rotation about the =COR bond. These resonance structures suggest hindered rotation about =COR bond. This is indeed observed:
O R O R' R O O R'
Energy

(E) Conformer
R R O C O

In the (E) conformation this lone pair is aligned to overlap with * CR.

* CR
O

R O C
barrier ~ 10-12 kcal/mol

O R O R O R
G ~ 2-3 kcal/mol

Since * CO is a better acceptor than * CR (where R is a carbon substituent) it follows that the (Z) conformation is stabilized by this interaction.

O R

O R R O

Esters versus Lactones: Questions to Ponder.


Esters strongly prefer to adopt the (Z) conformation while small-ring lactones such as 2 are constrained to exist in the (Z) conformation. From the preceding discussion explain the following: 1) Lactone 2 is significantly more susceptible to nucleophilic attack at the carbonyl carbon than 1? Explain.

Rotational barriers are ~ 10-12 kcal/mol. This is a measure of the strength of the pi bond.

O 1 Et CH3CH2 O O O 2

Lone Pair Conjugation: The oxygen lone pairs conjugate with the C=O.

versus

R
O

C R

The filled oxygen p-orbital interacts with pi (and pi*) C=O to form a 3-centered 4-electron bonding system.

2) Lactone 2 is significantly more prone to enolization than 1? In fact the pKa of 2 is ~25 while ester 1 is ~30 (DMSO). Explain. 3) In 1985 Burgi, on carefully studying O O O the X-ray structures of a number of lactones, noted that the O-C-C () & O O O O-C-O () bond angles were not equal. Explain the indicated trend in bond = 12.3 = 6.9 = 4.5 angle changes.

SP2 Hybridization

Oxygen Hybridization: Note that the alkyl oxygen is Sp2. Rehybridization is driven by system to optimize pi-bonding.

2-05 RCO2R Bonding 9/16/03 2:50 PM

D. A. Evans

Three-Center Bonds
Case 3: 2 p-Orbitals; 1 s-orbital

Chem 206

Consider the linear combination of three atomic orbitals. The resulting molecular orbitals (MOs) usually consist of one bonding, one nonbonding and one antibonding MO. Case 1: 3 p-Orbitals pi-orientation
antibonding

antibonding

nonbonding

Energy

nonbonding

bonding

Case 4: 2 s-Orbitals; 1 p-orbital


bonding
Note that the more nodes there are in the wave function, the higher its energy. H 2C H 2C CH CH + CH2 Allyl carbonium ion: both pi-electrons in bonding state

Do this as an exercise

Examples of three-center bonds in organic chemistry A. H-bonds: (3center, 4electron)


O CH3 O H O H O CH3

CH2

Allyl Radical: 2 electrons in bonding obital plus one in nonbonding MO.

The acetic acid dimer is stabilized by ca 15 kcal/mol

H 2C

CH

Allyl Carbanion: 2 electrons in bonding obital plus 2 in CH2 nonbonding MO.

B. H-B-H bonds: (3-center, 2 electron)


H B H B H H H H H B H H B H H

Case 2: 3 p-Orbitals sigma-orientation 3


nonbonding antibonding

diborane stabilized by 35 kcal/mol


Energy

C. The SN2 Transition state: (3center, 4electron)


H Nu C H H Br
The SN2 transition state approximates a case 2 situation with a central carbon p-orbital The three orbitals in reactant molecules used are: 1 nonbonding MO from Nucleophile (2 electrons) 1 bonding MO CBr (2 electrons) 1 antibonding MO * CBr

bonding

2-06 3-center bonds/review 10/28/03 12:00 PM

D. A. Evans

Substitution Reactions: General Considerations

Chem 206

Why do SN2 Reactions proceed with backside displacement?


R Nu: H H C X R
Nu X

Electrophilic substitution at saturated carbon may occur with either inversion of retention Inversion
+ M

R Nu C H H X:

C H H

Ra El(+) H Rb C M

+ Nu

Ra C H

Ra Nu C Rb H M+

Given the fact that the LUMO on the electrophile is the CX antibonding orblital, Nucleophilic attack could occur with either inversion or retention.

Rb

Inversion
R Nu

Retention
Ra El(+) Ra C M H Rb C M
+

Retention
R X H H C

Ra H Rb C El M+

C H H
LUMO

H Rb

El +

HOMO

Constructive overlap between Nu & *CX

Nu Overlap from this geometry results in no net bonding interaction El(+)


LUMO

Ra Ra C H Rb

C M H Rb

Expanded view of *CX

HOMO

Inversion
LUMO

El(+)

C
antibonding HOMO

X
bonding Br2 H Br

Retention Examples

Li H

CO2 CO2Li H

Nu
Fleming, page 75-76

predominant inversion

predominant retention

Stereochemistry frequently determined by electrophile structure


See A. Basu, Angew. Chem. Int. Ed. 2002, 41, 717-738

2-07-SN2-1 9/18/03 12:38 PM

D. A. Evans
The reaction under discussion:
R R C H X

SN2 Reaction: Stereoelectronic Effects


The use of isotope labels to probe mechanism.
C H H X R Nu C H H

Chem 206

Nu:

Nu

X:

1 and 2 containing deuterium labels either on the aromatic ring or on the methyl group were prepared. A 1:1-mixture of 1 and 2 were allowed to react. If the rxn was exclusively intramolecular, the products would only contain only three deuterium atoms:
O S O O CH3 SO3 D3C Nu

The NuCX bonding interaction is that of a 3-center, 4-electron bond. The frontier orbitals which are involved are the nonbonding orbital from Nu as well as CX and CX: CX
D 3C

Nu:

exclusively intramolecular

(CD3ArNuCH3)

CH3

O S

O O CD3

SO3 H 3C Nu

(CH3ArNuCD3

energy

Nu:
C X

Me

Nu:

exclusively intramolecular

CD3

CX RCH2X

Nu

Experiments have been designed to probe inherent requirement for achieving a 180 NuCX bond angle: Here both Nu and leaving group are constrained to be part of the same ring.
R R

2 If the reaction was exclusively intermolecular, products would only contain differing amounts of D-label depending on which two partners underwent reaction. The deuterium content might be analyzed by mass spectrometry. Here are the possibilities: 1 + 1 D3-product 2 CD3ArNuCH3 D'3-product 2 + 2 2 CH3ArNuCD3 1 CD3ArNuCD3 1 CH3ArNuCH3 Hence, for the strictly intermolecular situation one should see the following ratios D0 : D3 : D'3 : D6 = 1 : 2 : 2 : 1. 1 + 2 The product isotope distribution in the Eschenmoser expt was found to be exclusively that derived from the intermolecular pathway! Other Cases: exclusively intermolecular
(CH3)2N SO3CH3 (CH3)3N

Nu:

C X

C H H X

H H

Nu

D6-product D0-product

"tethered reactants"

"constrained transition state"

The Eschenmoser Experiment (1970): Helv. Chim Acta 1970, 53, 2059 The reaction illustrated below proceeds exclusively through bimolecular pathway in contrast to the apparent availability of the intramolecular path.
O S O O CH3 SO3

SO3

CH3

16% intramolecular 84% intermolecular

SO3CH3 N(CH3)2

SO3

Nu:

N(CH3)3

Nu

2-08-The SN2 RXN-FMO 9/16/03 2:56 PM

Hence, the NuCX 180 transition state bond angle must be rigidly maintained for the reaction to take place.

D. A. Evans
SO3CH3 N(CH3)2

Intramolecular methyl transfer: Speculation on the transition structures

(CH3)2N

Chem 206
+
SO3

SO3

SO3CH3

N(CH3)3

(CH3)3N

16% intramolecular; 84% intermolecular


9- membered cyclic transition state

exclusively intermolecular
8- membered cyclic transition state

174 174

2-09-Intra alk TS's 9/16/03 2:56 PM

00000 00000 00000 00000 00000


est CO bond length 2.1

000000 000000 000000 000000 000000


est CO bond length 2.1

00000 00000000 00000 00000000 00000 00000 00 00 00 00 0000 00 00 0000 00000 0000 00000

est CN bond length 2.1

est CN bond length 2.1

Approximate representation of the transition states of the intramolecular alkylation reactions. Transition state CO and CN bond lengths were estimated to be 1.5x(CX) bond length of 1.4

D. A. Evans
The General Reaction:
R R O

Olefin Epoxidation via Peracids: An Introduction Per-arachidonic acid Epoxidation


O

Chem 206

R O OH R

+
R R R

+
R OH

Me O

O H

HOMO CC

LUMO *OO

note labeled oxygen is transferfed

O-O bond energy: ~35 kcal/mol

Reaction rates are governed by olefin nucleophilicity. The rates of epoxidation of the indicated olefin relative to cyclohexene are provided below: OH
OH OAc

1.0

0.6

0.05

0.4

The indicated olefin in each of the diolefinic substrates may be oxidized selectively.
Me Me Me Me Me Me Me

H Me Me

The transition state:

Me

O H

E. J. Corey, JACS 101, 1586 (1979) For a more detailed study see P. Beak, JACS 113, 6281 (1991) View from below olefin
2-10 Epoxidation-1 9/16/03 2:58 PM

For theoretical studies of TS see R. D. Bach, JACS 1991, 113, 2338 R. D. Bach, J. Org. Chem 2000, 65, 6715

D. A. Evans
The General Reaction:
R R

Olefin Epoxidation with Dioxiranes


Asymmetric Epoxidation with Chiral Ketones
R R O R R

Chem 206

+
R R

R R

Review: Frohn & Shi, Syn Lett 2000, 1979-2000


Me O Me O O O Me

+
R R note labeled oxygen is transferfed

HOMO CC

LUMO *OO

chiral catalyst
O Me

O-O bond energy: ~35 kcal/mol


R2 H R1 R R O R2

Synthesis of the Dioxirane Oxidant


K+ O R O O S O O H R R O O SO3

O R2 R1 R2

oxone, CH3CN-H2O pH 7-8


Me Me Ph

O R

(Oxone)

Ph

Ph

Ph

Ph

>95% ee

84% ee

92% ee

Synthetically Useful Dioxirane Synthesis


O Me O F 3C CF3 Me

Question: First hour Exam 2000 (Database Problem 34)


Question 4. (15 points). The useful epoxidation reagent dimethyldioxirane (1) may be prepared from "oxone" (KO3SOOH) and acetone (eq 1). In an extension of this epoxidation concept, Shi has described a family of chiral fructose-derived ketones such as 2 that, in the presence of "oxone", mediate the asymmetric epoxidation of di- and tri-substituted olefins with excellent enantioselectivities (>90% ee) (JACS 1997, 119, 11224). Me O Me R2 R1 R2
KO3SOOH CH3CN-H2O pH 10.5 1 equiv 2 oxone, CH3CN-H2O pH 10.5

oxone

O Me

O Me O CF3

co-distill to give ~0.1 M soln of dioxirane in acetone co-distill to give ~0.6 M soln of dioxirane in hexafluoroacetone

oxone

O F 3C

Me

Me Me

O O
1 (1) 2

Me O

Curci, JOC, 1980, 4758 & 1988, 3890; JACS 1991, 7654. Transition State for the Dioxirane Mediated Olefin Epoxidation
O R R O O R R

O R2
(2)

O Me O Me

R1

R2

planar

>90% ee

spiro

rotate 90

Part A (8 points). Provide a mechanism for the epoxidation of ethylene with dimethyldioxirane (1). Use three-dimensional representations, where relevant, to illustrate the relative stereochemical aspects of the oxygen transfer step. Clearly identify the frontier orbitals involved in the epoxidation. Part B (7 points). Now superimpose chiral ketone 2 on to your mechanism proposed above and rationalize the sense of asymmetric induction of the epoxidation of trisubstituted olefins (eq 2). Use three-dimensional representations, where relevant, to illustrate the absolute stereochemical aspects of the oxygen transfer step.

stabilizing Olp * C=C cis olefins react ~10 times faster than trans Houk, JACS, 1997, 12982.

2-11 Epoxidation-2 9/16/03 3:01 PM

D. A. Evans
O C RL RS + RCO3H RCO2H

The Baeyer-Villiger Reaction: Stereoelectronic Effects


O RL C O major RS O Me O minor RS Me O + RCO3H CMe3 Me3C O H O O O
- MeCO2H

Chem 206
CMe3 Me O H O R O O

Let RL and RS be Sterically large and small substituents.


+
C RL

The major product is that wherein oxygen has been inserted into theRLCarbonyl bond. O kR kR / KMe R
R O O C R Me + CF3CO3H O C Me
H

Favored
CMe3 O Me O OH O R

Migrating group

major

CH3CH2 CH3(CH2)2 (CH3)3C

72 150 830 >2000 Conformer A


H

O Me O CMe3

kMe

C R O

Me

minor

PhCH2
RS OH C RL O O

Disfavored
Me O

Migrating group

The Intermediate

O OH Me O CMe3

Me3C O

The important stereoelectronic components to this rearrangement:


R

1. The RLCOO dihedral angle must be180 due to the HOMO LUMO interaction -RLCOO. 2. The COOC' dihedral angle will be ca. 60 due to the gauche effect (O-lone pairsCO). This gauche geometry is probably reinforced by intramolecular hydrogen bonding as illustrated on the opposite page:
2-12- Baeyer Villiger Rxn 9/16/03 5:33 PM

Conformer B

The destabilizing gauche interaction

Steric effects destabilize Conformer B relative to Conformer A; hence, the reaction is thought to proceed via a transition state similar to A. For relevant papers see: Crudden, Angew. Chem. Int. Ed 2000, 39, 2852-2855 (pdf) Kishi, JACS 1998, 120, 9392 (pdf)

D. A. Evans

The Baeyer-Villiger Reaction: Stereoelectronic Effects


CMe3 Me

Chem 206

Conformer A in three dimensions

O Me

+ RCO3H CMe3

Me3C O H

Me O
- MeCO2H

O O H O R O

Favored
H

Migrating group CMe3 O O OH O R O Me O CMe3

Me

Conformer A
H

Disfavored
Me O Me3C O R OH O

Migrating group

O Me
1

CMe3
2 3

Conformer B

The destabilizing gauche interaction

Steric effects destabilize Conformer B relative to Conformer A; hence, the reaction is thought to proceed via a transition state similar to A. For relevant papers see: Crudden, Angew. Chem. Int. Ed 2000, 39, 2852-2855 (pdf) Kishi, JACS 1998, 120, 9392 (pdf)
2-13- Baeyer Villiger Rxn-2 9/16/03 5:41 PM

23 dihedral angle ~ 178 from Chem 3D

B. Breit

FMO-Theory/HSAB Principle 1
Hard and Soft Acids and Bases (HSAB-Principle)
Reading Assignment: Fleming, Chapter 3, p33-46 Pearson, JACS 1963, 85, 3533.

Chem 206

FMO-Theory and Klopman-Salem equation provide an understanding of this empirical principle: Hard Acids have usually a positive charge, small ion radii (high charge density), energy rich (high lying) LUMO. Soft Acids are usually uncharged and large (low charge density), they have an energy poor (low lying ) LUMO (usually with large MO coefficient). Hard Bases usually have a negative charge, small ion radii (high charge density), energy poor (low lying) HOMO. Soft Bases are usually uncharged and large (low charge density), energy rich (high lying) HOMO (usually with large MO coefficient).

Hard Acids prefer to interact with hard bases Soft acids prefer to interact with soft bases.
Softness: Polarizability; soft nucleophiles have electron clouds, which can be polarized (deformed) easily. Charged species with small ion radii, high charge density.

Hardness:

Qualitative scaling possible: Molecular Orbital Energies of an

idealized Soft Species


E E small HOMO/LUMO gap

idealized Hard Species

large HOMO/LUMO gap

FMO-Theory for interaction:

Soft-Soft
E E

Hard-Hard

Acid

Base Acid Base

Significant Energy gain through HOMO/LUMO interaction

Only neglectable energy gain through orbital interaction.

2-14-FMO HSAB 1 9/20/00 8:30 AM

B. Breit

FMO-Theory/HSAB Principle 2

Chem 206

Klopman-Salem Equation for the interaction of a Nucleophile N (Lewis-Base) and an Electrophile E (Lewis-Acid).

E =

QNQE RNE Coulomb Term

2(cNcE)2 EHOMO(N) - ELUMO(E) Frontier Orbital Term

Q: Charge density : Dielectricity constant R: distance (N-E) c: coefficient of MO : Resonance Integral E: Energy of MO

Soft-Soft Interactions: Coulomb term small (low charge density). Dominant interaction is the frontier orbital interaction because of a small E(HOMON/LUMOE). formation of covalent bonds Hard-Hard Interactions: Frontier orbital term small because of large E(HOMON/LUMOE). Dominant interaction is described by the Coulomb term (Q is large for hard species), i.e. electrostatic interaction. formation of ionic bonds Hard-Soft Interactions: Neither energy term provides significant energy gain through interaction. Hence, Hard-Soft interactions are unfavorable.

2-15-FMO HSAB 2 9/20/00 8:27 AM

B. Breit

FMO-Theory/HSAB Principle 3
HSAB principle - Application to Chemoselectivity Issues
(c) SN2 vs E2

Chem 206

(a) Enolate Alkylation


O soft

CO2R

hard O soft C C

soft MeI

C-Alkylation
Me OTMS

HC(COOR)2 H Br hard OC2H5

CO2R

S N2

hard TMSCl

E2

O-Alkylation

(d) Ambident Nucleophiles (b) 1,2- vs. 1,4-addition to ,-unsaturated carbonyl compounds + 0.29
O H H

- 0.48
O

soft Ag S C N hard Na

soft MeI

H3 C

C O

S-Alkylation

+ 0.01 Charge density

+ 0.62 LUMO-coefficients

hard RCOX

N-Acylation

hard soft
O

hard MeLi

OH Me O Ag soft MeI H3C NO2

N-Alkylation

1,2-Addition soft Me2CuLi Conjugate Addition


Me O

O hard

N Na soft hard t-BuCl ONO

O-Alkylation

2-16-FMO HSAB 3 9/20/00 8:27 AM

D. A. Evans

Rules for Ring Closure: Introduction

Chem 206

http://www.courses.fas.harvard.edu/~chem206/

The Primary Literature Baldwin, J. Chem. Soc., Chem. Comm. 1976, 734, 736. Baldwin, J. Chem. Soc., Chem. Comm. 1977 233. Baldwin, J. Org. Chem. 1977, 42, 3846. Baldwin, Tetrahedron 1982, 38, 2939.

Chemistry 206 Advanced Organic Chemistry

Lecture Number 3

Stereoelectronic Effects-3 "Rules for Ring Closure: Baldwin's Rules"


Kirby, "Stereoelectronic Effects" Chapters 4, 5

Useful LIterature Reviews Johnson, C. D. (1993). Stereoelectronic effects in the formation of 5and 6-membered rings: the role of Baldwin's rules. Acc. Chem. Res. 26: 476-82. (Handout) Beak, P. (1992). Determinations of transition-state geometries by the endocyclic restriction test: mechanisms of substitution at nonstereogenic atoms. Acc. Chem. Res. 25: 215. (Handout)

Problems of the Day


Propose mechanisms for the following reactions
R O R HO H+ R R O O

+
HO O

D. A. Evans
3-00-Cover Page 9/19/03 8:36 AM

Friday, September 19, 2003


Me

NH2NH2 OMe Me

HN NH O

D. A. Evans, J. Johnson

Rules for Ring Closure: Introduction

Chem 206

Ring Closure and Stereoelectronic Connsiderations An Examination of Baldwin's Rules


"Baldwin's Rules" provides a qualitative set of generalizations on the probability of a given ring closure. There are circumstances where the "rules" don't apply. They do not apply to non-first-row elements participating in the cyclization event. The longer bond lengths and larger atomic radii of 2nd row elements result in relaxed geometrical constraints. For example, a change in a heteroatom from O to S could result in relaxation of a given geometric constraint. X = O vs S X Y

endo

C. Nucleophilic ring closures sub-classified according to hybridization state of electrophilic component: (tetrahedral = tet; trigonal = trig; digonal = dig) D. Nucleophilic ring closures further subclassified according to size of the fomed ring. For example:
5-exo-tet

X Y

5-exo-trig

X Y
5-exo-dig

X Y

The "rules" do not apply to electrocyclic processes.

Nomenclature
Classes of Ring Closing Processes A. Exo-cyclization modes identified by the breaking bond being positioned exocyclic to the forming cycle.
exo

Required trajectories (Baldwin):

X Y

= 180

X Y

X X

Y Y B. Endo-cyclization modes identified by the breaking bond being positioned endocyclic to the forming cycle.
endo

= 109

Y
Will come back to this case later

X Y X = first-row element N, O

* 120 *

Baldwin, J. Chem. Soc., Chem. Commun., 1976, 734.

3-01-Baldwin Rules-1 9/18/03 3:38 PM

D. A. Evans, J. Johnson

Rules for Ring Closure: SP3 Carbon & Related Systems


FRST-PLATTNER RULE

Chem 206

Tetrahedral Carbon
All exo cyclization modes are allowed: (n-exo-tet, n = 3)
exo

In this simple model, the transition-state leading to 1 involves the diaxial orientation of nucleophile and leaving group. This orientation affords the best overlap of the anti-bonding CY orbital and the nonbonding electron pairs on the nucleophile O. In the formation of the diastereomeric epoxide 2, the proper alignment of orbitals may only be achieved by cyclization through the less-favored boat conformer. Accordingly, while both cyclizations are "allowed", there are large rate differences the the rates of ring closure. While the FRST-PLATTNER RULE deals wilth the microscopic reverse, in the opening of epoxides by nucleophiles, the stereoelectronic arguments are the same.

C Y

C Y

There are stereoelectronic issues to consider for n-exo-tet cyclizations Formation of 3-Membered Rings (3-exo-tet)
H H H

X C H2

CH2 C H2

C H2

+ Y

Stereoelectronic Effects in Epoxide Ring Cleavage


Nu Me3C NuO H O Me3C H HO HO NuH Me Me3C H H Nu Me H H

Conformational Effects in Epoxide Ring Formation/cleavage


Those stereoelectronic effects that operate in ring cleavage also influence ring formation. Consider a rigid cyclohexene oxide system:
Y H H O H Y H O H O

faster
H H H Me3C

O
Y

slower
H O

Nu-

Nu HO

O H H

chair

boat

"The diaxial nucleophilic ring cleavage of epoxides" For more information on epoxide cleavage see Handout 03A.

3-02-Baldwin Rules-2 9/18/03 3:39 PM

D. A. Evans, J. Johnson

Rules for Ring Closure: SP3 Carbon & Related Systems


Case 2: King, J.C.S. Chem. Comm., 1979, 1140.
O S O Me NMe2 O O

Chem 206

Tetrahedral Carbon
Endo cyclization modes that are disallowed (n-endo-tet, n = 39)

8-endo-tet disfavored Rxn exclusively intermolecular 8-endo-tet disfavored Rxn exclusively intermolecular 9-endo-tet borderline 84% intermolecular, 16% intramolecular

O O

NMe3+

endo

Y C(SP3)

SO2OMe NMe2

SO3 NMe3+

The stereoelectronic requirement for a 180 XCY bond angle is only met when the endo cyclization ring size reaches 9 or 10 members. Case 1: Eschenmoser, Helvetica Chim. Acta 1970, 53, 2059.
O S O O CX3 O S O O S O OCX3 O S O SO2OMe NMe2

SO3 NMe3+

NaH 6-endo-tet disfavored Rxn exclusively intermolecular (lecture 2)

Conclusions
Allowed endo cyclization modes will require transition state ring sizes of at least nine members.

Intramolecular epoxidation has also been evaluated


CY3

CY3

O Cl OOH n
-

Beak, JACS 1991, 113, 6281. 8-endo-tet disfavored


Cl CO2H

Cyclization exclusively intermolecular. However the exocyclic analog is exclusively intramolecular


O S O O CX2I O S O O S O O

NaH 6-exo-tet favored Rxn exclusively intramolecular


O S

CX2 O

n = 1: rxn exclusively intermolecular n = 9: rxn is intramolecular

Beak states that the conclusions made with carbon substitution also hold for oxygen atom transfer.
Beak, P. (1992). Determinations of transition-state geometries by the endocyclic restriction test: mechanisms of substitution at nonstereogenic atoms. Acc. Chem. Res. 25: 215.

CY3

CY3

3-03-Baldwin Rules-3 9/18/03 4:07 PM

D. A. Evans, J. Johnson

Rules for Ring Closure: SP2 Carbon & Related Systems

Chem 206

Trigonal Carbon
Endo cyclization modes that are disallowed (3 to 5-endo-trig)
n-endo-trig X Y X C Y MeO2C NH2 CO2Me

X = first-row element The 5-endo-trig cyclization is a watershed case distance from reacting centers: 2.77 Case 1: Baldwin, J. Chem. Soc., Chem. Commun., 1976, 734.
CO2Me OH base
X

CO2Me O

however
CO2Me SH

5-endo-trig Disfavored base

It is possible that a "nonvertical" trajectory is operational like that suspected in C=O addition
CO2Me

Second row atom relaxes the cyclization geometrical requirement Case 2: Baldwin, J. Chem. Soc., Chem. Commun., 1976, 736.
MeO2C NH2 CO2Me
X

MeO2C HN

CO2Me

5-endo-trig 0%
MeO2C HN O

5-exo-trig 100%

3-04-Baldwin Rules-4 9/18/03 4:07 PM

D. A. Evans, J. Johnson
Case 2: continued...
MeO2C NH2 CO2Me
X

Rules for Ring Closure: SP2 Carbon & Related Systems

Chem 206

Apparent exceptions to disallowed 5-endo-trig cyclization process


MeO2C HN CO2Me
O

5-endo-trig 0%
MeO2C HN O

CH3CO2H

+ N

OH

Filer, J. Am. Chem. Soc. 1979, 44, 285.

5-exo-trig 100%
R HC N
1

CO2Me KO Bu R2 CO2Me
t

CO2Me R1 HN R CO2Me
2

CO2Me R 3:1
1

Control experiment: Intermolecular reaction favors conjugate addtion.


Me CO2Me PhCH2NH2 H N Me CO2Me 100% Me H N O 0% Ph

HN

CO2Me R2

Ph

R1 = aryl, R2 = aryl, alkyl

Grigg, J. Chem. Soc., Chem. Commun. 1980, 648.

Case 3:
O Ph OMe NH2NH2 65 oC Ph HN NH O

Does the illustrated ketalization process necessarily violate "the rules"?

R O R

(CH2OH)2 H+

R R

O O

O Ph MeI OK

1) EtO2CCl, pyridine 2) NH2NH2

Ph NH

R O R

(CH2OH)2

R HO

R O

OH

H+ H2O

R R O

OH

( )2

H 2N 200 oC O Ph OMe NH2NH2 65 oC HN X

( )2
5-endo-trig

5-endo-trig

H+
R O

disfavored ?
R R O O

R Ph CO2Me NH2 Ph O HN NH HO

OH 5-exo-tet

( )2

favored ?

5-exo-trig

Johnson, C. D. (1993). Stereoelectronic effects in the formation of 5- and 6-membered rings: the role of Baldwin's rules. Acc. Chem. Res. 26: 476-82.

3-05-Baldwin Rules-5 9/18/03 4:08 PM

D. A. Evans, J. Johnson
More Exceptions

Rules for Ring Closure: SP2 Carbon & Related Systems


Bu

Chem 206
Bu

Zard, Org. Lett. 2002, 4, 1135


MeO O MeO N S S OEt N O

X Y

NaH
HO

DMF, 60 C Y F H Cl Br Cond

Y O

ROOR heat 80% O


O

X F F

Yield 80 17 15

DMF, 60 C, 2 h DMF, 80 C, 43 h DMF, 60 C, 8 h DMF, 60 C, 5 h

O MeO

O MeO N

MeO O MeO N

Cl Br

5-endo-trig

Ichikawa, et al Synthesis 2002, 1917-1936, PDF on Course Website Numerous other cases are provided in this review.

Br

H O

Revisiting Case 2 with Fluorines


O O MeO2C

Bu3SnH AIBN 82%


O H

MeO2C N O

OMe

MeO2C N Ts

CO2Me

Ts

5-exo-trig

TsHN

5-endo-trig

Favored
H O

Not Observed

5-endo-trig
O O

O MeO2C N Ts O CF2 MeO2C TsHN

O OMe CF2 MeO2C N Ts F CO2Me

5-exo-trig

5-endo-trig

Chem. Comm 2088, 28 Review: "5-Endo-Trig Radical Cyclizatons" Ishibashi, et al Synthesis 2002, 695-713, PDF on Course Website

Not Observed

Favored

3-06-Baldwin Rules-6 9/18/03 5:10 PM

D. A. Evans, J. Johnson

Rules for Ring Closure: SP2 Carbon & Related Systems

Chem 206

Trigonal Carbon: Exocyclic Enolate Alkylation


exo

distance between reacting centers: 3.37


C O C C Y-

C C

C Y

Br
X

By definition, an exo-tet cyclization, but stereoelectronically behaves as an endo trig.


Me Me O Me Me MO Br
X

MO

Me Me O

(1)

The overlap for C-alkylation is poor due to geometrical constraints of 5-membered ring

only observed product

However:
Me Me O Me Br
KOt-Bu or LDA

distance between reacting centers: 3.04


Me Me O > 95% by NMR

The relaxed geometrical constraint provided by the added CH2 group now renders the 6-membered cyclization possible

Baldwin, J. Chem. Soc., Chem. Commun. 1977, 233.

Given the failure of the enolate alkylation shown above (eq 1), explain why these two cyclizations are successful.
Br NHAr O R O Ar NH R OMs base O N Ar base N O R Ar R

MO

Br

Favorskii Rearrangement (Carey, Pt B, pp 609-610) Your thoughts on the mechanism


O Cl MeO HCl O MeO

CO2Me

3-07-Baldwin Rules-7 9/18/03 4:09 PM

D. A. Evans, J. Johnson

Rules for Ring Closure: SP2 & SP Carbon & Related Systems

Chem 206

Trigonal Carbon: Intramolecular Aldol Condensations


Baldwin, Tetrahedron 1982, 38, 2939

Digonal Carbon: Cyclizations on to Acetylenes


DIGONAL: Angle of approach for attack on triple bonds

MO R

X Y

(Enolendo)-Exo-trig
O R

X YM

Nu120 120 E+

Baldwin: - 3 and 4-Exo-dig are disfavored - 5 to 7-Exo-dig are favored - 3 to 7-Endo-dig are favored

Favored: 6-7-(enolendo)-exo-trig Disfavored: 3-5-(enolendo)-exo-trig

X MO R Y

(Enolexo)-exo-trig
O R

X YM
H _

Ab initio SCF 4-31G calculations for the interaction of hydride with acetylene:
H
2.13 127 o 148o H

Favored: 3-7-(enolexo)-exo-trig
H O Me O
I

4-31G basis set Houk, J.ACS.1979, 101, 1340.

5-(Enolendo)-Exo-trig
Me Me O

6-(Enolendo)-Exo-trig
Me O H

H 156o 1.22

1.5-2.0

110o -120o

STO-3G minimal basis set H Dunitz, Helv Chim. Acta 1978, 61, 2538.

favored
Me O Me O Me
III

H O

Me Me O
II

Crystal Structures do not support Baldwin


O

Statistical Distribution, (I + II)/III = 2:1 Experimental Distribution, = 0:100


N

N
2.92 104o

(KOH, MeOH, r.t., 5 min, 77% y.)

ON
+

+
N

O
93

2.44
o

N
o

Caution: Baldwin's conclusions assume that the RDS is ring closure; however, it is well known (by some!) that the rate determining step is dehydration in a base-catalyzed aldol condensation.

86

J. Dunitz and J. Wallis J. C. S. Chem. Comm. 1984, 671.

3-08-Baldwin Rules-8 9/18/03 5:49 PM

D. A. Evans, J. Johnson

Rules for Ring Closure: SP Carbon & Related Systems


Indole synthesis:
CH3
2 equiv. LDA 2 equiv. RX -78 oC R = Me, Bu, CO2Me LiTMP
+

Chem 206

Endo Digonal versus Endo Trigonal Cyclizations


5-endo-trig
Y X:

CH2R N

C-

In-plane approach; nucleophile lone pair is orthogonal to *

Out-of-plane approach; nucleophile lone pair can't achieve Brgi-Dunitz angle

R
Saegusa, J. Am. Chem. Soc. 1977, 99, 3532. _

5-endo-dig
:X

Spiro dihydrofuranones:
O Li HO MeO
n n = 1,2

Li+

Allowed due to in-plane pi orbitals

OMe

KOtBu

OMe

X
For an opposing viewpoint to Baldwin's view of nucleophile trajectories, see Menger's article on directionality in solution organic chemistry: Tetrahedron 1983, 39, 1013.
O Me Me HO Ph NaOMe MeOH Me Me O Ph O

Developing negative charge on the central allenic carbon is in the same plane as the OMe group
Magnus, J. Am. Chem. Soc. 1978, 100, 7746.

5-endo-dig
O

5-exo-dig
O R OH
R = H, OMe

NaOMe
X

5-endo-trig

R O Ph

Li

Ph

Li

Ph

however, the acid catalyzed version does cyclize


Baldwin, J. Chem. Soc., Chem. Commun., 1976, 736. Johnson, Can. J. Chem. 1990, 68, 1780 J. Am. Chem. Soc. 1983, 105, 5090 J. Chem. Soc., Chem. Commun. 1982, 36.

4-endo-dig
X

Li

Li

Ph

3-09-Baldwin Rules-9 9/19/03 8:38 AM

D. A. Evans, J. Johnson

Rules for Ring Closure: SP Carbon & Related Systems

Chem 206

Digonal Cyclizations: Interesting Examples


MeO2C CN Et3N, Toluene, reflux 12 h, 65-70% y. CN

Trost, J. Am. Chem. Soc., 1979, 101, 1284. Proposes E-olefin geometry, E/Z > 95:5

5-exo-dig
O O CO2Me
:

R R'

30-40 kcal/mol ?

R R'
:

OH

HO2C H
Hirsutic Acid C

Conclusions and Caveats


OTBS

O Me

LiCH2NC; TBS-Cl 71%


+

Baldwin's Rules are an effective first line of analysis in evaluating the stereoelectronics of a given ring closure Baldwin's Rules have provided an important foundation for the study of reaction mechanism Competition studies between different modes of cyclization only give information about relative rates, and are not an absolute indicator of whether a process is "favored" or "disfavored" Structural modifications can dramatically affect the cyclization mode; beware of imines and epoxides
EXO Tet Trig Dig X X Tet ENDO Trig X X X X X X Dig

OTBS 1) RCOCl
2) AgBF4 86%

Me C-

N+ C O

Me R

Works for varying ring sizes and R groups; acylnitrilium ion can also work as an electophile in a Friedel-Crafts type of reaction 5-endo-dig Livinghouse, Tetrahedron 1992, 48, 2209.
O H N O Me R

3 4 5 6 7

3-10-Baldwin Rules-10 9/18/03 5:21 PM

D. A. Evans

Acyclic Conformational Analysis-1


Professor Frank Weinhold Univ. of Wisconsin, Dept of Chemistry B.A. 1962, University of Colorado, Boulder

Chem 206

http://www.courses.fas.harvard.edu/~chem206/

Chemistry 206 Advanced Organic Chemistry


Lecture Number 4

A.M. 1964, Harvard University Ph.D. 1967, Harvard University Physical and Theoretical Chemistry.

Useful LIterature Reviews


Eliel, E. L., S. H. Wilen, et al. (1994). Stereochemistry of Organic Compounds. New York, Wiley. Juaristi, E. (1991). Introduction to Stereochemistry and Conformational Analysis. New York, Wiley. Juaristi, E., Ed. (1995). Conformational Behavior of Six-Membered Rings: Analysis, Dynamics and Stereochemical Effects. (Series: Methods in Stereochemical Analysis). Weinheim, Germany, VCH. Schweizer, W. B. (1994). Conformational Analysis. Structure Correlation, Vol 1 and 2. H. B. Burgi and J. D. Dunitz. Weinheim, Germany, V C H Verlagsgesellschaft: 369-404. Kleinpeter, E. (1997). Conformational Analysis of Saturated Six-Membered Oxygen-Containing Heterocyclic Rings. Adv. Heterocycl. Chem. 69: 217-69. Glass, R. R., Ed. (1988). Conformational Analysis of Medium-Sized Ring Heterocycles. Weinheim, VCH. Bucourt, R. (1973). The Torsion Angle Concept in Conformational Analysis. Top. Stereochem. 8: 159.

Acyclic Conformational Analysis-1


Ethane, Propane, Butane & Pentane Conformations Simple Alkene Conformations

Reading Assignment for week A. Carey & Sundberg: Part A; Chapters 2 & 3
R. W. Hoffmann, Angew. Chem. Int. Ed. Engl. 2000, 39, 2054-2070 Conformation Design of Open-Chain Compounds (handout)

The Ethane Barrtier Problem


F. Weinhold, Nature 2001, 411, 539-541 "A New Twist on Molecular Shape" (handout) F. M. Bickemhaupt & E. J. Baerends, Angew. Chem. Int. Ed. 2003, 42, 4183-4188,"The Case for Steric Repulsion Causing the Staggered Conformation in Ethane" (handout) F. Weinhold,, Angew. Chem. Int. Ed. 2003, 42, 4188-4194,"Rebuttal of the BikelhauptBaerends Case for Steric Repulsion Causing the staggered Connformation of Ethane" (handout)

Problems of the Day


O Predict the most stable conformation of the indicated dioxospiran? O

D. A. Evans
4-00-Cover Page 9/22/03 9:01 AM

Monday, September 22, 2003

D. A. Evans

Acyclic Conformational Analysis-1

Chem 206

The following discussion is intended to provide a general overview of acyclic conformational analysis

Ethane Rotational Barrier: The FMO View


F. Weinhold, Angew. nature 2001, 411, 539-541"A New Twist on Molecular Shape"

Ethane & Propane


The conformational isomerism in these 2 structures reveals a gratifying level of internal consistency.
H H RH C H H

One can see from the space-filling models that the Van der Waals radii of the hydrogens do not overlap in the eclipsed ethane conformation. This makes the steric argument for the barrier untenable. One explanation for the rotational barrier in ethane is that better overlap is possible in the staggered conformation than in the eclipsed conformation as shown below. In the staggered conformation there are 3 anti-periplanar CH Bonds

eclipsed conformation
H E = +3.0 kcal mol-1 (R = H) Van derWaals radii of vicinal hydrogens do not overlap in ethane E = +3.4 kcal mol-1 (R = Me) C H C
CH HOMO

H
C C

* CH LUMO

CH

p. p.

CH

R H C H H
In propane there is a discernable interaction

H H

In the eclipsed conformation there are 3 syn-periplanar CH Bonds staggered conformation


H C H C
CH HOMO CH

H
C

H
C

* CH LUMO CH

For purposes of analysis, each eclipsed conformer may be broken up into its component destabilizing interactions. Incremental Contributions to the Barrier. Structure ethane propane Eclipsed atoms E (kcal mol -1) 3 (HH) 2 (HH) 1 (HMe) +1.0 kcal mol -1 +1.0 kcal mol -1 +1.4 kcal mol -1

Following this argument one might conclude that: The staggered conformer has a better orbital match between bonding and antibonding states. The staggered conformer can form more delocalized molecular orbitals. J. P. Lowe was the first to propose this explanation
"A Simple Molecuar Orbital Explanation for the Barrier to Internal Rotation in Ethane and Other Molecules" J. P. Lowe, JACS 1970, 92, 3799 Me Me Me

Calculate the the rotational barrier about the C1-C2 bond in isobutane

4-01-introduction 9/22/03 8:28 AM

D. A. Evans

Acyclic Conformational Analysis: Butane The 1,2-Dihaloethanes Butane

Chem 206

X
H C H H H H H

H H C

X X

X = Cl; H = + 0.91.3 kcal/mol X = Br; H = + 1.41.8 kcal/mol X = F; H = 0.6-0.9 kcal/mol

Using the eclipsing interactions extracted from propane & ethane we should be able to estimate all but one of the eclipsed butane conformations
Me C H Me H

Observation: While the anti conformers are favored for X = Cl, Br, the gauche conformation is prefered for 1,2-difluroethane. Explain. Discuss with class the origin of the gauche stabiliation of the difluoro anaolg. Recent Article: Chem. Commun 2002, 1226-1227 (handout)

staggered conformation

H H

Me H C MeH

eclipsed conformation
E=?

Eclipsed atoms 1 (HH) 2 (HMe)

E (kcal mol -1) +1.0 kcal mol -1 +2.8 kcal mol -1

Relationship between G and Keq and pKa


p. p. Recall that: or

G = RT Ln K G = 2.3RT Log10K
2.3RT = 1.4 (G in kcal Mol1 )

E est = 3.8 kcal mol -1 The estimated value of +3.8 agrees quite well with the value of +3.6 reported by Allinger (J. Comp. Chem. 1980, 1, 181-184)

At 298 K:

G298 = 1.4 Log10Keq


Since

n-Butane Torsional Energy Profile


H H HH C Me Me H C H

pKeq = Log10Keq
E1

E2

G298 = 1.4 pKeq


energy

H H

Me H C MeH H H

Hence, pK is proportional to the free energy change

H H

Me C

Keq 1.0 10 100

pKeq 0 1 2

G 0 1.4 2.8 kcal /mol

H Me A
Ref = 0

+3.6

Me Me G

+5.1

+0.88

4-02-introduction-2 9/22/03 8:33 AM

D. A. Evans Butane continued

Acyclic Conformational Analysis: Butane


Nomenclature for staggered conformers:
H H Me Me C H H H H H Me C H H Me Me

Chem 206
Me C H H H

From the torsional energy profile established by Allinger, we should be able to extract the contribution of the MeMe eclipsing interaction to the barrier:
H H C H H Me H H Me Me C HH

staggered conformation Me

eclipsed conformation

trans or t or (anti) Conformer population at 298 K: 70%

gauche(+) or g+ 15%

gauche(-) or g15%

E = +5.1 kcal mol-1

Let's extract out the magnitide of the MeMe interaction 2 (HH) + 1 (MeMe) = +5.1 1 (MeMe) = +5.1 2 (HH) p. p. 1 (MeMe) = +3.1

General nomenclature for diastereomers resulting from rotation about a single bond (Klyne, Prelog, Experientia 1960, 16, 521.)
RR C 0 R R C -60

R C

sp sc sc

+60

Incremental Contributions to the Barrier. Eclipsed atoms 2 (HH) 1 (MeMe) E (kcal mol -1) +2.0 +3.1
R Eclipsed Butane conformation

ac
R C -120

ac
+120 R C R 180 R C R

ap

From the energy profiles of ethane, propane, and n-butane, one may extract the useful eclipsing interactions summarized below: Hierarchy of Eclipsing Interactions

X
X Y

Y H

E kcal mol -1 +1.0 +1.4 +3.1


Energy Maxima Energy Minima Torsion angle 0 30 +60 30 +120 30 180 30 -120 30 -60 30

H
H

Designation syn periplanar + syn-clinal + anti-clinal antiperiplanar - anti-clinal - syn-clinal

Symbol sp + sc (g+) + ac

H H

C H

H Me Me Me

n-Butane Conformer E2 G E1 A E1 G

ap (anti or t) - ac - sc (g-)

4-03-butane 9/26/03 1:51 PM

D. A. Evans n-Pentane

Acyclic Conformational Analysis: Pentane The double-gauche pentane conformation


The new high-energy conformation: (g+g)
Me H
g-g-

Chem 206

Rotation about both the C2-C3 and C3-C4 bonds in either direction (+ or -):
Me Me H Me H
g+g+ g+t

Me H Me Me
tg-

H Me
g+g-

Me H

H Me

Me Me H

t,t

Me Me Me H H

H Me

Estimate of 1,3-Dimethyl Eclipsing Interaction Y

Me
tg+

Me H

X
g-t 1 g-g+

p. p.

1 (t,t) Anti(2,3)-Anti(3,4)

2 (g+t) Gauche(2,3)-Anti(3,4)

G = +5.5 kcal mol -1 G = X + 2Y where: X = 1,3(MeMe) & Y = 1,3(MeH) 1,3(MeH) = Skew-butane = 0.88 kcal mol -1 1,3(MeMe) = G 2Y = 5.5 1.76 = + 3.7 kcal mol -1

1 1 5 3

1,3(MeMe) = + 3.7 kcal mol -1

Estimates of In-Plane 1,2 &1,3-Dimethyl Eclipsing Interactions


3

4 (g+g) Gauche(2,3)-Gauche'(3,4) double gauche pentane

3 (g+g+)

Me

Me

Me

Me

Me

Me

Me

Me

Gauche(2,3)-Gauche(3,4) 3.1 ~ 3.7 ~3.9 ~ 7.6

From prior discussion, you should be able to estimate energies of 2 & 3 (relative to 1). On the other hand, the least stable conformer 4 requires additional data before is relative energy can be evaluated.

It may be concluded that in-plane 1,3(MeMe) interactions are Ca +4 kcal/mol while 1,2(MeMe) interactions are destabliizing by Ca 2.2 kcal/mol.

4-04-pentane 9/26/03 11:23 AM

D. A. Evans

Acyclic Conformational Analysis: Natural Products Lactol & Ketol Polyether Antibioitics

Chem 206

The syn-Pentane Interaction - Consequences


R. W. Hoffmann, Angew. Chem. Int. Ed. Engl. 2000, 39, 2054-2070 Conformation Design of Open-Chain Compounds (handout)

The conformation of these structures are strongly influenced by the acyclic stereocenters
Me O HO R O OH H Me Me OH O H O O Me OH Et Et OH Me

R Me R Me Me Me

R'

R'
or

Me H RR'
g -g -

Me H R'

Me H H Me
tt

Me

Me

Et

R'

Me
or

Me

Me H H R'
tg

H H R H
gt

Ferensimycin B, R = Me Lysocellin, R = H

Consequences for the preferred conformation of polyketide natural products p. p.


Analyze the conformation found in the crystal state of a bourgeanic acid derivative!

The conformation of these structures are strongly influenced by the acyclic stereocenters and internal H-bonding
Alborixin R = Me; X-206 R = H

Internal H-Bonding
OH Me Me Me Me O O OR Bourgeanic acid Me Me C O OH R Me OH H Me O OH O OH OH O Me Me Me H O Me Me OH O Et Me OH

Metal ion ligation sites (M = Ag, K)


Me Me O C O O R Me OH H Me O OH O OH OH O Me Me Me H O Me Me OH O Et Me OH

4-05-Natural Products 9/22/03 8:42 AM

D. A. Evans

Conformational Analysis: Ionophore X-206/X-rays X-ray of Ionophore X-206 H2O

Chem 206

X-ray of Ionophore X-206 - Ag+ - Complex


Metal ion ligation sites (M = Ag, K)
H Me Me OH Me O C O O R Me OH H Me O OH O OH OH O Me Me Me H O Me Me OH O Et Me OH

Internal H-Bonding
Me Me O C O OH Me OH H Me O OH O OH OH O Me Me Me O Me Me OH O Et

p. p.

"The Total Synthesis of the Polyether Antibiotic X-206". Evans, D. A.; Bender, S. L.; Morris, J. J. Am. Chem. Soc. 1988, 110, 2506-2526.

4-06-X-206 conformation 9/22/03 8:42 AM

D. A. Evans

Conformational Analysis: Ionophore X-206/X-ray overlay

Chem 206

p. p.

4-07-X-206 overlay 9/19/01 11:57 AM

D. A. Evans

Stabilized Eclipsed Conformations in Simple Olefins Butane versus 1-Butene


Me H C H H H Me H H Me Me

Chem 206

Simple olefins exhibit unusal conformational properties relative to their saturated counterparts
Propane versus Propene
109 H H H H Me H H H H H 120 CH2

staggered conformation

C H H

eclipsed conformation

G = +4 kcal mol-1

Me

Hybridilzation change opens up the CCC bond angle The Propylene Barrier
p. p.
H H C H CH2 H H CH2 C H

staggered conformation

CH2 C H H H

H H

Me CH2 C H

eclipsed conformation

= 50 staggered conformation

G = 0.83 kcal mol-1

=0

The Torsional Energy Profile = 50


Me H C H H C H H

+2.0 kcal/mol

= 180
H H C H H C Me H

eclipsed conformation

=0 New destabilizing effect


H C H Me C

H H H +1.32 kcal H C H H C H Me H +1.33 kcal

H X H C H H

repulsive interaction between CX & CH

+0.49 kcal

= 120
= 180

H X C H H

=0

Conforms to ab initio (3-21G) values: Wiberg, K. B.; Martin, E. J. Am. Chem. Soc. 1985, 107, 5035.

Acetaldehyde exhibits a similar conformational bias


O H H H H Me H H O H H H H O Me Me H H O Me

K. Wiberg, JACS 1985, 107, 5035-5041 K. Houk, JACS 1987, 109, 6591-6600

The low-energy conformation in each of above cases is eclisped

4-08-simple alkenes 9/22/03 8:54 AM

Useful Destabilizing Interactions to Remember

Hierarchy of Vicinal Eclipsing Interactions


X
X Y

Y H Me Me

E kcal mol -1 +1.0 +1.4 +3.1

H
H

H H

C H

H Me

Estimates of In-Plane 1,2 &1,3-Dimethyl Eclipsing Interactions


Me Me Me Me Me Me Me Me

~ 3.1

~ 3.7

~3.9

~ 7.6

It may be concluded that in-plane 1,3(MeMe) interactions are Ca +4 kcal/mol while 1,2(MeMe) interactions are destabliizing by Ca +3 kcal/mol.

0000 0000 000 0000 0000 0000 000 0000 0000 0000 000 0000 00 0 00 0 0 00 0 0 00 0 0000 00000000 0000 0000 00000000 0000 0000 00000000 0000

minimized structure

4-09-Destabilizing Effects 9/20/01 5:33 PM

D. A. Evans

Acyclic Conformational Analysis-2


Problems of the Day:

Chem 206

http://www.courses.fas.harvard.edu/~chem206/

Can you predict the stereochemical outcome of this reaction? O OTs Me


LiNR2

OLi EtO n-C4H9 H Me

OTs 1

Chemistry 206 Advanced Organic Chemistry


Lecture Number 5

EtO n-C4H9 H

+ 98:2

D. Kim & Co-workers, Tetrahedron Lett. 1986, 27, 943.

Acyclic Conformational Analysis-2


Conformations of Simple Olefinic Substrates Introduction to Allylic Strain Introduction to Allylic Strain-2: Amides and Enolates
O Me CH2OBn Me
B2H6 H2O2

OH O Me

CH2OBn

Me

diastereoselection 8:1

Reading Assignment for week A. Carey & Sundberg: Part A; Chapters 2 & 3
R. W. Hoffmann, Angew. Chem. Int. Ed. Engl. 2000, 39, 2054-2070 Conformation Design of Open-Chain Compounds (handout) R. W. Hoffmann, Chem. Rev. 1989, 89, 1841-1860 Allylic 1-3-Strain as a Controlling Element in Stereoselective Transformations (handout) F. Weinhold, Nature 2001, 411, 539-541 "A New Twist on Molecular Shape" (handout)

Y. Kishi & Co-workers, J. Am. Chem. Soc. 1979, 101, 259.

Me

Me
PhNCO Et3N

Me

Me

O N only one isomer

NO2

A. Kozikowski & Co-workers, Tetrahedron Lett. 1982, 23, 2081.

D. A. Evans

Wednesday, September 24, 2001

5-00-Cover Page 9/24/03 8:46 AM

D. A. Evans

Stabilized Eclipsed Conformations in Simple Olefins Butane versus 1-Butene


Me H C H H H Me H H Me Me

Chem 206

Simple olefins exhibit unusal conformational properties relative to their saturated counterparts
Propane versus Propene
109 H H H H Me H H H H H 120 CH2

staggered conformation

C H H

eclipsed conformation

G = +4 kcal mol-1

Me

Hybridilzation change opens up the CCC bond angle The Propylene Barrier
p. p.
H H C H CH2 H H CH2 C H

staggered conformation

CH2 C H H H

H H

Me CH2 C H

eclipsed conformation

= 50 staggered conformation

G = 0.83 kcal mol-1

=0

The Torsional Energy Profile = 50


Me H C H H C H H

+2.0 kcal/mol

= 180
H H C H H C Me H

eclipsed conformation

=0 New destabilizing effect


H C H Me C

H H H +1.32 kcal H C H H C H Me H +1.33 kcal

H X H C H H

repulsive interaction between CX & CH

+0.49 kcal

= 120
= 180

H X C H H

=0

Conforms to ab initio (3-21G) values: Wiberg, K. B.; Martin, E. J. Am. Chem. Soc. 1985, 107, 5035.

Acetaldehyde exhibits a similar conformational bias


O H H H H Me H H O H H H H O Me Me H H O Me

K. Wiberg, JACS 1985, 107, 5035-5041 K. Houk, JACS 1987, 109, 6591-6600

The low-energy conformation in each of above cases is eclisped

5-01-simple alkenes 9/22/03 8:54 AM

Evans, Duffy, & Ripin


5

Conformational Barriers to Rotation: Olefin A-1,2 Interactions


5

Chem 206

1-butene
4

2-propen-1-ol
H H H C C OH H H

H H H C C Me

H H

E (kcal/mol)

E (kcal/mol)

0 -180

-90

90

180

0 -180

-90

90

180

(Deg)
The Torsional Energy Profile = 50
Me H H H C H C H H H C H

(Deg)
The Torsional Energy Profile
H H C H C OH H

= 180
H C Me H H H

= 60
HO C H C H H

= 180

=0
C H Me H C

H H H

= 120
H C H C

OH H H

+1.32 kcal
H H C H C

Me H H

+1.33 kcal
H

+2.00 kcal

=0
C H HO C

H H H

+1.18 kcal

+0.49 kcal
=0

= 120
= 180

+0.37 kcal
= 180

=0

Conforms to ab initio (3-21G) values: Wiberg, K. B.; Martin, E. J. Am. Chem. Soc. 1985, 107, 5035.

5-02-1-butene & 2-propen-1-ol 9/23/03 2:59 PM

Evans, Duffy, & Ripin


5

Conformational Barriers to Rotation: Olefin A-1,2 Interactions-2


5

Chem 206

2-methyl-1-butene
4

2-methyl-2-propen-1-ol
4

H H H C C Me

H Me

H H H C C OH

H Me

E (kcal/mol)

E (kcal/mol)

0 -180

-90

90

180

0 -180

-90

90

180

(Deg)
The Torsional Energy Profile = 180 = 50
Me H H H C H C Me H H C H H H C H C H Me Me

(Deg)
The Torsional Energy Profile = 180 = 60
HO C Me H H H H C H C OH Me

= 120
OH

=0
C H Me =0 H C

= 110
H Me H

+1.39 kcal
H H C H C

Me Me H

+2.68 kcal

=0
C H HO = 180 =0 H C

H Me H

+1.16 kcal

H H

+2.01 kcal
Me

C H

C H

+0.06 kcal

+0.21 kcal
= 180

5-03-methylbutene etal 9/23/03 3:00 PM

Evans, Duffy, & Ripin


5

Conformational Barriers to Rotation: Olefin A-1,3 Interactions


5

Chem 206

(Z)-2-pentene
4 4

(Z)-2-buten-1-ol
H Me H C C OH H H

H Me

E (kcal/mol)

E (kcal/mol)

C Me

0 -180

-90

90

180

0 -180

-90

90

180

(Deg)
H Me C H Me C H H

(Deg)
The Torsional Energy Profile =0
Me C H HO C H H H Me

The Torsional Energy Profile =0

= 180
H C H C H OH H

+3.88 kcal

= 180 = 90
Me Me H C H C H H Me H H C H C Me H

+1.44 kcal
Me

= 120
OH C H C H H

+0.86 kcal

+0.52 kcal
= 180 =0

=0

= 180

Values calculated using MM2 (molecular mechanics) force fields via the Macromodel multiconformation search. 5-04-2-pentene/ z-2-buten-1-ol 9/23/03 3:00 PM

Review: Hoffman, R. W. Chem. Rev. 1989, 89, 1841.

Evans, Duffy, & Ripin


5

Conformational Barriers to Rotation: Olefin A-1,3 Interactions-2


OH Me Rotate clockwise C Me C H H

Chem 206

(Z)-2-hydroxy-3-pentene
4

Me OH Me

4.6 kcal/mol

HO

H C Me H

E (kcal/mol)

Me
3

Me

100
H OH

Me H

C H

0.3-0.4 kcal/mol

H Me H C C OH
-90 0 90 180

Me H

30

0 -180

Lowest energy conformer


Me Me Me OH H H C H

(Deg)
The Torsional Energy Profile
OH

60
Me

=0
H Me C H HO C H Me

Me C H Me

2.7 kcal/mol

C OH

H H

= -140

Needs to be redone = 110


Me Me C H H H OH C H OH

+2.72

????? = -80
HO Me C H C Me H H

+4.68

A(1,3) interaction 4.0 kcal/mol


R2
3

HO H H C C Me

H Me

30

= 80 Me
Me C C H H

Y X
2

R1

= 150
H C C Me H

R3

*
1

R large

+0.66H

Me

R small

Lowest energy conformer

+0.34
=0

+0.40 kcal

HO = 180

A(1,2) interaction 2.7 kcal/mol (MM2)

5-05-z-2-hydroxy-3-pentene 9/23/03 6:22 PM

D. A. Evans

Acyclic Conformational Analysis: Allylic Strain The Definition of Allylic Strain

Chem 206

Can you predict the stereochemical outcome of this reaction?


D. Kim & Co-workers, Tetrahedron Lett. 1986, 27, 943. O EtO R large 1 R small n-C4H9 H Me OTs
LiNR2

F. Johnson, Chem. Rev. 1968, 68, 375; Allylic Strain in Six-Membered Rings R. W. Hoffmann, Chem. Rev. 1989, 89, 1841-1860 (handout) Allylic 1-3-Strain as a Controlling Element in Stereoselective Transformations Houk, Hoffmann JACS 1991, 113, 5006

Consider the illustrated general structure where X & Y are permutations of C, N, and O:

R2

3 Y X 2

R1

OLi EtO n-C4H9 O H Me Me

OTs 1

+ 98:2

R3

Typical examples:
R2 R3 R1

R2 R large N

R1

R2 R large R + N

R1

R2 R large

R1 N

Relevant enolate
conformations
R large 1

EtO n-C4H9 H

major

O +

p. p.

R small Olefin

R small Imine

R small Imonium ion

R small Nitrone TsO(H2C)4 Me C Bu H C OR OLi Me Bu C C H (CH2)4OTs OR OLi Bu Me (CH2)4OTs OR C C OLi

In the above examples, the resident allylic stereocenter () and its associated substituents frequently impart a pronounced bias towards reactions occuring at the pi-bond. A(1,3) interaction R2 3 R1 Nonbonding interactions between the allylic Y substituents (Rlarge, Rsmall) & substituents at X R large the 2- & 3-positions play a critical role in R3 2 1 defining the stereochemical course of such R small reactions A(1,2) interaction Representative Reactions controlled by Allylic Strain Interactions
HO H Me HO O R OBn
Hg(OAc)2 NaBH4

A1

B1
critical conformations

C1 H

H OR Me TsO(H2C)4 C C Bu OLi Me

H C

Bu C

Bu OR OLi Me H C C OR OLi (CH2)4OTs

(CH2)4OTs

A2

B2

C2

R EtO2C OBn 2 n-C4H9

Me

HO Me

minor
H

diastereoselection 10:1 M. Isobe & Co-workers, Tetrahedron Lett. 1985, 26, 5199.

5-06-Allylic Strain-1 9/23/03 6:24 PM

D. A. Evans
O EtO n-C4H9 H O EtO n-C4H9 H
LiNR2 MeI

Allylic Strain & Enolate Diastereoface Selection


OTs Me
LiNR2

Chem 206
Ph
NH4Cl

O EtO n-C4H9 O EtO n-C4H9

Me diastereoselection 98:2 H Me3Si

Ph

OM OMe R
R-substituent R = Me diastereoselection 87:13 80:20 40:60

O OMe R

Me3Si

Me diastereoselection 89:11 H Ph Me3Si O

R = Et R = CHMe2

major diastereomer opposite to that shown

I. Fleming & Co-workers, Chem. Commun. 1985, 318. Y. Yamamoto & Co-workers, Chem. Commun. 1984, 904.

D. Kim & Co-workers, Tetrahedron Lett. 1986, 27, 943.

Ph
LiNR2

O OBn diastereoselection 90:10 at C3


one isomer at C2

OBn

MeCHO Me3Si 71% yield

MeO

p. p.

RO2C H CO2Me

MeO
LiNR2

RO2C H H CO2Me

O "one isomer" Me Bn N Boc O N S S


Sn(OTf)2 RCHO 91-95%

Me

OH

I. Fleming & Co-workers, Chem. Commun. 1986, 1198.

Br
95% yield

Me Bn N Boc R

O N OH

S S diastereoselection >95%

G. Stork & Co-workers, Tetrahedron Lett. 1987, 28, 2088.

TBSOCH2

Me CH2
LiNR2 MeI

Me CH2 TBSOCH2 Me H H CO2Me "one isomer" Me O


Ph(MeS)2CLi

T. Mukaiyama & Co-workers, Chem. Letters 1986, 637

H CO2Me

OMe

MeI 86%

MeS MeS MeS

Me

O OMe diastereoselection 99:1

T. Money & Co-workers, Chem. Commun. 1986, 288.

Me

K. Koga & Co-workers, Tetrahedron Letters 1985, 26, 3031.

R PhMe2Si

OM OEt

R
MeI

O OEt Me
R = Me: diastereoselection 99:1 R = Ph: diastereoselection 97:3

PhMe2Si

I CO2Et R

OLi O-t-Bu

CO2Et

R = H: one isomer

I. Fleming & Co-workers, Chem. Commun. 1984, 28.

KOt-Bu THF -78 C

CO2-t-Bu R = Me: > 15 :1 H R

Y. Yamaguchi & Co-workers, Tetrahedron Letters 1985, 26,1723.

5-07-A-strain enolates 9/20/01 4:45 PM

D. A. Evans
s The basic process
S H H
R

Allylic Strain & Olefin Hydroboration


Hydroborations dominated by A(1,3) Strain
S H
R

Chem 206

H
R

OH

H H
R

H2 B
R

H C R
R

O Me Me

CH2 OBn

B2 H6 H2 O2

O Me Me

CH2 OBn

C R

C R R

C R

diastereoselection 8:1 OMe Me O B2 H6 H2 O2 OH O Me Me Me diastereoselection 12:1 OH OMe OH

s Response to steric effects: Here is a good calibration system:


A
CH2 Me3 C H

Me

Me

Oxidant
MCPBA BH3, H2 O2 E

Ratio, A:E
69:31 34:66

Reference
JOC, 1967, 32, 1363 JOC, 1970, 35, 2654 BnO Me

Y. Kishi & Co-workers, J. Am. Chem. Soc. 1979, 101, 259.

OH OH Me Me B2 H6 H2 O2 BnO Me Me Me OH

s Acyclic hydroboration can be controlled by A(1,3) interactions:


OH RL RM Me OH R2 BH H2 O2 RL RM Me R R B C RL OH RL RM Me R2 BH H2 O2 OH RL RM Me R R RM B C RL H H C Me TrO CH2 OR OH H H C CH2 OR TrO

Diastereoselection = 3:1 C. H. Heathcock et. al. Tetrahedron Lett 1984 25 243. OH

major diastereomer

Me Me

control elements

A(1,3) allylic strain Steric effects; RL vs RM Staggered transition states

RM H

Me ThexylBH2 , OH then BH3 OTr TrO OH

Me

Me OTr OH OH 5:1

Diastereoselection;

major diastereomer

Me

Me

Me

Me

ThexylBH2 , then BH3 TrO

Me

Me

Me

Me OTr

OH

OH

OH

OH OTr

OH

OH

OH 4: 1

OH

Diastereoselection;

See Houk, Tetrahedron 1984, 40, 2257

Still, W.C.; Barrish, J. C. J. Am. Chem. Soc. 1983, 105, 2487.

5-07a-A-strain hydroboration 9/24/03 9:45 AM

D. A. Evans
Consider the resonance structures of an amide:
O R3 C N
1

Allylic Strain & Amide Conformation


R2 R3
3

Chem 206

Y X
2

R1 R large
1

The selection of amide protecting group may be done with the knowledge that altered conformational preferences may result:
O O H H

R1 R R

O R3

C N +

R1 R
1

R small

Favored for R = H, alkyl


H O H

N R

N R O

Favored for R = COR

A(1,3) interactions between the "allylic substituent" and the R1 moiety will strongly influence the torsion angle between N & C1. Favored
O Me C N Me

N H R H O N O H C R H H R

Disfavored

Me Me

Disfavored

H C O R

Favored

p. p.

conformations of cyclic amides


O R C N + R R C O H Me N R C Me O H R C O H N N R A(1,3)
Chow Can. J. Chem. 1968, 46, 2821

H R C O N

H strongly favored Me Me

R O 1 A(1,3) interaction between the C2 & amide C R substituents will strongly influence the torsion N angle between C1 & C2. R R
2

R O
1 2

N R + R

H strongly favored

As a result, amides afford (Z) enolates under all conditions


H H

O Me Ph N O

O published X-ray structure of this amide shows chair Me diaxial conformation


Quick, J. Org. Chem. 1978, 43, 2705

O Me

N H

L L

OM base favored O Me C N H L L H

Me

N L

Problem: Predict the stereochemical outcome of this cyclization.


OH D. Hart, JACS 1980, 102, 397 Ph N O HOCO
HCO2H

(Z)-Enolate
H

H O H C L N Me L base disfavored O H

H L

OM C N Me L H Me N L L

N Ph O diastereoselection >95%

identify HOMO-LUMO pair

5-08-A-strain Amides-1 9/23/03 6:09 PM

(E)-Enolate

D. A. Evans

Allylic Strain & Amide Conformation

Chem 206

A(1,3) Strain and Chiral Enolate Design


O Me N Bn H L L El O Me N Bn O LDA O or NaNTMS2 Me O M O O

Polypropionate Biosynthesis: The Acylation Event


O O SR O SR Me O SR Me OH O SR Me

N Bn

enolization selectivity >100:1


HO

R O

Acylation CO2

Reduction

El(+) JACS. 1982,104, 1737.


O O

O Me

N H

First laboratory analogue of the acylation event


O Et Cl O Me Li O N R O Me Me R O N O O O

favored enolization geometry p. p.

In the enolate alkylation process product epimerization is a serious problem. Allylic strain suppresses product enolization through the intervention of allylic strain
H O El C N Me L L O Me C N H El L L Me O C H El N L L

with M. Ennis JACS 1984, 106, 1154.

Diastereoselection ~ 97 : 3

Why does'nt the acylation product rapidy epimerize at the exocyclic stereocenter?? R O Me C
favored

H N H R R O R C N Me R R

While conformers B and C meet the stereoelectronic requirement for enolization, they are much higher in energy than conformer A. Further, as deprotonation is initiated, A(1,3) destabilization contributes significantly to reducing the kinetic acidity of the system These allylic strain attributes are an integral part of the design criteria of chiral amide and imide-based enolate systems
O O Me N Bn CH2OH Me N O O O Me N Me OH Me

Evans Tetr Lett. 1977, 29, 2495

Evans JACS 1982,104, 1737.

Myers JACS 1997, 119, 6496

X-ray structure

5-09-A-strain Amides-2 9/20/01 5:22 PM

D. A. Evans

Discodermolide

Chem 206

hinge Me HO O O H
16

Me
17

Me

Me Me Me OH Me

OH

O O

NH2

Me OH

- immunosuppressive activity - potent microtubule-stabilizing agent (antitumor activity similar to that of taxol)

The epimers at C16 and C17 have no or almost no biological activity.

The conformation about C16 and C17 is critical to discodermolide's biological activity.

S. L. Schreiber et al. JACS 1996, 118, 11061.


5-10-Discodermolide 9/19/01 12:14 AM

D. A. Evans

Conformational Analysis - Discodermolide X-ray 1


Me HO Me Me Me OH Me OH O O NH2 Me Me

Chem 206

O H

Me OH

5-11-Discodermolide X-ray1 9/21/01 8:34 AM

D. A. Evans

Conformational Analysis - Discodermolide X-ray 2


Me Me Me

Chem 206

HO O O H

16 Me Me Me OH Me OH O O NH2

Me OH

16

5-12-Discodermolide X-ray2 9/21/01 8:40 AM

D. A. Evans

Conformational Analysis: Part3

Chem 206

http://www.courses.fas.harvard.edu/~chem206/

Conformational Analysis of Cyclic Systems


Three Types of Strain:

Chemistry 206 Advanced Organic Chemistry

Prelog Strain: van der Waals interactions Baeyer Strain: bond angle distortion away from the ideal Pitzer Strain: torsional rotation about a sigma bond

Lecture Number 6

Baeyer Strain for selected ring sizes


"angle strain" size of ring Ht of Combustion Total StrainStrain per CH2 (kcal/mol) (kcal.mol) deviation from 10928' (kcal/mol)

Conformational Analysis-3
Conformational Analysis of C4 C6 Rings

Reading Assignment for week A. Carey & Sundberg: Part A; Chapter 3


Eliel & Wilen, "Stereochemistry of Organic Compounds, "Chapter 11, Configuration and Conformation of Cyclic Molecules, Wiley, 1994

3 4 5 6 7 8 9 10 11 12 13 14 15

499.8 656.1 793.5 944.8 1108.3 1269.2 1429.6 1586.8 1743.1 1893.4 2051.9 2206.1 2363.5

27.5 26.3 6.2 0.1 6.2 9.7 12.6 12.4 11.3 4.1 5.2 1.9 1.9

9.17 6.58 1.24 0.02 0.89 1.21 1.40 1.24 1.02 0.34 0.40 0.14 0.13

2444' 944' 044' -516'

Eliel, E. L., Wilen, S. H. Stereochemistry of Organic Compounds Chapter 11, John Wiley & Sons, 1994.

Ribeiro & Rittner, "The Role of Hyperconjugation in the Conformational Analysis of Methylcyclohexane and Methylheterocyclohexanes" J. Org. Chem., 2003, 68, 6780-6787 (handout)
de Meijere, "Bonding Properties of Cyclopropane & their Chemical Characteristics" Angew Chem. Int. Ed. 1979, 18, 809-826

Baeyer "angle strain" is calculated from the deviation of the planar bond angles from the ideal tetrahedral bond angle. Discrepancies between calculated strain/CH2 and the "angle strain" results from puckering to minimize van der Waals or eclipsing torsional strain between vicinal hydrogens. Why is there an increase in strain for medium sized rings even though they also can access puckered conformations free of angle strain? The answer is transannular strain- van der Waals interactions between hydrogens across the ring.

D. A. Evans
6-00-Cover Page 9/26/03 8:44 AM

Friday, September 26, 2003

Evans, Kim, Breit

Cyclopropane: Bonding, Conformation, Carbonium Ion Stabilization Cyclopropane


H

Chem 206

Carbocation Stabilization via Cyclopropylgroups

H H H

Necessarily planar. Subtituents are therefore eclipsed. Disubstitution prefers to be trans.

= 120 Almost sp2, not sp3 = 3080 cm-1


Me

Walsh Model for Strained Rings:


Rather than and * c-c bonds, cyclopropane has sp2 and p-type orbitals instead.

A rotational barrier of about 13.7 kcal/mol is observed in following example:


H

Me

NMR in super acids (CH3) = 2.6 and 3.2 ppm O

X-ray Structures support this orientation 1.302

1.517 1.478

1.222

R
1.474

(antibonding) H (antibonding) Nonbonding


side view

(antibonding)

1.464 1.541 1.409


H

1.444 1.534

Ph

(bonding)

(bonding)

1 (bonding) de Meijere, "Bonding Properties of Cyclopropane & their Chemical Characteristics" Angew Chem. Int. Ed. 1979, 18, 809-826 (handout)
de Meijere, A.; Wessjohann, L. "Tailoring the Reactivity of Small Ring Building Blocks for Organic Synthesis." Synlett 1990, 20.

R. F. Childs, JACS 1986, 108, 1692

6-01-Cyclopropane 9/25/03 7:55 PM

Evans, Kim, Breit


145-155
ax eq eq eq eq ax ax ax

Conformational Analysis: Cyclic Systems-2 Cyclobutane


H H H

Chem 206
H H H H H H H H H H H H H H H H H

Cyclopentane
H H H H H H

Eclipsing torsional strain overrides increased bond angle strain by puckering. Ring barrier to inversion is 1.45 kcal/mol.

H H H H

= 28

CsEnvelope

C2 Half-Chair

CsEnvelope

Two lowest energy conformations (10 envelope and 10 half chair conformations Cs favored by only 0.5 kcal/mol) in rapid conformational flux (pseudorotation) which causes the molecule to appear to have a single out-of-plane atom "bulge" which rotates about the ring.

(MM2)

Since there is no "natural" conformation of cyclopentane, the ring conforms to minimize interactions of any substituents present.

CsEnvelope (MM2)

H H

H H

H H

G = 1 kcal/mol favoring R = Me equatorial


1,3 Disubstitution prefers cis diequatorial to trans by 0.58 kcal/mol for di-bromo cmpd.

A single substituent prefers the equatorial position of the flap of the envelope (barrier ca. 3.4 kcal/mol, R = CH3). 1,2 Disubstitution prefers trans for steric/torsional reasons (alkyl groups) and dipole reasons (polar groups). Me Me X

X 1,3 Disubstitution: Cis-1,3-dimethyl cyclopentane 0.5 kcal/mol more stable than trans.

1,2 Disubstitution prefers trans diequatorial to cis by 1.3 kcal/mol for diacid (roughly equivalent to the cyclohexyl analogue.)

H H

A carbonyl or methylene prefers the planar position of the half-chair (barrier 1.15 kcal/mol for cyclopentanone). X

6-02-Conform/cyclic-2 9/25/03 7:55 PM

Evans, Kim, Breit

Conformational Analysis: Cyclic Systems-3

Chem 206

Methylenecyclopentane and Cyclopentene


Strain trends: > > Decrease in eclipsing strain more than compensates for the increase in angle strain.

Cyclohexane Energy Profile (kcal/mol)


Half-Chair

Boat +1.01.5 10.711.5

Relative to cyclohexane derivatives, those of cyclopentane prefer an sp2 center in the ring to minimize eclipsing interactions. "Reactions will proceed in such a manner as to favor the formation or retention of an exo double bond in the 5-ring and to avoid the formation or retention of the exo double bond in the 6-ring systems." Brown, H. C., Brewster, J. H.; Shechter, H. J. Am. Chem. Soc. 1954, 76, 467.

+5.5 +5.5 Chair Twist Boat Inverted Chair

Examples:
H H H H H H H H O O NaBH4 H

H OH

k6

H H H

NaBH4

H H OH H

k6 = 23 k5

k5

Brown, H. C.; Ichikawa, K. Tetrahedron 1957, 1, 221.


O O

E = +6.5-7.0 E = +5.5

hydrolyzes 13 times faster than

Conan, J-Y.; Natat, A.; Priolet, D. Bull. Soc. Chim., Fr. 1976, 1935.
O O OEt OH O OEt

The barrier: +10.7-11.5 E=0

95.5:4.5 keto:enol

76:24 enol:keto

Brown, H. C., Brewster, J. H.; Shechter, H. JACS 1954, 76, 467.

6-03-Conform/cyclic-3 9/25/03 7:57 PM

Evans, Breit

Conformational Analysis: Cyclic Systems-4

Chem 206

Monosubstituted Cyclohexanes: A Values


R H

A Values depend on the relative size of the particular substituent.


H H H H Me H H H Me Me H Me Me Me H

Keq
H

G = RTlnKeq

Meaxial has 2 gauche butane interactions more than Meequatorial. Expected destabilization: 2(0.88) kcal/mol = ~1.8 kcal/mol; Observed: 1.74 kcal/mol
Me H C H H H H Me H H C Me H C H

AValue

1.74

1.80

2.15

5.0

The "relative size" of a substituent and the associated A-value may not correlate. For example, consider the CMe3 and SiMe3 substituents. While the SiMe3 substituent has a larger covalent radius, it has a smaller A-value:
Me C Me H Me Si Me H Me Sn Me H

The A Value, or -G, is the preference of the substituent for the equatorial position.

Me

Me

Me

AValue

4.5-5.0

2.5

1.1

Can you explain these observations?

The impact of double bonds on A-values:


Lambert, Accts. Chem. Res. 1987, 20, 454
R H H R

substituent R = Me R = OMe R = OAc

G 0.8 0.8 0.6

A-value (cyclohexane) 1.74 0.6 0.71

The Me substituent appears to respond strictly to the decrease in nonbonding interactions in axial conformer. With the more polar substituents, electrostatic effects due to the trigonal ring carbon offset the decreased steric environment.

6-04-Conform/cyclic-4 9/25/03 7:59 PM

Rigberio & Rittner, The Role of Hyperconjugation in the Conformational Analysis of Methylcyclohexane and Methylheterocyclohexanes JOC 2003, 68, 6780 Commentary by Ken Houk University of California, Los Angeles Department of Chemistry Dear David, The calculations in the Ribeiro article look fine, but I am not convinced by the interpretation. It does seem to work pretty well for many systems, but not obviously for the isomeric 1,3-dioxane cases they note early on. There seems no explanation of why CH hyperconjugates better than CC. Further, the results with alkyls larger than methyl still require traditional steric arguments. I would say that the equatorial methyl preference has been attributed in part to hyperconjugative effects that occur when the CH bonds are anti-periplanar. But I would not yet go much beyond that! Best regards, (b. 1943) A.B. 1964, Ph.D. 1968, Harvard University; Assistant-Full Professor, Louisiana State University, 1968-1980; Alfred P. Sloan Fellow, 1975-1977; Camille and Henry Dreyfus Teacher-Scholar, 1972-1977; LSU Distinguished Research Master, 1978; Professor, University of Pittsburgh, 1980-1985; Alexander von Humboldt Senior U.S. Scientist Award, 1982; Akron Section, American Chemical Society Award, 1983; Arthur C. Cope Scholar Award, 1988; Director, Chemistry Division, National Science Foundation, 1988-1990; James Flack Norris Award in Physical Organic Chemistry, 1991; Schrdinger Medal, World Association of Theoretically Oriented Chemists, 1998; Tolman Medal, Southern California Section, American Chemical Society, 1999; Fellow of the American Academy of Arts and Sciences, 2002; American Chemical Society Award for Computers in Chemical and Pharmaceutical Research, 2003; International Academy of Quantum Molecular Science, 2003.

D. A. Evans

Bond Lengths and A-Values of Methyl Halides

Chem 206

CF: 1.39

CCl: 1.79

CBr: 1.95

CI: 2.16

F A-value: 0.250.42

Cl A-value: 0.5364

Br Avalue: 0.48-0.67

I A-value: 0.470.61

p. p.

Chem 3D Pro (Verson 5.0)


6-04a methyl halides 9/26/01 8:26 AM

Evans, Breit

Conformational Analysis: Cyclic Systems-5

Chem 206

The impact of trigonal Carbon


Let's now compare look at the carbonyl analog in the 3-position
Me H O O H Me

Polysubstituted Cyclohexane A Values


As long as the substituents on the ring do not interact in either conformation, their A-values are roughly additive 1,4 Disubstitution: A Values are roughly additive.
Me Me Me Me Me Me Me Me

G = 1.36 kcal/mol versus 1.74 for cyclohexane Let's now compare look at the carbonyl analog in the 2-position
Me Me3C O H base epimerization Me3C O H Me

G = 0 kcal/mol

G = 2(1.74) = 3.48 kcal/mol

G = 1.56 kcal/mol versus 1.74 for cyclohexane However, the larger alkyl groups do not follow the expected trend. Can you explain? (see Eliel, page 732)
CHMe2 Me3C O H base epimerization Me3C O H CHMe2

1,3 Disubstitution: A Values are only additive in the trans diastereomer


H X H Me H Me H X

G = A(Me) A(X)

H X H Me X H

G = 0.59 kcal/mol versus 2.15 for cyclohexane


CMe3 Me3C O H base epimerization Me3C O H

Me

The new interaction! For X = Me


H CMe3 Me H Me H H H H Me Me

G = 1.62 kcal/mol versus 5.0 for cyclohexane


6-05-Conform/cyclic-5 9/25/03 8:01 PM

+ 0.88

+ 0.88

G = 2(.9) + 1(+3.7)= 5.5 kcal/mol + 3.7

Evans, Breit
Me Ph

Conformational Analysis: Cyclic Systems-6 Let's now consider vicinal substitution


Me H Ph

Chem 206

Let's now consider geminal substitution


Me

Case 1:
Me

HH Me H

Me

The prediction:

G = A(Ph) A(Me) G = +2.8 1.7 = +1.1 kcal/mol

Observed:

G = 0.32 kcal/mol

The prediction:

G = 1 gauche butane 2A(Me) G = +0.88 2(1.74) = +2.6 kcal/mol

Hence, when the two substituents are mutually interacting you can predict neither the magnitude or the direction of the equilibrium. Let's analyze this case. Allinger, Tet. Lett. 1971, 3259

Observed:

G = +2.74 kcal/mol

If the added gauche butane destabilization in the di-equatorial conformer had not been added, the estimate would have been off.

6-06-Conform/cyclic-6 9/25/03 8:13 PM

000000 000000 0 0 00 00 0 000000 000


A G = +2.8

0000 00 00 00000 0000 00 00 0000 00


B G = 0.32

Case 2:
H Me Me OH H OH H H Me Me

The conformer which places the isopropyl group equatorial is much more strongly preferred than would be suggested by A- Values. This is due to a syn pentane OH/Me interaction. Problem: Can you rationalize the stereochemical outcome of this reaction?

00 0000000 0 00 0000000 00

000 0 0000000 0000000

O EtO LiNR2 MeI H EtO

Me

D
n-C4H9

n-C4H9

diastereoselection 89:11

Note the difference in the Ph substituent in A & B.

D. Kim & Co-workers, Tetrahedron Lett. 1986, 27, 943.

Evans, Breit

Conformational Analysis: Cyclic Systems-7

Chem 206

Heteroatom-Substituted 6-Membered Rings


A-values at the 2-position in both the O & N heterocycles are larger than expected. This is due to the shorter CO (1.43 ), and CN (1.47 ) bond lengths relative to carbon (CC; 1.53 ). The combination of bond length and bond angle change increases the indicated 1,3-diaxial interaction (see eq 1, 4).

A-Values for N-Substituents in Piperidine


H N

The Reference:
N H

G = 0.36 kcal/mol

Me H

Reference:

H Me

Me N N

G = 1.74 kcal/mol
Hydrogen is "bigger" than the Nlone Pair.
Me

Me

G = 3.0 kcal/mol

Me H O Me O

(1)

G = 2.86 kcal/mol

The A-value of Nsubstituents is slightly larger than the corresponding cyclohexane value. Rationalize

H H O H H Me O Me

Me

H H O O Me

(2)

O Me

G = 1.43 kcal/mol

O O

G = 4.0 kcal/mol

(3)

G = 1.95 kcal/mol

O Me H

H Me

O O

Me H

O O

G = 0.8 kcal/mol

H H N H H H N H H H Me N Me H Me

(4)
N

H Me

G = 2.5 kcal/mol

The indicated distance is 2.05 . The analogous HH distance in Cyclohexane is 2.45

(5)
H

N Me

G = 1.6 kcal/mol

(6) H N

G = 1.9 kcal/mol 2.05 2.45

6-07-Conform/cyclic-7 9/25/03 8:02 PM

Evans, Breit

Conformational Analysis: Bicyclic Ring Systems

Chem 206

Conformations of Bicyclic Ring Systems


Me Me H O 4 7 H H 1 10 14 11 13 17

Estimate the energy difference between the two methyl-decalins shown below.
Me Me

see Elier, p 780 Hydrindane Ring System (6/5)


H H

The steroid nucleus provided the stimulation for the development of conformational analysis, particularly of polycyclic ring systems. D. H. R. Barton was awarded a Nobel prize in 1969 for his contributions in this area.

flexible
H H

rigid

Decalin Ring System (6/6)


H H

G = 0.5 kcal/mol
rigid The turnover to favor the cis fusion results from the entropic preference for the less ordered cis isomer.

mobile

H H

The 5-5 Ring System


H H H H

favored

2.4 kcal/mol

Relative G

G = +6.4 kcal/mol
H

Let's identify the destabilizing gauche butane interactions in the cis isomer
H

3
H

Gauche-butane interactions
2

Me
A B

H
C D

R
A

H Me H
B

H
C D

4 1

C1 C2 C1 C3 C4 C3 G(est) = 3(0.88) = 2.64 kcal/mol

A/B Trans

A/B Cis

Rationalize the conformational flexibility of a A/B Trans vs. A/B Cis Steroid!

6-08-Conform/bicyclics 9/25/03 8:04 PM

Evans, Breit

Conformational Analysis: Axial vs Equatorial Reactivity


SN2 Reactions (Displacement with PhS)
H Me3C OTs Me3C OTs

Chem 206

Different reactivity for axial and equatorial substituents


Axial substituents are more hindered, thus less reactive in many transformations Acetylation with Ac2O/Py
H OH OH H

k rel

31

The axial diastereomer is not always slower reacting: Alcohol Oxidation with Cr(6+)
H Me3C OH Me3C OH H

k rel
Me3C

1
H OH Me3C

0.13

OH H

k rel

0.27

k rel

1
Me H OH Me Me Me

3.2
Me OH H

Acid-catalyzed esterification
H CO2H CO2H H

Me

k rel

3.36

k rel

1
H

0.04
CO2H CO2H Me3C H

The rate-determining step is breakdown of the chromate ester. This is an apparent case of strain acceleration

Steric Hindrance and Steric Assistance


TS TS G > G TS G < G

k rel

Me3C

0.05

Ester Saponification
H Me3C CO2Et Me3C CO2Et H

G A

G ref

G B

k rel

20

1
GS GS G G GS Reference Case

For a more detailed discussion of this topic see: Eliel, E. L., S. H. Wilen, et al. (1994). Stereochemistry of Organic Compounds pp 720-726
6-09-Axial-equat react 9/25/03 8:05 PM

GA > G ref

GB < G ref

D. A. Evans

Conformational Transmission

Chem 206

Observation: Relative enolate stability correlates to ring junction stereochemistry


H H H H H LiO H LiO

Let be defined as the torsion or dihedral angle for butane


Me C H H

Me

A Me B C Me

base
O H H

illustrated may be designated as B.

ratio: 13 : 87
H H H

Let's now consider cyclohexane


Perfect chair real chair

base
O H LiO H LiO H

= 60 CCC 109 28'

= 56 CCC 111

ratio: 70 : 30 House, JOC 1965, 30, 1341

Observation: Relative enolate stability seems to be correlated to position of C=C


H H H

Given cyclohexane with an identified torsion angle R, if R either increases or decreases wht effects in angle change are transmitted to O, M, and P?

R = 56
base
O H H LiO H H LiO H H

R = 0 15 44 61

[] 41 11 +6

R
ratio: 10 : 90 Operation: 56
LiO H LiO H

P O M

O M P

56 56 56

[] = R( 0) R( 56)
56 56 56 56 H2 15 0* 44 61

base
O H

56

ratio: 92 : 8

How do we explain the experimental observations shown above?

Hence, relative to cyclohexane, the following notation for torsion angle change may be denoted: 11 +6 For P For M

Readings: Velluz etal, Angew. Chemie, Int Ed. 1965, 4, 181-270

6-10-Conformal transmission 9/25/03 8:09 PM

D. A. Evans
Operation:
H

Conformational Transmission-2
H H R C R H H H R C R H Me Me O H H H Me Me O H LiO H LiO H

Chem 206
H

base
H

ratio: 10 : 90 effects opposing effects reinforcing

Operation:
H

base
H LiO H LiO H

Simple Application: Reinforcing Torsional Effects


R R

ratio: 92 : 8

effects reinforcing

effects opposing

A
R

versus

A
R

Question: Which is the more stable C=C isomer in the two THC structures?
Me OH Me OH

Which C=C bond isomer is more stable?


R Me

Me O C5H11 Me O C5H11

1) C=C will open up ring=B torsion angle B 2) Ring B will resist increase in its ring fusion torsion angle 3) Therefore torsion effects are opposed

Me

R. W. Kierstead, JACS 1967, 89, 5934 Question: Which enol acetate is more stable?

OBz

1) C=C will close down ring=B torsion angle 2) Ring B will accomodate decrease in its ring fusion torsion angle

OBz

OBz

Ac2O TSOH or
O AcO AcO

3) Therefore torsion effects are reinforcing

6-11-Conformal transmission-2 9/25/03 8:12 PM

Evans, Breit Cycloheptane

Conformational Analysis: Cylcoheptane Cyclooctane


Chair-Boat Lowest-energy conformation 7 1 3

Chem 206

Chair (2.16 kcal/mol)

Twist-Chair (0 kcal/mol)

Transannular strain between C3 & C7

Ring strain originates in eclipsing interactions and transannular van der Waals interactions Methyl position G 1 2 2.8 3 >4.5 4 -0.3 5 6.1

(pseudoeqatorial) 1.8 (pseudoaxial) (kcal/mol)

Underside view of boat-chair C3 & C7 eclipsing interactions Boat (3.02 kcal/mol) Twist-Boat (2.49 kcal/mol) 7

Hendrickson, J. B. JACS 1961, 83, 4537. See Eliel, page 762+

Olefins are preferentially orientated to eliminate eclipsing interactions. 1 7

6-12-Conform/cycloheptane 9/24/01 7:43 AM

Evans, Breit Cyclooctane continued...


Chair-Boat (BC) Lowest-energy conformation

Conformational Analysis: Cyclooctane

Chem 206

7 1 Transannular strain between C3 & C7 3 Chair-Boat (CB) conformation reference structure Chair-Chair (CC) conformation (+1-1.6 kcal/mol)

Methyl position G

2 2.8

3 >4.5

4 -0.3

5 6.1

(pseudoeqatorial) 1.8 (pseudoaxial) (kcal/mol)

Cyclooctane derivatives

Boat-Boat (BB) conformation (>+ 8 kcal/mol)


Olefin is positioned at C3-C4 or C6-C7
Me O Nu X Me

Carbonyl is positioned at C3 or C7

LINR2 MeI
Me

Predict stereochemistry

Me

Nu

LiCuMe2

Predict stereochemistry

Disubstitution occurs at C4 or C6

SN2 occurs at C1 and C5

Me

Me

Still, W. C.; Galynker, I. Tetrahedron 1981, 37, 1981.

6-13-Conform/cyclooctane 9/24/01 7:43 AM

D. A. Evans

Conformational Analysis: Part4


s Other Reading Material
The Curtin-Hammett Principle

Chem 206

http://www.courses.fas.harvard.edu/~chem206/

Chemistry 206 Advanced Organic Chemistry


Lecture Number 7 Leading References:

J. I. Seeman, J. Chem. Ed. 1986, 63, 42-48. J. I. Seeman, Chem Rev. 1983, 83, 83-134. Eliel, pp. 647-655 Carey & Sundberg,Part A, CH 4, pp 187-250

Conformational Analysis-4
s Ground State Torsional Effects (Conformational Transmission) s Conformational Analysis of C6 C8 Rings continued s Transition State Torsional Effects s CurtinHammett Principle (Will not cover in lecture)

s Problems of the Day: (To be discussed)


Predict the stereochemical outcome of this reaction. The diastereoselection is 99:1
O mCPBA H N Ph O Ph O N H

s Reading Assignment for week Eliel & Wilen, Stereochemistry of Carbon Compounds" Chapter 11 Configuration and Conformation of Cyclic Molecules (handout) A. Carey & Sundberg: Part A; Chapter 4 "Study & Descrption of Reaction Mechanisms"
K. Houk, Science. 1986, 231, 1108-1117 Theory & Modeling of Stereoselective Organic Reactions (Handout) Monday, September 29, 2003

Martinelli, et.al. Tett. Lett. 1989, 30, 3935 Rationalize the stereochemical outcome of this reaction.
CO2Me Me Me CO2Me Me

Me Me

O O

LiNR2 Me-I

Me Me

O O

diastereoselection is 8:1.

D. A. Evans
7-00-Cover Page 9/29/03 8:57 AM

Ladner, et.al. Angew. Chemie, Int. Ed 1982, 21, 449-450

D. A. Evans

Conformational Transmission

Chem 206

Observation: Relative enolate stability correlates to ring junction stereochemistry


H H H H H LiO H LiO

Let be defined as the torsion or dihedral angle for butane


Me C H H

Me

A Me B C Me

base
O H H

illustrated may be designated as B.

ratio: 13 : 87
H H H

Let's now consider cyclohexane


Perfect chair real chair

base
O H LiO H LiO H

= 60 CCC 109 28'

= 56 CCC 111

ratio: 70 : 30 House, JOC 1965, 30, 1341

Observation: Relative enolate stability seems to be correlated to position of C=C


H H H

Given cyclohexane with an identified torsion angle R, if R either increases or decreases wht effects in angle change are transmitted to O, M, and P?

R = 56
base
O H H LiO H H LiO H H

R = 0 15 44 61

[] 41 11 +6

R
ratio: 10 : 90 Operation: 56
LiO H LiO H

P O M

O M P

56 56 56

[] = R( 0) R( 56)
56 56 56 56 H2 15 0* 44 61

base
O H

56

ratio: 92 : 8

How do we explain the experimental observations shown above?

Hence, relative to cyclohexane, the following notation for torsion angle change may be denoted: 11 +6 For P For M

Readings: Velluz etal, Angew. Chemie, Int Ed. 1965, 4, 181-270

7-01-Conformal transmission 9/25/03 8:09 PM

D. A. Evans
Operation:
H

Conformational Transmission-2
H H R C R H H H R C R H Me Me O H H H Me Me O H LiO H LiO H

Chem 206
H

base
H

ratio: 10 : 90 effects opposing effects reinforcing

Operation:
H

base
H LiO H LiO H

Simple Application: Reinforcing Torsional Effects


R R

ratio: 92 : 8

effects reinforcing

effects opposing

A
R

versus

A
R

Question: Which is the more stable C=C isomer in the two THC structures?
Me OH Me OH

Which C=C bond isomer is more stable?


R Me

Me O C5H11 Me O C5H11

1) C=C will open up ring=B torsion angle B 2) Ring B will resist increase in its ring fusion torsion angle 3) Therefore torsion effects are opposed

Me

R. W. Kierstead, JACS 1967, 89, 5934 Question: Which enol acetate is more stable?

OBz

1) C=C will close down ring=B torsion angle 2) Ring B will accomodate decrease in its ring fusion torsion angle

OBz

OBz

Ac2O TSOH or
O AcO AcO

3) Therefore torsion effects are reinforcing

7-02-Conformal transmission-2 9/25/03 8:12 PM

Evans, Breit Cycloheptane

Conformational Analysis: Cylcoheptane Cyclooctane


Chair-Boat Lowest-energy conformation 7 1 3

Chem 206

Chair (2.16 kcal/mol)

Twist-Chair (0 kcal/mol)

Transannular strain between C3 & C7

Ring strain originates in eclipsing interactions and transannular van der Waals interactions Methyl position G 1 2 2.8 3 >4.5 4 -0.3 5 6.1

(pseudoeqatorial) 1.8 (pseudoaxial) (kcal/mol)

Underside view of boat-chair C3 & C7 eclipsing interactions Boat (3.02 kcal/mol) Twist-Boat (2.49 kcal/mol) 7

Hendrickson, J. B. JACS 1961, 83, 4537. See Eliel, page 762+

Olefins are preferentially orientated to eliminate eclipsing interactions. 1 7

7-03-Conform/cycloheptane 9/26/03 1:26 PM

Evans, Breit Cyclooctane continued...


Chair-Boat (BC) Lowest-energy conformation

Conformational Analysis: Cyclooctane

Chem 206

7 1 Transannular strain between C3 & C7 3 Chair-Boat (CB) conformation reference structure Chair-Chair (CC) conformation (+1-1.6 kcal/mol)

Methyl position G

2 2.8

3 >4.5

4 -0.3

5 6.1

(pseudoeqatorial) 1.8 (pseudoaxial) (kcal/mol)

Cyclooctane derivatives

Boat-Boat (BB) conformation (>+ 8 kcal/mol)


Olefin is positioned at C3-C4 or C6-C7
Me O Nu X Me

Carbonyl is positioned at C3 or C7

LINR2 MeI
Me

Predict stereochemistry

Me

Nu

LiCuMe2

Predict stereochemistry

Disubstitution occurs at C4 or C6

SN2 occurs at C1 and C5

Me

Me

Still, W. C.; Galynker, I. Tetrahedron 1981, 37, 3981.

7-04-Conform/cyclooctane 9/26/03 2:31 PM

D. A. Evans Torsional Effects

Ground State Torsional Effects


Relevant Orbital Interactions:
H H H H H H H H CH & electrons are destabilizing C C H H H H C C C C H H B H H H H A H C C H

Chem 206

Torsional Strain: the resistance to rotation about a bond Torsional energy: the energy required to obtain rotation about a bond Torsional Angle: also known as dihedral angle Torsional steering: Stereoselectivity originating from transition state torsional energy considerations

H H

H CH's properly aligned for overlap hence better delocalization

G = +3 kcal mol-1

Dorigo, A. E.; Pratt, D. W.; Houk, K. N. JACS 1987, 109, 6591-6600.

Conformational Preferences: Acetaldehyde staggered


Torsional Strain (Pitzer Strain): Ethane

eclipsed
A H O H CH2 C H H H H CH2 H

H H

H O H H H B

staggered conformation

The eclipsed conformation (conformation A) is preferred. Polarization of the carbonyl decreases the 4 electron destabilizing Rotational barrier: 1.14 kcal/mol Houk, JACS 1983, 105, 5980-5988.

+2.0 kcal/mol

eclipsed conformation H

C H

Conformational Preferences 1-Butene (X = CH2); Propanal (X = O)

H
Wiberg K. B.; Martin, E. J. Amer. Chem. Soc. 1985, 107, 5035-5041. A

H X H Me H B

Me X H

See Lecture 4 for previous discussion

Conformation A is preferred. There is little steric repulsion between the methyl and the X-group in conformation A.

7-05-torsional effects-1 9/29/03 8:14 AM

D. A. Evans

Transition State Torsional Effects According to Houk

Chem 206

Houk: "Torsional effects in transition states are more important than in ground states"
H*
H H 2C H C H H H H 2C C H

H*
H H H 2C H

H*
H C H H C RL C

Transition states
H-radical and H-anion: antiperiplanar CR orbital stabilized the TS illustrated for Nu addition

C-RL lumo C-Nu

C-RL

0 no H*
+

30 60 90 120
+1.6 +4.7

C-Nu homo

Nu

Forming bond

H
2 Kcalmol-1

Same trends are observed for H+ addition Houk, Science 1981, 231, 1108-1117 "The Theory and Modeling of Stereoselective Organic Reactions"

Forming bond

0 +2.4 0

Nu Transition state H H C H H C H

H0

+5.3

C-RL C-Nu

RL
Nu C R H

30 60 90 120 Ground state

Houk, JACS 1981, 103, 2438 Houk, JACS 1982, 104, 7162

C-RL

RL
7-06-Torsional effects-2 9/29/03 8:17 AM

C-Nu

D. A. Evans

Transition State Torsional Effects: Olefin Additions


Olefin Addition Reactions: Case two

Chem 206

Olefin Addition Reactions: Case one


How do we account for the high exo selectivities in addition reactions to norbornene? exo
H H

How do we account for the high selectilvities in the oxidation of the indicated olefin?
OH OH H N N Ph 98 : 2 diastereoselectivity O Ph 99 : 1 diastereoselectivity O O mCPBA H N H

Highly exo selective for electrophilic, nucleophilic and cycloaddition reactions Rate enhancement due to strain

OsO4

endo The Controversy over origin of high exoselectivities Steric effects Least nuclear motion Orbital distortion Schleyer: torsional steering B A

Ph O

Nitrogen protecting group does not affect selectivities

B A Favored

Schleyer, P. R. J. Amer. Chem. Soc. 1967, 89, 701.

Martinelli, et.al. Tett. Lett. 1989, 30, 3935

Addition from exo face avoids eclipsing A & B hydrogens (better hyperconjugative stabilization of transition state)

Addition from indicated olefin face avoids eclipsing A & B H's (better hyperconjugative stabilization of transition state) Martinelli has carried out further studies on related structures...........

7-07-torsional effects-3 9/29/03 8:18 AM

D. A. Evans

Transition State Torsional Effects: Olefin Additions Martinelli: Torsional steering important in selectivity
O mCPBA H

Chem 206

89

99 : 1 diastereoselectivity

major

mCPBA

Me

Me

62

50 : 50 diastereoselectivity

63
O

mCPBA H

major 99 : 1 diastereoselectivity 40

74

Authors propose that diastereoselection controlled by TS torsional effects Martinelli & Houk, J. Org. Chem. 1994, 59, 2204.
7-08-torsional effects-4 9/29/03 8:18 AM

D. A. Evans

Cyclohexanone Addition Reactions: Hydride Reduction


HA
OH OH H Me3C H 3 H

Chem 206
Nu:

Stereoselective Reductions: Cyclic Systems


O Me3C H H

HE

Attack Trajectories for Cyclohexanone (Torsional Argument)

[H]

Me3C

% Axial Diastereomer
0 10 20 30 40 50 60 70 80 90 100

HE HA

Nu:

This approach favored sterically

DIBAL-H 72:28 NaBH4 79:21 LiAlH(Ot-Bu)3 92:8 LiAlH4 93:7

L-Selectride 8:92 K-Selectride 3:97


H B
M+

Me C CH2Me H
3

The steric hindrance encountered by Nu-attack from the axial C=O face by the axial ring substituents (hydrogens in this case) at the 3-positions is more severe than the steric hinderance encountered from Nu-attack from the equatorial C=O face.

Axial Attack H:

Observation: Increasingly bulky hydride reagents prefer to attack from the equatorial C=O face. The most stereoselective Reductions
O Me3C H H OH OH H Me3C H H

OCCHe dihedral: +63.0

[H]

Me3C

OCCHe dihedral: +4.0

H:

Reagent KBH(s-Bu)3 Li in NH3


NPh Me3C H H Me3C H NHR Me C 3

Ratio 03 :97 99 :01


NHR H H

Equatorial Attack

OCCHe dihedral: 56.0

The Issues Associated with the Reduction Process


Steric Effects: Torsional Effects: Attack across equatorial C=O face sterically more favorable. However, attack across the axial face of the C=O group avoids development of eclipsing interactions in the transition state. (Note the dihedral angle sign changes between reactants & products shown above). These "torsional effects" favor axial attack. For "small" hydride reagents such as LiAlH4, torsional effects are felt to be dominant and this explains the predisposition for axial attack. For "large" hydride reagents such as HBR4, steric effects now are dominant and this explains the predisposition for equatorial attack.

[H]

(R) Ganem, Tet. Let 1981, 22, 3447 Hutchins, JOC 1983, 48, 3412 private communication R = Bn R = Ph

Reagent LiBH(s-Bu)3 LiBH(s-Bu)3 Al/Hg/MeOH

Ratio 03 :97 01 :99 ~90 :10 Prediction

Prediction

7-09-Cyclohexanon/C=O-1 9/29/03 8:21 AM

K. A. Beaver, D. A. Evans
Leading References:

Conformational Analysis and Reactivity: Curtin-Hammett Principle


Case 1: "Kinetic Quench"

Chem 206

J. I. Seeman, J. Chem. Ed. 1986, 63, 42-48. J. I. Seeman, Chem Rev. 1983, 83, 83-134. See also Eliel, pp. 647-655

How does the conformation of a molecule effect its reactivity?


Consider the following example:
N
MeI

k1, k2 >> kA, kB: If the rates of reaction are faster than the rate of interconversion, A and B cannot equilibrate during the course of the reaction, and the product distribution (PB/PA) will reflect the initial composition.
[PB] = [PA] [B]o [A]o

Me

Me

13

13

MeI

Energy

GAB G1 G2 G

PB

p. p. Do the two different conformers react at the same rate, or different rates? What factors determine the product distribution?

PA

A
Rxn. Coord.

The situation:
Consider two interconverting conformers, A and B, each of which can undergo a reaction resulting in two different products, PA and PB.

In this case, the product distribution depends solely on the initial ratio of the two conformers.
steric hindrance
less stable

Me Me H N O -78C G = -3.0 kcal/mol (ab initio calculations) Me

Me
more stable

PA
major

k1

kA kB

k2

PB
minor

(1)

H O
MeBr

We'll consider two limiting cases: (1) The rate of reaction is faster than the rate of conformational interconversion (2) The rate of reaction is slower than the rate of conformational interconversion
If the rates of conformationall interconversion and reaction are comparable, the reactants are not in equilibrium during the course of the reaction and complex mathmatical solutions are necessary. See Seeman, Chem. Rev. 1983, 83 - 144 for analytical solutions.

MeBr

Me Me N minor product O Me H

Me Me N O H Me major product

While enolate conformers can be equilibrated at higher temperatures, the products of alkylation at -78 C always reflect the initial ratio of enloate isomers. Padwa, JACS 1997 4565

7-10- Curtin-Hammett Intro 9/26/03 2:49 PM

K. A. Beaver, D. A. Evans
Case 2: Curtin-Hammett Conditons

Curtin - Hammett: Limiting Cases

Chem 206

k1, k2 << kA, kB: If the rates of reaction are much slower than the rate of interconversion, (GAB is small relative to G1 and G2), then the ratio of A to B is constant throughout the course of the reaction.
PA
k1

To relate this quantity to G values, recall that Go = -RT ln Keq or Keq = e-G/RT, k1 = e-G1/RT, and k2 = e-G2/RT. Substituting this into the above equation:
[PB] [PA] = k2 k1 Keq = e-G2/RT e-G1/RT (e-G/RT) = e-G2/RTe-G/RTeG1/RT

kA kB

k2

slow

slow

PB

(1)

(4)

fast
G

Combining terms:
[PB] [PA]

= e-(G2 + G-G1)/RT or

[PB] [PA]

e-G/RT

G1 Energy

GAB

G2

Where G = G2+G-G1
PB

p. p.

A
PA

Curtin - Hammett Principle: The product composition is not solely dependent on relative proportions of the conformational isomers in the substrate; it is controlled by the difference in standard Gibbs energies of the respective transition states.

major

Rxn. Coord.

minor

The Derivation:
d[PB] d[P ] Using the rate equations dt A = k1[A] and dt = k2[B] we can write:
d[PB] d[PA] = k2[B] k1[A]

Within these limits, we can envision three scenarios: If both conformers react at the same rate, the product distribution will be the same as the ratio of conformers at equilibrium. If the major conformer is also the faster reacting conformer, the product from the major conformer should prevail, and will not reflect the equilibrium distribution. If the minor conformer is the faster reacting conformer, the product ratio will depend on all three variables in eq (2), and the observed product distribution will not reflect the equilibrium distribution. This derivation implies that you could potentially isolate a product which is derived from a conformer that you can't even observe in the ground state!

or

d[PB] =

k2[B] k1[A]

d[PA]

(2)

Since A and B are in equilibrium, we can substitute Keq =


k2 k1

[B] [A]
(3)

d[PB] =

Keq d[PA]

Integrating, we get

[PB] [PA]

k2 K k1 eq

When A and B are in rapid equilibrium, we must consider the rates of reaction of the conformers as well as the equilibrium constant when analyzing the product ratio.

7-11-C-H Derivation 9/29/03 8:39 AM

K. A. Beaver, D. A. Evans
Tropane alkylation is a well-known example.
N Me Me N

Some Curtin-Hammett Examples


Enantioselective Lithiation:

Chem 206

N i-Pr2N O

less stable

more stable

H Me

i-Pr2N

Li Me

s-BuLi

(-)-Sparteine

faster

13

MeI

13

MeI

slower (-)-Sparteine
13

13

Me + Me N

Me + N

Me

major product

minor product

i-Pr2N

Because sparteine is chiral, these two complexes are diastereomeric and have different properties.
Lisparteine Me

Lisparteine Me

i-Pr2N

p. p.

The less stable conformer reacts much faster than the more stable conformer, resulting in an unexpected major product!
JOC 1974 319

faster
i-Pr2N O

Cl

slower

Oxidation of piperidines:
Me N N Me3C H

i-Pr2N Me

less stable

Me3C

Me

Me

more stable
82 - 87% ee

Keq = 10.5 k1
Me N+ O

slower

H2O2

k2 faster
O Me3C H N + Me

Enantioselectivities are the same, regardless of whether or not the starting material is chiral, even at low temperatures. Further, reaction in the absence of (-)-sparteine results in racemic product. Note that the two alkyllithium complexes MUST be in equilibrium, as the enantioselectivity is the same over the course of the reaction. If they were not equilibrating, the enantioselectivity should be higher at lower conversions.

Me3C

minor product

Ratio: 5 : 95

major product

When the equilibrium constant is known, the Curtin-Hammett derivation can be used to calculate the relative rates of reaction of the two conformers. Substituting the above data into [PB]/[PA] = k2K/k1, the ratio k2/k1 ~ 2.
Note that in this case, the more stable conformer is also the faster reacting conformer! Tet. 1972 573 Tet. 1977 915

This is a case of Dynamic Kinetic Resolution: Two enantiomeric alkyl lithium complexes are equilibrating during the course of a reaction with an electrophile. Beak, Acc. Chem. Res, 1996, 552

7-12-Simple C-H examples 9/29/03 8:38 AM

K. A. Beaver, D. A. Evans

Mechanism of Asymmetric Hydrogenation


P Rh S,S P S Ph S MeO2C NHAc

Chem 206

The asymmetric hydrogenation of prochiral olefins catalyzed by Rhodium is an important catalytic process.
[L2Rh]+

MeO2C

NHAc

MeO2C

NHAc

coordination
MeO2C NH HN

coordination
CO2Me P Ph O Me Me Rh P

Ph

Ph

> 95% ee Enantioselectivities are generally very high when the ligand is a chelating diphosphine. (ee's are given for S,S-CHIRAPHOS)

P Rh P Ph O

minor

major

When a chiral ligand is used, there are two diastereomeric complexes which may be formed:
MeO2C NH P HN P Ph O Me Me Ph O Rh P CO2Me

hydrogen addition
MeO2C P

faster

slower

hydrogen addition
CO2Me

NH Ph O Me Me

HN Ph O

P P Rh H H

p. p.

Rh P

H Rh P H

minor complex 1
H2 fast

major complex
(NMR, X-Ray) H2

slow

migration
P

+S

+S

migration

MeO2C R

NHAc

MeO2C S

NHAc

H Rh P S O Me

CH2Ph CO2Me R NH

PhH2C MeO2C S S HN

P H Rh P O Me

Ph

Ph

observed product
Observations: Complex 2 is the only diasteromer observed for the catalyst-substrate complex (1HNMR, X-Ray crystallography) in the absence of hydrogen The enantioselectivity is strongly dependant on the pressure of H2, and degrades rapidly at higher hydrogen pressures The observed enantiomer is exclusively derived from the minor complex 2
MeO2C R Ph NHAc

reductive elimination

-L2RhS2

-L2RhS2

reductive elimination

MeO2C S

NHAc

These observations may be explained using the Curtin - Hammett Principle

Ph

> 95% ee

Halpern, Science, 217, 1982, 401

7-13-Halpern Mechanism 9/25/01 9:59 PM

K. A. Beaver, D. A. Evans

Reactions Involving Interconverting Isomers

Chem 206

The Curtin-Hammett treatment can be extended to ANY case where different products are formed from two rapidly intereconverting starting materials, whether they are conformers, tautomers or isomers.
k1 kA kB k2

PA
major

PB
minor

"It was pointed out by Professor L. P. Hammett in 1950 (private communication) that ..." David Y. Curtin, 1954 " Because Curtin is very generous in attributing credit, this is sometimes referrred to as the Curtin-Hammett principle rather than the Curtin principle." Louis Plack Hammett, 1970

Stannylene ketals provide an efficient way to acylate the more hindered site of 1,2-diols.
O Cl O 2N Ph O SnBu2 O O 2N O Cl

p. p.

Ph

OCOAr OSnBu2Cl

Ph

O O

Ph Ar OSnBu2Cl

OSnBu2Cl OCOAr

more stable

less stable

Ratio 2:1
TMS-Cl faster Ar= p-NO2C6H4 slower TMS-Cl

Curtin - Hammett Principle: The product composition is not solely dependent on relative proportions of the conformational isomers in the substrate; it is controlled by the difference in standard Gibbs energies of the respective transition states.

Ph

OCOAr OTMS

Ph

OTMS OCOAr

Product Ratio 22:1

THE TAKE-HOME LESSON: Never assume that the most stable conformation of a compound is the most reactive. It may be, but then again, it may not.

The two stannyl esters are in equilibrium at room temperature, and the more stable isomer is initially formed more slowly. The stannyl esters are allowed to equilibrate before quenching with TMS-Cl, which reacts more rapidly with the less hindered primary alkoxystannane.
JOC 1996, 5257

7-14-Isomer examples 9/25/01 10:00 PM

D. A. Evans

Olefin Addition Reactions: Part1


Problems of the Day: (To be discussed)

Chem 206

http://www.courses.fas.harvard.edu/~chem206/

Rationalize the stereochemical outcome of these reactions.


OH
9-BBN H2O2

Chemistry 206 Advanced Organic Chemistry


Lecture Number 8

OH Me2CH

OH diastereoselection 24:1

Me2CH Me

Me

W. C. Still & J. C. Barrish, J. Am. Chem. Soc. 1983, 105, 2487. Me NHCONHPh Ph
m-CPBA CH2Cl2, 0 C 75 %

Me O

NHCONHPh Ph

Olefin Addition Reactions1


Hydroboration Epoxidation & Directed Epoxidation

Roush, J. Org. Chem. 1987, 52, 5127.

Diastereoselection = 95 : 5

Reading Assignment for week


A. Carey & Sundberg: Part B; Chapter 4 "Electrophilic Additions to CC Multilple Bonds" K. Houk, Science. 1986, 231, 1108-1117 Theory & Modeling of Stereoselective Organic Reactions (Handout) K. Houk, Tetrahedron. 1984, 40, 2257-2274 Theoretical Studies of Stereoselective Hydroboration Reactions (Handout) Hoveyda, Evans, Fu, Chem Rev. 1993, 93, 1307-1370 Substrate-Directable Chemical Reactions (Electronic Handout) E. Vedejs, JACS 2003, 125, 10502-3 A Mechanistic Alternative for the Intramolecular Hydroboration of Homoallylic Amine and Phosphine Borane Complexes (Electronic Handout)
+

Web Problem 163: The following is an idea that has been proposed to you by a fellow student. The proposal is based on the fact that boranemethyl sulfide complex is an effective hydroboration reagent (eq 1). It is proposed that homoallylic sulfides such as that illustrated below should be capable of "directing" the hydroboration process from this substituent through the borane-substrate complex.

H 2B
-SMe2

further reaction with cyclohexene

(1)

Me2SBH3 + SMe
Me2S +Me2S

Me

BH3
directed hydroboration

Me

S
(2)

???
Me Me

BH2

Me

D. A. Evans
8-00-Cover Page 10/1/03 8:02 AM

Wednesday, October 1, 2003

Part A. In order to begin your critique, you must possess a good working knowledge of the details of the hydroboration of olefins with boranemethyl sulfide. Provide a clear depiction of the transition state for the hydroboration process using ethylene as the olefinic substrate and boranemethyl sulfide as the hydroborating agent. Part B. Now, based on your knowledge of the hydroboration reaction and the principles learned thus far in Chem 206, critique the idea proposed in Eq 2. You must concisely state the logic upon which you base your assessment. Pictures speak a thousand words.

D. A. Evans

Olefin Addition Reactions: Introduction


Representative Trans-Addition Processes

Chem 206

Representative Cis-Addition Processes

Hydrometallation
R C C R H H

Halogenation
M H

M H

M = B, Al, etc

R C C R H H

R C C R H H

Br

Br

H R C C H Br

Br

Hydrogenation
R C C R H H

M-catalyst
H H

Oxymetallation (M = Hg(II), Tl(III)


R C C R H H

R C C R H H

Group Transfer (epoxidation)


R C C R H H O

+ Hg(OR)2

RO R H R C C H HgOR

RO2H

ROH

R C C R H H O Os O O O

Oxysulfenation (M = S(II), Se(II)


R C C R H H

Group Transfer (dihydroxylation)


R C C R H H

RSX

RO R H R C C H SR

OsO4

N2

Group Transfer (cyclopropanation)


R C C R H H

R C C R H H R2 C R C C R H H R R O

Attributes: Each process may proceed via an bridged intermediate where X is the initiating electrophile Olefin substitution may disrupt bridging
X R C C R H H

M-catalyst + R2C=N2 N2

Cycloadditions (one of many!)


R C C R H H

Addition of hydrogen halides


R C C R H H H R H H R H

+ R2C=C=O

N2

R C C R H H

+ HX

R C C H X

H C C R X

Attributes: Each process adds to the C=C via a stereospecific process Intermediates may be involved in some of the indicated reactions

Attributes: Process may proceed via an bridged intermediate where H+ is the initiating electrophile Olefin substitution, reaction conditions as well as halide type may disrupt bridging
H R C C R H H

8-01-Olefin addn-intro 9/29/03 11:29 AM

D. A. Evans
The basic process
S H B H H
R

Allylic Strain & Olefin Hydroboration


Hydroborations dominated by A(1,3) Strain
S H H
R R OH CH2OBn Me R Me
B2H6 H2O2

Chem 206

H
R

H 2B
R

H C

O Me

CH2OBn Me

R
OMe O Me Me Me OH
B2H6 H2O2

diastereoselection 8:1 OMe OH O Me Me Me diastereoselection 12:1 OH

+ R

Response to steric effects: Here is a good calibration system:


A CH2 Me3C

Oxidant
MCPBA BH3, H2O2

Ratio, A:E
69:31 34:66

Reference
JOC, 1967, 32, 1363 JOC, 1970, 35, 2654

Y. Kishi & Co-workers, J. Am. Chem. Soc. 1979, 101, 259.


OH BnO Me Me Me OH
B2H6 H2O2

p. p.

BnO Me Me Me

OH Diastereoselection = 3:1

Acyclic hydroboration can be controlled by A(1,3) interactions:


OH RL RM Me
B2 H 6

OH

RL RM Me

H2O2

OH major diastereomer

C. H. Heathcock et. al. Tetrahedron Lett 1984 25 243.

control elements

A(1,3) allylic strain Steric effects; RL vs RM Staggered transition states RM TrO Me H RL H C B H C H H CH2OR

Me

Me

Me
ThexylBH2, then BH3

Me OTr

TrO OH OH OH

H RM

H B C RL H H C CH2OR Me

OH

OTr

Diastereoselection;

5:1 Me OTr

major

minor

Me

Me

Me

Me

ThexylBH2, then BH3 TrO

Me

Me

Me

TrO

OH

OH

OH

OTr

OH

OH

OH

OH 4: 1

OH

Diastereoselection;

Houk, "Theoretical Studies of Stereoselective Hydroboration Reactions" Tetrahedron 1984, 40, 2257 (Handout)

Still, W.C.; Barrish, J. C. J. Am. Chem. Soc. 1983, 105, 2487.

8-02-hydroboration-1 9/29/03 11:31 AM

D. A. Evans

Allylic Strain & Olefin Hydroboration


Case II: Dialkylboranes
R
R2BH H2O2

Chem 206
R H H R R RL H C H Me Me RL OH
R2BH H2O2

Hydroborations dominated by A(1,2) Strain


Me RL RM RM
R2BH H2O2

H OH

Me RL

TS1 R2BH structure is a potential variable


major

RM Me C

B H C

Me RL RM OH favored for R2BH

Me RL RM OH

TS2
minor

H H

B RM C

R2BH H2O2

Houk's rules: Orient RL anti-periplanar to incoming reagents to avoid TS eclipsing: Case I: Borane
H H H H B RM C H C RL A(1,2) Strain H Me
BH3 H2O2

RM RL Midland finds that TS1 favored for R2BH reagents, but TS1 ~ TS2 for BH3 Others have found that TS1 favored over TS2 for BH3

TS2
major

Me RL RM OH

p. p. A
H RM Me C RL from Lecture 4:

Representative Examples
favored for BH3 Me Me Me OH H CH2 H
9-BBN H2O2

TS1
minor

B H C

H H H
BH3 H2O2

Me Me H H R2BH

CH2OH H

RL RM

The Torsional Energy Profile


H = 180 H C H C H OH C H Me H
9-BBN

diastereoselection 1:1 4:1 14 : 1 26 : 1

Me Me

M. M. Midland & Co-workers, J. Am. Chem. Soc. 1983, 105, 3725..

= 50
Me H C H

borane methylsulfide thexylborane 9-BBN dicyclohexylborane

OH R

OH

R Me

H2O2

R = n-Bu: diastereoselection 11:1 R = CHMe2: diastereoselection 24:1

=0
H C H Me
=0

= 110
H C Me H +0.06 kcal +1.39 kcal H C H H C H Me Me

+2.68 kcal

Me

Model is consistent if you presume HO = RM: R = RL

W. C. Still & J. C. Barrish, J. Am. Chem. Soc. 1983, 105, 2487.


= 180

8-03-hydroboration-2 10/1/03 8:17 AM

D. A. Evans
Case I: Dialkylboranes
H R R

Allylic Strain & Olefin Hydroboration


Case II: Borane
Me H H R R B RM C RL H H C RL Me R2BH H2O2 RM RL H Me OH H B RM C R2BH H2O2 RL RM OH H H RM Me C RL H H C RL Me BH3 H2O2 RL RM Me OH B H C H H H BH3 H2O2 Me RL RM OH B H C

Chem 206

TS1
major

RM Me C

TS1
minor

favored for R2BH

minor

TS2

H H

TS2
major

H H

favored for BH3 OMe Me A Me O OH H OMe Me Me Me O B C 9 O Me C-9 C10 Me D O E O O Me OMe Me OMe F Me OH OMe Me O BH3SMe2 85% OMe O O N Me O
1 5 5

p. p.

HO2C

Evans, Ratz, Huff, Sheppard, JACS 1995, 117, 3448-3467.

Me H

Lonomycin A

Me OH
10 9

OH

XP

Me Me OH
10 9

O OMe H

Me

Me

H H B RO H C H

H H C RL H Me RO Me H C RL B H C H H H

diastereoselection 92 : 8 OMe Me O XP
1 5

TS1 favored
R

TS2 disfavored
R

Me Bn

O OMe H

Me

Me

Me OH
10 9

OH

R H H

B RO C

H C RL

H Me

RO Me

H C RL

B H C

R H H

9-BBN 60%

Me

O OMe H

Me

Me

diastereoselection > 95 : 5

TA1 disfavored

TA2 favored

8-04-hydroboration-3 9/29/03 11:33 AM

D. A. Evans

Represetative Hydroboration Examples: Acyclic Control


Me CO2H CH2 Me
Wolinsky, J.; Eustace, E. J. J. Org. Chem. 1972, 37, 3376.

Chem 206
Me H O O

For each of the examples shown below, attempt to rationalize the stereochemical outcome of the reaction in terms of one of the models presented in the discussion.

1. 9-BBN 2. H2 O2 , NaOH

Me OH Et Et Me O Et H O Et OH
B2H6 H2O2

Me R Et Me

Me

Diastereoselection = 7:1 O O

Et Et

CO2H CH2 Me

Me

"one isomer"
Y. Kishi & Co-workers, J. Am. Chem. Soc. 1978, 100, 2933.

1. 9-BBN 2. H2 O2 , NaOH
H Me

Me O Me O Me

Me

Wolinsky, J.; Nelson, D. Tetrahedron. 1968, 25, 3767.

Diastereoselection = 10:1 Me

BH3 .THF

O Me O

Me Me Me OH CH2

p. p.
Mori, K. Tetrahedron 1976, 32, 1979

HO

B2H6 / [ O ]
OH

Me

diastereoselection 12:1

OH Diastereoselection = 19:1 Me

R H O O CH3 O OBn CH3 BH3 THF


.

R HO H O Me O CH3 O OBn CH3 OH Me CH2

B2H6 / [ O ]
OH

Me

Oikawa et. al. Tetrahedron Lett. 1983, 19, 1987.

R=H; Diastereoselection = 6.8:1 R=OBn Diastereoselection = 6.6:1

OH Diastereoselection = 32:1 Me

Me Me

B2H6 / [ O ]
OH

Me

OH BH3.THF

OH OH OH H Me Me Me OH CH2 CH2 OH Diastereoselection = 10:1

Birtwistle et. al. Tetrahedron Lett. 1986, 25, 243.

Diastereoselction = 6.7:1 R = CH3; R = isopropyl "One Compound"

B2H6 / [ O ]
OH

Me

Schulte-Ette, K.H.; Ohloff, G. Helv. Chim. Acta 1967, 50, 153.

OH Diastereoselection = 4.6:1

8-05-hydroboration-4 9/29/03 12:38 PM

D. A. Evans
CH2 Me3C BH3.THF

Representative Hydroboration Examples: Cyclic Systems


OH Me3C BnO HO OH H O H O Me
B. Fraser-Reid et. al. J. Am. Chem. Soc. 1984, 106, 731.

Chem 206
OH H O H OH O Me

BH3.THF

BnO HO

Diastereoselection = 2.1:1

Major isomer; no ratio given.

CH2 Me3C Me Me

BH3.THF Me3C Me

OH Me CH3 O H O CH2 O BH3.THF H N H O H

Diastereoselection = 1.2:1

H N H O

CH3

OH

p. p.
Me3C

CH2 Me

BH3 THF Me3C Me

OH

Sallay, S. I. J. Am. Chem. Soc. 1967, 89, 6762.

90% yield, no diastereoselection given

Diastereoselection = 3.3:1

Me CH2 Me3C BH3.THF Me3C Me OH Me

BH3.THF Me O H H CO2Me

Me OH

Me

O H CO2Me

CH2

Diastereoselection = 2.4:1

Ley, S. et.al. J. Chem. Soc. Chem. Commun. 1983 630.

55% yield with the diastereomeric alcohol produced in an unspecified amount. Recycling of the minor isomer further provided 15% of the desired material

CH2 Me

BH3.THF Me

OH Me

NNHAr BH3 THF Me


.

Me

NNHAr

Y. Senda et. al. Tetrahedron 1977, 33, 2933.

Diastereoselection = 4.9:1 (Compare with H.C. Brown's case, with 9-BBN; 1.5:1)

Me Me H

Me

OH

McMurry, J. E. J. Am. Chem. Soc. 1968, 90, 6321.

Minor diastereomer not detected

8-06-hydroboration-5 9/29/03 12:39 PM

D. A. Evans

Directed Reactions: An Introduction


Heteroatom-directed Reactions
Mechanism-based: (HO & C=C must be allylic) [3,3]
R OH disfavored R A B H H C C H
Et2Zn

Chem 206

Stereochemical Control Elements for all reactions


Steric & Electronic Factors Stereoelectronic Considerations Associative Substrate-Reagent Interactions Steric control:
R R

O R

R O CH2 R

A B

Claisen Rearrangement

A B

via Reagent Ligation

favored product p. p.

A B

favored

R OH

CH2I2

R O Zn CH2I

CH2 R OH

Nonbonding Interactions disfavor the syn diastereoface

Simmons-Smith Reaction

Directed Reactions
Review: Hoveyda, Evans, Fu Chem. Reviews 1993, 93, 1307
MCPBA

Hydroxyl-directed Reactions
Cl CO3H

Associative Substrate-Reagent Interactions


A B X A B X A B X
A C B favored product H H C H disfavored OH favored

ratio 92 : 8
O OH

Henbest J. Chem. Soc. 1958, (1957)

t-BuOOH VO(acac)2

ratio 98 : 2

Sharpless JACS 95, 6136, (1973)

Noncovalent Interaction favors the syn diastereoface Directed Oxidations


Epoxidation Hydroboration

A B

Et2Zn CH2I2

CH2 OH Winstein JACS 91, 6892, (1969)

ratio 90 : 10
Me
M(I) + H2

Agenda

Directed Reductions
Hydrogenation Hydride reduction

Me

Directed CC Bond Constructions


OH

8-07-Directed Rxns/intro 10/1/03 8:19 AM

OH

(Ir+) Stork JACS 105, 1072 (1983) (Rh+) Evans JACS 106, 3866 (1984)

D. A. Evans

Directed Reactions: An Introduction Orientation of the Directing Group


H

Chem 206 Peracid Epoxidation

A. Rao in Comprehensive Organic Synthesis, Trost, Ed., 1992, Vol 7, Chapter 3.1

H Me OH

Me

reagent

X Me Me OH

s General Reaction:
X = O, CH2
R R O R O OH q R R O

+
R R R

q O
R

+
R OH

maj
Reag

H Me OH C H Me H
Reag

Me C C OH H H

HOMO CC LUMO CC

LUMO *OO HOMO O lone pair

note labeled oxygen is transferred

O-O bond energy: ~35 kcal/mol

H Me

min TSminor

s Transition state: What about lone pairs. [Consider q to be Sp2 hybridized].


R O
H

R R O O O
H

R O O

TSmajor

q O

O
H

q O

q O
R C C H R C H H H R

q O
C C H H

Orientation of directing group is not the same for all reactions


R

R C C H H R

Reagent t-BuO2H, V +5 RCO3H CH2I2, ZnCu

Selectivity 71 : 29 95 : 5 > 99 : 1

Estimate ~ 50 ~ 120 ?

HOMO CC LUMO: OO

HOMO: O lone pair LUMO: CC

s Reaction rates governed by olefin nucleophilicity. The rates of epoxidation of the indicated olefin relative to cyclohexene are provided below: OH
OH OAc

The transition state bite angles for the above reactions are either not known or have been only crudely estimated. The "best guesses" are provided.

1.0

0.6

0.05

0.4

s The indicated olefin in each of the diolefinic substrates may be oxidized selectively.
Me Me Me Me Me Me Me

H Me Me

8-08-Directed Rxns/intro-2 10/1/03 9:27 AM

D. A. Evans

Diastereoselective Peracid Epoxidation


The Directed Peracid Epoxidation

Chem 206

Stereoelectronic Implications of intramolecular Peracid Epoxidation


Per-arachidonic acid Epoxidation: Corey, JACS 101, 1586 (1979)
O O O O H
H

Transition State Hydrogen Bonding: Substrate as H-bond donor (Henbest)


R O O O
H

R O

O
H

R O

O
R C C


R R C H C H O R

H O R

O R H C C

H O

C H2

C H2

C H2

require allylic or homoallylic alcohol

Transition State Hydrogen Bonding: Peracid as H-bond donor (Ganem) p. p.


O
H

CF3 O O H C C H2 C H R R O C H2 O
H

CF3 O

O
H

CF3 O

O
C C H R R

O C H2

O H C C H

require more acidic peracid both allylic alcohols and ethers OK

OH

Syn : Anti (m-CPBA) 24 : 1

Syn : Anti (CF3CO3H) 50 : 1

OTBS

Syn : Anti (m-CPBA) 1:7

Syn : Anti (CF3CO3H) 5:1

OH O H O O Me3C OH

OTBS

24 : 1

100 : 1
Me3C OTBS

1:8

12 : 1

5:1
Me3C

100 : 1
Me3C

1:4

1:6

8-09-peracid epoxid-2 9/29/03 12:52 PM

Ganem Tet. Let. 1985, 26, 4895

D. A. Evans

Diastereoselective Peracid Epoxidation


Epoxidation of Cyclic Homoallylic Alcohols

Chem 206

Epoxidation of Cyclic Olefins with Amide &Urethane Directing Groups

Substrate
O HN HO Me HO

Major Product
O HN Me

Selectivity

Substrate
OH O

Major Product
OH Me OH

Selectivity
9:1

O HO O HN Ph HN HO O Ph

"highly selective"

Me

OH

HO Et Me Me

HO Et

"highly selective"
O Me Me

p. p.
O O R O O R

"highly selective"
HO

16 : 1
HO Me O Me

a. R = NH2 b. R = NHBn

3:1 5:1 10 : 1
HO

1:1
HO Me O Me

c. R = NMe2
Me Me

a. R = OCONHBn b. R = OCONMe2

>20 : 1 >20 : 1
HO HOH2C HO O HOH2C

21 : 1

R RO

R RO

a. R = CONH2 b. R = CONHBn

6:1 >10 : 1 2:1


AcO AcO O

5:1

AcO

AcO

c. R = CONMe2

Conditions: Perbenzoic acid, or meta-chlorobenzoic acid in benzene.

Conditions: Perbenzoic acid, or meta-chloroperbenzoic acid in benzene or cyclopentane.

(Table 11, p1316, from the Evans, Hoveyda, Fu review article)

(Table 14, p1318, from the Evans, Hoveyda, Fu review article)

8-10-peracid epoxid-3 9/27/01 10:16 PM

D. A. Evans The Sharpless Epoxidation


OR RO O V O O RO HO V O O R O+ RDS

Sharpless Epoxidation (V+5)

Chem 206

The literature precedent: Sheng, Zajecek, J. Org. Chem. 1970, 35, 1839
TBHP
OR RO O V O O OH O O

80 oC Catalyst

OH

OH

VO(acac)2 Mo(CO)6

4:1 1:1

ROH OR RO O V OR O HO

ROOH

Next step: Sharpless, Michaelson JACS 1973, 95, 6136


OH OH Me

HO

O Me Me

VO(acac)2 TBHP 80 C
Me Me OH

Me O

Regioselection 20:1

p. p.

Aldrichimica Acta, 12, 63 (1979)

OH

Mo(CO)6 TBHP 80 C
O

Stereoselection 98:2 (90 % yield)

OC2C3C4 = 41 The Sharpless estimate: ~50

Relative Rates (Diastereoselectivities) for the Epoxidation of Cyclohexene Derivatives JACS 1973, 95, 6136 Substrate krela,b (diastereoselectivityc ) peracid
1.00
OH O-OtBu O OAc

t-BuOOH
HO HO

t-BuOH
O

Mo(CO)6
1.00

VO(acac)2
1.00

2 1 3 RO 4 O

tBu O V O tBu O O

OR V

0.55 (92 : 8)

4.5 (98 : 2)

>200 (98 : 2)

0.046 (37 : 63) 0.07 (40 : 60)


OH O

--

Chem 3D Transition State

slow

RO V O O

0.42 (60 : 40)


a,b

11.0 (98 : 2)

10.0 (98 : 2)

The relative rate data apply only to a given column. Values in parenthesis refer to the ratio of syn:anti epoxide.

8-11-Sharpless-1 9/29/03 1:07 PM

D. A. Evans
Allylic Alcohols:
Me OH
Reagent

Epoxidation of Acyclic Alcohols


OH
Reagent

Chem 206
OH Me O Me
+

OH Me O Me

Me O

OH Me
threo +

Me

OH Me
erythro

Me

Me

Me

Reagent m-CPBA t-BuOOH / VO(acac)2 t-BuOOH / Mo(CO)6 t-BuOOH / (t-BuO)3Al


OH R1 R2 SiMe3
t-BuOOH VO(acac)2

Estimate ~ 120 40-50

Reagent m-CPBA t-BuOOH / VO(acac)2 t-BuOOH / Mo(CO)6


OH H Me H Me C Me C

Ratio 95 : 5 71 : 29 84 : 16
H H OH Me OH
t BuOOH

Ratio 64 : 36 29 : 71 62 : 38 64 : 36
OH

OH R1

RCO3H Transition States: ~ 120 TSmajor p. p. V(+) Transition States: ~ 45


HO H Me CH C

O R1

R2 SiMe3

R1

R2 SiMe3

TSminor

Oshima, Tetrahedron Lett. 1982, 23, 3387.

R2 Bu Me
OH

Yield 84 % 70 %

Ratio 99 : 1 99 : 1

H C5H11

TSmajor

H Me

C H

Me H

H Me

C HO

H H

TSminor
EtO OEt OH

O O Me Me

VO(acac)2 60 %

O
EtO OEt OH

O O Me Me

only isomer

t-BuOOH

Me Me

OH
Reagent

Me Me O

OH Me
threo +

Me Me O

OH Me
erythro

O EtO OEt O Me Me

VO(acac)2 60 %

O
EtO OEt

O O Me Me

only isomer

Me

Depezay, Tetrahedron Lett. 1978, 19, 2869.

K. Oshima & Coworkers Tetrahedron Lett. 1980, 21, 1657, 4843. K. B. Sharpless & Coworkers Tetrahedron Lett. 1979, 20, 4733.

Reagent m-CPBA t-BuOOH / VO(acac)2 t-BuOOH / Mo(CO)6 t-BuOOH / (t-BuO)3Al

Ratio 95 : 5 86 : 14 95 : 5 100 : 0

Me HO OH
Boeckman, JACS 1977, 99, 2805.

t-BuOOH VO(acac)2 60 % HO

O Me OH

Diastereoselection = 7 : 1

Me

NHCONHPh Ph

m-CPBA CH2Cl2, 0 C 75 %

Me O

NHCONHPh Ph
+

Me O

NHCONHPh Ph

8-12-Acyclic substrates 9/29/03 1:01 PM

Roush, J. Org. Chem. 1987, 52, 5127.

Diastereoselection = 95 : 5

D. A. Evans

Epoxidation of Acyclic Homoallylic Alcohols


Anti diastereomer
OH OH t-BuOOH VO(acac)2 90 % O OH Me Me + Me Me O OH R1 Me R2 t-BuOOH VO(acac)2 O R1 Me OH R2 + R1 Me O

Chem 206

Homoallylic Alcohols (Mihelich, JACS 1981, 103, 7690)

OH R2

Me

Me

Control Elements
A(1,3) Strain Directed Rxn
H H Me H O L HV

Diastereoselection > 400 : 1


L' O R O H Me R1 Me HO Me Me R2

R1

R2 Me i-Pr
OH R2 +

Yield 92 % 97 %

Ratio 104 : 1 > 400 : 1


OH R2

Syn diastereomer
OH t-BuOOH VO(acac)2

C6H13 Me

O R1 Me

O R1 Me

OH

p. .
Et

Me

t-BuOOH VO(acac)2

O Et

OH Me +

O Et

OH Me

R1 C6H13 Me Me
OH Me O Et Me C5H11 t-BuOOH VO(acac)2 Me

R2 Me Me C5H11
OH Me C5H11

Yield 73 % 70 % 81 %
O
+ Me

Ratio 70 : 1 85 : 1 16 :1
OH Me C5H11

Diastereoselection 12 : 1

Control Elements
Directed Rxn
Me

H O

L HV

L' O R O H Et Me

OH

E. D. Mihelich & Coworkers J. Am. Chem. Soc. 1981, 103, 7690.

Diastereoselection = 211 : 1

Epoxidation of Homoallylic Alcohols with TBHP, VO(acac)2

Prediction
H R2 O R1 H L HV L' O R H L O R1 HV

Substrate
L' O R O H Me OH Me OH Me Me OH Hex R

Product
OH Me OH Me Me OH Hex

Selectivity
O

2:1

O H
Me

Anti should be more diastereoselective than syn

R2 H

4.6 : 1

Anti diastereomer

Syn diastereomer

1.4 : 1

8-13-Homoallylic ROHs 9/29/03 1:03 PM

R = (CH2)7CO2Me

D. A. Evans

Epoxidation of Acyclic Homoallylic Alcohols


Epoxidation & Cyclization of Bishomoallylic Alcohols
R Et VO(acac)2 R Et O iPr OH Me AcOH iPr O R

Chem 206

Bishomoallylic Alcohols (Kishi, Tet. Lett. 1978, 19, 2741)


OH Me Et CHMe2 t-BuOOH, VO(acac)2 C6H6, RT Me Et O OH CHMe2

Me Et OH

iPr

OH

Me

diastereoselection ~ 9 : 1
H H R O Et Et H V Me R OH O Me

The Kishi Lasalocid Synthesis (JACS 1978, 100, 2933)


Me OH HO CO2H Me Me Me Et VO(acac)2 TBHP Me AcOH Et O Me H Et

A
H O

O
O R

B
Me

OH Et

Me OH

Et

p. .

Me Et Me

CHMe2

t-BuOOH, VO(acac)2 C6H6, RT

O Me Et Me

OH CHMe2 Ar OH

A
Ar O Et

Diastereoselection 8:1
OH Me Et

Ar = p-MeOPh

diastereoselection ~ 20 : 1
H Et Me Me R O H V Me O O R OH R O Me O CHMe2 Et Me Ph O Me H R O H V Me O O R R OH O Et Me O Me N OBn Me O OH Et VO(acac)2 TBHP C6H6, RT OH Et Me Me A O Me OH H B O OH

2nd stereocenter is reinforcing

Evans X-206 Synthesis JACS 1988, 110, 2506.


Me Me C O OH OH Me D Me H E O O F Me OH Me O Et OH

OH Me Et Me CHMe2 t-BuOOH, VO(acac)2 C6H6, RT Me

OH

Me

diastereoselection ~ 6 : 1

O XN

OH

O Et

Me

Et

OBn HOAc O

Me

diastereoselection 20 : 1 (89 %)

XN OBn

D O

Et Me OH

8-14-Bishomoallylic ROHs 9/29/03 1:04 PM

D. A. Evans

Olefin Addition Reactions: Part2


Problems of the Day: (To be discussed)

Chem 206

http://www.courses.fas.harvard.edu/~chem206/

Predict the stereochemical outcome of the indicated reaction.


O Me HOOC I2, NaHCO3 Me Me Me Me I diastereoselection: 20:1 O Me

Chemistry 206 Advanced Organic Chemistry


Lecture Number 9

Kinetic Control: 3 eq. I2, MeCN, NaHCO3, 0C

Olefin Addition Reactions2


Epoxidation & Directed Epoxidation Hydrogenation

Bartlett, P. A.; Richardson, D.; Myerson, J. Tetrahedron 1984, 12, 2317

Rationalize the stereochemical outcome of the indicated reaction.


Me N O MH Me N H Me N OH R2AlH LiAlH4 97 : 3 28 : 72 H OH

Olefin Bromination

Reading Assignment for week A. Carey & Sundberg: Part B; Chapter 4 "Electrophilic Additions to CC Multilple Bonds"
Hoveyda, Evans, & Fu (1993). Substrate-directable chemical reactions. Chem. Rev. 93: 1307-70 (Handout) J. M. Brown, Angew. Chem. Int. Edit. 26, 190-203 (1987) (Handout) Investigation of the early Steps in Electrophilic Bromination through the Study of the Reaction of Sterically Encumbered Olefins R. S. Brown, Accts. Chem. Res. 1997, 30, 131 (handout) Bromoniun Ions or -Bromocarbocations in Olefin Bromination. A Kinetic Approach to Product Selectivities M-F. Ruasse, Accts. Chem. Res. 1990, 23, 87 (handout) Friday, October 3, 2003

R. Noyori Bull. Chem. Soc. Japan 47, 2617, (1974)

D. A. Evans
9-00-Cover Page 10/2/03 3:38 PM

D. A. Evans

Diastereoselective Hydrogenation: Introduction


The Hydrogenation Reaction

Chem 206

Relevant Review articles: J. M. Brown, Angew. Chem. Int. Edit. 26, 190-203 (1987).

Polar functional groups may play a role in controlling the diastereoselectivity of the hydrogenation process; however, the control elements were not well-defined.
CHMe2 CHMe2 O

General Mechanism
M

M C R H R C H C H H R H H CH3 O H2, Pd-C

M(0)

R C + H

C R H

R C H

only isomer
CH3

H C R H

M(0) +

M R H C C

H R H R C H

H C R H

however

CHMe2 OH H CH3

R C H

CHMe2 H2, Pd-C H CH3 OH

p. .

Historically, primary stereochemical control designed around analysis of steric environment in vicinity of C=C. However, the influence of polar effects was documented

trans:cis = 55:45

J. E. McMurry & Co-workers, Tetrahedron Lett.. 3731 (1970)

O O

CO2Et

CO2Et HO H H H H

H2 Pd-C EtOH
OMe

10% Pd-C
OMe

H2
HO H H OH H

Steric Control
sole product

LiAlH4

Pd(0)
O O CH2OH

Pd(II)
O

trans : cis 85 : 15

CH2OH O HO H H

H H

H2 Pd-C EtOH
OMe

5% Pd-Al2O3
OMe

H2
OH H

Directed ?
12 : 1

Thompson, J.Org. Chem. 36, 2577 (1971)

trans : cis 5 : 95

Y. Kishi & Co-workers, J. Am. Chem. Soc. 102, 7156 (1980)

9-01-Hydrogenation-1 10/2/03 3:22 PM

D. A. Evans

Diastereoselective Hydrogenation: Introduction-2


Cationic Hydrogenation Catalysts

Chem 206

The first rational attempt to identify those FGs which will direct the reaction
H O CH3O R O H2, 5% Pd-C CH3O
10

O Rh R O Ph2P
(CH2)n Schrock & Osborne, J. Am. Chem. Soc. 91, 2816 (1969)

BF4

PPh2 Py

Ir

PF6

PCy3

R CH2OH CHO CN COONa COOH COOMe COMe CONH2 p. .

cis : trans 95 : 5 93 : 7 75 : 25 55 : 45 18 : 82 15 : 85 14 : 86 10 : 90

R. Crabtree J. Organomet. Chem. 168, 183 (1979)

S H2 Ph2P Rh

S
BF4

H2

S H Rh H Ph2P

S = solvent

PPh2
(CH2)n

PPh2

16-e

18-e-

H. Thompson & Co-workers, J. Am. Chem. Soc. 95, 838 (1973) The first rational attempt to associate catalyst with substrate:

Rh(+I): d8 Mechanism of Hydrogenation Cationic Rhodium-(I) Catalysts. S = solvent


S CH2=CH2 PPh2 S Rh Ph2P CH3CH3 H2
(S)

CH2OK (Ph3P)3RhCl MeO H2 100 psi 50 C, C6H6

CH2OH

S H MeO

Rh Ph2P

S PPh2

cis : trans >98 : 2


CH2ORh(PPh3)3

Reductive Elimination
H H

Oxidative Addition
H

(+S)

Rxn Catalytic in Rh (4 mol%)


MeO

Rh

H 2C Ph2P

S PPh2

H Rh Ph2P

S PPh2

Thompson & Coworkers, J. Am. Chem. Soc. 97, 6232 (1974)

9-02-Hydrogenation-2 10/2/03 3:24 PM

D. A. Evans

Diastereoaselective Hydrogenation: Cationic Catalysts

Chem 206

Mechanism of Hydrogenation Cationic Rhodium-(I) Catalysts.

D. A. Evans & M. M. Morrissey JACS 106, 3866 (1984)


OH OH
H2

S 2H2 Rh Ph2P
+

S Rh

S
BF4

H2

S H Rh H PPh2 CH3

CH2Cl2

CH3

+2 S
BF4

Ph2P

PPh2

PPh2

16-eP P H2 Rh HA HB H C H H OH C

18-e

- Ph2P

Catalyst

Mol% Catalyst 17.5 3.5 20.0 2.5

H2 Pressure 15 psi H2 375 psi H2 15 psi H2 15 psi H2

trans:cis (Yield) 200 : 1 (89%) 300 : 1 (95%) 50 : 1 (82%) 150 : 1 (85%)

Rh(DIPHOS-4)+ +

R2 CH2

OH

R2 H

Ir(pyr)PCy3

p. .

P Rh P P R2 CH3 OH P Rh CH3 H R2 OH

Which hydrogen migrates ?? +


CH3 H C H

OH

Rh + H2

OH CH3 CH3

CH2OH

Rh + H2

CH2OH

65 : 1

CH3 19 : 1

Rh(DIPHOS-4)+ H2 1000 psi CH2Cl2

Me

Me

Excessive Steric Hindrance


Me H Me Me

A potential stereoelectronic effect


P P Rh HA HB H C H H OH C R2 H P P HB Rh Me H Me CO2H

Me

H Me Me

OH Me
}

OH Me

HA

HA H H

O R2

+ H

Retigeranic Acid Me
Rh H2
+

OH Me Me

That H atom lying parallel to the pi-system (HA) should migrate preferentially if the dihydride is an intermediate.

Me

H Me

OH

75 : 1 (95%)

Rh(DIPHOS-4)+ H2 800 psi THF THF is important to success of rxn to buffer the Lewis acidity of the catalyst which causes elimination of ROH under normal conditions

9-03-cationic H2 mech 10/2/03 3:25 PM

D. A. Evans

Diastereoaselective Hydrogenation: Cyclic Substrates


O X Ir(pyr)Pcy3+ H2 Me CO2Me Ir(pyr)Pcy3+ CO2Me Me O X X OMe NC4H8

Chem 206

Polar functional groups other than OH may also direct the process
CO2Me Rh(DIPHOS-4)+ H 2C H2 CH3 CO2Me

Diastereoselection 55:45 99:1

diastereoselection 91 : 9

O X CH3 Ir(pyr)Pcy3+ H2 CH3 X CH3

X OMe NC4H8

Diastereoselection 99:1 >99:1

CH3

H2

diastereoselection 89:11

CH3

CH3

p. .

Ir(pyr)Pcy3+ H2 Me

A.G. Schultz and P.J. McCloskey, J. Org. Chem., 1985, 50, 5907.

diastereoselection >99:1
H Me OCH3 15 psi H2 Ir(pyr)Pcy3+ CH3 R.H. Crabtree and M.W. Davis, J. Org. Chem., 1986, 51, 2655. OCH3

J.M. Brown and S.A. Hall, J. Organomet. Chem., 1985, 285, 333. Me O N N H O H Ir(pyr)Pcy3+ H2 H Me O N N H O CONC4H8 N CH3 CH2OMe O CH3 Ir(pyr)Pcy3+ H2 CH3 N CH2OMe O CH3 H CH3

diastereoselection >99:1

diastereoselection >99:1

15 psi H2 Ir(pyr)Pcy3+

CONC4H8

diastereoselection >99:1

diastereoselection >99:1
CH3

CH3

A.G. Schultz and P.J. McCloskey, J. Org. Chem., 1985, 50, 5907. A.G. Schultz and P.J. McCloskey, J. Org. Chem., 1985, 50, 5907.

9-04-Cyclic substrates 10/3/03 8:33 AM

D. A. Evans

Diastereoaselective Hydrogenation: Acyclic Substrates


Acyclic Allylic Alcohols
P P + Rh H P H Rh C R1 + OH H2 C CH2R2 H R2 Me OH R1 R2 CH2 OH R1 P P P + Rh P Rh H H C R1 OH C CH2R2 H
+

Chem 206
OH H2 R2 Me R1

anti > 93 : 7

favored
OH R2 CH2 P P + Rh R1

anti
OH P R2 CH3 R1 P + Rh R2 H

P P Rh C R1 OH C CH3 H H2 R2 Me
+

OH R1

H H

H C P Rh P C + OH

R1 CH2R2 H2 R2 Me

OH R1 OH R CH2 Me O N O O Me Ph H2 Rh(DIPHOS-4)+ R Me OH

syn > 91 : 9

p. .

disfavored

syn

COXn Me

D
P P P + Rh P R2 H OH R2 Me P P + Rh H C P Rh P C + OH R1 Me H2 R2 Me OH R1 Rh C R1 + OH C CH3 H OH H2 OH R2 Me R1 R Me Me O

anti

low pressure
Me N O O Ph OH R Me Me COXn

favored

H2 Rh(DIPHOS-4)+

syn

syn

Anti : Syn Ratio


Hydroxy-Olefin R = CH3 15 psi H2 25 : 75 (23%) 52 : 48 (35%) 71 : 29 (-) 13 : 87 (6%) 12 : 88 (8%) 21 : 79 (-) 640 psi H2 93 : 7 94 : 6 93 : 7 9 : 91 8 : 92 6 : 94

R2 H

D
R1

R = (CH3)2CH R = Ph R = CH3

disfavored

anti T

R = (CH3)2CH R = Ph

D. A. Evans & M. M. Morrissey JACS 106, 3866 (1984)

9-05-Acyclic ROH's 10/3/03 8:41 AM

D. A. Evans

Diastereoaselective Hydrogenation: Acyclic Substrates


The Premonensin Synthesis
HO O Me Me HO C Me Me Me OH O Me Me OH

Chem 206

Homoallylic Alcohols Evans, Morrissey Tetrahedron Lett. 26 6005 (1985)


P P + Rh OH R H C CH2 C Me P Rh P O+ H Me H H2 R

favored
OH R Me Me

Me Et Me O Me

syn

disfavored
P P + Rh

P H O+ P Rh CH2 R Me C Me C H H

OH H2 R Me Me

EtO2C Me HO Me Me H2 Rh +

Catalyst
Me

Ratio
+

Rh(DIPHOS-4)
EtO2C HO Me Me

85 : 15 65 : 35 98 : 2 (90%)

Rh()(BINAP) + Rh(+)(BINAP)
+

p. .

anti A(1,3) destabilization


Evans, DiMare, JACS, 1986, 108, 2476)

HO Me Me

OTBS

HO Me Me

OTBS

The Ionomycin Synthesis


Me Me

A
HO Me Me HO

syn

OH

OH Me O Me H O

O OTBS Me Me HOOC Me Me Me Me CH3O2C Me Me Me OH H2 Rh(DIPHOS-4)+ Me O Me

Me OH

B Olefin A A B

OTBS

anti

Catalyst (H2 Pressure) Rh(DIPHOS-4)+ (1000 psi) Ir(pyr)PCy3+ (15 psi, 2.5 mol%)

syn : anti 95 : 5 73 : 27

Me CH3O2C Me Me Me OH

Rh(DIPHOS-4)+ (1000 psi) 9 : 91

Diastereoselection: 94 : 6 (93%) with Dow, Shih, Zahler, Takacs, JACS 1990, 112, 5290
}

9-06-Acyclic ROH's 10/3/03 8:39 AM

D. A. Evans
R R R Br2 R

Olefin Bromination-1
R Br C

Chem 206

Introduction
R

Bromonium ion origin of the anti (trans) selectivity first suggested by Roberts, JACS 1937, 59, 947 +
R R R

Br R

R R

R R

Reaction is first order in alkene At low concentrations of Br2, rxn is also first order in Br2 At higher concentrations of Br2 in nonpolar solvents rxn is second order in Br2. Substituent Effects on Bromination Rates

Br2

Br R R Br

R Br C

Br R

First X-ray Structure of a bromonuium ion: Brown, JACS 1985, 107, 4504

Alkene
CH2=CH2 CH3CH=CH2 n-PrCH=CH2 i-PrCH=CH2 t-BuCH=CH2 (CH3)2C=CH2 cis-CH3CH=CHCH3 trans-CH3CH=CHCH3 (CH3)2C=CHCH3 (CH3)2C=C(CH3)2

krel
1 61 70 57 27 5470 2620 1700 130,000 1,800,000 Br-3 Br-2 C C 2 eq Br2 -2 eq Br2

Br3 Br

p. .

Br-4

X-ray structure
Stereochemical outcome versus structure (Br2 in HOAc @ 25) Alkene
H Me H Me H Ph H Br-1

% anti addition

Alkene
H Me

% anti addition
2.116 2.194

100%
Me Me Ph Me H H

83%

100%
H H Ph Me Me Me

63%

1.497

73%
Me Ph H

68%

9-07-Bromination-1 10/2/03 3:41 PM

D. A. Evans

Olefin Bromination-2
Overall Reaction Mechanism
Second Order Kinetics

Chem 206

Calculated Geometries of Substituted Bromonium Ions Ruasse, Chem Commun. 1990, 898 More recent calculations: Sigalas, Tetrahedron 2003, 59, 4749

Br
2.01

Br
1.88

Br
2.70 1.51 2.05

Br Br.HOR -complex Br2

Products

H H

1.47

H H

H H

Me Me

Me Me

1.51

Me Me

Br2

Note; the CBr bond lengths in previous X-ray structure are 2.116 . Bromonium Ions undergo fast exchange with olefins Brown, Accts. Chem. Res. 1997, 30, 131 p. . Unprecedented until 1991 (Bennet, JACS 1991, 113, 8532)

(-complex)

Br2

Br Br3 -complex

Products

Third Order Kinetics

AdC CAd Br3 Br

AdC CAd Br AdC CAd Br3

Bromination of Cyclohexene Derivatives Pasto, JACS 1970, 92, 7480


R Me3C H PyrBr R = H, Me Br
+

AdC CAd X

Br3 Me3C H Br H

exclusive product

X = Br: exchange rate: 2 x 106 M1 s-1 AdC CAd X = I : exchange rate: 8 x 106 M1 s-1

Diaxial opening of bromonium ions may be viewed as an extension of the Furst-Plattner Rule for epoxide ring opening (Lecture-3).
H Me3C H Br H MeOH Me3C H H H Me3C H H OMe H H Br Me3C H Br Br OMe H H

There is an intermediate in the halogen transfer (ab initio calcs):


PyrBr+ Br3 R R Me3C R R complex MeOH H

R R R

R R R

R R

47%

+ Br
R R complex R R R R

+ Br
R R TS R R

+ Br

53%

9-08-Bromination-2 10/3/03 8:46 AM

It appears that bromine attack from both olefin faces occurs wilth near equal probability.

D. A. Evans

Olefin Bromination-3

Chem 206

Bromination of Cyclohexene Derivatives Pasto, JACS 1970, 92, 7480 Diaxial opening of bromonium ions may be viewed as an extension of the Furst-Plattner Rule for epoxide ring opening. (Lecture-2)

Representative Examples of Diastereoselective Bromination


Me Br Br2 HOAc H HH Br H H

H H

Me

Case A
MeOH PyrBr+ Br3 H Me3C H MeOH Me3C H Me3C H Br

H H Br Me3C H Br

OMe H H

H R

47%

House 2nd Ed, pg 424 H Me R H H H Br H H R H H H Br Me R H R Major Product (70%)

Br H H Me3C H H OMe H Br Br Minor Product (7%)

53%

Case B p. .
PyrBr+ Br3 Me3C H Me MeOH OMe Me3C H Br Me H H Me

How to generate either epoxide from a conformationaly biased olefin exclusive product
H Me RCO3H H O Me Epoxidation controlled by steric effects imposed by cis-fused ring

Br + Me3C H H Me Me3C

How do we construct the other epoxide diastereomer?? Me Me Br2 H 2O H Br OH2 Me Br OH2 H

syn-Unreactive Br Me3C H + Me H HOMe

H anti-Reactive

Br H

Br Me3C H

not Me H observed
OMe H H O

Me base H Br H H H

Me Me Br OH minor major H

syn-Unreactive

H H OH

From Case A, one assumes that both bromonium ions are formed; however, for the syn isomer to react, ring opeing must proceed against the polarization due to Methyl substituent.

9-09-Bromination-3 10/3/03 8:47 AM

both bromohydrins afford same product

D. A. Evans
Oxymercuration Pasto, JACS 1970, 92, 7480 The basic process:
H R C C H H XHgX ROH XHg H C R C H H OR NaBH4 H

Olefin Oxymercuration-1

Chem 206

Bromination of Cyclohexene Derivatives Pasto, JACS 1970, 92, 7480


H C R C H H OR

Diaxial opening of bromonium ions may be viewed as an extension of the Furst-Plattner Rule for epoxide ring opening. (Lecture-2) Case A
MeOH PyrBr+ Br3 H Me3C H Br H MeOH Me3C H H H Me3C H H OMe H H H Br Me3C H Br Br OMe H H

Kinetics: Halpern, JACS 1967, 89, 6427 Reduction: Pasto, JACS 199, 91, 719 Overview: B rown, JOC 1981, 46, 3810.

47%

Oxy-Mercuration & bromination follow identical pathways (Pasto)


R Me3C H OH Hg(OAc)2 THF, H2O Me3C H H R Me3C H OH HgOAc H R

Me3C

53%

p. .

HgOAc

Case B
PyrBr+ Br3 Me3C Me MeOH OMe Me3C H Br Me H Br H anti-Reactive H Me

R=H R = Me
Me3C H Me Hg(OAc)2 THF, H2O Me3C

41% 100%

48%
OH H Me

exclusive product

exclusive product

HgOAc Br + + Me3C Me H Me3C H H Me

HgX Me3C H + Me H syn-Unreactive Me3C

HgX H anti-Reactive Me3C

syn-Unreactive Br + Me H H HOMe

Reduction of the HgC bond R HgX


HCO2 NaBH4

H+
Me3C H

Br H OMe Me

HgH

not observed

nonstereoselective radical chain process O O CO2 Formate is an excellent source of hydride ion for late transition and heavy main-group metals

syn-Unreactive

R Hg
H

From Case A, one assumes that both bromonium ions are formed; however, for the syn isomer to react, ring opeing must proceed against the polarization due to Methyl substituent.

9-10-oxymercuration-1 10/3/03 8:21 AM

D. A. Evans

Olefin Addition Reactions: Part3

Chem 206

http://www.courses.fas.harvard.edu/~chem206/

Bromoniun Ions or -Bromocarbocations in Olefin Bromination. A Kinetic Approach to Product Selectivities M-F. Ruasse, Accts. Chem. Res. 1990, 23, 87 (handout) Investigation of the early Steps in Electrophilic Bromination through the Study of the Reaction of Sterically Encumbered Olefins R. S. Brown, Accts. Chem. Res. 1997, 30, 131 (handout)

Chemistry 206 Advanced Organic Chemistry


Lecture Number 10

Predict stereochemical outcome


Me Me OH O OH Me Me Me 99% Me O
1

OR

OR

O Me

Olefin Addition Reactions3


Olefin Oxymercuration Halolactonization Simmons-Smith Reaction

X-206 Synthesis (with S. Bender, JACS 1988, 110, 2506)

Hg(OAc)2, CH2Cl2 -78 oC to -20 oC

OR

OR

O Me

O OH HgX Me Me

16

Reading Assignment for week


HO Me

99%, single diastereomer


H O Me Me OH

A. Carey & Sundberg: Part B; Chapter 4 "Electrophilic Additions to CC Multilple Bonds"


Me BnO O O Me Me Me

Hg(OAc)2, CH2Cl2 -78 oC to -20 oC

Hoveyda, Evans, & Fu (1993). Substrate-Directable Chemical Reactions. Chem. Rev. 93: 1307-70 (Handout)

XHg Me BnO

O O O Me Me Me H O Me

Me OH

D. A. Evans
10-00-Cover Page 10/6/03 8:35 AM

Monday, October 3, 2003

85%, dr = 93 : 7
Me

Ionomycin Synthesis (with Dow & Shih, JACS 1990, 112, 5290)

D. A. Evans

Olefin Oxymercuration-1
Oxymercuration Pasto, JACS 1970, 92, 7480

Chem 206

Bromination of Cyclohexene Derivatives Pasto, JACS 1970, 92, 7480


H

The basic process:


H R C C H H XHgX ROH XHg H C R C H H OR NaBH4 H H C R C H OR

Diaxial opening of bromonium ions may be viewed as an extension of the Furst-Plattner Rule for epoxide ring opening. (Lecture-2) Case A
MeOH PyrBr+ Br3 H Me3C H MeOH Me3C H H Me3C H Br H Me3C H Me Me3C H PyrBr+ Br3 H OMe H H H Br Me3C H Br Br OMe H H

Kinetics: Halpern, JACS 1967, 89, 6427 Reduction: Pasto, JACS 199, 91, 719 Overview: B rown, JOC 1981, 46, 3810.

47%

Oxy-Mercuration & bromination follow identical pathways (Pasto)


R Me3C H OH Hg(OAc)2 THF, H2O Me3C H H R Me3C H OH HgOAc H R

53%

p. .

HgOAc

Case B

R=H R = Me
Me3C H Me Hg(OAc)2 THF, H2O Me3C

41% 100%

48%
OH H Me

exclusive product

Br + Me3C H H syn-Unreactive MeOH Br H H OMe Br + Me3C H H Me + Me3C H Me Me3C H Me Me3C

Me H

HgOAc Me H

Br H anti-Reactive MeOH OMe H Br Me A Me

HgX Me3C H + Me H syn-Unreactive Me3C

H anti-Reactive

HgX

not observed
Me3C

exclusive product

Reduction of the HgC bond R HgX


HCO2 NaBH4

HgH

nonstereoselective radical chain process O O CO2 Formate is an excellent source of hydride ion for late transition and heavy main-group metals

R Hg
H

10-01-oxymercuration-1 10/5/03 11:20 AM

From Case A, one assumes that both bromonium ions are formed; however, for the syn isomer to react, ring opeing must proceed against the polarization due to Methyl substituent.

D. A. Evans

Oxymercuration Examples
s Acyclic allylic alcohols:
OH OBn O H H H OC6H11 Giese, Tet. Lett. 1985, 26, 1197 : = 96 : 4 R
Hg(OAc)2 R'OH

Chem 206

Diastereoselective ring closures via oxymercuration


H BnO BnO H OBn OH H OC6H11 H
Hg(OTFA)2 NaBH4

OH
NaBH4

OH HgOAc R R'OH HOH MeOH HOH HOH Me OR' R -Et -Et -Ph -tBu H C H H RL X RL OH HgOAc Ratio 76 : 24 93 : 07 88 : 12 98 : 02 OH yield 65% 72% 66% 70%

BnO BnO

R OR'

Mukaiyama, Chem. Lett. 1981, 683 H R' AcNH H OH CO2Me OR


Hg(OTFA)2 Ph3 SiH

H R' AcNH H O

OR CO2Me "one isomer" OH RL HOH OBn OR n-Bu OH


Hg(OAc)2

X Hg HO
Hg(OAc)2

H OBn

H OBn Sinay, Tet. Lett. 1984, 25, 3071 H O OBn OR


Hg(OAc)2 NaBH4

H O HO HO Me H

HO Me H

Hg HO H C RL Me

H C H Me HOH RL

OH

OH Me

Isobe, Tet. Lett. 1985, 26, 5199

HgOAc syn:anti = 80 :20

s Kinetic vs Thermodynamic control:


H Me H N BnO2 C H
Hg(OAc)2 NaBH4

Chamberlin, Tetrahedron 1984, 40, 2297 H CH2 HgX Me

O-acetate participation will turn over the stereochemical course of the rxn
OAc Et
Hg(OAc)2 HOH NaBH4

XHgCH2 N BnO2 C

Me H

H N BnO2 C H

OAc Et Me Et OH erythro

OH Me OAc Me COOMe BnO O OBn erythro:threo = 77 : 23

Harding, JOC 1984, 49, 2838

Hg(OAc)2: short rxn times : 40 : 60 Hg(OTFA)2: longer rxn times : 2 : 98


Me

Hg(OAc)2

COOMe

With more electrophilic Hg(II) salt, more polar solvents, and longer rxn times, the rxn may be rendered reversible.

BnO

BnOH NaBH4

Seebach, JACS 1983, 105, 7407

diastereoselection = 83 : 17 (79%)

10-02-oxymercuration-2 10/6/03 8:54 AM

D. A. Evans

Oxymercuration Examples
Proposed Mechanism
Lewis acid catalyzes formation of hemiketal
H R' R O HgCl H OH Me O Me H
5% Yb(OYt)3

Chem 206

Oxymercuration via Hemiketals & Hemiacetals


J. L. Leighton et. al, Org. Lett. 2000, 2, 3197-3199

General Reaction: diastereoselection >10:1

H O O

Me Me

OH R

+ H

O R'

HgClOAc 5% Yb(OYt)3

O R Me

Yb(X2)

Yb(X2)

The Oxymercuration Step (Kinetic Phase)


H Me
HgClOAc

Me O HgCl R H O O X R Hg

OH R

+ Me

O Me

H Me

HgClOAc 5% Yb(OYt)3

O R

H O O

Me Me R

H H O O HgCl

Me Me

p. .

Mechanistic Observations:
Me OH R + Me O Me
HgClOAc acetone, 2h rt

Yb(X2)
rate-determining step

Yb(X2)
low diastereoselectivity

Me O HgCl R H H O O Me
HgClOAc

O R

X Me

Hg R

H O O H

Me Me

~1:1-mixture of diastereomers Product formed in low yield. much recovered starting material
Lewis acid addends were surveyed. the logic for this step was two-fold: (A) Lewis acid would promote the formation of the putative hemiketal imtermediate. (B) Lewis acid would promote reversability of the oxymercuration process

Yb(X2)

Yb(X2)

Leighton presumes that mercurium ion formation is rate-determining under kinetic conditions. At higher temperatures and longer reaction times the products are shown to interconvert. ClHg H R O H O Me Me YbX3
HOAc, 5% Yb(OYt)3

OH Me3C

+ Me

O Me

HgClOAc 5% Yb(OYt)3 acetone, 2 min 0 C

Me O Me3C

Me O HgCl

Cl

Hg H R H O O H

Me Me

~1:1-mixture of diastereomers H Me Me H O O Me MM-2 Me Me H H Me O O Me Me Me O Me3C


HOAc, 5% Yb(OYt)3

YbX3 Cl Hg R H H O O H Me Me

Me O 93% yield HgCl

Erel = 0

Erel = +5.2 kcal/mol

OAc

10-03-oxymercuration-3 10/5/03 11:22 AM

D. A. Evans

Oxymercuration Examples: X-206 & Lonomycin Syntheses

Chem 206

X-206 Synthesis (with S. Bender, JACS 1988, 110, 2506)


Me Me O OH
A

Ionomycin Synthesis (with Dow & Shih, JACS 1990, 112, 5290)
Me Me OH O O O

Me OH
B

Me
C

Me
D

Me
E

H Me O
F

O Me

OH

Me H

OH OH

O Me OH Me

OH OH O Ca

aldol

Et Me O
D E

Me OH

Me

Ionomycin Calcium Complex

+
H

H O
F

O Me O Me Me OR

C1-C16 Subunit
Me OR
A

O Me OMe Me

Me Me OH Me Me R H O Me H O Me

Me Me OH

Me

Me

Me Me Me OH

OR

C17-C37 Subunit

Et

HO Me Me R OH OH

p. .

Me O
1

Me H

O OH

OH OH

Me

Me

16

Assemblage strategy for Ring A:

Me Me
A

OR OH

OR

O Me H Me H HO

H R1
16

Predicted stereochemical outcome: O


Me Me
7

OH

Me H

Me

O O Me

Me

Hg(OAc)2, CH2Cl2 -78 oC to -20 oC

HO Me H

Me Me OH

H H

OR OH
9

HgX H Me R2

Me BnO O

O Me Me

RL

HgX2 RDS

Me RO2C H H

O Me H

CO2R X RO H Hg C RL H C Me H

H RL

OR

HgX+

RDS

Me

85%, dr = 93 : 7
H R1 H Me H HO Me R2 H Hg H X O O Me Me XHg Me BnO O Me H O Me Me Me Me OH

Hg(OAc)2 CH2Cl2 99%


Me RO2C H H

H H O
7

OR
9

Me H

RL

Me

HgX

10-04-oxymerc/lono 10/5/03 5:22 PM

D. A. Evans

Related Olefin Addition Rxns: Halogen Electrophiles

Chem 206

Other electrophilic olefin addition reactions afford the same stereochemical outcome
X OH
Hg(OAc)2

Iodine-induced lactonization is also highly stereoselective


Chamberlin (JACS 1983, 105, 5819)

HO H

Hg C n-Bu

H C H Me n-Bu

OH

OH Me O RO Me OH
I2, HOH/THF

A HO Me
HCO3

I C CH2

n-Bu

Me

H Me

HO Me -O2C C

H C H Me

RDS

HgOAc ratio = 80 :20

H Me

R=H -O2C

OH
I2, HOAc

HO H C

CH2

H C H Me

OH
HOAc

OAc Me

n-Bu

Me

RDS

n-Bu

n-Bu I C n-Bu Chamberlin, Tetrahedron 1984, 40, 2297

I Ratio = 98 : 2 (78%) OH OAc Me I Ratio = 94 : 6 (85%)

As we have seen before, gauche B is more destabilizing than gauche A

HO

I Me Me O

HO

I Me Me O

p. .
n-Bu

OH

Me
I2, HOAc

HO H

H C Me H

HOAc

R = OMe

n-Bu

t-BuOOH VO(acac)2

Ratio 96 : 4 (85%) O
K2CO3 MeOH

RDS

O MeO

OH O Me MeO

OH O Me

This is an exceptional approach to the creation of either syn or anti 1,3-dioxygen relationships Evans, Kaldor, Jones, J. Am. Chem. Soc. 1990, 112, 7001.

Me

Me

Other cases:
O OH Me

Lactonization Ratio = 96 : 4 Epoxidation Ratio = 3 : 97


HO
I2, HOH/THF HCO3

I Me O H Ratio >95 : 5 (49%)

TIPSO

OH Me Me
I2, THF, 4 C 0.25 M KH2PO4,

TIPSO

OH

OH Me

HO

Me
n-Bu3SnH, toluene, 25 C TsOH, (CH3)2C(OCH3)2, 25 C

I Me TIPSO O Me O Me HO

OH

O HO
I2, HOH/THF HCO3

I R = H: 77 : 23 (74%) O R R = Me: 42 : 58 (81%)

Me O HO

Me

This methodology superior to oxymercuration alternative which was evaluated first

Me

67% overall
HO

I R = H: 87 : 13 (41%) O R R = Me: 90 : 10 (94%)

OH

I2, HOH/THF HCO3

diastereoselection 96 : 4

Me O

Me

10-05-iodination 10/5/03 5:40 PM

D. A. Evans

Related Olefin Addition Rxns

Chem 206

Halogen-induced heterocyclization in the synthesis of monensin


HO Me MeO O Me HO Me A O B O C O Me D O Me

Hypothesis-B: Stereocontrol through Reversal of Bromonium Ion Intermediate Me RL H OH Me RL bromonium ion-olefin exchange Me Me RL RL Ar H Br H C O Me Me Et H D O H Me Br Ar H C O Me Me Et H D O H Me

E O HO

Me CH2OH

Me H

Et H

Br

The Kishi Ring D Construction:


Me Me Ar H C O Et D OH Me

Kishi, JACS 1979, 101, 259, 260, 262 Still, JACS 1980, 103, 2117-2121

NBS MeCN 57%

only one diastereomer

OH

Br

Ring D disfavored

p. .

Hypothesis-A: Stereocontrol through A(1,3) Strain?? Me Ar Me Ar H C O Me Et H D O H Me H C O Me Et H D O H Me

The Still Ring E Construction:


Me Me R H D O Me E O HO Me CH2OH Me R H D O Me E O O I
Ag2CO3

Me D OH E HO O Me

R H Me

Br

KO2_ DMSO 47%

OH Me

H
50%

H Me H

Me RL H OH Me RL H

RL H OH Me RL Br

Stereocontrol through A(1,3) Strain?? Me D O O Me HO O Me E Me Me D O Me O Me I O O Me E Me

O Me O

KI3 HCO3 87%

Me Me RL H OH H

RL Me RL H OH

Me

RL Incorrect Stereochemistry Br

Me

El(+)-induced heterocyclization

Cardillo, Tetrahedron 1990, 46, 3321-3408 Bartlett, Asymmetric Synthesis 1984, 3, Chap 6, 411-454

10-06-Monensin examps 10/5/03 5:34 PM

D. A. Evans
A complete turnover in olefin diastereofacial selectivity is observed when adding internal and external nucleophiles I2

Applications in Synthesis Hehre's Analysis


HO H I Me O O cis : trans 95 : 5 OH Me H HO H I2 Bu I ratio 99 : 1 I2 H H OH R Me H R H H OH
Nu Nu

Chem 206
HO I Me O O Bu I
Addition product

OH

OH Me

O HO

OH Me

H
I2

Me OH O OH

R H H HO

Me H

R H H HO I2

Me RDS H

R H H HO I+

Me H

OH2

OH
I2

Bu

OH Me

Bu

Me

H 2O

Favored groundstate conformer

Disfavored complex

Favored iodonium ion

p. .

General Observation:

For electrophiles that react via onium intermediates (I2, Br2, Hg(OAc)2, PhSeCl), the major diastereomer from electrophile-induced cyclization is opposite to that observed in the analogous intermolecular electrophilic addition. For a review of elctrophilic induced olefin cyclization reactions see: G. Cardillo & M. Orena, Tetrahedron 1990, 46, 3321.

RDS

Me H H

OH

I + Me H R

More reactive ground-state conformer

Favored complex

Disfavored iodonium ion

Chamberlin & Hehre's Rationalization


"Facial preferences in electrophilic addition reactions are not invariant with respect to the location of the transition state along the reaction coordinate." Change in diastereoselectivity is a consequence of a change in the rate-limiting step Addition reactions: Formation of an onium ion intermediate
(subsequently trapped by a Nu from the medium)

HO

H O

I OH Me Bu I Me OH

Cyclization reactions: Intramolecular attack on a complex (not an onium ion) Analysis of the stereoselectivity of electrophilic addition to chiral olefins: 1. Relative abundances of conformational minima 2. Relative reactivities of the available forms 3. Stereoselectivies of the individual conformers

O
Cyclization product

Houk: Argument for the "inside alkoxy effect" in complex formation complex cyclizes if R contains a Nu and its formation is rate determining Onium ion formation is rate determing in the addition reactions "The presence or absence of an internal nucleophile acts to determine the stereochemical outcome of the reaction by modifying the nature (timing) of transition state.

Chamberlin & Hehre, J. Am. Chem. Soc. 1987, 109, 672-677.

10-07-Monensin examps-2 10/5/03 5:37 PM

D. A. Evans

The Simmons-Smith Reaction


R OH OH 79 % CH3 CH2 I2 Zn-Cu R M. Pereyre and Co-workers J. Chem. Res. (S) 1979, 179 CH3 Et tBu 57 64 67 R OH Ratio : : : 43 36 33 CH3 R

Chem 206
CH3 OH

For a recent general review of the Simmons-Smith reaction see: Charette & Beauchemin, Organic Reactions, 58, 1-415 (2001)
CH2 I2 , Zn-Cu OH

>99:1

S. Winstein, JACS 1959, 81, 6523; 1961, 83, 3235; 1969, 91, 6892

A large rate acceleration relative to simple olefins was observed.

s The classical mechanism


OR CH2 I2 Zn-Cu OH CH2 I2 Zn-Cu OH CH2 I2 Zn-Cu CH2 OH 1 CH2 3 PhCH2 CH2 CH2 OH Et2 Zn CH2 I2 PhCH2 CH2 CH2 OH CH2 OH OR

CH2 I2 + Zn R = OMe: >99:1 Dauben, JACS 1963, 85, 468 R = OAc: 4:1
Sawada, JOC 1968, 33, 1767 R R

ICH2 ZnI
ZnI CH2 I

ICH2 ZnI

+ ZnI2
R

9:1

s Enantioselective Simmons-Smith Variants: Kobayashi, Tet. Let. 1992, 33, 2575


80% ee (82% yield)

>99 : 1

NHSO2 Ar

10 mol% epoxidation also gives anti adduct

These results suggest that the transition state might be binuclear. Construct a reasonable transition structure which accomdates the data

OC1C2C3 dihedral = 165


S. Winstein, JACS, 1969, 91, 6892

NHSO2 Ar

Absolute control of stereochemistry is possible through chiral ketal auxiliaries


BnOH2 C O CH2 OBn O Me CH2 I2 Zn-Cu BnOH2 C O CH2 OBn O

s Low-valent Samarium Variants: Molander,JOC 1987, 52, 3942


Me3 C HO CH3 Sm or Sm/Hg CH2 I2 R'' HO OH R' Isolated alkenes and homoallylic alcohols are inert to these reaction conditions. G. A. Molander and J. B. Etter J. Org. Chem. 1987, 52, 3942 Ph Ph Ph tBu tBu R" nBu iPr tBu CH3 iPr Ratio 1 > 200 > 200 1 > 200 : : : : : 1.4 1 1 5.1 1 R' R'' R'' Me3 C OH R' R'' OH > 200 : 1 (99%)

Me CH2

diastereoselection 20:1

R'

Yamamoto, JACS, 1985, 107, 8254 Mash, JACS, 1985, 107, 8256 Yamamoto, Tetrahedron, 1986, 42, 6458

10-08-S-Smith rxn 10/6/03 9:23 AM

A. S. Kim, D. A. Evans
Radical Lead Tetraacetate Mechanism

Redox- Thallium.1
Vinyl and Aryl C-H Oxidations Thallium

Chem 206

ROH + Pb(OAc)4 ROPb(OAc)3 RO + R'H R' + Pb(OAc)4

ROPb(OAc)3 + OAc RO + Pb(OAc)3 ROH + R' R'OAc + Pb(OAc)3

CHO

(TFA)2Tl

CHO

CHO

Tl(TFA)3, TFA
N H N H

CuI, I2 DMF 94%


N H

Somei, Heterocycles 1984, 22, 797.

H RCH2 Me OH

LTA

Me RCH2 Me O R

OAc

RCH2

OAc

1) Tl(TFA)3, TFA 2) CuSO45H2O


O N Ac

42%
OH

N Ac

Somei, Heterocycles, 1988, 27, 2363.

19% (racemic)
O Me Me Me

33%

20%

Hauser, Helv. Chim. Acta 1964, 47, 1883.

Phenolic Oxidations:
O Me

For a review of oxidative aryl couplings, see: Dinsmore, C. Evening Seminar, February 1993.

LTA
HO

Me O O
Immer, Helv. Chim. Acta 1962, 45, 753.

Me O

45-72%
H

TTFA 75%
HO O

CF3COO

LTA
HO

OAc O O OAc

Coombs, Chem. Ind., 1972, 169.

O Br O O O O OAc HO
Cocker, JCS, 1965, 6.

28%
O O Me Me

42%

Haynes, JOC, 1866,31, 3067.

Ph

Me O

Me

N Ph

TTFA 56%

N O Br O

LTA, BF3Et2O PhH, RT 86%

Me

AcO

AcO

Taylor, JACS, 1981, 103, 6856.

10-09-REDOX-Thallium.1 10/6/03 8:36 AM

A. S. Kim, D. A. Evans
Oxythallation of Ketones (Enols) with Ring Contraction TTN
O Me CO2Me

Redox- Thallium.2
TTN = Tl(NO3)3 Oxythallation of Double and Triple Bonds

Chem 206

Me

MeOH
O

84%

Tl(ClO4)3
McKillop and Taylor, JACS, 1973, 95, 3340. McKillop and Taylor, JOC, 1978, 43, 3773.

COMe

AcOH 88-92%
OMe OMe

TTN MeCN
O

ONO2

97% TTN MeOH


OH

85%

1) TTN, AcOH 2) NaHCO3

McKillop and Taylor, JACS, 1973, 95, 3635.

OMe
McKillop and Taylor, JOC, 1972, 37, 3381.

84%

OMe

86%

1) TTN, AcOH 2) reflux, 30min.


O

COOH

O O

84%
O COOH
Salaun, Tet., 1974, 30, 1423.

1) TTN/HClO4 2) NaH,
n-Pentyl O

O n-Pentyl

TTN AcOH 95%


O

(MeO)2P(O)

70%

O
Corey, TL, 1971, 4753.

CO2Me

TTN MeOH 85%


Me Me Me Me Me

Jones, JOC, 1977, 42, 2176.

Me Me Me HO2C H

TTN MeOH 68%


Me Me Me Me

O
Jones, JACS, 1976, 98, 8476.

Tl(OAc)3 83%

Me
Inoue, Bull. Chem. Soc.

TTFA AcOH 76%


Me Jpn. 197, 51, 2439. Me O

Me

Romeo, Tet., 1972, 28, 5337.

10-10-REDOX-Thallium.2 10/6/03 8:36 AM

A. S. Kim, D. A. Evans
Oxidative Rearrangements of Styrenes TTN = Tl(NO3)3

Redox- Thallium.3
Oxidative Rearrangements of Chalcones
MeO

Chem 206

OMe Me O

1) TTN, MeOH 2) H3O+ 85%


Me O

H Me MeO
McKillop and Taylor, JACS, 1973, 95, 3635.

TTN MeOH 70%


MeO

Antus, Liebigs Ann. Chem. 1980, 1283.

1) TTN, MeOH 2) H3O+ 81%


O

Me

OH

TTN
OH OH

McKillop and Taylor, JACS, 1973, 95, 3635.

MeOH 74%

O
Eade,Aust. J. Chem. 1978, 31, 2699.

1) TTN, MeOH 2) H3O+ 95%


McKillop and Taylor, JACS, 1973, 95, 3635.

OBn

OM e

1) TTN, MeOH 2) H3O+


MOMO O OMOM OMe

75%

HO

OBn

van Heerden JCSPT1 1980, 2463.

Me

1) TTN, MeOH 2) H3O+ 95%


McKillop and Taylor, TL, 1977, 1827.

OH

O O

2TTN
OH O O

MeOH 65%

OMe

1) TTN, MeOH
MeO

Antus, Liebigs Ann. Chem. 1980,1271.

2) H3O+ 100%

MeO
McKillop and Taylor, TL, 1977, 1827.

OH

OO

2TTN
OH HO

MeOH 53%

Antus, Liebigs Ann. Chem. 1980,1271.

10-11-REDOX-Thallium.3 10/6/03 8:36 AM

A. S. Kim, D. A. Evans
Oxidation of Nitrogen Compounds
R N N H R R' HN N COR

Redox- Thallium.4
TTN = Tl(NO3)3

Chem 206

2TTN MeOH 78-95%

CO2Me

+ N2

Taylor and McKillop, ACIEE 1972, 11, 48.

TTN MeOH

R O R'
McKillop and Taylor, JACS 1971, 93, 4918.

R and R' alkyl or aryl, yields 82-95%


R N R' OH

TTN MeOH

R O R'
McKillop and Taylor, JACS 1971, 93, 4918.

R and R' alkyl or aryl, yields 72-96% Oxidation of Sulfur Compounds


SEt MeO NO2

OMe

MeO

TTN MeOH 98%

MeO

NO2
Fujita, JCSCC, 1976, 202.

MeO

O SEt

Ph

TTN MeOH 91%

OMe OMe
Fujita, TL, 1978, 4115.

Ph

Me S S S

TTN MeOH 97%


O

Me S

S
Smith, Synth. Commun. 1979, 9, 301.

10-12-REDOX-Thallium.4 10/6/03 8:37 AM

D. A. Evans, T. B. Dunn

Pericyclic Reactions: Part1


s Other Reading Material:

Chem 206

http://www.courses.fas.harvard.edu/~chem206/

Chemistry 206 Advanced Organic Chemistry

s Woodward-Hoffmann Theory R. B. Woodward and R. Hoffmann, The Conservation of Orbital Symmetry, Verlag Chemie, Weinheim, 1970. s Frontier Molecular Orbital Theory I. Fleming, Frontier Orbitals and Organic Chemical Reactions, John-Wiley and Sons, New York, 1976. s Dewar-Zimmerman Theory T. H. Lowry and K. S. Richardson, Mechanism and Theory in Organic Chemistry, 3rd Ed., Harper & Row, New York, 1987. s General Reference R. E. Lehr and A. P. Marchand, Orbital Symmetry: A Problem Solving Approach, Academic Press, New York, 1972.

Lecture Number 11

Pericyclic Reactions1
s Introduction to Pericyclic Reactions s Electrocyclic Reactions s Sigmatropic Reactions s Cycloaddition Reactions

s Problems of the Day:


Predict the stereochemical outcome of this reaction.
Ph O O O Ph
h

O O

s Reading Assignment for week: Carey & Sundberg: Part A; Chapter 11 Concerted Pericyclic Reactions Fleming: Chapter 4 Thermal Pericyclic Reactions
Ph

heat

Huisgen, TL, 1964, 3381.

Ph

Suggest a mechanism for the following reaction.


CO2Me MeO2 C CO2Me H

Travis Dunn

Wednesday, October 7, 2003

heat
CO2Me

Bloomfield, TL, 1969, 3719.

11-00-Cover Page 10/6/03 9:26 AM

D. A. Evans, B. Breit, T. B. Dunn

Pericyclic Reactions: Introduction The Theories:

Chem 206

Pericyclic Reactions - Introduction/Definitions


A pericyclic reaction is characterized as a change in bonding relationships that takes place as a continuous, concerted reorganization of electrons. The term "concerted" specifies that there is one single transition state and therefore no intermediates are involved in the process. To maintain continuous electron flow, pericyclic reactions occur through cyclic transition states. More precisely: The cyclic transition state must correspond to an arrangement of the participating orbitals which has to maintain a bonding interaction between the reaction components throughout the course of the reaction.

Three theories are commonly used to explain and predict pericyclic reactions. We will only concern ourselves with two of these theories. 1) Fukui: Frontier Molecular Orbital Interactions s Much easier to use than the original orbital symmetry arguments s HOMO/LUMO interactions 2) Dewar-Zimmerman: Aromatic Transition States s The easiest to apply for all reaction types, but it is not as easy to
understand why it it valid

Some factors to consider in our analysis:


The number of electrons involved has a profound influence on reactivity:

s Aromatic or antiaromatic transition states 3) Woodward-Hoffmann: Conservation of Orbital Symmetry

heat rarely observed 4 electrons Pericyclic reactions are stereospecific: A A A A heat A A heat

heat often observed 6 electrons

s First theory to explain and predict the outcome of many reactions s Correlation diagrams
On the three methods:
"There are several ways of applying the orbital-symmetry principle to cycloaddition reactions, three of which are used more frequently than others. Of these three, we will discuss two: the frontier-orbital method and the Mbius-Hckel method. The third, called the correlation diagram method, is less convenient to apply than the other two." Jerry March in "Advanced Organic Chemistry"

A A

The Five Major Categories of Pericyclic Reactions


(1) ELECTROCYCLIC RING CLOSURE/RING OPENING:
An electrocyclic ring closure is the creation of a new sigma bond at the expense of the terminal p orbitals of a conjugated pi system. There is a corresponding reorganization of the conjugated pi system. We usually classify the reaction according to the number of electrons involved. Examples: A 4 e- electrocyclic reaction A 6 e- electrocyclic reaction

Reactions behave differently depending on the conditions used (i.e. thermal versus photochemical conditions): A A A
11-01-Peri 10/11/00 7:53 AM

heat A

A A
Cyclobutene

or h
Butadiene

or h
1,3,5-Hexatriene 1,3-Cyclohexadiene

D. A. Evans, B. Breit, T. B. Dunn

Pericyclic Reactions: Major Classes


(4) SIGMATROPIC REARRANGEMENTS:

Chem 206

(2) CYCLOADDITION REACTIONS/CYCLOREVERSION REACTIONS:


A cycloaddition reaction is the union of two smaller, independent pi systems. Sigma bonds are created at the expense of pi bonds. A cycloaddition can occur in an intramolecular sense, but it must be between two independent pi systems. Cycloaddition reactions are referred to as [m + n] additions when a system of m conjugated atoms combines with a system of n conjugated atoms. A cycloreversion is simply the reverse of a cycloaddition. Examples:

A sigmatropic rearrangement is the migration of a sigma bond from one position in a conjugated system to another position in the system, accompanied by reorganization of the connecting pi bonds. The number of pi and sigma bonds remains constant. The rearrangement is an [m,n] shift when the sigma bond migrates across m atoms of one system and n atoms of the second system. Examples:
2 2 3 1

[1,3]-shift

2 3 1

3 4 5

R1 R2

[1,5]-shift

R2 R1 H

[2+2]

A 2+2 cycloaddition. The Paterno-Bchi reaction.

R
1 2 3 3' 2'

R
1 2 3 3' 2'

X
[4+2] +

[3,3]-shift

X R
1'

A 4+2 cycloaddition. The Diels-Alder reaction.

X=CR2, Cope rearrangement X=O, Claisen rearrangement

1'

(5) GROUP TRANSFER REACTIONS: (3) CHELETROPIC REACTIONS:


Cheletropic reactions are a special group of cycloaddition/cycloreversion reactions. Two bonds are formed or broken at a single atom. The nomenclature for cheletropic reactions is the same as for cycloadditions. Examples: In a group transfer reaction one or more groups get transferred to a second reaction partner. Examples: Hydrogen Transfer:

H
+

R
+

H H

O
+

[4+1]

O S O

H N N H
+

R N2 R'
+

H H

R R'

C O

[4+1]

O
Ene Reaction:

CR2

[2+1]

R R H
+

11-02-Peri 10/11/00 7:55 AM

D. A. Evans, B. Breit, T. B. Dunn


ELECTROCYCLIC RING CLOSURE/RING OPENING: The Stereochemical issues:

Electrocyclic Reactions
Butadiene to cyclobutene: A 4-electron (4q) system

Chem 206

Ring closure can occur in two distinct ways. This has consequences with regard to: s The orbital lobes that interact s The disposition of substituents on the termini Conrotatory Closure: The termini rotate in the same direction Me B B A B A B A A B A A B Me Me H H Me

Me
conrotation

heat

Me

conrotation

heat

Me

Me

Hextriene to cyclohexadiene: A 6-electron (4q+2) system

disrotation

Disrotatory Closure: The termini rotate in opposite directions

Me

H H

Me

Me B A Me Me
disrotation

Me

A A A B A B B A B B

Me Me It was also noted that changing the "reagent" from heat to light reversed this reactivity pattern. Under photochemical conditions 4 electron systems undergo disrotatory motion, while 6 electron systems undergo conrotatory motion. Me
disrotation

Empirical Observations: It was noted that butadienes undergo conrotatory closure under thermal conditions, while hexatrienes undergo disrotatory closure under thermal conditions. The microscopic reverse reactions also occur with the same rotational sense (i.e. cyclobutenes open in a conrotatory sense when heated, and cyclohexadienes open in a disrotatory sense when heated.)

Me Me Me Me

controtation

h Me

h Me Me

11-03-Peri 10/11/00 7:56 AM

D. A. Evans, T. B. Dunn

Conjugated pi systems

Chem 206

antibonding 5 p-orbitals 4 p-orbitals 3 p-orbitals 2 p-orbitals


3 3 4

6 p-orbitals

C nonbonding

C nonbonding

bonding

There are no nodal planes in the most stable bonding MO. With each higher MO, one additional nodal plane is added. The more nodes, the higher the orbital energy.

11-04-Peri 10/11/00 7:56 AM

D. A. Evans, B. Breit, T. B. Dunn

Electrocyclic Reactions: FMO Analysis

Chem 206

s FMO Treatment of Electrocyclic reactions.


s Examine the interactions that occur in the HOMO as the reaction proceeds. s If the overlap is constructive (i.e. of the same phase) then the reaction is "allowed." s If the overlap is destructive (i.e. of different phases) then the reaction is "forbidden." Thermal Activation: Conrotatory Closure: (Allowed and observed)
Constructive overlap

Photochemical Activation: When light is used to initiate an electrocyclic reaction, an electron is excited from 2 to 3. Treating 3 as the HOMO now shows that disrotatory closure is allowed and conrotatory closure is forbidden. 4 3 2 1 4 3 Me 2 1

h Me H H Me
Photon Me absorption

2 (HOMO) Disrotatory Closure: (Allowed and observed) H H H Me Me


Constructive overlap

3 (HOMO)

Me

Me

Me H

H Me

Me H

H Me

H Me

H Me

2 (diene HOMO)

Me Me

Disrotatory Closure: (Forbidden and not observed) H Me H H Me Me


Destructive overlap

3 (new HOMO)

H Me

H Me

H Me

Conrotatory Closure: (Forbidden and not observed) Me Me H H Me

2 (diene HOMO)

Me

Me

Me H

3 (new HOMO)

Destructive overlap

A similar analysis for the hexatriene system proves that under thermal conditions, disrotation is allowed and conrotation is forbidden.

We have so far proven which ring closures are allowed and which are forbidden. Do we now have to go back and examine all the ring openings?

NO!

The principle of microscopic reversiblity says that if the reaction is allowed in one direction, it must be allowed in the other direction.
11-05-electrocyclic/FMO 10/11/00 7:56 AM

D. A. Evans, B. Breit, T. B. Dunn

Electrocyclic Reactions: Dewar-Zimmerman


Connect Orbitals

Chem 206

The Dewar-Zimmerman analysis is based on identifying transition states as aromatic or antiaromatic. We will not go into the theory behind why this treatment works, but it will give the same predictions as FMO or Orbital Symmetry treatments, and is fundamentally equivalent to them.

Using the Dewar-Zimmerman model:


s Choose a basis set of 2p atomic orbitals for all atoms involved (1s for hydrogen atoms). s Assign phases to the orbitals. Any phases will suffice. It is not important to identify this basis set with any molecular orbital. s Connect the orbitals that interact in the starting material, before the reaction begins. s Allow the reaction to proceed according to the geometry postulated. Connect those lobes that begin to interact that were not interacting in the starting materials. s Count the number of phase inversions that occur as the electrons flow around the circuit. Note that a phase inversion within an orbital is not counted. s Based on the phase inversions, identify the topology of the system. Odd number of phase inversions: Even number of phase inversions: Mbius topology Hckel topology

Disrotatory Closure

Conrotatory Closure

One Phase Inversion Zero Phase Inversions Hckel Topology Mbius Topology 4 electrons in system 4 electrons in system Antiaromatic and Aromatic and Forbidden Allowed Note that I can change the phase of an abitrary orbital and the analysis is still valid! Connect Orbitals

Disrotatory Closure

Conrotatory Closure

s Assign the transition state as aromatic or antiaromatic, based on the number of electrons present. System Hckel Mbius Aromatic 4q + 2 4q Antiaromatic 4q 4q + 2 Two Phase Inversions Hckel Topology 4 electrons in system Antiaromatic and Forbidden Three Phase Inversions Mbius Topology 4 electrons in system Aromatic and Allowed

s If the transition state is aromatic, then the reaction will be allowed thermally. If the transition state is antiaromatic, then the reaction will be allowed photochemically.
11-06-electrocyclic/DZ 10/11/00 7:57 AM

D. A. Evans, B. Breit, T. B. Dunn

[1,3]-Sigmatropic Rearrangements: FMO Analysis


s [1,3] Sigmatropic Rearrangements (H migration)

Chem 206

The Stereochemical issues: The migrating group can migrate across the conjugated pi system in one of two ways. If the group migrates on the same side of the system, it is said to migrate suprafacially with respect to that system. If the group migrates from one side of the pi system to the other, it is said to migrate antarafacially with respect to that system. Suprafacial migration: The group moves across the same face. B A B A A B A B

s Construct TS by considering an allyl anion and the proton (or radical pair): H X Y X H H Y X Y

B X

H Y X

H Y X

H Y

Antarafacial migration: The group moves from one face to the other.
Proton 1S (LUMO)
bonding antibonding bonding

B A B A

B B A A B A B X

X
2 (allyl anion HOMO)

Y
bonding

s Sigmatropic Rearrangements: FMO Analysis


s Imagine the two pieces fragmenting into a cation/anion pair, (or a pair of radicals) and examine the HOMO/LUMO interaction. s If the overlap is constructive at both termini then the reaction is allowed. If the overlap is destructive at either terminus then the reaction is forbidden. s If the migrating atom is carbon, then we can also entertain the possiblity of the alkyl group migrating with inversion of configuration (antarafacial on the single atom). s If the migrating atom is hydrogen, then it cannot migrate with inversion.
11-07-[1,3]sig/FMO 10/11/00 7:58 AM

Suprafacial Geometry

Antarafacial Geometry

s The analysis works if you consider the other ionic reaction, or consider a radical reaction. In each case it is the same pair of orbitals interacting. s The suprafacial migration is forbidden and the bridging distance too great for the antarafacial migration. Hence, [1,3] hydrogen migrations are not observed under thermal conditions. s Under photochemical conditions, the [1,3] rearrangement is allowed suprafacially. How would you predict this using FMO?

D. A. Evans, B. Breit, T. B. Dunn

[1,3]-Sigmatropic Rearrangements

Chem 206

s [1,3] Sigmatropic Rearrangements (C migration) CH3 X Y X CH3 CH3 Y X Y

s Sigmatropic Rearrangements: Dewar-Zimmerman


Dewar-Zimmerman also predicts the [1,3] suprafacial migration to be forbidden. The basis set of s and p orbitals with arbitrary phase: Two Phase Inversions Hckel Topology Four Electrons Forbidden thermally
H H

s Construct TS by considering an allyl anion and the methyl cation: Retention at carbon H C H H
bonding

Inversion at carbon
2p on Carbon

H C H H X
H H
bonding

antibonding bonding

Orbital interactions in the parent system

Completing the circuit across the bottom face

H H

2 (allyl anion HOMO)

Y The [1,5] shift of a hydrogen atom across a diene.

Suprafacial on allyl fragment

Suprafacial on allyl fragment

s The analysis works if you consider the other ionic reaction, or consider a radical reaction. In each case it is the same pair of orbitals interacting. s Under photochemical conditions, the [1,3] rearrangement is allowed suprafacially with retention of stereochemistry. s The stereochemical constraints on the migration of carbon with inversion of configuration is highly disfavored on the basis of strain. Such rearrangements are rare and usually only occur in highly strained systems. Using a similar analysis, one can prove that [1,5] hydrogen and alkyl shifts should be allowed when suprafacial on the pi component and proceeding with retention. Please refer to Fleming for more applications of FMO theory to [1,n] sigmatropic shifts.

Zero Phase Inversions Hckel Topology Six Electrons Allowed thermally Orbital interactions in the Completing the circuit parent system across the bottom face

11-08-[1,3]sig/FMO/DZ 10/11/00 7:58 AM

D. A. Evans, B. Breit, T. B. Dunn

[3,3]-Sigmatropic Rearrangements s The Toggle Algorithm:

Chem 206

[3,3] Rearrangements:
A thermally allowed reaction in either of two geometries, the "chair" or the "boat" geometry. Depicted below is the "chair" geometry. You should be able to work out the details of the "boat" geometry yourself. X Z X & Z = C, O, N etc The FMO Analysis: Bring two Allyl radicals together to access for a possible bonding interaction between termini.

The toggle algorithm is a simple way to take one reaction of each class that you remember is allowed (or forbidden) and derive if the reaction is allowed or forbidden under new conditions.

X Z

X Z

s How does it work?


All of the various parameters of the pericyclic reaction are the input variables, the "switches." The output is either "allowed" or "forbidden." Write out all the relevant parameters of a reaction together with the known result. Each time you change a parameter by one incremental value ("toggle a switch"), the output will switch. This is the prediction of the reaction under the new parameters.

s So it's nothing really new, is it?


No, its just a convenient way to rederive predictions without memorizing a table of selection rules. An Example: Take the [1,3] sigmatropic rearrangement of an alkyl group. We know this is forbidden under thermal conditions in a supra-supra manner, and so we make it the first entry in the table.

The nonbonding allyl MO

bonding bonding

Rearrangement Conditions Component 1 Component 2 [1,3] [1,3] [1,3] [1,5] Two Phase Inversions Hckel Topology Six Electrons Allowed Thermally Heat Heat Light Heat Suprafacial Antarafacial Antarafacial Suprafacial Suprafacial Suprafacial Suprafacial Suprafacial

Output Forbidden Allowed Forbidden ?

The Dewar-Zimmerman Analysis:

Each incremental change in the "input" registers changes the "output" register by one. Multiple changes simply toggle the output back and forth. What is the prediction in the last line?

11-09-[3,3]sig/FMO/DZ 10/11/00 7:59 AM

D. A. Evans, B. Breit, T. B. Dunn


The Stereochemical issues:

Cycloaddition Reactions
The [4+2] Cycloaddition: Dewar-Zimmerman

Chem 206

In a cycloaddition, a pi system may be attacked in one of two distinct ways. If the pi system is attacked from the same face, then the reaction is suprafacial on that component. If the system is attacked from opposite faces, then the reaction is antarafacial on that component.

The most well known cycloaddition is the Diels-Alder reaction between a four pi component (the diene) and a two pi component (the dienophile). An exhaustive examination of this reaction is forthcoming, so we will limit ourselves to a simple examination.

Suprafacial attack

Antarafacial attack Zero Phase Inversions Hckel Topology Six Electrons Allowed thermally

The [2+2] Cycloaddition: FMO Analysis For the [2+2] cycloaddition two different geometries have to be considered. Suprafacial/Suprafacial
HOMO

Antarafacial/Suprafacial
bonding

HOMO

Summary:
s There are three fundamentally equivalent methods of analyzing pericyclic reactions: Two are much simpler than the third. s Fukui Frontier Molecular Orbital Theory s Dewar-Zimmerman Hckel-Mbius Aromatic Transition States s Woodward-Hoffmann Correlation Diagrams
LUMO
bonding

bonding

antibonding

LUMO

Forbidden

Allowed

s Some methods are easier to use than others, but all are equally correct and no one is superior to another. Conclusions drawn from the correct application of one theory will not be contradicted by another theory. s The principle of microscopic reversibility allows us to look at a reaction from either the forward direction or the reverse direction. s There is a general trend that reactions will behave fundamentally different under thermal conditions and photochemical conditions.

The simplest approach (Supra/Supra) is forbidden under thermal activation. The less obvious approach (Antara/Supra) is allowed thermally but geometrically rather congested. It is believed to occur in some very specific cases (e.g. ketenes) where the steric congestion is reduced.
11-10-Cycloaddition 10/11/00 7:59 AM

D. A. Evans

Pericyclic Reactions: Part2


Other Reading Material:

Chem 206

http://www.courses.fas.harvard.edu/~chem206/

C. Palomo, "Asymmetric Synthesis of -Lactams by Stauginger Ketene-Imine Cycloaddition Reaction, Eur. J. Org. Chem. 1999, 3223-3235. R R N O R N O S N O R N3 R N R O N R O R R R Correlate with R S N O con N3 R N R N3 H S N

Chemistry 206 Advanced Organic Chemistry


N3 C

N3

con

Lecture Number 12

Pericyclic Reactions2
Electrocyclic Reactions Cheletropic Reactions

Problems of the Day:


Predict the stereochemical outcome of this reaction.
Ph O O O Ph

O O O

Reading Assignment for week: Carey & Sundberg: Part A; Chapter 11 Concerted Pericyclic Reactions Fleming: Chapter 4 Thermal Pericyclic Reactions
Ph heat Huisgen, TL, 1964, 3381.

Ph

Suggest a mechanism for the following reaction.


H CO2Me heat H CO2Me Bloomfield, TL, 1969, 3719. MeO2C CO2Me H

D. A. Evans
12-00-Cover Page 10/13/03 9:03 AM

Monday, October 12, 2003

Evans, Breit
Electrocyclic Reaction - Selection Rules Ground State (Thermal process) 4n e(n = 1,2...) 4n+2 e(n = 0,1,2...) conrotatory disrotatory
Ground State Conrotatory

Electrocyclic Processes-1

Chem 206

Excited State (Photochemical Process) disrotatory conrotatory

Controtation 1 and 2 on to the indicated bonding and anti-bonding orbitals of cyclobutene:


LUMO

Con

HOMO

Examples

Excited State

1
Disrotatory

Con

LUMO HOMO

Disrotatory

Conrotatory

Activation Energy (kcal/mol) for electrocyclic ring opening

Conrotatory

Disrotatory

42

45

Disrotatory Conrotatory

Conrotatory
H

29

27
H

Disrotatory

Criegee, Chem. Ber. 1968, 101, 102. Ph O O O Ph O O

Conrotatory

Disrotatory

Disrotatory

Conrotatory

Ph Ph O O O O

R Con R R R R

Con

Ph O

Ph

R Sterically favored

Ph Huisgen, TL, 1964, 3381. Ph O

12-01-Electroclycliz-1 10/12/03 4:39 PM

Evans, Breit

Electrocyclic Processes-2: Torquoselectivity


How do we explain?
R R

Chem 206
Donor substituents prefer conout mode Pi acceptor substituents prefer conin mode

Torquoselectivilty is defined as the predisposition of a given R substituent for a given conrotatory motion Houk et al. Acc. Chem. Res 1996, 29, 471
R R

con in

View the 2 conrotatory modes by looking at the breaking sigma bond from this perspective
H

con
H

out
R R H H

Examples:

Donor substituents prefer conout mode Pi acceptor substituents prefer conin mode
R R H

Outward Motion +
R H H H

Inward Motion

con

R = Me R = CHO

only none

none only

H H LUMO + p

H H

H H H LUMO + p

CH2OBn CHO H H

CH2OBn

con +
CHO

CH2OBn

A
CHO

H H HOMO + p

H H

H H HOMO + p

H H H

ratio: >20:1
Me CN Me CN CN

con

Me

ratio: 4:1

As conrotation begins the energy of the breaking sigma bond rises steeply. Hyperconjugation with a pi* orbital, while possible in both A & B, is better in B. (Houk)

destabilizing 4 electron interation for donor substituents stabilizing 2 electron interation for acceptor substituents

12-02-Electroclycliz-2 10/12/03 4:40 PM

Evans, Breit

Electrocyclic Processes-3: 3-Atom Electrocyclizations


Solvolysis of Cyclopropyl Derivatives
A Con?? A H H R H A Case 1 X slow X 1 fast H fast

Chem 206

Three-Atom Electrocyclizations (2 electrons)


A H R A H Dis?? R C H A

Does solvolysis proceed via cation 1 followed by rearrangement to 2 (Case 1), or does it proceed directly to 2 (Case 2)?

2 +X

3 Case 2 2 nonbonding X slow X 2 Me H H fast +X X

TsO 1 cation anion H

TsO H

Me

TsO H

H Me

H Me

relative rate 1
LUMO

4
DePuy, Accts. Chem. Res. 1967, 1, 33

40,000

+
R

H A H A
R Dis

H A C H C A
HOMO

X
LUMO

H Me H C Me Me
HOMO Dis

Note that there are two disrotatory modes


R R X Dis R X R R Dis R Sterically favored R Favored for R = ring R R LUMO

Me H Me C C H
HOMO Dis

12-03-Electroclycliz-3 10/12/03 4:41 PM

+
Me H

H H

Me Me

Evans, Breit
Solvolysis Summary
TsO H TsO H

Electrocyclic Processes-3: 3-Atom Electrocyclizations


dis-in
Unfavorable Me Me H H TsO H H Me

Chem 206

dis-out
favorable H Me R A

Three-Atom Electrocyclizations (4 electrons)


H H A H A Dis?? R C H A Con?? R H A A H

relative rate

40,000
3

Ring-fused Cyclopropyl Systems When the cis substiltutents on the cyclopropyl ring are tied together in a ring the following observsations have been made
TsO H H H TsO H H

nonbonding

1 cation anion

favored
relative rate: > 10+6 H TsO H H TsO H H H H 2C CH2

dis-in
H 2C

A
R

Con R

B B C A C A

C A B

disavored

dis-out

Observation
Ar N

H CO2Me Con H CO2Me Ar N (+)

CO2Me ()

Revisiting the Favorski rearrangement: (Carey, Part A, pp 506-8)


O Cl base O Cl Cl

MeO2C MeO2C Con Ar N (+)

dis-in
Ar N

H CO2Me

3-exo-tet disallowed MeO2C products H

()

MeO2C

12-04-Electroclycliz-4 10/12/03 4:42 PM

Evans, Breit

Electrocyclic Processes-3
The Nazarov Reaction
O OH +H+ A A A A A H+ A O R Con?? A A

Chem 206

Five-Atom Electrocyclizations (4 electrons)


R

R Dis?? A A

Denmark, S. E. In Comprehensive Organic Synthesis; Trost, B. M., Fleming, I., Eds.; Pergamon Press: Oxford, 1991; Vol. 5; pp 751.
O O +H
+

nonbonding

predict stereochemistry H H

2
Eight-Atom Electrocyclizations (8 electrons)

1
Cation Anion

A Dis?? A A

A Con?? A

Pentadienyl Cation
A LUMO Con H

C
A

HOMO A

C
H

Let's use the "Ready" shortcut to find the homo: Nodes will appear at single bonds
symmetry of homo

Pentadienyl Anion
A H Dis A

C
A H

C
A

Closure should be conrotatory

12-05-Electroclycliz-5 10/12/03 4:44 PM

Evans, Breit

Cheletropic Processes-1

Chem 206

CHELETROPIC REACTIONS: [n+1] Cycloadditions (or Cycloreversions)


Concerted processes in which 2 -bonds are made (or broken) which terminate at a single atom.

2 + 1 CheletropicReaction: Olefins + Singlet Carbene


R C C R R R

[4+2]
+

Linear Approach: 2 HOMO-LUMO Interactions


O
+

[4+1] S

O O C C HOMO R R C C LUMO R R

S O

General
Reversion Addition Y
+

LUMO

HOMO

-system

Y X Z

-system

X Z

Nonlinear Approach: 2 HOMO-LUMO Interactions


R R R R C C HOMO LUMO HOMO LUMO

Y X Z C Singlet-Carbene Addition (and Reversion) C O N N N N O SO2 Reversion and Addition

C C

Cycloreversion only

Frontier Orbitals

0 p (empty)
Y X Z

E 0
2

sp2 (filled) 2

Carry out the analysis of the indicated hypothetical transformation


Me Y C C Z Me Me Z Me Y

Question: what is orientation of carbene relative to attacking olefin??

R C R

predict approach geometry of carbene


12-06-cheletropic-1 10/12/03 4:44 PM

Evans, Breit

Cheletropic Processes-2

Chem 206

Let's now consider SO2 as the one-atom component

Key step in the Ramberg Bcklund Rearrangement


R1 R2 S O R1 empty X S O O R2 base R1 S O O base R2 R1 E

O S O S

O S O

O O S

-SO2

4e in pi system

1 filled
Me

2 filled

R1

R2 Z

Me suprafacial

O S O Me Me O S O Me

-SO2

O Me Me S O suprafacial Me

reactions are: stereospecific & reversible

Clough, J. M. The Ramberg-Backlund Rearrangement.; Trost, B. M. and Fleming, I., Ed.; Pergamon Press: Oxford, 1991; Vol. 3, pp 861. "The Ramberg-Backlund Rearrangement.", Paquette, L. A. Org. React. (N.Y.) 1977, 2 5, 1.

Analysis of the Suprafacial SO2 Extrusion (nonlinear)


R1 R1 R2 S O O S O HOMO O S O

+
R
2

SO2

O S O S

O O

HOMO

empty (LUMO)

empty (LUMO)

O S O S

O S O LUMO

O O S

O O

LUMO

2 filled

1 filled
Similar to carbene geometry

12-07-cheletropic-2 10/12/03 4:50 PM

D. A. Evans

Sigmatropic Rearrangements-1
[1,3] Sigmatropic Rearrangements (C migration)
consider the 1,3-migration of Carbon X Consider the orbitals needed to contruct the transition state (TS). :X R X X H H
bonding

Chem 206
CH3 Y CH3 X H3 C X Y Y

Sigmatropic rearrangements are those reactions in which a sigma bond (& associated substituent) interchanges termini on a conjugated pi system Examples:
[1,3] Sigmatropic rearrangement R X [2,3] Sigmatropic rearrangement R X [3,3] Sigmatropic rearrangement X [1,5] Sigmatropic rearrangement Y: X R Y X

Construct TS by uniting an allyl and Me radicals: Retention at carbon


X C R H
antibonding bonding

Inversion at carbon
R C H X X
H H
bonding

Suprafacial on allyl fragment [1,3] Sigmatropic Rearrangements (H migration)


H consider the 1,3-migration of H X Y H X Y X H Y Sychronous bonding to both termini cannot be achieved from this geometry

Suprafacial on allyl fragment


Sychronous bonding to both termini is possible from this geometry

Consider the orbitals needed to contruct the transition state (TS).

The stereochemical constraints on the suprafacial migration of carbon with inversion of configuration is highly disfavored on the basis of strain.

[1,3]-Sigmatropic rearrangements are not common


D no observed scrambling of labels H Me
1 bonding 3 120 C 1

Construct TS by uniting an allyl and H radical:


bonding bonding
H

Me C H
3

D H
1 3

X
Me

antibonding

X X
H H

Y 2 (allyl HOMO)

Suprafacial Geometry

Antarafacial Geometry

Bridging distance too great for antarafacial migration.


12-08-Sigmatropic-1 10/12/03 5:29 PM

These rearrangements are only seen in systems that are highly strained, an attribute that lowers the activation for rearrangement.

D. A. Evans

Sigmatropic Rearrangements-2
[1,5] Sigmatropic Rearrangements (C migration)
5 3

Chem 206

SIGMATROPIC REACTIONS - FMO-Analysis


2 1 3 4 5

/h
R = H, CR3

2 1

[1s,5s] alkyl shift RETENTION [1a,5a] alkyl shift INVERSION disfavored

[1,5] Sigmatropic Rearrangements (H migration)

4
[1,5] (C migration): Stereochemical Evaluation
Me Me
RETENTION 230-280C

Me H Me [1,5s]H- shift Me

nonbonding

h
Me

[1,5s]C- shift

2
DewarZimmerman Analysis: Retention

1
thermal photochemical
H

R H H

View as cycloadditon between following species:


R H H R + R pentadienyl radical 3 pentadienyl radical 3 suprafacial preferred R R R H H H H H

R R H H H H H

0 phase inversions Huckel toplogy 6 electrons therefore, allowed thermally

either, or

12-09-Sigmatropic-2 10/12/03 5:29 PM

D. A. Evans

Sigmatropic Rearrangements-3
The Wittig Rearrangement [1,2]

Chem 206

[1,2] Sigmatropic Rearrangements: Carbon

[1,2] Concerted sigmatropic rearrangements to cationic centers allowed

"[2,3]-Wittig Sigmatropic Rearrangements in Organic Synthesis.", Nakai, T.; Mikami, K. Chem. Rev. 1986, 86, 885. Marshall, J. A. The Wittig Rearrangement.; Trost, B. M. and Fleming, I., Ed.; Pergamon Press: Oxford, 1991; Vol. 3, pp 975.
Li O R BuLi O R

consider as cycloaddition CR homoylsis

R
olefin radical cation transition state

This 1,2-sigmatropic rearrangement is non-concerted

R Li O Li

The Wittig Rearrangement [2,3]


[1,2] Concerted sigmatropic rearrangements to carbanionic centers not observed Li O R R OLi

stepwise

R
CR homoylsis

consider as cycloaddition

CR homoylsis

R
antibonding

R H R Allyl radical

ketyl radical

R
olefin radical anion transition state Allyl radical

12-10-Sigmatropic-3 10/12/03 5:29 PM

D. A. Evans

Pericyclic Reactions: Part3


Other Reading Material:

Chem 206

http://www.courses.fas.harvard.edu/~chem206/

[2,3] Sigmatropic Rearrangements


Trost, Ed., Comprehensive Organic Synthesis 1992, Vol 6, Chapter 4.6:

Chemistry 206 Advanced Organic Chemistry

Nakai, T.; Mikami, K. Org. React. (N.Y.) 1994, 46, 105-209. Hoffmann, Angew. Chem. Int. Ed. 1979, 18, 563-572 (Stereochemistry of) Nakai, Chem. Rev. 1986, 86, 885-902 (Wittig Rearrangement) Evans, Accts. Chem. Res. 1974, 7, 147-55 (Sulfoxide Rearrangement) Vedejs, Accts. Chem. Res. 1984, 17, 358-364 (Sulfur Ylilde Rearrangements)

Lecture Number 13 [3,3] Sigmatropic Rearrangements


Trost, Ed., Comprehensive Organic Synthesis 1992, Vol 5, Chapter 7.1: (Cope, oxy-Cope, Anionic oxy-Cope) Chapter 7.2, Claisen S. J. Rhoades, Organic Reactions 1974, 22, 1 (Cope, Claisen) S. R. Wilson, Organic Reactions 1993, 43, 93 (oxy-Cope) T. S. Ho, Tandem Organic Reactions 1992, Chapter 12 (Cope, Claisen)

Pericyclic Reactions3
Introduction to Sigmatropic Rearrangements [2,3] Sigmatropic Rearrangements

Reading Assignment for week: Carey & Sundberg: Part A; Chapter 11 Concerted Pericyclic Reactions Fleming: Chapter 4 Thermal Pericyclic Reactions

Paquette, L. A. (1990). Stereocontrolled construction of complex cyclic ketones by oxy-Cope rearrangement. Angew. Chem., Int. Ed. Engl. 29: 609.

Problems of the Day:


Provide a mechanism for this transformation.
Me Me S S Me Me heat Me Me S Me Me

D. A. Evans
13-00-Cover Page 10/15/03 8:53 AM

Wednesday, October 14, 2003

PPh3

S=PPh3

For study on this [2,3] rxn See Baldwin JACS 1971, 93, 6307

D. A. Evans

Sigmatropic Rearrangements-1
[1,3] Sigmatropic Rearrangements (C migration)
consider the 1,3-migration of Carbon X Consider the orbitals needed to contruct the transition state (TS). :X R X X H H
bonding

Chem 206
CH3 Y CH3 X H3 C X Y Y

Sigmatropic rearrangements are those reactions in which a sigma bond (& associated substituent) interchanges termini on a conjugated pi system Examples:
[1,3] Sigmatropic rearrangement R X [2,3] Sigmatropic rearrangement R X [3,3] Sigmatropic rearrangement X [1,5] Sigmatropic rearrangement Y: X R Y X

Construct TS by uniting an allyl and Me radicals: Retention at carbon


X C R H
antibonding bonding

Inversion at carbon
R C H X X
H H
bonding

Suprafacial on allyl fragment [1,3] Sigmatropic Rearrangements (H migration)


H consider the 1,3-migration of H X Y H X Y X H Y Sychronous bonding to both termini cannot be achieved from this geometry

Suprafacial on allyl fragment


Sychronous bonding to both termini is possible from this geometry

Consider the orbitals needed to contruct the transition state (TS).

The stereochemical constraints on the suprafacial migration of carbon with inversion of configuration is highly disfavored on the basis of strain.

[1,3]-Sigmatropic rearrangements are not common


D no observed scrambling of labels H Me
1 bonding 3 120 C 1

Construct TS by uniting an allyl and H radical:


bonding bonding
H

Me C H
3

D H
1 3

X
Me

antibonding

X X
H H

Y 2 (allyl HOMO)

Suprafacial Geometry

Antarafacial Geometry

Bridging distance too great for antarafacial migration.


13-01-Sigmatropic-1 10/15/03 8:53 AM

These rearrangements are only seen in systems that are highly strained, an attribute that lowers the activation for rearrangement.

D. A. Evans

Sigmatropic Rearrangements-2
[1,5] Sigmatropic Rearrangements (C migration)
5 3

Chem 206

SIGMATROPIC REACTIONS - FMO-Analysis


2 1 3 4 5

/h
R = H, CR3

2 1

[1s,5s] alkyl shift RETENTION [1a,5a] alkyl shift INVERSION disfavored

[1,5] Sigmatropic Rearrangements (H migration)

4
[1,5] (C migration): Stereochemical Evaluation
Me 5 Me
RETENTION 230-280C

Me H

nonbonding

5 1 [1,5s]H- shift Me

1 Me

[1,5s]C- shift

Me

2
DewarZimmerman Analysis: Retention

1
thermal photochemical
H

R H H

View as cycloadditon between following species:


R H H R + R pentadienyl radical 3 the transiton structure suprafacial preferred R R R H H H H H

R R H H H H H

0 phase inversions Huckel toplogy 6 electrons therefore, allowed thermally

either, or

13-02-Sigmatropic-2 10/15/03 8:53 AM

D. A. Evans

Sigmatropic Rearrangements: An Overview


The Wittig Rearrangement [1,2]

Chem 206

[1,2] Sigmatropic Rearrangements: Carbon [1,2]-Sigmatropic rearrangements to cationic centers allowed. Wagner-Meerwein Rearrangement R + + R

"[2,3]-Wittig Sigmatropic Rearrangements in Organic Synthesis.", Nakai, T.; Mikami, K. Chem. Rev. 1986, 86, 885. Marshall, J. A. The Wittig Rearrangement.; Trost, B. M. and Fleming, I., Ed.; Pergamon Press: Oxford, 1991; Vol. 3, pp 975.
Li O R BuLi O R

consider as cycloaddition

FMO analysis

Ea ~16 Kcal/mol R This 1,2-sigmatropic rearrangement is non-concerted


R

R Li O Li

R
olefin radical cation transition state

The Wittig Rearrangement [2,3]


R R concerted O OLi

[1,2]-Sigmatropic rearr to carbanionic centers not observed R

stepwise

consider as cycloaddition

FMO analysis

FMO analysis R

C antibonding

ketyl radical R H transition state

R H

R
olefin radical anion transition state Allyl radical

The G between concerted and non-concerted pathways can be quite small

13-03-Sigmatropics-3 10/15/03 8:53 AM

D. A. Evans

[2,3]-Sigmatropic Rearrangements: An Introduction


[2,3] Sigmatropic Rearrangements
O R2 R3 R :X Y R3 R :X R2

Chem 206
OH Me Me Me Me

The basic process:


R2 R

BuLi

O Me Me Me Me

OH

Me R3

Me Me

temp Me

Me

Me

Me

Me

Y:

Rautenstrauch, Chem Commun. 1979, 1970

25 C

~70%

~30%

X & Y = permutations of C, N, O, S, Se, P; however X is usually a heteroatom Attributes: Stereoselective olefin construction & chirality transfer Representative X-Y Pairs: NO (amine oxides) SC (sulfur ylids) OC (Wittig rearrangement) NC (nitrogen ylids) SS (disulfides) SP, SN, SO (sulfoxides) OP (phosphites) NN, Cl+C (haloium ylids) PC, CC (homoallylic anions).

X - S, Y = C; Sulfonium Ylide Rearrangement:

BuLi

[2,3] S Li+ S Lythgoe, Chem Commum 1972, 757 S S

S S

p. p.

X - N, Y = C; Ammonium Ylide Rearrangement: Sommelet-Hauser:


[2,3] CH2 Me H CH2 NMe2 base Me NMe2

An important early paper: Baldwin, J. Chem. Soc., Chem. Comm. 1970, 576 General Reviews:
Trost, Ed., Comprehensive Organic Synthesis 1992, Vol 6, Chapter 4.6: Nakai, T.; Mikami, K. Org. React. (N.Y.) 1994, 46, 105-209. Hoffmann, Angew. Chem. Int. Ed. 1979, 18, 563-572 (Stereochemistry of) Nakai, Chem. Rev. 1986, 86, 885-902 (Wittig Rearrangement) Evans, Accts. Chem. Res. 1974, 7, 147-55 (sulfoxide Rearrangement) Vedejs, Accts. Chem. Res. 1984, 17, 358-364 (Sulfur Ylilde Rearrangements)

BuLi N Me N Me

Me Me

X - O, Y = C; Wittig Rearrangement:
Me O Ph R O Ph
BuLi

Review, Pines, Org. Rxns 1970, 18, 416 [2,3] Me LiO Me Ph R1 Me N R Ph + CN

Me
BuLi

Me O

Me Li+ Ph

Modern versions of Stevens:


R2 R3 BuLi R1 Me N + CN Me2N CN R2 R3 [2,3] R1 R2 R3

Baldwin, JACS 1971, 93, 3556 [1,2] Li R Li O

R O Li+ Ph

O H Ph

Me

Me

Garst, JACS 1976, 98, 1526

13-04-[2,3] introduction 10/15/03 8:55 AM

Buchi, JACS 1974, 96, 7573 Mander, JOC 1973, 38, 2915 important extension lacking CN FG; Sato, JACS 1990, 112, 1999

D. A. Evans
X - O, Y = C; Wittig-like Rearrangements
R2 R1 OH OMe H NMe2 OMe R1 O C: NMe2 R3 140 C R1 O C R2 R3

[2,3]-Sigmatropic Rearrangements: Introduction-2


X - S, Y = O; Sulfoxide Rearrangement
R2 R1 O R3 NMe2 R1 Ar

Chem 206

R2 R3 O keq < 1 R1 + S

R2

R2 R3 thiophile BuLi PhSCl R1

R3

S Ar

OH

+ NMe2 R2 R3

Buchi, JACS 1974, 96, 5563

Evans, Accts. Chem. Res. 1974, 7, 147

In thinking about this rearrangement, also consider the carbenoid resonance form as well

X - Se, Y = N; Related Rearrangement


R2 R1 Ar

R2 R3 R1 Se Ar

R2 R3 Ts
selenophile R1

R3

X - O, Y = C; An all-carbon Rearrangement p. p.
R1 O N N note that the product contains the retrons for the enolate Claisen rearrangement Smith, Chem. Commun. 1974, 695; Smith, JOC 1977, 42, 3165 R2 R3 Cu(I) N2 O : H O C ROH R2 R1 O OR R3 R1 R2 R3 R1 R2 R3

+ Se

N Ts

NHTs Hopkins, Tet Let. 1984, 25, 15 Hopkins, JOC 1984, 49, 3647 Hopkins, JOC 1985, 50, 417

X - S, Y = N; Related Rearrangement
Na+ TsNCl

TsO S Ph

TsO Ts N S Ph

[2,3]

TsO

Ts N SPh (MeO)3P NaOH

X - N, Y = O; Meisenheimer Rearrangement
R2 R1 Me N + O R3 R1 Me N Me Tanabe, Tet Let. 1975, 3005 O R2 R3 Zn/HOAc R1 R2 R3 OH

85% yield overall

N Ts

TsO

N Ts

Me

Dolle, Tet Let. 1989, 30, 4723

13-05-[2,3] intro-2 10/15/03 8:55 AM

D. A. Evans

[2,3]-Sigmatropic Rearrangements: Olefin Geometry


Me O Ra Rb
H H

Chem 206

1,2-Disubstitution: Good Trans Olefin Selectivity Starting olefin: Trans favored


Ra X Y: Ra Rb X Y Ra Rb :X Y

RLi -75 to -50 C Ph

Me

(E) selectivity: "only isomer"


HO Ph

Me O

2 LDA -75 to -50 C CO2H

Me

(E) selectivity: 75%


HO CO2H Nakai, Tet. Lett 1981, 22, 69 R1 S Ar O R2 (MeO) P 3 MeOH OH (E) selectivity: >95% R1 R2

Rb
H

disfavored
Y

X Rb

Ra Y :X

R1 Ar S O

R2

p. p.

Ra & Rb prefer to orient in pseudo-equatorial positions during rearrangement; nevertheless, this is a delicately balanced situation Starting olefin: Cis favored
Ra X Y: Rb X Y
H

RLi R1X R2 Ar S

Evans, Accts. Chem. Res. 1974, 7, 147-55 O

Ra
H

Ra :X Y Rb

The preceeding transition state models do not explain some of the results: Cis selectivity has been observed: Still JACS 1978, 100, 1927.
R Me O SnBu3 Me n-BuLi -78 C R ratio, 65:35 OH R Me OH

Rb

highly disfavored
Y

Rb

Ra
H

Rb Ra Y :X

However, Cis selectivity is dependent on starting olefin geometry


R Bu3Sn O Me n-BuLi -78 C R Me OH only (E) isomer (91%)

Conclusions
Olefin geometry dictates sense of asymmetric induction in rearrangement (Z) Olefin rearrangements might exhibit higher levels of 1,3 induction Product olefin geometry can be either (E) or (Z) from (E) starting material Product olefin geometry will be (E) from (Z) starting material

Several theoretical studies have been published: Good reading


Houk JOC 1991, 56, 5657 (Sulfur ylide transition states) Houk JOC 1990, 55, 1421 (Wittig transition states)

13-06-Stereochemistry-1 10/15/03 8:56 AM

D. A. Evans
Starting olefin: (E) Trisubstituted
R1 Y H R2 X Y: R1 H Me R2

[2,3]-Sigmatropic Rearrangements: Olefin Geometry


(Z) selectivity has been observed: Still JACS 1978, 100, 1927.
Me Me R1 :X Y R2 O Li n-BuLi -78 C KH O SnBu3 n-Bu OH n-Bu n-Bu Me OH

Chem 206

95%, >96% (Z)

favored
Me R1

Me H Me Me R2 R1 Y :X n-Bu

Me SnBu3 halogen

disfavored
Y

R2

Still says that the TS is early, so that the 1,2 interactions in the TS are most important. destabilizing
O H 2C Li H H C 4H 9 C4H9 Me H LiO CH2 n-Bu Me

p. p.

R1Me interaction can destabilize the (E) transition state while (Z) TS might be destabilized by R1 interactions with both X-Y and allyl moiety. (E)-path Starting olefin: (Z) favored
Me R1 X Y: R2 X Y H R2 R1 Me H :X Y R1 Me R2 Me n-Bu O CH2Li

Me

(Z)-path
H2 C

O Li

H Me R1 CH2 LiO

highly disfavored
X Y

R2

R1 H Me

Me R2 R1 Y :X

(Z) selectivity has also been observed by others: Sato JACS 1990, 112, 1999.
Me LHMDS, NH3 -70 C Me N +Me Me XMe CsF in HMPA Me
X-

Me NMe2

Conclusions
Olefin geometry dictates sense of asymmetric induction in rearrangement (Z) Olefin rearrangements might exhibit higher levels of 1,3 induction Product olefin geometry can be either (E) or (Z) from (E) starting material Product olefin geometry will be (E) from (Z) starting material

76%, (Z):(E) 95:5


NMe2 25 C

N+ CH2TMS Me

Me

61%, (E):(Z) 100:0

13-07-Stereochemistry-2 10/15/03 8:57 AM

D. A. Evans

[2,3]-Sigmatropic Rearrangements: Olefin Geometry


R1
H H H

Chem 206

Trisubstituted olefins via [2,3]-rearrangement of sulfoxides:


PhS O R1 Me Me

Trisubstituted olefins via [2,3]-rearrangement of sulfonium ylides:


Me Bu Me CuSO4 S 100 C N2 C(COOMe)2 Bu S CO2Me Ph + CO2Me Bu CO2Me S Ph CO2Me Me

(E)-path
Me R1 Ar S O

favored

S Ph Me

O Ph

R1

(Z)-path disfavored

Ph O

Grieco, JOC 1973, 38, 2572 R1 S Ph O

(E):(Z) > 90:10 (70%)

S
H

Me

A general procedure for the direct synthesis of sulfur ylides:


R S R + :CR2 R R +S C R R
base

In contrast to the previous cases exhibiting (Z) selectivity rearrangements (E)-selective rearrangments has been observed: p. p.
Me n-BuLi N N S Me N N

Me S Li

Me

Me

R R +S C H R R
pKa ~ 18 (DMSO

Me Br

Me

N N

Me

Me

Me / = 90:10 (95%) RCO3H

Trisubstituted olefins via Wittig [2,3]-rearrangement:


Me C5H11 O
2 LDA

Accts. Chem. Res. 1974, 7, 147-55

Me C5H11 HO (E):(Z) > 95:5 (74%) CO2H Nakai, Tet Let 1981, 22, 69

Me

Me [2,3]

Me OH Et2NH, MeOH 25 C Me

Me CO2H

Me

N N Me

S Me

(E):(Z) > 97:3 (80-85%)


Me () N N S Me

However, this reaction is not general:


Me Me O
LDA

Me (E):(Z) 31:69 Me HO CO2Me Nakai, Tet Let 1986, 27, 4511

is operationally equivalent to:

() H OH

CO2Me

13-08-Stereochemistry-3 10/15/03 8:57 AM

D. A. Evans

[2,3]-Sigmatropic Rearrangements: Olefin Geometry


RL
H

Chem 206

Trisubstituted olefins via [2,3]-rearrangement:


X Y RM
H

An elegant squalene synthesis Ollis, Chem. Commun 1969, 99


Me RM Y :X Me For study on this [2,3] rxn See Baldwin JACS 1971, 93, 6307 Me RL :X Y Me PPh3 S=PPh3 Me Me Me S S Me [2,3] heat Me RM S Me S Me Me Me Me Me Me

RL

(E)-path
RL RM (RL = large) X Y:

RL

RM

(Z)-path
Y

X
H

One might project that the (E) path will be moderately favored with selectivity depending on size difference between RL & RM p. p.
Me Me Me S Me Me Rautenstrauch, Helv. Chim Acta 1971, 54, 739 Me Me Me OMe H NMe2 OMe Buchi, JACS 1974, 96, 5563 OH 140 C Me O C: NMe2 O Me Me Me poorly selective NMe2 Me Me Me Me Li/NH3 Me Me Me Me
n-BuLi

Me Me Me Me S

Me

Me Me F

Me Me

Me S Li Me Me Me Me SLi Me Me Me

benzyne MgBr Me Me + S Ph This rxn is probably not as stereoselective as advertised Me Me Me Me Me [2,3] For related [2,3] rxns See Baldwin JACS 1968, 90, 4758 Baldwin JACS 1969, 91, 3646 Me Me Me Me

Me

(E):(Z) = 3:2

Me

SPh Me Me

"gave one major product in high yield"

Me

Squalene

Me

Me

Me

13-09-Stereochemistry-4 10/15/03 8:57 AM

D. A. Evans

[2,3]-Sigmatropic Rearrangements: Chirality Transfer

Chem 206

[2,3] Sulfur Ylide Rearrangement Using a Chiral Auxiliary


Kurth JOC 1990, 55, 2286 and TL 1991, 32, 335 Me Me CO2H steps Me NH2 SH Me
steps

Chiral Auxiliaries can also be used in the Wittig Rearrangement

Me CO2H
Allylation

Me Me S CO2H

O CH2OR O ROCH2 Me N

BuLi

96% de (61%) HO O XC

Me
DBU, -78 C

Me O S+
BF4 HBF4 CH2Cl2

O O S O O N2 ROCH2 N CH2OR

BuLi Cp2ZrCl2

Me HO O Katsuki, Tet Lett 1986, 27, 4577 XC 97% syn; 96% de (43%)

Me

Me

p. p.
Me Me Me S Me
+

Me

Me O O Me S Me O O Me

Me O S Me Bu3Sn O

Internal Relay of Stereochemistry in CC Constructions


O Me Me C 3H 7 Me OBOM
n-BuLi, THF, -78 C

Me

Me

Me C 3H 7

66%, 94:4

OH

OBOM

diastereoselection > 100:1 (57%) Me Me S


+

Me O O Me S Me O O Me

Me O S Me O

Bu3Sn

Me Me C 3H 7 Me OBOM
n-BuLi, THF, -78 C

Me

Me

Me C 3H7

OH

OBOM

Kallmerten TL 1988, 29, 6901.

diastereoselection > 100:1 (64%) Kallmerten TL 1993, 34, 753. Kallmerten TL 1993, 34, 749. Kallmerten SynLet 1992, 845.

Me

64%, 4:93
See these papers for other applications

13-10-applications 10/15/03 8:58 AM

D. A. Evans

[2,3]-Sigmatropic Rearrangements: Chirality Transfer

Chem 206

Internal Relay of Stereochemistry in CO Constructions Tandem [ 4+2 ] & [ 2,3 ] Process:


Evans, Bryan, Sims J. Am. Chem. Soc. 1972, 2891.

Cases where the chirality is exocyclic to the rearrangement


Me O O Me
n-BuLi

Me O O HO ratio 79 : 6 HO Me

Me O O Me

N Me N S Me O Ph OMe HO
Na2S, MeOH

O SnBu3

Ph

A Felkin analysis predicts the major product

H 2C O H C O C

HO Bruckner, Angew. Chem. Int. Ed. 1988, 27, 278 N cepharamine Me

p. p.

MeO

N Me

Allylic Ethers to Make Three Contiguous Stereocenters


H O
15

C5H11
HSPh, KOt-Bu

O CO2Et C5H11
15

TBSO

Me
n-BuLi, -78 C

CO2Et H

TBSO O TMS TBSO Me

OH 94% TMS OH 4% TMS

SPh

SPh
1) MCPBA 2) P(OMe)3

O CO2H C5H11 OH
steps

O CO2Et
15

Nakai TL, 1988, 29, 4587. C5H11

Me

OH Taber J. Am. Chem. Soc. 1977, 99, 3513. Kondo Tet. Lett. 1978, 3927.

Can you rationalize the stereochemical outcome of this reaction?

13-11-applications-2 10/15/03 8:58 AM

D. A. Evans

[2,3]-Sigmatropic Rearrangements: Chirality Transfer

Chem 206

The Synthesis of Bakkenolide-A (Evans JACS 1977, 99, 5453)


Me O O CH2
H H

[2,3] Sigmatropic rearrangements respond to subtile steric effects


Me
H

Me Me HO O Me
H H

Me

Me HO C: H O Me

H Me3C H S Ph
N2=CHCO2Et

X :Y

Me3C H + Ph S C H CO2Et Me3C

X: favored

Candidate processes:
R3 OH H OMe NMe2 OMe R3
140 C

SPh H CO2Et

R3 O NMe2 CMe3 S O Ph

Cu(I) catalysis 25 C

C: NMe2

selectivity: 91:9 CMe3

Buchi, JACS 1974, 96, 5563 R3

p. p.
S C N

R3 SMe NHTs
NaH 65 C

25 C (MeO)3P MeOH

R3 S SMe CMe3 N Me Me Me Me Me +

Me3C H

OSPh

Me3C

OH

C: SMe

H selectivity: 92:8 Evans, JACS, 1972, 94, 3672

Baldwin, Chem Comm 1972, 354

The synthesis:
Me Me O MgBr H Me S
Note that rearrangement is not required to proceed via the carbenoid. propose altenate mechanism

C CN H

-10 C

Me

Me3C H

N CN

selectivity: 90:10 Mander, JOC, 1973, 38, 2915

CMe3 HO H Br C N SMe NHTs Me Me


NaH 65 C

The comparison of analogous [2,3] & [3,3] rearrangements:


OEt O O heat CMe3 O heat Me3C H CMe3 Me3C H H O selectivity: 75:25 OEt selectivity: 52:48

Me Bakkenolide-A
HgCl2, HOH MeS H2SeO3

Me
H

Me

S Me 65% (no other isomer) N

S C SMe

House, JOC 1975, 40, 86

NHTs

13-12-applications-3 10/15/03 8:58 AM

D. A. Evans

[2,3]-Sigmatropic Rearrangements: Ring Expansion & Contraction

Chem 206

Ring expansion reactions have been investigated


Methods based on sulfur ylides: (review) Vedejs, Accts. Chem. Res. 1984, 17, 358 Me

A ring contraction using the Wittig Rearrangement


Me
R2NLi

Me Me HO

S N2

Cu(O)

CO2Et +S

Me O Li N Ph With chiral amide Ph bases induction is observed! Me Me

Me

CO2Et

CO2Et

50%

82%, 69% ee Me Aristolactone Me O O

KOt-Bu

TfO S 72%

CO2Et
DBU

S +

S EtO2C

72%

Marshall, JACS 1988, 110, 2925

p. p.
O

Me

HO Me

Me O Et O

OH Methynolide has been synthesized by Vedejs using this ring-expansion methodology Me Vedejs, JACS 1989, 111, 8430

A ring contraction using the Stevens Rearrangement


Me Me Me Me N Me Br O + N Me Ph
MeO MeOH

Me Me 54% Me N Me Ph O

An early ring expansion using the Sommelet-Hauser Rearrangement

+ N Me Me

NaNH2 liq NH3

Me N+ CH2 N Me H

O Ph

salts readily separated Me O Me + N Me Ph


MeO MeOH

nonselective N-alkylation

Me Me 78% Me N O

83% Hauser, JACS 1957, 79, 4449

Me Ph Both rearrangements afford a single isomer

N Me

Stevenson, Tet. Lett 1990, 31, 4351

13-13-applications-4 10/15/03 8:58 AM

D. A. Evans

Pericyclic Reactions: Part4


Other Reading Material:

Chem 206

http://www.courses.fas.harvard.edu/~chem206/

[3,3] Sigmatropic Rearrangements


Trost, Ed., Comprehensive Organic Synthesis 1992, Vol 5, Chapter 7.1: (Cope, oxy-Cope, Anionic oxy-Cope) Chapter 7.2, Claisen S. J. Rhoades, Organic Reactions 1974, 22, 1 (Cope, Claisen) S. R. Wilson, Organic Reactions 1993, 43, 93 (oxy-Cope)

Chemistry 206 Advanced Organic Chemistry


Lecture Number 14

T. S. Ho, Tandem Organic Reactions 1992, Chapter 12 (Cope, Claisen) Paquette, L. A. (1990). Stereocontrolled construction of complex cyclic ketones by oxy-Cope rearrangement. Angew. Chem., Int. Ed. Engl. 29: 609.

Pericyclic Reactions4
[3,3] Sigmatropic Rearrangements: Introduction Cope Rearrangements & Variants Claisen Rearrangements & Variants

Problems of the Day:


Predict the stereochemical outcome of this Claisen rearrangement
Et Et O 144 C, 6h O

Reading Assignment for week: Carey & Sundberg: Part A; Chapter 11 Concerted Pericyclic Reactions Fleming: Chapter 4: Thermal Pericyclic Reactions
K. Houk, Transition Structures of Hydrocarbon Pericyclic Rxns Angew Chem. Int. Ed. Engl. 1992, 31, 682-708 K. Houk, Pericyclic Reaction Transition States: Passions & Punctilios, Accts. Chem. Res. 1995, 28, 81-90 Angew Chem. Int. Ed. Engl. 1992, 31, 682-708 CMe3

diastereoselection >87:13
CMe3 Ireland, JOC 1983, 48, 1829

Provide a mechanism for the indicated fransformation


OH Me
KH, THF H3O+ quench

O H Me H Me

D. A. Evans

Friday, October 16, 2003

Me

Schreiber, JACS 1984, 106, 4038

14-00-Cover Page 10/16/03 7:29 PM

D. A. Evans
General Reviews:

Introduction to [3, 3]-Sigmatropic Rearrangements


The CopeTransition States

Chem 206

S. J. Rhoades, Organic Reactions 1974, 22, 1 (Cope, Claisen) S. R. Wilson, Organic Reactions 1993, 43, 93 (oxy-Cope) T. S. Ho, Tandem Organic Reactions 1992, Chapter 12 (Cope, Claisen) Trost, Ed., Comprehensive Organic Synthesis 1992, Vol 5, Chapter 7.1: (Cope, oxy-Cope, Anionic oxy-Cope) Chapter 7.2, Claisen

X Z X Z X Z

CHAIR Relative Energy G: 0

BOAT + 5.8 kcal/mol

?
X

CHAIR Relative Energy G: 0

BOAT + 5.3 kcal/mol

p. p.
X

X & Z = C, O, N etc
X

The Boat and Chair geometries for these transition structures are well defined.
O O

The FMO Analysis (Fleming page 101)


Cope Rearrangement, Ea = 33.5 kcal/mol Claisen Rearrangement Ea = 30.6 kcal/mol

Bring two allyl radicals together to access for a possible bonding interaction between termini.

The Reaction Energetics Goldstein, JACS 1972, 94, 7147

X X

G523 = 46.3 G523 = 40.5

The nonbonding allyl MO

bonding bonding

It is evident that synchronous bonding is possible in this rearrangement

14-01-[3,3] intro 10/16/03 7:16 PM

D. A. Evans

The DoeringRoth Experiments


The Results
H Me Me H Me Me Me Me H Me H less favored H Me H disfavored H Me Me Me H Me Me H Me Me H

Chem 206
favored
H Me Me

Doering/Roth Experiments: Tetrahedron 18, 67, (1962): The Geometry of the transltion state (boat vs chair) can be analyzed via the rearrangement of substituted 1,5-dienes:
Me Me

trans-trans: 90%

Results: Threo isomer


Me

cis-cis: 10%

Threo isomer
Me Me Me Me

Me

trans-cis: < 1%

Meso isomer Measure product composition from rearrangement of each diene isomer p. p.
Me Me H Me H Me H less favored H Me H disfavored H Me H Me H Me Me Me H Me H Me disfavored H H H H H Me Me Me trans-trans H H 60 C Reese Chem. Commun. 1970, 1519
equilibrium stongly favors this isomer

favored
H H Me

Me

trans-cis: 99.7% G ~ 5.7 kcal/mol

favored

Me Me

trans-trans

Results: Meso isomer

H Me

Me H H

disfavored
H H

Me

trans-trans: 0.3%

Predictions: Threo isomer

H H Me Me

cis-cis

Ring Strain can be employed to drive the Cope process:


H

Me

trans-cis

H 5-20 C Brown Chem. Commun. 1973, 319

favored

trans-cis
H

120 C

Vogel Annalen 1958, 615, 1

Predictions: Meso isomer

14-02-Doering/Roth 10/16/03 7:17 PM

D. A. Evans

Strain Accelerated-Cope Rearrangements


Ring extension via divinylcyclopropane rearrangement
O
"quantitative"

Chem 206

Ring Strain can be employed to drive the Cope process:


20 C

O Me

Me Me
heat xylene

OMe

Me

90% Me Me
LDA MeI

W. von E. Doering's Bullvalene


Li (PhS)Cu

Me 1.5 equiv

Me O Me Me Me (EtO)2P
Li

OMe

Me
H2 (Ph3)3RhCl LDA

Bullvalene: Ea = 13.9 kcal/mol At 100 C one carbon is observed in nmr spectrum

Me

OMe Me

Me

EtNH2/THF

O Me

p. p.

Carey, Vol 1, page 630-630

Me

Me

(EtO)2PCl

-himachalene
O Me CHO Vogel Angew. Chem. Int. Ed. 1963, 2, 739
favored Wittig

Piers, Can J. Chem. 1983, 61, 1226, 1239 O Me CO2Et


140 C

Position of Equilibrium dictated by ring strain issues:


H

CO2Et Me

60-70%

+ S(O)Me2

Marino, J. Org. Chem. 1974, 39, 3175

H H Wharton J. Org. Chem. 1973, 38, 4117 H

Accelerated Cope Rearrangements


HO

k1

HO

HO

However, tautomerism can shift the equilibrium:


H 220 C 3h OH OH keq ~ 10+5 O 90% O Evans, Golob, JACS 1975, 97, 4765. Trost, Ed., Comprehensive Organic Synthesis 1992, Vol 5, Chapter 7.1: (Cope, oxy-Cope, Anionic oxy-Cope)
"Recent applications of anionic oxy-Cope rearrangements." Paquette, L. A. Tetrahedron 1997, 5 3, 13971-14020

k2

k2 = 10 + 10 k1

10 + 17

Marvell, Tet. Lett. 1970, 509 Energetically, how much does tautomerization give you? keq ~ 10+5 G = 1.4(pKeq) = 1.4X(-5) = -7 kcal/mmol

14-03-Strain Acceleration 10/15/03 12:03 PM

D. A. Evans

The Anionic Oxy-Cope Rearrangement


Documentation of Alkoxy Substituent Effect
OX OH O Me MeO OX H H OMe Me OH
3 4 6

Chem 206

The Aborted Oxy-Cope Reaction (circa 1969) The basic reaction


Me

OX OH

Half-life (66 yrs) no rxn 1.2 hrs 1.4 min 11 hrs 4.4 min 10 C 66 C

OH

Me

66 C THF

OLi ONa OK

Actual case studied:


1

Me
4 3 6

1 6

O
3

10 +12 rate acceleration

OK (HMPT) O K
-+

OH

Me XO Me MeO

Me Me Me

Me

Me

p. p.
HO

66 C THF

No Rxn

Gestimate = 15 kca mol -1 Gexperiment = 13 kca mol -1

Prediction of Substituent Effect (circa 1969) GOH


HO

with A. M. Golob, JACS. 1975, 97, 4765.


HO

GOH

Origin of the Rate Effect


pka (SM)
O

pka (TS)
O

GO ??

GO

HO

GO = GOH+ 2.3RT [ pka TS pka SM] GO = GOH GO + 2.3RT [18 29] (in DMSO) 1.4 [ 11]
HO

Effect probably comes from both reactant destabilization & transition state stabilization

GO = GOH +

~ 15 kcal/mol

= GOH 15 kcal/mol at 298 K (in DMSO)


Maximal rates are observed under conditions where reactant is maximally destabilized

14-04-oxy-Cope 10/15/03 12:04 PM

D. A. Evans

Anionic Substituent Effects: Bond Homolysis


Substituent Effects in Molecular Rearrangements
+ CR3

Chem 206

Substituent Effects in Bond Homolysis


HO C CR3 A O C CR3 DI I DI HO C B + CR3 X

[3,3]

X R

[1,3]

X R

O C
ketyl

Y R

X Y R

X Y

[2,3]

[1,2]

DI DI I = 2.3 RT[pka (A) pka (B)]


H Acidities of these radicals are known in H2O p. p. HO C Hayon, Accts. Chem. Res. 1974, 7, 114 (H2O) HO C H pKa = 10.7 Ph HO C Ph pKa = 9.2 Y X X H

X ene
H

In DMSO:

D = 2.3 RT[29 pka 18] = ~ 15 kcal/mol

C C

Y X

Substituent Effect based on ab initio calculations


(Evans, Goddard, JACS 1979, 101, 1994)

Y X H HO C H H BDE = 90.7 (BDE = 91.8 expt) H NaO C H H BDE = 80.6 H KO C H H BDE = 79.0 H O C H H BDE = 74.2

C C

C C C

Y X

C X C

D +16.5 kcal/mol

C X

Related papers: Evans, Baillargeon, Tet Lett. 1978, 36, 3315, 3319

X C

X C

X C

X C

RH

14-05-oxy-Cope-2 10/15/03 12:04 PM

D. A. Evans
MeO OMe MeO OMe

Anionic Oxy-Cope Rearrangement: Applications


O O O O Me Me O H Me OR Me O

Chem 206
Periplanone-B Synthesis
Me

O MgX

OH

NaH 75%

O
H H

OMe OMe H2 C

200 g from 75,000 virgin female cockroaches

H OH

KH THF
ROH2C Me

Me

Jung, JACS 1978, 100, 4309 Jung, JACS 1980, 102, 2463
OH

ROH2C

Me

Still, JACS 1979, 101, 2493


OH

KH, THF

O Levine, JOC 1981, 46, 2199 Me H Me

OR

KH THF 2 steps
Me H Me Me O Me

OH Me

O Me

Schreiber, JACS 1984, 106, 4038 KH, THF


Me Me

Gadwood, JOC, 1982, 47, 2268

Synthesis of (+)-CP-263,114: Shair, JACS 2000, 122, 7424-7425.


O O O HC CLi OH Me H Me H O O CO2H Me H OH MeO2C R H R H XMgO CH OR 2 H Me O C OMe R H R H XMgO CH OR 2 Me Me H O O H O R H R O H O H Me Me O

50 C 50 % yield

OH OH

Me OH

[3,3] 7823 C

Me

H Me

Me

Me

poitediol

dactylol

Dieckmann
CH2OR

53%

Gadwood, JACS, 1986, 108, 6343

14-06-Oxy-Cope appl 10/15/03 12:04 PM

J. Leighton, D. A. Evans
Me

Oxy-Cope Problems
Me Me Me O Me Me

Chem 206
Me H

Propose a synthesis of -amorphene using 1-methyl-1,3-cyclohexadiene. H

?
Me Me H

-amorphene
THPO

Me Me Gregson, R. P.; Mirrington, R. N. J. Chem. Soc., Chem. Commun. 1973, 598. Et Me By incorporating a carbonyl group into this structure generate all possible oxy-Cope retrons. Which is (are) the most reasonable? Brub, G.; Fallis, A. G. Tetrahedron Lett. 1989, 30, 4045.

THPO

Propose a three step synthesis of B from A. Koreeda, et al. J. Org. Chem. 1980, 45, 1172. Me Me OH MeO Me Me OH OMe KHMDS THF, 20 C 51% KHMDS THF, 20 C 88% Me Me O H H OMe H H Me Me O H

H H Me O H

By incorporating a double bond into this structure generate all possible oxy-Cope retrons. Which is (are) the most reasonable? Ireland, et al. J. Org. Chem. 1981, 46, 4863. O Et H H OMe OMe

H H By incorporating a double bond into this structure generate all possible oxy-Cope retrons. Which is (are) the most reasonable? Paquette, L. A. et al. Tetrahedron Lett. 1987, 28, 31. MeO Me H

Me

H Propose detailed mechanisms for these reactions. Rationalize the different behavior of these enol ether isomers. Paquette, L. A.; Reagan, J.; Schreiber, S. L.; Teleha, C. A. J. Am. Chem. Soc. 1989, 111, 2331-2332. O O Me HO O mesitylene, N Me Me 72 h 48% H O O O Me O Me Me

O H Li O

Me

H Me

Propose a synthesis of A using the illustrated dihydropyran synthon. Oplinger, J. A.; Paquette, L. A. Tetrahedron Lett. 1987, 28, 5441.

Propose a detailed mechanism paying particular attention to issues of chemo- and stereoselectivity. Jacobi, P. A.; Selnick, H. G. J. Org. Chem. 1990, 55, 202.

14-07 Oxy-Cope/probs 10/15/03 12:05 PM

D. A. Evans
General Reviews:

The Claisen Rearrangement

Chem 206

Recognition Pattern for Organic Synthesis: An Enforced SN2'


R O R R R

S. J. Rhoades, Organic Reactions 1974, 22, 1 (Cope, Claisen) Trost, Ed., Comprehensive Organic Synthesis 1992, Vol 5, Ch 7.2 Ziegler, Accts. Chem. Res. 1977, 10, 227 (Claisen) Bennett, Synthesis 1977, 589 (Claisen) Blechert, Synthesis 1989, 71 (HeteroCope) R. K. Hill, Asymmetric Synthesis vol 3, Ch 8, p503 (chirality transfer) Ziegler, Chem Rev. 1989, 89, 1423 (Claisen)

Claisen

O R

SN2'

The Reaction:
O R

O R

H ~ 20 kcal mol-1

Stereochemical outcome is syn and controlled by hydroxyl stereocenter


1 O

X R
2

There is good thermodynamic driving force for this reaction. Bonds Broken: C-C (65 kcal mol ) & C-O (85 kcal mol ) Bonds Made: C-O (85 kcal mol-1) and C-C (85 kcal mol-1) R
-1 -1

X O
1 O

p. p. Themodynamics of Claisen Variants:


X O O X

X R
2

Substituent X=H X = OH X = NH2

H (kcal mol-1) 16 31 30
(Benson estimates)

Control of stereocenter 2 evolves into a decision how to establish the hydroxyl-bearing stereocenter

X O

Rearrangements of Aryl Allyl Ethers: Traditional Applications


O O
180-200 oC

OH H 77%

~ 30
O O

OR ~ 20

O
180-200 oC

O
Cope

O H 91%

OH

~ 20 kcal/mol

~ 30 kcal/mol

OR
O

OH

E:Z = 6.7:1

Heteroatom substitution at the indicated position increases exothermicity as well as reaction rate

65%

14-08-Claisen-1 10/16/03 7:22 PM

D. A. Evans

The Claisen Rearrangement-2


Synthesis of Allyl Vinyl Ethers
Hg(OAc)2 -EtOH

Chem 206

Stereoelectronic & steric constraints


Endocyclic Olefins: Ireland, JOC 1983, 48, 1829
Et O O Et 144 C, 6h Me3C H CMe3 for endocyclic olefins, overlap between developing sigma and pi bonds required. Best overlap for forming chair geometry. As shown below, bring a radical up to either face of the allylic radical. As the bond is formed, overlap must be maintained. Parh A evolves into a chair conformation while Path B evolved into a boat conformation. H R H C H CH2 diastereoselection >87:13 OH OEt (solvent)

AcOHg

O OEt

75%

Watanabe, Conlon, JACS 1957, 79, 2828 Bronsted acids can also serve as catalysts O CH2 Cp2Ti AlMe2 Cl Ph O 96% Ph CH2 O

p. p.

H Me3C H2C

R B H Me3C H2 C

Use of Tebbe's Reagent: Evans, Grubbs, J. Am. Chem. Soc. 1980, 102, 3272. (review) S. H. Pines, Organic Reactions 1993, 43, 1 H

A H R

Me3C H2C R

The Ireland approach to the bicyclic acid A: Exocyclic Olefins: House, JOC 1975, 40, 86
OEt O O heat CMe3 O Me3C H H heat CMe3 Me3C H OEt Me3C O OEt HO H Me Me Me X O Claisen
X=H

JOC 1962, 27, 1118


OH

H Me O Me

HO Me Me H

HO Me Me
Me

ratio 52:48
O Me3C

Me O H H HO Me Me

OH Me
EtOCH=CH2 Hg(OAc)2

Me OH
HH

Me H

ratio 75:25

HO Me Me

53% overall

O H

for exoocyclic olefins, overlap between developing sigma and pi bonds is equally good from either olefin diastereoface. In this instance, steric effects dominate & this system shows a modest preference for "equatorial attack." A related case is provided below.

The new stereocenter () introduced via the rearrangement had the wrong configuration!

14-09-Claisen-2 10/16/03 7:23 PM

D. A. Evans

The Claisen Rearrangement: Stereoselective Olefin Synthesis

Chem 206

Claisen Rearrangement as vehicle for stereoselective olefin synthesis Consider the following rearrangement:
H ke CHO Me O

Faulkner suggests that the installation of other substituents on Claisen transition states will lead to enhanced reaction diastereoselection:
H ke X
110 C

O X O X Et Me O X O Et X

Et Me

O Me ka H

e
Me

Me O Ga - Ge = 1.5 kcal/mol O Me CHO Et Me ka


110 C

e
Et

(E)

H Me

(Z) a

a
Faulkner & Perrin (Tet. Lett. 2783 (1969) have made the correlation between G for rearrangement & G for the corrresponding cyclohexane# equilibria:

Me

The R2X interaction should destabilize a as X gets progressively larger. X


H Me MeO Me2N

p. p.
Me

H O H Me
#

Me H Me H O

(E):(Z) found
90:10 >99:1 >99:1 >98:2 Faulkner, Tet Let 1969, 3243 Faulkner, JACS 1973, 95, 553 Johnson, JACS 1970, 92, 741

G = +1.5 kcal/mol

G = +1.75 kcal/mol

Another comparison: (DAE) M. DiMare, Ph. D. Harvard University, 1988


OY X OPMB Et Me Et Me O R

Note: The A-value of 2-methyl-tetrahydropyran is +2.86 kcal/mol (LectureNo. 6)

They then suggest that there is a good correlation between cyclohexane "A-values" & G for the rearrangement process. Their case is fortified by the following expamples: CHO
110 C

O R2 R1

R2

CHO X

O OPMB Et Me

R2 R1 (E) R1 (Z)

R1
Me Me Et

R2
Et iPr Et

(E):(Z) found
90:10 93:07 90:10

(E):(Z) predicted
91:9 94:6 91:9 Faulkner, JACS 1973, 95, 553

procedure
Y = Ac, Ireland Y = H, Johnson Y = H, Eschenmoser

conditions
LDA, TMSCl HC(OMe)3, H+ MeC(OMe)2NMe2

X
TMSO MeO Me2N

T, C
-78+55 130 80

(E):(Z) ratio
97:3 94:6 97.5:2.5

14-10-Claisen-3 10/16/03 7:24 PM

D. A. Evans

Pericyclic Reactions: Part5


s Other Reading Material:

Chem 206

http://www.courses.fas.harvard.edu/~chem206/

Chemistry 206 Advanced Organic Chemistry


Lecture Number 15

Pericyclic Reactions5
s Claisen Rearrangements & Variants

Enders, D.; Knopp, M.; Schiffers, R. "Asymmetric [3.3]-sigmatropic rearrangements in organic synthesis." Tetrahedron: Asymmetry 1996, 7, 1847-1882 Ziegler, F. E. "The Thermal Aliphatic Claisen Rearrangement." Chem. Rev. 1988, 88, 1423. Gajewski, J. J. "The Claisen rearrangement. Response to solvents and substituents: The case for both hydrophobic and hydrogen bond acceleration in water and for a variable transition state." Acc. Chem. Res. 1997, 30, 219-225. Tietze, L. F. "Domino reactions in organic synthesis." Chem. Rev. 1996, 96, 115-136. Parsons, P. J.; Penkett, C. S.; Shell, A. J. "Tandem reactions in organic synthesis: Novel strategies for natural product elaboration and the development of new synthetic methodology." Chem. Rev. 1996, 96, 195-206. Pereira, S.; Srebnik, M. "The Ireland-Claisen rearrangement." Aldrichimica Acta 1993, 26, 17.

s Problems of the Day:


Propose a mechanism for this transformation
N O O Cl Me Me EtN(iPr)2 TiCl4 O N Me O Me

s Reading Assignment for week: Carey & Sundberg: Part A; Chapter 11 Concerted Pericyclic Reactions Carey & Sundberg: Part B; Chapter 6 Cycloadditions, Unimolecular Rearrangements Thermal Eliminations Fleming: Chapter 4 Thermal Pericyclic Reactions
Wipf, P. Claisen Rearrangements.; Trost, B. M. and Fleming, I., Ed.; Pergamon Press: Oxford, 1991; Vol. 5, pp 827. New Aspects of the Ireland and Related Rearrangements, Tetrahedron 2002, 58, 29052928 (handout)
Me

MacMillan, JACS 1999, 121, 9726

Predict the stereochemical outcome of this reaction


OH O O Me
2 LDA

OH Me
h

O OTMS

D. A. Evans
15-00-Cover Page 10/20/03 9:40 AM

Wednesday, October 20, 2003

Me h

Kurth, JOC 1985, 50, 1840

D. A. Evans
General Reviews:

The Claisen Rearrangement

Chem 206

Recognition Pattern for Organic Synthesis: An Enforced SN2'


R O R R R

S. J. Rhoades, Organic Reactions 1974, 22, 1 (Cope, Claisen) Trost, Ed., Comprehensive Organic Synthesis 1992, Vol 5, Ch 7.2 Ziegler, Accts. Chem. Res. 1977, 10, 227 (Claisen) Bennett, Synthesis 1977, 589 (Claisen) Blechert, Synthesis 1989, 71 (HeteroCope) R. K. Hill, Asymmetric Synthesis vol 3, Ch 8, p503 (chirality transfer) Ziegler, Chem Rev. 1989, 89, 1423 (Claisen)

Claisen

O R

SN2'

The Reaction:
O R

O R

H ~ 20 kcal mol-1

Stereochemical outcome is syn and controlled by hydroxyl stereocenter


1 O

X R
2

There is good thermodynamic driving force for this reaction. Bonds Broken: C-C (65 kcal mol ) & C-O (85 kcal mol ) Bonds Made: C-O (85 kcal mol-1) and C-C (85 kcal mol-1) R
-1 -1

X O
1 O

p. p. Themodynamics of Claisen Variants:


X O O X

X R
2

Substituent X=H X = OH X = NH2

H (kcal mol-1) 16 31 30
(Benson estimates)

Control of stereocenter 2 evolves into a decision how to establish the hydroxyl-bearing stereocenter

X O

Rearrangements of Aryl Allyl Ethers: Traditional Applications


O O
180-200 oC

OH H 77%

H O

~ 30
O

OR ~ 20

O
180-200 oC

O
Cope

O H 91%

OH

~ 20 kcal/mol

~ 30 kcal/mol

OR
O

OH

E:Z = 6.7:1

Heteroatom substitution at the indicated position increases exothermicity as well as reaction rate

65%

15-01-Claisen-1 10/19/03 5:47 PM

D. A. Evans

The Claisen Rearrangement-2


Synthesis of Allyl Vinyl Ethers
Hg(OAc)2 -EtOH

Chem 206

Stereoelectronic & steric constraints


Endocyclic Olefins: Ireland, JOC 1983, 48, 1829
Et O O Et 144 C, 6h Me3C H CMe3 for endocyclic olefins, overlap between developing sigma and pi bonds required. Best overlap for forming chair geometry. As shown below, bring a radical up to either face of the allylic radical. As the bond is formed, overlap must be maintained. Parh A evolves into a chair conformation while Path B evolved into a boat conformation. H R H C H CH2 diastereoselection >87:13 OH OEt (solvent)

AcOHg

O OEt

75%

Watanabe, Conlon, JACS 1957, 79, 2828 Bronsted acids can also serve as catalysts O CH2 Cp2Ti AlMe2 Cl Ph O 96% Ph CH2 O

p. p.

H Me3C H2C

R B H Me3C H2 C

Use of Tebbe's Reagent: Evans, Grubbs, J. Am. Chem. Soc. 1980, 102, 3272. (review) S. H. Pines, Organic Reactions 1993, 43, 1 H

A H R

Me3C H2C R

The Ireland approach to the bicyclic acid A: Exocyclic Olefins: House, JOC 1975, 40, 86
OEt O O heat CMe3 O Me3C H H heat CMe3 Me3C H OEt Me3C O OEt HO H Me Me Me X O Claisen
X=H

JOC 1962, 27, 1118

H Me O Me OH

HO Me Me H

HO Me Me
Me

ratio 52:48
O Me3C

Me O H H HO Me Me

OH Me
EtOCH=CH2 Hg(OAc)2

Me OH
HH

Me H

ratio 75:25

HO Me Me

53% overall

O H

for exoocyclic olefins, overlap between developing sigma and pi bonds is equally good from either olefin diastereoface. In this instance, steric effects dominate & this system shows a modest preference for "equatorial attack." A related case is provided below.

The new stereocenter () introduced via the rearrangement had the wrong configuration!

15-02-Claisen-2 10/19/03 5:48 PM

D. A. Evans

The Claisen Rearrangement: Stereoselective Olefin Synthesis

Chem 206

Claisen Rearrangement as vehicle for stereoselective olefin synthesis


Consider the following rearrangement: H ke CHO Me O

Faulkner suggests that the installation of other substituents on Claisen transition states will lead to enhanced reaction diastereoselection: H ke X
110 C

O X O X Et Me O X O Et X

Et Me

O Me ka H

e
Me

Me O Ga - Ge = 1.5 kcal/mol O Me CHO Et Me ka


110 C

e
Et

(E)

H Me

(Z) a

a
Faulkner & Perrin (Tet. Lett. 2783 (1969) have made the correlation between G for rearrangement & G for the corrresponding cyclohexane# equilibria:

Me

The R2X interaction should destabilize a as X gets progressively larger. X


H Me MeO Me2N

p. p.
Me

H O H Me
#

Me H Me H O

(E):(Z) found
90:10 >99:1 >99:1 >98:2 Faulkner, Tet Let 1969, 3243 Faulkner, JACS 1973, 95, 553 Johnson, JACS 1970, 92, 741

G = +1.5 kcal/mol

G = +1.75 kcal/mol

Another comparison: (DAE) M. DiMare, Ph. D. Harvard University, 1988


OY X OPMB Et Me Et Me O R

Note: The A-value of 2-methyl-tetrahydropyran is +2.86 kcal/mol (Lecture No. 6)

They then suggest that there is a good correlation between cyclohexane "A-values" & G for the rearrangement process. Their case is fortified by the following expamples: CHO
110 C

O R2 R1

R2

CHO X

O OPMB Et Me

R2 R1 (E) R1 (Z)

R1
Me Me Et

R2
Et iPr Et

(E):(Z) found
90:10 93:07 90:10

(E):(Z) predicted
91:9 94:6 91:9 Faulkner, JACS 1973, 95, 553

procedure
Y = Ac, Ireland Y = H, Johnson Y = H, Eschenmoser

conditions
LDA, TMSCl HC(OMe)3, H+ MeC(OMe)2NMe2

X
TMSO MeO Me2N

T, C
-78+55 130 80

(E):(Z) ratio
97:3 94:6 97.5:2.5

15-03-Claisen-3 10/19/03 5:44 PM

D. A. Evans

Johnson & Eschenmoser Claisen Rearrangements


Johnson Orthoester Claisen Eschenmoser-Claisen

Chem 206

Lead paper: Johnson, Faulkner, Peterson, JACS 1970, 92, 741


OEt OH Me C OEt OEt
EtCO2H (cat)

Eschenmoser, A. Helv. Chem. Acta 1964, 47, 2425; Helv. Chim.Acta 1969, 52, 1030.
Xylene, 150oC

Me EtO

O OEt O O OEt

OH

Me Et2N

OMe OMe

O NEt2

O Et2N Me Me Et 92% (E:Z = 98:2) OH Me OMe


H
CH3C(NMe2)(OMe)2

O NEt2
H

Compare the two variants:


Me Me
CH3C(OEt)3

OEt

NMe2 Et O Me
Faulkner and Peterson

NMe2 Me

CH3CH2CO2H (cat)

EtO2C

Et O

p. p.

OH
60% Hg(OAc)2, EVE

138oC

Xylene, 110oC

High yield, E:Z = 99:1 Me O Me CO2Me 70%

Me

Me
98oC

Me OHC

Me

CO2Me Me Me OH
CH2=C(OMe)(NEt2) Xylene, 140 C, 14h
o

Me2N

98% (E:Z = 86:14)

The Saucy Marbet Alternative


OMe Me Me OH Me Me
H3PO4 or TsOH 125oC

Synthesis of Amide Acetals


Et O Me NMe2 Et2O Me OEt
NaOEt NMe2 EtOH +

Me O 94% Me Me

O Me

EtO Me

OEt NMe2

BF4-

+ O Et

Et

Me

Reactions to ponder:
C 3H 5 R Me OLi N Me O Me OLi N Predict the major diastereomer Me Welch, JACS 1987, 109, 6716 CMe2(OBn) O Me diastereoselection 6:1 Predict the major diastereomer Stevenson, Tet. Let. 1991, 32, 4199 Me

Me Me

OH

Me

OEt
H3PO4 125oC

Me Me

Me 60% Me Me

Me Me

CHO

+N Me

OMe

Me

Me + N

OMe Et

Saucy, Marbet, Helv. Chim. Acta 1967, 50, 2091,2095

15-04-Johnson/Eschenm 10/19/03 5:52 PM

CMe2(OBn)

D. A. Evans
Ireland-Enolate Claisen
Reviews

Ireland Enolate Claisen Rearrangement

Chem 206

Substituted enolates afford an additional stereocenter


R2 Me O OTBS

New Aspects of the Ireland and Re;ated Rearrangements, Tetrahedron 2002, 58, 29052928 (handout)

R1 R2 H

Me

R1 O

R2 Me

R1 O OTBS

Ireland, R. E.; Mueller, R. H.; Willard, A. K. J. Am. Chem. Soc. 1976, 98, 2868 OH
t1/2 (32oC) = 3.5 h

(E)

OTBS

O O Me
LDA Me3SiCl

OTMS O O

LDA, TBSCl LDA, TBSCl DMPU

R2 Me O

R1

Et O

conditions OEt LDA, TBSCl


LDA, TBSCl DMPU

(E):(Z) 94:6 7:93

control kinetic thermo

66%

Enolization: Amide Bases & Ireland Enollization Model


Stereoelectronic Requirements: -C-H bond must be able to overlap with CO

R2 Me

R1 R2 Me H

R1 O
H

R2 Me O OTBS

R1

O OTBS (Z)

p. p.

CO

Ha C O Hb
O Li N H R R

OTBS

R Hc

base Ha+
R

Hc R

Hb O
OLi R Me

key study: Ireland, JOC 1991, 56, 650 and earlier cited papers

Double Claisen Rearrangements are also possible


Paterson, Tet Lett 1991, 32, 7601 Me Me Me Me Me Me Me Me

favored O R Me LMNR2 disfavored R

Me

O Me H Li R H N R

(E) Geometry
OLi R Me

OH

OH

Me O

O O
LDA, TMSCl Et3N

Me

1,2 syn aldol relation permuted into 1,5 syn relationship via Claisen rearrangement

(Z) Geometry
Me RO Me Me Me Me Me OR O O O 63% yield (diastereoselection 86%)
20-60 C

Me Me O

Me

Me

Me Me O OTBS

The Ireland Model (JACS 1976, 98, 2868); Narula, Tetrahedron Lett. 1981, 22, 4119; more recent study: Ireland, JOC 1991, 56, 650 For a recent study on the effect of amide base structure on (E)/(Z) selectivity in the cont3ext of the Ireland enolization model see: JOC 1997, 62, 7516.

OTBS

15-05-Ireland/enolate Claisen 10/20/03 8:54 AM

D. A. Evans

Claisen Rearrangement & Chirality Transfer


Yamamoto, JOC 1993, 58, 5301
Me Me N Me Li Me TMSCl, THF in situ OMe 1) LiN(TMS)2 THF-HMPA 1) TBSCl OR t-BuMe2SiO t-BuMe2SiO OTBS OMe Et

Chem 206

Recent studies on controlling enolization condiltions have apeared

These Chelating substituents can be benzyl ethers as well


Kalmerton, Tet Lett 1993, 34, 1103

OTMS OMe

BnO

OMOM Me LN(TMS)2/TMSCl CH2N2 OPMB O

BnO

OMOM Me Me O O O LI PMB

(A)
O t-BuMe2SiO

(E)
Me O

(B)

(Z)
BnO O Me OSiR3 Me O Me O OTMS OMOM OPMB OMe BnO OMOM Me

(A)
O t-BuMe2SiO

H Et H

OTBS H O

Et

ratio 95:5
Et

OTBS O OSiR3

85% yield

OPMB

p. p.

(B)

H O Et R

A Problem to consider
OH R 2 LDA O O

Predict the stereochemical outcome of this reaction


OTMS O R Me OTMS

OTBS

ratio 99:1 OTBS

2 LDA, TMSCl Me

Chelating substituents on -carbon afford (Z)-enolates


Fujisawa, Tet Lett 1983, 24, 729
O HO O 2 LN(TMS)2/TMSCl Me CH2N2 OH H (H) Me TMSO (Me) H O HO O Me O Me O OMe

Li O

Li

O O Me

predict product if this is rearranging R species ratio 92:8 (84%)


Me3SiCl Me

OTMS Me O

OTMS O H

R TMSO H Me

favored

Me TMSO H

(E):(Z) ratio 93:7

OTMS Me O

OTMS O

OTMS Me O OMe OH

larger smaller

H R

OTMS O R TMSO H

disfavored Me
TMSO

TMSO

2 LN(TMS)2/TMSCl CH2N2

ratio 97:3 (79%)

Me

15-06-Ireland/enolat Claisen-2 10/19/03 5:54 PM

A(1,3) strain orients stereocenter relative size of carbon & oxygen substituents dictate relative face selectivity

Kurth, JOC 1985, 50, 1840

D. A. Evans

Applications of the Claisen Rearrangement

Chem 206

Johnson Squalene Synthesis: JACS 1970, 92, 741


Me Me Me Me Me Me Me Me

Faulkner Juvenile Hormone Synthesis: JACS 1973, 95, 553


Et Me O Et Me CO2Me

Cecropia Juvenile Hormone

Observations: Molecule contains an obvious symmetry plane The trisubstituted C=C's are the issue
OHC CHO Me OH Li Me Me EtO2C CO2Et Me Me LiAlH4 CrO3-pyr Li MeC(OEt)3 EtCO2H 138 C Me Me OH MeC(OEt)3 EtCO2H 138 C

HO Me

Et

Et

Me CO2Me

OH HO Me
1O

Et
3

Et

Me CO2Me
6

Et
6

Me CO2Me

p. p.

Et

OR

Me

87% (E)-(E) 97%


Me EtO2C

Me CO2Me

Me CO2Me H+ 110 C O Et
6

Me Me LiAlH4 CrO3-pyr Me CO2Et OH

Me CO2Me

OMe Et Et Et Me
6

85-90% (E) NaBH4


CO2Me H+ 110 C Et
6

Me OHC

Me Me Me Me Et Et
3

CHO

Me CO2Me OMe OMe

Me Me Me Me Me

PPh3 HO Me Me
1O

OR

Me Me CO2Me

OH Me

Et

Et Me CO2Me

OR

Me

Me

Me

Me

HO NaBH4 Me

Et

Et

Isomeric purity is Ca 95%

nonselective [H]

OH

TSCl/pyr MeONa

Cecropia Juvenile Hormone

15-07-Applications-1 10/19/03 5:54 PM

D. A. Evans

Claisen Rearrangement & Chirality Transfer

Chem 206

Chirality transfer via the Claisen rxn is an integral aspect of the general utility of process
R. K. Hill, Asymmetric Synthesis vol 3, Ch 8, p503 (chirality transfer)
OEt OH R2

Sense of Asymmetric induction may be controlled by olefin geometry


R2

OEt O R1 R2 R1

R1

MeC(OEt)3 H+

R2
H H

R1 O

R2

R1 CO2Et

MeC(OEt)3 H R1
+

O R2

OH Na/NH3 H2 Pd CaCO3 R2

OEt

R1

Such chirality permutation processes are only as stereoselective as the energy difference between diastereomeric chair transiltion states:
R2 R1 O
H H

OH

R2 O

R1

favored
R2

R1

MeC(OEt)3 H+

R1 O R2
H

R2

R1 CO2Et

R1

X X R2 R2 OH

OEt

p. p.

disfavored
R2

R1
H

Since stereoselection in reduction of acetylenes is >98%, either product accessible


R1 O X HO

Me

tocopherol (Vitamin E) Cohen JOC 1976, 41, 3497 JACS 1979, 101, 6710
Me O Me Me Me O H Me MgBr OH Me Me Me Me Me OH Me Me Me NaAlH2(OR)2 Me Me Me Me Me

Note that chirality transfer is coupled to olefin geometry in product. Prior arguments (Faulkner) imply that the X substituent will play significant role in promoting selectivity.
Me

Me Me MeC(OEt)3 H+ X Me O Me

O X CO2Me N Me OH Me Me

ee > 90% (E)-selection > 90% R. K. Hill JOC 1972, 37, 3737
OH

Me

Me

50%

50%

Me

Me

CO2Me MeC(OEt)3 H+ N Me

68% enantiomerically pure Uskokovic, JACS 1979, 101, 6742

Me Me OH Me

H2 Pd/CaCO3 Me

CO2Me OTMS Me Me MeC(OEt)3 H+ ETO2C

Me

(Z):(E) = 98:2
R O EtO

(E):(Z) = >99:1
O Me Me Me Me

92%

OH

77%

Me

Heathcock JOC 1988, 53, 1922

15-08-Chirality transfer 10/19/03 5:55 PM

diastereoselection ~ 99% from both routes

D. A. Evans

Claisen Rearrangement & Chirality Transfer


The analysis:
O H H H O

Chem 206
H

Boat transition states more accessible in Claisen than in Cope rearrangements


A case where the chair-boat preference depends on enol geometry

OR

Ireland, JACS 1991, 56, 3572 Factors controlling diastereoselection Enolate geometry Chair vs Boat transition states Bartlett, JOC 1981, 46, 3896 Ireland, JACS 1991, 56, 3572
H H Me O OR H H
OR

H Me

O OR

Me H

Boat (Z)-enolate

Chair (Z)-enolate

Me H

O O

LDA,TBSCl
O

A
O OR

O Me OR O

Boat (Z)-enolate Ireland study supports Bartlett's conclusions

Chair (Z)-enolate

Me Me

conditions
O O

A:B 29:71 86:14

TS (E)-boat (Z)-boat

LDA,THF LDA,TBSCl

p. p.
Me

A
O OR

B
O OR

LDA,THF DMPU
Me

H O H
OR

H O H

conditions LDA,THF LDA,THF DMPU

(E):(Z) 83:17 04:96

A:B 84:16 72:28

TS (E)-chair (Z)-boat

Me

H H Me

Me H

O OR

H H
OR

A further example:
OTBS Me

Chair (E)-enolate

Boat (E)-enolate

H O Me Me H O OR

Boat geometies can be favored in these and related systems

O OMe boat-preferred RO O Me OMe Me O H O Me

O O

OMe

Chair (E)-enolate

Boat (E)-enolate

Me

Me

In this case the boat geometry is preferred from either enol geometry
O O O

OMe Me O H H Me O OTBS

LDA,TBSCl
O

A
O OR

O Me OR O

?
H

Me

destabilizing
TBSO

conditions LDA,THF LDA,THF DMPU

A:B 29:71 86:14

TS (E)-boat (Z)-boat

Me

It appears that both of the indicated interactions contribute to the destabilization of chair geometry

15-09-Boat geometries 10/19/03 5:56 PM

D. A. Evans

Claisen Rearrangement & Chirality Transfer

Chem 206

In this case the chair geometry is preferred from either enol geometry
O O

The Claisen Rearrangement has been used in fragment coupling


CO2H Me CO2H Me CO2H Me
6

LDA,TBSCl
Me Me

A
O OR

B
Me OR HO O O

conditions LDA,THF LDA,THF DMPU

A:B 75:25 40:60

TS (E)-chair (Z)-chair

O O O H Me H RO O H Me OR

O O O H RO
1O 3

O RO O H Me O H O

R2

O R2 R1 OTBS

This destabilizing interaction has been attenuated


H TBSO

TBSO O O Me H

R1

favored

disfavored

Chlorothricolide

OR

[3,3]
CO2H

OR

p. p.

Ireland, J. Org. Chem. 1986, 51, 635 Ireland, J. Org. Chem. 1981, 46, 4863
CO2H Me H O OR O O TBSO R1 R2 O O R2 R1 OTBS CO2H H Me H O O RO O H OR O OR

In this case the boat geometry is preferred from either enol geometry
O O Me O

Me

O O O H Me H

LDA,TBSCl

A
O OR

O O OR OH RO O

Me

Me

conditions LDA,THF LDA,THF DMPU

A:B 43:57 80:20

TS (E)-boat (Z)-boat

In the initial approaches to the synthesis, the Schlosser-Wittig was unsuccessfully attempted for the fragment coupling process.
OR Me OR Me

OR

Summary:
O O Me O O O O O O

disfavored

favored
O

X = OH

O O KHMDS TESCl RO X O O H Me H O H Bu3SnH O

boat-preferred TS from either geometry


Me O O

O MeO O H Me H O H

X = SePh

Me

Me

60-72%
OMOM

decarboxylation
OMOM

chair-preferred TS from either geometry

boat/chair TS dependant on enol geometry

15-10-Boat geometries-2 10/19/03 5:57 PM

key paper for decarboxylation: Ireland, JACS 1985, 107, 3285

D. A. Evans

Ireland Claisen Rearrangements: Ionophore Synthesis


The relevant rearrangements:
Me Me RO

Chem 206

The Ireland lasalocid synthesis: JACS 1983, 105, 1988


Me OH HO CO2H Me Me Et O Me

A
H O Et

A
Et H O O O

Claisen

A
RO Et H HO O

B
Me

OH Et Claisen construction Me

OO

B
Me

OR Et

OR Et Me

Me O HO CO2H Me H

aldol
O

Me

A
Me Et H O Et

B
Me

Me OR Et Me

A
BnO

H
CO2H

A
BnO Et H O Cl OH

O BnO Et H

A
O O O

Et O

B
OMOM Me

Me

Note that neither retron for Claisen exists in this intermediate

LDA THF/HMPA TMSCl

p. p.
RO Et

A
H O O O

add Claisen retrons


O

B
OMOM Me

50% overall yield

Claisen
RO

Me

B
OMOM Me

A
Et H O HO

B
Me

OR Et

OO

B
Me

OR Et

The Ireland monensin synthesis:


Claisen construction HO Me MeO O Me HO Me HO Me Me Me

Me OH

enolate face selectivity is anticipated problem. This results in CO2H epimers

A
O

B
O Me H

C
O Et H

D
O O

E
HO

Me CH2OH

A
RO Et H O COOR O

B
Me

OR Et

MeCOOH JACS 1993, 115, 7152-65 JACS 1993, 115, 7166-72 (and previously cited papers) A
O Me

Here is another potential Claisen construction


Me O

Methyl-bearing stereocenter to be established by selective reduction A


H Et Et O Me OR Et O

X Et

B
O

O O Me Me

B
Me

A
O

R Et

O RO

Me O

MeO

A
Et H O

R R Et Me

Claisen construction

C
O Et H

D
O O

E
HO

Me CH2OH

15-11-lasalocid 10/19/03 5:59 PM

D. A. Evans

Ireland Claisen Rearrangements: Prostaglandin Synthesis


Consider the prostaglandin nucleus

Chem 206

Prostaglandin A2 Synthesis, G. Stork, JACS 1976, 98, 1583


O COOH Me H
8

O COOH Me

O COOH Me
15

H COOH H OH Me

Aldol

Me O H H OH

MeO2C

HO

OH

OH
1O

Claisen retrons
O RO OR COOH Me H OH O OR H

Claisen retrons
H

C-8 center can be controlled by equilibration Claisen (chirality transfer)


1 OR

Dieckmann
H
3

MeO

COOH Me

COOH Me O OH RO
1O 3 5 3

MeO2C COOH Me

H 5

OH

OR

p. p.
O O

Claisen
OR COOH Me OH O HO HO OH COOR Me HO HO O O O OR OR

Claisen Claisen (trans C=C)


COOH OR Me O OH O COOH OR Me HO O O CONH2 O O M OH MeO2C(CH2)2 Me Me O Me MgBr Cl-CO2Me O Me MeO2C OH O Me OCO2Me MeC(OMe)3 H
+

HOOC COOH

COOR OH OH OR C4H9

OR

Me

Unrealized plan to generate the required enolate


OMe CONH2 O O O Me OMe

O Me O Me

OCO2Me H3O+ Et3N

83%

Li(0)

via erythrose
(MeO)3C OH O O O (CH2)3CO2Me H+ 160 C, 1h MeO

O (CH2)3CO2Me
12

An Application PGA1: Ireland JOC 1976, 41, 986; Aldrichimica Acta 1988, 21, 59

MeO2C

H O O O

15-12-PG applications 10/19/03 6:00 PM

C12 center and 13 C=C set in this rxn

59%

D. A. Evans

Ireland Claisen Rearrangements: Cyclic Enolates


OR R1 R3 R1 O R1 O Me OR Me H CO2H R O O R3 R O equivalent representations O O OR R3 Me H CO2H Me O Et Me H R3 Me Ph3C
+

Chem 206

The previous cases were derived from a connection between R2 &R3


OR O R2

The Indanomycin Synthesis, Burke, Tet. Lett. 1985, 26, 1163; ibid, 1986, 27, 6295
O O N H H N H H

X Me

O Et H Me Et H CO2H O

H O

Et

Consider consequence of connecting R1 &R2


R R O R1 R3 O O

"Right wing"

Et

The Left Wing: Tet. Lett. 1985, 26, 1163


Me

"Left wing"
Me Me

O R2

add Claisen retrons


Me H CO2H O

Claisen
O OR Me

p. p.

"Left wing"
O O H LDA/TMSCl Me H BnO

H CO2H

Examples:
Me MeO TBSO O

R3 Me H MeO2C Me Me CHO OM TMSO H O XCO X Me LDA/TMSCl O H O H OTMS Me O H TMSO H O O Me

OTMS 110 C 4h

Me Me H BnO O OTMS

H H BnO

80% yield

MeO2C

-78 C

The Right Wing: Tet. Lett. 1986, 27, 6295


O Y H TMSO H

Danishefsky, Tet. Let 29, 1371 (1988)


TMSO2C Me H Me Me TMSO H

[4+2]

Claisen
H

O H Et

H H

Et

Et O

CH2X OTMS H

OTMS

135 C 4h H Et

TMSO2C

Me

Et

65% yield

see also Danishefsky, JACS 1980, 102, 6889, 6891

Et

The "apparent" Claisen process is more complicated than anticipated.

15-13-cyclic enolates 10/19/03 6:00 PM

D. A. Evans The FK 506 application

Ireland Claisen Rearrangements: Cyclic Enolates


HO

Chem 206
R R1 R3 O R O

Now connect R1 &R3


H
26

MeO

Me
24 22 O

Schreiber Synthesis JACS 1990,112, 5583

O R2

O R2 R2

Me N O H O O
17

OH
19

The Schreiber Route:

Examples: Funk, JACS 1982, 104, 4030


Me O H O LDA/TMSCl TMSO H O O O O O LDA/TMSCl TMSO O H H O TMSO H

O Me

OH

O
14

Me OMe

FK 506

Claisen disconnection
R3SiO MeO
29

70%

OMe OR O

MeO

29

OR O MeO

81%
H

p. p.
O O TBSOTf Et3N MeO H

O O LDA/TMSCl CHMe2

H O 110 C OTBS MeO


29

TMSO O

OR O

89%
H

MeO

71%

Me2HC

The previous cases were derived from a connection between R2 &R3


OR O R2 R1 R3 R1 O OR R1 O OR

Recent improvments: Funk, JACS 1993, 115, 8847


O O O R1 R2 R3 LDA ClPO(OEt)2 R3 O O P(OEt)2 R1 R2 R3 O O O P(OEt)2 R1 R2

and R1 &R2
R O R1 O R2 R3 R3 R O

chrysanthemic acid application


O O Me OR Me R3 Me O LDA Me ClPO(OEt)2 Me O Me Me Me Me XO H H 65 C TMSO O H Me

83%
Me H Me

X = TMS; T1/2 = 58 min

15-14-cyclic enolates-2 10/19/03 6:01 PM

X = PO(OEt)2; T1/2 = 10 min

D. A. Evans

Claisen Rearrangements: Acyclic Cases with Stereoinduction


Exocyclic Stereochemical Issues
O O R2 Me

Chem 206

N-Allylketene-N-O-Acetals: Kurth, JACS 1985, 107, 443


R1 OR Me Br R' O R N O alkylate base N O R 185 C R O N O R' R' R' OH

R1 R2 Me

R1

OR O R2

These rearrangements present many of the same issues which were encountered during our discussion of carbonyl addition with regrad to assymmetric induction.

Chelate Control: -Hydroxy ester enolates: Kurth, JOC 1985, 50, 1840
OH Me OH O O 2 LDA, TMSCl H Me H H Me O O R 2 LDA Me O O Li O Li R El(+) Me El OH Me Me H O OTMS Me Me2HC OH O O R N Me O

-Hydroxy ester enolates: Kurth, JOC 1985, 50, 5769


Me2HC alkylate base Br N O 185 C Me2HC O N O OH

chelate control
Me O Li

R' Me Me

p. p.

Me

O O

diastereoselection 87:13
Me2HC N O Me2HC N H Me Me O

chair TS

diastereoselection 81:19

We again see the consequence of chelate-organized asymmetric induction

185 C

diastereoselection 94:6

Felkin Control ?: Cha, Tet. Lett. 1984, 25, 5263


Me O Me Me O OTMS O Me O Me O OMe OTMS O THPO Me O OEt 110 C Me O 25 C Me O 25 C Me Me O OMe OTMS O THPO OEt O O Me O OTMS O Me O Me O OTMS O Me O OMe OTMS O O

57%
Me Me

The Claisen Rearrangement is subject to acid catalysis


LA LA + O LA + O LA O

57:43

81:19

BF3-HOAc: Bryusova, J. Gen. Chem. (USSR) 1941, 11, 722 BCl3: Gerrard, Proc. Chem. Soc. 1957, 19; Schmid, Helv. Chem. Acta 1973. 56, 14 Et2AlCl: Sonnenberg, J. Org. Chem. 1970, 35, 3166 TiCl4: Mukaiyama, Chem. Lett. 1975, 35, 1041 (RO)2AlMe: Yamamoto, JACS. 1988, 110, 7922 (RO)2AlMe: Yamamoto, Tet. Lett. 1989, 30, 1265 (RO)2AlMe: Yamamoto, JACS. 1990, 112, 316 LiClO4: Reetz, Tetrahedron. 1993, 49, 6025

75:25 (Felkin model, RO = large)

Takano, Tet. Lett. 1985, 26, 865

15-15-Acyclic/Felkin 10/19/03 6:01 PM

D. A. Evans

Lewis Acid Catalyzed Claisen Rearrangements

Chem 206

Catalyzed Claisen Rearrangement of Allyl-phenyl Ethers


(RO)2AlMe: Yamamoto, Tet. Lett. 1990, 31, 377
CMe3 Me3C O Al CMe3 Me Me3C O Me Me heat Me R Me "high yield" Br

Catalyzed Claisen Rearrangement of Allyl-vinyl Ethers


(RO)2AlMe: Yamamoto, JACS. 1990, 112, 316

The thermal process


favored R H O X R R OH O X disfavored H O O X R X O X

The Lewis Acid Br (LA)

Me

LA

Me OH Me Me Me

The catalyzed process


disfavored R H Al O H R R R favored H O Al O H H O H

p. p.
Me

LA + O

+ LA
Me R O H

Me -78 C

B
Me Me O

nonbonding interactions favor this TS


R

+ LA
Me Claisen Cope Me

A:B = 3:4
Me R O H O H

Me O

Me OH Me iBu Claisen Cope Me O Me TMS O H H O H

43%

19:81

LA

40% 85%

07:93

94%

03:97

The hindered Lewis acid will alter the partitioning of the Claisen process to the two ortho positions

15-16-catalysis-1 10/19/03 6:02 PM

D. A. Evans

Lewis Acid Catalyzed Claisen Rearrangements


Yamamoto, JACS. 1990, 112, 7791 Yamamoto, Tet. Asymmetry 1991, 2, 647-662
Si(t-Bu)Ph2 O Al O Si(t-Bu)Ph2 X R Y Me

Chem 206

Chiral Lewis Acid Promoted Claisen Rearrangements

Allylic rearrangements may be included as a subset of other sigmatropic processes:


R1
1 3

R2

R1

R2

Note that these reactions are not catalytic


R1

X R2 R3

Y R2

[3, 3]

R1 X Y

R3

(R)-1 (R)-1 (1.1-2 equiv.)

X = C, N, O, S

O Ph

SiMe3

CH2Cl2, -20C 76-95%

O R

SiMe3

R Ph C6H11

% ee 88% 71%

X = O; Y = S Faulkner, Synthesis 1971 175 (see pg 183)


Me Et Me Me Et O S OPh O S OPh

pyr
OH Cl S OPh

Et

p. p.

(R)-1 (1.1-2 equiv.)


O R

CH2Cl2, -40C 68-76%

R SiMe2Ph GeMe3

% ee 90% 93%

(E) selectivity: 96.5%

Enantioselective Claisen Rearrangements: Metal-Centered Chirality


Corey, JACS. 1991, 113, 4026
Ph ArO2S N B Br O L2BBr i-Pr2NEt CH2Cl2 Me Me L2BBr Et3N toluene O Me Me O BL2 O OH Me Me 20 C O BL2 OH Me Me 20 C O Me Me Ph N SO2Ar CF3

A stereochemically related case Johnson, JACS 1970 92, 735


Me R SOCl2 OH R O Cl Me R O Cl S O Me

Ar =
CF3

(LA)

ether

S O

(E) selectivity: >98%

X = O; Y = N
Threo: erythro 99:1 >97% ee (75%)
R R 200 C O H R 200 C Ph O N ONa O Ph ONa N Ph R N O Ph H R N N Ph

O O

Roberts, JACS 1960 82, 1950 Hill, JOC 1968 33, 1111
R

Threo: erythro 10:90 96% ee (65%) 8 cases reported

O O

H N

H3O+ H

Ph

15-17-catalysis-2 10/19/03 6:02 PM

D. A. Evans
R2 R1 X Y R3 R2

Allylic Rearrangements via [3,3] Processes


Me R3 Me O OTBS 138 C Me TBSO HN O CF3 20 h TBSO Me O Me O

Chem 206

[3, 3]

R1 X Y

X = C, N, O, S

OTBS Me HN O CF3

X = O; Y = N:
The Trichloroacetimidate Rearrangement, Overman, JACS 1974, 96, 597
R O Cl3C H 3O CN
+

90% as a single isomer

steps

Thomas, Chem. Commun 1989 717


O OH CH2OH N HO OH NH2 O OH

R heat O N CCl3 O H CCl3 N

H RO OH

OH

O N H

HN CO2H O

This reaction is also catalyzed by Hg(II) ion


C 3H7 C 3H7 HN OH CCl3 O Hg(OTFA)2 THF, 0 C (or 138 C, 9 h) R Me C3H 7

NH2

79%
HN O CCl3

OH OH

The polyoxins (antifungal antibiotics)

p. p.

The "Burgess Reagent" is normally used for alcohol dehydration


O O N S O O O MeO N S

however, rearrangement is obsteved with allylic alcohols


+
NEt3 Me Et OM MeO2C N Me O S O MeO2C Et Me N Et

Hg(OTFA)2 Me Me Me OH THF, 0 C (or 138 C, 6 h)

79%
HN O CCl3

MeO

For an excellent review see: Overman, Angew. Chem. Int. Ed. 1984, 23, 579
NHBoc Me HN O CCl3 Me O heat BnO HN O CCl3 BnO Me O PdCl2(MeCN)2 THF, 25 C, 3h Me HN O CCl3 R NHBoc

+ NEt3
O

OM

E. Burgess, JACS 1970, 92, 5224

99% anti (45-62%)

A new approach to the synthesis of -amino acids


O OLi N O R' O Me LDA R N N O R' OLi O R N H H Me H 3O
+

OLi Me R N N O OLi Me

Me O

Me O

Gonda, Synthesis 1993 729

R'HN

a 1:1 mixture was obtained


HN O CCl3

Endo, SynLett 1991 649 This may not be a concerted process


R' N

Saksena, JOC 1986 51, 5024

moderate yields

15-18-ALLYLIC rearr 10/19/03 6:03 PM

D. A. Evans

Cycloaddition Reactions: Diels-Alder Reaction


The Diels-Alder Cycloaddition Reactions

Chem 206

http://www.courses.fas.harvard.edu/~chem206/

Chemistry 206 Advanced Organic Chemistry

"Diels-Alder Reactions". Evans, D. A.; Johnson J. S. In Comprehensive Asymmetric Catalysis, Jacobsen, E. N.; Pfaltz, A.; and Yamamoto, H. Editors; Springer Verlag: Heidelberg, 1999; Vol III, 1178-1235 (electronic handout)

The Diels-Alder Reaction in Total Synthesis, K. C. Nicolaou, Angew Chem. Int. Ed. 2002, 41, 1668-1698 (electronic handout) Catalytic Enantioselective DielsAlder Reactions: Methods, Mechanistic Fundamentals, Pathways, and Applications, E. J. Corey, Angew Chem. Int. Ed. 2002, 41, 1650-1667 (electronic handout) Chemistry and Biology of Biosynthetic DielsAlder Reactions Emily M. Stocking and Robert M. Williams, Angew Chem. Int. Ed. 2003, 42, 3078-3115 (electronic handout)

Lecture Number 16

Cycloaddition Reactions-1
Cycloadditions: Introduction Ketene Cycloadditions The Diels-Alder Reaction

Reading Assignment for week: Carey & Sundberg: Part A; Chapter 11 Concerted Pericyclic Reactions Carey & Sundberg: Part B; Chapter 6 Cycloadditions, Unimolecular Rearrangements Thermal Eliminations Fleming: Chapter 4 Thermal Pericyclic Reactions
Wednesday, October 22, 2003
X CHO

Problem of the Day:


Rationalize the sense of asymmetric induction for this Diels-Alder Reaction reported by MacMillan, JACS, 2000, 122, 4243. (pdf)
O catalyst PhCH2 X N Me Me

N H Me

D. A. Evans

+
R

5% catalyst MeOH-H2O

CHO R

16-00-Cover Page 10/22/03 11:23 AM

D. A. Evans

Cycloaddition Reactions-1
Consider [2 + 2] cycloaddition: Photochemical activation
new HOMO LUMO

Chem 206
[ 2s

Why does maleic anhydride react easily with 1,3-butadiene, but not with ethylene? So what are the "rules"?
O O O O O

+ 2s]

*
light

C C
bonding

X
[2+2]

O O

[4+2]
O O

bonding

The related reaction of 2 ethylenes is nonconcerted: [2 + 2] cycloaddition


Y Y Y

light

C C C
+

C
concerted

HOMO

heat
X


X X

C C

C C

C C

C C

+ energy

We also know that the photochemical variant is concerted p. . The frontier orbitals of the reacting species must have the proper symmetries Nomenclature suprafacial
C C

[2+2] Cycloaddition - Examples

2s

antarafacial
C

2a

Quadricyclane Dauben, Tet. 1961, 15, 197.

[2s + 2s]

Using this nomenclature, the Diels-Alder reaction is a 4s

+ 2s cycloaddition + 2s]
Me bonding Me Me

Me Me Me h Me Me Me

Me

Consider [2 + 2] cycloaddition: Thermal activation [ 2s

[2s + 2s]

Me Me

Schfer, AC 1967, 79, 54.

2s
bonding

C
antibonding

2s

C C C C

Dewarbenzene-Derivative

Prismane-Der.

2a
[2s + 2a]

2s
[ 2s

bonding

+ 2s] "forbidden"

[ 2s

+ 2a] "allowed"
must be antarafical for indicated stereochem

TL 1967, 4357, 4723.

16-01-Cycloaddition intro-1 10/22/03 6:15 AM

D. A. Evans
Summary of Ketene Cycloadditions
R R' R O R R' O
Carbonyl Alkene

Cycloaddition Reactions-2
O O O O O Ph R' R R' O
550 C _

Chem 206
O O O Br Zn _ ZnBr Ph2C C O
2

O
or

R R' R

C O H

R'

Ph Br

O R1 O R3 R O N R H R N2 O -CH2CH2 R O R'
h or
2

H 2C C O R1 O R3 R2 R C O R' R C O H

O C
1,3-Dipole

R'

p. .

Imine

h or

O R R'

X Y Z X
R = -CH=CH2

R'

Y O X Y O R R' R'

FMO Analysis Ketene Preparation


O R R'
X = Cl, Ts, AcO, DCC, etc... R3N

R C O R'
Staudinger Reaction (very general)

HOMO
[2s2a]
O

H
H 2C C O O
_ +

(CH3CO)2O
_ RCH2CO2Ar

AcOH -ArO
_

H
R C O

O LUMO

OH

_H O 2

OAr

16-02-Cycloaddition intro-2 10/22/03 8:16 AM

D. A. Evans
[2+2]: Stepwise Versus Concerted

Cycloaddition Reactions-3
Ketene-Alkene [2+2]
O Me
+

Chem 206

H H C R' H C

R O R

H H R' H

R C R O

O
Fast

R'

O Me

C Me O C

Me

Me

Me Me Me Me O

Me
+

least hindered bond rotation

Me
+

Me Me Me

Me Me Me

Stepwise
Very large polar effects E olefins yield a mixture of cis and trans products Solvent effects observed, but it could merely be a ground state effect KIE seen for many reactions support stepwise mechanism Calculations (Wang and Houk) show a highly asynchronus transition state in the gas phase reaction All stereochemical outcomes can be rationalized assuming a stepwise mechanism

Concerted
Ketenes add stereoselectively to Z alkenes Z olefins are much more reactive than E

Me

Me

Me

B 1:2 H C

Me
+ Me

H Me Me C

Me

Me Me

Me

O_ C Me + H

O_ C H + Me

Me Me

p. .

C O

Frey, H. M.; Isaacs, N. J. J. Chem Soc. B, 1970, 830-832.

Ketenes + Aldehydes Afford -Lactones Solvent Effects


+

Me

O
+

ab initio Calulations

O H H

O H H

Me C O X
X Cl Cl Cl Cl Solvent hexane Et3N CHCl3 CH3CN endo / exo 4.3 / 1 2.2 / 1 1.6 / 1 0.59 / 1

X
endo X Br Br Br

Me
exo Solvent hexane Et3N CH3CN endo / exo 0.71 / 1 0.28 / 1 0.14 / 1 +

O C H O H H H

path A

38 kcal/mol

O O
32 kcal/mol path B

O O H H H H

Solvent effects implicate a zwitterionic intermediate Brady,et. al, JACS 1970, 92, 146-148.

Pons, J. -M.; et. al. JACS 1997, 119, 3333.

Mechanism and Origin of Stereoselectivity in Lewis Acid Catalyzed [2 2] Cycloadditions of Ketenes with Aldehydes, Singleton, Angew. Chem. Int. Ed. 2002, 41, 1572

16-03-Cycloaddition intro-3 10/22/03 11:29 AM

D. A. Evans
Transformations of -Lactones
R2 R1 O O O O R2 R1
+S _

Cycloaddition Reactions-4
The Staudinger Reaction

Chem 206

or BF3 -CO2

Me2S

O O

In this process, the illustrated ketene, generated in situ from an acid chloride, undergoes reaction with the indicated substrates to form -lactams in a stereoselective process. When the azo-methine (RN=CHR) geometry in the reactant is (Z) the product stereochemistry is trans (eq 1). In a complementary fashion, the (E) imine affords the cis-substituted product (eq 2). While this transformatlion could be viewed as a [2s+2a] cycloaddition, it is felt that this reaction is stepwise. H H H R S S R Et3N O Cl R C O H R N N O R R R H H R
(2)

N O

(1)

O R2N O

CuCN R'Li (2eq)

R' R2N CO2H

Vederas et al JACS 1987, 107, 4649.

p. .
R2 O O OH
base or acid

The stepwise mechanism


O Nu R2 R C O H R

R1

R1

H N

H H conrotatory R S closure N O

(1)

Nu = OH2, ROH, R2NH

enantiomers
R

MeO

Application in Natural Product Synthesis: Ginkolide B, E. J. CoreyJACS 1988, 110, 649. O O CMe3
1. (COCl)2, PhH, 2. NBu3, toluene,

Ther are two contortaory modes. If you control the conrotatory mode, you control the absolute stereochemistry of the reaction: O CMe3 O N Bn Ph O Cl H N Ar

conrotatory R closure O

H H S N

O Et3N O N Ph O H H N O Ar + Bn Ph

O N O H H N Ar Bn

H O H

CO2H

Evans, SjogrenTet. Lett. 1985, 26, 3783, 3787.

H CH2COCl

NEt3 O C H O
JOC 1982, 47, 3470.

See also Evans, Williams, Tet. Lett. 1988, 29, 5065.

diastereoselection > 95:5 80-90% yields

"[2+2] photocycloaddition/fragmentation strategies for the synthesis of natural and unnatural products.", Winkler, J. D.; Bowen, C. M.; Liotta, F. Chem. Rev. 1995, 95, 2003. "Stereoselective intermolecular [2+2]-photocycloaddition reactions and their application in synthesis.", Bach, T. Synthesis 1998, 683.

16-04-Cycloaddition intro-4 10/22/03 8:24 AM

D. A. Evans

Cycloaddition Reactions-4
Me
1)

Chem 206
Me O N

Enantioselective Ketene-Aldehyde Cycloaddiitons


O O
+

+ _

O Br H
R3N

O
catalyst (10 mol%)

O R O C + Me3Si H H O

Me

R O C

i-Pr2NEt

[RCHO cat.]

N OTf Cu Me3C H2O OH2 CMe3 OTf PhMe2Si OEt 1 mol%, THF, 3 MS
-78 C, 24 h

O O

R3NHBr

CH2

EtO2C
77% yield, 93% ee

i-Pr N F3CO2S

Bn N Al R N

i-Pr SO2CF3
cat. = 5a: R = Me 5b: R = Cl

PhMe2Si O EtO2C
% ee 3 (configuration)

KF, CH3CN O EtO2C

p. .
entry a b Aldehyde 2 (R)

3: >99% yield, 92% ee catalyst [time (h), temp (C)] 5b (8, -40) 5a (16, -50) 5a (72, -78) 5b (16, -50) 5a (24, -50) 5b (16, -40) 5b (16, -40) 5a (16, -50) 5a (16, -50) 5b (24, -40) % yield 91 93 89 91 80 90 74 86 91 56
2+

Me BnOCH2 PhCH2CH2
PhCH2CH2 c d e f g h i CH2CH(CH2)8 Me2CHCH2 BnOCH2CH2 TBDPSOCH2 92 (R) 92 (S) 95 (S) 91 (S) 93 (S) 91 (S) 89 (R)

O H N N Cu O R RO OR2 R O H C C Me3Si
1

Me

Me3Si
observed product
2

O O O

R1 R O

Me Me
93 (R) 85 (R) 54 (R)

2+

BnOCH2 Me3C
C6H11

O N Cu Me3C H2 O N

OH2

CMe3

Nelson, S. G.; Peelen, T. J.; Wan, Z. JACS, 1999, 121, 9742-9743

+ 2 CF3SO3 with J. Janey, Org. Lett. 2001, 3, 2125-2128

X-ray

16-05-Cycloaddition intro-5 10/22/03 9:14 AM

D. A. Evans

The Diels-Alder Reaction


Articles and monographs of Significance
The Reaction: +

Chem 206

Comprehensive Organic Synthesis, Vol. 5, Trost, Ed. 1991


4.1 Intermolecular Diels-Alder Reactions, W. Oppolzer 4.2 Heterodienophile Additions to Dienes, S. M. Weinreb 4.3 Heterodiene Additions, D. L. Boger 4.4 Intramolecular Diels-Alder Reactions, W. R. Roush 4.5 Retrogade Diels-Alder Reactions, R. W. Sweger, A. W. Czarnik

Representative natural products displaying the Diels-Alder retron: These natural products could well have incorporated the DA rxn into the biosynthesis
HO O O O H O H H H H Me NMe2 H Et O O Me R H H O

The Diels-Alder Reaction in Total Synthesis, K. C. Nicolaou, Angew Chem. Int. Ed. 2002, 41, 1668-1698 (handout)
Catalytic Enantioselective DielsAlder Reactions: Methods, Mechanistic Fundamentals, Pathways, and Applications, E. J. Corey, Angew Chem. Int. Ed. 2002, 41, 1650-1667 (handout)

O Et O

Hetero Diels-Alder Methodology in Organic Synthesis Boger, D.L. and Weinreb, S.N., Academic Press, 1987

Me

p. .

Natural Products Synthesis Through Pericyclic Reactions Desimoni, Tacconi, Barco, Polini, ACS Monograph 180, 1983, Chapter 5,

Compactin: R = H Mevinolin: R = Me

Me

Asymmetric Diels-Alder Reactions with Chiral Enoates as Dienophiles Modern Synthetic Methods 1986, Scheffold, Ed. Springer-Verlag,
Intramolecular Diels-Alder and Alder Ene Rxns, D. F. Taber, Springer-Verlag, 1984 Synthetic Aspects of D-A Cycloadditions with Heterodienophiles Weinreb, Tetrahedron, 1982, 38, 3087-3128 The Intramolecular DA Rxn: recent advances and synthetic applications Fallis, Can. J. Chem., 1984, 62, 183-234 Intramolecular [4 +2] & [3 + 2] Cycloadditions in Organic Synthesis Oppolzer, Angew. Chem. Int. Ed., 1977, 16, 10-23 Preparation & DA Reactions of Heterosubstituted 1, 3-Dienes Petrzilka, Synthesis, 1981, 753-786 DA Reactions of Azadienes Boger, Tetrahedron, 1983, 39, 2869-2939 Silyloxydienes in Organic Synthesis Danishefsky, Acct. Chem. Res., 1981, 14, 400-406 DA Reactions Part I: New Preparative Aspects Sauer, Angew. Chem. Int. Ed., 1966, 5, 211-230 DA Reactions Part II: The Reaction Mechanism Sauer, Angew. Chem. Int. Ed., 1967, 6, 16-33 Mechanistic Aspects of Diels-Alder Reactions: A Critical Survey Sauer, Angew. Chem. Int. Ed., 1980, 19, 779-807
Me O COOH H Me

Lepicidin

(Synthesis) JACS, 1993, 115, 4497

(Biosynthesis) JACS 1985, 107, 3694 Clive, JACS 1988, 110, 6914 Kozikowski, JOC 1987, 52, 3541 O H Keck, JOC 1986, 51, 2487 H Me O Grieco, JACS 1986, 108, 5908 H OMe H Heathcock, JACS 1985, 107, 3731 MeO H OMe Girotra, Tet. Let. 1983, 24, 3687 Hirama, JACS 1982, 104, 4251
HN O H H HO2C H

H H H Ph

Et H Ph Et
H H H H

Endiandric Acid C

X-14547A

Roush JOC 1984, 49, 3429 Nicolaou JOC 1985, 50, 1440 Ley Chem. Commun. 1983, 630

CO2H
H

Endiandric Acid B
(Syntheses) Nicolaou, JACS 1982, 104, 5555-5562

16-06-Diels-Alder intro 10/22/03 6:47 AM

D. A. Evans
The Alder Endo Rule

Diels-Alder Reaction-Orbital Symmetry Considerations


The following observation illustrates an example of the Alder Rule which will be defined below.

Chem 206

Orbital Symmetry Considerations for Diels Alder Reaction


If the symmetries of the frontier MO's of reacting partners are "properly matched" the reaction is referred to as "symmetry-allowed". The Diels-Alder reaction is such a case. As illustrated, the HOMO and LUMO of both the diene and dienophile, which in this case are the same, will constructively overlap as indicated in formation of both sigma bonds.

H + H H
disfavored favored

"Exo product"

"Endo product"

C C C

HOMO-2

C C C

LUMO-3

Observation: The endo Diels-Alder adduct is formed faster even though the exo product is more stable. There is thus some special stabilization in the transition state leading to the endo product which is lacking the exo transition state. Exo TS Endo TS

C C

C C

LUMO-3

C C

C C

HOMO-2

p. .
Energy Primary orbital overlap leads directly to the formation of new chemical bonds.

2
H

Frontier MO Explanation for the Endo Rule


C Secondary (transient) orbital overlap can also occcur in the stabilization of certain transition state geometries. Such a transient stabilizing interaction can occur in the endo, but not exo, transition state: C C C C HOMO-2 C C LUMO-3

Of the two possible transition states, the one having the "greatest accumulation of interacting double bonds will be preferred" (the Alder Endo Rule). Secondary orbital overlap is noted below.

The Other Dimerization Possibility for Butadiene


Does the possibility for the following concerted dimerization exist?

Secondary orbital overlap

Note that the termini only match at one end for the HOMO-LUMO pairing. Hence we say that the symmetry C requirements for the reaction in question are not met. This does not mean that the reaction will not occur, only that the reaction will not be concerted. Such reactions are called "symmetry-forbidden".

C C

C HOMO-2

C C C

C LUMO-3

Exo TS

Endo TS

16-07-Diels-Alder intro-2 10/22/03 6:48 AM

Additional Reading: Lowry & Richardson, Chapter 10, theory of Pericyclic Rxns pp 839-900

D. A. Evans

Diels-Alder Reaction: The Transition Structure


Lewis Acid Catalysis of the reaction is possible:

Chem 206
Yates & Eaton, JACS 1960, 82, 4436

Transition State Modelling is Coming of Age


+

LUMO1

LUMO2

The lengths of the forming CC bonds are Ca. 1.5 times the normal bond distance. This factor comes out of the ab initio work of Jorgensen & Houk leading references:
Jorgensen, JACS 1993, 115, 2936-2942 Houk, Jorgensen, JACS 1989, 111, 9172

energy

The Critical Energy Difference:


HOMO1 HOMO2

E(LUMO1) - E(HOMO2) or E(LUMO2) - E(HOMO1)


Dienophile

Transition Structures of Hydrocarbon Pericyclic Reactions Houk Angew. chem. Int. Ed. 1992, 31, 682-708

Diene

Bond formation is not synchronus with substituted dienophiles (Jorgensen) p. .

The closer the two orbitals are in energy, the better they interact As E decreases for the relevant ground state FMOs, rxn rates increase

Ethylene & Butadiene


+

Vs Butadiene & Acrolein


O
+

O H H

2.193

2.193

2.091

2.325
LUMO2 LUMO1

exptl

H = 22.5 kcal/mol S = -32.1 eu rel rate = 1

H = 14.0 kcal/mol S = -38.3 eu rel rate = 10+5

E
HOMO1 HOMO2

LUMO3

Diene Reactivity as measured against Maleic anhydride


Me Me

HOMO3

E (LUMO3-HOMO1) < E (LUMO2-HOMO1)


log k = 2.12 log k = 1.83

Rate Acceleration

log k = 4.96

log k = 2.36 log k = 2.19

Sauer, Angew. Chem. Int. Ed., 1980, 19, 779-807

Lewis acid catalysis not only dramatically increases rates by ca 10+6 it also improves reaction regiochemistry & endo diastereoselectivity

16-08-DA transtion structure 10/22/03 6:52 AM

D. A. Evans
Orientation of Reacting Partners
CO2H CO2H CO2H

Diels-Alder Reaction: Regiochemistry

Chem 206

Here is an interesting problem in reaction design


CO2H CO2H RO CO2H COX RO favored disfavored COX RO COX

favored

disfavored However, what if you need the disfavored product?


PhS
MgBr2

4.5 : 01 @ 100 C Lewis acid catalysis improves orientation


COX Me Me favored disfavored COX Me COX

Ni(Raney)

PhS AcO disfavored

COMe PhS AcO favored COMe

AcO

COMe

Trost, JACS 1980, 102, 3554

p. .

toluene, 120 C C6H6, SnCl4, 25 C

59 : 41 96 : 04

By employing a removable substituent, it is possible to access the normally disfavored product diastereomer
O2N RO CO2Me RO NO2 CO2Me

In general, 1-substituted dienes are more regioselective than their 2-substituted counterparts: Sauer, Angew. Chem. Int. Ed., 1967, 6, 16-33 Lewis acid catalysis improves endo diastereoselection

Danishefsky, JACS 1978, 100, 2918: The NO2 FG completely dominates directivity It then can be removed by elimination NO2 RO CO2Me NO2 RO CO2Me O
83% base NO2

RO

CO2Me favored CH2Cl2, 0 C C6H6, SnCl4, 25 C

H CO2Me H disfavored 80 : 20 95 : 05

CO2Me or by reduction Ono, Tet. 1985, 4013

CO2Me

R3SnH RO O
R3SnH 86%

CO2Me

DA Reactions Part II: The Reaction Mechanism, Sauer, Angew. Chem. Int. Ed., 1967, 6, 16-33

NO2

Me

O2N

Me

Me

16-09-DA regiochem 10/22/03 6:58 AM

Ono, Chem. Commun. 1982, 33-34

mixture of ring-fusion isomers

D. A. Evans

Diels-Alder Reaction: Regiochemistry


MeOCH2 CH2OMe Me Cl CN
Cu(BF4)2 0 C

Chem 206
H H Cl CN CN CH2OMe Cl

Instructive Issues of Regiocontrol with Quinone Dienophiles


O Me MeO O Orientation of Reacting Partners controlled by Lewis acid structure Reusch JOC 1980, 45, 5013 F 3B O Me Me O O O + Me Me O Sn O Cl4 Me MeO O H Me + MeO O O H Me Me O Me Me O

MeO O

MeO O

Corey, JACS 1969, 91, 5675 H Me

Ratio: 90 : 10

Diels-Alder Reactions with Chiral Dienes


Comprehensive Organic Synthesis, Vol. 5, Trost, Ed. 1991
4.1 Intermolecular Diels-Alder Reactions, W. Oppolzer, See page 347

Conditions thermal (100 ) BF3OEt2 (-20 ) SnCl4 (-20 )

Ratio 50 : 50 80 : 20 <5 : 95 X

O NPh O
25-50 C

Me

O PhN O

Me Me

O PhN

p. .

selection 80 : 20

H X X= OH Ratio

H X

36 : 64 83 : 17 >97 : 3 Me O OR O NPh Me O

selection >95 :5

Overman, JACS 1988, 110, 4625

Me OMe

Me

OR

Similar results provided by Stoodley Chem. Comm. 1982, 929 Kelly Tet. Let. 1978, 4311 OMe
BF3OEt2

O NPh
25-50 C

Me

OR

NPh Me O

O RO

OMe

Me

O RO Me

selection >95 :5
Me OH O O

0.4 equiv

Franck, Tet. Lett. 1985, 26, 3187 Franck, JACS 1988,110, 3257

R = Me: Ratio; 83 : 17 R = Me3Si: Ratio; 88 : 12 Me Me RO OR better than Me PhN O O H

Me OH O OMe
MgI2 0.5 equiv

Comments on the Transition State Me Avoid Eclipsing allylic substituents better donor (Me) anti to forming bond avoid gauche OR interaction Me PhN O

RO

selection >95 :5
OH O OMe

16-10-DA regiochem-2 10/22/03 7:01 AM

D. A. Evans

Cycloaddition Reactions: Diels-Alder Reaction


The Diels-Alder Cycloaddition Reactions

Chem 206

http://www.courses.fas.harvard.edu/~chem206/

Chemistry 206 Advanced Organic Chemistry

"Diels-Alder Reactions". Evans, D. A.; Johnson J. S. In Comprehensive Asymmetric Catalysis, Jacobsen, E. N.; Pfaltz, A.; and Yamamoto, H. Editors; Springer Verlag: Heidelberg, 1999; Vol III, 1178-1235 (pdf) "Chiral Bis(oxazoline) Copper (II) Complexes: Versatile Catalysts for Enantioselective Cycloaddition, Adol, Michael and Carbonyl Ene Reactions". Johnson, J. S.; Evans, D. A. Acc. Chem. Res. 2000, 33, 325-335. (pdf) "New Strategies for Organic Catalysis: The first Highly Selective Organocatalytic Diels-alder Reaction", MacMillan, JACS, 2000, 122, 4243. (pdf) "New Strategies for Organic Catalysis: The first Enantioselective Organocatalytic 1,3-Dipolar Cycloaddition", MacMillan, JACS, 2000, 122, 9874. (pdf)

Lecture Number 17

Cycloaddition Reactions-2
The Diels-Alder Reaction

Reading Assignment for week: Carey & Sundberg: Part A; Chapter 11 Concerted Pericyclic Reactions Carey & Sundberg: Part B; Chapter 6 Cycloadditions, Unimolecular Rearrangements Thermal Eliminations Fleming: Chapter 4 Thermal Pericyclic Reactions

Problem of the Day:


O Me Me O Me OMe Me toluene, H Me Me OMe O O

97% yield

Me Fallis J. Org. Chem. 1993, 58, 2186. H Me

D. A. Evans
17-00-Cover Page 10/23/03 3:34 PM

Friday, October 24, 2003


Me (+)-Longifolene

D. A. Evans

Chiral Auxiliary Controlled Diels-Alder Reactions


Non-Chelate Ester-Type Chiral Auxiliaries
LA O O BnO H RL RS

Chem 206

Review: Oppolzer in Comprehensive Organic Synthesis 1992,Vol. 4, 315-399.

Ester-Type Chiral Auxiliaries Corey JACS 1975, 97, 6908.


Me O Ph H O H Me H Me O BnO

S-trans geometry

AlCl3 89%
Ph O

rxn from this face Lewis Acid-Ester Complex Conformation Dictates Diastereoselection
LA O O R H RS RL R O O H O R O H RL RS

Diastereomer Ratio Not Given

LA
RL RS

p. .
Chem 3D model
R LA O H O RL RS R RL RS

front Face
H

AlCl3
O O

See Oppolzer ACIEE 1984, 23, 876.

front Face

Me O Me O Cl Ti Cl Cl Cl Cl O Cl Cl Ti Cl

Chem 3D model

diene approach

Me O Me

X-ray

17-01-Aux-control/DA 10/23/03 4:12 PM

D. A. Evans

Lewis AcidCarbonyl Complexes-3

Chem 206

Representative 1titanium complexes with organic compounds


Me O Me O Cl Cl Ti Cl Cl Cl Me O Me Cl Cl Ti O Cl

Representative 1titanium complexes with organic compounds

X-ray structure X-ray structure

Cl Cl O O
Ti

Cl Cl O OEt Me

p. . X-ray structure

R O Ph

Side view of in-plane Ti coordination: Ti-O-C-C = 2.86 TiOC angle = 152


Ph

O Ph Ph O O N Ti

O O

Ph

Cl Cl O Me O
Ti

Cl Cl O Me
Narasaka JACS, 1989, 111, 5340 A. Jorgensen, JACS, 1995, 117, 4435

X-ray structure

17-02-LA-C=O complexes-3 10/23/03 4:11 PM

D. A. Evans

Chiral Auxiliary Controlled Diels-Alder Reactions-2


Chelating Imide-type Chiral Auxiliaries Metal ion Dependent Diastereoselection
O R O R OR* Me N Me2CH O Me O
M+

Chem 206
Evans JACS, 1984, 106, 4261. Evans JACS, 1988, 110, 1238.

Ester-Type Chiral Auxiliaries: Chelating Dienophiles


EtO Me 0.7 equiv TiCl4 Entry A O O TiCl4 O

H O C H XV O H C

Me H XV

Exo-1 + Exo-2

Me EtO O O

R H H Me

Entry A B O O AlEtCl2

Temp 63 C 63 C

Endo/Exo Endo dr 39:1 39:1 93:7 22:78

Lewis Acid (1.2 equiv) SnCl4 SnCl4 TiCl4

Endo-2 Endo-1 Endo-1 Temp. Endo-2 Endo / Exo 25 78 78 78 78 78 2.7 3.1 2.6 1.5 15.7 4.4 92 : 8 93 : 7 91 : 9 80 : 20 >99 : 1 92 : 8

p. .

Entry B 2.5 equiv EtAlCl2

EtO O Cl2EtAl

OR*

AlCl3 (2.0 equiv) (0.7 equiv) Et2AlCl Et2AlCl

Helmchen Tetrahedron Lett. 1984, 25, 2191. ACIEE 1985, 24, 112. Tetrahedron Lett. 1985, 26, 3095.

Binding Mode Dictates Diastereoselectivity


Me O O N R O 1 equiv Me2AlCl Cl Al O Me

+
N

1 Point Binding
C 5H 6

Cl Cl O O
Ti

Cl Cl O OEt Me

O O Me Al

Endo/Exo = 4 :1 Rel. Rate = 1

O R N R

O O

> 1 equiv Me2AlCl

Me Me2AlCl2

2 Point Binding
C 5H 6 O

O + O R N R

X-ray structure Why Imide Dienophiles?? They are ~ 100-1000X more reactive

Endo/Exo = >20:1 Rel. Rate = 100

R = CH2Ph much more stereoselective than R = CHMe2

17-03-Aux-control/DA-2 10/23/03 4:11 PM

D. A. Evans

Chiral Auxiliary Controlled Diels-Alder Reactions-2


Angew Chem, Int Ed. 1987, 26, 1184 The Chiral Dienophile
Me2AlCl2 Me Me O Me
*

Chem 206

A Case for -Stacking:

Compare the alkylation rxn which is dominated by steric effects with the DA rxn which may be controlled by both steric and electronic effects Et O Me N R O Et N R O O LDA Et O O Et2AlCl Me O Al N R O Li O N R O MeI 30 Et * Me R Et O O N R O N O O O O

Me

Model

Al O + O N O

Al

30 Me

CH3

PhCH2

~3A

A Complex Application
OTIPS Et O O Me OTES H O O N Bn O Me2AlCl, 0 C 71% Et O O H H Me H O H H N Bn OTBS O O OTIPS OTES

p. .

G = 2.3 RT Log P1/P2 PLOT G FOR EACH RXN AS A FUNCTION OF THE SUBST., R.

20
CH2c-C6H11

CH2Ph

15
A1 10 A2
Et i-C3H7 Me

CH2p-MeOPh CH2p-ClPh CH2p-CF3Ph OTBS

diastereoselection 10:1

Alkylation,

The control experiment with no chiral auxiliary: OTIPS Et Me O OTES H O O N O Me2AlCl, 0 C 74% O H Et O Me H H O H N O O OTIPS OTES

Ph

0 0 5 10 15
Diels-Alder,

Conclusions:

20 D1
D2

25

30

Steric effects correlate well for the two reactions Added electronic effects from Bn group enhance facial bias
17-04-Aux-control/DA-3 10/23/03 4:36 PM

diastereoselection 6:1
OTBS

H OTBS

Evans, BlackLepicidin Synthesis, JACS, 1993, 115, 4497

D. A. Evans
Type I intramolecular Diels-Alder Reaction:

Diels-Alder Reaction: Intramolecular Reactions


A Type I Intramolecular Diels-Alder:
CO2H Me O O O O RO O OH OH O O O O CO2R Me R3SiO

Chem 206

Me

O t-Bu

HO O

Type II intramolecular Diels-Alder Reaction:

OR

Me H

Me H

H OR

Me H SiR3 OR

OH

(-)- Chlorothricolide
A Type II Intramolecular Diels-Alder:
O O O O O H CO2H O O H O Me Me O O O O H H O O Me C 5H 9 R3SiO

2 Diels-Alder Retrons 45% yield


R3SiO Me O t-Bu O O O O Me t-Bu

C8H15

OR

()-CP-263,114

CO2H Me Me O O C 5H 9 H SiR3 O OMe

Me

toluene, 120 C

Me O O N O O O

Me O C5 H 9

RO2C SiR3 OR

ZnCl2 CH2Cl2 Endo T.S.


X

Roush, Sciotti J. Am. Chem. Soc. 1998, 120, 7411-7419.


EtS C8H15

EtS Bn MeO2C

C8H15

CO2Me

approx. 60% yield MeO C CO Me 2 2

Some Intramolecular Diels-Alder Reviews: Shea Angew. Chem. Int. Ed. 2001, 40, 820. Fallis Acc. Chem. Res. 1999, 32, 464-474.

Fukuyama et al JACS 2000, 122, 7825-7826. The concept: Evans, et al Angew. Chem. Int. Engl. 1997, 36, 2119-2121.

17-05- Intramolecular DA 10/23/03 4:41 PM

D. A. Evans

The Diels-Alder Reaction: Enantioselective Catalysis


Articles and monographs of Significance

Chem 206

The conformation of the dienophile is also an issue


The S-cis versus S-trans dienophile conformation is coupled to the geometry of the Lewis acid-dienophile complex & both issues determine face selection
L S M X M R' O R O X R R' S R O X R M L M S

"Diels-Alder Reactions". Evans, D. A.; Johnson J. S. In Comprehensive Asymmetric Catalysis, Jacobsen, E. N.; Pfaltz, A.; and Yamamoto, H. Editors; Springer Verlag: Heidelberg, 1999; Vol III, 1178-1235. Review: Kagan, H. B.; Riant, O. Chem. Rev. 1992, 92, 1007-1019 Comprehensive Organic Synthesis, Vol. 5, Trost, Ed. 1991

re-face

re-face
X L

4.1 Intermolecular Diels-Alder Reactions, W. Oppolzer 4.2 Heterodienophile Additions to Dienes, S. M. Weinreb 4.3 Heterodiene Additions, D. L. Boger 4.4 Intramolecular Diels-Alder Reactions, W. R. Roush 4.5 Retrogade Diels-Alder Reactions, R. W. Sweger, A. W. Czarnik
Catalytic Asymmetric Synthesis, I. Ojima, Ed. 1993

(E) s-cis
M O X M L

M M X O

si-face

si-face

Chapter 9, Asymmetric Rxns with Chiral Lewis Acid Catalysts

(Z) s-cis

p. .

Chiral Lewis Acids in Catalytic Asymmetric Reactions Narasaka, Synthesis, 1991, 1-11 (Carbonyl-Lewis Acid Complexes) Schreiber, Angew. Chem. Int. Ed., 1990, 29, 256-272 Rotational barriers in Aldehydes & Ketones Coordinated to Neutral Lewis Acids Wiberg, JACS, 1988, 110, 6642 Theoretical Studies on Conformations of Acrolein, Acrylic Acid, Methyl Acrylate & their Lewis Acid Complexes Houk, JACS, 1987, 109, 14-23 C2 Symmetry and Asymmetric Induction, Whitesell, Chem. Rev., 1989, 89, 1581-1590

Theoretical Studies on Conformations of Acrolein & Methyl Acrylate & their Lewis Acid Complexes Houk, JACS, 1987, 109, 14-23

Stereoelectronic Effects (?) in Lewis acid-C=O Complexes


Y X M H Z O R Let X be the most electronegative ligand in the Lewis acid The stabilizing hyperconjugative interaction between the O-lone pair and * MX will provide a stabilizing interaction for the illustrated conformation.
J. M. Goodman, Tet. Lett. 1992, 33, 7219

The Design of Enantioselective Diels-Alder Catalysts


R' O X O X R' R R chiral Lewis acid catalyst R H R' O X

However, there is no evidence for this orienting effect in this X-ray structure reported by Reetz, JACS, 1986, 108, 2405 F
F 3B

O+

However pi-bonding to coordinated C=O-Al complexes has been reported Barron, JACS, 1990, 112, 3446, JACS, 1990, 112, 2950 Theory predicts a small rotational barrier about BO bond: Wiberg JACS, 1988, 110, 6642

17-06-DA enantioselect-1 10/23/03 4:48 PM

D. A. Evans

The Diels-Alder Reaction: Enantioselective Catalysis-2

Chem 206

Boron-Based Catalysts: Hawkins JACS 1991, 113, 7794


10 mol % H BCl2 OMe R re-face R H H Me CO2Me Temp (C) -78 -20 -78 Endo ee 97 % 91 % 92 %

Titanium-Based Catalysts: Narasaka JACS 1989, 111, 5340.


Ph Ph O R N O O + 10 mol % O R Temp (C) -40 0 25 10 mol % Ph O Ph O MeO2C N O O + Me O Ph 0 C Endo/Exo 96:4 92:8 88:12 Ph O O Ph O 94% ee TiCl2 CO2Me O N O Endo ee 64 % 91 % 64 % N O Me O O Ph Ph O O Ph O TiCl2 R

Endo/Exo Ratios not provided. R CO2Me H Me Ph

reported X-ray similar to this model

p. .

re-face R CO2Me

R O Ph R si-face CO2Me Ph O O Ph N Ti

R O Ph O O O Ph

X-ray
Chem 3D Models A. Jorgensen, JACS, 1995, 117, 4435

17-07-DA enantioselect-2 10/23/03 4:49 PM

D. A. Evans

The Diels-Alder Reaction: Enantioselective Catalysis-2

Chem 206

Mg(2+)-Based Catalysts: Corey Tetrahedron Lett. 1992, 33, 6807.


Me 10 mol% Me O O + Me Ph Me O Me Me Ph O N O H O Me N O O O

O N

N N Mg I I -78 C

Stereochemical Model:

Endo/Exo = 98:2 Endo ee = 91%

X-ray structure

PM3 model

Mg
p. .

Tetrahedral metal geometry


R H O

O N

O O
5 mol% cat. 25 C

O yield 98% 95% 95% R O N yield 89% 95% 100%

R H H X

PM3 model

Re-face

R O R = Me R = Ph R = Cl

time 8h 8h 8h

endo ee 96% ee 96% ee 94% ee


R

Limitations: Scope limited to illustrated reaction


O O N R R = Me + 2 SbF6 CMe3 R O O X N O O
10 mol% cat. CH2Cl2 +25 C

O O

Cu(2+)-Based Catalysts:

Evans, Miller, Lectka JACS 1993, 115, 6460. Angew. Chem. Int. Engl. 1995, 34, 798-800. JACS 1999, 121, 7559-7573. JACS 1999, 121, 7582-7594
2+

2-5 mol% cat. CH2Cl2

temp 25 C -20 C 0 C

endo ee 94% ee 97% ee 97% ee

Me Me O O R + N O O Me3C N Cu H2 O OH2 N O

R = Ph R = OAc

0 C

93% ee, 90% yield

17-08-DA enantioselect-3 10/23/03 4:50 PM

D. A. Evans

Chiral Cu(2+) Complexes as Chiral Lewis Acids

Chem 206

Cu(box) and Cu(pybox) catalyst-substrate complexes implicated in enantioselective reactions.


"Chiral Bis(oxazoline) Copper (II) Complexes: Versatile Catalysts for Enantioselective Cycloaddition, Adol, Michael and Carbonyl Ene Reactions". Johnson, Evans, Acc. Chem. Res. 2000, 33, 325-335. (pdf)

Me O N Cu Me3C R O N

Me O N O O

2+
O

Me

Me O

2+ 2 X
CMe3 O O

Me

Me O

2+ 2 X
CMe3 Ph OMe O N N Cu O H N OBn Ph O

2+

2X
CMe3

N Cu Me3C RO2C N N O N

N O

N Cu Me3C MeO R O

N O

2 X

Cycloaddition Reactions Michael Reactions

Enol Amination Reactons

Michael Reactions

Aldol Reactions

p. .
Me O N Cu Me3C O O OR' R R CMe3 N Me O

2+ 2 X
O

Me

Me O

2+
O

Me

Me O

2+ 2 X
CMe3 Bn O N X N Cu O N Bn O

2+

N Cu Me3C O

N O P

2X
CMe3 OR'

N Cu Me3C O R

N O OR'

2 X

OR'

Hetero Diels-Alder Reactions

R = H, Cycloaddition Reactions Ene Reactions R = Alkyl, Cycloaddition Reactions Aldol Reactions

Diels-Alder Reactions

17-09-DA enantioselect-4 10/23/03 4:51 PM

D. A. Evans

TemperatureEnantioselection Profile for [Cu((S,S)-t-Bu-box)](X)2 Catalysts


2+
O N Cu Me3C CMe3 N

Chem 206

Me O

Me

Me

Me O

2+

2X

a: X = OTf b: X = SbF6

O N Cu Me3C H O 2 N

(A)
2 X

cat. 1b
N O

C5 H6 -78 C: >98% ee 25 C: 94% ee


O

H O N O

OH2 CMe3

2
OTMS

100
(D) (B)

Me cat. 1b
O O N O O Pyr Me CO2Et O N O O

98
EtO2C

(CF3)2CHOH -78 C: 98% ee -20 C: 94% ee

(E)

96

enantiomeric excess (%)

p. .

(B)

(A)
O MeO O

94
(C)

(C)

Me

OTMS StBu

cat. 1a -78 C: 99% ee 25 C: 92% ee

Me MeO O

OH O StBu

92
(A)
O

Ph OEt O OEt

90

(B) (C)

(D)

cat. 2a 3 mol. sieves -40 C: 99% ee 25 C: 94% ee


EtO2C O OEt

Ph

88

(D) (E)

86
N

OTMS

84 -80 -70 -60 -50 -40 -30 -20 -10 0 10 20 30

(E)

cat. 2a
O O N O N

O Pyr

Troc N N H

O N

O O

CF3CH2OH

temperature (C)

Troc

"Chiral Bis(oxazoline) Copper (II) Complexes: Versatile Catalysts for Enantioselective Cycloaddition, Adol, Michael and Carbonyl Ene Reactions". Johnson, Evans, Acc. Chem. Res. 2000, 33, 325-335. (electronic pdf)

-78 C: 99% ee 25 C: 96% ee

17-10-DA enantioselect-5 10/22/03 4:31 PM

D. A. Evans Ethylene & Butadiene


+

Diels-Alder Reaction: Molecular Orbital Analysis


Vs Butadiene & Acrolein
O
+

Chem 206

O H H RO O

"Inverse-Electron Demand" Hetero-Diels-Alder Reactions


X RO
+

X
+

OR

OR

LUMO2 LUMO1 LUMO3

LUMO

LUMO

LUMO

E
HOMO1 HOMO2 HOMO3

E
HOMO

LUMO

HOMO HOMO

HOMO

E (LUMO3-HOMO1) < E (LUMO2-HOMO1)

Rate Acceleration

"Inverse-Electron Demand" Diels-Alder Reactions


CO2R RO RO
+ +

Lewis Acid Catalysis of the reaction is possible:

CO2R XOC O OR RO
+

LA O

O X X O
+

OR

XOC

OR

LUMO LUMO LUMO

LUMO

LUMO

E
HOMO

LUMO

LUMO

E
HOMO HOMO HOMO HOMO

LUMO

HOMO HOMO HOMO

17-11-IED-diels alder 10/22/01 8:08 AM

D. A. Evans

Hetero-Diels-Alder Reactions Chiral Catalysis


Chiral Copper Lewis Acids:
O Me O N O Me3C Me O Cu N
2 X 2+

Chem 206

Heteroatom-substituted reactions are also possible:


O O

Me

Me O N

2+ 2 X

+
O O

A: X = OTf B: X = SbF6

O N

CMe3

Cu Me3C H O OH CMe3 2 2

O EtO2C N

O O

10 mol% 2B OSiR3 Ph Me

Ph

OSiR3 O

OMe N O O OSiR3 Me OMe H O 10 mol% 1A O OEt Endo T.S. N O O OMe >99:1 Endo:Exo 96% yield 99% ee O Me O CO2Et R3SiO Me O CO2Et

p. .

Me CO2Et

O Ph Me

CO2EtO N

O O

(CF3)2CHOH

Ph Me

OSiR3 O

CO2Et Evans, Scheidt, Johnston, Willis J. Am. Chem. Soc. 2001, 123, 4480.

CF3CO2H
R3SiO Me

O CO2Et

Me + EtO2C O Me 2 mol% 2A + EtO2C O O EtO2C OEt 2 mol% 2A EtO2C

Me

Jorgensen J. Am. Chem. Soc. 1998, 120, 8599. 24:1 Endo:Exo 87% yield 97% ee

O Me

OEt

C4H10OTBS O O N O 5 mol% 1B H H

C4H10OTBS N O O O

16:1 Endo:Exo 96% yield O 97%ee

CH2Cl2

81% yield 96% ee, 99:1 dr Evans, Johnson J. Org. Chem. 1997, 62, 786-787.

Evans, Johnson, Olhava J. Am. Chem. Soc. 2000, 122, 1635.

17-12-Hetero DA 10/23/03 4:51 PM

D. A. Evans

Diels-Alder Reactions: Synthesis of FR182877


The Transannular Diels-Alder Step
8

Chem 206

Structures of FR182877 & Hexacyclinic acid


HO H Me
11 8

Me OH O

Structural Differences: Acylation at C8 hydroxyl

AcO H HO2C
11

Me OH O OH H Me

TBSO TBSO Me O

CO2Et Me OTBS
Ph2Se2O3 R3N, RT 63%

TBSO TBSO Me O

CO2Et
18

Me OTBS

H H Me

H
2

Oxidation at C11

H H Me

H H O O
17

19

endo vs. exo Diels-Alder retron

O O

17

Me

Hydration across C2-C17 double bond

Br

Br Me
50 C

Me Diels-Alder cycloaddn

()-FR182877
Sato, B. J. Antibiot. 2000, 123, 204, 615. Corrected Structure: J. Antibiot. 2002, C-1.

Hexacyclinic Acid
Hofs, R., et al. Angew. Chem. Int. Ed. 2000, 39, 3258.

TBSO H Br H H
TBSO

Me OTBS CO2Et Hetero-DA Cycloaddiiton


63%

Me MeO H

OMe H H CO2Me Me O Br H Me OMe

p. .

Hypothesis: A Transannular Diels-Alder Cycloaddition Cascade


TBSO TBSO Me O CO2Et Me OTBS Br Me A B H C Me DO O OH O H Me
FR182877 endo-TS

Me 1. Diels-Alder cycloaddn 2. Hetero-DA cycloaddiiton

O Me

C18 & C19 stereocenters exert complete control over the first cycloaddition (Evans)

HO H X
11

Me RO Me H Br H

-120
OR H H H Me
11

CO2R

kcal/mol (PM3)

OR Me H

MeO MeO
8

6
2

CO2Me
18

-130 -140 -150 -160 -170

Me

O
19

Me OMe

endo-I

endo-II

Br Evans & Starr, JACS, 2003, 125, ASAP Sorensen et al, JACS, 2003, 125, 5393

Me

(natural configuration)

-180 2.1 2.3 2.5 2.7 2.9


-Face Separation ()

E = 59 kcal/mol @ 2.1
17-13-FR DA Rxns 10/22/03 4:01 PM

Problem of the Day:


O Me O Me OMe Me Me H Me Me Me OMe O O H Me

Me

toluene,

Fallis J. Org. Chem. 1993, 58, 2186.

97% yield

(+)-Longifolene

O H Me O Me OMe OMe

1,5-H shift

H Me Me O Me

Me

1,5-H shift

Me H

Me

O Me

OMe

Me

Me

Me

OMe O O

Me Me O OMe O Me H

Me Me O MeO

H O Me

[4+2]
Me

Exo T.S.

Me

Me

Me Me O OMe

O Me

OMe O O

Me Me

OMe O O

Me

97% yield

17-14-Fallis Longifolene 10/22/01 8:06 AM

D. A. Evans

Dipolar Cycloadditions
Dipolar Cycloaddition Reactions

Chem 206

http://www.courses.fas.harvard.edu/~chem206/

General References Carruthers, W. Cycloaddition Reactions in Organic Synthesis.; Pergamon: Elmsford, NY, 1990. Padwa, A. 1,3-Dipolar Cycloaddition Chemistry, John Wiley, 1984, Volumes 1 & 2, Jorgensen, Asymmetric 1,3-Dipolar cycloadditions, Chem Rev. 1998, 98, 863-909 Padwa, A. Generation and utilization of carbonyl ylides via the tandem cyclization-cycloaddition method." Acc. Chem. Res. 1991, 24, 22. (handout) Confalone, P. N.; Huie, E. M. The [3+2] Nitrone-Olefin Cycloaddition Reaction Org. React. (N.Y.) 1988, 36, 1. S. Kanemasa, Metal Assisted 1,3-Dipolar Cycloaddition Reactions, SynLett 2002, 1371-1387 (handout)

Chemistry 206 Advanced Organic Chemistry

Lecture Number 17

Dipolar cycloaddition Reactions

Reading Assignment for week: Carey & Sundberg: Part A; Chapter 11, p 647-648 Concerted Pericyclic Reactions
Carruthers, W. Cycloaddition Reactions in Organic Synthesis.; Pergamon: Elmsford, NY, 1990.pp 294-331 (Handout) D. Ripin, W. Use of Isoxazoles as 1,3-Dicarbonyl equivalents in Organic synthesis; Evans Group Seminar 1997 (ElectronicHandout) Problem of the Day
Provide a plausible mechanism for this transformation in the space below. In attacking this question, it is important that you are aware of the transformation that transpires when terminal acetylenes are treated with Cu(I) or Ag(I) in the presence of an amine base. O Ph H Ar N H R
13% CuX R3N, RT

Ph N O

R
major

Ar

D. A. Evans

Monday, October 27, 2003

In 1995 Miura and co-workers reported the remarkable reaction illlustrated below (J. Org. Chem. 1995, 60, 4999). Recently, Fu has reported an enantioselective variant of this transformation (J. Am. Chem. Soc. 2002, 124, 4572). In most instances, the cis adduct is formed in large excess (>90%). There is really no thoughtful mechanism in the literature for this transformation. You will be graded on "reasonability"

18-00-Cover Page 10/27/03 9:17 AM

D. A. Evans

Dipolar Cycloaddition Reactions: An Introduction


The General Reaction Family
B

Chem 206

Classification of 1,3-Dipoles Containing C, N, & O Atoms


A D E C

B A C A D

B C E

[4S + 2S]

isoelectronic with allylanion

HOMO FMO Analysis

B A

LUMO B A

E LUMO D HOMO D

p. .

The specific set of reaction partners, will define the dominant frontier orbitals

Reaction Stereospecificity: The Dipolariphile (Padwa, Vol 1, pp 61-90)


Me MeO2C Me CO2Me H C Me MeO2C CO2Me Me H Me MeO2C N N N N CO2Me Me Me MeO2C N N Me CO2Me

H MeO2C

H CO2Me

Rinehart, JACS 1962, 84, 3736

N O

nitrone

MeO2C

CO2Me

Fumarate gives trans cycloadduct

Huisgen, Chem. Ber. 1969, 102, 736

18-01-Introduction-1 10/27/03 8:56 AM

D. A. Evans
Reaction Stereospecificity: Dipolariphile continued
Bn H N CO2Me Bn 100 N CO2Me RO O

Dipolar Cycloaddition Reactions: An Introduction


Bn MeO2C major R O Bn Me MeO2C N Me N

Chem 206

Rng strain factors may control regioselectivity


H 2C H O N

Ratio 98:02

Me

heat in toluene
Me O Me N H Me H H H

70%

Strain Energy 43.8 kcal/mol

DeShong, JOC 1985, 50, 2309; Tet. Lett. 1986, 27, 3979

We will return to the other stereochemical element of this reaction shortly Reaction Regiochemistry (Padwa, Vol 1, pp 135)
HO Ph Ph N N N CO2Me N N N

MeI LiAlH4

Strain Energy 37.6 kcal/mol

p. .

LUMO
N Ph N N N C4H9 N N

Me CO2Me N H Me H O C4 H 9 H

Here is an elegant example of regiochemical control. While the authors offer no explanation for the outcome, it appears that the reaction is being governed by transition state steric strain factors. Oppolzer, JACS 1978, 98, 6722

Ph

HOMO

The specific set of reaction partners, will define the dominant frontier orbitals Steric Effects will frequently alter regioselectivity (Padwa, Vol 1, pp 144)
H C H Me Me C Me N N Me Me H CO2Me N N N H H Me Me N N N CO2Me Me Me CO2Me

Reaction Diastereoselectivity
H Ar N CO2Me H

Huisgen et al, Angew.Chem. Int. Ed. 1969, 8, 604


Ar Ar N

CO2Me

100

MeO2C

CO2Me

MeO2C

CO2Me

MeO2C

CO2Me

100:0

conrotation

MeO2C Ar

CO2Me Ar MeO2C N CO2Me

Me

Ar N

100

MeO2C

N CO2Me

96:4

CO2Me CO2Me

MeO2C

CO2Me

Steric effects are also important considerations in reaction regiochemistry

18-02-Introduction-2 10/25/03 5:19 PM

D. A. Evans
Relative Orientation of Reaction Partners

Dipolar Cycloaddition Reactions: An Introduction


Cis Disubstituted Olefins.
N MeO2C H R N Bn H Me Me H N Bn R H H Me R O Me Me N O H H MeO2C H N Bn H H O R Me O MeO2C Bn N O H O N H O O N

Chem 206

DeShong, JOC 1985, 50, 2309; Tet. Lett. 1986, 27, 3979

H O

HH

MeO2C H R H O

Major cycloadduct (30:1)

The preceding trend appears to be reinforced by cis disubstitution I. Washita et al, Chem lett 1979, 1337

H H MeO2C H H O

Ratio 98:02
Bn N

Intramolecular Reaction Variants


Me

N. LeBel et al, JACS 1964, 86, 3759


Me H O N Me N H Me

Me O

p. .

The above analysis is clouded by the fact that the geometry of the 1, 3-dipole is not fixed.
MeO2C H H N Ph H H H CO2Me N Ph H

Orientation probably driven by ring strain as in Oppolzer case (previous page) Intramolecular Reaction Variants

"highly diastereoselective"

P. Confalone et al, Tet. Lett. 1984, 25, 4613

Monosubstituted Olefins. In the following study, 1,3-dipole isomerism is not an issue


N N H O H R H H R O N O O N O O CO2H CHO H H2O N Bn H

HNTMS2

O H O Bn N H

Ph 78%

Me

CO2Me

100% 82%

highly stereoselective
O CO2 Ar H R CH2 N Bn Ar H H H CH2 N Bn

H R R H N H O

22%

0%

18%
Ar

R N H Bn

Diastereoselection appears to be dictated by steric effects Tufariello, Accounts. Chem. Res. 1979, 12, 396-1403

Conclusions on Reaction Diastereoselection


In general reaction diastereoselection appears to be dictated by steric and torsional rather than electronic factors

18-03-Introduction-3 10/27/03 9:01 AM

D. A. Evans

Dipolar Cycloaddition Reactions: Nitrile Oxide Cycloadditions


The Basic Reactions Reactions with olefins are stereospecific

Chem 206

Padwa, A. 1,3-Dipolar Cycloaddition Chemistry, John Wiley, 1984, Vol 1, Chapter 3


R R C N O R N O R R R C N O R N O R

Carruthers, W. Cycloaddition Reactions in Organic Synthesis.; Pergamon: Elmsford, NY, 1990.Chapter 6, pp2 69298 Me OH Me Et3N 0 C CO2Et Me CO2Et Me Me O O N

Isoxazoles

+
Me

Cl

Isoxazolines

+
Me Me

O N

Et3N PhN=C=O 0rt Me

Methods of Generation
N R OH NCS H R N

H O Et3N

Me

Method A

Oxazoline Cleavage

p. .

Cl Me O N Me H O Me Me Me H2 Raney Ni Me OH O Me Me OH O Me Me Me O R O R

Method B
R

Et3N PhN=C=O

NHPh

Non-aldol approach to aldol adducts

Stability

Nitrile oxides are usually prepared in the presence of the olefin or acetylene acceptor. These intermediates are generall unstable and will dimerize if not given an alternative reaction course
N R O N O R N O N O R

O N

H2 Raney Ni

Me

R R

C C

O R

H2 Raney Ni

Non-aldol approach to 1,3-dicarbonyls

Regioselectivity Nitrile oxide cycloadditions with olefins and acetylenes are usually quite regioselective and in the direction as illustrated above. DeShong, JOC 1985, 50, 2309; Tet. Lett. 1986, 27, 3979

Preferred method for reducing oxazoles and oxazolines: Nittta et al, Chem. Comm. 1982, 877-878: Mo(CO)6 MeCN

18-04-Nitrile oxides 10/27/03 9:03 AM

D. A. Evans
Miyakolide Synthesis:
Me Me

Nitrile Oxide Cycloadditions: Applications in Synthesis


with David Ripin & David Halstead, JACS 1999, 121, 6816-6826

Chem 206
Me

The Intermolecular Case


Me Me O 3-ClPhNCO i-Pr2NEt 90 C 96%

Me O OTBS Me BnO OTBS O Me


1

O O Me
17 13 19

O O Me OH O O O
19 17

OH O

biomimmetic aldol reaction?

O 2N Me Me O O Me OR O Xp OH CO2Me O Me Me O Me

OTBS N O Me BnO OTBS O O Me Xp

OMe

Me

OH

Me

OH

CO2Me

H O

OMe

Me

OPMB

CO2Me

Me

O
13

Me OH
11

OPMB

CO2Me

p. .
Me O

O M O H Me H
H 11

R OH

Me Me Me O O O Me OH O

The Intramolecular Case

R N
19 17

Me Me Me OR O 2N Me O Me OTES O
19

Transannular Aldol

O O O Me OMe

Me

3-ClPhNCO i-Pr2NEt 90 C 68 % Me Me

Me

OH

CO2Me Me OPMB O O Me Me N
19

Competing olefin chlorination eliminated this approach to the nitrile oxide precursor
Me Me HO N O OTES Me HO Me

Xp

OMe

O O Me
1

O
13

Me OTES

OMe

X
Cl

O OTES Me Xp

Me

OPMB

OMe

18-05-Nitrile oxide Appl-1 10/26/03 9:01 PM

D. A. Evans

Nitrile Oxide Cycloadditions: Applications in Synthesis


Reaction Diastereoselectivities
N Et OH O O Et OH Ph N O

Chem 206

Development of Directed Cycloadditions


Kanemasa at al, JACS 1994, 116, 2324-2339 (electronic handout) Kanemasa at al, Metal-Assisted Stereocontrol of 1,3-Dipolar Cycloaddition Reactions SynLett 2002, 1371-1387 (electronic handout)

A
base

Ph

Et OH

Et3N
Me Ph Ph C N O N O R' Me Ph R' N O Me

67:33 95:5

60% 75%

Me

EtMgBr

O Me OBn

69:3174:26
conditions; no cat, ZnI2, Ti(OiPr)4
OH

N Me

O Me OH Ph

O Me OH

A
base

Ph

p. .

While lewis acid activation is known, no change in regiochemistry was noted under above connditions Magnesium alkoxides found to effect regiochemical control
N Ph R Me OH OH + Et3N OMgBr (2 equiv) O CH2OH Ph N O R CH2OH OH Me nBu

Et3N EtMgBr
N O

61:39 96:4
N nBu Ph

60% 75%
O Me nBu OH

Ph R OH

A
base

Me

Ph OH

46:54 >99:1

82% 66%

Et3N EtMgBr Stereochemical Rationale


Ph C H H H Ph N O X M

60:40 96:4 85%

N Ph

Me

Cl

Me Pr

OMgBr (2 equiv)

>99:1

68%

Ph H H

X M

OMgBr (2 equiv) N Ph OH O

98:2

73%

H O R

syn product

Anti product

H O R H

A
OH base

Rate acceleratons
Me OH Me OH OH Me OH

OH

Et3N EtMgBr

67:33 100:0 2030

6900

490

16,000

18-06-Nitrile oxides/Kanemasa 10/27/03 9:20 AM

D. A. Evans

Nitrile Oxide Cycloadditions: Applications in Synthesis

Chem 206

Applications in Polypropionate Synthesis


Carreira at al, Angew. Chem. Int. Ed. 2001, 40, 20822085

Applications to the Synthesis of Epothilones A, B


Carreira & Bode JACS 2001, 123, 20822085 Epothilone A: R = H Epothilone B: R = Me
O N Me O O Me OH Me Me OH O Me S Me N Me O Me OH O Me

TBSO

O Me

Me

TBSO

O Me

S Me

82%
Me OH

87%
Me OH TBSO H N OH Me OH

Me

Me

OH TBSO

synsyn
N

antisyn

Me

H Me N O OTIPS MgBr chelate control Me N N Me S Me Me HO Me Me ~10:1 N O R Me O M O Me OTIPS TBSO N O Me Me OH Me OH O Me S

Me TBSO N O Me Me Me OH

p. .
antianti

73%
Me Me

68%

synanti

a, Oxime Chlorination: t-BuOCl; b, 3 Equiv EtMgBr, room temp, 12 h

Oxazoline Reduction
EtO TBSO N O Me Me Me OTES TBSO O OH Me EtO

O P Me

OH Me H OH t-BuOCl EtMgBr 79% EtO EtO

O P Me

O Me OH N Me S CHO

Raney Ni 90%

synanti

Me

Me

OTES

LiCl, DBU Oxidation

Lit Conditions: Curran, JACS 1983, 105, 5826; JOC 1984, 49, 3474 N TBSO N O Me Me Me OTES O

Raney Ni 94%

TBSO

OH Me

O Me Me

antisyn

Me S

Me

Me

OTES

18-07-Nitrile oxides/Carreira 10/26/03 8:57 PM

D. A. Evans
The Basic Reactions

Carbonyl Ylides: Introduction


Carbenes Plus Carbonyl Groups
R M R O R R O

Chem 206

Padwa, A.; Hornbuckle, S. F. Chem. Rev. 1991, 91, 263-309. Reviews - Ylides Padwa, A.; Krumpe, K. E. Tetrahedron. 1992, 48, 5385-5483. Padwa, A. Acc. Chem. Res. 1991, 24, 22-28.

R1 A

O B

Tandem Intramolecular CyclizationIntermolecular Cycloaddition


R R O CHN2 O CF3 NPh CF3 R O O H O R O CO2CH3 O O O X Padwa, A.; Hornbuckle, S. F.; Fryxell, G. E.; Stull, P. D. J. Org. Chem. 1989, 54, 817-824. CO2CH3 O NPh O O O R O N C CO2Et O N CO2Et

Stabilized (Isolable) Carbonyl Ylides


NMe2 N2 F 3C CF3 H3C CF3 N CH3 O N CH3 Me2N O

Rh2(OAc)4

F 3C F 3C

RCHO DMAD
R O R H

p. .

F 3C

CH3

O H

Arduengo, A. J., III; Janulis, E. P., Jr. J. Am. Chem. Soc. 1983, 105, 5929-5930

Hamaguchi, M.; Ibata, T. Tetrahedron Lett. 1974, 4475-4476.

O N O

CH3

O 2N

O N N2

O Cu(acac)2 X

O 2N

CH3

PhH, 80 C

O N O

CH3

Intramolecular Variants
O

X = H, Br, OMe
O 2N C6H4NO2 CO2CH3 CH3 N Ph CO2CH3 CH3O2C CO2CH3 O X O CHN2 O H O

O O

Rh2(OAc)4

75% Can make 5-7 membered rings

quantitative
Hamaguchi, M.; Ibata, T. Chem. Lett. 1975, 499-502.

18-08-Carbonyl Ylids 10/26/03 9:30 PM

D. A. Evans

Carbonyl Ylides: Applications


Carbonyl Ylide Cycloadditions of Diazoimides
O O N Bn N2 O Me Rh2(OAc)4 PhCH3, 110 C Bn N O COMe O O A H CH3 O A B O O

Chem 206

Cycloadditions with Oxidopyrylium Ylides

Me

74%
H O Me Me N Bn N2 O O Me Rh2(OAc)4 PhCH3, 110 C Me Bn N O COMe O Me R O OH Br2, MeOH -45 C MeO O OMe OH R

1) H3O+ 2) Ac2O / pyr AcO O

O R

Maier, M. E.; Evertz, K. Tetrahedron Lett. 1988, 29, 1677-1680.

73%
O O Me O O N H H N O AcO O H DBN, CH2Cl2, RT O O

80% from furfuraldehyde

p. .

O N

O Me N2

75%
H O

Rh2(OAc)4 PhCH3, 110 C

"high yield"
O Me O N

AcO

pyrolysis

O cat. HOAc / CH3CN H OH O 150 C, 16h O

O N

52%

Sammes, P. G.; Street, L. J. J. Chem. Soc., Chem. Commun. 1982, 1056-1057 Hertzog, D. L.; Austin, D. J.; Nadler, W. R.; Padwa, A. Tetrahedron Lett. 1992, 33, 4731-4734.

18-09-Carbonyl Ylids-2 10/26/03 9:47 PM

D. A. Evans

Carbonyl Ylides: Applications


Cyclizations with 5-Hydroxy-4-Pyrones

Chem 206

Phorbol: The Hydroxypyrone Approach


OH Me OH Me Me OH O OH OH H

O OH

H O A B A

OH O H B Me

Phorbol

O OH O O N

OAc O

1) PhH, reflux, 12 h 2) acetylation


Me

O N

O H t-BuMe2Si O

p. .

Me

Me TBSO O O OTBS

55%
O OH O O Me N Me OAc

PhCH3, 200 C

Me H O

1) CH3CN,reflux, 60 h
O

O O

OTBS

2) acetylation
N

42%
O OH O

71%
Me O

1) heat
( )n Z Z Z O

OAc O ( )n Z O O H O Me

Me H O TBSO OTBS

2) acetylation

Z = CO2Me

OTBS

n = 1 : 70% n = 2 : 65%
Garst, M. E.; McBride, B. J.; Douglass, J. G. III. Tetrahedron Lett. 1983, 24, 1675-1678. Wender, P. A.; McDonald, F. E. J. Am. Chem. Soc. 1990, 112, 4956-4958

18-10-Carbonyl Ylids-3 10/26/03 10:00 PM

D. A. Evans

Carbonyl Ylides: Problems

Chem 206

Problem 53. Williams recently reported an approach to the synthesis of quinocarcinamide (1) (J. Org. Chem. 1995, 60, 6791). The pivotal process that establishes the tetracyclic nucleus is the two-step transformation shown below (eq 1). CO2H CO2Me H Me Me Me N N N H N (1) N two steps N OMe
CH2OH O

OMe

CH2OH O

OMe

CH2OH O

Devise a strategy for transforming A into B and clearly illustrate your answer in the space below. Full credit will be awarded to concise answers. Problem 55. The following transformation was recently reported by Heathcock during studies directed toward the synthesis of sarain A (Tetrahedron Lett. 1995, 6, 2381). From your knowledge of the functionality present in the starting material, deduce the structure (including stereochemistry) of the reaction product which has the same molecular weight as the starting material. Hint. the 1H NMR spectrum of the product reveals that the olefinic resonances have disappeared. O

p. .
MeO

N Ts

N CO2Et

Ph

110 C 45-55%

Your mechanism for the transformation

product structure

Problem 65. The following stereoselective nitrile oxide cycloaddition has been reported by Kozikowski (Tetrahedron Lett. 1982, 23, 2081; J. Org. Chem. 1984, 49, 2762). Provide the stereostructure of the major product and rationalize the stereochemical outcome as indicated in the directions. Me O 2N Me
PhNCO, Et3N

The product ? Stereoselection: 16/1 product structure

Problem 87. The illustrated transformation has been utilized by Coldham (Chem. Commun. 1999, 1757) to construct the core ring system of the manzamines. N H S S BocN CHO CH2 O OEt
i

H S N N S Me N

N H

Pr2NEt, toluene, 45%

BocN

OH

NHMeHCl

EtO2C "one diastereomer"

18-11-Problems-1 10/26/03 10:19 PM

D. A. Evans

Carbonyl Ylides: Problems

Chem 206

Problem 90. Padwa and co-workers recently disclosed the illustrated multistep polycyclisation as a possible route to the strychnine core (Org. Lett. 2001, ASAP)

N2 O O O

Ph N

H Rh2(pfb)4 145 C O N O

Ph

In the space below, provide a mechanism for the indicated transformation. Hint: The management suggests that a carefull bidirectional analysis might help you to arrive at a sollution of this question. Problem 136. A recent paper by Harwood and Park highlights the rapidity which whch one may assemble complex architecture in a single chemical operation (Tetrahedron Lett. 1999, 40, 2907 and earlier cited references). The transformation in question is illustrated below. You are asked to address two aspects of this transformation. Ph H N O O H

benzene, heat

H Ph N H O O

p. .

Part A. Provide a concise mechanism for the indicated transformation. For now, ignore the stereochemical aspects of the reaction. Part B. Predict the stereochemical outcome of the reaction at the three new stereocenters, and provide a three-dimensional drawing of the transition state wherein these centers are produced.

Problem 171. A recent paper by Dolle (Tetrahedron Lett. 1999, 40, 2907) highlights the rapidity which whch one may assemmble complex architecture in a single chemical operation. The transformation in question is illustrated below. CO2Me O 1. BocNHNH2 CO2Me 2. EtOH, reflux N NBoc 75% yield, one diastereomer

Provide a concise mechanism for the indicated transformation. In that step where the complex stereochemical relationships are established, a carefully rendered three dimensional illustration is requested.

Problem 189. This question is taken from recent work reported by Jack Baldwin (Org. Lett., 1999, 1933 and 1937). Provide a mechanism for the conversion of I to II. O O O N2 O Rh2(OAc)4; H3O+ quench O O

I
O

OH

II

18-12-Problems-2 10/26/03 10:18 PM

D. A. Evans

Acid-Base Properties of Organic Molecules

Chem 206

http://www.courses.fas.harvard.edu/~chem206/

Articles on the Acidities of Organic Molecules

Chemistry 206 Advanced Organic Chemistry

Lowry & Richardson: 3rd Edition, Chapter 3 Acids and Bases

Here is a web site containing Brodwell pKa data Lecture Number 19

http://www.chem.wisc.edu/areas/reich/pkatable/index.htm

Acid-Base Properties of Organic Molecules


Bronsted Acidity Concepts in the Activation of Organic Structures Medium Effects on Bronsted Acidity Substituent & Hybridization Effects on Bronsted Acidity Kinetic & Thermodynamic Acidity of Ketones Kinetic Acidity: Carbon vs. Oxygen Acids Tabulation of Acid Dissociation Constants in DMSO

Problems of the Day:


Explain why 1 and 3 are ~4 pKa units more acidic than their acyclic counterparts 2 and 4. (J. Org. Chem. 1994, 59, 6456)
O O O Me 2 O Et HN 3 O O Me N H 1 4 O O Et

Reading Assignment for this Lecture: Carey & Sundberg: Part A; Chapter 7 Carbanions & Other Nucleophilic Carbon Species
"Equilibrium acidities in DMSO Solution", F. G. Bordwell. Acc. Chem. Res. 1988, 21, 456-463. (handout) Wednesday, October 29, 2003

The thermodynamic acidities of phenol and nitromethane are both ~10; however, using a common base, phenol is deprotonated 10+6 times as fast. Rationalize
pKa(H2O) ~10 Base H rel rate: 10+6

D. A. Evans
19-00-Cover Page 10/27/03 11:36 AM

pKa(H2O) ~10

O H 3C N O

Base rel rate: 1

O H2 C N O

D. A. Evans Activation of Organic Molecules


Base Activation
R1 C R2 R3 H - H-base R2 base R1 C R3

Acidity Concepts-1
Definition of Ka

Chem 206

Let HX be any Bronsted acid. In water ionization takes place: HX + HOH H3O+ + X

Nucleophile

where

Keq =

[H3O+] [X] [HX] [HOH]

where [HOH] = 55.5 mol L-1

(A)

Since [HOH] is, for all practical purposes, a constant value, the acid dissociation constant Ka is defined wilthout regard to this entity. e.g. HX Hence Ka = H+ + X [H+] [X] [HX] where H+ = H3O+ (B)

pKa , describes quantitatively a molecule's propensity to act as an acid, i.e. to release a proton. - Medium effects - Structural effects (influence of substituents R1)

Acid Activation p. .
R1 X R2 X = e.g. O, NR ... R2 acid (protic or lewis acid) R1 X acid

From the above definitions, Ka is related to Keq by the relation: Ka(HX) = 55.5 Keq(HX) (C)

Electrophile Autoionization of water


HOH + HOH H3O+ + HO Keq = 3.3 X 1018

The Aldol Example


O HOMO R M R O O base catalysis H R R R base O R R O O R acid LUMO O R SiMe3 R M O acid catalysis H R R R R R R OH R R O M From Eq C: Hence

Ka = 55.5 Keq = 55.5(3.3 X 1018) Ka = 1.8 X 1016

Since pKa is defined in the following equation: pKa = log10 [Ka] The pKa of HOH is + 15.7 Keep in mind that the strongest base that can exist in water is HO.

Ca 10+6 Activation
O O SiMe3

Lets now calculate the acid dissociation constant for hydronium ion.
H3O+ + H 2O H3O+ Keq = 1 + H 2O

obviously:

Ka = [HOH] x Keq hence Ka = 55.5 pKa = log10 Ka = 1.7

The strongest acid that can exist in water is H3O+.

19-01-Acidity Concepts-1 10/27/03 12:51 PM

D. A. Evans
The Gibbs Relationship G = RT ln K or

Acidity Trends for Carbonyl & Related Compounds


Medium Effects on the pKa of HOH 2.3 RT = 1.4 at T = 298 K in kcal mol-1 ** The gas phase ionization of HOH is endothermic by 391 kcal/mol !!! Representative pKa Data with pK = log10 K Substrate HOH HSH
Energy

Chem 206
HOH pKa 15.7 31 279 (est)** Medium HOH DMSO Vacuum

G = 2.3 RT log10 K G298 = 1.4 log10 Keq G298 = 1.4 pKeq 1.4 pKa

DMSO 31.2 14.7 29.0 18.0 17.2 24.6

HOH 15.7 7.0 15.3 9.9 10.0 17

pKa 15.5 7.7 13.7 8.1 7.2 7.6

Hence, pKa is proportional to the free energy change

Keq 1 10

pKeq 0 -1 -2

G 0 1.4 2.8 kcal/mol


Reaction coordinate

MeOH
A HA G

C6H5OH O2NCH3

p. .

100

Medium Effects

O Ph C CH3

Consider the ionization process: + solvent A: + solvent(H+)

H A

In the ionization of an acid in solution, the acid donates a proton to the medium. The more basic the medium, the larger the dissociation equilibrium. The ability of the medium to stabilize the conjugate base also plays an important role in the promotion of ionization. Let us consider two solvents, HOH and DMSO and the performance of these solvents in the ionization process.

The change in pKa in going from water to DMSO is increasingly diminished as the conjugate base becomes resonance stabilized (Internal solvation!). Substrate DMSO 18.1 HOH 16.0 pKa 2.1

The Protonated Solvent


H

Conjug. Base Stabiliz.

O
O H A H No H-bonding Capacity

O OEt
16.4 13.3 3.1

Water

H O H Me

EtO O Me NC O

DMSO

HO S Me

Me CN

13.3 11.1

8.9 11.2

4.5 0

As shown above, although HOH can stabilize anions via H-bonding, DMSO cannot. Hence, a given acid will show a greater propensity to dissociate in HOH. As illustrated below the acidity constants of water in HOH, DMSO and in a vacuum dramatically reflect this trend.

19-02-Acidity Concepts-2 10/27/03 12:54 PM

D. A. Evans

Acidity Trends for Carbonyl & Related Compounds Substituent Effects

Chem 206

Electrons in 2S states "see" a greater effective nuclear charge than electrons in 2P states.
This becomes apparent when the radial probability functions for S and P-states are examined: The radial probability functions for the hydrogen atom S & P states are shown below.

Electronegativity e.g. Compare Carboxylic Acids vs. Ketones

H H C H C

O H O H

H C H C

O CH2H R

(H2O)

pKA = 4.8

pKA 19 pKA 26.5 - S-character of carbon hybridization sp3-orbitals 25% s-character sp2-orbitals 33% s-character sp-orbitals 50% s-character

(DMSO) pKA = 12.3 Hybridization p. . Remember:

Radial Probability

Radial Probability

Carboxylate ion more stabile than enolate because R C O more O electronegative than C O

O C CH2

100 %

100 %

1 S Orbital

2 S Orbital

2 S Orbital 2 P Orbital

3 S Orbital

3 P Orbital

Carbon Acids
R R H R H H RR H R R RR

S-states have greater radial penetration due to the nodal properties of the wave function. Electrons in s states see a higher nuclear charge. The above observation correctly implies that the stability of nonbonding electron pairs is directly proportional to the % of S-character in the doubly occupied orbital. Carbanions
CSP3 Least stable CSP2 CSP Most stable CSP2 CSP Least stable

Hybridzation Bond Angle pKa(DMSO)

sp 180 23

sp2 120 44

sp2 120 39

sp3 109 60

Carbenium ions

CSP3 Most stable

The above trends indicate that the greater the % of S-character at a given atom, the greater the electronegativity of that atom.

19-03-Acidity Concepts-3 10/27/03 12:55 PM

D. A. Evans

Acidity Trends for Carbonyl & Related Compounds


Substituent Effects

Chem 206

Hybridization vs Electronegativity There is a linear relationship between %S character & Pauling electronegativity
5

Alkyl Substituents on Localized Carbanions are Destabilizilng: Steric hinderance of anion solvation

NSP
4.5

Compare:
pKA (DMSO) S S S S

(JACS 1975, 97, 190) pKA (DMSO)

Pauling Electronegativity

H H Me H

N
3.5

PhSO2-CHH H
C SP

SP2

29 31

31.1

NSP3

PhSO2-CHMe H

38.3

C
2.5

SP2

CSP3

Heteroatom-Substituents: - 1st row elements of periodic table


pKA (DMSO)
25 30 35 40 45 50 55

p. .

2 20

% S-Character

PhSO2-CH-OCH3 H PhSO2-CH-OPh H PhSO2-CH-NMe3 H

30.7

There is a direct relationship between %S character & hydrocarbon acidity


60

Inductive Stabilization versus Lone Pair Repulsion (-I vs +M -Effect)

27.9 19.4
Inductive Stabilization

CH (56)
55
4

50

Pka of Carbon Acid

45

C H (44)
6 6

Heteroatom-Substituents: - 2nd row elements of periodic table Strong carbanion stabilizing effect
pKA (DMSO) PhSO2-CH-H H PhSO2-CH-SPh H pKA (DMSO) PhSO2-CH-SO2Ph H PhSO2-CH-PPh2 H

40

35

PhCC-H (29)
30

29 20.5

12.2 20.5

25 20 25 30 35 40 45 50 55

% S-Character

19-04-Acidity Concepts-4 10/27/03 12:56 PM

D. A. Evans

Acidity Trends for Carbonyl & Related Compounds


Conjugative Stabilization of Conjugate Base
H H C NO2 H H C C H
(JACS 1976, 98, 7498; JACS 1977, 99, 5633; JACS 1978, 100, 200).

Chem 206

Carbanion Stabilization by 2rdRow Atoms: SR, SO2R, PR3 etc


Me S CH3 Me O H3C S CH3 O S S H H O H3C S CH3

O C N O O C C

17.2 pKA (DMSO)

H O CH3 H H H C C N

18.2 (DMSO)
Ph Ph P CH3 Ph

31

31

35

26.5
CH3

H H

22.5

C C N H

31.5

The accepted explanation for carbanion stabilization in 3rd row elements is delocalization into vicinal antibonding orbitals

For efficient conjugative stabilization, rehybridization of carbanion orbital from nsp3 to np is required for efficient overlap with low-lying *-orbital of stabilizing group. However, the cost of rehybridization must be considered.

p. .

Cn

SX*

X C S
SX* (empty) Stereoelectronic Requirement for Carbanion Overlap: Enolization of Carbonyl Compounds
Stereoelectronic Requirements: The -C-H bond must be able to overlap with CO

Cn (filled)

This argument suggests a specific orientation requirement. This has been noted:
Anti (or syn) periplanar orientation of Carbanion-orbital and * orbital mandatory for efficient orbital overlap. H H Me Me He S S Ha He R

CO
CH3 O

Ha
base

Hc R

Hb O

R Hc

O Hb

Ha+

Rates for deprotonation with n-BuLi He : Ha = 8.6


(JACS 1974, 96, 1811) O H O

pKA 5.2 Ph3CH

pKA (DMSO) 31.5

S H

He : Ha = 30
Ha

(JACS 1978, 100, 200) O H O

C-H acidity not detectable

47.7
H

19-05-Acidity Concepts-5 10/24/01 8:13 AM

D. A. Evans
Phenol Acidity:
FG OH

Phenol Acidity: An Analysis of Resonance & Inductive Effects


O + H+

Chem 206

Is the benzene ring somehow special. i.e "larger resonance space." (1) Acetone enol:
acetone Me O Me Keq = 10
-8

G FG

acetone enol H 2C Me OH pKa = 10.9

acetone enolate H2C Me O + H+ (1)

This topic has a number of take-home lessons. Most importantly, is is a useful construct on which to discuss the role of FG's in influencing the acidity of this oxygen acid.

How does one analyze the impact of structure on pKa of a weak acid (pKa > 0) ? O The Approach:
For equilibria such as that presented above, analyze the effect of stabilizing (or destabilizing) interactions on the more energetic constituent which in this case is the conjugate base.
FG

The surprising facts is that the acetone enol has nearly the same pKa as phenol. Hence, the answer to the above question is no!

How important are inductive effects in the stabilization of C6H5O?


Consider the following general oxygen acid XOH where X can only stabilize the conjugate base through induction: pKa(H2O) X OH X OH X O + H+ 15.5 CH3 OH As the electronegativity of X increases 12.4 CF3CH2 OH the acidity of XOH increases. 7.5 Cl OH If you take the calculated electronegativity of an SP2 carbon (2.75) you can see that there is a linear correlation between the electronegativity of X and the pKa of XOH.

+ solvent(H+)

Energy

OH FG

Why is phenol so much more acidic than cyclohexanol?


OH O

This argument suggests that the acidity of acetone enol is largely due to inductive stabilization, not resonance.
H+ pKa (H2O) = 10
3.2

G
OH O

HOCl (7.5)

pKa (H2O) = 17
Electronegativity of X
2.8

Loudon (pg 730): "The enhanced acidity of phenol is due largely to stabilization of its conjugate base by resonance." O O O O

acetone enol (10.9) phenol (10.0)

2.6

2.4

from previous discussion, G298 = 1.4 Log10 Keq = 1.4 pKeq G (stab) = 1.4(Pkaphenol pKacyclohenanol) = 1.4(-7) = 9.8 kcal/mol
19-06-Phenol acidity 10/27/03 12:58 PM

2.2

HOH (15.7)
2 6 8 10 12 14 16

pKa of XOH

D. A. Evans

Weak Acids: Impact of Structure on Acidity


Case III: Carboxylic Acids vs Ketones:
O H C C OH H H H O H C C CH2H H

Chem 206

The General Reaction: Ionization of a weak acid (pKa > 0)


O R X H + solvent O + solvent(H+)

Variables:
X = O (carboxylic acid) X = NH (amide) Energy X = CH2 (Ketone/ester) R = CR3 R = OR R = NR2 O R

O X

pKa = 4.8
+ solvent(H )
+

pKa ~ 19

O more stabile than O enolate because R C R C O more O CH2 electronegative than C

Carboxylate ion

Case IV: Carboxylic Acids, Esters, Amides & Ketones:


R

G
R XH O Me C CH2H O EtO C CH2H Me2N O C CH2H O

O C CH2H O C CH2H

The Question: How does one analyze the impact of structure on pKa ? The Approach: For equilibria such as that presented above, analyze the effect of stabilizing (or destabilizing) interactions on the more energetic constituent which in this case is the conjugate base. Case I: Carboxylic Acids: Inductive Effects
O H C C OH H H O C C OH Cl Cl Cl Cl Carboxylate ion O stabilized by increased C C electron-withdrawing O Cl CCl3 group.

pKa ~ 26 The Analysis:

pKa ~ 30

pKa ~ 34

pKa > 34 < 40

In this series of compounds, there are two variables to consider: Inductive Effect: OEt > Me2N > H3C but (O?) Resonance Effect:

Cl

O C

pKa = 4.8

pKa = 0.6

Case II: Carboxylic Acids: Inductive Effects & Carbon Hybridization


O H C C C OH H H H H O H C C C OH O Carboxylate ion stabilized by increased HC C C O electron-withdrawing SP-hybridized carbon

The degree to which substituent X: CH2 CH2 "contributes" electron density into enolate represents a destabilizing interaction: Trend: O > Me2N > OEt

O C

Resonance donation dominates inductive electron withdrawal as indicated by the data. Substituents on the -carbon: Stabilization by either resonance, induction, or both is observed:
O Ph C CH2CH3 O Ph C CH2OCH3 O Ph C CH2Ph O Ph C CH2SPh

pKa = 4.9

pKa = 1.9

pKa = 24.4
19-07 Weak Acids/Gen anal 10/24/01 8:16 AM

pKa = 22.9

pKa = 17.7

pKa = 17.1

D. A. Evans

Acidity of Carboxylic Acids, Esters, & Lactones: Anomeric Effects Again?


Hyperconjugation: (Z) Conformer
O O R' O H H O H

Chem 206

Conformations: There are 2 planar conformations.


O O O O R' R

Let us now focus on the oxygen lone pair in the hybrid orbital lying in the sigma framework of the C=O plane. * CO
O R R O

(Z) Conformer

(E) Conformer

Specific Case: Formic Acid H

G = +2 kcal/mol

C R

In the (Z) conformation this lone pair is aligned to overlap with * CO.

The (E) conformation of both acids and esters is less stable by 2-3 kcal/mol. If this equilibrium were governed only by steric effects one would predict that the (E) conformation of formic acid would be more stable (H smaller than =O). Since this is not the case, there are electronic effects which must also be considered. These effects will be introduced shortly. Rotational Barriers: There is hindered rotation about the =COR bond. These resonance structures suggest hindered rotation about =COR bond. This is indeed observed:
O R O R' R O + O R'
Energy

(E) Conformer
R R O C O

In the (E) conformation this lone pair is aligned to overlap with * CR.

* CR
O

R O C
barrier ~ 10 kcal/mol

O R O R O R
G ~ 2-3 kcal/mol

Since * CO is a better acceptor than * CR (where R is a carbon substituent) it follows that the (Z) conformation is stabilized by this interaction.

O R

O R R O

Lone pair orientation & Impact on pKa (DMSO)


See Bordwell, J. Org. Chem. 1994, 59, 6456-6458
O CH3CH2 O Et O O Me O Meldrum's Acid O O O Me O O Me O Me O

Rotational barriers are ~ 10 kcal/mol This is a measure of the strength of the pi bond.

Lone Pair Conjugation: The oxygen lone pairs conjugate with the C=O.

pKa ~ 30

pKa = 25.2

pKa = 15.9

R
O

pKa = 7.3

C R

The filled oxygen p-orbital interacts with pi (and pi*) C=O to form a 3-centered 4-electron bonding system.
O Me N H O Et HN O O

Is this a dipole effect? See Bordwell Me


+

SP2 Hybridization

O O R

Me
+

O O

Oxygen Hybridization: Note that the alkyl oxygen is Sp2. Rehybridization is driven by system to optimize pi-bonding.

pKa = 24.5

pKa = 20.6

Houk, JACS 1988, 110, 1870 supports the dipole argument

E(rel) = 0

E(rel) = +3.8 kcal

19-08-Lone pair orientation 10/27/03 12:59 PM

D. A. Evans

Kinetic & Thermodynamic Acidity of Ketones

Chem 206

Kinetic Acidity: Rates of proton removal Consider enolization of the illustrated ketone under non-equilibrating conditions:
OLi HA O OLi Me

Kinetic & Equilibrium Ratios of Enolates Resulting from Enolization with LDA & Subsequent Equilibration

HB

Me

HB
LiNR2

HA

Me

HB
LiNR2

O
(99) (1) Me (10)

O
(90) Me (98)

O
(2) (34)

O
(66)

kB B

HB

kA K A

HB

Kinetic Ratios

B GB
Energy

Equilibrium Ratios

Kinetic Ratios

Equilibrium Ratios

GA
O
(13) H

O
(53)

(47) H

O C3H7
(16)

O CH3 C3H7 CH3

K B
LiNR2

(87)

p. .

(84)

(87)

(13)

Kinetic Ratios A
Reaction Coordinate Kinetic acidity refers to the rate of proton removal. e.g. kA vs kB. For example, in reading the above energy diagram you would say that HA has a lower kinetic acidity than HB. As such, the structure of the base (hindered vs unhindered) employed plays a role in determining the magnitude of kA and kB. For the case shown above, GA will increase more than GB as the base becomes more hindered since the proton HA resides in a more sterically hindered environment. The example shown below shows the high level of selectivity which may be achieved with the sterically hindered base lithium diisopropylamide (LDA). Me Me N Li Me Me H O OLi H THF 78 C H OLi Me

Equilibrium Ratios
O

Kinetic Ratios

Equilibrium Ratios

O Ph
(14)

(86)

CH3

Ph
(99)

(1)

CH3

Kinetic Ratios

Equilibrium Ratios

Note that alkyl substitution stabilizes the enolate (Why??). This effect

shows up in the equilibrium ratios shown above.

Me

Me H

Hence, enolization under "kinetic control with LDA allows you to produce the less-substituted enolate while subsequent equilibration by simply heating the enolate mixture allows equilibration to the more substituted enolate.

LDA

Kinetic Ratio 99 : 1 Equilibrium Ratio 10 : 90

19-09-kinetic Acidity-1 10/27/03 1:00 PM

Evans, Annis
Kinetic Acidity

Kinetic Acidity: Carbon versus Oxygen Acids

Chem 206

Kinetic Acidity vs. Leaving Group Ability: E1cb Elimination Reactions Stirling, Chem. Commun. 1975, 940
O Ph S O O base O Ph S O O Ph S O O rds O Ph S O O

Observation: The thermodynamic acidities of phenol and nitromethane are both approximately 10; however, using a common base, phenol is deprotonated 10+6 times as fast. pKa(H2O) ~10
O H

Base rel rate: 10+6

pKa(H2O) ~10

H3C N

Base rel rate: 1

O H2 C N O

OPh

Ph S O

+ PPh3

Ph S O

CN

krel = 1 pKa HX 10

krel = 10+4 0

krel = <108 9.5

Proton transfers from C-H Bonds are slow. Why??? p. . Most carbon acids are stabilized by resonance. Hence significant structural reorganization must accompany deprotonation.

The greater the structural reorganization of the leaving group during E1cb elimination, the slower the rate of elimination.

O H

Base

Protonation of Conjugate bases


O H N H O H3O+ O-H electron density is still here. OLi O H N H O OH

Kinetic product

O-H electron density is here.

OH

H3O+ Keq ~ 10+5 H

H H N H

O O

Base

H N H

O O

Kinetic product

C-H electron density is here.

electron density now resides here, and nuclei have moved to accomodate rehybridization.

Jack Hine: Least Motion Principle (Adv. Phys. Org. Chem. 1977, 15, 1) Lowry & Richardson, 3rd Edition, pp 205-206 Those elementary reactions that involve the least change in atomic posiitons will be favored

The greater the structural reorganization during deprotonation, the lower the kinetic acidity

19-10-kinetic Acidity-2 10/27/03 1:00 PM

D. A. Evans

Carbonyl and Azomethine Electrophiles-1


Additional Reading Material Provided

Chem 206

http://www.courses.fas.harvard.edu/~chem206/

Additions to 5- & 6-Membered oxocarbenium Ions:

Chemistry 206 Advanced Organic Chemistry


Lecture Number 20

Woerpel etal. JACS 1999, 121, 12208 (Handout) Woerpel etal. JACS 2000, 122, 168 (Handout) Woerpel etal. JACS 2003, 125, ASAP (Handout) "From Crystal Statics to Chemical Dynamics", Accounts Chem. Research 1983, 16, 153. (Electronic Handout) "Theoretical Interpretation of 1,2-Asymmetric Induction. The Importance of Antiperiplanarity", N. T. Anh, O. Eisenstein Nouv. J. Chem. 1977, 1, 61-70. (Handout) "Around and Beyond Cram's Rule" A. Mengel & O. Reiser, Chem Rev. 1999, 99, 1191-1223 (Electronic Handout)

Carbonyl and Azomethine Electrophiles-1


R C O R R R C O R R R C N R R R C N R R

Relevant Dunitz Articles Reactivity Trends C=X Stereoelectronic Effects Carbonyl Addition: Theoretical Models The Felkin-Anh-Eisenstein Model for C=O Addition Diastereoselective Ketone Reduction "Geometrical Reaction Coordinates. II. Nucleophilic Addition to a Carbonyl Group", JACS 1973, 95, 5065. "Stereochemistry of Reaction Paths at Carbonyl Centers", Tetrahedron 1974, 30, 1563 "From Crystal Statics to Chemical Dynamics", Accounts Chem. Research 1983, 16, 153. (Electronic Handout) "Stereochemistry of Reaction Paths as Determined from Crystal Structure Data. A Relationship Between Structure and Energy.", Burgi, H.-B. Angew. Chem., Int. Ed. Engl. 1975, 14, 460.

Reading Assignment for this Week: Carey & Sundberg: Part A; Chapter 8 Reactions of Carbonyl Compounds Carey & Sundberg: Part B; Chapter 2 Reactions of Carbon Nucleophiles with Carbonyl Compounds Carey & Sundberg: Part B; Chapter 5 Reduction of Carbonyl & Other Functional Groups
Friday, October 31, 2003

D. A. Evans
20-00-Cover Page 10/30/03 11:44 AM

Chemistry Reviews Issue on Diastereoselection in C=O Addition


Chem Rev. 1999, 99, (5), 1069-1480
Mengel, A. and O. Reiser, Around and beyond Cram's rule. Chem. Rev. 1999, 1191-1223. Reetz, M. T., Synthesis and diastereoselective reactions of N,N-dibenzylamino aldehydes and related compounds. Chem. Rev. 1999,1121-1162. Dannenberg, J. J., Using perturbation and frontier molecular orbital theory to predict diastereofacial selectivity. Chem. Rev. 1999, 1225-1241. Tomoda, S., The exterior frontier orbital extension model. Chem. Rev. 1999,1243-1263. Cieplak, A. S., Inductive and resonance effects of substituents on pi-face selection. Chem. Rev. 1999,1265-1336. Ohwada, T., Orbital-controlled stereoselections in sterically unbiased cyclic systems. Chem. Rev. 1999,1337-1375. Gung, B. W., Structure distortions in heteroatom-substituted cyclohexanones, adamantanones, and adamantanes: Origin of diastereofacial selectivity. Chem. Rev. 1999,1377-1386. Kaselj, M., W. S. Chung, et al., Face selection in addition and elimination in sterically unbiased systems. Chem. Rev. 1999, 1387-1413. Adcock, W. and N. A. Trout, Nature of the electronic factor governing diastereofacial selectivity in some reactions of rigid saturated model substrates. Chem. Rev. 1999,1415-1435. Mehta, G. and J. Chandrasekhar, Electronic control of facial selection in additions to sterically unbiased ketones and olefins. Chem. Rev. 1999,1437-1467. Wipf, P. and J. K. Jung, Nucleophilic additions to 4,4-disubstituted 2,5-cyclohexadienones: Can dipole effects control facial selectivity?. Chem. Rev. 1999,1469-1480.

20-000-chem reviews 1999 10/29/03 4:15 PM

D. A. Evans
The Set of Functional Groups:
R C O R Aldehyde Ketone R C O R

C=X Electrophiles: Carbonyls, Imines & Their Conjugate Acids

Chem 206

Stereoelectronic Considerations for C=O Addition


R C N R R C N R

LUMO is CO; HOMO Provided by Nu: CO CO

R R R Aldimine Iminium ion Ketimine (Imine) These functional groups are among the most versatile sources of electrophilic carbon in both synthesis and biosynthesis. The ensuing discussion is aimed at providing a more advanced discussion of this topic.

R Oxocarbenium ion

R C R

LUMO

R
C=X Polarization

R C O R C O
Dunitz-Burgi trajectory HOMO (Nu) CO Nu NuC ~107 The forming bond

R
p. .

Partial Charge: As the familiar polar resonance structure above indicates, the carbonyl carbon supports a partial positive charge due to the polarization of the sigma and pi system by the more electronegative heteroatom. The partial charges for this family of functional groups derived from molecular orbital calclulations (ab initio, 3-21(G)*, HF) are illustrated below: R R R R R R R C R + 0.33 N R + 0.51 C O R + 0.54 C N R C O R + 0.61 (R = H) + 0.63 (R = Me)

R R

electrophilic reactivity Proton Activation of C=X Functional groups


R R C R O + 0.51 R C O + 0.61 H

What about C=O vs C=O-R(+)?

HA

R C O

A
R

C R

The electrophilic potential of the C=O FG may be greatly increased by either Lewis acid coordination of by protonation. The magnitide of this increase in reactivity is ~ 10+6. Among the weakest Bronsted acids that may be used for C=O actilvation (ketalization) is pyridinium ion (pKa = 5). Hence, the Keq below, while quite low, is still functional. R R H

+
N H R

O R

O N

Keq ~ 10-11

The LUMO coefficient on carbon for B will be considerably larger than for A. Does this mean that there is a lower constraint on the approach angle for the attacking nucleophile? There is no experimental proof for this question; however, it is worthy of consideration What was the basis for the Dunitz-Burgi analysis?

pka = +5

pka = 6

20-01-Introduction 10/30/03 11:46 AM

D. A. Evans
Relevant Dunitz Articles

The Dunitz-Burgi Trajectory for C=O Addition

Chem 206

"Geometrical Reaction Coordinates. II. Nucleophilic Addition to a Carbonyl Group", JACS 1973, 95, 5065. "Stereochemistry of Reaction Paths at Carbonyl Centers", Tetrahedron 1974, 30, 1563 "From Crystal Statics to Chemical Dynamics", Accounts Chem. Research 1983, 16, 153. "Stereochemistry of Reaction Paths as Determined from Crystal Structure Data. A Relationship Between Structure and Energy.", Burgi, H.-B. Angew. Chem., Int. Ed. Engl. 1975, 14, 460. Dunitz Method of Analysis A series of organic structures containing both C=O and Nu FG's disposed in a geometry for mutual interaction were designed. These structures positioned the interacting FGs an increasingly closer distances. The X-ray structures of these structures were determined to ascertain the direction of C=O distortion. The two families of structures that were evaluated are shown below.
1,8-Disubstituted Naphthalenes. Substituents located at these positions are strongly interacting as illustrated by the MM2 minimized di-methyl-naphthalene structure shown below. In this structure (A), at 2.56 the C=O is starting to pyramidalize Cyclic aminoketones. Medium-ring ketones of various ring sizes were analyzed for the interaction of amine an C=O FGs. One example is shown below. 2.56

p. .

Nu

O 2.29

Dunitz, Helv. Chem. Acta 1978, 61, 2783

Me N

Me

Me

~ 7.6 kcal/moll Lecture 5

Me2N

Me

MeO

Me

Analysis of distortion of C=O in this and related structures formed the basis of the 107 attack angle. This value should be taken as approximate.

A (shown)

Sekirkine Birnbaum JACS 1974, 96 6165

20-02-Dunitz-Burgi 10/29/03 4:38 PM

D. A. Evans

The Dunitz-Burgi Trajectory for C=O Addition

Chem 206

Cyclic aminoketones. Medium-ring ketones of various ring sizes were analyzed for the interaction of amine an C=O FGs. Two examples are shown below.
Sekirkine Birnbaum JACS 1974, 96 6165

Should these crystallographic date be relevant to the addition to complexed C=O & Iminium Ions?
R C R N R R R C O R R C N R R R R C O R

+ 0.33
2.46

+ 0.51

+ 0.54

+ 0.61 (R = H) + 0.63 (R = Me)

(antibonding)

C
p. .

Energy

R R

C O (bonding) C O

(antibonding)

C R
2.69

Energy

C O (bonding) C O

Nu

DB angel ~107

DB angel >107 ??

Nu R R C O

R R
20-03-Dunitz-Burgi-2 10/29/03 4:39 PM

D. A. Evans
Pivotal Articles

Stereoelectronic Effects in the Addition to Iminium and Oxo-carbenium Ions


An early example from Eliel; JACS 1969, 91, 536
H Me Me OMe O O H H

Chem 206

R. V. Stevens in "Strategies and Tactics in Organic Synthesis", Vol. 1. On the Stereochemistry of Nucleophilic Additions to Tetrahydropyridinium Salts: a Powerful Heuristic Principle for the Stereorationale Design of Alkaloid Synthesis.; Lindberg, T., Ed.; Academic Press, 1984; Eliel etal. , JACS 1969, 91, 536 Kishi etal. , JACS 1982, 104, 4976-8

H H O O H Me Me

H O O

Ph H

PhMgBr

Me Me

dioxolenium ion

trans : cis 95:5 (95%)

The Proposal for Oxo-carbenium Ions (Eliel, Kishi)


H Nu H R H C O Path A R O Nu

Eliel was the first to attibute stereoelectronic factors to the addition of nucleophiles to cyclic oxo-carbenium ions.

Kishi Examples; JACS 1982, 104, 4976-8


OBn BnO H H O Nu R O Nu BnO OBn OBn Et3SiH BF3OEt2 OH N R O CH2OBn BnO H PMBO O CH2OBn stereoselection 10:1 (55%) OBn O OBn CH2OBn OBn SiMe3 BF3OEt2 BnO H OBn O OBn CH2OBn

H kinetic product conformations Path B R H

p. .
Nu

The Proposal for Iminium Ions (Stevens)


H Nu H R H C N R Nu Nu Path A R Nu

H kinetic product conformations Path B R H N

stereoselection 10:1 (55%) Chair-aixal attack on oxo-carbenium ion occurs for both carbon and hydride nucleophiles

H H R R N Nu R

Iminium Ions (Stevens) cited reference

NaCNBH3
N Me C 4H 9 Me N C 4H 9 only one stereoisomer

It was proposed that chair-axial addition would be preferred as a consequence of the intervention of a transition state anomeric effect (Path A). Attack through Path B would necessitate the generation of the twist-boat kinetic product conformation thus destabilizing attack from the equatorial diastereoface. While Stevens espoused this concept for iminium ions in the late 70's, his untimely death at the age of 42 significantly delayed his cited publication.

n-PrMgBr
N C 4H9 N n-Pr C 4H 9

20-04-cyclic onium addns-1 10/29/03 4:40 PM

D. A. Evans

Stereoelectronic Effects in the Addition to Iminium and Oxo-carbenium Ions


H O Me H C O SiMe3 Me BF3OEt2 OAc Me H

Chem 206
Allyl O H trans:cis 99:1 (69%) BnO H Allyl

5-Membered oxocarbenium Ions: Woerpel etal. JACS 1999, 121, 12208.


O OAc SnBr4 Me O O

Me

Me SiMe3

stereoselection 99:1 O O BnO BF3OEt2 OAc

BnO H H C O SiMe3

OAc SnBr4 BnO

O H cis:trans 89:11(75%) Allyl

BnO

BnO

stereoselection >95:5

BF3OEt2 BnO OAc H H

These cases provide dramatic evidence for the importance of electrostatic effects in controlling face selecticity. OBn

BnO H

p. .

6-Membered oxocarbenium Ions: Woerpel etal. JACS 2000, 122, 168.


O OAc BF3OEt2 Me Me O C H SiMe3 O OAc BF3OEt2 BnO H O C H H OBn OBn O H Allyl trans:cis 99:1 (75%) O AlCl3 H BnO H O Allyl N N H 2C Cl Cl Cl Tet. Lett. 1988, 29, 6593 Cl OSiR3 93 : 7 H H H Me H O H Allyl cis:trans 94:6 (74%)

H cis:trans 83:17(84%)

These cases provide dramatic evidence for the importance of electrostatic effects in controlling face selecticity.

Are the preceding addition reactions somehow related to the apparently contrasteric reactions shown below??
O R3SiO EtO OSiR3 HgI2 EtO2C >94 : 6 O OSiR3 JOC 1991, 56, 387 OSiR3

This analysis presumes that only pseudo-chair transition states need be considered.

OSiR3 H H N

Woerpel's model states that axial attack from the most stable chair conformer predicts the major product.

N exclusive adduct

20-05-cyclic onium addns-2 10/26/01 8:17 AM

D. A. Evans

Diastereoselective Oxocarbenium Ion Additions in the Phorboxazole Synthesis

Chem 206

Phorboxazole B
Evans, Fitch, Smith, Cee, JACS 2000, 122, 10033 HO O N Br B HSi Me
H

B: The C-22 Reduction


OH B H O
Me

Et3SiH H OTPS Me Et3SiH BF3OEt2 R O R H


Me

20 13

RO2C O A H H H MeO O
4

H OTPS Me H 91%

H H H

C
9

N CH2 Me O

Me

> 95:5 Diastereoselection


H R O
Me

O
Me

H O
HO

OH

N O

Me

H OTPS Me

p. .

MeO

Stereochemical analogies: Kishi et. al.: JACS 1982, 104, 4976-8

N Me O

Me

A: The C-11 Reduction


TIPSO R H H
13

A O OH H R
9

TIPSO Et3SiH BF3OEt2


H

13

C: The C-9 CC Bond Construction


OTMS C OAc H
4

R H H

O R Et3SiH H 9 O BnOCH2 O Me O

OTMS Me H H O O

CH2

TMSOTf

TIPSO

13

O BnOCH2

>95:5 Diastereoselection

R H H

O H H O R
9

O Me
H

C
9

O O

89%

CH2

Diastereoselection 89:11

H O BnOCH2

20-06-phorboxazole cases 10/25/01 5:06 PM

D. A. Evans

Carbonyl Addition Reactions: Transition State Geometry


Do these results relate to "real" reactions? Yes!
R' C O
+

Chem 206
O ZnR C R

4- vs 6-Membered Transition Structures for C=O Addition Consider carbonyl hydration: H2C=O
H O H C H H O H + HOH H O H H C O O H H

R Zn R

slow

R' H

n HOH

H2C(OH)2

(n-1) HOH

Observation: catalytic amounts of ZnI2 dramatically catalyze addition process.


R Zn R I Zn I C O

T1

bimetallic transition state

T2

H R

+42.2

4 Versus 6Center Transition States for Boron 4-Centered +6.7


L B Me L
R2C=O

L Me R C R B O L R R

Me C OBL2

H2C=O + HOH H2C(OH)2

H2C=O + 2 HOH

disfavored : rxn does not proceed!)


H2C(OH)2 +HOH

Schowen J. Am. Chem. Soc. 105, 31, (1983).

6Centered
CH CH2 L
R2C=O

L L R R C OBL2

Overall Process: The valure of the proton shuttle


H C O H H O H H O H H H C O H H O H H O H H O

H H C O H H O H H H C O H L H O fast H O

B L

H 2C R R C O

favored 6Centered
R B L H R R
R2C=O

R C H R R C O CH2 B

L L R R

R H H 2C C R OBL2

Transiton structure T2 ~40 kcal/mol more stable than transition structure T1.

favored

20-07-C=O addn TS-1 10/30/03 12:25 PM

D. A. Evans

Carbonyl Addition Reactions: Transition State Geometry


The Schlenk Equilibrium
R Mg S Me R C Br Br Mg R S Me Br Mg Br R Mg R S Me Br Mg

Chem 206
R Mg R S Br

Carbonyl Addition: 4 Versus 6Center Transition States for Aluminum


L Me Al O L L Al Me R R L Al C O L

4Centered disfavored
L

R C R

Solution structure of RMgBr is in dynamic eqkuilibrium through Schlenck equilib The Bimetallic (Binuclear) Mechanism for C=O Addition
OAlL2 Recent theoretical study: Yamazaki & Yambe, J. Org. Chem. 2002, 67, 9346 Me Mg R C O R
+ Br Br

n
L

R Al Me R C O

Me Me

S fast R C Me R

Me Mg Br Br O S Mg

S slow R R Me C

S Mg Br O Mg

Br

6Centered favored

Mg S

The Bimetallic Transition States are preferred


Me R C R L Al L O Al Me 4-Centered L Me Bicyclic TS R C R Al O L L Al Me Me Me R C R L Al O Me L L Me R C R Al O L L Al Me Me R C R L L Al O L Al Me 6-Centered Chair L L Al Me R Swain JACS 1951, 73, 870 Ashby JACS 1974, 89, 1967 Me

Me

Me L Bicyclic TS Br Mg O Br R L Mg Me

S Me R Me

Me 6-Centered Boat

Yamazaki TS

The Mononuclear) Mechanism is now in disfavor


S C O
+

R R2 fast R C O Mg S Br R2
+ solvent (S)

Grignard Reagents:
R

C O

+ R2 Mg

Br

R R

O MgBr C R2

Mg Br

R C O slow Br R C O S Br Mg R2

R R O MgBr C R2

The molecularity and transition structure for this reaction have not been carefully elucidated. The fact that the Grignard reagent is not a single species in solution greatly complicates the kinetic analysis.

R S

R2 Mg

20-08-C=O addn TS-2 10/30/03 1:49 PM

D. A. Evans

The Evolution of Models for Carbonyl Addition

Chem 206

Mengel, A. and O. Reiser, Around and Beyond Cram's Rule. Chem. Rev. 1999, 1191-1223

Fischer

Cram

Cornforth

Felkin

Anh/Eisenstein

Cieplak

Tomoda

Humor provided by Sarah Siska


20-09 C=O addition Models 10/30/03 2:17 PM

D. A. Evans
Dauben, JACS 1956, 78, 2579 O Me3C H
0 10 20 30 40

Evolution of a Model for C=O Addition


Assumptions in Felkin Model:
OH OH Me C 3 H % Axial Diastereomer
50 60 70 80 90 100

Chem 206

Product Development & Steric Approach Control:


H [H] Me3C

Transition states are all reactant-like rather than product-like.


H

Torsional strain considerations are dominant. Staggered TS conformations preferred The principal steric interactions are between Nu & R.
RL R C O predicted to be favored TS RM O H RL C R RM
destabilizing interaction

DIBAL-H 72:28 NaBH4 79:21 LiAlH(Ot-Bu)3 92:8 LiAlH4 93:7

L-Selectride 8:92 K-Selectride 3:97 Me H B C CH2Me H M+

Nu:

Nu:
RL

The flaw in the Felkin model: A problem with aldehydes!!


3

RL H H C O RM
destabilizing interaction

Observation: Increasingly bulky hydride reagents prefer to attack from the equatorial C=O face. Assumption: Hindered reagents react through more highly developed transition states than unhindered reagents

O H

H RM

predicted to be favored TS wrong prediction

Nu:

Nu:

Carbonyl Addition: Evolution of Acyclic Models


H RL RM O

Stereoelectronic Effect: Nu:


R RL OH Nu RM favored RL R OH Nu

The HOMO-LUMO interaction dictates the following reaction geometry: CO


LUMO

RM disfavored

Nu
~107

RL R

R
RL R C RM H OR M R H RL C O RM

C R

HOMO

Nu: CO

Nu:
H

C O

The Dunitz-Brgi Angle

Nu:
Karabatsos JACS 1967, 89, 1367

Nu:
Felkin TL. 1968, 2199-2208

Cram JACS 1952, 74, 5828

attack angle greater than 90 ; estimates place it in the 100110 range Burgi, Dunitz, Acc. Chem. Res. 1983, 16, 153-161

20-10-C=O addn/traj 10/30/03 1:52 PM

D. A. Evans

The Felkin-Anh Eisenstein Model


RL O RM
destabilizing interaction

Chem 206

The flaw in the Felkin model: A problem with aldehydes!!


RL H H C O H

Houk:
H RM
predicted to be favored TS wrong prediction

"The tendency for the staggering of partially formed vicinal bonds is greater than for fully formed bonds" Lecture7

Lets begin with ground state effects: Ethane Rotational Barrier

Nu:

Nu:

Anh & Eisenstein Noveau J. Chim. 1977, 1, 61-70 Anh Topics in Current Chemistry. 1980, No 88, 146-162 RL

G =+3 kcal mol -1

RL H OH C H Nu RM

Nu: Felkin
H RL RM RL

H H

O RM

One explanation for the rotational barrier in ethane is that better overlap is achieved in the staggered conformation than in the eclipsed conformation.

In the staggered conformation there are 3 anti-periplanar CH Bonds


H

Nu:
O

favored H
C
CH HOMO CH * CH LUMO

RL HO C H Nu RM H

C H

Nu: anti-Felkin

C
CH

O H

H RM

Nu: New Additions to Felkin Model:

disfavored In the eclipsed conformation there are 3 syn-periplanar CH Bonds


H C H C
CH HOMO

Y
C

Dunitz-Brgi C=ONu orientation applied to Felkin model. The antiperiplanar effect: Hyperconjugative interactions between C-RL which will lower *C=O will stablize the transition state. Theoretical Support for Staggered Transition states (Lecture 7)
(Read this) Houk, JACS 1982, 104, 7162-6 (Read this) Houk, Science 1986, 231, 1108-17

X
C

CH * CH LUMO

CH

Following this argument one might conclude that: The staggered conformer has a better orbital match between bonding and antibonding states. The staggered conformer can form more delocalized molecular orbitals.

20-11-C=O Felkin/Anh 10/30/03 1:55 PM

D. A. Evans

The Felkin-Anh Eisenstein Model


Hierarchy of Donor & Acceptor States

Chem 206

The tendency for the staggering of partially formed vicinal bonds is greater than for fully formed bonds Ground State
C

The following trends are made on the basis of comparing the bonding and antibonding states for the molecule CH3-X where X = C, N, O, F, and H. -anti-bonding States: (CX)
CH3 H * CRL LUMO CH3 CH3 CH3 NH2 CH3 OH

RL C Nu C

RL C

Transition State

X
C CNu HOMO C

* CRL LUMO

Nu C CNu HOMO

X
C

Nu

Nu

Increasing -acceptor capacity

CH3 F best acceptor

CRL CNu CNu

CRL

-bonding States: (CX)


CH3 CH3 CH3 H

Under debate

CH3 NH2 CH3 OH

Best acceptor * orbital is oriented anti periplanar to forming bond


RL H H Nu C O RM

decreasing -donor capacity

CH3 F poorest donor

RL

The following are trends for the energy levels of nonbonding states of several common molecules. The trend was established by photoelectron spectroscopy. Nonbonding States

Nu

C Theoretical support:
Padden-Row, Chem. Commun. 1990, 456; ibid 1991, 327 Houk, J. Am. Chem. Soc. 1991, 113, 5018 Frenking & Reetz, Tetrahedron 1991, 47, 8091

RL

H3P: H2S: H3N: H2O: HCl: decreasing donor capacity


poorest donor

20-12-C=O Felkin/Anh-2 10/29/03 4:17 PM

D. A. Evans
Addition of Enolate & Enol Nucleophiles
RL

The Felkin-Anh Eisenstein Model: Verification


This trend carries over to organometallic reagents as well

Chem 206

H OH RL RM Nu H H
Cram

Nu O Me Me H H H
Reagent Ratio 1:1 5:1 + Anti-Felkin Isomer

Nu: Felkin
O RL RM H H C O RM

Me Me H

OH

Nu:
H RL

(Felkin) favored

ClMg C CEt Li Me Me

OH RL RM Nu

Nu: anti-Felkin

C. Djerassi & Co-workers, J. Org, Chem. 1979, 44, 3374.

O H

H RM

H O Me

Nu: Trend-1:

disfavored

RTi (OiProp)3

OH R Me
R-Titanium R = Me R = n-Bu Ratio >90 : 10 >90 : 10 + Anti-Felkin Isomer

For Li enolates, increased steric hindrance at enolate carbon results in enhanced selectivity OLi H R O Me Me
Enolate (R) R = Me R = OtBu Ratio 3:1 4:1

M. Reetz & Co-workers, Angew Chemie Int. Ed.. 1982, 21, 135.

OH

O R
+ Anti-Felkin Isomer

(RMgX gives Ca 3:1 ratios)

Trend-2:

Lewis acid catalyzed reactions are more diastereoselective


OSiMe2tBu H R2 O Me
BF3-Et2O -78 C

L. Flippin & Co-workers, Tetrahedron Lett.. 1985, 26, 973.

OH R1 Me
Ratio

O
+ Anti-Felkin Isomer

OLi Me H R Me
L. Flippin & Co-workers, Tetrahedron Lett.. 1985, 26, 973.

R1

R2

OMe R

Me O

OH

O OMe
+ Anti-Felkin Isomer

Ketone (R1) R = Ph R = Ph R = Ph R = Ph R = c-C6H11

Enolate (R2) R = Me R = t-Bu R = OMe R = Ot-Bu R = Ot-Bu

Ratio Li enolate 3:1

MeMe Me
Ketone (R) R = Ph R = c-C6H11 Ratio >200 : 1 9:1

10 : 1 24 : 1 15 : 1 36 : 1 16 : 1

4:1

C. Heathcock & L. Flippin J. Am. Chem. Soc. 1983, 105, 1667.

20-13-C=O Felkin/Anh-verify 10/29/03 4:32 PM

D. A. Evans

The Felkin-Anh Eisenstein Model: Ketone Reduction


Addition of Hydride Nucleophiles
RL

Chem 206

O Me OH

Me H [H]

HO Me H

Me H
+ Anti-Felkin Isomer

Hydride Felkin
O RL RM R H C O RM

RL RM

H H

H
Cram Reagent Ratio TS

Nu:
R RL

(Felkin) favored

M. M. Midland & Co-workers, J. Am. Chem. Soc. 1983, 105, 3725.

OH RL RM R

Hydride anti-Felkin

O H

R RM

Li+HB(sec-Bu)3 NaBH4 LiAlH4 HB(Sia)2

54 : 1 5:1 3:1 1 : 10

Felkin Felkin Felkin Anti-Felkin

Nu:
R H2 C Me O (CH2)2Ph
MH 78 C

disfavored

Note: Borane reducing agents do not follow the normal trend

R H2C Me

OH
+ Anti-Felkin Isomer

Transition States for C=O-Borane Reductions


RL

(CH2)2Ph

R2BH
Reagent
+

OH RL RM R

R H

C H

O RM B R R RL

Ketone (R) G. Tsuchihashi & Co-workers, Tetrahedron Lett. 1984, 25, 2479.

Ratio

Felkin
O RL RM R

R=H R=H R = Me R = Me

Li HB (sec-Bu)3 DIBAL Li+HB(sec-Bu)3 DIBAL OH

96 : 4 47 : 53 >99 : 1 88 : 12 OH Me

(Felkin) disavored

OH RL RM R

O
MH

R2BH anti-Felkin

Me Me
M. M. Midland & Co-workers, J. Am. Chem. Soc. 1983, 105, 3725.

Me Me
Reagent Ratio

Me
TS

O H R B R

C H

R RM

favored

Li+HB(sec-Bu)3 HB(Sia)2

22 : 1 1:4

Felkin Anti-Felkin

Nonspherical nucleophiles are unreliable in the Felkin Analysis Exercise: Draw the analogous bis(R2BH)2 transition structures

20-14-C=O Cram reductions 10/30/03 1:58 PM

D. A. Evans

The Felkin-Anh Eisenstein Model: A Breakdown

Chem 206

Are there cases not handled by the Anh-Eisenstein Model?


Anh-Eisenstein: "Best acceptor * orbital is oriented anti periplanar to forming bond." CSP3CSP2 is lower in energy than CSP3CSP3 bond.

Felkin-Anh analysis predicts B for R = electronegative substituent.


O Me-Li R R R HO Me Me OH

Ph
H H Nu C O RM H H Nu

Cyclohexyl
C O RM

A (R) Substituent G. Mehta, JACS 1990, 112, 6140


O C OMe

B
(Felkin-Anh Prediction)

A/B Ratio >90:10 34:66 27:73 17:83

C C

Cyclohexyl
Felkin-Anh analysis predicts the wrong product!

Ph
C

CH2OMe CH=CH2 CH2CH3

Cyclohexyl

C Case I:
O

Ph
O HO Me-Li Me Me OH NaBH4 CO2Me CO2Me CO2Me CO2Me HO NaBH4 H CO2Me CO2Me OH HO H H OH

CO2Me CO2Me M Nu

CO2Me CO2Me

CO2Me CO2Me

70: 30
H

Electronegative -CO2Me substituent will stabilize both CC bonding & antibonding states CO2Me CO2Me

Et Et Et

Et

Et

Felkin-Anh analysis predicts B

NaBH4 MeLi

39: 61 34: 66

Et

20-15-C=O cases not handled 10/30/03 2:02 PM

G. Mehta, Chem. Commun. 1992, 1711-2: "These results can be reconciled in terms of the Cieplak model."

D. A. Evans

The Felkin-Anh Eisenstein Model: A Breakdown Cieplak Model for C=O Addition

Chem 206

Case II: The Le Noble Examples Le Noble, JACS 1992, 114, 1916
O NaBH4 Me N i-PrOH, 25 C Me N Me N
Felkin-Anh Prediction

OH

HO

Cieplak, JACS 1981,103, 4540; Cieplak/Johnson, JACS 1989, 111, 8447

Point A: TS is stabilized by antiperiplanar allylic bond, but.... Point B: Nature of the stabilizing secondary orbital interactions differ: C

X R H Nu C O R

Ratio, 95:5

Pyramidally distorted C=O ruled out from inspection of X-ray structures.


O
NaBH4 i-PrOH, 25 C

OH

X
C

Nu
C

+ Syn Isomer R (R) R=F R = CO2Me Anti:Syn Ratio 62:38 61:39 45:55 43:57

X
C

Nu
Cieplak

Felkin Anh C C

Nu
C

Le Noble, J. Org. Chem. 1989, 54, 3836

R = SiMe3 R = OH

Point C: CX Electron donating ability follows the order: CH > CC > CN > CO
H
(Houk disputes the ordering of CH, CC)

NaBH4 MeOH, 0 C

+ Syn Isomer OH (R) R = NO2 Anti:Syn Ratio 79:21 63:37 43:57 36:64

Point D:

Importance of torsional effects (Felkin, Anh, Houk, Padden-Row) disputed.

"Structures are stabilized by stabilizing their highest energy filled states. This is one of the fundamendal assumptions in frontier molecular orbital theory." The Cieplak hypothesis is nonsense." "Just because a hypothesis correlates a set of observations doesn't make that hypothesis correct." The management

Halterman, JACS 1990, 112, 6690

R = Cl R = OMe R = NH2

20-16-C=O cases not handled 10/30/00 8:06 AM

Quotes for the Day

"Every generation of scientists starts where the previous generation left off, and the most advanced discoveries of one age constitute elementary axioms of the next." Aldous Huxley

It is a capital mistake to theorise before one has data. Insensibly one begins to twist facts to suit theories, instead of theories to suit facts. (Sherlock Holmes, A Scandal in Bohemia)

Quotes-1 11/3/03 9:18 AM

D. A. Evans

Carbonyl and Azomethine Electrophiles-2

Chem 206

http://www.courses.fas.harvard.edu/~chem206/

1. "Theoretical Interpretation of 1,2-Asymmetric Induction. The Importance of Antiperiplanarity", N. T. Anh, O. Eisenstein Nouv. J. Chem. 1977, 1, 61-70. (pdf) 2. Structural, mechanistic, and theoretical aspects of chelation controlled carbonyl addition reactions.Reetz, Acc. Chem. Res. 1993 26: 462. (pdf) 3. "A Stereochemical Model for Merged 1,2- and 1,3-Asymmetric Induction in Diastereoselective Mukaiyama Aldol Addition Reactions and Related Processes." Evans, et. al. JACS 1996, 118, 4322-4343. (pdf) 4. The Exceptional Chelating Ability of Dimethylaluminum Chloride and Methylaluminum Dichloride. The Merged Stereochemical Impact of - and Stereocenters in Chelate-Controlled Carbonyl Addition Reactions with Enolsilane and Hydride Nucleophiles. Evans, Allison,Yang, Masse, 2001, 123, 10840-10852. (pdf)
R

Chemistry 206 Advanced Organic Chemistry


Lecture Number 21

Carbonyl and Azomethine Electrophiles-2


R C R O R R C O R R R C N R R R C N R

Me2AlCl is the most powerful chelating Lewis acid yet documented


Me O R N Bn O O
2 Me2 AlCl

Me Al N O O

1+ Me2 AlCl2

O R

(1)

s s s s

Breakdown in the Felkin-Anh Model Cyclohexanone Revisited Diastereoselective Additions to Cyclic Ketones Chelate Controlled Carbonyl Additions

Bn

s Reading Assignment for this Week: Carey & Sundberg: Part A; Chapter 8 Reactions of Carbonyl Compounds Carey & Sundberg: Part B; Chapter 2 Reactions of Carbon Nucleophiles with Carbonyl Compounds Carey & Sundberg: Part B; Chapter 5 Reduction of Carbonyl & Other Functional Groups
Monday, November 3, 2003

"Asymmetric Diels-Alder Cycloaddition Reactions with Chiral ,-Unsaturated-N Acyloxazolidinones". Evans, D. A.; Chapman, K. T.; Bisaha, J. J. Am. Chem. Soc. 1988, 110, 1238-1256.

Syn Diastereomer:
O H OP R R A O H R B OP R O H R Me2AlCl H R M O M

&
O P

Centers Reinforcing
Nu

OH Nu R

OP
(2)

Me2AlCl

P
Nu

OH Nu R

OP
(3)

D. A. Evans
21-00-Cover Page 11/3/03 9:33 AM

Anti Diastereomer:

&

Centers Opposing

D. A. Evans

The Felkin-Anh Eisenstein Model: A Breakdown

Chem 206

Are there cases not handled by the Anh-Eisenstein Model?


Anh-Eisenstein: "Best acceptor * orbital is oriented anti periplanar to forming bond." CSP3CSP2 is lower in energy than CSP3CSP3 bond.

Felkin-Anh analysis predicts B for R = electronegative substituent.


O Me-Li R R R HO Me Me OH

Ph
H H Nu C O RM H H Nu

Cyclohexyl
C O RM

A (R) Substituent G. Mehta, JACS 1990, 112, 6140


O C OMe

B
(Felkin-Anh Prediction)

A/B Ratio >90:10 34:66 27:73 17:83

C C

Cyclohexyl
Felkin-Anh analysis predicts the wrong product!

Ph
C

CH2OMe CH=CH2 CH2CH3

Cyclohexyl

C Case I:
O

Ph
O HO Me-Li Me Me OH NaBH4 CO2Me CO2Me CO2Me CO2Me HO NaBH4 H CO2Me CO2Me OH HO H H OH

CO2Me CO2Me M Nu

CO2Me CO2Me

CO2Me CO2Me

70: 30
H

Electronegative -CO2Me substituent will stabilize both CC bonding & antibonding states CO2Me CO2Me

Et Et Et

Et

Et

Felkin-Anh analysis predicts B

NaBH4 MeLi

39: 61 34: 66

Et

21-01-C=O cases not handled 11/2/03 12:33 PM

G. Mehta, Chem. Commun. 1992, 1711-2: "These results can be reconciled in terms of the Cieplak model."

D. A. Evans

The Felkin-Anh Eisenstein Model: A Breakdown Cieplak Model for C=O Addition

Chem 206

Case II: The Le Noble Examples Le Noble, JACS 1992, 114, 1916
O NaBH4 Me N i-PrOH, 25 C Me N Me N
Felkin-Anh Prediction

OH

HO

Cieplak, JACS 1981,103, 4540; Cieplak/Johnson, JACS 1989, 111, 8447

Point A: TS is stabilized by antiperiplanar allylic bond, but.... Point B: Nature of the stabilizing secondary orbital interactions differ: C

X R H Nu C O R

Ratio, 95:5

Pyramidally distorted C=O ruled out from inspection of X-ray structures.


O
NaBH4 i-PrOH, 25 C

OH

X
C

Nu
C

+ Syn Isomer R (R) R=F R = CO2Me Anti:Syn Ratio 62:38 61:39 45:55 43:57

X
C

Nu
Cieplak

Felkin Anh C C

Nu
C

Le Noble, J. Org. Chem. 1989, 54, 3836

R = SiMe3 R = OH

Point C: CX Electron donating ability follows the order: CH > CC > CN > CO
H
(Houk disputes the ordering of CH, CC)

NaBH4 MeOH, 0 C

+ Syn Isomer OH (R) R = NO2 Anti:Syn Ratio 79:21 63:37 43:57 36:64

Point D:

Importance of torsional effects (Felkin, Anh, Houk, Padden-Row) disputed.

"Structures are stabilized by stabilizing their highest energy filled states. This one of the fundamendal assumptions in frontier molecular orbital theory." The Cieplak hypothesis is nonsense." "Just because a hypothesis correlates a set of observations doesn't make that hypothesis correct." The management It is a capital mistake to theorise before one has data. Insensibly one begins to twist facts to suit theories, instead of theories to suit facts. (Sherlock Holmes, A Scandal in Bohemia)

Halterman, JACS 1990, 112, 6690

R = Cl R = OMe R = NH2

21-02-C=O cases not handled 10/31/03 11:28 AM

D. A. Evans
Observation
H O MeMetal H H

The Cyclohexanone Case Revisited: Frenking & Houk


As R becomes more electronegative, percentage of axial attack increases.
H Me OH H OH Me

Chem 206

Houk: Electrostatic rather than covalent considerations may be dominant.


"Equatorial electronegative substituents should interact more strongly with the C(2-3) and C(9-10) bonds than axial substituents." OH H O NaBH4 H OH MeOH, 25 C R R R
3

(R) Substituent R=H R = C6F5 R = CF3

R % Axial Attack Me-Li Me2Cu-Li

21% 34% 50%

6% 21% 42%

(R) Substituent R=H R = OAc R = Cl

Product Ratio 60:40 71:29 71:29

Felkin-Anh predicts opposite trend. Cieplak argument consistent with results. The Frenking Position: Cieplak stabilizing interaction is "dubious." Why not stabilize the forming sigma bond? Enhanced rate of axial Nu attack on cyclohexanone is caused by better electrostatic interactions of the ketone with the attacking reagent and not by torsional considerations. Nonequivalence of the *C=O LUMO with a greater extension on the axial face dictates stereoselection (Klein, 1973).

"If nucleophilic addition occurs anti to the better donor bond (Cieplak), the equatorial isomers should have considerably more axial attack than the parent while the axial isomers should have only a slight increase in axial attack." OH H O NaBH4 H OH MeOH, 25 C H H H R R R

(R) Substituent
"Exactly the opposite is observed."

Product Ratio 83:17 88:12

R = OAc R = Cl

Axial 4-substituents favor axial attack for electrostatic reasons:


"Since interactions between the C=O & C=O and the bonding & anti-bonding () CH & () CC orbitals are all symmetry allowed, it is difficult to predict a priori which interactions are dominant without carrying out quantum mechanical calculations." "Disfavored"
O M H +
R

"Favored"

Nu

M O

Nu

H +
R

Frenking & Reetz, Angew. Chem. Int. Ed. 1991, 30, 1146

(DAE: Bimetallic transition states were not considered)

K. Houk & Co-workers, J. Am. Chem. Soc. 1991, 113, 5018

21-03-C=O Frencking/Houk 10/31/03 11:31 AM

D. A. Evans

The Felkin-Anh Eisenstein Model: Electronic Effects


Are there electronic effects in the reaction?

Chem 206

The Polar Felkin-Anh Model


Premise: Transition state hyperconjugation between forming bond (HOMO) and best antiperiplanar acceptor (* CX, LUMO). Steric effects alre also considered; X = Halogen, OR, SR etc
X H H O R X R C

Several cases have already been presented which may be relevant


L. Flippin & Co-workers, Tetrahedron Lett.. 1985, 26, 973. H R O Me Me Me H R O Me Me Me Me OMe + Anti-Felkin Isomer OLi OMe OH O Me Me Me OMe OH O + Anti-Felkin Isomer

OH R X Nu

O R

Ratio 9 : 1

Nu Nucleophile
X O H C H R

favored

OH R X Nu

Ratio > 200 : 1 The molecular volume occupied by cyclohexyl acknowledged to be larger than that for phenyl. Because of shape phenyl "can get out of the way."

Nu

disfavored

Modified Cornforth Model


Premise: Transition State dipole minimization between polar CX substituent and the transforming carbonyl function dictate preferred TS geometrics. Steric effects alre also considered; X = Halogen, OR, SR etc
R H X C

Anh-Eisenstein Explanation based on HOMO-LUMO Analysis:


"Best acceptor * orbital is oriented anti periplanar to forming bond."

CSP3CSP2 is lower in energy than CSP3CSP3 bond.


Ph
H H Nu C O RM H H Nu C

OH R X Nu

Cyclohexyl
O RM R X

O H

O R

Nu Nucleophile
R O X C H H

favored

C C
Ph

Cyclohexyl

OH R X Nu

C C
Ph

Cyclohexyl

Nu

disfavored

Both models lead to the same stereochemical prediction.


21-04-Felkin-anh Polar Model 11/2/03 12:35 PM

D. A. Evans

Carbonyl Addition Reactions: Chelate Organization


O H OR SnBu3
Lewis Acid

Chem 206
OH C6H11 H C 5 3 OR
Chelate

Chelate organization also provides a powerful control element in carbonyl addition reactions
R R R R R O O M
NuM

OH H5C3

C6H11 OR
Felkin: RL=OR

R R R R O M

Nu O M
H
+

R R R R O

Nu OH

(OR) O Nu R
H+

Acid

Solv.

Ratio 20 : 80 >99 : 1 >99 : 1 5 : 95

O R

NuM

OH Nu R

Reviews

Reetz, Accts. Chem. Res. 1993, 26, 462-468 (pdf) Reetz, Angew. Chem. Int. Ed. 1984, 23, 556-569

MgBr2 R = CH2OBn THF (0) MgBr2 CH2Cl2 (-20) R = CH2OBn TiCl4 CH2Cl2 (-78) R = CH2OBn R = SiMe2(t)Bu BF3-Et2O CH2Cl2 (-78)

G. Keck & Co-workers, Tetrahedron Lett. 1984, 25, 265

Me

Addition of Carbon Nucleopiles


OR

Me TBSO OH RL OR
Felkin: RL=OR

MgBr O OCH2OBn
Chelate Model

Me TBSO OH OCH2OBn

Me

path A
O R OR H

H H

O R

Nu Me TBSO
Me-MgBr

diastereoselection 50 : 1

Nu:
R

Me TBSO OH OCH2OBn

Me

O OCH2OBn

Chelate Model

path B

OH H H R OR Nu

O RO

W. C. Still & Co-workers, Tetrahedron Lett. 1980, 21, 1031

diastereoselection >100 : 1

Chelation model
R

Nu:
MgBr

OH R OR
Chelate

H O H

path C

Me Et OBn

Me
THF

M O O R

H H

Nu

Me

H OH

Me Et OBn

Nu:

Y. Kishi & Co-workers, Tetrahedron Lett. 1978, 19, 2745

"only one isomer" Chelate Model

21-05-Chelation VS PFA-1 11/2/03 4:29 PM

D. A. Evans

Chelate Organization in C=O Addition


O H Me R OR OR Me3C O Me3C Me Me Me OBn TiCl4 Me3C OTMS Me O OH OBn

Chem 206

Chelate organization provides a powerful control element in carbonyl addition reactions


O Me Me Me OR
LiAlH4 -10 C

OH Me R

OH

diastereoselection 95 : 5 (Chelation)
OH OTBS

Polar Felkin Anh


(OR)
Overman Tet Lett. 1982, 23, 2355

Chelation
O OTBS TiCl4

Solv. THF Et2O

Ratio 30 : 70 2 : 98 95 : 5

R = CH2OBn R = CH2OBn

Model Chelate Chelate FA: RL=OR

R = SiPh2(t)Bu THF

Degree of chelate organization may be regulated by choice of solvent and protecting group. Note that SiPh2(t)Bu group prevents chelation.

2 Me2AlCl

Me O Al

Me TBS O

diastereoselection 93 : 7 (Felkin)
1+ O Me3C OH OBn

Case Study
Nu:

See Lecture 17 slide 03 for this Lewis acid

Nu:

H Me Me2AlCl2

Me

Felkin Control O H OP

(M)

Me MO

OH Nu Me

OP

diastereoselection 97 : 3 (Chelation)

H H

Me2AlCl & MeAlCl2 only Lewis acids that will chelate strongly to OSiR3 Groups. Evans, Allison, Yang,Masse, JACS 2001, 123, 1084010852 O H OP
t

D
Lewis acid (M) P O M

CH2OP

1,2-Syn (Felkin)

OTMS Bu
Lewis Acid
t

O Bu

OH

OP
t

OH

OP

Me 1, P = Bn 2, P = TBS Chelate Control

Bu Me Felkin 10 12
2 P = TBS 11 : 12 (%) 09 : 91 07 : 93 07 : 93 97 : 03 77 : 23 (55) (41) (55) (62)c (55)

Me
H

Me OH Nu Me 1,2-Anti (Chelation)
entry A B C D E

(M)

C
Nu:

OP

1 2

CH2Cl2 P = Bn P = TBS
Lewis acidb BF3OEt2 SnCl4 TiCl4 Me2AlCl MeAlCl2

Me Chelation 9 11
1 P = Bn 9 : 10 (%) 26 : 74 50 : 50 97 : 03 90 : 10 78 : 22 (76) (87) (74) (45) (70)

M O P O

Me H H
Nu:

21-06-Chelation VS PFA-2 11/2/03 8:57 PM

D. A. Evans

Carbonyl Addition Reactions: Chelate Organization


AlphaVersus Beta-Chelation
Me Me TBSO MgBr O OCH2OBn
Chelate Model

Chem 206

Kinetic Evidence for Chelate-Controlled C=O Additon


R R O M

Since

much more reactive than

R R Substrates which can participate in C=O chelation will be more reactive since the effective concentration of chelated intermediate will be higher. R

Me TBSO OH OCH2OBn

Me

Ketone +

RM

O M R + RM
OMgX OR Me OMgX

product

W. C. Still & Co-workers, Tetrahedron Lett. 1980, 21, 1031

diastereoselection 50 : 1

Me

H O

R-M -78 C Chelate Model

Me BnO

R OH

O Me O Me Bu OR

+ isomer

Me2Mg k2 Me

R
Me Bn CMe3 SiMe3 Si-i-Pr3

rel rate
213 174 9 7 1

BnO

M. T. Reetz & Co-workers J. Am. Chem. Soc.. 1983, 105, 4833. Other nucleophiles reported

R-M Me-MgCl Me-TiCl3

Solv. THF CH2Cl2

Ratio 40 : 60 90 : 10

Me2Mg k1 Me Me

Bu

reference rxn

rxn run inTHF at - 78C

Eliel, Frye, JACS 1992, 114, 1778-84 (read)

However, these trends are not transmitted strongly to -chelation


O Me O Me OSi(iPr)3 Me2Mg k2 R OBn Me2Mg k1 Me OMgX Me OR

Me BnO O

SiMe3
Acid -78 C CH2Cl2

Me BnO OH

+ isomer

Chelate Model Acid TiCl4 SnCl4 BF3-OEt2 Ratio 95 : 5 95 : 5 85 : 15

k1 = 2.5 k2

M. T. Reetz & Co-workers Tetrahedron Lett. 1984, 25, 729.

Hence, organization through

R R O

R O better than R

O R

Note that beta chelation can be developed as a control element by varying solvent & Nu. Note BF3 gives "apparent" chelate control

21-07-More chelation 10/31/03 2:41 PM

D. A. Evans

Carbonyl Addition Reactions: Chelate Organization


1,3-Stereoinduction Polar Model:
O H OP iPr R BF3OEt2 CH2Cl2 OTMS R 1,3-Anti 3 5 (%) (82) (91) (89) O OH OP iPr R 1,3-Syn 4 6 5:6 (P = TBS) 84 : 16 80 : 20 93 : 07 O OH

Chem 206

1,3-Stereoinduction Models for Chelate & non-Chelate Rxns


1,3-Stereoinduction Polar Model: Evans, Dart, Duffy,Yang, JACS 1996, 118, 43224343 1,3-Stereoinduction Chelate Model: Evans, Allison, Yang,Masse, JACS 2001, 123, 1084010852
Nu:

OP

(8)
iPr

Hb OM H
Nu:

1 P = PMB 2 P = TBS
1,3-Anti

(M)

Ha H PO

C C R

entry A B C

R t-Bu i-Pr Me

3:4 (P = PMB) 89 : 11 92 : 08 91 : 09

(%) (79) (84) (87)

O H

OP R P (M) O M H

E
R C
Nu:

OH

OP R

Nu

Steric effects appear to play a minor role in stereoinduction: OTMS


Nu:

F1

O H

Me Me Me

1,3-Anti (Chelation)

BF3OEt2 CH2Cl2 (88%)

1,3-Anti (58%) 1,3-Syn (42%)

Me2CH

F2

P O M O

C
Nu: H

H R O OP Me OH Nu 1,3-Anti (P = Bn) anti : syn 85 : 15 79 : 29 70 : 30 OP Me Nu 1,3-Syn (P = MOM) anti : syn OH OP Me

1,3-Anti Relationship is favored by either polar or chelate models H OTMS O H OBn Me Ph TiCl4 OTMS O H OPMB R
R = (CH2)2Ph
i

MLn

O Ph

OH

OBn Me

P = Bn (benzy) P = MOM (methoxymethyl) entry A conditions

diastereoselection 92 : 8

SiMe3 BF3OEt2 SnPh3 BF3OEt2 SnMe3 BF3OEt2

Pr BF3OEt2
i

O Pr

OH

OPMB R

B C

diastereoselection 81 : 19

21-08-betaOR Model 11/2/03 6:03 PM

D. A. Evans

Carbonyl Addition Reactions: Chelate Organization


1,3-Stereoinduction Polar Model The Anti Diastereomer
O OP OTMS Nu Me Felkin 15 17 17 : 18 (P = TBS) 99 : 01 95 : 05 71 : 29 OH OP iPr Nu OH

Chem 206

Evans, Dart, Duffy,Yang, JACS 1996, 118, 43224343

OP iPr

Can one develop a Rational model for & Stereocenters?


O H Me O H Me OR R R BF3OEt2 R OR R BF3OEt2 OTMS O R O OH Me OH Me OR R OR R

iPr R BF3OEt2 Me CH2Cl2 13 P = PMB 14 P = TBS entry A B C R t-Bu i-Pr Me

Me Anti-Felkin 16 18

15 : 16 (P = PMB) 99 : 01 98 : 02 97 : 03

Which of the two stereochemical representations is reinforcing? Non-reinforcing?

The Syn Diastereomer Integration of 1,3- Polar Model & Felkin-Anh Model
Nu

H O R OH OPMB

OP iPr Me R

OTMS Nu

OH

OP iPr Nu

OH

OP iPr

O H

OPMB
R

Ha F3BO H

C C iPr

Hb H OP

BF3OEt2 solvent

iPr

OTMS R O H
RL

19 P = PMB 20 P = TBS entry R t-Bu t-Bu i-Pr i-Pr Me Me Solvent CH2Cl2 toluene CH2Cl2 toluene CH2Cl2 toluene

Me Felkin 21 23 21 : 22 (P = PMB) 96 : 04 88 : 12 56 : 44 32 : 68 17 : 83 06 : 94

Me Anti-Felkin 22 24 23 : 24 (P = TBS) 96 : 04 94 : 06 87 : 13 75 : 25 58 : 42 40 : 60

BF3OEt2
Nu

OH
RL

Me F3BO

C RL

H H

Me

Me
Nu

O H

If both models are correct, they should integrate when the two stereocenters are reinforcing

Me F3BO H

C C iPr

Hb H OP

OR
R

A B C D E F

Conclusions
A: Anti diastereomer is reinforcing. Both models integrate. B: Syn diastereomer transitions from Felkin control (Large Nu) to 1,3-control (Small Nu).

Me & reinnforcing?

21-09-Merged Model-1 11/2/03 5:52 PM

D. A. Evans

Reinforcing & Non-reinforcing Relationsips

Chem 206

The Anti Diastereomer: Both Centers Reinforcing


O R O H Me entry Conditions A B C BF3OEt2, toluene BF3OEt2, toluene Ph3CClO4, CH2Cl2 R Me H H OPMB iPr Bu3Sn R Me Felkin : anti-Felkin >99 : 1 >99 : 1 >99 : 1 H Me O OPMB iPr TBSO Me Felkin OH OPMB iPr H Me OPMB iPr Bu3Sn
BF3OEt2 CH2Cl2

OH

OPMB iPr Felkin

TBSO OTBS OH

Me

Diastereoselection > 99 : 1 OPMB iPr Me TBSO Anti-Felkin OH OPMB iPr

Diastereoselection 59 : 32 : 9a
a

The third unpictured product is the Felkin-3,4-anti diastereomer.

The Syn Diastereomer: Stereocenters are Non-reinforcing

The Syn Diastereomer: Stereocenters are Non-reinforcing


O OP iPr Me 19 P = PMB 20 P = TBS entry A B C R t-Bu i-Pr Me R O Me Nu Me Felkin 21 23 OH OP iPr Nu Me Anti-Felkin 22 24 23 : 24 (P = TBS) (%) 08 : 92 13 : 87 14 : 86 (91) (64) (88) OH OP iPr

R O H Me entry A B C Conditions BF3OEt2, toluene BF3OEt2, toluene Ph3CClO4, CH2Cl2 R Me H H OPMB iPr Bu3Sn R Me Felkin : anti-Felkin 20 : 80 13 : 87 62 : 38 OH OPMB iPr

LDA, THF

21 : 22 (P = PMB) (%) 11 : 89 14 : 86 22 : 78 (71) (95) (73)

In this example, the OR substituent is the dominant stereo-control element

21-10-Merged Model-2 11/2/03 8:19 PM

D. A. Evans

Carbonyl Addition Reactions: Chelate Organization


Beta Chelation with Organometals
OTBS Me Me BnOCH2O OH
R-M MeMgBr Me2CuLi Ratio 50 : 50 97 : 3 + isomer

Chem 206

OTBS OH CHO O M O Me O XV
Metal M = MgCl M = ZnCl Ratio 70 : 30 97 : 3 + isomer

Me H BnOCH2O O
Me-M -78 C

Me

Chelate Model

Me2CH
D. A. Evans & E. Sjogren Tetrahedron Lett. 1986, 27, 4961.

Me O O O H
Me2CuLi -78 C Et2O

Me O O Me OH

+ isomer

Me TBSO
R = BOM (CH2OBn)

RO

O H

RO

OH

Me Me O Me

Chelate Model

diastereoselection > 95 : 5 Me Me BnOCH2O O H


Me2CuLi -78 C Et2O

Me O

Me Me BnOCH2O Me OH diastereoselection > 95 : 5


+ isomer

Me Me O Me

Me
Metal M = [CuCN]1/2 M = Li

Me
Ratio 98 : 2 33 : 67

D. A. Evans and S.L. Bender J. Am. Chem. Soc.. 1988, in press.

Chelate Model

Me

Me Me BnOCH2O O H
Me2CuLi -78 C Et2O

Me Me BnOCH2O Me OH BnO diastereoselection 70 : 30 O


+ isomer

OTMS Me H Me Ph
TiCl4 -78 C CH2Cl2

Me Me BnO OH O Ph
+ isomer

Chelate Model

Chelate Model

Me BnOCH2O O

H Me2CuLi
-78 C Et2O

Me BnOCH2O

Me OH

M. T. Reetz & Co-workers Tetrahedron Lett. 1984, 25, 729. + isomer

diastereoselection >92 % OH

O diastereoselection 50 : 50 H H O CO2R H Me
TiCl4 Me2Zn

W.C. Still & Co-workers, Tetrahedron Lett. 1980, 21, 1035.

Me
chelate model

CO2R H Me

S. W. Baldwin & Co-workers J. Org. Chem. 1987, 52, 320.

diastereoselection 96 : 4

21-11-Chelation beta-2 10/31/00 8:58 PM

D. A. Evans

Carbonyl Addition Reactions: Chelate Organization


Beta Chelate-Controlled Reduction Directed reductions of -hydroxyketones Evans, Chapman, Carreira, JACS 110, 3560 (1988)
Ph MeO OH + isomer OH O R2 R3 Me Ph LiAlH4 Et2O, 0 C 91-99% Ph NH2 OH Ph + isomer R1 H R R2
3

Chem 206

Me Ph MeO O Zn(BH4)2 Et2O, 0 C 91-99%

Me

H H

O+ C O H

OAc B

OH OAc favored R1 R3 R2 OH

T. Oishi & Co-workers Chem. Pharm Bull. 1984, 32, 1411. Me Ph NH2 O

Chelate Model

diastereoselection 97 : 3

R1

Me4NBH(OAc)3

H R2 R1 H R3 C O

OAc B

OH OAc R1 R3 OH R2

J. Barluenga & Co-workers J. Org. Chem. 1985, 50, 4052. O O OEt C3H 5 G. R. Brown & Co-workers Chem. Commun. 1985, 455. M-H

Chelate Model

diastereoselection 88 : 12

H O H OH OH Me Me Me

OH OH C3H 5 M-H Zn(BH4)2 Et2O LiAlH4 THF

OH Me OH C 3H 5 Ratio 100 : 0 0 : 100 Me

OH

O Me Me NaBH(OAc)3 HOAc, -20 C Me

+ isomer

diastereoselection 96 : 4
OH Me Me Me Me O Me NaBH(OAc)3 HOAc, -20 C Me Me Me Me OH OH Me + isomer

O Me Me

O N

CH2MOM M-H Me Me

OH

O XC Me Me Me

OH

O XC Me

diastereoselection 98 : 2
OH Me O Me Me Me Me NaBH(OAc)3 HOAc, -20 C Me Me Me Me OH OH Me + isomer

Me MOMCH2

M. Yamaguchi & Co-workers Tetrahedron Lett. 1985, 26, 4643. T. Oishi & Co-workers Tetrahedron Lett. 1980, 21, 1641 (Zn(BH4)2 on esters. Bu OH Ph O Ph Ph O B Bu O Ph NaBH4 -100 C K. Narasaka & Co-workers Chem. Lett. 1980, 1415. Ph

M-H Zn(BH4)2 Et2O KBH3H THF

Ratio 100 : 0 0 : 100

diastereoselection 98 : 2

OH

OH Ph + isomer

Propose a mechanism for tihs highly diastereoselective transformation, Evans, Hoveyda JACS 112, 6447 (1990)
O OH Me O R2CHO R1 Me 15% SmX3 catalyst R2 Me Me O OH R1

diastereoselection 96 : 4

diastereoselection > 100:1

21-12-Chelation beta-3 11/1/00 9:00 AM

D. A. Evans
Me OH H Me Me OH
M-H

Remote Heteroatom Effects in Carbonyl Addition


O C OH + Me H Me O Me H Me
Ratio 40 : 60 50 : 50 60 : 40 100 : 0 R. Frenette & Co-workers J. Org. Chem. 52, 304 (1987)

Chem 206
O O O O

Me OH

O OH Me O COOH
M-H

Me

Me

Me

Temp. & Solvent not specified in this study in first three cases

Reagent

Reagent NaBH2(OR)2 NaBH4 DIBAL (2.4 equiv) DIBAL (2.4 equiv) with ZnCl2

Ratio 50 : 50 70 : 30 95 : 5 99 : 1

R. Baker & Co-workers Chem. Commun. 1984, 74

Li-AlH4 Na-BH4 Zn-BH4


Li-BHEt3, (THF, -78C)

Me Et O O
M-H

Me Et O OH + OMe Et

Me Me O OH Ph OMe
1:1 3:1 11 : 1 R. Frenette & Co-workers J. Org. Chem. 52, 304 (1987) M-H

Me Ph O Me O Ph O Me
Ratio 50 : 50 70 : 30

Me O

COOMe

OMe
Y. Kishi & Co-workers Tetrahedron Lett. 1978, 2741.
* di-2-(o-toluidinomethyl)pyrrolidine

Solvent THF C 6H6

Li-BH(R)3, Et2O, 25C LiAlH4, Et2O, 25C LiAlH4-diamine,* -78C

OH O OH
MeM -78 C

OCH2OCH3 Me
M-H

Me

OCH2OCH3 +

OCH2OCH3 Me

t-BuPhMe2SiO

t-BuPhMe2SiO

Me OH

+ Isomer

67-87% yields

HO
Li-BH(R)3, LiAlH4

HO
73 : 27 76 : 24 G. Tsuchihashi & Co-workers Tetrahedron Lett. 1987,28, 6335.

Reagent MeLi MeMgBr Me3Al MeTiCl3 MeTi(O-iProp)3

Solvent Et2O THF CH2Cl2 CH2Cl2 CH2Cl2

Ratio 1.7 : 1 1.3 : 1 1.1 : 1 8.4 : 1 12 : 1

Me

OCH2OCH3
M-H

OCH2OCH3 Me + HO
Li-BH(R)3, LiAlH4

Me

OCH2OCH3 OH

HO
28 : 72 72 : 28

O R
R = Me, Ph(CH2)2

MeTi(O-iProp)3 0 C

OH R

OH Me
+ Isomer

W. G. Dauben & Co-workers Tetrahedron Lett. 1978, 2741.

Ratio 4-5.5 : 1

21-13-C=O ADDN (REMOTE-OR) 10/31/00 9:00 PM

Quote for the Day


Richard P. Feynman (as an undergraduate) in Surely you're Joking Mr. Feynman "When I was an undergraduate student at MIT I loved it. I thought it was a great place, and I wanted to go to graduate school there too of course. But when I went to Professor Slater and told him of my intentions, he said, 'We won't let you in here.' I said, "what"? Slater asked, 'Why do you think that you should go to graduate school at MIT'? "Because it is the best school for science in the country." 'You think that'? "Yeah." 'That's why you should go to some other school. You should find out how the rest of the world is.'

Quotes-2 11/5/03 9:11 AM

Experimental Support for Cornforth or FelkinAnh Models


Polar Felkin-Anh Model L R' H L M O H O H X RZ I R syn-pentane L L M O O X H RE H H R R' R' O
2 3

Cornforth Model L O R'


2 3

OH
4

M O O R

R' H X RZ H

RZ

matched for Cornforth Model L OH


4

III

M O O R

R' RE X H H syn-pentane

RE

II

matched for polar Felkin-Anh Model

IV

PFA Model Prediction: The (Z) enolate substituent causes a destabilizing syn-pentane interaction (I), while the (E) enolate substituent experiences no such interaction (II). Therefore, (E) enolates are predicted to give superior 3,4-anti selectivity relative to (Z) enolates. Cornforth Model Prediction: The (E) enolate substituent causes a destabilizing syn-pentane interaction (IV) while the (Z) enolate substituent experiences no such interaction (III). Therefore, (Z) enolates are predicted to give superior 3,4-anti selectivity relative to (E) enolates. Due to this dichotomy, the experimentally determined relationship between enolate geometry and aldehyde diastereofacial selectivity should validate a single model for asymmetric induction. Angew. Chem. Int. Ed., 2003, 42, 1761-1765.
CornforthSupport 11/5/03 9:27 AM

D. A. Evans

Diastereoselective & Enantioselective Carbonyl Addition


Relevant Problems:

Chem 206

http://www.courses.fas.harvard.edu/~chem206/

Chemistry 206 Advanced Organic Chemistry


Lecture Number 22

Database Problem 27: Chiral amino alcohol 1 efficiently mediates the addition of diethylzinc to aromatic aldehydes. While a number of other amino alcohols are also effective in controlling the absolute course of the addition process, this amino alcohol has been the focus of a recent computational investigation that addresses the preferred transition state geometry for this addition process (Pericas, et al. J. Org. Chem. 2000, 65, 7303 and references cited therein). It should be noted that, while 1 is not the actual catalyst, it is modified under the reaction conditions to the competent catalytic agent. Provide a detailed mechanism for the overall transformation. Use 3-dimensional representations to illustrate the absolute stereochemical aspects of the indicated transformation.

O N Ph Ph
(R) 1

OH H
0.06 equiv 1 Et2Zn, 0 C toluene
(S)

Enantioselective Carbonyl Addition


Enantioselective addition of R2Zn to aldehydes Enantioselective Reduction of Ketones & Imines Introduction to Enolate-based Nucleophiles

Et

97% ee

Ph OH

Reading Assignment for this Week:


Carey & Sundberg: Part A; Chapter 8 Reactions of Carbonyl Compounds Carey & Sundberg: Part B; Chapter 2 Reactions of Carbon Nucleophiles with Carbonyl Compounds Carey & Sundberg: Part B; Chapter 5 Reduction of Carbonyl & Other Functional Groups Enantioselective Carbonyl Reduction: Corey Angew. Chem. Int Ed. 1998, 37, 1986-2012 (handout) Enantioselective Carbonyl Addition (R2Zn): Noyori Angew. Chem. Int Ed. 1991, 30, 49-69 (handout)

Cume Question, 2000: Corey's introduction of chiral oxazaborolidine catalysts 1 in the borane-mediated enantioselective reduction of ketones represents an important advance in asymmetric synthesis (Corey & Helal, Angew. Chem. Int. Ed. 1998, 37, 1986-2012). Provide a detailed mechanism for the overall transformation. Use 3-dimensional representations to illustrate the absolute stereochemical aspects of the indicated transformation.

Ph Ph H N B
1

O Me
0.1 equiv 1

OH
(R)

Me

97% ee

Me

1 equiv BH3THF

Database Problem 151: The following stereoselective transformation has been reported by Fujisawa (Chem.Lett. 1991, 1555). Given the structure of the product, rationalize the stereochemical outcome of the process.

Ph N R H OMe
R"-Li

Ph HN R R" OMe dr> 97:3

D. A. Evans
22-00-Cover Page 11/5/03 8:51 AM

Wednesday, November 5, 2003

D. A. Evans

Enantioselective C=O Addition: Noyori Catalyst


The Catalytic Cycle
Me N Me Zn O Et H Me Me Me N Me Zn O Et H O H C Me Me

Chem 206

Catalytic Asymmetric Carbonyl Addition


R Zn R I Zn I Me H replace with chiral controller Me Me

Me N Me Zn O O Et H C

H C O R Noyori & co-workers, J. Am. Chem. Soc. 1986, 108, 6072. J. Am. Chem. Soc. 1989, 111, 4028. Review: Noyori Angew. Chem. Int. Ed. 1991, 30, 49 Review: L. Pu, Chem. Reviews 2001, 101, 757-824

the catalyst

Me

ZnEt O H Et

Me

90-98% ee

Et2Zn

Me2Zn PhCHO

Zn Zn Me Me Me Et Me

p. p.
Zn

Me

Me N Me Zn O O Et

Me N Me Zn O O Et Et H C

00 0000000 0000 00000

RDS
H C Me H Et H

(DAIB-Zn) ArCHO + R2Zn


Et2Zn Me2Zn Et2Zn Et2Zn Et2Zn Et2Zn Et2Zn the catalyst
0C, toluene 59 - 97% Ar OH R

Me

Zn

Zn

Et

C6H5CHO " p-ClC6H4CHO p-MeOC6H4CHO Cinnamyl PhCH2CH2CHO n-C6H13CHO

98% e.e. 91% e.e. 93% e.e. 93% e.e. 96% e.e. 90% e.e. 61% e.e.

Catalyst must be sterically hindered so that association is precluded


R R' 4 R'O ZnR R R' O Zn Zn O R' R R O R' Zn R R R' O O R' Zn Zn Zn O R' O R' R

Zn

Zn R

The method is catalytic in aminoalcohol. Two zinc species per aldehyde are involved in the alkylation step.
22-01-Et2Zn-1 11/5/03 8:46 AM

Product is taken out of the picture by aggregation

D. A. Evans

Enantioselective C=O Addition: Noyori Catalyst-2


Explanation for Nonlinearity of DAIB Catalyst
Me2 Me N Zn O O Zn N Me Me 2 Me2 Me N Zn O O Zn N Me Me 2

Chem 206

Other Catalysts for the R2Zn Addition Process


H Ph Ph O Zn Et Me3C N H Ph H O Zn Et H Me N Me N Li O Ph Me

N Me

97% e.e. (S)


Me Me N Me N Li O Ph Me

100% e.e. (R)


Li N

95% e.e. (S)

(S,S) dimer

(R,R) dimer

Bu N Et Zn O

Bu Me Me2 N Zn Me O Me2 N Zn Me O

N Li

90% e.e. (R)

Ph

90% e.e. (R)

90% e.e. (R)

p. p.

(Results are cited for the reaction of benzaldehyde and Et2Zn)

Problem: Rationalize the stereochemical course of each of the catalysts Non-linear effects observed with the Noyori Catalyst (DAIB-Zn)
100

(S) catalyst

(R) catalyst

There is no correlation between catalyst and product ee.

(S,R) dimer

Product (%ee)

Et2Zn + PhCHO (S) catalyst OH

Me2 Me N Zn O O Zn N Me Me 2

Observations (S,S) dimer dissociates upon addition of RCHO & effects catalysis

25% ee Catalyst affords product in 95% ee.

Ph

Me

(S,R) dimer is overwhelmingly more stable than (S,S) homodimer (S,R) dimer is ineffective as a catalyst
100

%ee of catalyst

22-02-Et2Zn-2 11/5/03 8:49 AM

D. A. Evans

Enantioselective C=O Addition: Dialkylzinc Addn Scope


Me2 N Zn Me O

Chem 206

Improved Selectivity with Aliphatic Aldehydes


Soai, J. Org. Chem. 1991, 56, 4264
Bu Et Zn O Ph R O H H Me 0 C, hexane 70 - 100% Bu N Me (6%) OH Et

Scope of the DAIB Catalyst

(S) catalyst
O H OH

(S) catalyst Et2Zn

Et

98% ee
OH

RCHO

Et2Zn

O H

(S) catalyst Me2Zn

O Me

O H

Me

O H

91% ee
OH

n-Bu

88% ee
H

78% ee

95% ee

93% ee

(S) catalyst

p. p.
O

Et2Zn

96% ee
OH

Lepicidin Application: The reaction functions in complex systems


C5H11 OR Bu Et Zn O Ph O Evans, Black, JACS 1993, 115, 44974513 H
9

(S) catalyst
Bu3Sn H

(n-Pen)2Zn

Bu3Sn

Bu N Me Et 21

85% ee
OH

the catalyst
O

Me H
3

(S) catalyst
H

Et2Zn

H H

90% ee
OH

OR

(S) catalyst
n-Bu H

n-Bu

Et

21

OTIPS
15 OTES

H 21

OTIPS
15 OTES

Et2Zn

60% ee

OH MeO

Me H
3

Et2Zn,
0 C, hexane

O MeO O

Me H
3

Review: Noyori Angew. Chem. Int. Ed. 1991, 30, 49 O

diastereoselection 10:1 (98%)


SnBu3

11

SnBu3

22-03-Et2Zn-Scope 11/5/03 8:49 AM

D. A. Evans

Enantioselective C=O Addition: Corey-Itsuno Catalyst


O Y B N Y X BH3 + N

Chem 206
O B X

Discovery of a Catalytic Process


Enantioselective Carbonyl Reduction: Corey Angew. Chem. Int Ed. 1998, 37, 1986-2012 (handout)

The Catalytic Cycle

BH3

The Stoichiometric Process: Itsuno, 1983-1985


RL Me2HC H 2N O R Ph Ph Ph OH

OBH2 C RS RL Y + N RS H 2B H

O C RS

2 equiv BH3 30 C, 10 hr
OH H R Ph

Chiral Boron Hydride ( HBXC )

H Y N O B X O

B + O

000 000000 000000 000000 000 000

( HBXC )

R = Me, 94 % ee R = Et, 94 % ee R = n-Bu 100 % ee

H 2B

O H

RS C RL

C RL

00 000 00 000 00 000 00 00 00 00 00 00 00 00

Itsuno, Chem. Commun. 1983, 469

p. p.

Itsuno, J. Org. Chem. 1984, 49, 555 Itsuno, J. Chem. Soc. Perkin Trans I. 1985, 2615

Ph

The Catalytic Process: Corey, 1987


H Ph Ph N O B R H Ph

B
Ph

BH3
N+ O B H 3B R

+ X O N B Ph B H Me H O+ H Ph

00000 00 00 00 00 00 00 00 00 000

R=H R = Me

O R Me

(0.1 equiv) BH3

OH R H Me

Corey, JACS 1987, 109, 5551 Corey, JACS 1987, 109, 7925 Corey, JOC 1988, 53, 2861

Catalyst X-ray, Corey, Tet. Let 1992, 33, 3429

R = Ph, R = t-Bu, R = c-C6H11


22-04-Corey Cat 11/5/03 8:55 AM

97 % ee 97 % ee 91 % ee
N

H Ph Ph O B Me

Mathre, JOC 1993, 58, 2880 catalyst prep: Mathre, JOC 1993, 58, 799 Mathre, JOC 1991, 56, 751 (Review) Martens, Tertrahedron Asymmetry 1992, 3, 1475

Improved version

But how does it really work ?

Chem 206

Enantioselective C=O Addition: Corey-Itsuno Catalyst

B
00000 00000 000 000 000 000 000 000 000 000 000 000 000 00000 000 00000 0000 0000 0000 000 0000 0000 0000 000 0000 0000 000 0000
Ph + X O N B Ph B H Me H O+

B
00000000 0000 00000000 0000 000 0000 000 000 000 000 000 000 00000 000

p. p.

22-05-Corey Cat 11/3/03 1:51 PM

D. A. Evans

Ph

D. A. Evans
H Ph Ph O The catalyst B Me

Enantioselective C=O Addition: Catalyst Scope


Representative Reductions
Me O Me O Me n-C5H11 n-C5H11 F Me Me Me Me Me (R) cat (0.2 equiv) Me Me Me

Chem 206
Me Me Me F Me

N O O

(S) cat (0.1 equiv)

1.5 equiv
O BH O

HO Me

92% ee

BH3 (0.6 equiv) THF 23C, 2 min

Tet. Let 1992, 33, 2319

ArCOO O

OH

ArCOO

91 : 9

Fluoxetine (Prozac) Synthesis


O BH3 Cl
H Ph

(R)-cat, as above
O n-C5H11

OH Cl
Ph

94% ee (>99%)
NaI MeNH2 OH

0.1 equiv CF3

p. p.

Corey, JACS 1987, 109, 7925


ArCOO

O B Me

OH

90 : 10
O OH O

NaH

NHMe CF3

(S) cat BH3

86% ee Prozac

NHMe

Cl

Tet. Let 1989, 30, 5207

O Br

OH

(S) cat BH3


O Me (S) cat BH3

Br

An -Amino Acid Synthesis 91% ee


R Me O CCl3 O BH OH O
H Ph N O B Ph

R
OH R CCl3

ee 95% 92% 98%

n-C5H11 c-C6H11 t-C4H9

91% ee
OH R CCl3 N3

0.1 equiv

n-Bu N3 R COOH

Corey, JACS 1992, 114, 1906 Tet. Let 1992, 33, 3435 Tet. Let 1992, 33, 3431

HO

rm temp

22-06-Corey Cat-2 10/31/01 7:43 AM

D. A. Evans

Enantioselective C=O Addition: Ketone Reduction


OH

Chem 206
Me B Cl OH RL RS

Stoichiometric Chloroborane Reducing Agents


NMe2 Me O RL RS

Enantioselective Reducing Agents

Ph

Me B H Al OEt O O

N-Methylephedrine, LiAlH4, (3,5-xylenol)2 [LiAl(lig)(OAr)2H]


Me Bn Ph Me2N OH

THF 50 - 90%

Ketone
acetophenone butyrophenone 2,2-dimethylpropiophenone 3,3-dimethyl-2-butanone 2,2-dimethylcyclohexanone

Reaction Conditions
25 C 25 C 25 C 25C, 12 days 25C, 2 days

% ee
98% 98% 79% 95% 91%

(R)-Alpine Borane (S)-BINAL-H

Li

Darvon alcohol, LiAlH4 [LiAl(lig)2H]

Reviews: Midland, Asymmetric Synthesis, Vol 2, p 45Granbois, Asymmetric Synthesis, Vol 2, p 71Brown, Accts. Chem. Res. 1992, 25, 16-24 Singh, Synthesis 1992, 605-617

p. p.

Reductions of Representative Carbonyl Compounds


O O R R O R Me Me

Brown's Model

Reagent
Alpine-Borane

72 - 92% e.e.

59 - 89% e.e.

78% e.e. R=Me 90% e.e. R=CO2Me 95 - 100% e.e. (71% ee, R=i-Pr) 15 - 75% e.e.

Large ligand (RL)

Me B H Favored TS RL

R Cl

BINAL-H

84 - 96% e.e. (57% ee, R=i-Pr) 34 - 90% e.e.

>95% e.e.

Small ligand (Rs) Less hindered aliphatic ketones are not reduced with useful levels of enantioselectivity

RS

Darvon-LiAlH4

25% e.e.

N-Methylephedrine- 75 - 90% e.e. LiAlH4

78 - 98% e.e. (cyclic ketones)

----

Brown, J. Org. Chem. 1985, 50, 5446 Brown, J. Org. Chem. 1986, 51, 3394 Brown, J. Org. Chem. 1988, 53, 2916

22-07-Asym Redn-1 11/5/03 8:55 AM

D. A. Evans

Enolates & Metalloenamines: Introduction


Important References

Chem 206

"Structure and Reactivity of Lithium Enolates. From Pinacolone to Selective C-Alkylations of Peptides. Difficulties and Opportunities Afforded by Complex Structures". D. Seebach Angew. Chem. Int. Ed. Engl., 27, 1624 (1983). "Stereoselective Alkylation Reactions of Chiral Metal Enolates". D. A. Evans Asymmetric Synthesis, 3, 1 (1984). "Generation of Simple Enols in Solution". B. Capon, B.-Z. Guo, F. C. Kwok, A. K. Siddhanta, and C. Zucco Acc. Chem. Res. 21, 121 (1988). "pKa and Keto-Enol Equilibrium Constant of Acetone in Aqueous Solution". Y. Chiang, A. J. Kresge, and Y. S. Tang J. Am. Chem. Soc. 106, 460 (1984).

Tautomers: Structural isomers generated as a consequence of the 1,3-shift of a proton adjacent to a X=Y bond. for example:

H Z X Y Z X

H Y

Keto-Enol Tautomers: Tautomerism may be catalyzed by either acids or bases:


O
H+

O R H H

+H+

OH R H H

base catalysis:

R O

CH3

Enols & Enolates are the most important nucleophiles in organic & biological chemistry.
H+ O Ra Rb base Ra O Rb El(+) Ra OH Rb

acid catalysis:

+H+

O R

H CH3

H+

OH R H H

CH3

El(+) O Ra El H 3C OH H 3C H El(+) H H 3C H O CH3 H3 C H Kresge, JACS 1984, 106, 460 On the origin of the acidity of enols: Wiberg, JACS 1996, 118, 8291-8299 H
+H+

Rb

Acidity of Keto and Enol Tautomers: Consider Acetone:


O CH3 H 3C H O
+ H + H pK = 10.94 (measured)

OH H

pK = 8.22 (measured)

Enamines & metalloenamines, their nitrogen counterparts, are equally important.


enamine H+ R Ra N Rb base R Ra R Ra N M El(+) Rb N H Rb

R Ra

N Rb El H3C

pK = 19.16 (calculated)

metalloenamine

22-08-Enolates/intro 11/5/03 8:57 AM

D. A. Evans

Enolization with Metal Amides bases


Stereochemistry
O O OH Me Base LMNR2 OLi R (E)

Chem 206
OLi

Tautomeric Equilibria: Ketones vs. Imines Keq ~ 103


Can you rationalize these differences? H N Me N Me

+
Me

R (Z)

Me

R-Substituent -OMe, O-t-Bu -S-t-Bu -Et -CHMe2 -CMe3 -C6H5 -NEt2 -NEt2

Ratio, (E):(Z) 95 : 5 95 : 5 77 : 23 40 : 60 0 : 100 0 : 100 0 : 100 25 : 75

MeNH2

H2O

LDA (THF) LDA (THF) LDA (THF)

Keq > 10

LDA (THF) LDA (THF)

The enamine content in an analogous imine is invariably higher than its carbonyl counterpart. In the case above, ring conjugation now stabilizes the enamine tautomer as the major tautomer in solution.

LDA (THF) LDA (THF) s-BuLi (THF)

Enolization: Amide Bases


H O R Me LMNR2 R N R O Li H

R Me

R R

OLi

Me (E) Geometry OLi R Me

Solvent

Base LDA (THF)

R-Substituent -OMe -OMe

Ratio, (E):(Z) 95 : 5 16 : 84

O Me H Li R H N R

LDA (THF, HMPA)

Base Structure
O R

Masamune (J. Am. Chem. Soc. 1982, 104, 5526)

(Z) Geometry

OLi Me LiNR2 THF, -78 C R Me


(E) Geometry

OLi

The Ireland Model (J. Am. Chem. Soc. 1976, 98, 2868) Narula, Tetrahedron Lett. 1981, 22, 4119 more recent study: Ireland, JOC 1991, 56, 650 For the latest word on this subject see: Xie, JOC 1997, 62, 7516-9 Stereoelectronic Requirements: The -C-H bond must be able to overlap with CO

Me
(Z) Geometry

Base LiN(i-Pr)2 LiN(SiMe3)2 LiN(SiEt3)2 LiN(SiMe2Ph)2 at equilibrium

R = Et, (E):(Z) 70 : 30 30 : 70 1 : 99 0 : 100 16 : 84

R = Cy, (E):(Z) 39 : 61 15 : 85 4 : 96 0 : 100

Ha

CO

R Hc

O Hb

base Ha+

Hc R

Hb O

22-09-Enolates/intro-2 11/3/03 2:03 PM

D. A. Evans
Base Structure
O Et Me

Enolization with Metal Amides bases


Corey & Co-workers, Tetrahedron Lett. 1984, 25, 491, 495

Chem 206
OLi OLi Me Me

Regioselective Enolization
O Me Mbase

OLi LiNR2 THF, -78 C Et

OLi

Et

Me

A
Base LiN(i-Pr)2 Me LiN(SiMe3)2 Ph3CLi Ph3CLi NaH KH temp 78 78 78 heat heat heat control kinetic kinetic kinetic thermo thermo thermo

Me (E) Geometry Me Me3C Me

(Z) Geometry Me

Ratio (A:B) 99:1 95:5 90:10 10:90 26:74 38:62

LiN(i-Pr)2 (LDA) 77 : 23 Me N Me Li Me (LiTMP) 86 : 14

N Me Li Me (LOBA) 98 : 2

Lithium Halide Effects

Collum (J. Am. Chem. Soc. 1991, 113, 9572) Collum (J. Am. Chem. Soc. 1991, 113, 9575) Collum (J. Am. Chem. Soc. 1991, 113, 5053)

A: Alkyl groups stabilize metal enolate A: As MO bond becomes more ionic A is attenuated Kinetic Selection sensitive to structure
O OLi Ph Mbase Ph OLi Ph

For the latest in the series of Column papers see: JACS 2000, 122, 2452-2458 O Et Me LiNR2 THF, -78 C Et Me Ratio, (E):(Z) LiTMP LiTMP, 10% LiBr 86 : 14 98 : 2 OLi OLi

Et

Me

A
O

+
ratio: 99:1

Enolization in Non-Ethereal Solvents Collum (JACS 2003, 125, ASAP)


O Me NaN(TMS)2 toluene, 78 6 equiv R3N OLi Me

estimated pKa's (Bordwell)

(25)

(18) Ph

pKeq est: 7 (10+7)

Unsaturated Ketones
1-Bu3N 1 KO Me Me KOtBu t-BuOH O LiN(i-Pr)2 LiO Me

R 3N krel

none Me2NEt 1 200

n-Bu3N 3000

Reaction kinetics suggest (TMS2NLi)2(R3N)(Ketone)

thermodynamic enolate

kinetic enolate
see kinetic acidity handout for an extensive compilation of cases.

22-10-Enolates/intro-3 11/3/03 2:28 PM

D. A. Evans

Enolization with Metal Amides bases

Chem 206

Kinetic Selection sensitive to structure


O Me Me O Me Me O Ph O Ph LDA Ph 99:1 O MeO LDA MeO 85:15 O LDA ~90:10 Me O LDA Et N Et N ~83:17 Et N Me OLi Me OLi Me LDA Me LDA OLi Me CH2 71:29 LDA OLi Me Me OLi Ph CH2 14:86 OLi Ph OLi Ph CH2 99:1 Me Me OLi Me OLi Me Me OLi Me

Kinetic Selection in Enolization of Unsaturated Ketones

Me

O LDA

Me

OLi

only enolate
O

O Me LDA O O LDA Me

Me

only enolate
LiO OLi

only enolate
Me

OLi MeO

OLi O LDA OLi

only enolate
Me

OLi

OLi

Me O LDA Me Me

OLi

only enolate
Me Me Me

Me

22-11-Enolates/intro-4 11/4/03 3:14 PM

Quote for the Day


Professor Robert Milikan (1928), Nobel Laureate in Chemistry

"There is no likelihood man can ever tap the power of the atom. The glib supposition of utilizing atomic energy when our coal has run out is a completely unscientific Utopian dream, a childish bug-a-boo. Nature has introduced a few foolproof devices into the great majority of elements that constitute the bulk of the world, and they have no energy to give up in the process of disintegration."

Quotes-3 11/7/03 8:42 AM

D. A. Evans

Enolates & Metalloenamines-1


Assigned Journal Articles

Chem 206

http://www.courses.fas.harvard.edu/~chem206/

Chemistry 206 Advanced Organic Chemistry


Lecture Number 23

"Structure and Reactivity of Lithium Enolates. From Pinacolone to Selective C-Alkylations of Peptides. Difficulties and Opportunities Afforded by Complex Structures". D. Seebach Angew. Chem. Int. Ed. Engl., 27, 1624 (1983). (handout) "Stereoselective Alkylation Reactions of Chiral Metal Enolates". D. A. Evans Asymmetric Synthesis, 3, 1 (1984). (handout)

Other Useful References


"Recent Advances in Dianion Chemistry". C. M. Thompson and D. L. C. Green Tetrahedron, 47, 4223 (1991). The Reactions of Dianions of Carboxylic Acids and Ester Enolates". N. Petragnani and M. Yonashiro Synthesis, 521 (1982).

Enolates & Metalloenamines-1


M R O R M R N R R

"Generation of Simple Enols in Solution". Capon, Guo, Kwok, Siddhanta, and Zucco Acc. Chem. Res. 21, 121 (1988). "Keto-Enol Equilibrium Constants of Simple Monofunctional Aldehydes and Ketones in Aqueous Solution". Keeffe, Kresge, and Schepp JACS, 112, 4862 (1990). "pKa and Keto-Enol Equilibrium Constant of Acetone in Aqueous Solution". Chiang, Kresge, and Tang JACS 106, 460 (1984).

Tautomerism in C=O and C=NR Systems C=O Enolization with Metal Amide Bases C=O Enolization: Kinetic Acidities Mild Methods for Enolate Generation Enolate Structure: A Survey of X-ray Structures Metallo-Enamine X-ray Structures

Database Problem 314


Kawabata and co-workers recently reported the remarkable enolate alkylation illustrated below (JACS 2003, 125, 13012). When the indicated -aminoacid ester is treated with KHMDS in DMF at 60 C, the derived cyclic amino acid ester is formed in high yield and enantioselectivity. The stereochemical outcome represents a formal retention of configuration. This reaction exhibits some generality as the 4- 5-, 6-, and 7-membered lactams may be obtained in high ee. R Br N CO2Et Boc R
KN(TMS)2 DMF 60 C, 30 min
R ee

Reading Assignment for this Week: Carey & Sundberg: Part A; Chapter 7 Carbanions & Other Nucleophilic Carbon Species Carey & Sundberg: Part B; Chapter 2 Reactions of Carbon Nucleophiles with Carbonyl Compounds
D. A. Evans
23-00-Cover Page 11/7/03 8:28 AM

CO2Et N Boc

Bn Me2CH

98 (S) 94 (S)

Friday, November 7, 2003

Provide a rationalization of these results. Three-dimensional drawings are recommended.

D. A. Evans

Enolates & Metalloenamines: Introduction


Important References

Chem 206

"Structure and Reactivity of Lithium Enolates. From Pinacolone to Selective C-Alkylations of Peptides. Difficulties and Opportunities Afforded by Complex Structures". D. Seebach Angew. Chem. Int. Ed. Engl., 27, 1624 (1983). "Stereoselective Alkylation Reactions of Chiral Metal Enolates". D. A. Evans Asymmetric Synthesis, 3, 1 (1984). "Generation of Simple Enols in Solution". B. Capon, B.-Z. Guo, F. C. Kwok, A. K. Siddhanta, and C. Zucco Acc. Chem. Res. 21, 121 (1988). "pKa and Keto-Enol Equilibrium Constant of Acetone in Aqueous Solution". Y. Chiang, A. J. Kresge, and Y. S. Tang J. Am. Chem. Soc. 106, 460 (1984).

Tautomers: Structural isomers generated as a consequence of the 1,3-shift of a proton adjacent to a X=Y bond. for example:

H Z X Y Z X

H Y

Keto-Enol Tautomers: Tautomerism may be catalyzed by either acids or bases:


O
H+

O R H H

+H+

OH R H H

base catalysis:

R O

CH3

Enols & Enolates are the most important nucleophiles in organic & biological chemistry.

acid catalysis:

+H+

O R

H CH3

H+

OH R H H

CH3

H+
O Ra Rb Ra

OH Rb

El(+) O Ra Rb El H 3C OH H 3C H El(+) H H 3C H O CH3 H3 C H Kresge, JACS 1984, 106, 460 On the origin of the acidity of enols: Wiberg, JACS 1996, 118, 8291-8299 H
+H+

Acidity of Keto and Enol Tautomers: Consider Acetone:


O CH3 H 3C H O
+ H + H pK = 10.94 (measured)

base
Ra

O Rb

El(+)

OH H

pK = 8.22 (measured)

Enamines & metalloenamines, their nitrogen counterparts, are equally important.


enamine

H+
R Ra N Rb

R Ra

H Rb M

R Ra El(+)

N Rb El H3C

pK = 19.16 (calculated)

base

R Ra

Rb

metalloenamine

23-01-Enolates/intro 11/7/03 8:16 AM

D. A. Evans

Enolization with Metal Amides bases


Stereochemistry
O O OH Me Base LMNR2 OLi R (E)

Chem 206
OLi

Tautomeric Equilibria: Ketones vs. Imines Keq ~ 103


Can you rationalize these differences? H N Me N Me

+
Me

R (Z)

Me

R-Substituent -OMe, O-t-Bu -S-t-Bu -Et -CHMe2 -CMe3 -C6H5 -NEt2 -NEt2

Ratio, (E):(Z) 95 : 5 95 : 5 77 : 23 40 : 60 0 : 100 0 : 100 0 : 100 25 : 75

MeNH2

H2O

LDA (THF) LDA (THF) LDA (THF)

Keq > 10

LDA (THF) LDA (THF)

The enamine content in an analogous imine is invariably higher than its carbonyl counterpart. In the case above, ring conjugation now stabilizes the enamine tautomer as the major tautomer in solution.

LDA (THF) LDA (THF) s-BuLi (THF)

Enolization: Amide Bases


H O R Me LMNR2 R N R O Li H

R Me

R R

OLi

Me (E) Geometry OLi R Me

Solvent

Base LDA (THF)

R-Substituent -OMe -OMe

Ratio, (E):(Z) 95 : 5 16 : 84

O Me H Li R H N R

LDA (THF, HMPA)

Base Structure
O R

Masamune (J. Am. Chem. Soc. 1982, 104, 5526)

(Z) Geometry

OLi Me LiNR2 THF, -78 C R Me


(E) Geometry

OLi

The Ireland Model (J. Am. Chem. Soc. 1976, 98, 2868) Narula, Tetrahedron Lett. 1981, 22, 4119 more recent study: Ireland, JOC 1991, 56, 650 For the latest word on this subject see: Xie, JOC 1997, 62, 7516-9 Stereoelectronic Requirements: The -C-H bond must be able to overlap with CO

Me
(Z) Geometry

Base LiN(i-Pr)2 LiN(SiMe3)2 LiN(SiEt3)2 LiN(SiMe2Ph)2 at equilibrium

R = Et, (E):(Z) 70 : 30 30 : 70 1 : 99 0 : 100 16 : 84

R = Cy, (E):(Z) 39 : 61 15 : 85 4 : 96 0 : 100

Ha

CO

R Hc

O Hb

base Ha+

Hc R

Hb O

23-02-Enolates/intro-2 11/7/03 8:16 AM

D. A. Evans
Base Structure
O Et Me

Enolization with Metal Amides bases


Corey & Co-workers, Tetrahedron Lett. 1984, 25, 491, 495

Chem 206

Regioselective Enolization
O OLi Me Mbase Me OLi Me

OLi LiNR2 THF, -78 C Et

OLi

Et

Me

A
Base LiN(i-Pr)2 Me LiN(SiMe3)2 Ph3CLi Ph3CLi NaH KH temp 78 78 78 heat heat heat control kinetic kinetic kinetic thermo thermo thermo

Me (E) Geometry Me Me3C Me

(Z) Geometry Me

Ratio (A:B) 99:1 95:5 90:10 10:90 26:74 38:62

LiN(i-Pr)2 (LDA) 77 : 23 Me N Me Li Me (LiTMP) 86 : 14

N Me Li Me (LOBA) 98 : 2

Lithium Halide Effects

Collum (J. Am. Chem. Soc. 1991, 113, 9572) Collum (J. Am. Chem. Soc. 1991, 113, 9575) Collum (J. Am. Chem. Soc. 1991, 113, 5053)

A: Alkyl groups stabilize metal enolate A: As MO bond becomes more ionic A is attenuated Kinetic Selection sensitive to structure
O OLi Ph Mbase Ph OLi Ph

For the latest in the series of Column papers see: JACS 2000, 122, 2452-2458 O Et Me LiNR2 THF, -78 C Et Me Ratio, (E):(Z) LiTMP LiTMP, 10% LiBr 86 : 14 98 : 2 OLi OLi

Et

Me

A
O

+
ratio: 99:1

Enolization in Non-Ethereal Solvents Collum (JACS 2003, 125, ASAP)


O Me NaN(TMS)2 toluene, 78 6 equiv R3N OLi Me

estimated pKa's (Bordwell)

(25)

(18) Ph

pKeq est: 7 (10+7)

Unsaturated Ketones
1-Bu3N 1 KO Me Me KOtBu t-BuOH O LiN(i-Pr)2 LiO Me

R 3N krel

none Me2NEt 1 200

n-Bu3N 3000

Reaction kinetics suggest (TMS2NLi)2(R3N)(Ketone)

thermodynamic enolate

kinetic enolate
see kinetic acidity handout for an extensive compilation of cases.

23-03-Enolates/intro-3 11/7/03 8:15 AM

D. A. Evans

Enolization with Metal Amides bases

Chem 206

Kinetic Selection sensitive to structure


O Me Me O Me Me O Ph O Ph LDA Ph 99:1 O MeO LDA MeO 85:15 O LDA ~90:10 Me O LDA Et N Et N ~83:17 Et N Me OLi Me OLi Me LDA Me LDA OLi Me CH2 71:29 LDA OLi Me Me OLi Ph CH2 14:86 OLi Ph OLi Ph CH2 99:1 Me Me OLi Me OLi Me Me OLi Me

Kinetic Selection in Enolization of Unsaturated Ketones

Me

O LDA

Me

OLi

only enolate
O

O Me LDA O O LDA Me

Me

only enolate
LiO OLi

only enolate
Me

OLi MeO

OLi O LDA OLi

only enolate
Me

OLi

OLi

Me O LDA Me Me

OLi

only enolate
Me Me Me

Me

23-04-Enolates/intro-4 11/1/01 12:38 PM

D. A. Evans
M R

Enolate Structures from X-ray Diffraction


O R M R O R O Ab initio calculations (Spartan) indicate that the partial negatilve charge on the alpha carbon is ~ 0.22 for the Li enolate Me M H H

Chem 206
M=H M = Li 0.19 0.22

Metal Tautomerism

For alkali metal enolates (M = Li, Na, K etc.) the O-metal tautomer is strongly favored. This generalization holds for most alkaline earth enolates (Mg+2) as well. These are the generally useful enolate nucleophiles For certain metal enolates from heavy metals such as M = Hg+2 the C-metal tautomer is sometimes favored. M R R

O Me

CMe3

O
O
M

O R

O R R

Me2 N O Li N Me2

Li N Li

Crystallized as the dimer

Resonance Structures

O resonance structure

resonance structure

Since enolates usually function as carbon nucleophiles, it is therefore of some interest to assess the relative importance of the illustrated contributing polar resonance structures. Within the last decade good X-ray crystal structures of a number of metal enolates have been obtained.

One would predict that as the relative importance of the C structure increases, the CO bond would shorten and the CC bond would lengthen. Me The prediction stated above does hold, but the net change in 1.36 the CC bond length is < 2 % ! R 1.32 O 1.35 H R H 1.34 H H O Li

Seebach & co-workers, J. Am. Chem. Soc. 1985,107, 5403.

Mg Mg Br
H Me CMe3 Br O Mg OEt2

In solution and in the solid state metal enolates have a strong tendency to aggregate into dimers and tetramers to satisfy metal solvation requirements.

M R O + M O R R O

M O R M

M RO M O M O R

OR M

Crystallized as the dimer

23-05 enolate structure-1 11/7/03 8:15 AM

D. A. Evans

Enolate Structures: Lithiun Enolates


Me O Me Me Me O Si Me CMe3 LDA O H 2C Me Li Me O Me

Chem 206

Li Enolates

Si

Me CMe3

Williard, P. G.; Hintze, M. J. J. Am. Chem. Soc. 1987, 109, 5539-5541. Li Li O OLi Li O Li O Li O N Li Li O O

O Li O Li N

Si

O N Li Li O Li

Si

Li Li

Li

Li O Li O O Li Li O O Li O Li

Li Li Li

Williard, P. G.; Carpenter, G. B. J. Am. Chem. Soc. 1986, 108, 462-468.

23-06 Enolate Structrure-2 11/7/03 8:14 AM

D. A. Evans

Enolate Structures: The Reformatsky Reagent


O Me3C O
Zn(0)

Chem 206

Zn Enolates
O

ZnBr Me3C O THF O Zn Br

The THF Complex metal tautomers


O Br Zn O O CMe3 THF

Me3C

Br

Br Zn O Me3C O

O CMe3

Me3C

ZnBr

O Zn Br

Br Zn Zn C O C Zn 2.0 Br Dekker, J.; Boersma, J.; van der Kerk, G. J. M. J. Chem. Soc. Chem. Commun. 1983, 553. Dekker, J.; Budzelaar, J.; Boersma, J.; van der Kerk, G. J. M. Organometallics 1984, 3, 1403.
23-07 Zn Enolate structure 11/7/03 8:14 AM

Br

O Zn 2.0

Br O

D. A. Evans

The Reformatsky Reaction


The "Classical" Reformatsky Process The Samarium(II) Variant

Chem 206

Review: Comprehensive Organic Synthesis, 1991; Vol 2, Chapter 1.8, pp 277-299 Frstner, A. "Recent Advances in the Reformatsky Reaction." Synthesis 1989, 571. CHO Zn, Et2AlCl
-20 C

Molander, "Reductions with Samarium (II) Iodide." Org. Reactions1994, 4 6, 211-367.

O O

O O

OH

R O O Br CHO

SmI2/0 C

R O O OH

R = H: 85% yield R = Me: 82% yield aldol diastereoselection 70:30 no config assignment

Br

48% yield

H. Nozaki & Co-workers, J. Am. Chem. Soc. 197, 99, 7705

Both cyclic and acyclic cases studied (11 cases).

R = H: 76% yield

Me O

CHO Br

OH
Zn, Et2AlCl

Me O O
Based on the Nozaki recipe JACS 1977, 99, 7705

O C6H13
diastereoselection 10:1 55% yield

Br CHO R

SmI2/0 C

O O
SmI2/0 C

R = Me: 82% yield aldol diastereoselection 62:38 OH no config assignment

C6H13 O

T. Nishida & Co-workers, Tetrahedron 1991, 47, 6623.

O O

Br

CHO

O O OH

82% yield

O Ph Me
PhCHO

Br

Zn in HOH 25 C

O Ph Me

OH Ph Ph

OH
Good entry into prior literature

Ph Me Br

O O R Me O Br O R Me Me
G. A. Molander & Co-workers, J. Am. Chem. Soc. 1987, 109, 6556.

T. Tabushi & Co-workers, Tetrahedron Lett. 1986, 27, 3889.

O O Me
2 SmI2/THF -20 C

diastereoselection 60:40 87% yield Rxns carried out in water with either activated Zn or Sn. 19 cases reported.

O R Me O Et OH

8: 1 diastereoselection 97% yield

T. H. Chan & Co-workers, Chem. Commun. 1990, 505.

Br

Br

O Me

2 SmI2/THF -78 C

O R Me Et OH

one isomer? 86% yield

O Bn N Br

O
Zn/THF

O Bn N

OZnBr

25 C

O Bn N

Me

Me

O
Proposed Transition structure

R(H) Et O O Sm

78% yield C. H. Heathcock & Co-workers, J. Org. Chem. 1987, 52, 5745.

R (R)H H

23-08-Reformatsky Rxn 11/6/03 4:15 PM

D.A. Evans

Design of Soft Enolization Systems


Mild Methods for Forming Enolates
Some qualitative observations (Evans Group, Unpublished)) Cl4 O O H CH3 F 3B BF3 pKa = 36 BF3 pKa = 24 H Hgp H2SO4 = 304 O CH3 Hgp HI = 314 Hgp = 50 kcal/mol O N Bn estimated pKa (DMSO)??? Hgp = 24 kcal/mol Et3NH Cl4 O Ti N Bn O Me Et3NH O Bn % deuterium 100 100 80 70 70 25 10 0 O O N D Bn *2.0 equiv required O O Me O N Bn O Me
DCl/D2O
H

Chem 206

Lewis Acid C=O Complexation Enhances CH Acidity (Computation)

Cl4
TiCl4 (1.0 equiv) Et3N (1.0 equiv)

Ti

O Me
Et3N

O O

Ti N

pKa 9 (DMSO) Me Et3NH

Keq~100 Bn pK + Et3N Cl4


TiCl4 (1.0 equiv) Et3N (1.0 equiv)

Hgas phase pKaHOH

+366 +17

+316 7

+9

BF3 complexation generates a "superacid" comparable to the acidity of H2SO4 Ren et al, JACS 1999, 121, 2633-2634 (pdf)

O Me
Et3N

O O

Ti N

O Me 2

Some qualitative observations (Evans Group, Unpublished)) Xn O O N Bn Lewis Acid (MXn) TiCl4 TiCl3(Oi-Pr) TiCl4(THF)2 TiCl2(Oi-Pr)2 AlCl3 MgBr2OEt2* TiCl(Oi-Pr)3 SnCl4, Et2AlCl, ZrCl4 O Me
Lewis Acid (1.0 equiv) Et3N (1.0 equiv)

pKa 9 (DMSO) Bn Et3NH Cl4 Ti N Bn estimated pKa (DMSO) ~ +7 Hence TiCl4 complexation lowers acidity by ~20 pka units. this number is the same mgnitude as the BF3acetaldehyde case just discussed O Me O O Cl4 Ti N Bn O Me
H

O O

M N

CH2Cl2 O

pKa ~ +7

In these experiments, the Lewis acid was added first followed by the amine.

pKa 9 (DMSO)~ +27

23-09-soft enoliz-1 11/7/03 8:05 AM

D.A. Evans

Design of Soft Enolization Systems Strategy Lithium Enolates


Horner-Wadsworth-Emmons Reaction.
O (EtO)2P
1.2 equiv

Chem 206

Choose Lewis Acid (LA) which can reversibly associate with amine base (B:).
() (+)

LA

B:

LA

O OEt

LiCl, 1.2 equiv Base, 1.0 equiv

This system has the potential to enolize carbonyl functional groups:


O

+
R CH3

+O LA
R

LA CH2H R

LA CH2

Me Base = DBU, 5 min, 99%, >50:1 E:Z i-PrCHO, 1 equiv = DIPEA, 7 h, 97%, >50:1 E:Z OEt Base MeCN Me rt

+ BH

(+)

pKa 19.2 (DMSO), K+ counterion pKa 12.2 (Diglyme), Li+ counterion Roush & Masamune, Tet. Lett. 1984, 25, 2183-2186

+ B:
() (+)

LA

B
Complexation Reversible Reversible Reversible (Et3N, EtNi-Pr2) Reversible (Et3N, EtNi-Pr2) Reversible (Et3N, EtNi-Pr2) Reversible (Et3N, EtNi-Pr2) Irreversible (Et3N, EtNi-Pr2) Reversible (Et3N, EtNi-Pr2) Reversible (Et3N, EtNi-Pr2) Reversible (Et3N, EtNi-Pr2)
O (EtO)2P
1 equiv

Useful Lewis Acid Pairs MgBr2 + NEt3 LiX + NR3 Sn(OTf)2 + NR3 R2B-OTf + NR3 R2BCl + NR3 PhBCl2 + NR3 TiCl4 + NR3 i-PrOTiCl3 + NR3 (i-PrO)2TiCl2 + NR3 (i-PrO)3TiCl + NR3
O R CH3 RS O CH3

OTf = OSO2CF3

NHCbz CHO Me

Above conditions using DIPEA 24 h, rt

NHCbz

O OEt

Me
85% + 10% recovered aldehyde

Me

Me

Conventional methods of deprotonation (NaH) resulted in epimerization (Overman JACS 1978, 5179).

Magnesium Enolates
O OEt
MgBr2, 1.2 equiv Et3N, 1.1 equiv RCHO, 1 equiv THF, rt

All of the above systems will enolize simple ketones to some extent.
O PhO CH3 EtO O CH3 R2N O CH3

O R OEt

R= i-Pr, 40% yield R= n-C6H13, 100% yield

100% enolization for B, Sn, Ti partial enolization for Li, Mg


23-10-soft enoliz-2 11/7/03 8:14 AM

Rathke, Nowak J. Org. Chem. 1985, 50, 2624-2626.

D.A. Evans

A Survey of 'Soft' Enolization Techniques Magnesium Enolates


Rathke
J. Org. Chem. 1985, 50, 2622-2624. J. Org. Chem. 1985, 50, 4877-4879. Syn. Comm. 1986, 16, 1133-1139.

Chem 206

Titanium Enolates
The Early Literature Lehnert, W. Tetrahedron Lett. 1970, 4723-4724.

Diethylmalonate acylations
O EtO O OEt Me O Cl
MgCl2, 1 equiv Et3N, 2 equiv MeCN, rt

O EtO

O OEt EtO

O OEt Et

O Et

TiCl4 Pyridine THF

O EtO Et

O OEt Et

Ketone Carboxylation
O

O Me 85% yield
MgCl2, 2 equiv NaI, 2 equiv Et3N, 4 equiv CO2, MeCN rt

75% yield

Harrison, C. R. Tetrahedron Lett. 1987, 28, 4135-4138.


O COOH 70 % yield Ph O Me
+ PhCHO TiCl4 Et3N CH2Cl2, 0 C 30 min

O Ph

OH Ph Me

MgCl2, 2 equiv Et3N, 4 equiv +CO2 MeCN rt

Mg

O
MVK

O O
EtOH -CO2

O Me 75% yield

Ketone and aldehyde combined followed by sequential addn of TiCl4 and then amine

91% yield 95:5 syn/anti

O Ph

TiCl4 Me
+

O Ph H Ph

O Me

O Ph

TiCl4 H

Michael reaction
O O N Bn O Et3NH O O Me
MgBr2OEt2

O CO2Me Brn
Mg
Et3N CH2Cl2, 0 C

O N Me Bn CO2Me Brocchini, Eberle, Lawton J. Am. Chem. Soc. 1988, 110, 5211-5212. O S
TiCl4 OH DIPEA THF -40 C

O CHO S OH NO2
10-15:1 Z/E

73% yield, 93:7 diastereomer ratio Me


Evans, Bilodeau unpublished results.

O NO2

Bn Deuterium quench indicates 25% enolization of N-propionyloxazolidinone

23-11-soft enoliz-3 11/7/03 8:14 AM

M. Bilodeau, D.A. Evans


Titanium Enolates
Cl Cl Ti Cl O O Cl O N Bn

A Survey of 'Soft' Enolization Techniques


Reactions with Representative Electrophiles
Cl4 O Me R 3N O Ti N Bn O Me Xp Me 78%, 98:2
H2C=CHCO2Me

Chem 206

O Xp O O OMe
BOMCl PhSCH2Cl

O O N

O Me 1

CH2OBn Me 99%, >99:1 Xp Me O CH2SPh 93%, 97:3

TiCl4

Bn

Enolization process not responsive to tertiary amine structure DIPEA, Et3N, N-Ethylpiperidine all suitable bases. DBU and tetramethylguanidine do not provide enolate. CH2Cl2 is the only suitable solvent for these enolizations.

Cln
Et

N-Propionyloxazolidone (1)
Lewis Acid TiCl4 i-PrOTiCl3 TiCl42THF (i-PrO)2TiCl2 (i-PrO)3TiCl % Enolization 100 100 80 70 ~10

Ethylisopropylketone
Lewis Acid TiCl4 i-PrOTiCl3 (i-PrO)2TiCl2 % Enolization 100 80 50

O Xp Me 88%, >99:1
HC(OMe)3

O O

Ti N

O Me
ClCH2NHCOPh

O Xp Me 89%, 97:3
MeOCH2NHCbz

(CH2)2COEt

CH2NHCOPh

Bn

O Me Me Me
Irreversible Complexation

O Xp Me 99%, >99:1 Xp CH(OMe)2 O

O Xp CH2NHCBz Me 91%, 96:4

Order of addition of reagents is important for TiCl4. R 3N + TiCl4 + R3N-TiCl4

CH2OH Me 93%, 99:1

Order of addition of reagents is not important for i-PrOTiCl3 or (i-PrO)2TiCl2. R 3N + i-PrOTiCl3 + R3N-TiCl3(OiPr) Reversible Complexation

Enolizable substrates:

J. Am. Chem. Soc. 1990, 112, 8215-8216.; J. Org. Chem. 1991, 56, 5750-5752.

O R Me t-Bu

O Me i-Pr

O Me O MeO

O OMe PhS O R Me

O Me O O N Me Bn O O Me
1. TiCl4 DIPEA 2. i-PrCHO

O O N

OH Me

Substrates Which present problems:

O MeO Me

Me Bn

Me

Me

R=Ar, R<i-Pr self condensation self condensation

86% yield, >99:1 Evans, Clark, Metternich, Novack, Sheppard J. Am. Chem. Soc. 1990, 112, 866.

23-12-soft enoliz-4 11/7/03 8:12 AM

D.A. Evans

A Survey of 'Soft' Enolization Techniques: Boron Enolates-1 Dialkylboron Triflates


O Me Me R2BX, R3N O Me BR2 Me Me

Chem 206
O BR2

Di-n-butylboron triflate
Mukaiyama, Inoue Chem. Lett. 1976, 559-562. Bull. Chem. Soc. Jpn. 1980, 53, 174-178. Enolizes ketones with 2,6-lutidine or DIPEA in ethereal solvents.

9-BBN-OTf, Et3N Cy2B-Cl, Et3N Bu2B-OTf, Et3N

ratio ~97: 3 ratio 21:79 ratio ~97: 3

Mr

Diastereoselective Aldol Reactions of Boron Enolates.


Evans, Vogel, Nelson J. Am. Chem. Soc. 1979, 101, 6120. Evans, Nelson, Vogel, Taber J. Am. Chem. Soc. 1981, 103, 3099-3111. Evans, Bartroli, Shih J. Am. Chem. Soc. 1981, 103, 2127. Masamune, S. et. al. Tetrahedron Lett. 1979, 2225, 2229, 3937. Masamune, S. et. al. J. Am Chem. Soc. 1981, 103, 1566-1568.

Borane and lutidine or DIPEA form 1:1 complex with L2B-OTf. Complexation reversible as enolization will occur upon addition of ketone. Less hindered nitrogen bases - pyridine, Dabco, DBU, irreversibly complex with L2B-OTf.

The Ketone-Boron Complexes as enolate precursors: Chiral dialkylboron triflates


Masamune, Sato, Kim, Wollmann J. Am. Chem. Soc. 1986, 108, 8279-8281. Paterson, I. et. al. Tetrahedron 1990, 46, 4663-4684. Tetrahedron Lett. 1989, 30, 997-1000. Tetrahedron Lett. 1986, 27, 4787-4790. Me R TfOB X Me Me Me (-)-(Ipc)2BOTf BOTf OBL2 R
NR'3

R B R O Me -X

R B O R

R B O Me charged R

R -X +X Me OBL2 R

R O B

R X syn Me

anti

+X

Me

Enolate Stereochemistry
Evans, Nelson, Vogel, Taber J. Am. Chem. Soc. 1981, 103, 3099-3111. Goodman, Tetrahedron Lett. 1992, 33, 7219. Enolization Model: Paterson, Tetrahedron Lett. 1992, 33, 7223. O Me Me 9-BBN-OTf, Et3N Cy2B-Cl, Et3N Me R2B-X, R3N Me Me ratio ~ 88: 12 ratio ~ 3: 97 O BR2 Me Me Me Mr O BR2 anti H R H

R Me syn H
NR'3

O Me

BL2OTf

R Me

BL2X

Cy2BCl-ketone complex may deprotonate through syn complex R2BOTf-ketone complex may deprotonate through charged complex with (Z) preference

Brown, J. Org. Chem. 1993, 58, 147-153

23-13-soft enoliz-5 11/7/03 8:13 AM

D. A. Evans

C versus O Enolate Reactivity & the Hammond Postulate

Chem 206

Question: Why do we generally show enolates reacting with electrophiles at carbon as opposed to oxygen ?? Let's begin the the discussion with an observation: "As electrophile reactivity increases, the percentage of reaction at the enolate oxygen increases." For example, consider the reactions of cyclohexanone enolate with the two electrophiles, methyl iodide and the much more reactive acetyl chloride:
O OEl

The Hammond Postulate is also relevant to this issue and is broadly used to make qualitative statements about transition state structure. Hammond, JACS 1955, 77, 334 (handout) In attempting to grasp the Hammond Postulate, let's consider two extreme reactions, one which is strongly endothermic and one which is strongly exothermic. T

1
El(+)

El(+)
O

C/O Rxn Ratio << 1 >> 1

B
Energy

Me C Cl O O Me I

Strongly Exothermic Reactions H > 20 kcal/mol B A

El

A
Rxn Coordinate

The very reactive acid chloride gives almost exclusively the O-acylation product while the less reactive methyl iodide affords the alternate C-alkylation product. These results may be understood in the context of qualitative statements made by Hammond (The Hammond Postulate) and Hine (The Principle of Least Motion) The Principle of Least Motion: "As reactions become more exothermic, the favored reaction becomes that path which results in the least structural (electronic) reorganization."

Hammond Postulate "For strongly exothermic reactions, the transition state T looks like reactant(s) e.g. B." As applied to the enolate-electrophile reaction, for very exothermic reactions, e.g. the reaction with acetyl chloride, the transition state for the process will involve little enolate structural reorganization. Hence in this instance the electrophile heads for the site of highest electron density

Carey & Sundberg: Part A; Chapter 4, pp217-220 for discussion of Hammond's Postulate
See Hine in Advances in Phys. Org. Chem. 1977, 15, 1-61 Since the X-ray data clearly support the picture that resonance structure 1 best represents the enolate structure, highly reactive electrophiles will favor O-attack according to Hine's generalization.
23-14 C vs O Enolate React 11/6/03 5:29 PM

Based upon the above discussion draw a detailed mechanism for the protonation of cyclohexanone enolate.
O
H+

D. A. Evans
Metalloenamines:

Enols, Enolates, Enamines & Metalloenamines: Reactivity Hierarchy


Decreasing Nucleophilicity
Nucleophile

Chem 206

Imines may be transformed into their conjugate bases (enolate counterparts) with strong bases: R pKa~ 29-33 N Li-NR2 Metal R N R-MgX R N

NR C C C C

O C C

NR2 C C

OMe

Electrophile

Br2, O3 H3O+
O R C Cl O R C H O R C R

+ + + + + + + + +

+ + + + + + +

+ + + +

+ +

The usual bases employed are either lithium amides (LDA) or Grignard reagents. Note that Grignard reagents do not add to the C=N pi-bond due to the reduced dipole. With this functional group, deprotonation is observed to be the preferred reaction.

When to use a metalloenamine:


Metalloenamines are significantly more nucleophilic than ketone or aldehyde enolates. They are used when less reactive electrophiles are under consideration. For example: OLi O Me Me
syn relationship

Decreasing Electrophilicity

X
no reaction

However:
Metal R N I

Me Me N
good yield

Me

Me Me

O R C OR O H 2C CH2 I

Metalloenamines are reactive enough to open epoxides in good yield. Ketone enolates are only marginally reactive enough for this family of electrophiles. O
Li-NR2

Me2CH

R N Li CH2 N R OH

Me

O R C NR2

Nature uses enamines, "stabilized" enolates, and enol derivatives in CC bond constructions extensively.

23-15 Enolate Reactivity Grid 11/6/03 5:28 PM

Quote for the Day

"640K ought to be enough for anybody." Bill Gates, 1981

Today's astrological forecast, Boston Globe, Monday, November 10, 2003 Capricorn (Dec 22Jan 19th)

"Be prepared for someone to try to steal your ideas or take credit for your work. You're on to something tangible and you need to act fast."

Quotes-3 11/10/03 9:12 AM

D. A. Evans

Enolates & Metalloenamines-2


Assigned Journal Articles

Chem 206

http://www.courses.fas.harvard.edu/~chem206/

Chemistry 206 Advanced Organic Chemistry


Lecture Number 24

"Structure and Reactivity of Lithium Enolates. From Pinacolone to Selective C-Alkylations of Peptides. Difficulties and Opportunities Afforded by Complex Structures". D. Seebach Angew. Chem. Int. Ed. Engl., 27, 1624 (1983). (handout) "Stereoselective Alkylation Reactions of Chiral Metal Enolates". D. A. Evans Asymmetric Synthesis, 3, 1 (1984). (handout)

Other Useful References


"Advances in Asymmetric Enolate Methodology" Arya, Qin, Tetrahedron 2000, 56, 917-947 (pdf) "Recent Advances in Dianion Chemistry". C. M. Thompson and D. L. C. Green Tetrahedron, 47, 4223 (1991). R The Reactions of Dianions of Carboxylic Acids and Ester Enolates". N. Petragnani and M. Yonashiro Synthesis, 521 (1982). "Generation of Simple Enols in Solution". Capon, Guo, Kwok, Siddhanta, and Zucco Acc. Chem. Res. 21, 121 (1988). "Keto-Enol Equilibrium Constants of Simple Monofunctional Aldehydes and Ketones in Aqueous Solution". Keeffe, Kresge, and Schepp JACS, 112, 4862 (1990). "pKa and Keto-Enol Equilibrium Constant of Acetone in Aqueous Solution". Chiang, Kresge, and Tang JACS 106, 460 (1984). Explain why A is favored for X = O while B is favored for X = NNHR X Me A Me base Me B X Me

Enolates & Metalloenamines-2


M R O R M R N R

Introduction and General Trends Enolate Alkylation: Electronic & Steric Control Elements Enolate Alkylation: Unusual Cases Chiral Amide Enolates Chiral Ester Enolates Chiral Imide Enolates Chiral Metalloenamines

Reading Assignment for this Week: Carey & Sundberg: Part A; Chapter 7 Carbanions & Other Nucleophilic Carbon Species Carey & Sundberg: Part B; Chapter 2 Reactions of Carbon Nucleophiles with Carbonyl Compounds
D. A. Evans
24-00-Cover Page 11/9/03 11:46 AM

Monday, November 10, 2003

D. A. Evans
Metalloenamines:

Enols, Enolates, Enamines & Metalloenamines: Reactivity Hierarchy


Decreasing Nucleophilicity
Nucleophile

Chem 206

Imines may be transformed into their conjugate bases (enolate counterparts) with strong bases: R pKa~ 29-33 N Li-NR2 Metal R N R-MgX R N

NR
C C C C

O C C

NR2 C C

OMe

Electrophile

Br2, O3 H3O+
O R C Cl O R C H O R C R

+ + + + + + + + +

+ + + + + + +

+ + + +

+ +

The usual bases employed are either lithium amides (LDA) or Grignard reagents. Note that Grignard reagents do not add to the C=N pi-bond due to the reduced dipole. With this functional group, deprotonation is observed to be the preferred reaction.

When to use a metalloenamine:


Metalloenamines are significantly more nucleophilic than ketone or aldehyde enolates. They are used when less reactive electrophiles are under consideration. For example: OLi O Me Me
syn relationship

Decreasing Electrophilicity

X
no reaction

However:
Metal R N I

Me Me N
good yield

Me

Me Me

O R C OR O H 2C CH2 I

Metalloenamines are reactive enough to open epoxides in good yield. Ketone enolates are only marginally reactive enough for this family of electrophiles. O
Li-NR2

Me2CH

R N Li CH2 N R OH

Me

O R C NR2

Nature uses enamines, "stabilized" enolates, and enol derivatives in CC bond constructions extensively.

24-01 Enolate Reactivity Grid 11/9/03 11:46 AM

D. A. Evans

C versus O Enolate Reactivity: Hine "Least Motion Principle"

Chem 206

Question: Why do we generally show enolates reacting with electrophiles at carbon as opposed to oxygen ?? Let's begin the the discussion with an observation: "As electrophile reactivity increases, the percentage of reaction at the enolate oxygen increases." For example, consider the reactions of cyclohexanone enolate with the two electrophiles, methyl iodide and the much more reactive acetyl chloride:
O OEl

C versus O Enolate Reactivity: Enolate Acylation


Kinetic CAcylation of ketone enolates can be carried out:
OLi R1 R2
+

O 78 C NC 1 OMe fast R1

Li

O OMe 2 CN

R2

Enolate acylation with 1 is fast 1


El(+)

slow

El(+)
O

C/O Rxn Ratio << 1 >> 1

Intermediate 2 breaks down to product more slowly than the acylation step Under these conditions, proton transfer from product to enolate does not occur. See accompanying "Acylation Handout"
O R1 R2 O OMe
+ LiCN

Me C Cl O O Me I

El

Kinetic Acidities nicely illustrate LNM Principle: *Lecture 18" pKa(H2O) ~10
O H O H 3C N O

The very reactive acid chloride gives almost exclusively the O-acylation product while the less reactive methyl iodide affords the alternate C-alkylation product. These results may be understood in the context of qualitative statements made by Hammond (The Hammond Postulate) and Hine (The Principle of Least Motion) The Principle of Least Nuclear Motion: "As reactions become more exothermic, the favored reaction becomes that path which results in the least structural (electronic) reorganization." Hine in Advances in Phys. Org. Chem. 1977, 15, 1-61 (handout) Since the X-ray data clearly support the picture that resonance structure 1 best represents the enolate structure, highly reactive electrophiles will favor O-attack according to Hine's generalization. See reinforcing examples on the accompanying page.
24-02 C vs O Enolate React 11/9/03 12:16 PM

Base rel rate: 10 Base rel rate: 1


+6

pKa(H2O) ~10

O H 2C N O

Proton kinetically controlled transfers from C-H Bonds are slow due to the extensive reorganization required in conjugate base. Leaving Group Ability: Stirling, Chem. Commun. 1975, 940
O Ph S O base O Ph S O rds O Ph S O O Ph S O

X
O

Ph S The greater the structural OPh O reorganization of the leaving group during E1cb elimination, the slower krel = 1 the rate of elimination. pKa = 10

CN

krel = <108 pKa = 9.5

D. A. Evans

C versus O Enolate Reactivity: "The Hammond Postulate"


Strongly Endothermic Reactions H > +20 kcal/mol B A T

Chem 206

The Hammond Postulate is also relevant to this issue and is broadly used to make qualitative statements about transition state structure. Hammond, JACS 1955, 77, 334 In attempting to grasp the Hammond Postulate, let's consider two extreme reactions, one which is strongly endothermic and one which is strongly exothermic. T B
Energy

SN1 reactions are typical examples of strongly endothermic processes Hammond: "The transition state will look like products."

Energy

Strongly Exothermic Reactions H > 20 kcal/mol B A

Rxn Coordinate

G A
Rxn Coordinate GA GE H Me3C H OTs

Hammond Postulate "For strongly exothermic reactions, the transition state T looks like reactant(s) e.g. B." As applied to the enolate-electrophile reaction, for very exothermic reactions, e.g. the reaction with acetyl chloride, the transition state for the process will involve little enolate structural reorganization. Hence in this instance the electrophile heads for the site of highest electron density

Does GE GA = G (E A)??
GA: 26.9 GA:27.6 OTs Me3C H H

Carey & Sundberg: Part A; Chapter 4, pp217-220 for discussion of Hammond's Postulate
Based upon the above discussion draw a detailed mechanism for the protonation of cyclohexanone enolate.
O H+ O Me3C

G = +0.50.7 kcal/mol H OTs

G= +0.7

E
Wnstein & Holness, JACS 1955, 55, 5562

24-02a Hammond Postulate 11/10/03 8:31 AM

D. A. Evans
Review

Enolate Alkylation: Stereoelectronic Control Elements

Chem 206

Evans, D. A. Stereoselective Alkylation Reactions of Chiral Metal Enolates.; Morrison, J. D., Ed.; AP: New York, 1984; Vol. 3, pp 1-110.

Examples where stereoelectronic factors are dominant


Pilli, Tetrahedron, 1999, 55, 13321

Stereoelectronic Issues
Enolization: Breaking CH bond must overlap with CO in TS Alkylation: Forming CEl bond must overlap with CO in TS H C C R H base R M O H C C R
El(+)

LDA O N Boc Me RX

R O N Boc Me

RX BnBr AllylBr

ratio >99:1 93:7

R M O

Issue: Degree of rehybridization in TS?

good illustration of the impact of allylic strain Me


Me

Me C H H N O H O R

El R M O H C C R

R M O

El C C R H

RO O

H N

H O H

LDA

H C Boc N LiO RO

RX

Cyclohexanone Enolate:
El(+)

a
H

The C19 Angular Methyl Group in the steroid nucleus


H path A C C OLi favored Me3C O chair conformation R O Me O R Me-I R O (90%) one isomer Me
H H H

Me3C R e

El Me O
H H

OH Li/NH3 R H LiO
H

Me

OH

El(+)

H path E disfavored Me3C El

Me-I Me OH

Me3C R El twist boat conformation Metal Li Li

O Ratio, a:e 70:30 83:17

R-substituent Me CO2Me

Electrophile CD3I Me-I

Chair vs boat geometries not stongly reflected in diastereomeric TS s. The transition states is early and enolate-like.

The enolate (Chem 3D)

24-03-Enolate alk-1 11/10/03 8:46 AM

D. A. Evans
Steric Effects
El(+) a Me3C H El(+)

Enolate Alkylation: Steric Control Elements


In this case, both e and a paths are stereoelectronically equivalent. Diastereoselectivity is now determined by the differential steric effects encountered in the two TSs. CO2Me OMe Me3C El OLi El H E Me3C -78 C e CO2Me H Electrophile Me-I n-Bu-Br

Chem 206

Cases with Opposed steric & electronic effects


R El(+) LiO Me H Li/NH3 R R H H O Me Me Me El(+) Et-I CD3I CD3I Et-I Ratio 95:05 83:17 07:93 >5:95 O El
H

R + O Me

Me El Dominant Control element


stereoelectronic stereoelectronic steric steric

A
Ratio, E:A 84:16 87:13

Representative cases
CO2Me Me LDA MeI Me LDA MeI Me CO2Me Me Me CO2Me Me Ratio, 80 : 20

Li
The enolate R = Me (Chem 3D)

CO2Me

CO2Me

Me

CO2Me Ratio, 90 : 10

Me

Me

Me

Based on above data, this case is reasonable:


C 3H 5 O Ph3COCH2 O H H O Me O H LDA MeI Me H LDA allyl-Br Ph3COCH2 O H Me O Ratio, >97 : 3 O O H CO2Me O Ratio, >97 : 3 O H CN OTHP NaH Me-I Me LiNH2 Me-I Me (58%) >90 : 10 O H NC Me OTHP (67%) 93 : 7 O Me H CO2Me

diastereoselectivity depends stongly on O-protecting group

However
Me

Me

24-04-Enolate alk-2 11/10/03 8:46 AM

D. A. Evans

Enolate Alkylation: Unusual Cases


Sterically Expected Results:
O H N O OLi OMe BnBr 97% ds MeI >98% ds BzCl >95% ds t-Bu O O N H R

Chem 206

Cases which do not appear to give the expected product based on the analysis of steric effects
CO2Me LDA R allyl-Br Me Me O H R C 3H 5 CO2Me

Me Me

O O

t-Bu

CO2Me

R = Me, Et, CO2Me

88 : 12
Seebach, Angew. Chem. Int. Ed 1981, 20, 1030 Ladner, Angew. Chem. Int. Ed 1982, 21, 449

Seebach, Helv. Chim. Acta 1987, 70, 1194.

Contrasteric relatives:
t-Bu O O OLi S-t-Bu MeOD >95%ds 60% ds BnBr acetone 95%ds t-Bu O O R COS-t-Bu

However:
Me Me O O CO2Me LDA

Me Me

acetone CO2Me

CO2Me MeO2C Me Me CO2Me O Me O OH O O Me O HMe Me H O

Seebach, Helv. Chim. Acta 1987, 70, 1194.

Li

ratio, 80 : 20

OLi t-Bu O OMe O Me OLi t-Bu O OMe Me


MeI MeI

t-Bu O

Me CO2Me 70 : 30 Me

The enolate (MM-2)

t-Bu Me O

Me CO2Me Me

Here is another example of a contrasteric alkylation


HO2C (+)-menthylO2C Me Me
(+)-menthol

Me O HO2C Me

>95 : 5

HO2C
LDA, conditions

Ladner, Chem. Ber. 1983, 116, 3413-3426. Me CO2R X R-Cl R-Br CO2R Ratio 80:20 02:98

conditions THF, 23 C THF-HMPA -78-20 C

Those factors defining olefin face selection are currently being defined: Would you have predicted the outcome of the following reaction? OSiR
O
3

R3SiO EtO OSiR3

HgI2 EtO2C

>94 : 6
OSiR3

OH CMe2

K. Yamada, Organic Synthesis Past Present, and Future, p 525

Danishefsky J. Org. Chem. 1991, 56, 387

24-05-Enolate alk-3 11/10/03 8:47 AM

D. A. Evans

Chiral Enolate Enolate Alkylation Circa 1978

Chem 206

Chiral Enolate Design Requirements Circa 1978


Overall enantioselection will be the sum total of the defects introduced through:
Enolization selectivity Enolate-electrophile face selectivity Racemization attendant with Xc removal

Enolization selectivity: Amide-based controllers XC limited by enolization selectivity (Lecture 22)


O Et N Et R R Base LDA (THF) Ratio, (E):(Z) 0 : 100 25 : 75 N Li H Me Et N Et R Me LMNR2 OLi Et N Me R N OLi Et Me (Z) Me O Li H H N Et Et

N Et

(E) Et H O

disfavored

O XC R
enolization

favored

OM XC
El(+)

s-BuLi (THF) R

Amide Based Chiral Auxiliaries


Li O Me O N CH2OH M O
2 LiNR2

O Me N

O
El(+)

CH2OH N

Me El

O RO El R
hydrolysis

XC El

R With Takacs,Tetrahedron Lett. 1980, 4233

diastereoselection Ca 95 %

Allylic Strain controls Enolate Geometry: Enolization selectivity: Ester-based chiral controllers XC limited by enolization selectivity (Lecture 23)
O RX Me Base LDA (THF) LDA (THF) OLi LMNR2 RX (E) OLi H strongly favored O Me C R N R H O H H C R N R Me strongly disfavored

+ RX
Me (Z) Ratio, (E):(Z) 95 : 5 95 : 5

Me

R-Substituent -OMe, O-t-Bu -S-t-Bu

Allylic Strain Prevents Product Enolization:


El strongly favored O Me C R N R H O El H C R N R Me strongly disfavored

24-06-Enolate alk-4 11/10/03 8:47 AM

D. A. Evans
Chiral Amide Enolates
RX M O Li O M-NR2 Me HOH2C RX N N Me HOH2C N

Enolate Alkylation: Chiral Amide Enolates


O Me (major) R R Me

Chem 206

Amide Hydrolysis
O N CH2OH
H
+

O R HO N Me

O O H2 N +
HCO3H2O, 5 min

HOH2C N

O O Me (minor) R R Me OH

H R O C H O O H N H
H 2O R

O O Me

HN

Evans, Takacs, Tet. Lett. 1980, 21, 4233-4236


Me Me

Br

96:4 (98%)

intramoleclar general base catalysis

Br

98:2 (84%)

Applications in Ionomycin synthesis Ionomycin Calcium Complex


JACS 1990, 112, 5290-5313 O Me
14

Chem 3D model

Li Li

Li

Me Me
17

Me H

Me Me OH O

OH OH Ca O
14 12

O N O

O
9

LDA RX The nature of enolate chelation is ambiguous. Nitrogen chelation is a real possibility. Me N OH Me O Me Ph
14

Me2HC O Me
14

Me O O 84% Ph

Me O
14

Me O N

Me

Me

Li

Me2HC O
12

PhCH=CHCH2Br

Me Me2HC diastereoselection 99:1 K Li O N

CH2OH N 83% Ph
14

O I Me

Myers, JACS 1997, 119, 6496

Me Me diastereoselection 97:3

Me

24-07-Enolate alk-5 11/9/03 9:05 PM

D. A. Evans
Alkali Metal enolates:
R LDA or NaNTMS2 O R2 R O M N Me2CH O M N Me O O Ph

Enolate Alkylation: Chiral Imide Enolates


O El(+) R El O R O O El(+) R El Alkyl Halide ArCH2Br CH2C=CHCH2Br ArCH2OCH2Br marginal reaction CH3CH2I CH3I Ratio 50-120 : 1 50 : 1 50 : 1 25 : 1 13 : 1
Me2HC

Chem 206
O O N N3 Bn O
JACS 1987,109, 6881. JACS 1990,112, 4012-4030

Enolate Amination
XC M=K M N Bn M = Li
CHMe2 SO2N3

Trisyl-N3 HOAc O O

O R N R1

diastereoselection 91-99+ %

O XC

O BocN=NBoc R N

O O JACS 1986,108, 3695.


Tet. 1988, 44, 5525-40

enolization selectivity >100:1

BocHNNBoc Bn diastereoselection 97-99+ %

M = Li, THF < 0 C M = Na, THF -78 to 0 C

(Trisyl-N3) CHMe2

Enolate Hydroxylation
O R N Me Na enolate is required. Why? O O Ph Imide (R) PhCH2CH2=CHCH2MeO2CCH2CH2CH2PhMe3CPhHC O NSO2Ph R OH Me Ratio 94 : 6 95 : 5 96 : 4 90 : 10 >99 : 1 Ph Yield * 86 % 91 % 68 % 77 % 94 %
Na-N(TMS)2

Enolate Acylation
O Me N Me2CH O O
LiNR2

JACS. 1982,104, 1737. O O R Cl Et O N Me Me2CH Diastereoselection ~ 97 : 3 O O JACS 1984, 106, 1154.

O N

O O

New stereocenter not lost through enolization

JACS. 1985,107, 4346.

X-ray structure
24-08-Enolate alk-6 11/9/03 9:06 PM

For all indicated rxns, as the R on the enolate grp increases in size enolate-El face selectivity increases. Explain.

D. A. Evans
Chiral Ester Enolates

Enolate Alkylation: Chiral Ester Enolates


Helmchen, Angew. Chem. Int.Ed. 1981, 20, 207-208 Me Me

Chem 206

Chiral -Keto Ester Enolates


Me2HC

CO2t-Bu N El O OMe A OMe Me2HC


HMPT

Li N Me Me

(LiCHIPA) THF

Me

Me

Me

SO2Ph N O R H O Me

Me
LiCHIPA THF, HMPA 4:1

CO2t-Bu Me2HC NH O OMe


SO2Ph N H Me

RO Me2HC
LDA toluene

O N Li O

THF

SO2Ph N Me

Me
Me

Me

CO2t-Bu N El O OMe B

O Me
H

H LiO Me (E)-enolate
El(+)

O Me Me LiO Me (Z)-enolate
H
El(+)

Rationalize the effect of HMPA on the stereochemical outcome of reaction. El(+) MeI Koga, JACS 1984, 106, 2718-2719 MeI Bn-Br Bn-Br addend THF Yield 63% 57% 48% 77%

Ratio (A:B) 96:04 01:99 99:01 15:85

Me

Me

SO2Ph N Me X O O

Me

Me

Me
H

El El(+) n-C14H29I n-C14H29I

SO2Ph N El X O O

HMPT THF HMPT

Me
H

Me

enolate (E) (Z)

Ratio 98.5:1.5 06:94


enolate contamination

Chiral -Keto Ester Dienolates


Me Me OLi N Me
t-BuLi El(+)

Me Me O N Me Me E(+) = MeI, EtI, BnBr diastereoselection 98% O Major diastereomer

Helmchen, Angew. Chem. Int.Ed. 1984, 23, 60-61 Helmchen,Tet. Lett. 1983, 24, 1235-1238 Helmchen,Tet. Lett. 1983, 24, 3213-3216 Me Me SO2Ph N O O Me Me Me Br
KO-t-Bu

Me

SO2PhH

Me Me O N
Rationalize the stereochemical outcome of reaction.

Me
H

N X O Me
H

H Me

Ratio, 93:7 (74%)


Helmchen,Tet. Lett. 1985, 26, 3319-3322

Schlessinger,Tet. Lett. 1988, 29, 1489-1492

24-09-Enolate alk-7 11/9/03 9:09 PM

D. A. Evans
O EtO n-C4H9 H O EtO n-C4H9 H
LiNR2 MeI

Allylic Strain & Enolate Diastereoface Selection


OTs Me
LiNR2

Chem 206
Ph
NH4Cl

O EtO n-C4H9 O EtO n-C4H9

Me diastereoselection 98:2 H Me3Si

Ph

OM OMe R
R-substituent R = Me diastereoselection 87:13 80:20 40:60

O OMe R

Me3Si

Me diastereoselection 89:11 H Ph Me3Si O

R = Et R = CHMe2

major diastereomer opposite to that shown

I. Fleming & Co-workers, Chem. Commun. 1985, 318. Y. Yamamoto & Co-workers, Chem. Commun. 1984, 904.

D. Kim & Co-workers, Tetrahedron Lett. 1986, 27, 943.

Ph
LiNR2

O OBn diastereoselection 90:10 at C3


one isomer at C2

OBn

MeCHO Me3Si 71% yield

MeO

p. p.

RO2C H CO2Me

MeO
LiNR2

RO2C H H CO2Me

O "one isomer" Me Bn N Boc O N S S


Sn(OTf)2 RCHO 91-95%

Me

OH

I. Fleming & Co-workers, Chem. Commun. 1986, 1198.

Br
95% yield

Me Bn N Boc R

G. Stork & Co-workers, Tetrahedron Lett. 1987, 28, 2088.

O N OH

S S diastereoselection >95%

TBSOCH2

Me CH2
LiNR2 MeI

Me CH2 TBSOCH2 Me H H CO2Me


T. Money & Co-workers, Chem. Commun. 1986, 288.

T. Mukaiyama & Co-workers, Chem. Letters 1986, 637

"one isomer" Me

O (MeS)3CLi OMe MeI


86%

H CO2Me

MeS MeS MeS

Me

O OMe diastereoselection 99:1

Me

K. Koga & Co-workers, Tetrahedron Letters 1985, 26, 3031.

R PhMe2Si

OM OEt

R
MeI

O OEt Me
R = Me: diastereoselection 99:1 R = Ph: diastereoselection 97:3

PhMe2Si

I CO2Et R

OLi O-t-Bu

CO2Et

R = H: one isomer

KOt-Bu THF -78 C I. Fleming & Co-workers, Chem. Commun. 1984, 28.

CO2-t-Bu R = Me: > 15 :1 H R

Y. Yamaguchi & Co-workers, Tetrahedron Letters 1985, 26,1723.

24-10-enolate alk/A-strain 11/9/03 9:10 PM

D. A. Evans
Seminal Paper: Stork & Dowd, JACS, 1963, 85, 2178-2180 Reviews:

Metalloenamines-1
Representative Reactions:
Me Me
EtMgCl H3O+

Chem 206

Me

Martin in Comprehensive Organic Synthesis, 1991; Vol 2, Chapter 1.16, pp 475-502 Whitesell Synthesis, 1983, 517-535 Bregbreiter in Asymmetric Synthesis, 1983; Vol 2, Chapter 9, pp 243-273 Enders in Asymmetric Synthesis, 1984; Vol 3, Chapter 4, pp 275-339

N
conditions: base + R-X in refluxing THF

Me2CHI

Me Me

N
60% overall

O Me

Me Me 83% overall O Me H

Generation & Structure:


M pKa~ 29-33 N R (Z) anion
base

R
El(+)

R El syn product Me Me CMe3 N

EtMgCl MeI

H3O+

EtMgCl n-BuBr H3O+

H3O+

Me Me

60% overall

The base: RLi; RMgX; R2NLi (E) anion

M
El(+)

N El anti product

Stork & Dowd, JACS, 1963, 85, 2178-2180

Nature of N-substituent, base, and solvent additive can play a role in


deprotonation regioselectivity: Hosomi, JACS, 1982, 104, 2081-2082 Me +
base H 3O

Acidity Measurements: (Streitwiser, JOC 1991, 56, 1989; Fraser, ibid. 1985, 50, 3234):

Kinetic product geometry strongly favors the syn isomer (>99%) (Fraser)

Fraser, JACS 1978, 100, 7999 Fraser, Chem. Commun. 1979, 47

N Me

MeI

O Me R base Me Me ratio 10:90 100:0 O Bu Bn R base Bn ratio 74:26 100:0

Solid State & Solution Structure:


X-ray structure reveals the following: Anion geometry is (Z) For M = Li, anion is delocalized rather than localized as pictured Collum, JACS 1984, 106, 4865-4869 Collum, JACS 1985, 107, 2078-2082 Collum, JACS 1986, 108, 3415-3422 Collum, JACS 1993, 115, 789-790 R nonbonding N-lone pair may be stabilized by delocalizatin into antibonding orbital of C=C. Remember, (Z) geometry also favored for enol ethers Me

cyclohexyl s-BuLi s-BuLi NMe2 N R


base H3O+

O Bu

Geometry Rationalization:

Me

BnBr

Me

+ 1 equiv HMPA

cyclohexyl s-BuLi cyclohexyl s-BuLi

24-11-met-enamines-1 11/9/03 9:11 PM

D. A. Evans
Stereoelectronic Issues:
N Bn LDA Li N Bn H MeI Me3C H CMe3 CMe3 X Ratio, 97:3 Me Me3C Me H

Metalloenamines-2
Chiral Metalloenamines:
early papers: H full papers: X R2 O M N Me H Ratio 94:06 60:40 N H LDA Me3C MeI Me Me3C NC NNMe2 Ratio, 96:04 Me NNMe2 Meyers, J. Am. Chem. Soc 1981, 103, 3081 RX MeI EtI n-PrI H CN Bn
LDA El(+) H3O+

Chem 206

Meyers, J. Am. Chem. Soc 1976, 98, 3032 Whitesell, J. Org. Chem. 1978, 42, 377-378 Meyers, J. Org. Chem 1978, 43, 892 Meyers, J. Am. Chem. Soc 1981, 103, 3081 Meyers, J. Am. Chem. Soc 1981, 103, 3088 R2 O

Fraser, JACS 1978, 100, 7999 (handout) Tendency for axial-chair alkylation is significantly greater that for ketones H Me3C H X Me H Me3C H X X Me Me H Me3C X H Me H

R1
base

R1
El(+)

R El

LDA MeI

Fraser, JACS 1978, 100, 7999 (handout) X = N-Bn X=O NNMe2 CN

The base: RLi; RMgX; R2NLi MeO

Major Product

O El

ee 87 94 99

CMe3

NNMe2 Me LDA MeI

H Me

Me H NNMe2 Ratio, 90:10 Me

H Me NNMe2

Chiral Metallated Hydrazones


N CH2OMe LDA
RX

N R CH2OMe

Collum, JACS 1984, 106, 4865-4869 (handout)


Enders in Asymmetric Synthesis, 1984; Vol 3, Chapter 4, pp 275-339

24-12-met-enamines-2 11/10/03 8:42 AM

D. A. Evans, K. Scheidt

Metalloenamine X-ray Structures


LI

Chem 206

LI

LI

Me Li N N Me H Li N H

LI

LI

Collum & Clardy, JACS 1984, 106, 4865

LI

24-13 Metalloenamline X-rays 11/3/00 7:34 AM

D. A. Evans
Chiral Metallated Hydrazones
Me N N CH2OMe
LDA

SAMP-Metalloenamine X-ray Structure

Chem 206

O Li N N
RX

MeO N N R

THF deleted

Li

H N N Li O Me R

Me R O N N Li B H R diastereotopic face attacked by El(+) R

A (Enders)

Which of the reactive chelate conformations are we to begin our analysis from?

Li

diastereotopic face attacked by El(+)

For a review of this methodology see Enders, D. in Asymmetric Synthesis.; Morrison, J. D., Ed.; AP: New York, 1984; Vol. 3, p 275-339.

24-14 X-ray 11/10/03 8:37 AM

Chemistry 206 Advanced Organic Chemistry


Handout24A

Enolate Acylation

D. A. Evans
H24-00-Cover page 11/10/03 8:03 AM

Monday , November 10, 2003

D. A. Evans
The Reaction:
Acylation R1 R2 OM Carboalkoxylation R1 R2
+

Enolate Acylation Acylation & Carboxylation


OM
+

Chem 206

O X O X OR3 R1 R3 R1

O R3 R2

Deacylation: When an acyl residue is employed in the one of the illustrated bond constructions, it may then be removed by nucleophilic deacylation: Several examples are provided. Deformylation:
O Me CHO O
Me Me HCO3

O Me Me Me

O OR3 R2

Me Me

Situations where the reaction is employed: Acyl moiety is a constituent of the target structure:
O OH R1 R2 R1 R2 OR3 R1 O OR3 R1 O R2 O O OR3 R1 O
() +

competitive ring cleavage not a problem due to more electrophilic formyl C=O

Decarboxylation:
O X (+) OR3 O X
+

Alkyl-Oxygen Cleavage: tert-butyl esters


O
NaH, DMF

O CO2-t-Bu
RBr

O R CO2-t-Bu H
CF3CO2H

R2 O

OR3 R2

O O

CO2

Acyl moiety employed in assisting bond construction but not part of the target structure:
O R O R O H O
()

O H
+ RX (+)

Decarboxylation in this system is a sigmatropic rearrangement involving C=O participation representative procedure: Henderson, Synthesis 1983, 996

Alkyl-Oxygen Cleavage: Methyl esters


O R CO2-Me
LiI/H2O

Me

Me

O CO2Me Me CO2Me CO2Me


R=H

R CO2

H3O+ CO2

O R

Me

Me

Me

Me

CO2Me O

O O OR

O C O

HO

leading references JOC. 1991, 56, 5301-7 Tet Let. 1990, 31, 1401-4

Acylketene intermediate

H24-01-Acylation Intro 11/5/00 5:20 PM

D. A. Evans

Claisen Condensation & Related Processes


Analysis of the two processes:
O R R Me CO2Me CO2Me
RO H3O+

Chem 206

Claisen Condensation: Condensation of 2 esters


O R OR
+

O
RO

O OR

Conventional Carbomethoxylation: Equilibrium achieved between all species


O
Me2CO3

OR H O+ 3 R

CO2Me O
Keq ~ 10+4

CO2Me O

Intramolecular Variant: Dieckmann Condensation


Me O CO2Me

A
+ MeOH

Keq ~ 10-2

+ MeO

O Keq > 10+4 CO2Me

Strictly speaking, the Claisen and Dieckmann condensations are defined as condensations between ester enolates & ester electrophiles. In this discussion, we choose to liberalize the classifcation to include ketone enolates as well.

Me2CO3

B
CO2Me

Reaction Thermodynamics: Overall Keq ~ 1


O RO
+

O OR R R
+4

O OR + RO

2 R

Critical issue: Product enolate A is significantly destabilized by peri-interaction with aromatic ring disrupting the required planarity of the delocalized enolate. Hence, the greater stability of B dictates the product. MeO O H O O Keq >> 1

Final enolization Step: Keq ~ 10


O RO

A
O OR R
+

CO2Me

O OR R pKa 12 R

This type of control is general:


ROH
pKa 16

CO2Et
HCO2Et KOtBu

CO2Et OH Me Me O
Meyers, JOC 1976, 41, 1976

Contrary to popular belief, final enolization step does not render the process irreversible

Reaction Control Elements: These reactions can be manipulated to give either kinetic or thermodynamic control:
CO2Me
LDA

Me

Me

HCO2Et

OH Me O Me Me O OH
JACS 1965, 87, 5728 Piers, Tet. Let 1968, 583

O NC

-78 C

O kinetic product

Me

O Me

MeO benzene

OMe
HCO2Et MeO benzene

NaH

O MeO

0 C

OMe

Thermodynamic product

Me

CO2Me

H24-02-Claisen Condensation 11/5/00 5:17 PM

D. A. Evans

Kinetic Enolate Acylation: The Mander Reagent


The Tetrahedral Intermediate 2; Why is it so stable?
Li O OMe 2 CN R1 R2 O Li O
slow

Chem 206

Kinetic Acylation: Methyl Cyanoformate (1):


OLi R1 R2
+

O -78 C NC 1 OMe fast R1

O R1 R2

O OMe
+ LiCN

R2

OMe CN 2

Enolate acylation with 1 is fast Intermediate 2 breaks down to product more slowly than the acylation step Under these conditions, proton transfer from product to enolate does not occur.
R1 O

slow

Consider this process in the broader context of elimination reactions of the E1cb classification where: O OMe R2 O CO2Me 84% O
+ LiCN

Y might be either C or some heteroatom X might be various leaving groups such as CN, OR etc. R X R Y H Data is available for the case where X = CN, OR & Y = carbanion: Stirling, Chem. Commun. 1975, 940-941 FG H
base base

R X

R Y
slow

R R Y
+ X

Examples: O
LDA

1 O Me O Me Me OTMS Me
Me-Li LDA

Me

Me O Me

CO2Me

65%

FG

slow +

FG
X

LDA

CO2Me
Mander Tet. Lett. 1983, 24, 5425

leaving grp (X) OPh CN C(Me)2-NO2 OMe

pKa HX 10 9.5 ~10 16

log

kX kOPh 1 <-7 <-9 -3.9

1 CMe3 O
R2Cu(CN)2Li2

Me3C H

O Me CO2Me 75%

+ isomer

7%

O 1 CO2Me

Mander, SynLett. 1990, 169

Above data makes the point that CN is a poor LG but it also leads one to the faulty conclusion that 2 should partition to acyl cyanide rather than methyl ester! O O CN R2
+ LiOMe

O R1

Li

O OMe CN 2 R1

O OMe R2

OTBS 82%
Hashimoto, Chem. Lett. 1989, 1063

R1

R2

LiCN

H24-03-Mander Reagent 11/5/00 5:21 PM

J. L. Leighton, D. A. Evans

Carbon Acylation with N-Methoxy-N-methylamides


O OR O N Me Li OMe Nu O O CbzHN R Bn N Me
THF, -78 C 73% H3O+

Chem 206
OR MeO2C O O P(OMe)2

Acylating agents can be desiged where the tetrahedral intermediate exhibits exceptional stability:
O R N Me
Nu(-)

O R Nu

Li O Me N Me
H3O+

MeO2C O R

P(OMe)2

THF/Et2O -110 C to -80 C 62%

OMe O R' N Me

P. Thiesen and C. Heathcock J. Org. Chem. 1988, 53, 2374.

BrMg

OEt CbzHN Bn

O OEt

R-Li or R-MgBr THF, 0 C

R'

OMe

OMe

R = Me, n-Bu, or Ph; yields > 90%


Weinreb Tet. Lett. 1981, 22, 3815.

M. Angelastro, N. Peet and P. Bey J. Org. Chem. 1989, 54, 3913.

O Me N N Me
R1M H3O+

O R1 N Me
R2M H3O+

O R1 R2

Nucleophiles: Acceptable
RLi, RMgX OLi RO DIBAL R LiN R LiAlH4 Li Ar Li(MgX) O
S

Unacceptable
RZnX & other colalent metal alkyls OLi CH2Li R other colalent metal enolates Weak hydride reagents: NaBH4

MeO

OMe

OMe
W. Wipple, H. Reich J. Org. Chem. 1991, 56, 2911-2.

Enolates and Metalloenamines:


OLi O O N Me OLi Me
THF, -78 C to R. T.

O t-Bu

O O t-Bu 83%

O LiB(R)3H

THF, -78 C

O Me 47%

OMe An excellent review on all aspects of Weinreb amide chemistry: M. Sibi, Organic Preparations and Procedures Int., 1993, 25 (1), 15-40.

Hydride Reductions: Representative Organometals:


OTBS Me N O Ar O N Me OMe
MeMgBr THF, 0 C 99%

J. Org. Chem. 1989, 54, 4229.

OMe OMe OTBS Me N O Ar


J. Prasad and L. Liebeskind Tetrahedron Lett. 1987, 28, 1857.

OMe

TBSO

O N OMe
DIBAl-H THF, -78 C

O Me

O2N Me OMe O2N


D. Evans and S. Miller J. Org. Chem. 1993, 58, 471.

OMe Me

Me

Me

Me

95%

OMe OMe

OMe

TBSO

O H

Several other examples reported.

Me OMe

OMe Me

Me

Me

H24-04 Weinreb Amides-1 11/5/00 5:22 PM

J. L. Leighton, D. A. Evans

Carbon Acylation with N-Methoxy-N-methylamides-2


Me Me O Me
26 32 A

Chem 206
Me

The Rutamycin B Synthesis, H. Ng, Ph. D. Thesis, Harvard University, 1993


Me Li N NMe2 I O Me O Me N NMe2 Me O Me O

OR O
7

OR
11

O Me
16

O
D
27

29

H O
F
35

17

21

O OR

Me
26 32

97%

OH

Me

Me

Me

Me

O Me OMe Me

Me OH

Me

C1-C16 Subunit

C17-C37 Subunit
Me

Et

Me TESO
20

Me

Problem is to control C=O reactivity on central D-fragment


O H M
17

O N Me Me OMe

LDA

O
D

Me H O Me OMOP O
28

O
29

H O
F

PMBO Et

23

80%
Me Me O NMe2

X Me Me

21

OBn Me

Et

TESO
20

O
32

O
23 26

O
D
21

PMBO HF MeCN-H2O

Me H Me
5

MeHO
28 30

Me
F
35

80%
Me HO
H H
20 32

Et

Me Me O Me 33
H H H H

Me Me OH O
8
1

Me

OH

O Me

Me OH

O
H

Et

Me O
O

O O 25 Me

OH

Me Me O

The Solution:
OH

Me H

Me Li Me CH2 Me Me Me O MeN H Me Li O Me

Me H O Me OMOP N
29

PMBO Et

Et2O, -78 C

Me

Me

26

several steps
Et

Me Me

C20C21

MeO

NMe2
Me

20

HO

12

Evans, Rieger, Jones, Kaldor, JOC, 1990, 55, 6260-6268


17

Me

MOP =

C OMe Me

tetrahedral intermediate stable for hours at 0C

The X-206 Synthesis, S. L. Bender, Ph. D. Thesis, Harvard University, 1986


Me Me O Me Me O OH
A A

H O Me N CH2 OMOP

NMe2 O Me H O Et H OBn Me Et OBn Me

Me OH O
7 12

Me
C

Me
21

Me
D
27

29

LDA, 0 C
H O
F
35

15

OH

Me

OH

OH OH

O O Me OH Me

83% C C 28 29
Me OH Me
30

Et OH
7

Me H

Me

Me O

OH
11

O
15

OH

O
D
21 27

MeHO

O
1

Me

Me

Me

Me

OH

O Me

Me O
35

Me OH

O H O Me Me NH O CH2 OMOP NMe2

Et

Evans, Bender, Morris J. Am. Chem. Soc. 1988, 110, 2506.

H24-05-Weinreb Amides-2 11/5/00 5:25 PM

D. H. Ripin, D. A. Evans

The Eschenmoser Coupling Reaction


N
(+)

Chem 206

Key Bond Construction Needed for the B12 Synthesis:


N
(+)

H N

O
()

The Problem:

H H N

O
()

H N

R 3P

R3P=S

H N

H N

N H

The Solution:
O O Me N H Me (CH2)2CO2Me
(+)

N S

O O Me H N O Me Me (CH2)2CO2Me N Me (CH2)2CO2Me N N H S X H N

O
()

??

E. Knott J. Chem. Soc. 916 (1955)

H N O

Me Me (CH2)2CO2Me

The General Reaction: Acylation of an Amide C=O


O S R O X R3
Base, Thiophile

R3 R N R R

MeO2CCH2 Me O R2 Me R2 Me R2 Me R3 OO NH N Me H N Me Me R3 N Me R3 MeO2CCH2 Me MeO2CCH2 Me O NH N Me () (CH2)2CO2Me

O O Me H N O (+) Me Me (CH2)2CO2Me N Me R

N R

Key papers:

A. Eschenmoser Helv. Chim. Acta. 54, 710 (1971) A. Eschenmoser Angew. Chem., Int. Ed. Engl. 6, 866 (1967) A. Eschenmoser Angew. Chem., Int. Ed. Engl. 8, 343 (1969) Review: Trost Comp. Org. Synth. Vol. 2, Ch. 3.7 (1991)

S N R R Reagents R N R R

??

The Thioamide component: R


p-MeOPh P S S P PhOMe S S

A. Eschenmoser Science 196, 1410 (1977)

P4S10 Lawesson's Reagent P4S10, Et3N or NaHCO3

RCS2R' + R2NH Imidate +H2S

JOC 46, 3558 (1981), Synthesis 149 (1973) Bull. Chim. Soc. Belg. 87, 229 & 293 (1978) Indian J. Chem., Sect. B 14, 999, (1976) JACS 102, 2392 (1980) Chem. Ind. (London) 803 (1974) Thioamide ThioamideAngew. Chem. 79, 865 (1967)

H24-06-Eschenmoser-1 11/5/00 5:27 PM

D. A. Evans and P.H. Carter The Dieckmann Condensation

Intramolecular Enolate AcylationDieckmann Condensation


Regioselectivity:
EtO2C
A

Chem 206
ONa

Reviews: Schaefer, Bloomfield, Organic Reactions 1967, 15, 1.


Davis & Garratt, Comprehensive Organic Synthesis 1991, 2, 806-829

CO2Et CO2Et

NaH/C6H6

CO2Et

Kinetic Control?
CO2Et
A

O CO2RCO2R
( )n ( )n

ONa CO2R EtO2C


B NaOEt/EtOH

EtO2C
B1

CH2CO2Et

CO2Et CO2Et

Thermodynamic Control?
ONa

Accesible Ring Sizes


O CO2R
not viable excellent

O CO2R

O CO2R

O CO2R

O CO2R
NaOEt/EtOH EtO2C

Not observed
CO2Et

Explanation:
excellent acceptable situation dependent high dilution required

B2

The individual steps: Enolization:


EtO
+ base

Enolization at (A) preferred on basis of inductive effects. Hence, Path A preferred in kinetically controlled situation Enolates (B1) and (B2) both more stable than enolate (A) Under equilibrating conditions (B1) appears to be preferred over (B2)

O OEt O EtO

O OEt
+ base-H

The effect of beta heteroatoms: classical kinetic vs. thermodynamic control

A variety of bases may be considered for the enolization step. Either alkoxide or a non-nucleophilic base such as NaH are commonly used. Choice of base can be important (Vide infra).

KOtBu / PhH

MeO

N CO2Et CO2Et
NaOEt / EtOH

CO2Et OH

Ring Closure:
O EtO

Keq (cycliz) ~1 OEt O


+

Keq (enoliz) ~ 10+4 CO2Et


Keq (enoliz)

O CO2Et

MeO

N OH CO2Et

NaOEt EtOH

+ EtOH R.H. Schlessinger, et al. Heterocycles 1987, 25, 315.

EtO

Statements claiming that the final enolization step renders the process irreversible are simply incorrect.
H24-07-Dieckmann-1 11/5/00 5:29 PM

D. H. Ripin, D. A. Evans Reagents for the Reaction: Bases: Inorganic: Organic:

The Eschenmoser Coupling Reaction-2


CO2Bn TfO t-BuO2C O S O O Me
mix Et3N, PPh3

Chem 206

MHCO3, MOH, MH, MOR R3N, N-methylmorpholine, buffered solutions


N

t-BuO2C

N Bn CO2Bn

Me O O

Thiophiles: Ar3P, R3P, (RO)3P Combination:


PhP NMe2 PhP
2

Bn

H. Rapoport J. Org. Chem. 46, 3230 (1981)

N
2

H. Rapoport J. Org. Chem. 46, 3230 (1981) A. Eschenmoser Helv. Chim. Acta. 54, 710 (1971)

CO2Et MeN OAc S Ar


DBU 79%

CO2Et MeN OAc


*

OMe Me
NaH CuBr

CO2Me MeO Me N NMe EtO2C OAc

A. Eschenmoser Science 196, 1410 (1977) O O Me N H Me (CH2)2CO2Me


(+)

O O Me H N N Me Me (CH2)2CO2Me MeO2CCH2 O R2 OO NH N Me H N Me Me R3 R3 Me (CH2)2CO2Me


t-BuOK t-BuOH, 25 C P(CH2CH2CN)3, TFA, sulfolane

N H Me (CH2)2CO2Me N
This center readily epimerizes to a 2:1 mix of diaster. in favor of the shown.

N H

Br CO2Me

99%

a) 1.05 eqiv. (PhCOO)2 b) P(OEt)3, Xylene,

Br

CO2Me

T. Kametani J. Chem. Soc., Perkin Trans. I 1607 (1980)

H O

Me

84% O

Me (CH2)2CO2Me O O O NH N Br S O Me R N

N2

S HN

N2
NaOH

S
ii

O N CO2t-Bu N CO2t-Bu
Ra-Ni, 68%

O CO2t-Bu
Rh(OAc)2

Me Me R3 N MeO2CCH2 Me MeO2CCH2

+ S
S. Danishefsky Tet. Lett. 30, 3625 (1989)

Me R2 Me R2 Me

O N CO2t-Bu

S N CO2t-Bu

H N

Me Me (CH2)2CO2Me R C: R carbenes + S R Thioethers R

64%

R R S+ R R S-Ylids

H24-08-Eschenmoser-2 11/5/00 5:27 PM

D. A. Evans and P.H. Carter

Intramolecular Enolate AcylationDieckmann Condensation


Intramolecular Ketone Acylation
O R EtS CO2Et R.Danieli, J.Org.Chem. 1983, 48, 123. Me Me CO2Et Me Me O NaH Me Me O H

Chem 206

Miscellaneous Dieckmann Reactions of Potential Interest


EtO2C R NaH DMSO O

CO2Et SEt

O H.-J. Liu and Co-workers, Tet.Lett. 1982, 23, 295.

Me

TMS tBOC N CO2Et S CO2Et LDA (tBOC)HN

OTMS OEt S CO2Et

8:1 mixture

CO2Et Me

H KH, THF TsHN N

O no loss of stereochemical integrity O

TsHN J.L. Adams, J.Org.Chem. 1985, 50, 2730. X X Me CO2Et tBuOK / PhH Me

N O

T.M. Harris and Co-workers, J.Org.Chem. 1984, 49, 3681.

O O CO2Et CO2Et Peterset, Recl.Trav.Chim.Pays-Bas 1977, 96, 219. X Me O O Me Me (TMS)2NLi THF, -78oC HO

O O Me Me

When X = NR2, this is a good reaction, but when X = OR, it is a poor reaction.

Deduce the mechanism of this multistep process.


O Me CO2Et G.Stork and Co-workers, J.Am.Chem.Soc. 1960, 82, 4315. NaOEt Et2O Me O O Cl CO2Et R Me Me O O Me

S.Brandawge and Co-workers, Tet.Lett. 1992, 33, 3025.

Cl

O OMe O O OMe Kocienski and Co-workers, Tet. 1990, 46, 1716

LDA -78 C

H24-09-Dieckmann-2 11/5/00 5:30 PM

D. A. Evans and P.H. Carter

Intramolecular Enolate AcylationDieckmann Condensation


Multistep Condensations
O LDA N CO2Me CO2Me Me Heterocycles, 1987, 25, 315 MeO O MeO2C CO2Me Cl CO2Me NaH MeOH MeO O OH OH Cl

Chem 206

Kinetically controlled Cyclizations


CO2tBu CO2Me N Me CO2Me

CO2Me

Me

R Li-TMP

Me

R NMe2 OH JACS, 1979, 101, 5060 Woodward, JACS, 1962, 84, 3222 O OH O OH OH CONH2 O

CO2Me

60-70%
CO2Me H

CO2Me H COSPh N O CO2Bn PhOCH2CONH H H Li-NTMS2 COSPh CO2tBu 3 equiv O Li-NTMS2 N

6-demethyl-6-deoxy-tetracycline

O CO2Bn

Tet. Let, 1981, 22, 3883

MeO2C O MeO2C

DMSO CO2Me 41% O MeO2C

CO2Me

PhOCH2CONH

Danishefsky, JACS, 1973, 95, 2410

N O

N O

OH CO2tBu

O CO2Me Me EtO2C CO2Et EtO 41% O O O CO2Me

Hatanaka, Tet. Let, 1983, 24, 4837 O CO2Me H HO C C OCOMe H Li-NTMS2 3 equiv H LiO C C C O H OLi HO O CO2Me OMe HO O

O Prostaglandin E2 Sih, JACS, 1975, 97, 865 CO2Me

CO2Me

CO2Me

78%

Brandange, JOC, 1984, 49, 927

HO

OH

H24-10-Dieckmann-3 11/5/00 5:31 PM

D. A. Evans

Synthetic Applications of Metalloenamine Nucleophiles


JACS 1993, 115, 11446-11459. Me Me Me Me N Me 97% Me
26

Chem 206
Me O O
D
27

The Rutamycin B Synthesis, H. Ng, Ph. D. Thesis, Harvard University, 1993


Me Me O Me
32
1

OR
7

OR
11

O Me
16

29

H
F
35

Li Me
26

NMe2

O
32

NMe2

O OH Me

17

21

O OR

Me

Me

Me

Me

O Me OMe Me O

C1-C16 Subunit

C17-C37 Subunit
Me H O
F

Et

Me OH

Me

Problem is to control C=O reactivity on central D-fragment


O H M
17

TESO PMBO
20 23

O N Me OMe

LDA

O
D

Me H O Me O OMOP
28

O
29

Et

Me H NMe2

80% Me Me

X Me Me

21

OBn Me

Et

TESO
HF MeCN-H2O

O
32

O O O
D
21

PMBO 80% HO
H H
20

20 23 26

Me H Me
5

Me HO
28 30

Me O
35

Et Me
32

Me Me O Me
33

Me Me OH O
8
1

Me

OH

O Me O

Me OH

H H

O
H H

Et

Me

OH

O O
H
25

Me Me O

The Solution:
OH Me Me

Me H

Me Li Me CH2 Me Et2O, -78 C

O Me N MeO H O Me OMOP N

Me
29

PMBO

Me

Et

Me

26

several steps Et

C20C21
Me Me O Li MeN O Me H O Me N CH2 OMOP

NMe2
Me

20

HO

12

Evans, Rieger, Jones, Kaldor, JOC, 1990, 55, 6260-6268


17

Me

Me

MOP =

tetrahedral intermediate stable for hours at 0C

C OMe Me

The X-206 Synthesis, S. L. Bender, Ph. D. Thesis, Harvard University, 1986


Me Me O Me Me O
1 1

NMe2 O Me H O Et H OBn Me

Me
A

Me
C

Me
21

Me
D
27

OH
7

29

12

LDA, 0 C H
F
35

15

O OH

H Me

OH

OH OH

O Me OH MeO

83% C C 28 29 Me OH Me
30

Et
A

Me H

Me

Me H O Me O Me NH O OMOP NMe2

OH
7

OH
11

O
15

OH

O
D
21

Me HO
27

O OH

Me

Me

Me

Me

OH

O Me O

Me O
35

Me OH

CH2

OBn Me

Et

Et Evans, Bender, Morris J. Am. Chem. Soc. 1988, 110, 2506.

H24-11-met-enamine acylation 11/4/01 7:08 PM

D. A. Evans

Synthetic Applications of Metalloenamine Nucleophiles


Me
Et-Li Et2O

Chem 206
Me Me O Li O Et2NLi MeN
THF

The Ferensimycin B Synthesis, JACS 1991, 113, 7613-7630


Me O HO Me OH Me O
B C

Me Et OH O MeO Me Me
B C

O Li MeN O Me

Me
B
17

Me Me
B

O OH H Me Me

Me

Et

O Me OH Et

Et

Me N

Me Me
B
18

NMe2

Me

Et

18

Me N NMe2 Li

Ferensimycin B

N Me

11

Me O HO

Me
A

O H Me

O H O O Me OH Et Et OH

Et

Me N

Et NMe2 Me O
B
11 21

Et

Me

O OH H Me Me

18

O Mg Br

Et

C10-C23 Synthon The BC Fragment (C10-C23 Synthon)


Me O
B
11

Me

Et

21 Et O Me NH Et OH NMe2

Me

18

Me O Et
11

NaHSO4 H 2O

Me
B
18

Et Et OH OH Me O
B
11

Me

Et

O Me OH Et

OH

Me

Et

Me O

Me

Me

18

Me O
21

Me
B

18

Et

O Me OH Et

Et Me

11

OH

Et

O Me OH Et

21

Et

OH

Me The C-11 ketone must be protected during the C-18C-19 bond construction Me O
B
11

Me
18

Et Et O OH Me O
B PPTS MeOH

Et

()

Me

C18 diastereoselection 9 : 1 (48%)


18

PPTS MeOH

The In situ protection of the C11 Carbonyl


Me O MeO
B

Me

Me O Et Et Me Et H
B

Me

18

Me Me
18

N Me

RM

11

Et

O Li MeN O R

Me Me
B
17

Me

Et

Me O

Me O

Et Et

epi-C18 6

Me N

NMe2

Et

Me N

NMe2

carbonyl-protected intermediate

H24-12-Ferensimycin construct 11/4/01 12:32 PM

Quote for the Day


Quote and limrick by J. W. Cornforth

"Nature, it seems, is an organic chemist having some predilection for the aldol and related condensations."

"That Outpost of Empire, Australia Produces some Curious Mammalia The Kangaroo Rat The Blood-sucking Bat and Aurthur J. Birch, inter alia."

25-12 Quotes-4 11/12/03 8:17 AM

D. A. Evans

The Aldol Reaction1


Suggested Reading

Chem 206

http://www.courses.fas.harvard.edu/~chem206/

Chemistry 206 Advanced Organic Chemistry


Lecture Number 25

Stereoselective Aldol Reactionsw in the Synthesis of Polyketide natural Products, I. Paterson et al. in Modern Carbonyl Chemistry, pp 249-297, J. Otera, Ed. Wiley VCH, 2000 (handout) Ager, D. J., I. Prakash, et al. (1997). Chiral oxazolidinones in asymmetric synthesis. Aldrichimica Acta 30(1): 3-12

Other Useful References


Evans, D. A., J. V. Nelson, et al. (1982). Stereoselective Aldol Condensations. Top. Stereochem. 13: 1. Heathcock, C. H. (1984). The Aldol Addition Reaction. Asymmetric Synthesis. Stereodifferentiating Reactions, Part B. J. D. Morrison. New York, AP. 3: 111. R Me Oppolzer, W. (1987). Camphor Derivatives as Chiral Auxiliaries in Asymmetric Synthesis. Tetrahedron 43: 1969. Heathcock, C. H. (1991). The Aldol Reaction: Acid and General Base Catalysis. Comprehensive Organic Synthesis. B. M. Trost and I. Fleming. Oxford, Pergamon Press. 2: 133. Heathcock, C. H. (1991). The Aldol Reaction: Group I and Group II Enolates. Comprehensive Organic Synthesis. B. M. Trost and I. Fleming. Oxford, Pergamon Press. 2: 181. Kim, B. M., S. F. Williams, et al. (1991). The Aldol Reaction: Group III Enolates. Comprehensive Organic Synthesis. B. M. Trost and I. Fleming. Oxford, Pergamon Press. 2: 239. Franklin, A. S. and I. Paterson (1994). Recent Developments in Asymmetric Aldol Methodology. Contemporary Organic Synthesis 1: 317-338. Cowden, C. J. and I. Paterson (1997). Asymmetric aldol reactions using boron enolates. Org. React. (N.Y.) 51: 1-200. Nelson, S. G. (1998). Catalyzed enantioselective aldol additions of latent enolate equivalents. Tetrahedron: Asymmetry 9(3): 357-389. Mahrwald, R. (1999). Diastereoselection in Lewis-acid-mediated aldol additions. Chem. Rev. 99(5): 1095-1120.

The Aldol Reaction1


O R M Me H O R R O M O

Polyketide Biosynthesis Historical Perspective on the Aldol Reaction Aldol Diastereoselectivity Enolate Diastereoface Selectivity Absolute Control in the Aldol Process

Reading Assignment for this Week: Carey & Sundberg: Part A; Chapter 7 Carbanions & Other Nucleophilic Carbon Species Carey & Sundberg: Part B; Chapter 2 Reactions of Carbon Nucleophiles with Carbonyl Compounds
D. A. Evans
25-00-Cover Page 11/12/03 8:56 AM

Wednesday, November 12, 2003

Aldol Reaction Variants: Each has its Merits & Liabilities


Metal Aldol Process
O X R1 M O H R2 X R1 O M O R2

Teruaki Mukaiyama
Chem Lett. 1973, 1011

BM
O X R1

BM
O X

BM(H+)
OH R2 R1 X R1 O TMS O H

Mukaiyama Aldol Process


M TMS M

Generally stoichiometric in Metal Good diastereomer control now possible

R2

X R1

R2

M
O H R2 X

M
O O TMS

R2 R1

Generally catalytic in Metal Diastereomer control not highly regulated

Clayton Heathcock
30-aldol Variants 11/12/03 9:34 AM

Satoru Masamune

D. A. Evans

The Aldol Reaction: Polypropionate Biosynthesis

Chem 206

"Nature, it seems, is an organic chemist having some predilection for the aldol and related condensations." J. W. Cornforth O
Me Me Et O Me Me MeO OH H R OH Me Me OH Me O O O Me R O H NMe2 OH Me

Polypropionate Biosynthesis: The Elementary Steps


O R O SR O SR Me OH

O Acylation CO2 R Me

O SR Reduction R

OH

O SR

J. W. Cornforth Nobel Prize, 1975

Me

Erythromycin A, R = OH Erythromycin B, R = H

HO

O SR

OH Acylation CO2 R

O Reduction SR R

OH

OH

O SR

Retro-biosynthesis: Erythromycin A
O Me HO Me Et O Me Me MeO OH OH OH O OH OH O OH Me Me Me Me Me Me Me Et O Me O H O OH Me Me OH Me O O O Me Me Me H OH Me O H NMe2 OH Me erythronolide aglycon O Me HO Me Et O Me O Me H Me Me OH OH H O O OH Me Me OH Me OH OH Me

Me OH OH O OH

Me OH

Me O OH

Me

Me

Me

Me

Me

Me

Me

Me

Erythromycin Seco Acid


OH

The 7 Propionate Subunits


OH O OH OH O OH

The Acylation Event


Decarboxylation-Acylation could either be stepwise (Option A) or concerted (Option B). The stepwise Option
Me C O O O SR Me SR O RS Me H O C O C C O SR O R Me

O SR

Erythromycin Seco Acid

Recent overview: Staunton, "Polyketide biosynthesis: a millennium review." Nat. Prod. Rep. 2001, 18, 380-416.

The overall acylation is stereospecific

HO

See Lecture 24; page 24-08 for first laboratory example

25-01-aldol intro 11/12/03 8:32 AM

D. A. Evans

Polypropionate Biosynthesis: A Laboratory Simulation Latter Stages of Lonomycin Biosynthesis


OMe O OH Me Me OH Me OH O Me Me Me OH Me Me OH O OH Me Me Me Me Me Me Me OMe Me HO2C Me O OH H OMe Me Me O Me Me HO2C O OH H OMe Me Me O O Me Me Me H O Me H O O Me Me O O Me OMe OH OMe Me

Chem 206

Polypropionate & Polyacetate Biosynthesis: Develop a Laboratory Simulation


O Me Me Et O Me H OH Me Me OH Me OR OR OH OH O HO OH OH

OMe Me

OMe Me

OHMe O Me

Erythrolide B

The 7 Propionate Subunits

Me

Polypropionate Biosynthesis: The Elementary Steps


O R O HO Me OH R

O Acylation SR O SR CO2 R Me

O SR Reduction R

OH

O SR HO2C Me

OMe OH

OH

OMe O

OMe

OMe O Me

Me

Me

Me

Me

Me

Me

Me

Me

Me

Me

Cane, Celmer, Westley JACS 1983, 105, 3594


OH Acylation O O OH Reduction R SR

O SR

OH

O SR Sn(OTf)2 EtN(iPr)2 O O N Me Me Bn TiCl4 EtN(iPr)2 O Me O OMe O RCHO O

Bn N

OH R

CO2

Me

Me

Me

Me

Me

Me O

Me

The Laboratory Mimic:


Dipropionyl Synthon O () R H Me Me Me Me Bn O O O

95:5 (85% yield)

Aldol
XC

Bn N O O

O N

O O

O Me

O Me

Me

See Lecture 23; page 23-08: with M. Ennis JACS 1984, 106, 1154.

93:7 (86% yield)

with Ratz, Huff, & Sheppard, JACS 1995, 117, 3448

25-02-aldol intro-2 11/12/03 8:34 AM

D. A. Evans
General Reviews of the Aldol Literature:

The Aldol Reaction: Early Contributions

Chem 206

DuBois 1965-67: Rough correlation between enolate stucture & product stereochemistry for alkali and alkaline earth enolates
O X Me (E) Enolate O X M Me H O R X Me O M H O R X Me O OH R X Me O OH R

Mukaiyama in Organic Reactions, 1982; Vol 28, pp 203-331 Evans in Topics in Stereochemistry, 1982; Vol 13, pp 1-115 Heathcock in Asymmetric Synthesis, 1984; Vol 3, pp 111-212 Comprehensive Organic Synthesis, 1991; Vol 2 Group I & II metal enolates: Heathcock; Chapter 1.6, pp 181 Group III metal enolates: Masamune; Chapter 1.7, pp 239 Transition metal enolates: Paterson; Chapter 1.9, pp 301

anti diastereomers OH R X O OH R

Control relative stereochemical relationships Zimmerman 1957: Proposed chair-like geometry for the Ivanov Reaction
OMgX H O H Ph
i-PrMgBr

(Z) Enolate

Me syn diastereomers

Zimmerman-Traxler Model for (Z) Enolates


X L H H M Me H Me H M = Li M = MgBr M = Li M = Li M = AlEt2 C Me R2 O O favored L R2 O C M O syn:anti X = CMe3 X = C 6H 5 O M 48 : 52 50 : 50 House 1971 > 98 : 2 > 95 : 5 80 : 20 M

L X

OMgBr OMgBr Ph

MgBr O Ph

PhCHO H3O
+

Ph

OMgX Ph O H H O

MgBr O X

OH R2 Me

R2CHO

syn diastereomer

L X

OH R2

O Ph OH Ph

OH

O OH Ph

OH

O OH

R2CHO

Ph

ratio, 75:25

Ph

Me anti diastereomer Heathcock 1977 DuBois 1972

Zimmerman recognized that diastereoselection should be a function of the relative sizes of the substituents on the carbonyl component. He also speculated on the role that the metal center might play in controlling the process. The only flaw in the study was that he failed to determine whether the aldol adducts were stable to the reaction conditions.
Zimmerman, J. Am. Chem. Soc 1956, 79, 1920 (handout)

Stereocontrol optimal for "large" X; the reaction is not general.

25-03-aldol-1 11/12/03 8:40 AM

D. A. Evans
Why Boron?
To tighten up the transition state. Design TS where control can come exclusively from metal center X
R2CHO

The Aldol Reaction: Boron Enolates


Are (E) enolates intrinsically less diastereoselective?
MO 1.9-2.2 L H H C Me R2 O L O favored B X Me syn diastereomer O O L X OH Ph R2 Me anti diastereomer Heathcock 1977 DuBois 1972 DuBois 1972 Me
9-BBN-Cl DIPEA Et2O

Chem 206

BO MC 1.4-1.5 2.0-2.2

BC 1.5-1.6

Now that there are good methods for preparing (E) enolates, it appears that both enolate geometries are nearly equivalent. JACS. 1989, 111, 3441-3442.
Chx2BCl Et3N Et2O

Dialkylboron chlorides (Brown)


JACS. 1989, 111, 3441-3442. J. Org. Chem. 1992, 57, 499-504. J. Org. Chem. 1992, 57, 2716-2721. J. Org. Chem. 1992, 57, 3767-3772. J. Org. Chem. 1993, 58, 147-153.

OH R2 OBChx2

O
PhCHO

OH Ph

O X

BL2 Me H

Ph Me ~99% (E) OB-9-BBN Ph Me ~99% (Z)

Ph Me 95% anti O
PhCHO

X R2 O C Me H L B O

R2CHO

OH Ph

Ph Me 98% syn

disfavored syn:anti X = CMe3 M = Li M = MgBr M = BBu2 X = C 6H 5 X = Et M = Li M = BBu2 M = Li M = BBu2 M = Li M = AlEt2 M = BBu2 M = BBu2 M = BCy(thex) > 98 : 2 > 95 : 5 > 97 : 3 80 : 20 > 97 : 3 80 : 20 > 97 : 3 48 : 52 50 : 50

It appears that there is not a great difference in aldol diastereoselectivity Dissection of the Aldol Problem: Select for one product diastereomer
O X M O X Me M Me O H R X Me O X Me OH R X Me O X O X H O X El Me M Me Relevant stereochemical information could be included in either X or M El Me O O OH R X Me OH R O OH R

Evans, Masamune 1979-81

syn diastereomers

House 1971 Yamamoto 1977

33 : 67 (ether) 17 : 83 (pentane) 6 : 94 (CH2Cl2) Evans, Masamune 1979-81

anti diastereomers

O (t)BuS

Control attack on the two enolate enantiofaces El(+)

M = B(Cyp)2 <5 : 95 (pentane) Me

Evans et al. JACS 1979, 101, 6120-6123; JACS 1981, 103, 3099-3111 Masamune, Tet. Lett 1979, 1665, 2225, 2229, 3937

El(+)

25-04-aldol-boron 11/12/03 8:50 AM

D. A. Evans

The Aldol Reaction: Boron Imide Enolates


R R H B O X R Me2CHCHO O

Chem 206

Imide Enolates: The problem of enolate face selectivity


O O N R O Me LDA O N R Et3N R R O O B N R R Chelate organization precluded, R therefore face selectivity uncertain N O O O B R H O Me O R R N O Me O OH R O Me R RCHO O O N O B R H O Me R O O N Me R O OH R O Li O Me O Br-CH2R O N Me R Face selectivity predicated on chelate organization Bu2B-OTf O R

The Alpha substituent, X, plays pivotal role in aldol diastereoselection


R N O X R Substituent X = Me How can we rationalize these data ? X = SMe X=H Ratio > 300 : 1 60 : 1 1:1 O OH R O O N X OH R

R N O O

+ O

Model for Asymmetric Induction (unpublished)


R N O O RCHO O BL2 Me O H R Me BL2 Me H H R R Me O H N O O R OL B L disfavored O O N Me R O O N H L R H O O L B favored L O O L B L O R N Me R R O B L O

R O O N O

The aldol reaction selects for the opposite enolate diastereoface

R R N O O O B

R H O Me O R R N O Me O N Me R O OH R O L O B L O

G (273 K) ~ 2.6 kcal mol -1

Result discovered but not predicted

diastereoselection > 98%

J. Am. Chem. Soc 1981, 103, 2127-2129 (Handout)

disfavored product diastereomer: The destabilizing interaction?

25-05-aldol-boron/imide-1 11/12/03 8:52 AM

D. A. Evans
Imide Hydrolysis

The Aldol Reaction: Imide Transformations


Trans-esterification
OMe O N3 O (1) O O R'O

Chem 206

Imides may suffer attack at either of the two C=O functions (eq 1, eq 2)
O R * El O O R * El N H Bn N O Bn R'O

OMe
Ti(OBn)4

O R * El OR' HN

Bn Bn O (2) OC(O)OR' OCMe3

N3 O

exocyclic

N O

O O

OBn 90-93% yield OCMe3

pKa 20

endocyclic

(OF-4949 Synthesis) JACS1989, 111, 1063

Product distribution a function of attacking nucleophile (Tet. Lett. 1987, 28, 6141)

Trans-thioesterification:
MeO Me OMe O N O O OTIPS Et Me O O H H O H H N O O OTBS
(Lepicidin Synthesis) J. Am. Chem. Soc 1993, 115, 4497-4513

Substrate
Me O N Me O O Bn

Exo:Endo Reagent Ratio LiOH LiOOH 0 : 100 76 : 16 Me

Substrate
Me O N Bn

Exo:Endo Reagent Ratio LiOH O LiOOH 06 : 89 96 : 04

Me Ph
Bn-SLi THF, 0 C

MeO Me

OMe O SBn

90-94%

PhH2C O OMe

Damon, Tet. Lett. 1990, 31, 2849-2852

OTIPS Et O
LiSEt THF, 25 C

OMe O O OBn NH N3 O O N BocHN Bn O O NH2 O


LiOOH

OTES Bn

Me H O H H H

OTES O H X X = SEt X=H


Et3SiH

OBn NH

N3 O OH

BocHN O NH2

O 89% yield H

97%

OTBS *5% Pd/CaCO3/PbO 96%

(OF-4949 Synthesis) JACS 1989, 111, 1063

RCOSR O MeO Me Bn
M. Bilodeau, unpublished results

RCHO Fukuyama, J. Am. Chem. Soc 1990, 112, 7050-7051

O N

O O
LiO2H HOH/THF

O MeO Me

O OH HN Bn

O O

Transamination to Weinreb Amides (see Handout 24A)


OH R Me Bn O N O
Me(OMe)NHMe

OH O
Me3Al

O N OMe O-Protect
Rmetal

OP R Me

O R

R Me

complete hydroytic selectivity possible

Me

25-06-imide transformations 11/12/03 8:53 AM

for recent examples see, J. Am. Chem. Soc 1992, 114, 9434-9453

D. A. Evans

The Aldol Reaction: Syn Aldol Rxns of Chiral Ethyl Ketones


O O M Me RM R-CHO RL RM Me O H RM RL R O C H H Me H RL RM O RL RM Me OH R TBSO R1 Me O Me L2BOTf Pr2NEt RCHO TBSO R1 Me L This system does not give a completely clean (Z) enolate Bu 9-BBN O Bu2BOTf, i Pr2NEt RCHO Me (-)-Ipc O
t

Chem 206

General Reaction for Syn Aldols: M = B, Ti


O OH R Me TBSO TBS = SiMe2tBu Me

n-Bu2BOTf R 3N RCHO TBSO R Ph

OH R Me Diastereoselection 97:3 98:2 96:4 >99:<1

M = B, Ti

RL

The Transition States:


O RL RM M Me RM RCHO disfavored L M L M Me RCHO favored L M L O O R C H H

OH R

Masamune, JACS 1981, 103, 1566.

Et BnOCH2CH2 Me2CH

RL RM Me

O RL

OH R Me Diastereoselection 63:37 - 84:16 83:17 - 85:15 72:28 91:9 - 94:6

Evans, JACS 1991, 113, 1047.

Examples:
OH R Me Me TBSO Me Me Me TBSO Me Me Me Me Me TBSO Me O Me TiCl4 EtNiPr2 Me2CHCHO Evans, JACS 1991, 113, 1047. O Me TiCl4 EtNiPr2 Me2CHCHO O Me TBSO Evans, JACS 1991, 113, 1047. TiCl4 EtNiPr2 Me2CHCHO O OH

BuMe2Si

Me

BuMe2Si

Paterson, McClure, Tet.Lett. 1987, 28, 1229. (+)-Ipc

Enders ACIEE 1988, 27, 581.

Diastereoselection = 96-98%

TBSO Me Me TBSO Me Me Me

OH Me Me Me OH Me

Diastereoselection: 99:1 (81%)

Me

Me

Me

Diastereoselection: 95:5 (80-90%)

25-07-Ketone aldols-1 11/11/03 8:27 PM

D. A. Evans
M

The Aldol Reaction: Anti Aldol Rxns of Chiral Ethyl Ketones


O

Chem 206
Diastereoselection major : others Me 94:6 (90%)

Examples:
O Me R-CHO RL RM Me O R-CHO RL RM Me OH R Me Me Me OH R Me Me TBSO Me O Me TBSO O TBSO (Chx)2BCl Me Et3N iPrCHO O OH

General Reaction for Syn Aldols:

RL RM O M

Me Me Me Me syn-anti diastereomer

General Reaction R L for Anti Aldols: RM

Me

OH TBSO O (Chx)2BCl Et3N Me Me iPrCHO Me Me Me Me Me

96:4 (75%)

The Transition States:


O RL RM Me M
RCHO

L M L

O O

R C H RM R O C H RL

H O Me H RL H O Me H RM RL RM Me OH R RL RM Me OH

However, the preceding precedent does not extend to these systems:


O R O N Me Bn Me O O O (Chx)2BCl Et3N Xq iPrCHO O OH R Xq Me Me diastereoselection 84:16 Me Me O O OH R

favored

O RL RM

M
RCHO

L M L

D. A. Evans, H. P. Ng, J. S. Clark, D. L. Rieger Tetrahedron, 1992, 48, 2127-2

Me

disfavored

An analogous case:
O BnO (Chx)2BCl Et3N BnO RCHO O OH R Me Me BnO Me Me O OH R

Evans, JACS 1991, 113, 1047.

(E) Enolate Facial Bias


RCHO

OH R

Me

Me

disfavored ?
O RL Me Me RL Me
O M H

RL Me Me

diastereoselection 95:5 I. Patterson, J. M. Goodman, M. Isaka Tetrahedron Lett. 1989, 30, 7121-71 O O N Me Bn Me O O BnO Me Me O

C H

anti-anti diastereomer RCHO O RL Me Me OH R

Me

favored ?

syn-anti diastereomer

These enolates do not comply with steric analysis: electronic effects?


Tetrahedron, 1992, 48, 2127-2142.

25-08-Ketone aldols-2 11/11/03 8:28 PM

D. A. Evans

The Aldol Reaction: Metal-Centered Chirality


H Me TfOB (95 % ee) Me O O DIPEA SCEt3 O BR*2 3 36 h SCEt3 + RCHO -78 C 0 C, 1 h SCEt3 Me HO R Me RCHO n-PrCHO i-PrCHO t-BuCHO c-C6H11CHO PhCHO Yield, % 91 85 95 82 (71) anti/syn ee % (corrected) 33:1 30:1 30:1 32:1 33:1 93 (98) 95 (99) 96 (99.9) 93 (98) 96 (99.8)
Chem 3D

Chem 206

Masamune, Sato, Kim, Wollmann J. Am. Chem. Soc. 1986, 108, 8279-8281.

Me R S O O B Me

HO

O SCEt3 Me

disfavored

HR Me H BR*2 R Me H S

O O Me R B

Me

HO

O SCEt3 Me

favored

Me

O SCEt3

Me

BR*2 3 10 h SCEt3 + RCHO -78 C

HO R

O SCEt3

RCHO n-PrCHO i-PrCHO t-BuCHO c-C6H11CHO PhCHO

Yield, % ee % (corrected) 82 81 71 95 78 87 (91) 87 (92) 94 (98) 86 (90) 88 (92) Ph ClB Ph

Analogous Carbonyl Allylation


Masamune, Sato, Kim, Wollmann J. Org. Chem. 1987, 52, 4831 HO R Me Me HO R Me syn:anti, 96:4 enantioselection: 95-97%

Me Me B
+ RCHO

favored

See analogous study by Reetz


Reetz Tetrahedron Lett. 1986, 4721

25-09-Chiral aldol-metal-1 11/11/03 8:29 PM

D. A. Evans, D. H. Ripin
Masamune-Reetz Analogy:
R H O B R SCEt3 Me RCHO Me S R H O O B

The Aldol Reaction: Metal-Centered Chirality


O R favored R Me SCEt3 HO O OtBu X (R) Phchex(X) Me Me HO R X % ee Yield 94 75 93 % 82 % O OtBu

Chem 206

Chiral Anti Aldol Reaction: JACS 1990,112, 4977; TL 1991,32, 2857.


enolization RCHO Ratio syn:anti 2 : 98 6 : 94

R R

Metal-Based Chiral Auxilliary:


CF3 Ph N Ph N CF3 (R) (X) Br Br Ratio syn:anti 2 : 98 2 : 98 % ee Yield 96 91 86 % 65 %

References:
CF3 (Corey) JACS. 1989,111, 5494 (Corey) JACS. 1990,112, 4977 (Corey) TL. 1991,32, 2857 (Corey) TL. 1993,34, 1737.

Phchex-

F3C

S O O

S O O Br

Chiral Syn Aldol Reaction JACS 1989,111, 5494.


O Me Me enolization RCHO Ratio syn:anti 94 : 6 98 : 2 >98 : 2 R Me OH O Me

1
Does this reagent perform in accord with the Masamune-Reetz analogy? Note: The sulfonamide nitrogens are pseudo-tetrahedral

Enolization:
Either enolate geometry possible with proper choice of base, solvent, and substrate. O
t

(R) PhMe2CHMeCH2-

% ee Yield 97 95 >98 86 % 91 % 68 %

1, Triethylamine Me PhCH3 / Hexane tBuO -78C

B * R2

BuO

Me B * R2 Me

Chiral Acetate Aldol Reaction JACS 1989,111, 5494.


O Me SPh (R) PhMe2CHenolization RCHO R OH O SPh

O PhS Me

1, Hunig's Base CH2Cl2 -78C PhS

A mechanistic proposal for enolization control is presented in paper (Corey) JACS. 1989,111, 5494

% ee Yield 91 83 84 % 82 %

25-10-Chiral aldol-metal-2 11/11/03 8:30 PM

D. A. Evans
M Me RO O M Me RO

The Aldol Reaction: Chelate vs Steric Control


O O R-CHO RL RO O R-CHO RL RO Me O N Me Me Bn
TiCl4 EtNiPr2 Me2CHCHO
t

Chem 206

OH R Me OH R

Complimentary aldol reactions may be obtained by changing metal as well as enolate geometry
TiCl4 i-Pr2NEt RCHO O Xq Me Me O OH R

Reference Rxn

RL

syn-syn
OH R

Chelate-Organized RL Variant

Sn(OTf)2 Et3N RCHO

O Xq Me

Me

Nonchelate Reaction
O Me
t

anti-syn
OH R

OH Me

(Chx)2BCl Et3N RCHO

O Xq Me

BuMe2SiO

BuMe2SiO

Me

Me

Me

anti-anti

Masamune, JACS 1981, 103, 1566 (boron enolate) Evans, JACS 1991, 113, 1047 (titanium enolate) O Me Me3SiO
LDA

Diastereoselection: 99:1

JACS, 1990, 112, 866; Tetrahedron, 1992, 48, 2127-2142. chiral reagent needed

LDA PhCHO

OH Ph

(+)(IPC)2-OTf i-Pr2NEt RCHO

O BnO Me O BnO Me Me

OH R

Me3SiO

Me O BnO Me Me Sn(OTf)2 Et3N RCHO

anti-syn
OH R

Diastereoselection: 90:10

L L R O RL Chelation possible for R = Bn, TMS but marginal for TBS Li O Me


PhCHO

Me O Li RL O R O H H Ph

Me O

syn-syn
OH R

Paterson & co-workers

(Chx)2BCl Et3N RCHO

BnO Me Me

Thorton, Tet. Let. 1990, 31, 6001

Tetrahedron Lett. 1988, 29, 585-588 Tetrahedron Lett. 1989, 30, 7121-7124 Tetrahedron Lett. 1992, 33, 4233-4236

anti-anti

25-11-Ketone aldols-3/chelat 11/7/01 7:38 AM

D. A. Evans

The Aldol Reaction2


Assigned Reading

Chem 206

http://www.courses.fas.harvard.edu/~chem206/

Chemistry 206 Advanced Organic Chemistry

Lithium Diisopropylamide-Mediated Lithiations of Imines: Insights into highly Structure -Dependent Rates and Selectivities. D. Colum, JACS 2003, 125, ASAP (handout) W. R. Roush, J. Org. Chem. 1991, 56, 4151-4157. (handout)

Other Useful References Lecture Number 26


Evans, D. A., J. V. Nelson, et al. (1982). Stereoselective Aldol Condensations. Top. Stereochem. 13: 1. Heathcock, C. H. (1984). The Aldol Addition Reaction. Asymmetric Synthesis. Stereodifferentiating Reactions, Part B. J. D. Morrison. New York, AP. 3: 111. O R Me Oppolzer, W. (1987). Camphor Derivatives as Chiral Auxiliaries in Asymmetric Synthesis. Tetrahedron 43: 1969. Heathcock, C. H. (1991). The Aldol Reaction: Acid and General Base Catalysis. Comprehensive Organic Synthesis. B. M. Trost and I. Fleming. Oxford, Pergamon Press. 2: 133. Heathcock, C. H. (1991). The Aldol Reaction: Group I and Group II Enolates. Comprehensive Organic Synthesis. B. M. Trost and I. Fleming. Oxford, Pergamon Press. 2: 181. Kim, B. M., S. F. Williams, et al. (1991). The Aldol Reaction: Group III Enolates. Comprehensive Organic Synthesis. B. M. Trost and I. Fleming. Oxford, Pergamon Press. 2: 239. Franklin, A. S. and I. Paterson (1994). Recent Developments in Asymmetric Aldol Methodology. Contemporary Organic Synthesis 1: 317-338. Cowden, C. J. and I. Paterson (1997). Asymmetric aldol reactions using boron enolates. Org. React. (N.Y.) 51: 1-200. Nelson, S. G. (1998). Catalyzed enantioselective aldol additions of latent enolate equivalents. Tetrahedron: Asymmetry 9(3): 357-389. Mahrwald, R. (1999). Diastereoselection in Lewis-acid-mediated aldol additions. Chem. Rev. 99(5): 1095-1120.

The Aldol Reaction2


O R M Me H O R R O M

(E) & (Z) Enolates: Felkin Selectivity Double Stereodifferentiating Aldol Reactions The Mukaiyama Aldol Reaction Variant Allylmetal Nucleophiles as Enolate Synthons

Reading Assignment for this Week: Carey & Sundberg: Part A; Chapter 7 Carbanions & Other Nucleophilic Carbon Species Carey & Sundberg: Part B; Chapter 2 Reactions of Carbon Nucleophiles with Carbonyl Compounds
D. A. Evans
26-00-Cover Page 11/17/03 8:40 AM

Monday, November 17, 2002

D. A. Evans

Carbonyl Addition Reactions: (E)-Enolate Nucleophiles

Chem 206

(E) Enolates Exhibit Felkin Aldehyde Diastereoface Selection


Me H Favored L nM O R Me MLn H Me Me Me R O RL H RL O C O
H

The Non-Reinforcing syn- RCHO is the most Interesting Dependence of the Selectivity of Felkin-controlled Reactions on Nu Size

+ + H Me R Me Me O OH RL H O OP R
i

RL O C O
H

OTMS O BF3OEt2 78 C R 20 Felkin/1,3-syn Me OH OP


i

OH

OP
i

R + + R MLn

Pr

Pr R Me

Pr

Felkin
O OH RL Me Me

Me 19 P = PMB 6 P = TBS

21 anti-Felkin/1,3-anti

P = PMB R 20 : 21 96 : 04 56 : 44 17 : 83

P = TBS 20 : 21 94 : 06 75 : 25 40 : 60

Disfavored

Anti-Felkin

-substituent dominates for Large Nu -substituent dominates for small Nu

The illustrated syn-pentane interaction disfavors the anti-Felkin pathway. Evans, Nelson, Taber, Topics in Stereochemistry 1982, 13, 1-115. W. R. Roush, J. Org. Chem. 1991, 56, 4151-4157.

t-Bu i-Pr Me

Background Information: The influence of -OR substituents on RCHO


Evans, JACS 1996, 118, 4322-4343
Nu:

OB(Chx)2 Me H Me Me

OR Me Me Me major : minors R = TBS R = PMB OR Me Me Me Me 99 : 1 93 : 7 Me

OH

OR Me

Ha H

C R
Nu:

Me OM

O R Me Nu Lewis acid Nu

OH R Me Felkin Selecton

Me

Me

Me

Me

both centers reinforcing

Felkin

(77% yield) (84% yield) O OH OR Me Me Me Me Felkin Me

Ha H PO

C C R

Hb OM H

O H OR R Nu Lewis acid Nu OH OR R 1,3-selection Me Me

OB(Chx)2 H Me

Therefore, one might conclude that: OR O H Me OR Me stereocenters Me

O H

major : minors centers non-reinforcing R = TBS (79% yield) 94 : 6 R = PMB 74 : 26 (82% yield)

Me

Me Me

stereocenters reinforcing

non-reinforcing

Achiral (E) enolates preferentially add to the Felkin diastereoface High anti:syn diastereoselectivity ( 97 : 3) is observed in all cases
Evans etal. JACS 1995, 117, 9073

26-01-E enol/RCHO 11/16/03 7:05 PM

D. A. Evans

Carbonyl Addition Reactions: (Z)-Enolate Nucleophiles


+ + O H R + + RL O C O R
H

Chem 206

(Z) Enolates Exhibit Anti-Felkin Aldehyde Diastereoface Selection


Me Me H O RL Me Me H Me OTBS Me R OM Me R Me Me Felkin OLi RL Me Me
D. W. Brooks & Co-workers Tetrahedron Lett. 1982, 23, 4991-4994.

Disfavored L nM O R MLn Me H Me O RL

RL O C O
H

OH

OH

OTBS Me R

OH

OTBS Me

Me

Me

Felkin
O R MLn OH

anti-Felkin (Cram-Chelate) Felkin : anti-Felkin Me 9BBN Me


29 : 71 27 : 73

H Me Me H

TMSO Me Me O PhS

Favored

Anti-Felkin

The illustrated syn-pentane interaction disfavors the Felkin pathway.


Evans, Nelson, Taber, Topics in Stereochemistry 1982, 13, 1-115. W. R. Roush, J. Org. Chem. 1991, 56, 4151-4157. The bulky OTBS group disfavors chelation. (see Keck, JACS 1986, 108, 3847.) The boron and lithium enolates display nearly equal levels of anti-Felkin selectivity.

An Early study rationalized results through chelated transition states:


Li Me O H Me R OCH2OBn R" Si-face Me Me O OH OCH2OBn R" R Me Me O OH OCH2OBn R"

Titanium enolates exhibit the same trend


O iPr Me Me OH OPMB iPr

O R

O iPr

TiCln Me H

OPMB iPr Me

Felkin

Anti-Felkin (Cram Chelate)

anti-Felkin : Felkin 77 : 23 (78%) O iPr Me Me OH OPMB iPr

(R) Et C6H11 Et C6H11


H H

(R")
O OCH2OBn Et Me O OCH2OBn CHMe2 Me

Felkin : Anti-Felkin 17 : 83 10 : 90 13 : 87 8 : 92 Nu:


H Me O Li O CH2OBn R" iPr

TiCln Me H

OPMB iPr Me

anti-Felkin : Felkin 56 : 44 (84%)

Evans etal. JACS 1995, 117, 9073

Masamune JACS 1982, 104, 5526

26-02-Z enol/RCHO 11/16/03 7:07 PM

D. A. Evans

Double Stereodifferentiating Bond Constructions-1

Chem 206

Double Stereodifferentiating Aldol Bond Constructions


O
()

Matched reactant pair: Stereo-induction from both partners reinforcing

O H Me Me

OH

The reference reactions:


OM O Me Me O Me H Me Me Me Me H

O Me Me Me

OH

Me

Me

Me

Me

Me

Stereochemical Control Elements Enolate geometry

Me

Me

[aldehyde prod ratio] = 10/1


OM O OH Me Me

Enolate facial bias

Product Stereochemistry

Aldehyde facial bias

Me

[enolate prod ratio] = 10/1

The Issue:

Can one reliably take the diastereoselectivites of the individual reaction partners and use this information in the illustrated extrapolation:

The double stereodifferentiating situation: Stereoselectivity?


OM Me Me H O

OH

The model reactions:


O Nu () H Me OM Me Me Me El (+) OH

Me

G (rxn)

Me

Me

Me

Nu
Me O

G (aldehyde)

The assumption: (Masamune, Heathcock)


It is presumed that useful information can be obtained from related achiral enolate & RCHO addition reactions and that the free energy contributions will be additive:

El
Me

G (enolate)

G (Rxn) ~ G (enolate) + G (RCHO) log [Product ratio] ~ log [enolate ratio] + log [aldehyde ratio] [Product ratio] ~ [enolate prod ratio] x [aldehyde prod ratio]

The extrapolation:
OM Me Me H Me O

OH

Hence, for the case at hand: [Product ratio] ~ [10] x [10] ~ 100
Mismatched reactant pair: Stereo-induction from partners nonreinforcing
OM Me O H

G (rxn) = ?

Me

Me

Me

OH

Masamune, Angew. Chem. Int. Ed. 1985, 24, 1-76

Me

Me

G (rxn)
Me

Me

Me

26-03-stereodif aldol-1 11/16/03 7:08 PM

G (Rxn) ~ G (enolate) G (RCHO)

D. A. Evans

Double Stereodifferentiating Bond Constructions-2


(Z)-Titanium Enolates: The reference reactions
TBSO Me O OTBS Me Me Me Me Me Me Me O OH OTBS Me Me H Me Me Me Me Me Me Me Me OM Me O OPMB Me Me X Me Me O OH OPMB Me Me X Me O OH Me Me Me O TiCl4, EtN-iPr2 R-CHO Me Me Me Me TBSO O OH

Chem 206

The Masamune-Heathcock generalizations hold to a point:


(E)-Boron Enolates: The reference reactions
O Me H Me Me B(c-hex)2

Me Me

diastereoselection 96 : 4
OPMB Me Me Me

diastereoselection 94 : 6
TBSO Me Me Me Me O (c-hex)2BCl, Et3N R-CHO Me TBSO O OH

diastereoselection 96 : 4 (E)-Boron Enolates: The matched cases


TBSO Me Me Me Me O BR2 H Me O OR Me Me Me Me Me Me Me TBSO O OH OR Me Me Me

M = TiCl4

M = B(9-BBN) syn: 10: 69 + syn: 21: 71 +

21% anti 8% anti

(Z)-Titanium Enolates: The matched cases


TBSO O TiCln Me Me O H Me Me OTBS Me Me TBSO R Me Me Me O OH OTBS TBSO R R Me Me Me O OH OTBS R

diastereoselection: anti : others R = TBS: >99 : 1 (85% yield) R = PMB: >99 : 1 (84% yield) (E)-Boron Enolates: The mismatched cases
TBSO Me Me O H Me Me OR Me Me Me O BR2 TBSO R Me Me Me O OH OR R TBSO R Me Me Me H O OH OR R Me

R = TBS: 87 : 13 (76%)

(Z)-Titanium Enolates: The mismatched cases


TBSO O TiCln Me Me O Me OTBS Me Me Me TBSO R Me Me Me O OH OTBS TBSO R R Me Me Me O OH OTBS R

R = TBS: 52 : 48 (83% yield) R = PMB: 81 : 19 (79% yield)

diastereoselection 62 : 38 (87%)

-center on RCHO can play a significant role in this marginal situation

"Double Stereodifferentiating Aldol Reactions. The Documentation of "Partially Matched" Aldol Bond Constructions". Evans, D. A.; Dart, M. J.; Duffy, J. L.; Rieger, D. L. JACS 1995, 117, 9073-9074.

26-04-stereodif aldol-2 11/16/03 7:09 PM

D. A. Evans

Introduction to Complex Aldol Bond Constructions


Synthesis of Polyketide chains
Evans, Ratz, Huff, Sheppard JACS 1995, 117, 3448

Chem 206

The Lonomycin Synthesis: An example of polypropionate assembage


OH OH R OMe Me O HO O OH H OMe Me Me O O Me Me Me H O Me H O O Me O HO OR ' Me ' Me O O N O
5 1

Given a polyprpionate chain of alternating Me & OH substitutents, select a disconnection point sectioning the fragments into subunits of comparable complexity by adding C=O as illustrated. OR R Me Me Me Me Me Me T1A OH O OR OR R

RO R Me

OH

OH

Me

Me

Me

Me

Me

OMe

Me OMe Me OH OH
9 11

OR R Me T1B OR R Me Me OR Me

OR

OH ' Me aldol

OR ' Me R O
3

Me

Me

Me

Me

OMe OH
5 7

OH
9

O
11

O H HO
1

O
3

OMe OH
5 7

Focusing on the =O FG, there are 2 1st-order aldol disconnections highlighted. Let's proceed forward with T1B. Carry out the dissconnection to subunits 2K and 2A.

O H Me

Me R

Me

Me

Me

Me

Me

Me

Me

Me

Me

BH3 Transform: See Lecture No. 8

2K center important center ignore

2A Both centers important

C1C11 Assemblage
O Me
7

O Sn(OTf)2 Et3N O H
9

O
7

OH
9

For substituted enolate and enolsilane-based processes, there are at least three identifiable stereochemical determinants that influence reaction diastereoselectivity (eq 1). Two of these determinants are associated with the local chirality of the individual reaction partners. For example, enolate (enolsilane) chirality influences the absolute stereochemistry of the forming methyl-bearing stereocenter, and in a similar fashion, aldehyde chirality controls the absolute stereochemical outcome of the incipient hydroxyl-bearing stereocenter. The third determinant, the pericyclic transition state, imposes a relative stereochemical relationship between the developing stereocenters. This important control element is present in the aldol reactions of metal enolates (M = BR2, TiX3, Li, etc.), but is absent in the Lewis acid catalyzed (Mukaiyama) enolsilanes aldol variants that proceed via open transition states. O R1 Me M Me H O
' R2

XP

1. NaBH(OAc)3 2. (MeO)2CMe2, H+ 93%

Me Bn A

Me

Me

Me

(85%) Diastereoselection 95 : 5 Me

Me Me O
5

Me O
9

JACS, 1990, 112, 866


H Stereochemically Matched aldol addition

Me O
9

O LiBH4, EtOH Swern XP


5

Me

Me

Me

Me

Me

Me

86% Me Me O
9

O R1 Me M = BR2

OH
' R2

(1) O

O N

O
1

O
3

Me

Me

Me

Me Sn(OTf)2 Et3N XP

O
1

OH
5

Stereochemical Determinants enolate facial bias aldehyde facial bias pericyclic transition state

M = SiR3

Me Bn A

Me

Me

Me

Me

Me

Anti-Felkin Adduct Diastereoselection >95 : 5 (86%) The Sn(OTf)2 aldol reaction of A: seethis lecture + JACS, 1990, 112, 866

26-05-PK synth-1 11/16/03 7:10 PM

D. A. Evans

Introduction to Complex Aldol Bond Constructions


Model Studies
Me Me OAc H OH Me
H

Chem 206
OB(c-Hex)2 Et Me Me -78 C Et O OH TBSO O Me Me O

The Altohyrtin Synthesis: An example of polypropionate assembage


Me O MeO O H H O H O O H AcO O H O O H H H H H OH O OH X
HO

Me H O H

14

Me TBSO O O

Diastereoselection 97:3

OH Me

Background
OB(Chx)2 iPr Me O Me O TESO O H
TESO

O iPr O OR iPr Me iPr O Me

OH

OR iPr

major : others R = TBS R = PMB 99 : 1 93 : 7

HO OH

Me OR

Evans, Trotter, Coleman, Ct, Dias, Rajapakse, Tetrahedron 1999, 55, 8671-8726. MeO O TrO O H Me H Me

BR2

OH

R = PMB iPr Me

69:31

iPr

H O M OTBS

?
O

Aldol O Reaction Me Me

The Aldol Fragment Coupling


MeO Me O H H O Me O TESO O H
TESO

H Me OTBS Me O H OH O
TESO

MeO O TrO

O TrO

H O B(cHex)2 OTBS

H OTBS

TESO O H

H Me Me Me O H OH O TESO O H
TESO

OTBS

The stereochemical determinants from each fragment were evaluated Model Studies
MeO O TrO O H Me H pentane, -78 C O TrO B(c-Hex)
2

OTBS

O MeO O

Me O Me MeO O O H Me Me Me OH TrO

H OTBS

H OTBS

H O OTBS

Diastereoselection 90:10 (70%)

OTBS

H 2:1 mixture of diastereomers

26-06-PK synth-2 11/16/03 7:11 PM

D. A. Evans

Introduction to Complex Aldol Bond Constructions


Evans, Calter, Tetrahedron Lett. 1993, 34, 6871 Me Me OMe O Me O 2 1 Me OMe Me OH Me OH O Me Me Me Me Me OR TBSO diastereoselection Me >99:1 O Me O O H Me Me O OR OR Me Me TMSO Me O Me Me Me Me OH O Me Enolization Conditions: PhBCl2, i-Pr2NEt, CH2Cl2, -78oC. tBu TBSO O H Me tBu O Si tBu O Me Me Me O O Si tBu O

Chem 206

Bafilomycin A1 Synthesis: An example of polypropionate assembage

Bafilomycin A1

HO

Me Me

Critical Aldol Disconnection


Me RO Me OMe

Critical Aldol Disconnection


Me Me OMe O O H Me
o

tBu O O Si

tBu O Me

Me

OMe Me

Required: Syn aldol addition Aldehyde Fragment: Target contains syn aldol retron wilth anti-Felkin relationship at 1 & 2 Enolate Fragment: Can the needed enolate facial bias be built into the reaction??

Me

Me

Me

OMe Me 60%

Aldol Model Studies


OH Me Me Me Me O

Enolization Conditions: PhBCl2, i-Pr2NEt, CH2Cl2, -78 C. Me Me OMe O Me O O Me2CHCHO Me Me OTBSOTES Me Me Me Me tBu O Me2CHCHO Me Me Me diastereoselection >99:1 Me O Si tBu O Me Me Me OMe Me OH Me HO Me O Me OMe O OH O Me OMe Me Me Me OH

diastereoselection >95:5

OTBSOTES Me Me

TMSO

tBu OH O O Si

tBu O Me

diastereoselection 62:38 9

Me

Me

Me

The Critical Observation


tBu OH Me Me Me Me O O Si tBu O

94% HF.pyridine, THF, 25oC.

Bafilomycin A1

26-07-PK synth-3 11/16/03 7:12 PM

D. A. Evans

The Mukayama Aldol Reaction-1


Recent Reviews

Chem 206

Type I Aldol Reaction: Metal Aldol Process


This reaction may be run with either a stoichiometric or catalytic amount of base.
O X R1 M O H R2 X R1 O M O R2

R. Mahrwald, Diatereoselection in Lewis Acid Mediated Aldol Additions, Chem. Rev. 1999, 99, 1095-1120 S. G. Nelson, Catalyzed enantioselective aldol additions of latent enolate equivalents Tetrahedron: Asymmetry 1998, 9, 357-389. Mukaiyama Aldol Reaction, E. Carreira In Comprehensive Asymmetric Catalysis, Jacobsen, E. N.; Pfaltz, A.; and Yamamoto, H. Editors; Springer Verlag: Heidelberg, 1999; Vol III, 998-1059.

BM
O X R1

HBM
O X

slow
OH R2 R1

Reaction Mechanism: "Closed" versus "Open" Transition States The Mukaiyama aldol reaction proceeds through an "open" transition state. The two illustrated competing TS orientations do not differ significantly in energy. For most reactions in this family there is not a good understanding of reactans-pair orientation. There is a prevalent view that the anti-periplanar TS is favored on the basis of electrostatic effects.
X O H X R1 TMSX R2 H C Me R2 O O L M R L H R O C C M R TMSO R2 OTMS R O M C C H R R R2

Catalytic Version: Slow step in the catalytic variant is protonation of the intermediate metal aldolate

Type II Aldol Reaction: Mukaiyama Aldol Process


This reaction may be run with either a stoichiometric or catalytic amount of Lewis acid. The minimalist mechanism: MX = Lewis acid
O X R1 TMS O H M R2 TMS X R1 O O M R2

M
O H R2 X

M
O

slow
O TMS R2 R1

Metal aldolate TS "Closed"

anti-periplanar TS "Open"

synclinal TS "Open"

Other events are also taking place: Carreira Tet. Lett 1994, 35, 4323

Silyl transfer is not necessarily intramolecular

26-08-Mukaiyama-1 11/16/03 7:12 PM

D. A. Evans

The Mukayama Aldol Reaction-2


Syn-Anti Aldol Diastereoselection
Heathcock: J. Org. Chem 1986, 51, 3027 OTMS Me Me Me BF3OEt2 PhCHO Me O O XM H H HO O OTMS Me Me Me O R Me BF3OEt2 PhCHO R Me O R Me O OTMS Ph R

Chem 206

Denmark has designed a nice substrate to distinguish between synclinal and anntiperiplanat transition states: Denmark, J. Org. Chem. 1994, 59, 707-709

OTMS Ph Me

synclinal
TMS

56:44
OTMS Ph R Me O OTMS Ph

Me CHO OTMS TMS O H

S
Me MX H O HO

56:44
OTMS Me3C O Me BF3OEt2 PhCHO OTMS Ph O H H Me3C C C M Me Ph OTMS

antiperiplanar

AP syn:anti 21:79 18:82 29:71 27:73 78:22

AP

The effectice size of the enol substituents are probably dominant.

>95:5

Lewis Acid TiCl4 SnCl4 BF3OEt2 TrClO4 SnCl2

The transition state?

These reactions "exhibit little simple diastereoselection except in special cases."....Heathcock

conclusion: there is a modestpreference for the antiperiplanar TS

26-09-Mukaiyama-2 11/16/03 7:14 PM

D. A. Evans

The Mukayama Aldol Reaction-3


Enolslane Face Selection
TBSO O R Me 95 : 5 (95%) R = iPr Felkin : anti-Felkin > 99 : 1 Me OH OTBS R R = iPr TBSO R OTBS R
BF3OEt2

Chem 206

Merged Syn-Anti & Felkin Diastereoselection


Evans: JACS 1995, 117, 9598 O H Me OTMS Me Me O H Me
Conclusions: Moderate to Good syn diastereoselectlion Excellent Felkin diastereoselectlion

OTMS Me Me iPrCHO

TBSO R Me

OH R Me

TBSO R Me

OH R Me

OTBS
BF3OEt2

O R Me

OH

OTBS R

Me

59 : 41 (82%) Enolsilane Face Selectivity 95 : 5 TBSO iPrCHO R R Me Me R R O OH TBSO O OH

OTMS

Me O R Me Me 70 : 30 (89%) Felkin : anti-Felkin 99 : 1 OH OTBS R R Me Me O OH OTBS R

Me

Me

favored
O C RL TMS C H C

Me Me 95 : 5 (80%) Enolsilane Face Selectivity 90 : 10 A(1-3) control is good for the (E) enolsilane

Me

H Me

O H Me OTMS Me Me O H Me Me

OTBS
BF3OEt2

O R Me R = iPr

OH

OTBS R R

OH

OTBS R

Double Stereodifferentiating Syn Aldol Rxns with Enolsilanes

Me 87 : 13 (68%) Felkin : anti-Felkin > 99 : 1

Me

Me H

OTBS R Me TBSO R OTBS R Me Me Me OTMS

TBSO R Me

OH

OTBS R

Me

Me

R = iPr OTBS R
BF3OEt2

98 : 2 (72%) TBSO R Me Me Me 98 : 2 (83%) O OH OTBS R

O R Me

OH

OTBS R R

OH

OTBS R O H

Me 91 : 9 (75%)

Me

Me

Conclusions: Moderate to Good syn diastereoselectlion Excellent Felkin diastereoselectlion

Felkin : anti-Felkin 87 : 13

26-10-Mukaiyama-3 11/16/03 7:15 PM

D. A. Evans, D. M. Barnes

Allyl and Crotylmetal Species1 : Boron


The Tartrate-derived Allylboronic Esters
iPrO2C iPrO2C O O B O H O O B Me Me O B Me Me OH OH

Chem 206

General Reviews of Allyl Metal Reagents: Comprehensive Organic Synthesis, 1991;Vol. 2.

Yield = 72% Chex ee = 87%

The General Reactions


R R B R R B Me R R B Me Me
RCHO RCHO RCHO

iPrO2C OH R OH R Me OH R O BnN O iPrO2C iPrO2C iPrO2C O

Chex Yield = 100% ee = 91% OH Chex Yield = 90% ee = 83%

W. Roush, J. Am. Chem. Soc. 1985, 107, 8186-8190. Tetrahedron Lett. 1988, 29, 5579-5582.

O H B OH Yield = 40% Chex ee = 97%


W. Roush, J. Am. Chem. Soc. 1988, 110, 3979-3982.

The Hoffman Chiral Allylboronic Esters


Me Me O Me O Ph B R
1) CH3CHO 2) N(CH2CH2OH)3

OH Me R R = Me: Yield = 93% ee = 60-70% R = H: Yield = 92% ee = 65%

BnN O

A Reagent for the Generation of Anti-1,2-Diols


iPrO2C iPrO2C O O O B SiMe2(OChex) (OChex)Me2Si OH
H2O2, KF, KHCO3

R. Hoffman Tetrahedron Lett. 1979, 4653-4656. ACIEE, 1978, 17, 768-769.

OH Chex

Me

Me Ms N B O OH
EtCHO

Et Yield = 92% ee = 92%


M. Reetz Chem. Ind. (London) 1988, 663-664.

Me

Chex OH

93% Yield 72% ee

W. Roush, Tetrahedron Lett. 1990, 31, 7563-7566.

26-11 - Allyl/Crotyl 1 11/16/03 7:15 PM

D. A. Evans, D. M. Barnes

Allyl and Crotylmetal Species2 : Boron


The Masamune Borolane
Me OH Yield = 56% ee = 92% Chex B Me Me B Me Me Me
70%

Chem 206

Allenylboronic Esters: Tartrate-derived Controllers and Internal Delivery


O iPrO2C iPrO2C O O B H C CH2

OH Me O H Me Me Me
76%

Me 96:4 97% ee Me

C OH Ph O Me H2C C CHB(OH)2 HO B Ph O

CH2 Me Me Ph O O B OH

OH

96:4 Me 93% ee

H 2C C

Me

S. Masamune, J. Org. Chem. 1987, 52, 4831-4832. H. Yamamoto, J. Am. Chem. Soc. 1982, 104, 7667-7669 Tetrahedron Lett. 1986, 27, 1175-1178.

HO Me OH

95% Yield >99:1

The Brown IPC Controller


CH3CHO

OH Me > 99% ee

Me B

Me

The Corey Stein Controller


Ph Ph N B O H Ph Ph N B Ts OH 98% ee R Me
E. J. Corey, J. Am. Chem. Soc. 1989, 111, 5495-5496. J. Am. Chem. Soc. 1990, 112, 878-879.

H. C. Brown, J. Am. Chem. Soc. 1983, 105, 2092-2093. J. Org. Chem. 1991, 56, 401-404. J. Org. Chem. 1992, 57, 6614.

Ts OH 98% ee R Me B R2 Me R1
CH3CHO

OH Me R1 = H, R2 = Me: ee = 90% R2 R1
R1 = Me, R2 = H: ee = 90% R1 = H, R2 = OMe: ee = 90%

Ts

H. C. Brown, J. Am. Chem. Soc. 1988, 110, 1535-1538. See also: Tetrahedron Lett. 1990, 31, 455-458.

Ts

Me B H

1) THF, RT 2) CH3CHO

H OH

OH Me ee = 94%

H N 3) 2

H. C. Brown, J. Chem. Soc., Perkin Trans. 1, 1991, 2633.

26-12 - Allyl/Crotyl 2 11/16/03 7:15 PM

D. A. Evans, D. M. Barnes

Allyl and Crotylmetal Species3


Another Enantioselective Allyltitanium Reagent
Ph Ph O Ti O O O Ph Ph OH Ph

Chem 206

The Allylboron Reagents Add to Carbonyl Compounds via a Zimmerman-Traxler Transition State
Masamune, Sato, Kim, Wollmann J. Org. Chem. 1987, 52, 4831 Me Me B + RCHO Me H H Me R H H R Me H H Me O B Me O B Me HO Me HO R Me anti:syn, 96:4 HO R Me

PhCHO

95% ee

favored

R. Duthaler, J. Am. Chem. Soc. 1992, 114, 2321-2336.

Chiral -Substituted Allyl Metal Reagents: Boron


Me Me Me Me O O B Cl
PhCHO

favored
R Me enantioselection: 95-97%

OH R Cl R Ph R = H: 92% ee R = Me: 98% ee

disfavored

HO R Me Chex O Chex

O B Me Me

OH
PhCHO

Me Me

Ph

68% Yield 99% ee

An Enantioselective Allyltitanium Reagent


RCHO

OH Me ee = 85 - 94%
R. Hoffman, Chem. Ber. 1986, 119, 2013-2024. Chem. Ber. 1988, 121, 1501-1507. ACIEE, 1986, 25, 1028-1030.

R*O Ti R*O Me Me
R*OH =

R = alkyl, aryl

O O H HO O H O O Me Me Ph R H O O

Me

Me Me Me Ph

Chex H O Me H O B Me O Chex

B H O Cl

M. Riediker, R. Duthaler, ACIEE, 1989, 28, 494-495. In Organic Synthesis via Organometallics, 1991, 285-309. J. Am. Chem. Soc. 1992, 114, 2321-2336. Duthaler Chem. Rev. 1992, 92, 807

The favored transition states

26-13- Allyl/Crotyl 3 11/16/03 7:16 PM

D. A. Evans, D. M. Barnes

Allyl and Crotylmetal Species4: Catalytic Systems

Chem 206

Three Catalytic Asymmetric Allylations of Aldehydes are Known


iPrO O COOH COOH OiPr OH OH
Ti(OiPr)4, 4 sieves

+ BH3-THF

BLn*

O O

OH

OiPr Ti OiPr

1
Me Me Me SiMe3 E/Z = 61/39
H. Yamamoto, Synlett 1991, 561-562.

OH Ph OH OH
Ti(OiPr)4, 4 sieves CF3COOH or TfOH

+ PhCHO

20 mol % BLn*

Me 63% yield 90% ee

O O

O Ti O

2
OH R
R Catalyst 1 2 1 2 Yield (%) 88 98 66 95 42 78 ee (%) 95 92 94 92 89 77

SnBu3

1 or 2 (10 mol %), RCHO

O SnBu3 O

Ti

Cl Cl 20 mol % OH n-C7H15 81% yield 97.4% ee

Ph Chex

+ n-C7H15CHO

O H

1 2

G. Keck J. Am. Chem. Soc. 1993, 115, 8467-8468.

E. Tagliavini, A. Umani-Ronchi J. Am. Chem. Soc. 1993, 115, 7001-7002.

Many Other Metals Have Been Employed in the Allylation Reaction ...
Pb: S. Torii, Chem. Lett. 1986, 1461-1462. Mo: J. Faller, Tetrahedron Lett. 1991, 32, 1271-1274. Cr: Y. Kishi, Tetrahedron Lett. 1982, 23, 2343-2346. P. Knochel, J. Org. Chem. 1992, 57, 6384-6386. Sb: Y. Butsugan, Tetrahedron Lett. 1987, 28, 3707-3708. Mn: T. Hiyama, Organometallics, 1982, 1, 1249-1251. Zn: T. Shono, Chem. Lett. 1990, 449-452. Ba: H. Yamamoto, J. Am. Chem. Soc. 1991, 113, 8955-8956.

26-14 - Allyl/Crotyl 4 11/9/00 4:33 PM

Chemistry 206 Advanced Organic Chemistry

Handout 26A

The Asymmetric Baylis-Hillman Reaction


An Evans Group Afternoon Seminar Jake Janey March 29th, 2001
XH

EWG

+
R R'

cat. base
R R'

EWG

EWG

J. Janey

Monday, November 17, 2003

26A-00-Cover page 11/17/03 8:28 AM

J. Janey

The Asymmetric Baylis-Hillman Reaction

Chem 206

An Evans Group Afternoon Seminar


Jake Janey March 29th, 2001
EWG
+

An anti propionate aldol equivalent...


OH XH * R EWG EWG Rh(I), H2 R OH EWG

X R R'

cat. base

R R'

EWG

Early Synthetic Examples


10 years after the Baylis-Hillman German patent...used in a C10 integerrinecic acid synthesis:

p. p. Leading References: Langer, P. Angew. Chem. Int. Ed. Engl. 2000, 39, 3049-3052. Ciganek, E. Org. React. 1997, 51, 201-350. Basavaiah, D.; et. al. Tetrahedron, 1996, 52, 8001-8062. Drewes, S. E.; Roos, G. H. P. Tetrahedron, 1988, 44, 4653-4670.

CO2Et +

MeCHO

5% DABCO 25 C, 7d

OH Me CO2Et

Me

Me

OH CO2H

HO2C
94% yield

Me

Drewes, S. E. J. Chem. Soc., Perkin Trans. 1 1982, 2079-2083.

Baylis-Hillman Reaction Scope


X R1 R2
+

Shortly thereafter, a more extensive, published study:

EWG

cat. base

R2 XH R1 EWG CO2Me
+

RCHO

15% DABCO 25 C, 0.5 to 7d

OH R CO2Me

X = O, NTs, NCO2R, NPPh2, NSO2Ph R1 = alkyl, aryl R2 = H, alkyl, EWG

EWG = CO2R, CN, POEt2, CHO, COR, SO2Ph, SO3Ph

R = alkyl or aromatic

94% yield

All reactions run neat in a sealed tube with 1.5-2 equivalents of acrylate. Hoffmann, H. M. R.; Rabe, J. Angew. Chem. Int. Ed. Engl. 1983, 22, 795-797.

OH
cat. bases:

N N DABCO N quinuclidine

n-Bu3P:

3-hydroxyquinuclidine (3-QDL)

26B-01 11/9/01 1:06 PM

J. Janey

The Asymmetric Baylis-Hillman Reaction


Hill, J. S.; Isaacs, N. S. J. Phys. Org. Chem. 1990, 3, 285-288. Kaye, P. T.; Bode, M. L. Tetrahedron Lett. 1991, 32, 5611-5614.

Chem 206

Evidence for an Intermediate


Drewes, S. E.; et. al. Syn. Comm. 1993, 23, 2807-2815. Coumarin Salt

EWG OH R' EWG O O R' R 3N + EWG


R 3N +

O H O
1 eq DABCO CH2Cl2, r.t., 2.5 h

+ N

Cl

O
X-ray 81% yield

EWG

O H
+

O Cl

R 3N +
Or...

OH O H OH
+

Drewes: "...the counter ion was chloride (presumably originating from the dichloromethane...)."

p. p.

O
1 eq DABCO

E2 elimination... R' O

R'CHO

:B H H H O

OMe

+ N

H H

CH2Cl2, 0 C 40% yield

Cl

EWG EWG NR3 NR3 + + initially formed eliminates E1cB is also possible

Effects of Acrylate Ester Substituent


CO2R
+

PhCHO
1.0 eq

13% DABCO neat, r.t

OH Ph

O OR

rate = Kobs[aldehyde][alkene][amine] pseudo-second order if [amine]constant addition to aldehyde is r.d.s. because the dipole is increased by further charge seperation acrylonitrile and methyl acrylate studied enolate geometry not considered ethereal solvent inhibits reaction whereas alcohols (especially diols) accelerate reaction huge volume of activation: V of -79 cm3 mol-1 (the Diels-Alder is -35 cm3 mol-1) found by plotting lnkobs vs. P. 5000 bar increases rate by 1.1 x 106 Reaction is reversible (i.e. a Grob type fragmentation), thus mechanism could be ternary, with no discrete enolate intermediate (supported by V and temperature effects).

1.3 eq R Me Et Bn n-C10H21 t-Bu 2-adamantyl CH2CH2F CH2CH2Br CH2CF3 CH2CH2OMe CH2CH2NMe2 (CH2)6Cl

26B-02 11/9/01 1:07 PM

time (days) yield (%) 6 89 7 79 For aryl substituted benzyl ethers, no clear 2 88 relation between values and reactivity was 14 75 observed. 65 65 62 40 Trends hold for furfural. 3 81 2 NR The products undergo retro Baylis-Hillman, i.e. the reaction is reversible. 15 h 58 4 89 8 82 15 NR Caubere, P.; et. al. Tetrahedron 1992, 48, 6371-6384.

J. Janey

The Asymmetric Baylis-Hillman Reaction Bases for Catalysis

Chem 206

Reaction is accelerated for a wide variety of aldehydes when conducted at 0 C Temperature effect not seen with acrylonitrile (cannot form enolate) Author concludes that one enolate must react faster than another (i.e. a kinetic versus a thermodynamic enolate).

pKa H2O (DMSO)

OH N

>

N N

>

N Quinuclidine 10.9 (9.80)

>>

+ NR3 O

R 3N +

CO2Me OMe

+ R 3N

O OMe
Z

DABCO 3-Hydroxyquinuclidine (3-QDL) 2.97, 8.82 (2.97, 8.93) 9.5 (~8.5)

NMe2 NMe2

>>

OAc N
or

N O N 3-Quinuclidone 6.9
Proton sponge 12.0 (7.50)

Which enolate is more stable and which is more reactive?

N
DBU (~12)

Leahy, J. W.; Rafel, S. J. Org. Chem. 1997, 62, 1521-1522.

3-Acetoxyquinuclidine

p. p.

Sterics also important: Me2NH > Me2NEt > MeNEt2 > NEt3 10.75 (9.00) Many, many phosphines screened...the winner: n-Bu3P ~9 n-Bu3P is only a slightly better catalyst than DABCO. E + NR3 O +

Enolate Geometry
O OMe
...or could accelerate protonation of intermediate, as any alcohol additive will accelerate reaction R 3N

OMe

O OMe
+ R3N

+ R 3N

O OMe
Z Kinetic

O H P P
unreactive

Thermodynamic less charge seperation less reactive less stable

O OR

more charge separation enolate twists out of plane by PM3

Temperature Effects
*
CO2Me
0.1 mol% DABCO

*
MeO
4 eHOMO + NR3 better conjugation into *

OH Me CO2Me

MeCHO
2M in dioxane 74% yield

+ NR3

MeO
25 C 1 week 0 C 8 hours!

26B-03 11/9/01 1:07 PM

J. Janey

The Asymmetric Baylis-Hillman Reaction Salt Additive


OH Ph CO2Me CO2t-Bu +

Chem 206
Lewis Acid Catalysis
1 eq DABCO

OH Ph CO2t-Bu

CO2Me
1.2 eq

5% DABCO

PhCHO
MeCN, r.t., 1 d 5 mol% ligand, 5 mol% metal

PhCHO
Et2O, 0 C, 20 h 1.0 eq yield (%) trace 12 40 63 72 (81)a 25 12 trace ClO4

LiClO4 (mol%) 0 5 10 50 70

Relative Reaction Rates ligand none + R3N (+)BINOL (+)diethyl tartrate (+)diisopropyl tartrate (+)TMTDA (+)hydrobenzoin (+)triphenylethanediol (+)TADDOL ethylene glycol triethanolamine salen box N-methylephedrine Sc(OTf)3 3.3 9.4 5.2 3.5 4.1 3.5 3.2 2.9 3.3 4.65 2.31 3.6 2.87 5.8 3.2 4.4 6.3 5.2 10.8 4.0 Yb(OTf)3 3.6 14.4 9.7 9.5 8.0 16.2 5.2 4.5 Eu(OTf)3 3.5 12.8 5.5 4.6 3.6 5.8 2.2 3.8 La(OTf)3 4.7 14.6 7.3 8.1 4.0 5.3 5.9 4.7

O Li OMe

p. p.

100 200 500


a

Stablize enolate?

15 mol% DABCO was used.

Ether was found to be optimal from solvent screening. General for a variety of alkenes and aldehydes. Kobayashi, S.; Kawamura, M. Tetrahedron Lett. 1999, 40, 1539-1542.

no enantioselectivity observed DABCO loading dropped to <10 mol% with (+)-BINOL rac -BINOL showed no rate acceleration Aggarwal, V. K.; et. al. Chem. Commun. 1996, 2713-2714. Aggarwal, V. K.; et. al. J. Org. Chem. 1998, 63, 7183-7189.

26B-04 11/9/01 1:07 PM

J. Janey

The Asymmetric Baylis-Hillman Reaction Possible Stereoisomers


+ H NR 3 O H H R' H

Chem 206

H R' MeO

+ H NR 3 O H H

+ R 3N

H H
O

+ R 3N

H R'
O

H R' OMe

H H OMe
Assumptions: E enolate formed

MeO

H H

+ O H NR3

H R' CO2Me

H R'

+ O H NR3

+ R 3N

H H

+ R 3N

H R'

H H CO2Me

E2 favored over E1 pathway

R' CO2Me

H CO2Me

-NR3+ is orthogonal to face (stereoelectronics)

p. p.
+ R 3N

H OH

+ R3N

OH H

HO H

H + NR3 H CO2Me

H HO

H + NR3 H CO2Me

MeO2C

H R'

MeO2C

H R'

R'

R'

120 rotation then E2 elim.

OH R' CO2Me R'

OH CO2Me

26B-05 11/9/01 1:08 PM

J. Janey

The Asymmetric Baylis-Hillman Reaction E/Z Selectivity with Crotononitrile

Chem 206

CN
+

OH PhCHO
1 eq E/Z ratio 1.2 : 1 1.4 : 1 1.5 : 1 3.1 : 1 4:1 Base DABCO 3-QDL NEt3 E E/Z ratio 1:1 2:1 4:1 r.t.

OH CN Me
+

Ph

Ph Me
Z

CN

Me
1 eq Solvent neat THF CHCl3 CH3CN MeOH

p. p.

5 mol% DABCO, 8 kbar, 17 h, solvent 50 vol%

10 mol% base, 8 kbar, 17 h, CHCl3 50 vol%

E and Z crotononitrile is easily isomerized under the reaction conditions. Products did not undergo retro-Baylis-Hillman. Rozendaal, E. L. M.; Voss, B. M. W.; Scheeren, H. W. Tetrahedron 1993, 49, 6931-6936.

5 mol% DABCO,17 h, solvent 50 vol%

26B-06 11/9/01 1:08 PM

J. Janey

The Asymmetric Baylis-Hillman Reaction

Chem 206

Possible Stereoisomers for Methylcrotonate


H R' MeO
+ H NR 3 O Me H

H R' MeO

+ Me NR 3 O H H

+ Me NR 3 O H H R' H

+ H NR 3 O H Me R' H

Assumptions: E enolate formed E2 favored over E1 pathway, only after rotation of ammonium to anti conformation -NR3+ is orthogonal to face (stereoelectronics) only one face of enolate considered, thus there are an additional 4 stereoisomers possible starting geometry of methylcrotonate and in situ isomerization not considered retro-Baylis-Hillman not considered

MeO

MeO

+ H NR O 3 H Me H R' CO2Me

+ Me NR O 3 H H H R' CO2Me + R3N

+ Me NR O 3 H H R' H CO2Me + R 3N

+ H NR O 3 H Me R' H CO2Me + R 3N

p. p.

+ R 3N

OH H

Me OH H

Me H OH

H OH

MeO2C

Me R' H

MeO2C

H R' H

MeO2C

H R' H

MeO2C

Me R' H

H MeO2C

Me OH H

Me MeO2C

OH H

Me MeO2C

H OH

H MeO2C

Me H OH

NR3 R' +

NR3 R' +

NR3 R' +

NR3 R' +

OH R' Me CO2Me R'

OH CO2Me Me R'

OH CO2Me Me R'

OH CO2Me Me

26B-07 11/9/01 1:09 PM

J. Janey

The Asymmetric Baylis-Hillman Reaction

Chem 206

Camphorsultam Acrylate Baylis-Hillman


O N S O O
R Me Et n-Pr i-Pr PhCH2CH2 AcOCH2 (CH3)2CHCH2 Ph yield (%) 85 98 70 33 68 68 67 0 ee (%) >99 >99 >99 >99 >99 >99 >99

-Branched Aldehydes: Modest Felkin-Anh Selection


CO2Me
+ (85%) r.t

O
RCHO, 10% DABCO CH2Cl2, 0 C, 12 h

R = Et MeOH, CSA

R1 R2 OH
anti R2 Me Me Me Ph Conditions DABCO, 4 d 3-QDL, 1.5 d DABCO, 6 d DABCO, 10 d 3-QDL, 60 d DABCO, 55 d DABCO, 7 d DABCO, 3.5 d DABCO, 11 d yield (%) 55 60 42 42 30 62 80 28 43

R1 CO2Me
+

O R O R

R2 OH
syn

CO2Me

R1R2CHCHO R1 MeOCH2O MeOCH2O BnOCH2O MeOCH2O Me

anti:syn 70:30 72:28 70:30 37:63 35:65 69:31 26:74 46:54 89:11

MeO2C OH

Me

Rh(I), H2 (85%)

n-Pr -OC(Me)2OCH2Me Me

p. p.

Me MeO2C OH Me

NHCO2t-Bu N-Phthalimidyl

-N(CO2t-Bu)C(Me)2OCH2-

Leahy, J. W.; et. al. J. Am. Chem Soc. 1997, 119, 4317-4318.

Varying the amount of catalyst only affects the rate, not selectivity.

Camphorsultam Acrylate Mechanism


Author's model: + NR3 Dipole minimized O + NR3 RCHO O

Anti and syn drawn incorrectly in review, should be reversed. + NR3

N S O O

H H

XcN

t-BuO2C N Me
Ciganek, E. Org. React. 1997, 51, 217-218.

N S O O O

O H syn selective H

O O R O R R O

O
+ NR3

XcN O
O

+ NR3

O XcN R3 N +

O R

R R

26B-08 11/9/01 1:09 PM

J. Janey

The Asymmetric Baylis-Hillman Reaction Chiral Aldehydes: Chromium Auxiliary


O MeO2C R OH CO2Me
cat. = 50% DABCO neat, r.t. aldehyde R OMe Cl F Me OMe Cl

Chem 206
OH CO2Me
10 mol% cat. neat, r.t. 9-94 h

CHO +

CO2Me N
18-83% yield 2-19% ee

R Cr(CO)3

H +

CO2Me
excess

OH R H Cr(CO)3
time (h) 93 6 7 58 93 8
1

h, air CH3CN yield (%) 87 89 92 90 85 97 dr >98:2 >98:2 92:8 84:16 >98:2 >98:2

R R'O P Ph R'O R

R = Me and R' = H gave highest reactivity

rac rac rac rac S-(+) S-(+)

Zhang, X.; et. al. J. Org. Chem. 2000, 65, 3489-3496.

p. p.

The High Point of Chiral Phosphine Catalysts


N CHO
+

dr determined by 200 MHz H NMR N-Tosyl arylimine chromium complex also reacts Kundig, P. E.; et. al. Tetrahedron Lett. 1993, 34, 7049-7052.

CO2R2

OH
20 mol% (S)-BINAP CHCl3, r.t. 3-14 d

N R1 N

CO2R2

R1

2.4 eq R1 H H H Me Me
a

Chiral Phosphine Catalysts


O CO2Et
18 mol% (-)-CAMP neat, r.t., 10 d

R2 i-Pr Et Me Me Me

time (d) 4 3 4 14 3

yield (%) ee (%) 8 12 24 18 26 9 25 44 37 30a

OH CO2Et
75% yield (40% isolated) 14% ee

(-)-CAMP =

P Me OMe 62% ee

Tol-BINAP was used

Frater, G.; et. al. Tetrahedron Lett. 1992, 33, 1045-1048.

other phosphines screened gave ~racemic products: DIOP, NORPHOS, BPPFOH, and MOP Soai, K.; et. al. Chem. Commun. 1998, 1271-1272.

26B-09 11/9/01 1:09 PM

J. Janey

The Asymmetric Baylis-Hillman Reaction Model For Quinidine Catalyst


Author's model:

Chem 206

Naturally Occurring Alkaloids as Chiral Catalysts


CN
+

MeCHO

? mol% cat. 9 kbar, 25-60 C

OH Me CN
0-81% yield 3-17% ee

H H

N H O

H H

H O

(-)-quinine, (1R,2S) N-methylephedrine, S-(-)-nicotine, S-(-)-N-methylprolinol screened (-)-menthyl acrylate ester gave 100% de with aromatic aldehydes and DABCO under high P Isaacs, N. S.; et. al. Tetrahedron: Asymm. 1991, 2, 969-972.

MeO

O H

N+ R O H

Me

MeO

O H

N+ H O R

Me

O Me
+ RCHO

10 mol% quinidine CH2Cl2, r.t. 20 h

OH R

O Me R

OH

O Me R

OH

O Me

p. p.
R n-Pr i-Pr c-hex

40-50% yield Pressure 3 kbar 3 kbar 3 kbar ee (%) 18 31 37 45

minor

major

n-C9H19 10 kbar

C hydrogens control face of the aldehyde bulky R should enhance selectivity, a trend that they say is "...clearly visible." H-bonding plays a "clear role" as O-acyl quinidine gives no enantioselectivity

Alternative:

N H H

3-QDL, quinine, cinchonine, cinchonidine, O-acetyl quinidine, N-methylprolinol, N-methylephedrine also screened ee is highly pressure dependent, optimized pressure is shown in table Marko, I. E.; Giles, P. R.; Hindley, N. J. Tetrahedron 1997, 53, 1015-1024.

major

H MeO H

O
O

N+ H H R H

Me

Marko, I. E.; Giles, P. R.; Hindley, N. J. Tetrahedron 1997, 53, 1015-1024.

26B-10 11/9/01 1:10 PM

J. Janey

The Asymmetric Baylis-Hillman Reaction C2 Symmetric DABCO Catalyst


CHO
+

Chem 206

Chiral Pyrrolizidine Catalyst


O OH O Me ArCHO +
10 mol% cat.

OH Ar

O Me

O
15 mol% cat.

Me

Me

O2N

1% Hydroquinone 5-10 kbar, THF, 30 C

1 eq NaBF4 CH3CN, -40 C, 0.5-3 d yield (%) 71 31 58 63 51 83 93 63 17 ee (%) 67 63 72 71 37 21 49 70 39

O2N
yield (%) 45 23 33 60 67 66 9 68 63 ee (%) 47 34 19 35 16 42 11 15 21 Author's model:

Ar 2-NO2 2-F 2-Cl 2-Br 3-NO2 2-pyridyl 3-pyridyl 4-quinolinyl 4-NO2

R Bn TBDPS TIPS cat. =

time (h) 12 12 28 16 28 16 24 17 24

O2N H N

N N H

H OR OR

cat. =

H HO

Ph Mesityl 1-naphthyl 1-anthranyl 1-napththoyl N-Cbz-Gly

p. p.

racemic alcohol product can be easily resolved by kinetic resolution with Sharpless asymmetric epoxidation other chiral DABCO's made, but not tested...

O H Ph N H H N Ph Me R

+ N

H N N H Ph Ph H N H Ph N Ph O

+ Na

O H NO2 Me

O O R

+ N

+ Na

O H NO2

favored

disfavored

Barrett, A. G. M.; et. al. Chem. Commun. 1998, 2533-2534. Hirama, M.; et. al. Tetrahedron: Asymm. 1995, 6, 1241-1244.

26B-11 11/9/01 1:10 PM

J. Janey

The Asymmetric Baylis-Hillman Reaction C2 Symmetric DABCO Catalyst


CHO
+

Chem 206

Chiral Pyrrolizidine Catalyst


O OH O Me ArCHO +
10 mol% cat.

OH Ar

O Me

O
15 mol% cat.

Me

Me

O2N

1% Hydroquinone 5-10 kbar, THF, 30 C

1 eq NaBF4 CH3CN, -40 C, 0.5-3 d yield (%) 71 31 58 63 51 83 93 63 17 ee (%) 67 63 72 71 37 21 49 70 39

O2N
yield (%) 45 23 33 60 67 66 9 68 63 ee (%) 47 34 19 35 16 42 11 15 21 Author's model:

Ar 2-NO2 2-F 2-Cl 2-Br 3-NO2 2-pyridyl 3-pyridyl 4-quinolinyl 4-NO2

R Bn TBDPS TIPS cat. =

time (h) 12 12 28 16 28 16 24 17 24

O2N H N

N N H

H OR OR

cat. =

H HO

Ph Mesityl 1-naphthyl 1-anthranyl 1-napththoyl N-Cbz-Gly

p. p.

racemic alcohol product can be easily resolved by kinetic resolution with Sharpless asymmetric epoxidation other chiral DABCO's made, but not tested...

O H Ph N H H N Ph Me R

+ N

H N N H Ph Ph H N H Ph N Ph O

+ Na

O H NO2 Me

O O R

+ N

+ Na

O H NO2

favored

disfavored

Barrett, A. G. M.; et. al. Chem. Commun. 1998, 2533-2534. Hirama, M.; et. al. Tetrahedron: Asymm. 1995, 6, 1241-1244.

26B-12 11/9/01 1:10 PM

J. Janey

The Asymmetric Baylis-Hillman Reaction Quinidine Ether Catalyst


O CF3 O CF3
10 mol% cat. DMF, -55 C, 0.5-3 d

Chem 206

OH R R

O O
ester +

CF3 CF3 R O R S O O

Proposed Mechanism: Partial Kinetic Resolution


Et H
obscures inherent facial selectivity of the catalyst RCHO

RCHO

O N+ O

OR' OH H

ee (%), R p-NO2 Ph (E)-PhCH=CH Et i-Bu i-Pr c-Hex t-Bu yield (%) 58 57 50 40 51 36 31 -(config) 91 (R) 95 (R) 92 (R) 97 (R) 99 (R) 99 (R) 99 (R) -yield (%) 11 --22 18 25 23 --

ee (%), (config) 4 (R) --27 (S) 18 (S) 25 (S) 23 (S) --

RCHO

dioxanone

Et H N O N+ H
O X=R O

H CO2R' Et

Y=R

Et H O N+ H O H
O

p. p.

O H

CO2R' H R

H O H

X
O

Y N

CO2R' H R

RCHO

RCHO

Quinidine and other acyclic derivatives showed no enantioselection and very low reactivity. Free hydroxyl on quinoline is essential for enantioselectivity. Reactions conducted at room temperature showed lower enantioselection. Racemic ester does not react to give dioxanone under the reaction conditions. Hatakeyama, S.; et. al J. Am. Chem. Soc. 1999, 121, 10219-10220.

R OH R
R

R O O R O S O O R S OH O OR'

O OR' R

O R

O
cat. =

N
prepared in 65% yield from quinidine in 85% phosphoric acid and KBr (100 C, 5 d).

OH

26B-13 11/9/01 1:10 PM

J. Janey

The Asymmetric Baylis-Hillman Reaction A Model for Facial Selectivity


O H
+ RCHO

Chem 206

BINOL as an Additive or Ligand


20 mol% n-Bu3P: 10 mol% BINOL THF, r.t. 2-24 h

OH R

O H R

R n-C7H15 Ph MEMO(CH2)3 Et PhCH2CH2

yield (%) quant. 92 98 91 quant. ee were all <10% phenol also accelerates reaction other acrylates also tolerated

p. p.
Favored PM3 minimized: C-N bond to enolate constrained to 1.6 Catalyst orthogonal to opposite face of the enolate leads to same major enantiomer after elimination. 16 mol%

O O

Ca

O R H

Ph

CHO

10 mol% n-Bu3P: THF, r.t. 7 h

OH Ph

62% yield, 56% ee Ikegami, S.; Yamada, Y. M. A. Tetrahedron Lett. 2000, 41, 2165-2169.

Disfavored

26B-14 11/9/01 1:11 PM

J. Janey

The Asymmetric Baylis-Hillman Reaction A Related Phosphine Catalyzed Reaction


CO2i-Bu
88% yield 100:0 A:B 93% ee

Chem 206

Phosphine Catalyzed Addition


RO2C
NuH + NuH 10 mol% cat. NaOAc/HOAc r.t. PhCH3 R Me yield (%) 80 76 74 ee (%) 73 74 75 product

EtO2C

:PR3

CO2i-Bu

10 mol% cat. PhCH3, 0 C

CO2i-Bu CO2Et B

Nu

CO2R

10 eq

CO2Et A Ph

cat. =

CO2R
+ PR3

i-Pr CO2i-Bu

:PR3

:PR3

O CO2Me

O CO2Me CO2R

Et t-Bu

i-Pr

CO2i-Bu R 3P +
CO2Et

p. p.

CO2i-Bu

R 3P +

CO2Et

O COMe
Et 67 56

O COMe CO2Et O
Et 83 48

CO2i-Bu
+ R 3P +

O CO2i-Bu
R 3P +

O i-Pr i-BuO2C Ph P i-Pr O O2N

COMe O

COMe CO2Et O

CO2Et

CO2Et

other catalyst and conditions give lower regio- and enantio- selection EtO2C Zhang, X.; et. al. J. Am. Chem. Soc. 1997, 119, 3836-3837. Lu, X.; et. al. J. Org. Chem. 1995, 60, 2906-2908.

Et

31

41

NO2 CO2Et

NC

CO2Et
Me 47 45

NC

CO2Et CO2Me

Zhang, X.; et. al. J. Org. Chem. 1998, 63, 5631-5635.

Ph
cat. =

Me

Me

26B-15 11/9/01 1:11 PM

J. Janey

The Asymmetric Baylis-Hillman Reaction Addition Mechanism Conclusions

Chem 206

Author's proposal:

:PR3

CO2R

cat. :PR3 HOAc/NaOAc PhCH3

Nu

CO2R

:PR3 H+ shift

The Baylis-Hillman reaction provides convenient access to valuable allylic alcohol building blocks which may serve as synthetic equivalents to anti-propionate aldol addition products. The basics of the reaction mechanism are understood, but the mechanistic details still remain elusive at best.

+ PR3

CO2R

NuH + PR3

CO2R
+ Nu

Nu

+ PR3

CO2R

Few examples of a general, diastereoselective Baylis-Hillman have been reported and the successful ones are rather limited in scope. Only one synthetically useful enantioselective, base catalyzed Baylis-Hillman reaction exists. There is no rational design, nor models for asymmetric catalysis.

p. p.

Recipe for a Good Catalyst?


O
Control enolate geometry... + NR3 E

OR
vs. + NR3 Z

OR

The asymmetric, catalytic Baylis-Hillman reaction is very promising and attractive methodology, but remains an elusive goal of chiral Lewis base catalysis.

Control aldehyde face...

H R NR2 + O OR
vs.

R H NR2 + O OR

for substituted acrylates, must control enolate facial selectivity chirality on catalyst may also gear ester substituent to influence aldehyde approach

or

O X

R H NR2 + O OR

X R2 O R1 N H O R OR

26B-16 11/9/01 1:11 PM

Chemistry 206 Advanced Organic Chemistry


Handout 26B

Asymmetric Carbonyl Ene Reactions

Evans Group Seminar by Steven Tregay, December 12, 1997

O R H R H

R R

O R R

R R

D. A. Evans

Monday, November 17, 2003

H26B-00 11/16/03 6:46 PM

Chem 206 Problems Containing the Ene Reaction


The problems provided on this and the following page deal with the ene reaction either directly or indirectly. In the latter cases, this reaction is imbedded within a multistep rearrangement sequence. Answers to these questions may be obtained by entering the descriptors "Rearrangement" and "Ene" into the problems database: http://evans.harvard.edu/problems/ Problem 210. The carbonyl ene reaction is illustrated below. Using FMO analysis, evaluate the transition state of this reaction. Your answer should include: a transition state drawing; clear orbital depictions and HOMO-LUMO assignments; an indication of the number of electrons from each segment; and indication of whether the reaction is thermally allowed. O + Ra H Rb Ra CH3 O H

Rb

Problem 19. The following transformation was recently reported by Barriault and Deon in conjunction with their synthesis of arteanniun M (Org. Lett. 2001, 3, 1925-1927). Provide a mechanism for the illustrated thermal rearrangement(s) of A to B. Where stereochemical issues are at stake, provide clear three dimensional drawings to support your answer. H Me H Single product diastereomer Me OH

DBU
OR toluene, 120 C

Me

OH

A
N N

OR

DBU is a useful amidine base; pKa ~ 12

Problem 83. Chiral methyl groups are commonly used to probe the stereochemical outcome of biological reaction mechanisms. Many interesting strategies have been developed to synthesize chiral methyl groups in high enantiomeric excess. The first approach, designed by Arigoni (Chem. Commun. 1975, 921), is illustrated below.
D O D OMe D T

HO2C
H

D T

260 C

*
H

Provide a mechanism for the following transformation that accounts for the (H,D,T) stereochemistry of the chiral methyl group. You do not need to account for the stereochemistry at the starred carbon (it was not determined by the investigators).

H26B-01 11/16/03 6:26 PM

Chem 206 Problems Containing the Ene Reaction


Problem 177. Provide a mechanism that predicts the observed stereochemistry at the starred (*) carbon atoms (Rajagopalan, Tetrahedron Lett. 1998, 39, 4133). Draw the starting material, each intermediate, and the product clearly in 3D. Me Me CO2Me Me H CO2Me OH **

OH Me Me Me Me

Problem 184. The key step in Kim's synthesis of perhydrohistrionicotoxin, 3, was the conversion of intermediate 1 to ketone 2 in a single acid-catalyzed transformation (Chem. Commun., 1997, 2263). Provide a mechanism for the conversion of 1 to 2 that accounts for the observed stereochemistry. OH C5H11 Me OMe H OH n-Bu 1 O p-TsOH/H2O O O H n-Bu 2 3 n-Bu H steps HN HO

Problem 203. Provide mechanisms that account for the stereoselective formation of the products obtained by treatment of aldehyde A to the conditions shown below. Briefly comment on the difference in reactivity under the two sets of conditions (JOC, 1998, 7586). O O OH B N H Ts A O 10 mol% SnCl4 H 0 C O

240 C

Ts N

O O C H

O N Ts

Problem 233. Snapper and co-workers have reported an approach to the [5.3.0] ring system that is commmon to a number of sesquiterpenes(JACS 2001, 123, 5152). One cited example is alismol whose structure is provided for reference. Upon thermolysis, the illustrated tetracyclic ester is transformed into the illustrated bicyclic ring system in 64% yield (eq 1). H CO2Me >130 C H TBSO OTBS Provide a plausible mechanism for this transformation. Your answer should include an explanatin of the somewhat unusual stereochemical inversion of the center carrying the flagged "red" hydrogen. CO2Me H (1) Me Me H HO

Me

Alismol

H26B-02 11/14/03 5:48 PM

Seminar Topics

H O

OH R R

Covered in this Seminar: Asymmetric Ene (Metallo-Ene not covered) Chiral Auxilaries Catalytic/Promoted

Chiral

+
H R

R = Si, Sn Reviews: Snider in Comprehensive Organic Chemistry, 1991, vol 2, 527. Mikami, Chem. Rev. 1992, 92, 1021. Mikami, Advances in Asymmetric Synthesis, 1995, 1. Bolm, ACIEE, 1995, 34, 1717. Mikami, Advances in Catalytic Processes, 1995, 1, 123. Mikami, Pure & Applied Chem., 1996, 68, 639.

History: Alder, Ber., 1943, 76, 27. Alder, Ann., 1962, 651, 141.

Early Work on Chiral Glyoxylates

Me

Me O

O O H

2 equiv. SnCl4 (1.0 equiv) 0 oC, CH3NO2

*RO OH

Me

31 % ee, 87 % yield Thermal reaction (160 oC) gave no induction

Achmatzowicz, JOC, 1972, 37, 964.

H26B-03 12/12/97 11:24 AM

Ene reactions of 8-Phenylmenthol Glyoxylate Ester


O H *RO O R O *RO OH R

2 equiv. SnCl4, 0 oC, 3 hrs

*R = 8-Phenylmenthol

>97 % de < 6 % cis olefin observed


Me Me O O Me H SnCl4 O *RO OH O *RO OH O OR *RO OH OR

Alkene
O

Product

Yield

99 %

92 %

R=H 100 % (2:1 cis : trans) R=OAc 59 % R=Bn 99 %

Whitesell, JCS CC, 1982, 989. Whitesell, Tetrahedron, 1986, 42, 2993.

R=TBS 89 %

Mechanism for 1-Substituted Olefins Olefin Geometry Absolute Configuration


O Me *RO H O Me H O *RO OH R *RO OH O R O S n L O
n

O H R *RO

SnLn O H

Me

SnLn

Competing Pathway for Free Hydroxyl


O O *RO H SnLn O H *RO OH O OH

H26B-04 12/12/97 11:24 AM

Mechanism for 1-Substituted Olefins Olefin Geometry Absolute Configuration


O Me *RO H O Me H O *RO OH R *RO OH O R O SnLn O Me O SnLn H R *RO H SnLn O H R

Competing Pathway for Free Hydroxyl


O O *RO H SnLn O H *RO OH O OH

1,1 Disubstituted Olefins


R O H *RO O R' O

2 equiv.
*RO OH R R'

SnCl4, 0 oC, 3 hrs

*R = 8-Phenylmenthol

>97 % de Alkene
O *RO OH O *RO OH

Product

Yield

94 %

3:1
H

84%

O OR *RO OH OR

R=H

"Dominant product"

Whitesell, JCS CC, 1982, 989. Whitesell, Tetrahedron, 1986, 42, 2993.

R=OAc 42 % 24 % trans olefin 20 % cis olefin

H26B-05 12/12/97 11:24 AM

1,2 Disubstituted Olefins


R O H *RO O R' *RO O
2

R
3

2 equiv.

SnCl4, -78 oC, 3 hrs

R'

*R = 8-Phenylmenthol

OH

C-2 >97 % de Alkene


O
2 3

Product

Yield C-3 92 % de "Exo TS" 90 % yield

*RO H OH O *RO
2

Me
3

C-3 15 : 1 85 % yield C-3 8 : 1

OH O *RO
2

one compound
3

86 % yield

OH

Whitesell, JCS CC, 1982, 989. Whitesell, Tetrahedron, 1986, 42, 2993.

1,2 Disubstituted Olefin Mechanism


O H *RO Me

+
R

*R = 8-Phenylmenthol

O *RO

SnLn H O R H

O H *RO

SnLn R O H H

O *RO OH

R *RO

OH

R = Me trans-butene 15 : 1 cis-butene 8:1 R = i-Pr 4-methyl-cis-2-pentene one compound

Note: cis-butene does not isomerize in presence of SnCl4 or SnCl4 / isopropyl alcohol at -78 oC Isomerization does occur in the presence of gyloxylate

H26B-06 12/12/97 11:28 AM

Asymmetric Desymmetrization using 8-Phenylmenthol Glyoxylate Ester

OH H Me Me Ph O Me H O O

OH H COOR* H COOR*

SnCl4 (1.25 equiv.) -78 oC, 3.5 h

+
H

Excess

8 : 1 (72 % yield)

H Ph

SnCl4 (1.5 equiv.)


O

OH H H COOR*

+
H Me

O H

-78 to -30 C, 3.5 h


H

one diastereomer Whitesell, JACS, 1988, 110, 3585. Whitesell, JACS, 1986, 108, 6802. Whitesell, JOC, 1985, 50, 3025. (81 % yield) Note: 1 gives opposite bridgehead selectivity

Phenylmenthol Imine-Ene Reaction

O H *RO N Bn

Me Me O *RO NBn Me

2 equiv. SnCl4, 20 oC

*R = 8-Phenylmenthol

97 % de 76 % yield

O H *RO N Ts O

2 equiv. SnCl4, 0 oC
*RO NTs

*R = 8-Phenylmenthol

> 98 % de 60 % yield Use of Ts rather than Bn was not discussed

Mikami, TL, 1993, 34, 4841.

H26B-07 12/12/97 11:28 AM

Ene reaction of (S)-2-(Ethylthio)-3-siloxy-1-butene

OMe SMe H OTBS O

OMe

Me OTBS

Me2AlCl
OTBS OTBS OH SMe

99 % ee 100 % yield

Referenced in Mikami, Chem. Rev., 1992, 92, 1021 Kuwajima, Annual Meeting of the Chemical Society of Japan, 1991.

Ene Reactions of N-Glyoxyloyl -(2R)-bornane-10,2-Sultam

O H N S O O O R O *RO OH R

2 equiv. Cat (1.1 equiv.)

R= Et
H N S O O O O

Catalyst SnCl4 ZnBr2

Temp (oC) -78 5 -78 5

% de 84 : 16 90 : 10 75 : 25 89 : 11

% Yield 78 50 93 43

n-Pr

SnCl4 ZnBr2

Most Reactive Conformation according to PM3 and Ab initio calculations Chapuis, Helv. Chim. Acta, in preparation

Jurczak, Tet.: Asymm., 1997, 8, 1741.

H26B-08 12/12/97 11:30 AM

Zn BINOL Promoted Intramolecular Ene Cyclizations


Me Me Me

Me

3 equiv. Zn BINOL CH2Cl2, -78 to 0 oC


OH Me

CHO

88 % ee 86 % yield

Note: using 1.5 equiv. reagent lowers % ee

Me

Me

O Me Zn Me Me Me O

CHO

3 equiv. Zn BINOL
Me

OH

88 % ee 89 % yield (E)-methylfarnesal afforded other regiochemistry in 20 % ee

CH2Cl2, -78 to 0 oC

(Z)- methylfarnesal Yamamoto, Tetrahedron, 1986, 42, 2203.

Zn BINOL Promoted Intramolecular Ene Cyclizations 0 % ee 31 % yield


OH Me

3 equiv. Zn BINOL
CHO

CH2Cl2, -78 to 0 oC
Me Me

Me

Me

3 equiv. Zn BINOL
CHO

one compound
OH Me

CH2Cl2, -78 to 0 oC
Me Me

(R)- citronellal
Me

(R)-isopulegol
Me

Note : ZnBr2 gives 95:5 ratio of products of idenical configuration See: Nakatani, Syn. Comm., 1978, 147.

CHO

3 equiv. Zn BINOL OH CH2Cl2, -78 to 0 C


o

one compound

Me

Me

Me

(S)- citronellal

(S)-isopulegol

Yamamoto, Tetrahedron, 1986, 42, 2203.

H26B-09 12/12/97 11:32 AM

TADDOL Promoted Intramolecular Ene

O R R N

O O

TADDOL TiCl2 (1.1 equiv) Solvent, 0 oC MS, 2-4 days

R O N O

+
O Me Me O O

Me

Me

Me

R= H

Solvent Toluene (20 days) Toluene

1 % ee (% yield) ND 82 86 97 84 (17) (39) (32) (47) (ND)

2 % ee (% yield) ND 92 >98 ND >98 (37) (36) (37) (16) (ND)


Ph Me Ph Ph Ph OH OH Ph

Me

1,3,5 Trimethylbenzene CFCl2CF2Cl/ CH2Cl2

-SCH2CH2S-

CFCl2CF2Cl/ CH2Cl2

Narasaka, Chem. Lett., 1988, 1609.

3-3'-bis(triphenylsilyl)BINOL Aluminium Catalyst

Me

+
C6F6 H SMe

Cat (20 mol %) CH2Cl2, -78 oC 4 A MS


C6F6

OH

SMe

88 % ee 88% yield

SiPh3

Note: Use of less reactive aldehydes (ie Chloral) afforded lower % ee and stoic. LA were required Use of MS is required for catalytic reaction

O Al O Me

Use of 3-3'-diphenylbinaphthol complex gave 0 % ee

SiPh3

Yamamoto, TL, 1988, 29, 3967.

H26B-10 12/12/97 11:33 AM

(i-PrO)2TiCl2 (R)-BINOL Catalyzed Ene Reaction

Preliminary Result:

OH OH

MS 4A + (i-PrO)2TiCl2 CH2Cl2 Cat.

Cat. (10 mol %)


OEt

(R)
OEt OH

H O

CH2Cl2, -30 oC

86 % ee 82 % yield Nakai, JACS, 1989, 111, 1940. Nakai; Mikami, JACS, 1990, 112, 3949.

(i-PrO)2TiBr2 (R)-BINOL Catalyzed Ene Reaction : 1,1 Disubstituted Olefin Product


O

Cat. Mol %

% yield

% ee

(R)
OEt OH

73

98

O Me Me Me OEt OH O Ph Me Ph OEt OH O OEt OH

10

87

94

1.0

98

94

92

89

Reaction conditions: Ethyl glyoxylate, -30 oC, 3 hr, MS, CH2Cl2 Nakai, JACS, 1989, 111, 1940. Nakai; Mikami, JACS, 1990, 112, 3949. Nakai, Org. Syn., 1993, 14.

H26B-11 12/12/97 11:35 AM

(i-PrO)2TiBr2 (R)-BINOL Catalyzed Ene Reaction : More 1,1 Disubstituted

Olefin
O OEt OH O OEt OH

Products
O

E 91 % yeild 98 % ee
OEt OH O

Z 9 % yeild >90 % ee

39 % yeild 91% ee
OH

OEt

E 57% yeild >98 % ee

Z 4% yeild >90 % ee

O OEt OH

83 % yeild 92% ee
OH

OEt

17% yeild >98 % ee

Reaction conditions: Ethyl glyoxylate, 5 -10 mol % cat., -30 oC, 3 hr, MS, CH2Cl2 Nakai, JACS, 1989, 111, 1940. Nakai; Mikami, JACS, 1990, 112, 3949.

Importance of Molecular Sieves

Additive BINOL-(OH)2 + (i-PrO)2TiCl2 CH2Cl2 + Additive TiCl4 CH2Cl2 Method B ref: Reetz, Chem. Ind. (London), 1986, 824. Method A

BINOL-(OLi)2

For reaction of -methyl Styrene and ethyl gloxylate (CH2Cl2, -30 oC) Additive 4 A MS (g/mmol) Method A 5 0 5 then filter Additive 4 A MS (g/mmol) 0 0 5 5

Yield 100 81 96

% ee 97 10 97 Method B

Additive i-PrOH 0 10 mol % 0 10 mol %

Yield 95 90 100 98

% ee 93 95 95 96

Note : By 13C NMR no Ti BINOL complexation occurs until MS are added. (i-PrO)2TiCl2 is a viable catalyst for the reaction.

Note : MeOH, t-BuOH give similar results

Nakai; Mikami, JACS, 1990, 112, 3949.

H26B-12 12/15/97 7:22 PM

NonLinear Effect in the(i-PrO)2TiX2 BINOL Catalyzed Ene Reaction

X X O O Ti X X Ti O O

L*Ti

RCHO (R)(R)-(BINOL Ti X2)2 Fast


H

O COOEt

RCHO Slow

(R)(S)-(BINOL Ti X2)2

For chiral poisoning of racemic BINOL Complxes see: Faller, TL, 1996, 37, 3449. Mikami, Nature, 1997, 385, 613.

Note : For X-ray crystal structure of dimeric ((PhO)2TiCl2)2 See: Watenpaugh, Inorg. Chem. 1966, 5, 1782. Mikami, Tetrahedron, 1992, 48, 5671. Mikami, JACS, 1994, 116, 2812.

(i-PrO)2TiBr2 (R)-BINOL Catalyzed Ene Reaction : Other Enophiles


O O

(i-PrO)2TiBr2 (R)-BINOL +
n

(R)
R OH

20 mol %, RT, CH2Cl2, MS

Enophile
O H COOMe O H COOEt

n= 0 1

% Yield 85 70

% ee 87 94

0 1

80 60

72 86

Mikami, TL, 1996, 47, 8515.

H26B-13 12/12/97 11:38 AM

Asymmetric Desymmetrization using (i-PrO)2TiBr2 (R)-BINOL

OR

COOMe O

(i-PrO)2TiBr2 (R)-BINOL +
H COOMe OTBDMS OR COOH

20 mol %, -30 C CH2Cl2, MS

1 (89 % ee)
OTBDMS

COOMe

2
n-C5H11 OH OH OTBDMS

isocarbacyclin

1 : 2 ratio 92 : 8 Mikami, TL, 1996, 47, 8515. Mikami, Synlett, 1995, 29.

Asymmetric Desymmetrization / Resolution using (i-PrO)2TiCl2 (R)-BINOL


OR O

(i-PrO)2TiCl2 (R)-BINOL
OEt O

OR Me

O OEt OH

+ H
Me Me

10 mol %, RT, CH2Cl2, MS

R = Dimethylthexylsilyl
OR Me Me O

> 99 % syn (97 % ee) (i-PrO)2TiCl2 (R)-BINOL


OEt H O OR
2

O OEt OH

10 mol %, RT, CH2Cl2, MS

Me

1 > 97% syn (>95 % ee)

OR Me Me

Catalyst (i-PrO)2TiCl2 (S)-BINOL (i-PrO)2TiCl2 (R)-BINOL

Product ent-1 (>99 % syn, 71 % yield) ent-1 (50 : 50 at C-2, 31 % yield)

Mikami, Annual Meeting of the Chemical Scoiety of Japan, 1990 and 1991. See : Mikami, Synlett, 1992, 255.

H26B-14 12/15/97 7:18 PM

Synthesis of C10 - C15 and C30- C35 fragments of Rapamycin

RO

(i-PrO)2TiCl2 BINOL ethyl glyoxylate


Me

Me

O OEt

Me

O OEt

10 mol %, -30 oC CH2Cl2, MS

OR

OH

OR

OH

R= Dimethylthexylsilyl (R)-BINOL (83 % yield) (S)-BINOL (61 % yield) >99 3 <1 97

Mikami, TL, 1994, 35, 7793.


Me O OR OR OH OR

Me

O OEt O OH

C10 - C15 fragment

C30- C35 fragment

(i-PrO)2Ti(ClO4)2 (R)-BINOL Catalyzed {3, 4} exo, exo Intramolecular Ene Reaction

O X H

(i-PrO)2TiCl2 (R)-BINOL Additive 20 mol %, 0 oC CH2Cl2, MS,


X

OH Me X

OH Me

trans

cis

Me

Me

X=
H

Additive none AgClO4 AgClO4

time 24 h 24 h 48 h

% Yield 73 50 66

Ratio trans : cis 47 : 53 80 : 20 69 : 31

trans % ee 70 84 55

3 2 1

O O -CH2-

Type I

Mikami, TL, 1991, 32, 6571. Mikami, Tet. :Asymm., 1991, 2, 1403.

H26B-15 12/12/97 11:41 AM

(i-PrO)2Ti(ClO4)2 (R)-BINOL Catalyzed {2, 4} exo, exo Intramolecular Ene Reaction

O R R
n

O H Me

(i-PrO)2TiCl2 (R)-BINOL AgClO4 20 mol %, 0 oC CH2Cl2, MS,


O

OH

n=
4 3 2 1 5

R= H or Me H Me

% yield NR 43 40

% ee -91 82

0 1
H

Type II

Mikami, TL, 1991, 32, 6571. Mikami, Tet. :Asymm., 1991, 2, 1403.

(i-PrO)2Ti(Cl)2 (S)-BINOL Catalyzed {2, 4} exo, exo Intramolecular Ene Reaction


Me OHC CHO

1) (i-PrO)2Ti(Cl)2 (S)-BINOL (25 mol %), CH2Cl2, 12h 2) Acylation


CHO

OAc Me

OAc

+
CHO

Me

Me

4.5 (38 % ee)

H Me

H O

OH H O Me OAc CHO M

H O

CHO Me

AcO

Me

Trichothecene Anguidine

Ziegler, JACS, 1990, 112, 2749.

H26B-16 12/12/97 12:10 PM

Approaches toward Ipsdienol


O

+
XPh H O

(i-PrO)2TiCl2 (R)-BINOL
OEt PhX

O OEt OH

0.5 mol %, -30 oC CH2Cl2, MS X= S 94 % yield >99 % ee

X= Se 95 % yield >99 % ee

+
H O

OEt

(i-PrO)2TiBr2 Ligand 2 mol %, -30 oC CH2Cl2, MS Ene

O OEt OH

O COOEt

Hetero D.A. Product Ratio Hetero D. A. Ene 79 (97 % ee) 21

OH

Ligand BINOL
Br

Yield 94

(R)-Ipsdienol
OH OH

84

92 (>99 % ee)

Mikami, JCS CC, 1995, 2391. Mikami, JCS CC, 1993, 327.

Br

Corey's Model for Ti BINOL Ene Reactions


H X X O O O H R H H H

BINOL Ti X2 Aldehyde Complex

(R)
OEt OH X X O O O H R OR Me OEt OH O X X HH H O O O H R RO H CH3

OR

Corey, TL, 1997, 38, 6513.


Me Me

R= Dimethylthexylsilyl

H26B-17 12/12/97 11:45 AM

BF3 Menthylethyl Etherate catalyzed Ene reaction

Me

+
Cl3C H Me

Cat (20 mol %) -70 oC, CH2Cl2


Cl3C

OH

Me

2 equiv.

22 % ee 55 % yield

Me

BF3 Complex
OEt

Demir, Syn. Comm. 1994, 24, 137.

Carreira's Catalytic Ene


O OMe

+
R H Me

Cat (2-10 mol %) 0- 23 oC


R

OH

OMe

2 N HCl Et2O
R

OH

O Me

used as solvent ~ $50 per L b.p. 34- 36 oC Aldehyde


Ph(CH2)3 CHO

O3, Ph3P % eea 98 93 91 90 66 % yield 99 OsO4, NMO 85 99 98 83 Acetone/H2O


R

OH R

O OMe

CH2Cl2

OH

O OH

TBSOCH2 Ph

CHO CHO CHO

t-Bu

Ph

PhCHO
Only branched aldehyde which reacts

N Ti O O i-Pr-O O-i-Pr

Br

75
CHO

79 Carreira, JACS, 1995, 117, 3649.

a % ee detrmind by NMR analysis of (S)-MPTA ester of methyl ketone

H26B-18 12/12/97 11:46 AM

Yb(OTf)3 BINOL Catalyzed Ene Reaction

O MeO H O

Ph

+
Me

Cat (20 mol %) CH2Cl2, 0 oC 24 h


Ph

OH COOMe

yields 78 - 82 %

Ligand R=
R

% ee 12 38 25

H Br
OH OH

Ph + Yb(OTf)3
TMS

29

Qain, TL, 1997, 38, 6721.

Jorgensen's Ene byproducts

O H O OET +

10 mol % cat. RT, 12 hr

OH

+
COOET

COOEt

Me O N Cu Ph

Me O N Ph

Solvent CH2Cl2 CH3NO2

Diels-Alder % ee 85 90

Ene Product % ee 83 78

DA :Ene Ratio 1 : 1.8 1 : 0.8

2 TfO

Jorgensen, Tetrahedron, 1996, 52, 7321. For optimization of hetero Diels-Alder reaction products See: Jorgensen, JCS PT 2, 1997, 1183.

H26B-19 12/12/97 11:48 AM

Work from the Evans Groups "C2-Symmetric Copper(II) Complexes as Chiral Lewis Acids. Catalytic Enantioselective Carbonyl-Ene Reactions with Glyoxylate and Pyruvate Esters". Evans, D. A.; Tregay, S. W.; Burgey, C. S.; Paras, N. A.; Vojkovsky, T. J. Am. Chem. Soc. 2000, 122, 7936-7943.
Me O N Cu Me3C CMe3 N Me O

2+
O

Me

Me O

2+
O

Me Me O N Cu Ph Ph N

2+

N Cu Me3C H O 2

N OH2 CMe3

2 SbF6

2 SbF6

2 TfO

olefin

producta
O
(S)

catalyst mol% % yield % ee


2 3 1 10 90 99 97 (S) 87 (R)

OEt OH O Me Me Me OH O Ph Me Ph OH O OEt OH C4H9 C4H9 O OEt Me OTBDPS OTBDPS OH O OEt Me OBn OBn OH O OEt Me OH O OEt OH C3H7 H 7 C3 OH O OEt OEt OEt

2 3

1 10

83 92

96 (S) 92 (R)

2 3

1 10

97 99

93 (S) 89 (R)

2 3

1 10

95 97

96 (S) 76 (R)

2 3

1 10

89 81

96 (S) 92 (R)

Regiochemistry 75 : 25 90 : 10

2 3

1 10

72 85

96 (S) 91 (R)

one regioisomer

2 3

10 2

62 88

98 (S) 92 (R)

one regioisomer

1 3

10 10

95 70

98 (S) 94 (R)

exo:endo 86:14 95:5

10

96

98 (S)

E: Z 96 : 4

H26B-20 11/16/03 6:30 PM

Origins of Enantioselectivity in Cu Box Ene Reactions

(S,S )-t-Bu Box Cu Glyoxylate (PM3tm)

However, (S,S)-Ph-Box Cu (OTf)2 gives (R) configured alcohols: Tetrahedral Cu center?? Jorgensen, JOC, 1995, 60, 5757.

000 000 000 000 000 000 000 000 000 000 000 000 0000 000 0000 00 000 0000 00 000 0000 00

(S)
OEt OH

(S,S)-Ph Box Cu (OTf)2(H2O)2 X-ray


H26B-21 12/12/97 11:51 AM

Quotes for the Day


"In the last third of his life, there came over Laslo Jamf-so it seemed to those who from out of the wood lecture halls watched his eyelids slowly granulate, spots and wrinkles grow across his image, disintegrating it towards old-age hostility, a strangely personal hatred, for the covalent bond." Thomas Pynchon, "Gravity's Rainbow"

"Faced with the choice between changing one's mind and proving that there is no need to do so, almost everyone gets busy with the proof. "

John Kenneth Galbraith

Quotes-5 11/19/03 9:36 AM

Chemistry 206 Advanced Organic Chemistry


Handout27A

An Organizational Format for the Classification of Functional Groups. Applications to the Construction of Difunctional Relationships

D. A. Evans
27A-00-Cover page 11/17/03 1:11 PM

Wednesday , November 19, 2003

D. A. Evans

Functional Group Classification Scheme for Polar Bond Constructions


Papers of Historical Interest:
"Arthur Lapworth: The Genesis of Reaction Mechanism." M. Saltzman J. Chem. Ed. 1972, 49, 750. (Handout)

Chem 206

http://www.courses.fas.harvard.edu/~chem206/

Chemistry 206 Advanced Organic Chemistry


Lecture Number 27

"A Theoretical Derivation of the Principle of Induced Alternate Polarities." A. Lapworth J. Chem. Soc. 1922, 121, 416. "The Electron Theory of Valence as Applied to Organic Compounds." J. Steiglitz J. Am. Chem. Soc. 1922, 44, 1293.

Monographs:
Hase, T. A. "Umpoled Synthons. A Survey of Sources and Uses in Synthesis".; John Wiley & Sons, Inc.: New York, 1987. Ho, T.-L. "Polarity Control for Synthesis"; John Wiley & Sons, Inc.: NY, 1991. Ono, N., "The Nitro Group in Organic Synthesis", Wiley-VCH, 2001

Functional Group Classification Scheme for Polar Bond Constructions


Historical Perspective Charge Affinity Patterns Functional Group Classification Scheme The Chemistry of the NO2 Group The Chemistry of the N2 Group

Several Interesting Problems


Provide a mechanism for the Nef reaction

O N O

R H R

1) HO 2) H3O +

R O R

Reading Assignment for this Week:


"An Organizational Scheme for the Classification of Functional Groups. Applications to the Construction of Difunctional Relationships." D. A. Evans Unpublished manuscript. (Handout) "Methods of Reactivity Umpolung." D. Seebach Angew. Chem. Int. Ed. Engl. 1979, 18, 239. (Handout) "Nitroaliphatic CompoundsIdeal Intermediates in Organic Synthesis"' Seebach, D. et. al, Chimia, 1979, 33, 1-18. (Handout)

The von Richter reaction is illustrated in the accompanying equation. Please provide a plausible mechanism for this transformation taking into account the following observations. (a) If 15N-labeled KCN is used, the N2 formed is half labeled; (b) 3-bromo-benzonitrile does not form 3-bromo-benzoic acid under the reaction conditions.

Br
KCN

Br
+ heat, aqueous EtOH

N2

CO2H

NO2
Stoltz and co-workers recently reported the interesting rearrangement illustrated below (JACS 2003, 125, 13624). Please provide a mechanism for the illustrated transformation. Your answer should include clear 3-D drawings where relevant. the answer may be found in the database.

D. A. Evans
27-00-Cover Page 11/18/03 4:53 PM

Wednesday, November 19, 2003

MeO

O N2 H Me
AgOBz, Et3N THF, 45

MeO

O Me
95% yield

D. A. Evans
Required Reading:

Ambiphilic Functional Groups


Arthur Lapworth (18721941)

Chem 206

"An Organizational Scheme for the Classification of Functional Groups. Applications to the Construction of Difunctional Relationships." D. A. Evans Unpublished manuscript. "Methods of Reactivity Umpolung." D. Seebach Angew. Chem. Int. Ed. Engl. 1979, 18, 239. "Nitroaliphatic CompoundsIdeal Intermediates in Organic Synthesis"' Seebach, D. et. al, Chimia, 1979, 33, 1-18. Papers of Historical Interest: "Arthur Lapworth: The Genesis of Reaction Mechanism." M. Saltzman J. Chem. Ed. 1972, 49, 750. "A Theoretical Derivation of the Principle of Induced Alternate Polarities." A. Lapworth J. Chem. Soc. 1922, 121, 416.

Lapworth was among the first to understand and conceptualize the effect of heteroatomic substituents on the reactivity of individual carbon centers, and how this effect is propagated through the carbon framework of organic molecules. Lapworth's Theory of Alternating Polarities: "Latent Polarities of Atoms and Mechanism of Reaction, with Special Reference to Carbonyl Compounds." A. Lapworth Mem. Manchester. Lit. Phil. Soc. 1920, 64 (3), 1. "The addition of electrolytes to the carbonyl compound invariably proceeded as if the carbon were more positive than the oxygen atom, and invariably selected the negative ion; for example:"
C O C NC H NC O H

"The Electron Theory of Valence as Applied to Organic Compounds." J. Steiglitz J. Am. Chem. Soc. 1922, 44, 1293. "Displacement of Aliphatic Nitro Groups by Carbon & Heteroatom Nucleophiles." R. Tamura, A. Kamimura, N. Ono Synthesis 1991, 423. "Functionalized Nitroalkanes as Useful Reagents for Alkyl Anion Synthons." G. Rosini, R. Ballini Synthesis 1988, 833. "Conjugated Nitroalkenes: Versatile Intermediates in Organic Synthesis." A. G. M. Barrett, G. G. Graboski Chem. Rev. 1986, 86, 751. Monographs: Hase, T. A. "Umpoled Synthons. A Survey of Sources and Uses in Synthesis".; John Wiley & Sons, Inc.: New York, 1987. Ho, T.-L. "Polarity Control for Synthesis"; John Wiley & Sons, Inc.: New York, 1991.
27-01-Historical 11/17/03 12:57 PM

"The extension of the influence of the directing, or "key atom," over a long range seems to require for its fullest display the presence of double bonds, and usually in conjugated positions...."
C C C O NC H C C C O NC H

The "key atom" is the one with the most electronegative character, in this case the carbonyl oxygen. anionoid/cationoid nucleophilic/electrophilic

The Lapworth polarity designations can be used to form the basis of a functional group classification scheme.

D. A. Evans

Reactivity Patterns of Functional Groups: Charge Affinity Patterns

Chem 206

Polar rxns form the basis set of bond constructions in synthesis Generalizations on conferred site reactivity will therefore be important
O

The actual reaction associated with this transform is the addition of organometals to carbonyl substrates.
O M CH3 CH3 OM CH CH3

Given this target

and the desire to form this bond

CH3

CH

The functional group =O "dictates" the following bond construction


O O R M R R O M R O R CH3 O R CH +

+M CH3

When one considers the polar resonance structure for the C=O group it is clear that an O atom is very good at stabilizing an adjacent (+) charge through resonance.

Consider polar disconnections of the illustrated -hydroxy ketone 1: ()

Conferred site reactivity of =O

R (+)

(+) R
R

TA
O () C CH2 (+) () CH2 OH (+) ()

O () R C (+) CH3 () CH2 (+) O ()

Charge Affinity Patterns


Use the descriptors (+) and () to denote the polar disconnections shown. () (+)

TB
R

O () C CH (+) () CH2 (+) OH2 ()

A B
() (+)

A: A: +

B+ B:

It is evident that the heteroatom functional groups, =O and -OH, strongly bias the indicated polar disconnections.

A B

In the transforms illustrated above, symbols (+) & () are used to denote the particular polar transform illustrated. In the present case there is NO INTRINSIC BIAS in favoring one transform over the other. Let's now add an OH functional group (FG) to propane at C-2 and see whether one creates a bias in the favoring of one or the other transforms:
OH CH3 CH () OH CH3 CH (+) CH3 () CH3 (+)

Charge Affinity Patterns of Common Functional Groups


Me CH2 CH2 Br H O C C (+) C (+) C C () C C () C (+) C (+) C (+) C (+) C E1 E2 E3 E4

Me CH2 C H 2C

TA

OH CH3 CH: OH CH3 CH + : CH3 + CH3

CH CH2 OH OR CH C O

disfavored
H 2C

TB

favored

27-02-Chg Affinity 11/18/03 9:21 PM

D. A. Evans

Classification of Functional Groups


A-Functions:

Chem 206

Functional groups activate the carbon skeleton at the point of attachment by either induction & resonance. Induction (+) (+) () ()

C
Resonance (+) (+) Symbol

F1

C
()

F2

C
(+)

F3

C
() ()

F4

A 3rd hypothetical FG, designated as A, may be defined that has an unbiased charge affinity pattern as in 1. Such an idealized FG's activates all sites to both nucleophilic and electrophilic reactions, and as such include those functions classifies as either E or G. The importance of introducing this third class designation is that it includes those functional groups having non-alternate charge affinity patterns such as 24.
(+) C () C (+) C () C (+) C A A A 2 3 4

()

G
Hypothetical A-function
(+) (+) (+) C C C 1 A

E = electrophilic at the point of attachment A = ambiphilic at the pont of attachment G = nucleophilic at the point of attachment For simplicity, we will designate three FG classes according to the designations provided above. E & G-Functions: To organize activating functions into common categories it is worthwhile to define "hypothetical" functional groups E, and G, having the charge affinity patterns denoted below. Hypothetical E-function
(+) () (+)

FG-Classification Rules In the proposed classification scheme the following rules followed in the assignment of class designation of a given FG. Activating functions are to be considered as heteroatoms appended to or included within the carbon skeleton. Activating functions are inspected and classified according to their observed polar site reactivities. Since proton removal and addition processes are frequently an integral aspect of FG activation, the FG, its conjugate acid or base, and its proton tautomers are considered together in determining its class designation. The oxidation state of the FG is deemphasized since this is a subordinate strategic consideration.

Hypothetical G-function
() (+) ()

Given the appropriate oxidation state of the carbon skeleton, such functional groups confer the indicated polar site reactivity patterns toward both electrophiles and nucleophiles. Any FG that conforms either to the ideal charge affinity parrern or a sub-pattern thereof will thus be classified as either an E- or G-function. Representative E-functions:
Me CH2 CH2 Br H O C C (+) C (+) C C () C C () C (+) C (+) C (+) C (+) C E1 E2 E3 E4 (+) () (+) C C C

Common E-Functions: Symbol: (+) C


OR NR2 O NR

E
exception: exception:

Me CH2 C H 2C H 2C

O N

CH CH2 OH OR CH C O

X, X = halogen
Also consider all combinations of of above FGs; e.g =O + OR

27-03-FG Classification-1 11/18/03 9:23 PM

D. A. Evans

Classification of Functional Groups


G
O N CH2R O () C (+) C () C G O H-tautomer HO N CHR O conjugate base O N CHR HO

Chem 206
conjugate acid HO N CHR

Common G-Functions: Symbol: () C

Typical G-class functions are the Group I-IV metals whose reactivity patterns, falls into a subset of the idealized G-FG 5.
() H 2C CH () CH2 Li

() CH3 CH2 MgBr

FG

C ()

FG

C (+)

The Reaction:

Common A-Functions: Symbol: () C

A-functions are usually more structurally complex FGs composed of polyatomic assemblages of nitrogen, oxygen and their heavier Group V and VI relatives (P, As, S, Se). Typical A-functions, classified by inspection, are provided below

O + N CH2R O

base

O + N CHR O

El(+)

O +N O

R El

pKa ~ 10

O + N CHR O The charge affinity pattern:

NO2 SR PR2

NOR S(O)R P(O)R2

NNR2 SO2R PR3


+

N(O)R
+

N2

O () + N CH2R O

SR2

This reactivity pattern may be extended via conjugation: The Reaction:


O + N CH CH R O O + + N CH CH R O

These FG's are capable of conferring both (+) and () at point of attachment.
X: N R H X: N (+) X:

Nu()

() N

O () (+) + N CH2 CH O R

Nu

R (+) H

R () H

X = OR, NR2

Remarkably, the dual electronic properties of oximes were first discussed by Lapworth in 1924 before the modern concepts of valence bond resonance were developed.

Charge affinity pattern:

Lapworth, A. Chemistry and Industry 1924, 43, 1294-1295.

O () (+) + N CH CH R O

The Nitro Functional Group


As an example, the class designation of the nitro function is determined by an evaluation of the parent function, its nitronic acid tautomer, as well as conjugate acid and base.

The resonance feature which has been exploited:


X: R X: H

()N

(+) X:

X = OR, NR2
R () H

R (+) H

27-04-FG Classification-2 11/18/03 9:24 PM

D. A. Evans

Charge Affinity Patterns & the Nitro Functional Group


Some Reactions of the Nitro Functional Group
O

Chem 206

The Nef Reaction


OH

O () (+) + N CH CH R O

R ()

NO2 H (+) R

R 3N EtOH

R NO2

Overall Transformation:

O +N O

R H R

1) HO 2) H3O +

R O R

O R () NO2 (+) R 3N EtOH

Mechanism
R NO2

O +N O

R H R

HO

O +N O

R R

H+

HO +N O

R R

nitronate anion

nitronic acid
H+

O MeO

O Me

O2N (+)

R 3N EtOH Me

O2N O HO N H HO
+

CO2Me O2N (+) EtNH2 EtOH Me O2N N H

R R

HO N HO

R R OH

H 2O -H
+

HO +N HO

R R

The charge affinity patterns represented


O +N O R H R () HO O +N O R () R H+ HO +N O R () R

Important Transformations of the NO2 Functional Group


O O H2 R NO2 R3SnH O R O HO R NO2

nitronate anion

nitronic acid
H+ H 2O HO +N HO R (+) R

Reduction:

Pd, Ni, Pt etc R NH2 O

rxn is quite facile


R (+) R

O2N
HO N HO

C()
R R OH

Ono, N.; Kaji, A. Synthesis 1986, 693.

+ O H

-H

O2N
The resonance features which have been exploited:
() N O Pinnick, Org. Reactions 1990, 38, 655 X:

C(+)

Nef Reaction:

H 3O +

X: H

(+) X:

X = OR, NR2
R () H

R (+) H

27-05 NO2 Chem-1 11/18/03 9:25 PM

O2N

C(+)

O2N

C()

D. A. Evans

Charge Affinity Patterns & the Nitro Functional Group Other Nonalternate Behavior of NO2 FG Representative examples: O2N
O +N O Ph 2 LDA O +N O

Chem 206
() ()

O +N O

CH2R

base

O +N O

H CHR H

C
Ph

C
Bn PhCH2Br O +N O Ph

base

O +N O

CH2 H

base O +N O CH2R

40% yield O N O H CH2 Bn CO2Et O CO2Et 2 LDA +N O 80% yield Seebach et. al. Tetrahedron Lett. 1977, 1161-1164 O +N O CO2Et PhCH2Br

nitronate dianion

nitronate dianion

O +N O

2()

() ()

O2N

O2N

Reactivity Patterns

NO2 As a Leaving Group


Seebach et. al. Tetrahedron Lett. 1977, 1161-1164 Review: Tamura et. al. Synthesis 1991, 423-434. "Nitroaliphatic CompoundsIdeal Intermediates in Organic Synthesis"' Seebach, D. etal, Chimia, 1979, 33, 1-18

Representative examples: O2N


O +N O CH2Me 2 BuLi O +N O Et

2()

Representative examples: O2N


O +N O CH2Me Bn O +N O R CH R Nu() R Nu CH R SnCl4 74%

C(+)

PhCH2Br

+ NO2

51% yield Ph O +N O Ph 2 BuLi O +N O Ph

EtI

O +N O

Ph Et NO2 NO2 Me SPh Me TiCl4 65% SPh SiMe3 Me Me Ph

80% yield

27-06 NO2 Chem-2 11/18/03 9:26 PM

D. A. Evans

The Nitro Function as a Leaving Group NO2 As a Leaving Group


O N CO2Et O N CO2Et

Chem 206

Representative examples: O2N


O +N O R CH R R Nu CH R

C (+)

(+) NO2

(+)

O2N
NO2 H3O+

Nu()

+ NO2 O CO2Et R2NH NO2

(+)
Ph SnCl4 74% Ph SiMe3 Me TiCl4 65% SPh (+) () NO2 Pd(PPh3)3 N H CH(CO2Me)2 NaCH(CO2Me)2 Pd(PPh3)3 MeO SO2Ph NaO2SPh Pd(PPh3)3 CO2Et N(CH2)5 NO2 Me O R 3N CO2Et O

CO2Et (+) NO2

O2N

NO2 NO2 Me SPh Me

(+) (+) McMurry etal. Chem Comm. 1971 488-489.

O2N

O R2NH CO2Et NO2 CO2Et

() Bakuzis etal. Tetrahedron Lett. 1978 2371.

O2N

(+)

(+) NO 2 MeO DBU CO2Et NO2

(+) NO2

(+) (+)

O2N

C
MeO CO2Et

Danishefsky etal. JACS 1978, 100, 2918.

27-07 NO2 Chem-3 11/18/03 9:26 PM

D. A. Evans

Nonalternate Reactivity Patterns of Diazo Functional Groups The Diazo Functional Group

Chem 206

Acid Catalyzed Reactions of Diazo Compounds


Review: Smith, Tet. 1981 2407 O H 3C N CH3 O H3 C N N Diazonium CH3

H +C N N R

H + C N N R

H + C N N R

Both (+) and () reactivity patterns suggested by resonance structures Rxns with acids:
H X H + C N N R + H C N N R H X

H+

N2

()

H H C X R N N

N Diazocarbonyl

Common acids include BF3OEt2, HBF4, TFA, etc.

N2

()

N2

(+)

Mechanism of activation is unclear for both Lewis and protic acids; activation may occur by protonation on C or O

Initiating reactivity is (); subsequent reactivity is (+) Ring expansion reactions:


O CH2N2 EtOH
+

Acid-Catalyzed Reactions
O N2
TFA, -20C

N2
O

()

N2

()

HO

CH2N2

N2

(+)

R
HO

(96%)

O
Mander, Chem. Comm. 1971 773 Tet., 1991 134

N2

(+)

Restriction: Starting ketone must be more reactive than product ketone Precursors to Carbenes:
H R C N N
+

()

N2
N N

(+)

H C: R

empty (+)

"Having become familiar with the peculiarities of diazoketone chemistry while preparing [other compounds] (and, I might add, inured to handling uncomfortably large quantites of diazomethane), it occurred to us that we might be able to substitute a diazo group for bromine."
Lewis Mander

H R

Cl3COCO

O N2 TFA -25C, 2 min (82%)

OCOCCl3 O

filled ()

Gibberrellic Acid
O

H (+, ) R C

E G

E,G

H C: R OMe

Mander, JACS 1980 6626

27-08 diazo chem 11/19/03 8:47 AM

D. A. Evans
DiazoCarbonyl Insertations:
N2 OO R BF3OEt2

Nonalternate Reactivity Patterns of Diazo Functional Groups

Chem 206

O O N2 OBF3 R Mander, Aust. J. Chem. 1979 1975 N2 R O

Web Problem 109. The following is a general reaction for the formation of pyrroles. In this condensation, any of the three reaction constituents may be widely varied. (Ono, "The Nitro Group in Organic Synthesis" Wiley-VCH, 2001. Chapter 10, pp 326-328). Siince it is not clear what the "inorganic" reaction product is, provide us with anything that is mechanistically sound using the reagents illustrated. Key descriptor for answer, "Nitro". O O Me O Me Ph NO2 NH3 Me N H Me Ph

Wolff Rearrangements
Web Problem 332. Stoltz and co-workers recently reported the interesting rearrangement illustrated below (JACS 2003, 125, 13624). MeO O N2 H Me AgOBz, Et3N THF, 45 MeO O Me
95% yield

In the space below provide a plausible mechanism for this transformation.

Web Problem 188. Provide a mechanism for the following reaction that predicts the stereochemistry at the starred (*) carbon atoms (Valentin, TL, 1983, 1621). Key descriptor for answer, "Nitro". i-Pr H H NO2 O NO2 * ** N * i-Pr Me Me N O

AgOBz, Et3N

N2 O H Me H

Cope Rearr

O H C Me H H H
cis olefin

Wolff Rearr

Web Problem 150. Provide a concise mechanism for the indicated reaction in the space below. Key descriptor for answer, "Carbene". N2 O CF3COOH O MeO Web Problem 13. The following transformation was recently reported by Stoltz (J. Am. Chem. Soc 2002, 124, 12426). In addition to the illustrated product, styrene and dinitrogen are produced as by-products in this transformation. Key descriptor for answer, "Carbene". Ph O R N Ph N O
130 C styrene

Diazo-mediated Ring Construction:


Evans, Mitch, JACS 1980, 102, 5956 Ph 1. HClO4 N Me 2. CH2N2 N Me

Morphine alkaloid skeleton CH2


30% yield

C ()
Ph

HClO4

A
CH2N2

C (+)

N2 N Me H CH2N2 A

Intermediate
1

cat Rh2(OAc)4 N2

H Ph

N H

Me

anti diastereoselection > 20:1, 80% yield

Provide a plausible mechanism for this transformation and identify intermediate 1. Your mechanism should provide a rationalization for the product stereochemical relationship.

27-09 diazo chem-2 11/19/03 9:18 AM

D. A. Evans, N. Finney

Hydrazone Transformations-1
O Me2N N CH2
+

Chem 206
Me2N RT, ca. 24h R' CH2Cl2 R 1. O3; 2. DMS

Hydrazone Anions: A useful Reversed Polarity Equivalent


R () N N: R N R N: H R N:

NO2 R'

R (+) H

R () H

C(+)

C()

C()

A
NO2 R' R

C(+)

(40-92%)
O

Bu

N H

N H

Pr

n-BuLi, 0C THF

Bu

N H

Pr

Bu

N H

R=alkyl or aryl R'=H or alkyl


Lassaletta, J-M, et al.Tet. Lett. 1992, 33, 3691.

Pr

J. E. Baldwin, et al. JCS Chem. Comm. 1983, 1040.

PhCHO O Me H CH3CO2 H Me Me

Wolff-Kishner Reduction Procedures


Me R N 2H 4 , NaOCH2CH2OCH2CH2OH, (HOCH2CH2)2O reflux and then heat to 210C HO Me H H Me Me H Me R

O 95% HO H

Pr Ph

Bu

1. n-BuLi 2. H+/H2O

LiO

Pr H Ph

H O

N Me

tBu NH H

tBu 1. n-BuLi, -78C Me O OCH3 Me N H N OCH3 Me O

Barton, D. H. R., Ives, D. A. J., and Thomas, B. R. J. Chem. Soc. 1955, 2056.

For particulary hindered ketones: anhydrous hydrazine or formation of hydrazone under acid catalysis (hydrazine/hydrazine dihydrochloride), then basify. Under these forcing conditions, saponification, epimerization, and methyl ether cleavage can occur.

A A
O Me Me O OCH3

C() C(+)

H+, H2O

Mechanism
R 2C N NH2 + OH R2C ROH N NH R2C H N NH

tBu N Me Me O N H OCH3

hydrolysis 58%

C()

OH (RDS)*

J. E. Baldwin, et al. JCS Chem. Comm. 1984, 1095.

R 2C H

N2

R2 C H ROH

R2 C H H RO

C()

27-10 Hydrazones-1 11/18/03 9:28 PM

Chemistry 206 Advanced Organic Chemistry


Handout27A

An Organizational Format for the Classification of Functional Groups. Applications to the Construction of Difunctional Relationships

D. A. Evans
27A-00-Cover page 11/17/03 1:11 PM

Wednesday , November 19, 2003

Functional Group Classification

Chemistry 206, 2001

An Organizational Format for the Classification of Functional Groups. Applications to the Construction of Difunctional Relationships
D. A. Evans Department of Chemistry & Chemical Biology, Harvard University, Cambridge, MA, 02318 Introduction Among the subdisciplines of chemistry the area of organic synthesis is probably the least organized in terms of unifying concepts and general methodology. This conclusion has been made quite obvious by the relative scarcity of critical monographs covering this important topic.1 The wide structural diversity of organic molecules, the vast abundance of organic reactions, and the restrictions imposed upon these reactions when applied to the synthesis of a complex structure all contribute to the magnitude of the problem of making generalizations in this area. However difficult the overall task of explicitly defining a priori a total synthesis of an organic structure may be, there are certain simplifying features which can be developed to generate logical sets of potential synthetic pathways to a given molecular target . Some of the general guidelines which help to define this task have been outlined.2 Recently, some of the problems associated with reducing synthetic design to a mathematical basis and the application of machine computation to synthetic analysis have been reported.3,4 Difunctional Relationships. One aspect of the synthesis of any polyfunctional target structure deals with strategies associated with the construction of arrays of relationships between heteroatom functional groups which may be denoted as F1, F 2 , etc. The general reactions illustrated below simply represent the union of two monofunctional organic fragments where the functional groups F 1 , F 2 provide the necessary activation for the coupling process. In these reactions, the oxidation states of the associated carbon fragments are purposely left undefined. In relating the generalized notation below to a real situation, if F1-C-C were an enolate, Equation 1 might be used to represent a generalized aldol or Mannich reaction while equation 3 might represent a Michael reaction.
F1 C C F2 F1 C C F2 C

(1)

F1 C C

F2

F1 C C C

F2 C

(2)

F1 C C

F2

F1 C C C C

F2 C

(3)

Henrickson has provided some useful generalizations on the construction of difunctional relationships which are worth summarizing. For example, he defines the construction span as the number of carbons linking F1 and F2. In the cases illustrated above, the product of the reaction illustrated in Equation 1 has a construction span of three. The construction fragments are then defined as the monofunctional reactants, such as F1-C-C and F1-C. In general, construction spans are limited to six or less. This is a consequence of the fact that the operational utility of a given functional group diminishes as it is removed
1) (a) Corey, E. J.; Cheng, X.-M. The Logic of Chemical Synthesis; Wiley, New York, 1979. (b) Fuhrhop, J.; Penzlin, G. Organic Synthesis: Concepts, Methods, Startimg Materials; Verlag Chemie, Weinheim, 1983. (c) Carruthers, W. Some Modern Methods of Organic Synthesis, 3nd ed.; Cambridge Univ. Press, Cambridge, 1987. (d) Organic Synthesis, The Disconnection Approach; Wiley, New York, 1982. (e) Payne, C. A.; Payne, L.B. How To Do An Organic Synthesis; Allyn and Bacon., Boston, 1969. (f) Ireland, R. E. Organic Synthesis, Prentice-Hall, Inc., Englewood Cliffs, 1969. 2) (a) Corey, E. J. Pure Appl. Chem. 1967, 14, 19. (b) Corey, E. J. Quart. Rev. 1971, 25, 455. 3) (a) Hendrickson, J. B. J. Am. Chem. Soc. 1971, 93, 6487. (b) Ugi, I; Gillespie, P. Angew. Chem. Int. Ed. 1971, 10, 914. (c) Corey, E. J.; Wipke, W. T.; Cramer, III, R. D.; Howe, W. J. J. Am. Chem. Soc. 1972, 94, 421. (d) Corey, E. J.; Cramer, III, R. D.; Howe, W. J. ibid. 1972, 94, 440, and earlier references cited therein. (e) Corey, E. J.; Howe, W. J.; Pensak, D. A. ibid. 1974, 96, 7724. (f) Blair, J.; Gasteiger, J.; Gillespie, C.; Gillespie, P. D.; Ugi, I. Tetrahedron 1974, 30, 1845. (g) Bersohn, M. J. Chem. Soc., Perkin I 1973, 1239. 4) (a) Thakkar, A. J. Fortschritte Chem. Forschung 1973, 39, 3. (b) Dungundji, J.; Ugi, I. ibid. 1973, 39, 19. (c) Gelernter, H.; Sridharan, N. S.; Hart, A. J.; Yen, S. C.; Fowler, F. W.; Shue, J.-J. ibid. 1973, 41, 113.

Functional Group Classification

page 2

from the C-C bond being formed. The problem of site or ambident reactivity in systems possessing extended conjugation is the principal liability in the extension of the construction span. This point is illustrated below for both conjugate addition and enolate alkylation (Scheme I).
Scheme I The Problem of Ambident Reactivity
Nu(-) Nu MeO2 C R O

1,6 Addition
4 MeO2C 6 R

H+

-alkylation
OM MeO R

El(+) H
+

MeO El O

Nu Nu(-) H+ MeO2 C R

1,4 Addition

-alkylation

El(+) H
+

MeO El

The objectives of the present discourse are to present an organizational format which can serve to correlate strategies for the construction simple pairwise functional group relationships. As a result of the overwhelming predisposition of nature to employ polar rather than free radical processes in the biosynthesis of organic compounds the chosen organizational format reflects this bias in reaction type. The designation of reactions as polar is recognized to be rather arbitrary since known reactions vary widely in their polar character, ranging from essentially nonpolar radical reactions and weakly polar electrocyclic reactions to strongly polar ionic processes. Of primary concern in this discussion will be those reactions that involve charged species at some point along the reaction coordinate. Charge Affinity Patterns. In order to describe an organizational model for the (4) A: B+ classification and synthesis of heteroatom-heteroatom A B (difunctional) relationships in organic molecules, two familiar (5) ideas will be employed. The first is that in a given target molecule A B A: + B: the various bonds can be ionically "disconnected" (eq 4, 5). That is, if the A-B bond could be cleaved heterolytically, the indicated set of polar fragments would result. This antithetic process suggests ionic precursors suitable for the construction of the target molecule via polar coupling processes. The second well accepted idea is that functional groups determine site reactivities on a carbon skeleton based upon known reactions. That is, the oxygen atom in both acetone and anisole dictates the site reactivities that are displayed for each molecule with nucleophilic and electrophilic reagents. Thus, if the molecule A-B contained one or more functional groups proximal to the bond to be disconnected, () (+) A: A B B+ (6) one pair of ionic precursors, eq 6 or 7, would be strongly favored as plausible precursors. In such a case the favored ionic (7) A: + B: precursors to A-B could be symbolized with either (+) or (-) in the A B target molecule, e.g.5 As an example, two possible polar disconnections for ketone 1 are illustrated below. The parity labels in the target structure suggest plausible monofunctional precursors from which the target structure can be assembled by polar processes. It is also evident that the heteroatom functional groups, =O and OH, strongly bias the indicated polar disconnections.
cheme II Polar Disconnections and Charge Affinity Pattterns
TA
O() R C CH2 (+) () CH2 OH (+) () O () R C CH3 (+) () O() R C CH (+) () CH2 (+) OH 2 () CH2 (+) O ()

TB

5) The use of the symbols, (+) and (-), in no way represents formal positive or negative charges and will always be bracketed to denote this distinction. Other forms of notation have been considered such as (0) and (1) to denote a potential site of electrophilicity or nucleophilicity; however, the chosen symbols convey more direct information to the organic chemist.

() (+)

Functional Group Classification

page 3

For any given atom or heteroatom assemblage which is defined as a functional group linked to a carbon skeleton, the parity labels, (+) and (-), may be employed to denote the positional polar site reactivity, or charge affinity pattern which the functional group confers upon the carbon framework. For the simple molecules shown below (Scheme III) containing a homogeneous set of activating functions, E, there are associated charge affinity patterns 2 - 5 of which each is a sub-pattern of the generalized structure 6. Note that the carbonyl function is defined as =O rather that C=O in this discussion. You might contemplate why this functional group is defined in this fashion.
Scheme III Charge Affinity Patterns of Common Functional Groups
H3 C H3C H 2C CH 2 CH 2 CH CH 2 H C CH 2 OR H2C CH C O (+) C () C (+) C E4 O Br C C () C C (+) C (+) C (+) C E1 E2 E3

2 3 4 5

C OH (+) C

(+) C

() C

(+) C

The notion that an organic structure can be viewed as an "ion assemblage" has an interesting history originating with the work of Lapworth and others.6, 7 Although the ion assemblage viewpoint was developed historically to predict site reactivity in both aliphatic and aromatic systems, this description of an organic structure is equally instructive in defining rational sets of synthetic pathways for a given target structure employing heterolytic processes as the primary set of coupling reactions. Indeed, the thought processes associated with the construction of organic molecules operate intuitively to recognize many subunits of a given structure in terms of polar fragments. The present use of parity labels to denote viable polar fragments simply formalizes this intuition. Classification of Functional Groups (FG). In order to organize general strategies that have been developed to construct heteroatom-heteroatom relationships from monofunctional precursors it is useful to develop a self-consistent classification scheme for single functional groups (FG) based on the concepts of polar disconnection and conferred site reactivity towards nucleophiles and electrophiles. The proposed scheme recognizes the dominate inductive and resonance components of various substituents and establishes8 broad categories for activating functions which correlate similar conferred chemical properties to carbon.9 Four possible functional group categories (F1-F4) are shown below. Those FGs which are more electronegative than carbon provide inductive activation defining the electrophilic potential at the point of attachment denoted as (+). In a complementary fashion, FGs which are less electronegative than carbon provide inductive activation creating nucleophilic potential at the point of attachment denoted as (). Since FG activation through induction and resonance are independent variables which contribute to the overall FG reactivity pattern, four possible classes of functional groups can be defined (Scheme IV). This discussion is reminiscent of the classification of FGs according to their impact on electrophilic aromatic substitution.10
Scheme IV Classification of Functional Groups
Induction Resonance (+)
C

(+)
F1 C

()
F2 C

()
F3 C

(+) (+)

()

(+) ()

() ()

F4

Symbol

6) (a) Lapworth, A. Mem. Proc. Manchester Lit. Phil. Soc. 1920, 64, 1. (b) Lapworth, A. J. Chem. Soc. 1922, 121, 416. (c) Lapworth, A. Chem. Ind. 1924, 43, 1294. (d) Lapworth, A. ibid. 1925, 44, 397. For an excellent review of Arthur Lapworth's contributions to chemistry see: Saltzman, M. J. Chem. Ed. 1972, 49, 750-753. 7) (a) Vorlnder, D. Chem. Ber. 1919, 52B, 263. (b) Stieglitz, J. J. Am. Chem. Soc. 1922, 44, 1293. 8) See reference 3c for an alternate classification scheme for functional groups. 9) For an analysis of the relative importance of field and resonance components of substitutent effects see: Swain, C. G.; Lupton, Jr., E. C. J. Am. Chem. Soc. 1968, 90, 4328. 10) March, J. Advanced Organic Chemistry, 4th ed.; Wiley-Interscience: New York, 1992; pp 507-512.

Functional Group Classification

page 4

E & G-Functions. From the preceding discussion, one might opt for the creation of four classes of functional groups; however, for the sake of ypothetical E-function simplicity, three FG class designations will be chosen. To organize activating (+) () (+) C C C E functions into common categories it is worthwhile to define "hypothetical" 11 having the charge affinity patterns denoted in 6 functional groups E, and G , 6 and 7 respectively. Given the appropriate oxidation state of the carbon skeleton, such functional groups confer the indicated potential site reactivity patterns ypothetical G-function towards both electrophilic and nucleophilic reagents. Any functional groups () (+) () whose reactivity pattern conforms to the ideal pattern or to a sub-pattern of these C C C G hypothetical functions will be thus classified as an E- or G-function respectively. 7 For example, the halogen and oxygen-based functional groups in four molecules illustrated in Scheme III may be classified as E-functions since their respective charge affinity patterns conform to a subset of the charge affinity pattern of the hypothetical E-function. A-Functions. A third hypothetical function, A, (A for (+) (+) C C A 9 amphoteric!) can be defined which has an unbiased charge ypothetical A-function affinity pattern as in 8. Such an idealized functional group (+) (+) (+) () () C C C A C C A 10 activates all sites to both nucleophilic and electrophilic reactions and, as such, include those functions classified as either E or G . 8 (+) The importance of introducing this third class designation is that C A 11 it includes those functional groups having non-alternate charge affinity patterns as in 9, 10 and 11. The differentiation of polar reactivity patterns can be described in an alternative manner. Starting with an ideal A-function, one could imagine a process in which the reactivity pattern is gradually polarized towards E- or G-behavior (Scheme V). Since site reactivity is not an on-off property but varies continuously over a wide range, one could further subdivide A-class functions into those functions with a bias towards E-class or G-class properties. Such a bias could be denoted by the dominant subordinate charge affinity notation in 12 and 13; however, for the concepts to be presented in this discourse, such A-function subclasses are nonessential. It should be emphasized that the purpose of the E- and G-classification is not to rigidly pigeon-hole functional groups based on site reactivity, but only to separate those which are strongly polarized toward E or G behavior. The decision has been made to avoid the pursuit of an overly detailed FG classification scheme since such attempts will dangerously oversimplify problems since an essentially contiguous function cannot be segmented in to discrete parts. Scheme V Alternate vs Nonalternate Reactivity Patterns
Hypothetical A-function
(+) (+) (+) C C C C () C A () C G () C () () C A () C () C () C () C A

12

13

(+) C

(+) C

(+) C

Hypothetical E-function

Hypothetical G-function

11) The symbol E was selected to denote electrophilic at the point of attachment to the carbon skeleton Unfortunately, the symbol N cannot be used to represent those FGs which are nucleophilic at the point of attachment since this is also the symbol for nitrogen. To avoid this conflict, the symbol G was chosen for this FG class designation.

Functional Group Classification

page 5

FG Classification Rules. In the proposed classification scheme the following rules are followed in the assignment of class designations to functional groups. s Activating functions are to be considered as heteroatoms appended to or included within the carbon skeleton. s Activating functions are inspected and classified according to their observed polar site reactivities. s Since both proton removals and addition processes are frequently an integral component in functional group activation, the function, its conjugate acid or base, and its possible proton tautomers are considered together in determining its class designation. s The oxidation state of the FG is de-emphasized since this is a subordinate strategic consideration. E-Functions. For example, carbonyls and carbonyl derivatives will be represented as =X where X may be either oxygen or substituted nitrogen. Well recognized exceptions to the polar class designations illustrated in Scheme I may be found in the chemistry of CO and HCN. In these instances the carbon bearing the heteroatom exhibits well-defined nucleophilic properties. Accordingly these two functional groups will be classified as A-functions by inspection (vide infra).
(+) Table I. Common E-Functions: Symbol: C
OR NR2 O NR E O N

exception: exception:

X, X = halogen

Also consider all combinations of of above FGs; e.g =O + OR

G-Functions. Typical G-class functions are the Group I-IV metals whose reactivity pattern, falls into a subset of 7.
()
2C

() CH CH2 Li () CH3 CH2 MgBr

() C

(+) C

() C

A-Functions. A-functions are usually more structurally complex FGs composed of polyatomic assemblages of nitrogen, oxygen and their heavier Group V and VI relatives (P, As, S, Se). Typical Afunctions, classified by inspection, are provided in Table II.
() Table II. Common A-Functions: Symbol: C
NO 2 SR PR2 NOR S(O)R P(O)R2 NNR2 SO2R
+

A N2 N

N(O)R
+

SR2

PR 3

Functional groups possessing the following general structure, =N-X where X is a hetroatom bearing a nonbonding electron pair, have an expanded set of resonance options which create either an electrophilic or nucleophilic potential at the point of attachment. Remarkably, the dual electronic properties of oximes were first discussed by Lapworth12 in 1924 before the modern concepts of valence bond resonance was developed.
s These FG's are capable of conferring both (+) and () at the point of attachment.
X: () N R (+) H R N H X: N (+) X:

X = OR, NR 2
R () H

12) Lapworth, A. Chemistry and Industry 1924, 43, 1294-1295.

Functional Group Classification

page 6

A Case Study: The Nitro Group. As an example, the class designation of the nitro function is determined by an evaluation of the parent function, its nitronic acid tautomer, as well as conjugate acid and base 14 and 15.
H-tautomer
O HO CH2R

conjugate base
O

conjugate base
HO

+N O

+N O

CHR

+N O

CHR

+N
HO

CHR

nitronic acid

nitronate anion, 14

15

From the collection of transformations of the nitro group one finds that the dominate mode of reactivity of the nitronate anion 14 is that of a G-function while the protonated nitronic acid 15 mirrors the reactivity of an E-function.
14 O + N O
HO

()
CHR

FG

The dominate polar site reactivity (+)


CHR

15

+N
HO

FG

The typical behavior of nitronate anions 14 is summarized in the representative transformations provided in Scheme VI. These moderately nucleophilic species, although they are not readily alkylated, readily undergo aldol and conjugate addition reactions.
cheme VI Selected Reactions of the Nitronate Anion
The Reaction:
O +N O CH2R O +N O CHR

base
O
qq

El(+)
CHR

O +N O

R El

pKa ~ 10
O () + N CH2 R

+N O

The charge affinity pattern:

s This reactivity pattern may be extended via conjugation:

It is no surprise that the charge affinity pattern of this FG may be extended by conjugation, and ,-unsaturated nitro compounds readily participate in conjugate addition reactions (Scheme VII).
Scheme VII Selected Reactions of the Nitronate Anion
O O CH CH R +N O O () (+) CH CH R O +N O O (+) (+) (+) () (+) CH2 CH R

The Reaction:

+N O

+ CH CH R

Nu()

Nu

The charge affinity pattern:

+N O

Nitro aromatics:
(+) X: N R () H

+N O

X:

X: N R H

s The resonance feature which has been exploited:

() N R (+) H

X = OR, NR2

Functional Group Classification

page 7

The non-alternate behavior of the nitro functional group is dramatically illustrated in the transformations provided in Scheme VIII. In both instances the derived anions 16 and 17 are highly nucleophilic.13 The non-alternate charge affinity patterns of these nucleophiles is provided.
Scheme VIII Deprotonated Nitronate Anions
O +N O O +N O
C H CH3

LDA -78 C

O +N O 16 O
C

CH3

()
FG

Li

()

(8)

CH3 C CH3

CH2Li C

n-BuLi -78 C

+N O 17

() ()
FG C C

(9)

CH3

The nitro group also exhibits the potential of undergoing direct displacement under specific conditions, a general transformation characteristic of E-functions. A recent review by Tamura provides numerous literature precedents for this general class of reactions.14 while table III provides some of the cited reactions. Although the NO2 group cannot be considered as a general leaving group, there are a number of conditions under which this moiety can be exploited, particularly when it is either allylic or tertiary.
O + N O R CH R

Nu()
Nu

R CH R + NO2

(+)
FG C

(10)

Table III. Representative Substitution Reactions of the Nitro Group (eq 10).
NO2 Pd(PPh3)3 Me N H CH(CO2Me)2 NaCH(CO2 Me)2 Pd(PPh3)3 Me Me SO2Ph Pd(PPh3)3 Me Ph SPh SPh NO2 NaO2SPh Me 3 SiCN TiCl4 73% Me SPh TiCl 4 65% Me Me NO 2 NO2 N(CH2)5 Ph SnCl4 74% NO2 Anisole SnCl4 94% SiMe3 Me SPh CN Ph Me t-Bu OMe Ph

A particularly useful transformation of the nitro group is the Nef Reaction, a process which transforms NO2 into =O (Scheme IX). A recent comprehensive review of this transformation provides a detailed discussion of this process.15 In addition to the Pinnick review, Seebach has also written a comprehensive review of the diverse chemistry of the nitro functional group.16

13) (a) Henning, R.; Lehr, F.; Seebach, D. Helv. Chim. Acta 1976, 59, 2213-2217; (b) Seebach, D.; Henning, R.; Lehr, F.; Gonnermann J. Tetrahedron Lett. 1977, 1161-1164. 14) Tamura, R.; Kamimura, A.; Ono, N. Synthesis 1991, 423-434. 15) Pinnick, H. W.; Org. Reactions 1990, 38, 655-792. 16) Seebach, D.; Colvin, E. W.; Lehr, F.; Weller, T.; Chimia 1979, 33, 1-18.

Functional Group Classification

page 8
O R H R R O R

Scheme IX The Nef Reaction


s Overall Transformation:
+N O 1) HO

s Mechanism

O +N O

R H R

HO

O +N O

R R

HO +N O

R R

G-Property

nitronate anion

nitronic acid
H+

HO N HO H + O H

R R

HO N HO

R R OH

H 2O -H
+

HO +N HO

R R

E-Property

The Diazo Functional Group. This functional group provides one of the best illustrations of an A-function. As illustrated in Scheme X, both () and (+) polar site reactivity is observed in is reactions with carboxylic acids.
Scheme IX The Nef Reaction
O R H R

s Overall Transformation:

+N O

1) HO O

R R

s Mechanism

O +N O

R H R

HO

O +N O

R R

HO +N O

R R

G-Property

nitronate anion

nitronic acid
H+

HO N H HO + H O

R R

HO N HO

R R OH

H2 O -H
+

HO +N HO

R R

E-Property

The same overall reactivity pattern is expressed by the diazo functional group in the TiffeneauDemjanov ring expansion reaction17 wherein diazomethane functions as the nucleophilic agent in the first step and the functional group is lost as a leaving group in the subsequent step (Scheme XI).
Scheme XI The Tiffeneau-Demyanov Ring Expansion
O + HO CH2 N2 O

CH 2 N2 EtOH

-N2

N2

()

N2

(+)

Restriction: Starting ketone must be more reactive than product ketone

17) For a monograph on ring expansion reactions see: Hesse, M. Ring Enlargement in Organic Chemistry; VCH: New York, 1991.

Functional Group Classification

page 9

Sulfur-Based Functional Groups Sulfonium Salts. The dual electronic behavior of sulfur functions may be illustrated in the reactions of sulfur ylids which are excellent examples of A-functions. As illustrated in Scheme XII, sulfonium salts are effective in carbanion stabilization, a characteristic of G-functions, and sulfonium salts are effective leaving groups, a characteristic of E-functions.
Scheme XII. Sulfonium Salts: Modes of Reactivity
R + S CH3 R R + S
qq

s Carbanion Stablization:
R

+
H
+

CH2

()
R 2S C
R

pKa (DMSO) ~ 18
+ S CH3

s Leaving Group Potential: Good


R

Nu:

S N2

R S R

Me

Nu

qq

(+)
R 2S C

The non-alternate reactivity pattern of trimethylsulfonium ylids is revealed in the cyclopropanation of unsaturated ketones as illustrated in the case below (Scheme XIII).18
Scheme XIII. Reactions of Sulfonium Ylids: Conjugate Addition
qq

Me Me

+ S CH2

(+)

Me

+
S Me

(+)
R 2S C

()
R 2S C

Me

+
S Me

O O

s Nonalternate reactivity pattern revealed in consecutive reactions

Sulfones. Other types of sulfur-derived functional groups exhibit reactivity profiles similar to that exhibited by sulfonium salts. A number of excellent applications of the arylsulfonyl functional group illustrate this point. Two applications utilizing the sulfone functional groups are presented below. The phenylsufonyl moiety strongly stabilizes carbanions and may be equated with the CN FG in its potential for hydrocarbon acidification.19 In addition, this FG is a respectable leaving group in selected situations. In comparisons with sulfonium ions (Scheme XV), arylsulfonyl-stabilized carbanions are more nucleophilic than sulfonium ylids (G-property), while ArSO2- is a poorer leaving group than Me2S- (EProperty).
Scheme XV. Sulfones: Modes of Reactivity
Me Ph Me O S O Me Ph Me O S O BuLi Me O S O Me Li Ph

more nucleophilic S CH2 than: R


R + S CH3 R

pKa ~ 25

poorer leaving group than:

18) Corey, E. J.; Chaykovsky, M. J. Am. Chem. Soc. 1965, 87, 1352-1364. 19) For an excellent compilation of pKa data for organic functional groups in DMSO see: Bordwell, F. G. Acc. Chem. Res. 1988, 21, 456-463.

qq

Functional Group Classification

page 10

Julia's use of phenylsulfonyl carbanions in the synthesis of trans-chrysanthemic acid provides the justification for defining this functional group as an A-function (Scheme XVI).20
Scheme XVI. The Julia Chrysanthemic Acid Synthesis
Me Me Me O Li Ph S O Me (+) O OEt Me OEt OLi Me Me SO2Ph

trans chrysanthemic acid


Me H CO2Et H

()
R2SO2 C R2SO 2

(+)
C

The dual electronic properties of the sulfone functional group are illustrated in the Julia synthesis of vitamin A (Scheme XVII).21 In this application, the E-property of the FG is exploited in the base-induced elimination reaction to generate the fully conjugated polyene.
Scheme XVII. The Julia Vitamin A Synthesis
Me Me Me SO2Ph () Li Me Br Me Me CO 2R Me CO 2R Me Me Me SO2 Ph Me CO2R

()
R2SO2 C

Me KOH/MeOH

Me

Me

- PhSO 2

SO2Ph Me R (+)

O OR

Me

(+)
R2SO2 C

Julia & Co-workers, Bull. Soc. Chim. Fr. 1985, 130

For additional reading on the utility of the utility of sulfones in organic synthesis a monograph on this subject has recently appeared.22 Several other reviews providing extensive literature coverage are worth reading.23 Organoboranes. The boron atom exhibits many of the common reactions normally attributed to metals, and when bound to carbon, serves as an excellent source of nucleophilic carbon.24 The transformations provided in (Table IV) represent but a few cases which demonstrate the G-properties of this activating function.25,26,27,28,29,30,31
20) (a) Julia, M.; Guy-Rouault, Bull. Soc. Chim. Fr. 1967, 1141. (b) Campbell, R. V. M.; Crombie, L.; Findley, D. A. R.; King, R. W.; Pattenden, G.; Whiting, J. J. Chem. Soc., Perkin Trans. I 1975, 897. 21) Arnould, D.; Chabardes, P. Farge, G.; Julia, M. Bull. Soc. Chim. Fr. 1985, 130. 22) Simpkins, N. S. Sulfones in Organic Synthesis, Pergamon Press, New York 1993. 23) (a) Trost, B. M. Bull. Chem. Soc. Jpn. 1988, 61, 107-124. (b) Magnus, P. D. Tetrahedron, 1977, 33, 2019-2045. 24) (a) Brown, H. C. Boranes in Organic Chemistry, Cornell University Press, New York 1973. (b) Cragg, G. M. L. Organoboranes in Organic Synthesis, Marcel Dekker, New York, 1973 25) (a) Kow, R.; Rathke, M. J. Am. Chem. Soc. 1973, 95, 2715. (b) Zweifel, G.; Fisher, R. P.; Horng, A. Synthesis 1973, 37. (c) Matteson, D. S. ibid. 1975, 147. 26) Negishi, E.; Abramovitch, A.; Merrill, R. E. J. Chem. Soc., Chem. Commun. 1975, 138. 27 For a recent citation on allylboron-based nucleophiles see: Wang, Z.; meng, X. J.; Kablaka, G. W. Tetrahedron Lett. 1991, 32, 5677-5680 and references cited therein. 28) Marshall, J. A. Synthesis 1971, 229. 29) (a) Brown, H. C.; Rhodes, S. P. J. Am. Chem. Soc. 1969, 2149, 2149. (b) Hawthorne, M. F.; Dupont, J. A. J. A m . Chem. Soc. 1958, 80, 5830. 30) (a) Pelter, A.; Subrahmanyan, C.; Laub, R. J.; Gould, K. J.; Harrison, C. R. Tetrahedron Lett. 1975, 1633. (b) Pelter, A.; Harrison, C. R.; Kirkpatrick, D. ibid. 1973, 4491. (c) Pelter, A.; Harrison, C. R. J. Chem. Soc., Chem. Comm. 1974, 828. (d) Naruse, M.; Utimoto, K.; Nozaki, H. Tetrahedron 1974, 30, 3037.

Functional Group Classification

page 11

Table IV. Reactivity Patterns for Organoboranes


Entry
25 R B R R O Ph R

Reaction
base
R B R R Me R O OBR2 B R

Charge Affinity Pattern


El(+)
R B R CH2 R ()

CH2

26 R B R

Cl

()

Ph

C Me

27 R 2B

() R R

()

Me

OTs

Me

OTs

Me ()

28

HO

B(OH)2

B(OH)3
Ph Cl Ph ()

29 R 2B

Ph

HO

(HO)3B

C
Cl

30 R

R R'

B R

El(+)

El + R'

El

() (+)

B R

C
R2B R'

El(+) = n-C 6H13I, C3H5Br, ethylene oxide, CH2I2, MeSO3 H


OH 31 R R O CH CH2 1) addition 2) H 2O2 R OH () (+)

B R

stereochemical aspects of this reaction not determined

The potential for non-alternate charge affinity patterns for boron have been revealed in the reactions of acetylenic and vinylic boron ate complexes (Table IV, entries F, G).30,31 These compounds exhibit high nucleophilicity towards a variety of electrophiles to the boron atom. The origin of such -nucleophilicity could be a consequence of conjugation32 (e.g., 19) not observed with the heavier metallic elements which are attacked by electrophiles to the metal where the alternate mode of conjugation 1 8 is possible.33 In principle, both types of conjugative stabilization are possible with a range of organometaloids; however, in practice this is not the case. It would be expected that the effects of
31) (a) Utimoto, K.; Uchida, K.; Nozaki, H. Tetrahedron 1973, 30, 4527. (b) Utimoto, K.; Uchida, K.; Nozaki, H. Chem Lett. 1974, 1493. 32) (a) Harmon, G. D.; Traylor, T. G. Tetrahedron Lett. 1975, 939, and reference cited therein. (b) for example of delocalization of type 25 involving R3B see Hanstein W.; Traylor, T. G. ibid. 1967, 4451; (c) for the reaction of vinylsilanes electrophiles see Miller, R. B.; Reichenbach, T. ibid. 1974, 543, and references cited therein. 33) Kitching, W. in "Organometallic Reactions," Vol. 3, E. I. Becker and M. Tsutsui, Ed., Wiley-Interscience, New York 1972, pp. 319-398.

Functional Group Classification

page 12

conjugation, such as that illustrated in 24, would be more important in those systems having shorter C-M bonds, a situation which may be unique to boron. It is noteworthy that the other group III and IV organometallic compounds, R3MCH=CH2 (M = Al, Si, Ge, Sn) react with electrophilic reagents to the metal. These elements all exhibit polar reactivity patterns common to G-class functions.
M H El
18

+
C

R H H

R H C

-attack

R R

M H

-attack

R R

+
M C H El
19

C H

H C H

(11)

El(+)

Metals. In deriving a class designation for metals, M, bound to carbon, two reaction types are considered. Metals undergoing exclusive substitution at the metal-carbon bond by electrophiles, El+, are classified as G-functions (eq 12), while metals which are involved in redox processes (eq 13) are classified as A-functions since such organometallic compounds also exhibit G-type behavior.
() (+)

R
(+)

M
()

+ El(+) + Nu()

R R

El Nu

+ +

M(+) M()

(12) (13)

TlX2 I The organic chemistry of Tl(III),34and 2 KI (14) (eq 14-16) illustrate the role of metals as - TlX leaving groups (reductive elimination). Oxidative addition reactions of metal carbonyl anions and alkyl OMe OMe MeOH halides provide examples of the reverse process.36 C A (15) - TlX In general, transition metal-mediated cross-coupling OMe TlX2 reactions provide a useful illustration of the APdX classification of redox metals (eq 17).37 The CH(CO 2 Me)2 NaCH(CO2 Me)2 (16) assignment of charge affinity labels to R1 and R2 in - Pd(0) this case is arbitrary. Employing polar processes as the basis set of synthetic reactions, existing functional groups may reductive R (+) elimination (17) be organized according to their known chemical L M R R + LnM n properties. Any number of positions may be taken R () relative to the classification of atom reactivity. The M = Pd, Fe, Cu, Ni goal of this section has been to define a general classification scheme which may be used to organize the multitude of different strategies which have been developed to construct pairwise functional group relationships in organic molecules.

Pd(II)35

34) 35) 36) 37)

Taylor, E. C.; McKillop, A. Acc. Chem. Res. 1970, 3, 338. Trost, B. M. Acc. Chem. Res. 1980, 13, 385-393. Ellis, J. E. J. Organomet. Chem. 1975, 86, 1. (a) Neuman, S. M.; Kochi, J. K. J. Org. Chem. 1975, 40, 599. (b) Normant, J. F. Synthesis 1972, 63. (c) Tamao, K.; Kiso, Y.; Sumitani, K.; Kumada, M. J. Am. Chem. Soc. 1972, 94, 9268. (d) Tamaki, A.; Magennis, S. A.; Kochi, J. K. ibid. 1973, 95, 6487.

Functional Group Classification

page 13

Classification of Difunctional Relationships. One of the basic assumptions employed in synthetic design involves the maximum utilization of existing functionality at all intermediate points in the construction of a polyfunctional molecule. Such guidelines aid in minimizing the number of side reactions and protection-deprotection steps during the assemblage operation. In the synthesis of even simple difunctional organic molecules, the relative positioning of the two activating functions on the carbon framework strongly influences the reaction types that will usually be employed to establish the difunctional relationship. Using the general notation developed in the previous section for activating functions, two distinct classes of difunctional relationships which may be defined between ideal E- and G-functions which may be defined are illustrated in Table V. Paths. Difunctional relationships between heteroatoms having "matched" charge affinity patterns will be defined as consonant while unmatched relationships will be labeled dissonant. It should be pointed out that the charge affinity notation is unnecessary to define the appropriate relation; other parity labels could serve equally well. For example, the number of bonds between E- and G-functions could be used to define the appropriate relationship. Employing E-functions for the purpose of illustration, the two carbonyl groups in 20a have a matched charge affinity pattern along the potential construction path. Since they are separated by three atoms they can be defined as 1,3-consonant (1,3-C). The symbol notation 20b transmits information relative to the EE' positioning along the construction path and since the E-symbol represents a homogeneous class of electronically equivalent functional groups, a common symbol is employed. In those cases where it is necessary to recognize oxidation states of carbon to derive a symbolic structural notation, one may easily do so.
Table V. Consonant & Dissonant Difunctional Pairwise Functional Group Relationships
Consonant Relationships
O Me O Me Cl Me NMe2 OMe OMe Me O OMe O E1 E2 E2 OR NH 2 O CO2 R Me E2 O H OTHP O OMe E E1 E2 E1 E1

Symbol

Notation

Dissonant Relationships

Symbol
G

Notation

Li

CH2 Cl

1,1D

1,3C

1,2D

1,4D

1,5C

1,4C
Li O G E O O

O E E

D-cycles

O NH E

C-cycles
O O E1 E2

1,2D, 1,4D

(+) O Me O OMe ()E 1 E 2 () O (+) E2 (+) E1 (+) (-) symbol deleted for convenience

(+) () (+)

20a

20b

21a

21b

Functional Group Classification

page 14

Cycles. In cyclic structures, a heteroatom attached to or contained within the cycle creates a relationship with itself. For non-arbitrary mathematical considerations it is convenient to define an evenmembered ring with or without a single functional group as consonant and corresponding odd-membered rings as dissonant. For the bicyclic ketone 21a, both of the oxygen heteroatoms, denoted as E1 and E2, establish consonant relationships with each other via all bond paths and individually by virtue of their position either attached to or contained within an even-membered ring. Consonant and Dissonant Bond Paths. In contrast to the uniformity with which consonant relationships may be established through common classes of polar processes, the synthetic methods and functional groups required for the construction of the bonds define a D-relationship are quite varied and involve either more steps, more functional groups or more reactive intermediates than reactions leading to C-paths. This statement will be reinforced in a series of case studies (vide infra); however a single case is presented to reinforce this assertion. Consider the Michael transform executed on the 1,5- and 1,4-diketones shown below (eq 18, 19). In the first instance, the transform may be executed using only the functional groups illustrated; however, this is not possible with the dissonant dicarbonyl relationship since one of the resulting polar fragments will be electronically mismatched with its associated FG. In the illustrated disconnection (eq 19), the electronically mismatched fragment is the carbonyl anion.
O O (+) () (+) R 2

,5-Consonant Relationship R 1 (+) ()

Michael

O R 1 (+) (+) () ()

O (+) R2

(18)

1,4-Dissonant Relationship

R 1 () O

(+) () (+) R 2

Michael

O R1 O () (+) () (+) R 2

(19)

Acyl anion

Nef
O () NO 2 (+) R 2 () NO2 R1 (+) O

Michael

R1

One possible solution to the construction of this dissonant relationship is through FG manipulation. In the present instance the application of the Nef transform (vide supra) provides the opportunity to match the charge affinity patterns so that the Michael transform may be properly executed. The use of Afunctions in this fashion is just one of a number of strategies which may be employed to construct dissonant difunctional relationships. In conclusion, dissonant pairwise relationships, either identified in simple acyclic molecules or within complex cyclic structures, generally pose a greater synthetic challenge and represent seams of lower flexibility within the carbon framework. At this point, it may be instructive to the reader to contemplate a synthesis strategy based on how and when D-relationships are incorporated into target structures. This point will be addressed later in the discussion. Synthesis of Consonant Difunctional Relationships. E2 Every complex polyfunctional molecule may be analyzed structurally CH2OH (+) in terms of its individual consonant or dissonant construction paths or (+) (+) cycles. For example, in the alkaloid lupinine (22) all possible construction (+) paths interconnecting E1 and E2 are consonant. On the other hand, N E1 mesembrine (23) 38 contains the potential dissonant paths and cycles (+) (+) illustrated in heavy lines. Consonant paths within the polyatomic 22 (lupinine) framework define seams in the structure that may be constructed using aldol and related condensation processes.
38) (a) Curphey, T. J.; Kim, H. L. Tetrahedron Lett. 1968, 1441. (b) Keely, S. L.; Tahk, F. C. J. Am. Chem. Soc. 1968, 90, 5584. (c) Stevens, R. V.; Wentland, M. P. ibid. 1968, 90, 5580; (d) Shamma, M.; Rodrigues, H. R. Tetrahedron 1968, 24, 6583.

()

(+) ()

(+) R2

(20)

Acyl anion equivalent

Functional Group Classification

page 15

s Regarding the number of different possibilities available for the synthesis of a consonant difunctional relationship interconnected by n bonds, there exists a set of n different connective operations that may be employed to establish any bond along the construction path from monofunctional or consonant polyfunctional precursors.39
OMe OMe E1 E2 E1 E2 Ar Ar Ar

dissonant bond paths (cycles) Ar O N Me Ar O N

23
E1 E2 E1 E2

Me

23 (lupinine)

Shortest consonant bond path

In the analysis of potential routes to structures like lupinine, identify the shortest consonant bond path and then proceed to carry out all polar disconnections along that bond path (Scheme XVIII). Since there four bonds interconnecting =O and N (E1 and E2), there will be four associated transforms which one may execute using the illustrated functional groups. In each set of precursors the intrinsic polar reactivity patterns of the heteroatoms are accommodated in the coupling process. The resulting adducts containing the requisite nitrogen-oxygen relationship may then be ranked in order of desirability by considering criteria such as chemical feasibility of the coupling step, ease of subsequent transformation to the target structure, and availability of precursor fragments. In the present example, transforms A and B might be more highly ranked that transform C while transform D might be discarded since it does not lead to structural simplification.
Scheme XVIII
Ar Ar equivalent to: E1 (+) (+) E2 () () Ar Ar O HN Me Ar equivalent to: E E 1 (+) () (+) 2() () E1 (+) (+) E2 () () Ar HO Ar equivalent to: E E 1 (+) () (+) 2() RO2C N Me Ar Ar equivalent to: () E1 (+) (+) E2 () () HO N+ Me N+ Me

Shortest consonant bond path

(+) In those cases when a given consonant or dissonant relationship is 6 4 separated by a significant number of bonds, it is strategically worthwhile to consider the option of incorporating additional functions to aid in the 2 (+) R R E1 (+) construction of the desired target molecule. The relative placement of such a O 24 functional group is of prime importance in dictating the subsequent polar disconnections that are perceived in generating a plausible synthetic tree. This point is illustrated when considering plausible precursors to ketone 24 (Scheme XIX). In this structure, the =O FG establishes a 1,5-relationship with itself on the six-membered ring. Through the addition of an appropriate second

39) The presence of a quaternary or bridgehead center along the construction path limits bond construction to those adjacent to the center.

Functional Group Classification

page 16

activating function to the target molecule 24, an expanded set of potential disconnections is created. In the placement of the second FG, the charge affinity pattern of the resident FG should be used. For example, consider the installation of a second FG, E2, at the (+) sites on the ring to set up aldol or Claisen transforms. In a complementary fashion the addition of C-E2 fragments to the () sites will open up the execution of the two possible Dieckmann transforms.40 The preceding analysis leads to the three precursors 26a-26c. Each of which contains a 1,5-consonant difunctional relationship between the carbonyl functions. These subgoals now become the focus of the next level of analysis wherein the preceding logic is again applied. It should be emphasized that the precursors illustrated in Scheme XIX are not inclusive but represent one set which leads to the generation of a synthetic tree based upon aldol and related reactions. The point to be emphasized is that in the first stage of the analysis where functionality is being added to the target structure, consonant, rather than dissonant relationships should be created.
Scheme XIX
(+)

A
E1 E2 R E2

Aldol
equivalent to: O OH OH equivalent to: E1 E2 R O R R O Me

O R

25a
E1 (+) (+) R

26a
O

Where (+) add E2

Aldol
O

Me R

25b

26b Dieckmann
RO 2C R R

C
() E1 () E1 () R R

RO 2C equivalent to: O

25c

Where (-) add C-E2

D
E1 E2 R equivalent to: O

Dieckmann
R CO2R

26c CO 2R

25d

40) To be completely rigorous with regard to this analysis, the addition of C-E2 to the 4-position should also be considered; however, the E1-E2 construction span from such a precursor is sufficiently large as to render this precursor less attractive than the other precursors 25a-25d.

Chemistry 206 Advanced Organic Chemistry


Handout 27B

Synthetic Applications of -Diazocarbonyl Compounds

An Evans Group Afternoon Seminar Krista B. Goodman January 15, 1999

D. A. Evans

Monday, November 17, 2003

27B-00-Cover page 11/17/03 1:16 PM

Krista Beaver

Synthetic Applications of -Diazocarbonyl Derivatives

Chem 206

Synthetic Applications of -Diazocarbonyl Compounds


An Evans Group Afternoon Seminar Krista Beaver January 15, 1999 Leading References: McKervey and Ye, Chem. Rev. 1994 1091 Doyle, McKervey and Ye, Modern Methods for Organic Synthesis with Diazo Compounds, Wiley, 1998

Synthesis of -Diazocarbonyl Compounds


First synthesized by Curtius in 1883 by diazotization of -amino acids Arndt-Eistert synthesis (1927)
O R OH 1. (ClCO)2, DMF 2. CH2N2 R O N2

Arndt and Eistert, Ber. Dtsch. Chem. Ges. 1927 60B 1122 Pettit, JOC 1986 1282

Diazo Transfer
O O R' RSO2N3, base R=Me, Ts, etc. R N2 Regitz, ACIEE 1967 733 O R' R , hv or M R' N N O R acid R' N N O R For temporary activation of carbonyl compounds prior to diazo transfer, see Danheiser, JOC 1990 1960 R'

Diazocarbonyl Compounds: Structure and Nomenclature

Diazocarbonyl

Diazonium

Acyl Transfer
cyclopropanation insertion rearrangement ylide formation displacement rearrangement solvolysis
O O O N O N2 ROH R O O N2

Badet, JOC 1993 1641

27B-01 11/11/01 7:59 PM

Krista Beaver

Synthetic Applications of -Diazocarbonyl Derivatives

Chem 206

Some Reactions of Diazocarbonyl Compounds


O O R O R H O R HO O R R1 R CNS SeR O R AcO R1 SeR O R1 R2 H O R1 H SeR R O R1 R2 R3 R RS O R1 R SR H O R1 SR R O R H R1 PO(OR)2 H O R1 O2P(OR2)2 R H N2 O R1 OCOR2 R1 OH O R1 R H R R1 NR2R3 H R1 SiR3 R H R1 BR2 R X O R1 X R H O R1 X R H O R1 OH O R H R1 OR O R1 R2 OR O R1 OTs

Acid Catalyzed Reactions of Diazo Compounds


Review: Smith, Tet. 1981 2407
O H 3C N N CH3 acid H 3C N N O CH3

Diazocarbonyl

Diazonium

Common acids include BF3OEt2, HBF4, TFA, etc. Mechanism of activation is unclear for both Lewis and protic acids; activation may occur by protonation on C or O

Acid-Catalyzed Reactions
O N2 TFA, -20C (96%) HO O Mander, Chem. Comm. 1971 773 Tet., 1991 134 "Having become familiar with the peculiarities of diazoketone chemistry while preparing [other compounds] (and, I might add, inured to handling uncomfortably large quantites of diazomethane), it occurred to us that we might be able to substitute a diazo group for bromine." O N2 TFA -25C, 2 min (82%) OMe O Mander, JACS 1980 6626 Lewis Mander OCOCCl3 O O

Adapted from McKervey, Chem. Rev. 1994 1090

Cl3COCO

Gibberrellic Acid

27B-02 12/20/99 4:05 PM

Krista Beaver

Synthetic Applications of -Diazocarbonyl Derivatives


More Acid Catalysis -Ketoester synthesis:
O Me HCl N2 (100%) Cl Me EtO O N2 H + R H O SnCl2 (50 - 90%) EtO O O R'

Chem 206

Olefins as nucleophiles:
Me O

Me

Roskamp, JOC 1989 3258


O

Smith's cyclopentenone annulation:


O N2 Me Me BF3OEt2 (40 - 65%) Me

Mander

Yields are good when R is aliphatic; moderate when aromatic

Ring expansion:
Me Me O Me O Me BF3OEt2 EtO2CCHN2 (81%) Me O Me O Me CO2Et

Lindlar's cat. (100%)


O

Smith, TL 1975 4225

Me Me

Jasmone

Me

Ghosh, Chem. Comm. 1988 1421

Br

Aplysin
Me O Me

Me

Rearrangement:
N2 O O BF3OEt2 O O N2 OBF3 R

Substitution:
N2 R O H O S N O O Me Me CO2CH2CCl3 ROH, BF3OEt2 ">60%" N O RO H O O S Me Me CO2CH2CCl3

Mander, Aust. J. Chem. 1979 1975

Thiols also work well

John and ThomasTL 1978 995

Polyene cyclizations:
O BF3OEt2 N2 Me Me H 46% 12% Me O + TESO Me O

Tetrahydrofuran Synthesis:
R O BF3OEt2 BnO2CCHN2 (53 - 87%) O CO2Et R OH

diastereoselection 3:1 - 20:1

Smith, JACS 1981 2009

Diastereoselectivity increases with size of R; independent of Lewis acid or protecting group

27B-03 12/20/99 4:14 PM

Angle, TL 1998 3119 TL 1998 8195

Krista Beaver

Synthetic Applications of -Diazocarbonyl Derivatives Substitution Reactions Base-Induced Reactions


H Me Mg N O O S

Chem 206

Deamination:
H 2N H O S N O O Me Me COOH NaNO2, Br2 Br Br O H O O S N O Me Me COOH R N2

Aldol-type reactions:
O Li + R1 R2 O O R N2 OH R1 R2

(90%)

Me COOH

Kapur and Fasel, TL 1985 3875

LDA is the optimal base for lithiation

Pellicciari, JCS Perkins I 1985 493

Synthesis of -substituted chiral acids:


OH Me H2N COOH H Me N2 H O H Me Nu
-

O O

Me Nu

COOH H

Ester alkylation:
O + O N2 Li CO2Et HO N2 O CO2Et Rh2(OAc)4 100% O CO2Et O

Nu = Br, Cl, F Ingold, Nature 1950 179 For other examples, see McKervey, Chem. Rev. 1994 1091

Displacement occurs with retention of stereochemistry

Rapoport, JOC 1985 5223

Gilbert-Seyferth Reagent:
(MeO)2OP H + N2 R1 O R2 KOt-Bu

Reaction with Boranes


BH3 B 3 D EtO2CCHN2, then D2O (97%, 100% d1) H CO2Et

R1

R2

Seyferth, JOC 1971 1379 Gilbert, JOC 1982 1837

Hooz, JACS 1969 6195


O N2 Bu3B N2 OBBu2 Bu n-BuLi, then MeI (61%) O Bu Me MeO

Mechanism?

CF3COOH O

Wojtkowski, JOC 1971 1790

27B-04 12/20/99 4:24 PM

Krista Beaver

Synthetic Applications of -Diazocarbonyl Derivatives Carbenoid Reactions: The Catalysts


Review: Padwa, ACIEE 1994 1797

Chem 206

Ligand Effects: Selectivity


Methine versus methyl:
O O H 3C N2 O Rh(II) O H3COC H3COC O + O O

Decomposition can be catalyzed by:


Heat or light Transition metals, including CuII, RhII, MnII, FeII, CoII, Ni0, NiII, ZnII, MoII, RuII, RuIII, PdIII

Most common catalysts:


Copper (I): Rhodium (II): CuOTf, Cu(OTf)2, CuSO4, CuX, Cu(acac)2 Much milder catalyst than Cu (introduced in 1973 by Tessi)
CH3 H 3C O

Doyle, JACS 1993 958

Rh2(OAc)4 Rh2(pfb)4 Rh2(acam)4

90:10 38:61 100:0

Cyclopropanation versus C-H Insertion:

Structures generally contain bridging ligands and contain a Rh-Rh single bond Reaction pathways are highly sensitive to steric and electronic effects
Me O L O Me Rh O O

Rh(II) N2 H 3C O +

H 3C

CH3

Rhodium Carboxylates:
Me O L

Rh2(OAc)4, Rh2(tfa)4, Rh2(oct)4, Rh2(tpa)4, Rh2(pfb)4 Rh2(acm)4, Rh2(cap)4, Rh2(CF3CF2CF2CONH)4 Padwa and Doyle, JACS 1993 8669

O CH3 Rh2(OAc)4 Rh2(pfb)4 Rh2(cap)4 44:56 0:100 100:0

O O Rh O

Rhodium Carboxamidates:

Rhodium Acetate

More Competition Experiments


Dipolar Cycloaddition versus C-H insertion:
O H N2 O E E E O E Rh2(OAc)4 Padwa and Moody, Tet. 1993 5109 Rh2(pfb)4 Rh2(cap)4 75:25 0:100 100:0 CH3 CH3 Rh(II) + O Ar O O CH3

Me

Transition Metal Catalyzed Diazo Decomposition


SCR2 S:

MLn B

L nM

LnM CR2

R 2C N 2

LnM CR2 N2

N2 Doyle, Chem. Rev. 1986 919

Conclusions: These results imply that the metal is involved in the transition state Reaction pathways can be controlled by tuning the ligands on the metal

27B-05 12/20/99 4:33 PM

Krista Beaver

Synthetic Applications of -Diazocarbonyl Derivatives


Me

Chem 206

Generalizations: Sigma Bond Insertion


O R N2 X-H OEt catalyst R X H O OEt Me O

Tandem O-H Insertion/Claisen Rearrangement


O OMe N2 98% ee + Me OH Rh2(OAc)4 PhH, , 20 min (66%) Me O 95% ee OCH3 HO O

Reviews

O-H Insertion: Moody Tet. 1995 10811 C-H Insertion: Sulikowski Tet. Asymm. 1998 3145

When X is a heteroatom, insertion is facile


O

H O Me CO2Me [3,3] fast Me

H O Me CO2Me

When X is carbon: Only intramolecular processes are generally useful 5 - membered ring formation is favored in general Order of selectivity: methine > methylene > methyl

Me

Z-Enol Transition State slow O COOH Me Me O O

Wood, JACS 1997 9641 Wood, JACS 1999, in press


Me Me

O-H Insertion Reactions


CO2Me MeO2C OH OMEM N2 CO2Me O OMEM CO2Me CO2Me CO2Me OH O COOH

The opposite enantiomer is observed!


O OCH3 O 47% ee

OMe

PhH, , 18 hrs (75%)

HO

O Me CO2Me

HO Me

Rh2(OAc)4 (75%)

E-Enol Transition State

Ganem, JACS 1982 6787


O TMSO Me H AcO H H O Me OH EtO2C N2 PO(OEt)2 Me H H

Chorismic Acid
Me O TMSO H O OH Me O NH N2 O

Merck Thienamycin Process


H H OH O O NO2 Rh2(oct)4 PhH, 80 C 100% OH NH3 S CO2 O CO2p-NB Me N H H OH

CO2Et

Rh2(OAc)4, then NaH (75%, 2 steps) AcO

"the most complex alkoxyphosphonate yet described"

Fuchs, TL 1994 7163


O

Thienamycin
Salzmann, JACS 1980 6161

27B-06 12/20/99 4:39 PM

Krista Beaver

Synthetic Applications of -Diazocarbonyl Derivatives C-H Insertion: Reactions


O R S N O H hv Ph N O H S Me Me CO2Me N2 OEt + R2 R1

Chem 206

Generalizations: Cyclopropanation
O R1 catalyst R OEt R2

N2 Ph

Me Me

CO2Me

Reviews: Davies, Ald. Acta. 1997 107 Davies, Tet. 1993 5203 For subsequent reactions: Calter, Evening Seminar 1992

Corey, JACS 1965 2518


Me Me H AcO H Me OAc N2 Rh2(OAc)4 (59%) AcO Me H H Me OAc MeO2C O O Me

Electron rich olefins work best Both concerted asynchronous and stepwise mechanisms have been proposed Cyclopropanes can participate in tandem reactions

Cyclopropanation Followed by Rearrangement


N2 O OEt Rh2(oct)4 H CO2Me OEt Et2AlCl (88%) EtO CO2Me

Wenkert, JOC 1982 3243


N2 O Rh2(OAc)4 (65%) AcO H3CO

O +

(87%)

Davies, JOC 1992 4309; TL 1992 453


AcO H3CO

AcO H3CO

Adams, JACS 1994 3296

Diastereoselection > 99:1 Cu(TBS)2 TBSO O O + OBn O BnOH2C 50% ee OBn OH Me O O N2 (84%) TBSO H Me O O O TBSO H Me O O

BnO BnO

O O N2 Rh2(5R-MEPY)4

BnOH2C

Et2AlCl (80%)

Doyle, JACS 1994 4507

97% ee

Diastereoselection 93:7 O

For a review of catalytic enantioselective carbene reactions, see: Doyle, Chem. Rev. 1998 911

HO Me O H CO2Me

TBSO

Me O O

Corey, JACS 1985 5574

Antheridic Acid

27B-07 12/20/99 4:46 PM

Krista Beaver

Synthetic Applications of -Diazocarbonyl Derivatives


More Cyclopropanation
Me H CO2Me (79%) 90% ee Ph Me Ph Bchner, Liebigs Ann. Chem. 1893 214 Doering, JACS 1956 5448 CO2Me N2

Chem 206

Reaction with Aromatic Rings


Discovered by Bchner (1893)
O OEt E E

N2 Ph CO2Me Rh2(TBSP)4 Me

hv

Davies, TL 1994 8939


DMB N O Rh2(OAc)4 pinacolone 120C (62%) N H N H DMB N O N O O DMB

Initial experiments gave poor selectivity, but transition metals help...


H3CO O N2 OCH3 + O N2 OEt Rh (II) (73%) E E OEt

(35%) E H3CO

+ 6 other products

N2

N H

N H

OCH3 +

Wood, JACS 1997 9461

Staurosporine
N H N H

Tessi, Chem. Comm. 1980 765 JOC 1981 873

Bchner Reaction: Confertin Synthesis


Me Me AcO Rh2(mandalate)4 AcO N2 Me O Me O

O + Ph N2 Davies, TL 1993 7243 O CO2Me CO2Me Cl Cl Cl Cl Rh2(S-TBSP)4 OMe (79%) 94% ee H

Ph Ph CO2Me 1. KMnO4, NaIO4 2. Me2SO4, K2CO3, acetone (97%, two steps) Ph (82%) H CO2Me CO2Me O Me H Me O H Me H

McKervey, Chem. Comm. 1988 1028 JCS Perkins I 1991 2565


Me O H

1. 6N HCl, 2. ClSO3H (84%)

Li2CuCnAr2

Confertin

Corey, TL 1994 5373

Me OTBS

27B-08 12/20/99 4:59 PM

Krista Beaver

Synthetic Applications of -Diazocarbonyl Derivatives Ylide Formation


O R N2 OEt O R2X catalyst R X R R OEt MeO2C O Me OAc N2 O R

Chem 206

Dipolar Cycloadditions: Carbonyl Ylides


TMSO Rh2(OAc)4 MeO2C O O R TMSO (66%) MeO2C O O R TMSO

Reviews:

Padwa, Chem. Rev. 1991 263 Padwa, Chem. Rev. 1996 223 Barnes, Evening Seminar, March 16, 1993
H

Merck, TL 1994 9185


Me H O N2 H Rh2(OAc)4 CO2Et O H CO2Et O H O H Me AcO Me H H

Zaragozic Acid Skeleton


AcO H Me Me

X is generally S, O or N and can be sp2 or sp3 hybridized Ylides often undergo sigmatropic rearrangements or cycloadditions [2,3]-Sigmatropic rearrangement:
Ph SPh O O OMe N2 Rh2(OAc)4 E S O

(86%)

H O H O

CO2Et

Dauben, JOC 1993 7635


Me N O N SPh E Bz O N2 O O OMe Rh2(pfb)4 MeO2C O N Me (93%) O MeO2C

Tigilane Skeleton
O N O H H Me

Kido and Kato, JCS Perkins 1 1992 229

Acorenone B

Padwa, JOC 1995 2704

N Bz

N Bz

Lysergic Acid Skeleton

Stevens Rearrangement ([1,2] alkyl shift):


O O R2 N R1 O N2 Rh2(OAc)4 N R1 R 2 N R2 O Me N N Et R1 Rh2(OAc)4 N N2 O O CO2Me N (95%) N Me N

O Et O

Et O

West, JACS 1993 1177 Padwa, JOC 1995 6258

CO2Me

Vindoline Skeleton

Me

CO2Me

27B-09 12/20/99 5:14 PM

Krista Beaver

Synthetic Applications of -Diazocarbonyl Derivatives Wolff Rearrangement Catalyst Glossary


Cu(TBS)2 Copper t-Butylsalicylaldimine R1 R1 H Rh2(OAc)4 Rhodium Acetate CH3CO2 CH3CONH Rh2(MEPY)4 O Rh2(MEOX)4 O O OH N t-Bu Rh2(MACIM)4 O Ac

Chem 206

N O H N2 R1 H O N H O N H CO2Me CO2Me

Arndt-Eistert Homologation:
OMe OMe O O 2N Me OMe N2 Ag , H2O
+

Rh2(acam)4 or Rhodium Acetamidate Rh2(acm)4 OMe OMe O 2N Me OMe Evans, JOC 1993 471 Rh2(oct)4 Rh2(pfb)4 Rhodium Octanoate Rhodium Perfluorobutyrate Rhodium Trifluoroacetate Rhodium Trifluoroacetamidate Rhodium Triphenylacetate Rh2(cap)4 COOH Rhodium Caprolactamate

O M N M

N H O

CO2Me

Ph CH3(CH2)6CO2 CF3CF2CF2CO2 CF3CO2 CF3CONH Ph3CCO2 Rh2(MPPIM)4 O

N N H CO2Me

Wolff Rearrangement - [2+2] Cycloaddition


TMS TMS R O N2 O H O O H SiO2 (60%) R R Me Me [2+2] Me O Me

Rh2(tfa)4 Rh2(tfm)4 Rh2(tpa)4

Rh2(S-TBSP)4

N Ts

CO2

O R TMS O O Rh O O R O O Rh O R O

Rh2(S-DOSP)4

CO2

Me O

All ligands on Rhodium are bridging through the carboxylate or the amide
(CH2)11CH3

Ireland, JACS 1981 2446; JOC 1984 1001

Aphidicolin
O O H

O OE

27B-10 12/20/99 5:22 PM

D. A. Evans

Ambiphilic Functional Groups2: Hydrazone-Based Transformations


Relevant Background Reading

Chem 206

http://www.courses.fas.harvard.edu/~chem206/

Chemistry 206 Advanced Organic Chemistry


Lecture Number 28

Hutchins, R. O. (1991). "Reduction of C=X to CH2 by Wolff-Kishner and Other Hydrazone Methods". Comprehensive Organic Synthesis. Trost and Fleming. Oxford, Pergamon Press. 8: 327. Shapiro, R. H. (1976). Alkenes from Tosylhydrazones. Org. React. (N.Y.) 23: 405. Addlington, R. M. and A. G. M. Barrett (1983). Recent Applications of the Shapiro Reaction. Acc. Chem. Res. 16: 55. Chamberlin, and Bloom (1990). Lithioalkenes from arylsulphonyl-hydrazones. Org. React. (N.Y.) 39: 1. Bergbreiter, and Momongan (1991). "Hydrazone Anions". Comprehensive Organic Synthesis. Trost and Fleming. Oxford, Pergamon Press. 2: 503.
Cume Question, November, 2000 Sorensen and coworkers recently reported the synthesis of ()-hispidospermidin (Sorensen JACS. 2000, 122, 9556). The Shapiro Reaction, along with methodology developed by Whitesell, was use in the construction of intermediate 3 from the indicated building blocks 1 and 2 (eq 1). R HN H O O H Me H Me Me Me O H Ph Me O Me O Me Me Me 1 2 ()-hispidospermidin
2,4,6-triisoproylbenzenesulfonyl hydrazine, HCl, CH3CN, 75% Intermediate n-BuLi (2.05 equiv)

Ambiphilic Functional Groups2 Hydrazone-Based Transformations


Wolff-Kischner Reduction Wharton Rearrangement Eschenmoser-Tanabe Fragmentation Reduction of Tosyl Hydrazones: "The Alkene Walk" Tosyl Hydrazone-Based Fragment Coupling The Shapiro Reaction Bamford-Stevens Reaction

Reading Assignment for this Week:


Shapiro Reaction: Chamberlin, and Bloom. Lithioalkenes from arylsulphonyl-hydrazones. Org. Reactions 1990, 39: 1. (handout) Wolff-Kishner & Related Reactions: Hutchins, (1991). "Reduction of C=X to CH2 by Wolff-Kishner and Other Hydrazone Methods". Comprehensive Organic Synthesis. Trost and Fleming. Oxford, Pergamon Press. 8: 327. (in library)

Intermediate B
55% MgBr2, -78 C then add 2

Me

O Me O Me Me Me 3
(eq 1)

D. A. Evans
28-00-Cover Page 11/19/03 2:02 PM

Friday, November 21, 2003

Me

HO

Ph

D. A. Evans, N. Finney

Hydrazone Transformations-1
O Me2N N CH2
+

Chem 206

Hydrazone Anions: A useful Reversed Polarity Equivalent


R () N N: R N R N: H R N:

O RT, ca. 24h R' CH2Cl2

Me2N

NO2 R' R

R (+) H

R () H

C(+)

C()

C()

1. O3; 2. DMS (40-92%) O R=alkyl or aryl R'=H or alkyl H R

A
NO2 R'

C(+)

Bu

N H

N H

Pr

n-BuLi, 0C THF

Bu

N H

Pr

Bu

N H

Pr Lassaletta, J-M, et al.Tet. Lett. 1992, 33, 3691.

Chiral Hydrazones as Carbonyl Anion Equivalents


J. E. Baldwin, et al. JCS Chem. Comm. 1983, 1040. PhCHO Ph Ph Ph OMe O 95% HO H Ph
i

Ph OBn

MgI2 CH2Cl2, 78

N H

Pr

Bu

1. n-BuLi 2. H+/H2O

LiO

Pr H Ph Ph

N H

N H

CO2Me CO2Me R

CO2Me CO2Me R H R X X

N Me

tBu NH H

tBu 1. n-BuLi, -78C Me O OCH3 Me N H N H

N H

O Me Ph CO2Me CO2Me

R = Me; 89:11 R = Ph; >98:2 O O I Mg I

H OCH3 Me O R

A A
O Me Me O OCH3

C() C(+)

H+, H2O

Lassaletta, J-M, et al. Chem Commun 2002, 498-499 OMe N H OCH3 H N H O dr = 90:10 D. Enders et al. Syn Lett 1999, 629-631 O O TBS N
TBSOTf

OMe N

tBu N Me Me O N

hydrolysis 58%

O CH2Cl2, 78

H O OTBS

J. E. Baldwin, et al. JCS Chem. Comm. 1984, 1095.

28-01 Hydrazones-1 11/24/03 2:23 PM

D. A. Evans, N. Finney

Hydrazone Transformations-2 The Wharton Rearrangement


Me R Me H HO H Me Me H R R O NNH R R
N 2H 4

Chem 206

Wolff-Kishner Reduction
O Me H CH3CO2 H Me Me H O Me R N 2H 4 , NaOCH2CH2OCH2CH2OH, (HOCH2CH2)2O reflux and then heat to 210C

R R

OH

Barton, D. H. R., Ives, D. A. J., and Thomas, B. R. J. Chem. Soc. 1955, 2056.

This example illustrates the 2 possible modes for the decomposition of A.

Mechanism
R 2C N NH2 + OH R 2C ROH R 2C H N N
N2

Me Me N NH R2C H N NH

O Me Me Me O

H N Me

H
:B

Me Me OH

NH Me

A
OH (RDS)*

C()

Me

N 2H 4 CH3OH,

Me

Me A

R 2C H ROH

R 2C H H RO Me Me WolffKishner NNH2 OH

C(+)
N2, H

C()

radical pathway

N2

polar pathway

Elimination of -Leaving Groups


BrCH2 Me Me O BrCH2 N2H4, H+ Me Me

Me Me OH

Me Me OH Me Me Me
20%

A
Me Me H N N H N2

C()

30%

Me Me

Me
G. Stork et al. JACS 1977, 99, 7067.

Br

Me

Some procedural improvements:


Me Me O O Me Me NH2NH2H2O KDA, KO Bu, etc.
t

Me Me OH Me Me 76%

-B

HBr Me CH2

For stable hydrazones, strongly basic conditions favor the ionic pathway. C. Dupuy, J. L. Luche Tetrahedron Lett. 1989, 44, 3437.

D. H. Gusyafson, W. F. Erman J. Org. Chem. 1965, 30, 1665.

28-02 Hydrazones-2 11/19/03 4:05 PM

D. A. Evans, N. Finney

Hydrazone Transformations-3
R C NNHTs + R O O BH R Ts

Chem 206

Tosylhydrazones Better Than Hydrazones


Tosylhydrazones are isolable, stable, and easily prepared. The presence of the tosyl leaving group strongly biases the system towards polar reaction pathways under hydridic reducing conditions.
H TsNHNH2 THF, N NHTs LAH (xs) THF,

NaOAc3H2O

R C N NH H B OR RO

R H

Ts R R C N NH H R R C H H

CHO

R C N NH
B OAc RO OR

H N

Ts Ts

H N N

G. W. Kabalka, et al. J. Org. Chem. 1975, 40, 1834.

Another Interesting Leaving Group


Ph H+/H2O CH3 N N Me L. Caglioti, M. Magi Tetrahedron 1963, 19, 1127.
H

Ph Ph
LAH

Ph N R
-stilbene

N R

N Me

Ph

N Ph Me

Further Refinements Very mild reduction with NaBH3CN under slightly acidic conditions (pH 4-5). No reduction in the absence of acid; carbonyl, nitro, nitrile FGs unaffected. Aromatic, sterically hindered carbonyls very poor substrates.
59% A. R. Chamberlin, et al. Tetrahedron Lett. 1991, 32, 1691.

Me

RH

Me

N2 R

N Me

NHTs

NaBH3CN, CH3CO2H

94%

R. O. Hutchins, et al. JACS 1973, 95, 3662.

28-03 Hydrazones-3 11/19/03 1:49 PM

D. A. Evans

Hydrazone Transformations-4
The EschenmoserTanabe Fragmentation

Chem 206

Tosylhydrazone Reductions: The Alkene Walk


Me Me
NaBH3CN

CH3
TsNHNH2, AcOH CH2Cl2, RT

CH3 H H O+ N N Ts TsNHN H Me CH3 N2 O N N Ts O


68%

Me
NaBD3CN

CH3

O S Ar

NNHTs

base

C. Djerassi, et al. JACS 1976, 98, 2275.

A. Eschenmoser, et al. Helv. Chem. Acta 1967, 50, 708.

O O CH3
+

Ph
HOAc

N N : N NH2
Et2O, 0C

Ph

H-

N N + H Ts

N Ts

O +

CH3 H

This has been developed into a reliable reduction


O Me 84% Me

N N + CH3 O H

Ph H 3C
+

CHO
94%

Me Me Me

O Me 81%

Me Me Me Me

Ph

CH2

A. Eschenmoser, et al. Helv. Chem. Acta 1967, 50, 2108.

16 cases reported: Hutchins, et al. JOC 1975, 40, 923

28-04 Hydrazones-4 11/19/03 1:50 PM

D. A. Evans, N. Finney

Hydrazone Transformations-5

Chem 206

Alkene Walk: Syntheses

Sulfonylhydrazone Reductions: Alcohol Deoxygenation


EtO2CN NCO2Et O RCH2 N S Ar ~0 C O RCH2 N NH In H RCH2 H 80% Bu

MeO

MeO Me 1. NH2NHTs, THF, 25C 2. Catecholborane 3. NaOAc3H2O O AcO

RCH2 Me H

OH Ph3P, 30 C SO2NHNH2 NO2

NH2 O

AcO [H]

(Mitsunobo Reaction) Org Rxns Volume 29

S O

Ar

RCH2 MeO H MeO Me OAc H H HSO2Ts N N Ts Compactin, Wendler, N. L., et al. Tet. Lett. 1982, 23, 5501. MeO OMe H AcO H N H N OH 86% H OH Bu

Me

AcO

Me MeO OMe

The stereochemical course of the hydrazone reduction may be stereospecifically transferred via the 1, 3-rearrangement

The intervention of radicals has been implicated (again):

OH CO2Et CO2Et O H (60%) Me N OH

86%

Me

1. NH2NHTs, THF, 25C 2. CB 3. NaOAc3H2O

O Me N

Topiramate, Maryanoff, B. E., et al. Tet. Lett. 1992, 33, 5009.


10 cases reported: A. Myers, et al. JACS 1997, 119, 8572.

28-05 Hydrazones-5 11/19/03 2:26 PM

D. A. Evans, N. Finney

Hydrazone Transformations-5

Chem 206

Tosylhydrazone-Based Fragment Coupling


O R H R' Li R R'

Stereoselective Construction of Trisubstituted Olefins


A. G. Myers, P. J. Kukkola JACS, 1990, 112, 8208. Me Me O 1) H2NNHTs 2) Et3N, TBSOTf 3) RLi 4) AcOH, F3CCH2OH O Me Me O O O H RLi H Ratio Z:E Yield Me H Li H Me Li Me O Me O Me O O TBS N N Li H Me Me Me O O O N H Et Me OMe Me H H Me Me CHO Li
(as above)

Me O

Me O Et Me (Z)

Ts N R N H TBS R' Li LiN R

Ts N R' TBS AcOH CF3CH2OH 78 rt R N NH R' A

Me Me

O O O

78 C

CHO

TBS = t-BuMe2Si NHSO2AR H Me LI

The monoalkyl azene A decomposes via a radical pathway N RO Me Ph 95% RO Me Me Ph Et

50:50

79% Me O O Me

O O

Me Me

(E) Et

Me

<5:95

81%

O H Me O O Me

16 cases reported: A. G. Myers etal. JACS, 1998, 120, 8891.

A Complex Application: A. Smith etal. JACS 1999, 121, 7423


Me I Ts N N Bu OMe t-BuLi (1.8 eq.) ether, -78 C (73%) RO Bu Me Me Bu HO HO Bu Me OH OH MeO Bu MeO OMe RO Me Me Me H Me Bu Me Me O

Me H N H Me

TBS MeO

MeO

OMe

SO2Ar

H Me

Me

Me

H Me

90%, ca. 2:1 Me H

Et

Me CHO Li

Me Me

Me

Cylindrocyclophane-F

Et H 82%, >20:1 CH3

(as above)

28-06 Hydrazones-6 11/19/03 1:52 PM

D. A. Evans
O C C (+)
HO Br

Hydrazone Transformations-7 -Keto Carbonium Ion Equivalents


base N R base HO Br O N R N R very electrophilic Nu: acceptor Stereoelectronic Issues H Br H base H H PhMgBr N NPh H Nu F3C Cycloadditions N OH Br Na2CO3 F3C N O

Chem 206

90% yield

Ressig, J. Org. Chem. 1992, 57, 339

Review: T. L. Gilchrist, Chem. Soc. Rev. 1983, 12, 53-72 Reactions with Nucleophiles

there is the expected strong chair-axial bias

N Ph

OH Br

O base N H HBr rate-det step

O Ph

HO Ph

H N N O

N NHPh

Ph H

R2NH

anti oxime is kinetic product

Stereoelectronic vs Steric Issues Me Br PhMgBr H H N NHPh H N PhHN Me H

N NHPh

T. Kaiser, JACS. 1972, 94, 9276-9277 OH Me OH H t-Bu OH Br Me OLi R = (CH2)3CH=CH2 2 equiv HO R

OH LiCuMe2 O

Ph

LiCuMe2

Bozzini, Tetrahedron 1982, 38, 1459 OLi

Corey, Tetradedron Lett. 1975, 3117

H NaSPh N NPh t-Bu PhHN

SPh H N

there is the expected strong chair-axial bias

N R

N Bozzini, Tetrahedron 1982, 39, 3413

Me O 92% yield

Corey, Helv. Chem. Acta. 1977, 60, 2964

28-07 Hydrazones-7 11/20/03 11:35 AM

D. A. Evans

The Shapiro Reaction-1


NHTs R

Chem 206

N R

1. Strong Base 2. Quench R

Deprotonation of the monoanion occurs predominantly at the kinetically more acidic site giving after elimination the less substituted alkene product.
R + N2 TrisHN Me N C5H11 BuLi THF BuLi TMEDA/hexane Li C5H11

General Reviews:
Trost, Ed., Comprehensive Organic Synthesis 1992, Vol 6, Chapters 4.3 . Trost, Ed., Comprehensive Organic Synthesis 1992, Vol 6, Chapters 4.6. Shapiro, Organic Reactions 1976, Vol 23, pp 405-507.

Li C5H11 + Me 4:1

Li C5H11

(E):(Z) = 4:1

Mechanism:
SO2Ar NHTs N nBuLi N N Li Li nBuLi N

SO2Ar N N Li TsLi N Li

In THF solution regiochemical ratios generally reflect the starting hydrazone geometries
Me H O Li Me H O O 1. TsNHNH2 Me 2. LDA 65 % Me H O Me H Me

Bond J. Org. Chem. 1978, 43, 154.

p. p.
O S NR O H O S NR O

nBuLi

O Grieco J. Org. Chem. 1977, 42, 1717. Me nBuLi, THF MeO H

Me Me Me Me Trisyl = Me Me MeO H NNHTs

Side Reaction

Li

80 %

the Triisopropylsulfonyl (Trisyl) group is used (Roberts Tet. Let. 1981, 22, 4895).

SO2 Me O Me + Me Me (2 %)

Nemoto et. al. JCS, Perkin Trans. 1 1985, 927. 1. TsNHNH2 2. K2CO3, Me Me O 1. TsNHNH2 2. LDA Me Me O O

Me O

Me 1. TsNHNH2 2. MeLi, Et2O (98 %)

via dianion

Regiochemistry

via trianion

O Me Me Me

1. TsNHNH2 2. MeLi, Et2O

Me Me (100 %) Me

O CO2Et

1. TsNHNH2 2. LDA

Me EtO2C (95 %) Me (5 %)

CO2Et

28-08-Shapiro Reaction-1 11/19/03 1:54 PM

D. A. Evans
Trapping of the intermediate alkenyllithium
CH2R C6H13 C6H13 O R R RCH2Br Li C6H13 Br+ Br C6H13 NHTris N Cl BusLi Li Cl 0C 82 % TMEDA/hexane -78C CO2

The Shapiro Reaction-2


Ts N CO2H C6H13 Ph Me O O Me2NCHO CHO C6H13 O O O + Li N N(TBS)Ts Et N TBS 1. nBuLi H 2. HOAc Ph Me Me H 81 % >20:1 (E):(Z) O O O H N Ts N TBS N H C 4H 9 H N

Chem 206

Ph C 4H 9 Me (E):(Z) = 12:1

R R

OH

-"TsTBS" C 4H 9 Ph H Me

O O Me Et

SiMe3 C6H13

Me3SiCl

Applications
O Me Me O O Me H H 1. TsNHNH2, TsOH 2. nBuLi, TMEDA 82 %

H Myers, J. Am. Chem. Soc. 1990, 112, 8208

Me Me O O H

p. p.

Aphidicolin

Bloom Tet Lett. 1984, 25, 4901

van Tamelen J. Am. Chem. Soc. 1983, 105, 142.

Carbonyl Transposition
NNHTs Me 1. 2.1 equiv. nBuLi TMEDA/THF 2. MeSSMe Me 3. 1 3quiv BuLi warm 2 eq. HgCl2 82 % Me SMe

Me

Me

Me BuLi NNHTris TMEDA/hexane HO RCHO 75 % Me Me

Me Li Me Me

HO O

Me

Nakai Chem. Lett. 1980, 1099

Shapiro alternatives
CO2Me O H H 2N N Ph Me OMe Me OMe H CO2Me N N 2.7 eq. LDA 0C THF Ph - styrene -N2 Me H HO CO2Me

Me

Me OH OH

Me

Me

Me

Me Me Me

Me

Me

OMe HO Me

(Juvabione) Evans J. Am. Chem. Soc. 1980, 102, 774

loroxanthin

Me Me Me Me Batikofer Helv. Chim. Acta. 1983, 66, 1148

28-09-Shapiro Reaction-2 11/19/03 1:55 PM

D. A. Evans
A Recent Aplication of the Shapiro Reaction

The Shapiro Reaction-Applications

Chem 206

Web Problem 24. Sorensen and coworkers recently reported the synthesis of ()-hispidospermidin (Sorensen JACS. 2000, 122, 9556). The Shapiro Reaction, along with methodology developed by Whitesell, was use in the construction of intermediate 3 from the indicated building blocks 1 and 2 (eq 1).

Part B (7 points). Provide a mechanism for the transformation of intermediate B to the illustrated product 3. Use 3-dimensional representations to illustrate the stereochemical aspects of this individual step.

X Li Me Me
MgBr2, -78 C then add 2

H Me 1

O Me Me O

O Ph 2

H Me O Me H Me Me

Me

HN

R H

MgBr Me Me

Back(Si) face of C=O attacked by Nu

X Mg O O

Ph Me 2

Me O Me

Me

O Me

Front (Re) face of C=O blocked by Aryl moiety

2,4,6-triisoproylbenzenesulfonyl hydrazine, HCl, CH3CN, 75% Intermediate

()-hispidospermidin
n-BuLi (2.05 equiv)

O Me HO Ph

Me O Me 3

Me
(eq 1)

Intermediate B
55% MgBr2, -78 C then add 2

Me

Me

p. p.

O Me HO Ph

Me O Me 3

Mattox-Kendall Dehydrohalogenation (Paquette, Reagents, Vol 5, p 3509) Me Me


(eq 1)

Me H2NNHCO2Et H3O+ N NH Me AOAc, heat

Me

Me

O
Part A (8 points). Provide a mechanism for the Shapiro Reaction of 1 to intermediate B in the space below. Feel free to use a simplified analog of 1 such as 2,2-dimethylcyclopentanone to answer this question.

Br

H EtO2C

Me H H O Br O Problem: The syn relationship between Br and H renders the direct dehydrohalogenation with base unfavorable (relative to other potential reactions. Solution; proceed via the hydrazone.

H O N Me Me SO2Ar H 2N N H Li N N Me Me Me Me
n-BuLi (2.05 equiv)

N SO2Ar

Li N N Li Me Me

O S Ar O

O S Ar N N Me Me EtO2C Li O N N H Br H EtO2C Me AOAc, heat N N H H Br EtO2C N NH tautomerization Me Me

28-10-Shapiro Reaction-3 11/19/03 2:13 PM

D. A. Evans

The Bamford-Stevens Reaction


Me Me
NaOMe, diglyme R N R' NHTs

Chem 206

An Alternate Decomposition Pathway for Tosyl Hydrazones

Me

Me CH2 Me

base

R
:

R' N NTs

R N+ N

R'

Me

ca. 180 C

N NHTs

Me
45% 55%

Me

Me
R
:

Me
CH3Li, Et2O, 0 C

Me

Me
quantitative

R'

products

Me
p. p. Bamford-Stevens vs. Shapiro

N NHTs

Me

Directed Bamford-Stevens
CH3 CH3 NNHTs NaOCH3, NMP, CH3
4% 63%

Hb R R' Ha Ha R Hb R' Ha R Hb R'


1,2 Ha 1,2 Hb

CH3
27%

R N

R' NNHX

CH3

trace

PhCH3, 145C

TMS
CH3 NNHTs CH3 2 eq. BuLi, 0 C
98%

Bn

Ph N TMS N Bn
PhCH3, 145C Rh2(OAc)4

66% (4:1 E:Z)

TMS

Bn

66% (14:86 E:Z) R. H. Shapiro Org. React. 1976, 23, 405. T. K. Sarkar, et al. JCS Chem. Comm. 1992, 1184.

28-11-Bamford-Stevens 11/19/03 1:58 PM

Chemistry 206 Advanced Organic Chemistry


Handout28A

Oxidative Coupling of Enolates & Enol Derivatives

OM R R OM
2 electrons

O R O R

Chuck Scales Evans Group Seminar, March, 1995

D. A. Evans
28A-00-Cover page 11/20/03 5:52 PM

Friday , November 21, 2003

Reaction Background The Oxidative Coupling of Enolates and Enol Derivatives


Evans Research Seminars
March 14, 1995 O

Synthesis of 3,4-diphenyl succinic acid


ONa

1) i-PrMgBr / Et2O 2) O2 3) H3O+

O OH OH + O O HO OH

[O]

38% 1) i-PrMgBr/Et2O 2) Br2/CCl4 3) H3O+


O O HO

12%

O ONa O

An underappreciated umpolung of enolates

OH

1. 2. 3. 4. 5.

Ketone Enolates Ester Enolates Carboxylic Acid Dianions Silyl Enolate Derivates Applications to Organic Synthesis

22% Also obtained a "high" yield of a-bromophenylacetic acid Proposed radical dimerization as mechanism for production of -diphenylsuccinic acid.
Ivanoff, Bull. Soc. Chem. Fr., 1935, 2, 76

Leading Papers: Saegusa, T. JACS, 1977, 99, 1487 Fox, M.A. JOC, 1988, 53, 3745 Narasaka, K. Chem. Letters, 1992, 2099

Synthesis of 1,4 Diketones

Chuck Scales

(AcO)2, 75 C

52%
O

Kharasch, M.S., McBay, H.C., and Urry, W.H., JACS, 1948, 70, 1269

28A-01.handouts 1/11/00 11:21 AM

Reaction Background
Coupling of Stabilized Anions

Synthesis of 1,4-Diketones

Methyl Ketone Dimerization


X R)n M

1) n-BuLi 2) Cu
II

X R)n M M X (R)n O R

1) LDA, THF, -78C 2) CuCl2, DMF, -78C


R

O R O

M = C, P, S X = O, BH3, S

R phenyl ferrocenyl furanyl cyclopropyl -C(CH2)2CO2Et


CH CH

Yield 95% 78% 41% 46% 64% 82%

Mislow, K. JACS, 1973, 95, 5839 Tamaru, Y. JACS, 1978, 100, 1923 A. Muci and K. Campos, unpublished results

Anion Coupling Models

Type 1 Oxidants

-Substituted Ketone Dimerization


OM R

-1e

O R

dimerization

O R O O R

LDA, THF CuCl2, DMF -78 C


O
2

73%

Type 2 Oxidants
OM OM R

E+
R

O E R R

O R O O

LDA, THF CuCl2, DMF -78 C


O
2

60%

Product mixture of exo-exo, exo-endo, and endo-endo isomers


Saegusa, JACS 1977, 99, 1487

28A-02.handouts 3/14/95 1:03 PM

Synthesis of 1,4-Diketones
Cross-Coupling of Methyl Ketones
O R

1,4-Diketones with Copper (II) Triflate


From enolates

3 eq acetone 4.5 eq LDA 4.5 eq CuCl2 THF/DMF, -78 C

O R

O Ph O

LDA, THF, -78 C Cu(OTf)2, i-PrCN


Ph

O Ph O

83%

Yield R
Et

Cross-couple 68%

Dimer 1%

O Ph

LDA, THF, -78 C Cu(OTf)2, i-PrCN


Ph

O Ph O

80%

53%

1%

LDA, THF, -78 C Cu(OTf)2, i-PrCN


Et

O Et O

63%

59%
O

4%

65%

2%

Anions Make a Difference


O O

CuCl2
Ph Ph O O

Ph

28%

Coupling of ,-unsaturated ketones


O O

LDA, THF/HMPA (4:1) CuCl2, DMF -78C


O O

33% , coupling product No , coupling product seen , couple not produced from thermal rearrangment of , product
O O O O

Yield difference may be due to lability of triflate versus chloride ions

Kobayashi, et al. TL, 18, 3741 (1977) Kobayashi, et al. Chem. Pharm. Bull., 28, 262 (1980)

32% , coupling product


Saegusa, T. et al. JACS 1977, 99, 1487

28A-03.handouts 3/14/95 1:05 PM

Coupling of Ketone Enolates


Oxidative Coupling with Ferric Chloride
O O

Intramolecular Coupling
Ketone cyclization
O O

1) LDA, THF, -78 C 52% 2) FeCl3, DMF, RT


O O Ph

1) LDA (2 eq), THF, -78 C


Ph

38%

2) Cu(OTF)2, i-PrCN
O

Dimers obtained from hindered enolates in moderate yields (40-60%) Prepared from kinetic and thermodynamic enolates
Frazier, R.H. et al. JOC, 1980, 45, 5408

Generally poor yields; spirocyclic examples included.

Kobayashi, et al. TL, 1978, 19, 3555

Oxidation with Manganic Acetate


O OAc

Mn(OAc)3 AcOH 70 C

Tetraketone Synthesis
O O

22%
O Ph

KOH (10 eq.) 80% aq. MeOH 0 C e- , 30 min.

Ph Ph O O

62%

Proposed radical coupling mechanism for transformation


Dessau, R.M. et al. JOC, 1974, 39, 3457

Synthesized various aryl substituted 3,4-aroyl-2,5-hexanediones Observed that EDG favor oxidation and EWG disfavor oxidation (correlated to Hammett plot)

Oxidative Dimerization of Aldehydes


O O O H + O H O

MnO2, reflux
H

48 hours

Hammett Plot Here


All examples produced ca 80% yield of dimer in 45:55 ratio (C-C:C-O).

Leffingwell, J.C. JCS Chem. Comm., 1970, 357 Lacan, M. et al. Croat. Chem. Acta, 1973, 45, 465

28A-04.handouts 3/14/95 1:07 PM

Ester Enolate Coupling

Oxidative Coupling Mechanism


Proposed Mechanism

Synthesis of Succinate Esters


R

OLi

SET
R

Radical

O R O R

Recombination

O RCH2 OR'

1) LiN(Cy)iPr, THF -78 C


R'O

R OR' R O

Radical may be associated with oxidizing agent

2) 1 eq CuX2, -78 C 15 min.

Evidence for Radical Mechanism


O O

1) LiN(Cy)i-Pr, THF, -78C 2) CuBr2


O

+
2

Br

Yield R -H -Me -n-Bu -Me2 -i-Pr -Ph R' -t-Bu -Et -Et -Et -Et -Et CuBr2 85 81 63 25 20 75 Cu(O2C5H9)2 95 50 ---20 20 60

85% Investigation of possible SN2 mechanism


OLi O O

"minor side product"

Br

THF
O

-78 C

O
2

10%

Exclusion of bromide source also leads to product Increasing alkyl substitution decreases yield of dimer Yield of a-bromoester increased with increasing alkyl substitution when copper (II) bromide used as oxidizing agent Yield of dimer not increased with copper (II) valerate Product obtained as an unspecified mixture of stereoisomers
OLi O O

Cu(O2C5H9)2 THF, -78 C


O
2

95%

Rathke, J. Am. Chem. Soc. 1971, 93, 4605

Rathke, J. Am. Chem. Soc. 1971, 93, 4605

28A-05.handouts 3/14/95 1:09 PM

Succinate Esters with Other Oxidants

Intramolecular Coupling

Oxidation with Iodine


O R1 R2 O R3 O R3 O R2 R1 O

Ester cyclization
COOR

1) LDA, THF, -78 C 2) I2 (0.5eq), warm to RT

R1

R2 O R3

(CH2)n COOR

LDA (2 eq) THF, -78 C CuCl2 Yield 88% (3:1 cis:trans)a

COOR (CH2)n-2 COOR

R1 H Me Me Me

R2 H H Me Me

R3 t-Bu t-Bu Me Et

Yield 97% 80% 90% 85%

n 3 4 5 6

R Me t-Bu t-Bu Me

20% (undetermined mixture of stereoisomers)b >50% (undetermined mixture of stereoisomers)b 93% (0.6:1 cis:trans)a

Authors propose an SN2 mechanism for this transformation


Brocksom, T.J., et al. Synthesis 1987, 396

Equilibration between cis and trans isomers noted for all reactions. Dimethyl adipate and dimethyl pimelate gave exclusively Dieckmann cyclization under the reaction conditions.
a b

Oxidation with TiCl4 Oxidation of dimethyl ,-dimethylglutarate


O OMe Me Me Me Me

Chung, S.K. et al. JOC, 1983, 48, 1125 Babler, J.H. et al. JOC, 1987, 52, 3462

1) LDA, THF, -78 C 2) TiCl4, CH2Cl2, -78 C

CO2Me

O OMe

80 %
CO2Me MeO

1) LDA (2 eq) THF, -78 C 2) [O], DMF Temp. -78 0 -78 -78 Yield 71% 83% 76% 81%

H MeO2C

CO2Me H

Excellent yields with -substituted esters (i-Pr, BnO) Other Lewis acids (BF3.OEt2, SnCl4) do not promote oxidation ZrCl4 resulted in Claisen condensation products Not applicable for ketones or amides
Ojima, et al. Chem. Letters 1992, 1591

Oxidant CuCl2 CuCl2 I2 AgCl

cis:trans 57:43 68:32 19:81 80:20

No attempt to rationalize stereochemical outcome of reaction


Babler, J.H. et al. Synth. Comm., 1983, 13, 905

28A-06.handouts 3/14/95 1:12 PM

Carboxylic Acid Dianion Coupling


Coupling of Acid Dianions
O Ph OH

Iodine Oxidation Mechanism


Mechanistic Investigations

1) n-BuLi (2 eq), THF, -78 C 2) I2 (O.5 eq), -65 C

O HO Ph

Ph OH O COOH

1) 2 eq LDA 2) 1.1 eq I2 THF, -78 C 2 min.


HOOC

COOH

+
I

COOH

90%, 11:1 dl:meso Yields increased with dianion salt isolation

16%

72%

Stereochemical Model
Li O O O O I L S
Belletire, J.L. et al. TL, 1984, 25, 5969

+
COOLi Li I COOLi

THF/HMPA -78 C 10 min.


HOOC

COOH

<10%

S L

"Dimerization reaction is much faster than nucleophilic substitution under the reaction conditions."

Coupling of ,-Unsaturated Carboxylic Acids


R' O OH R

1) 2 Et2NLi, THF -70 C 2) AgNO3 R H Me H Me R' H H Me Me Yield 49 63% 38% 61%

O HO R

R'

R OH R' O COOH

1) 2 LDA, THF, -78 C 2) [O]

Ph

COOH

+
Ph COOH

HOOC

E,E:E,Z:Z,Z 1:0:0 1:0:0 1:1:1 7:3:0


O

1 [O] eI2 1 16 23 2 40 38
COOH

O OH

1) 2 Et2NLi, THF -70 C 2) AgNO3


O

OH OH

Formation of ,para coupling product 2 supports radical anion intermediate 31%, E,E only


COOLi COOLi

No rational for observed stereochemistry Also observed unspecified yields of , coupled product
Mestres, R. TL, 1988, 29, 6181

Fox, M.A. et al. JACS 1988, 53, 3745

28A-07.handouts 3/14/95 1:14 PM

Iodine Oxidation Mechanism

Mechanism for Iodine Promoted Coupling

5-Hexenyl Radical Trap

R R'

OLi

COOH

1) 2 eq LDA 2) I2 (1 eq) +

I COOH

OLi

70%
R COOLi I R R'

COOH

COOLi

12%
COOH

R'

C
R R'

Ph COOH

1) 2 eq LDA 2) [O]

Ph O O Ph O

COOLi

R' R

COOLi

+ [O] I2 e1 26% 8% 2 32% 36%


Ph COOH

Proposed initial step is SET to form radical anion (D). Radical anion (D) may iodinate, then form dimer (B) via SN2 reaction

COOH

Radical anion may form dimer directly, especially if R and R' are large (>H) Direct iodination of dianion neither supported or excluded by experiments

No cyclopentylmethyl products seen!

Fox, M.A. et al. JACS 1988, 53, 3745

Fox, M.A. et al. JACS 1988, 53, 3745

28A-08.handouts 3/14/95 1:16 PM

Silyl Ketene Acetal Dimerization

Silyl Ketene Acetal Coupling


Carbocycle Synthesis
MeO OMe OTMS

Silyl Ketene Acetals


R
R R' OTMS OMe

R' Me i-Pr t-Bu Cy Ph Me Et

Yield
TMSO

TiCl4, CH2Cl2 RT

H MeO2C

CO2Me H

TiCl4 (1 eq) CH2Cl2, RT

R' R R R'

CO2Me CO2Me

69% Yield 100% trans

H H H H H Me Et

79%a 62%b 72%b 57%b 73%a 80%a, 93%b 67%b

Stereochemical course under thermodynamic control No coupling from ketone-derived enol silyl ethers Generally poor yields; exclusively 1,3-trans product.
Chan, T.H. Tetrahedron, 1983, 39, 847

No Claisen type products observed SET (TiIV --> TiIII) followed by radical coupling mechanism proposed by both authors Other reagents (CuII salts, FeCl3) ineffective for coupling reaction.
a

Acrylonitrile Trapping

Me Me

OMe OTMS

CN MeO2C

CN CO2Me CN

Ojima, I. et al. TL 1977, 18, 2009 b Rhodes, Y.E. et al. Synth. Comm. 1985, 15, 301

TiCl4, CH2Cl2 -78 C

63% Yield 1:1 diast. ratio

Vinyl Ketene Silyl Acetal Coupling


TMSO MeO R R'

OMe R R' CO2Me , coupling R' R (CH2)3 OTMS OMe OTMS

CN

NC

CO2Me

TiCl4 CH2Cl2 0 C

MeO2C

TiCl4, CH2Cl2 -78 C

52% Yield Mixture of all diastereomers


CO2Me

NC

+
R R'

R H H Me

R' H Me H

Yield 98% 76% 98%

, : , 83:17 64:36 96:4

Authors propose enoxyradical trapping by acrylonitrile, followed by dimerization , coupling Reactions with methacrylonitrile gave poor yields. Attempts to trap putative radical intermediate with FeCl3, CCl4, CBr4, and tributyltin hydride failed.
Rousseau, G. Tetrahedron, 1990, 46, 7011

MeO2C R' R CO2Me

Ojima, I. et al. TL 1983, 24, 785

28A-09.handouts 3/14/95 1:19 PM

Silyl Enol Ether Dimerization


Hypervalent Iodine Oxidants
O Ar

Silyl Enol Ether Dimerization


Metal Oxidants

1) Et3N, TMSCl 2) (PhIO)n-BF3Et2O CH2Cl2, -40 C, then RT


Ar

O Ar O

OTMS

Cu(OTf)2, Cu2O i-PrCN, 0 C


Ph

O Ph O

43-62% Oxidant

55%

Ph

# eq 1,4 1 0.5 2

Solvent i-PrCN DMSO 2:1 CH2Cl2:THF CH2Cl2

Temp.(C) 0 65 -78, then 23 -75, then -40

Time (hr) Yield 2 2 1.5;1 3;4 55% 73% 45% 30%

Ref. a b c d

Ar = Ph, p-HOC6H4, p-MeOC6H4, p-ClC6H4, p-MeC6H4


O t-Bu O t-Bu O

1) Et3N, TMSCl
t-Bu 2) (PhIO)n-BF3Et2O CH2Cl2, -40 C, then RT

Cu(OTf)2/Cu2O Ag2O Pb(OAc)4 VO(OEt)Cl2 57%

Proposed Mechanism
OTMS OTMS R
+ -

All authors also reported yields for non-styrenyl silyl enol ethers. Yields are extremely substrate dependent. Generally, increasing steric hindrance decreases yields. All authors propose oxidation to cation radical, followed by loss of trimethylsilyl cation and radical coupling.
a b

(PhI -OBF3 )
R

O I(Ph)OBF2 R R

O R O

Kobayashi, Y. et al. Chem. Pharm. Bull., 1980, 28, 262 Saegusa, T. et al. JACS 1975, 97, 649 Moriarty, R.M. et al. TL 1987, 28, 873 d Ohshiro, Y. et al. TL 1992, 33, 5823
c

Cross Coupling Experiments


NuH Nucleophile Trapping
O R Nu OTMS Ph OTMS

+ 1 eq
OTMS

VO(OEt)Cl2 (3 eq) CH2Cl2, -75 C, 3h, then -30 C, 4h.

O Ph O

H20, MeOH, EtOH

93%

2 eq
OTMS

Nucleophile trapping accomplished in "good yields."


Ph

+ 1 eq

VO(OEt)Cl2 (3 eq) CH2Cl2, -75 C, 3h, then -30 C, 4h.


O

Ph

77%

2 eq
Moriarty, R., et al. JCS Chem. Comm., 420 (1985) Moriarty, R., et al. J. Chem. Soc. Perk. Trans. I, 559 (1987) Caple, R., et al. JOC, 54, 2609 (1989)

Less reactive substrate added first, followed by more reactive substrate. In all cases, trace amounts of dimers isolated.
Ohshiro, Y. et al. ibid.

28A-10.handouts 3/14/95 1:21 PM

Silyl Enol Ether Coupling


Alkene Trapping
OTBDMS O

Application: Ketone Enolates


Cerorubenic Acid III: Construction of the Tetracyclic Core

H H

Cu(OTf)2, Cu2O CH3CN, 0 C


H

90%, 20:1


H COOH

H H 3C H CH
3

Cerorubenic Acid III Also observed for , olefins. Kinetic product can be isomerized in KOH/MeOH.
Snider, et al. JOC, 1990, 55, 4786

O O

1) LDA (2 eq), THF, -78 C 2) FeCl3, DMF, -78 C, 2 hr.

O O

H O CH3

6-Oxo-,-Unsaturated Carbonyl Compounds


CAN (2 eq) 0 C CH3CN
O H O

46-55% yield
OTMS

CH3

OTMS

77%

Other oxidants (CuCl2, Cu(OTf)2) coupled with poor yields


Paquette, L.A. JOC, 1993, 58, 4245

10 eq

Proposed Mechanism
+.
OTMS R R' O R OTMS O R

C16 Hexaquinacene
R' COCH3 COCH3

CAN

-TMS+

R' TMSO R''' O R''' R'' O R R''

1) LDA, THF -78 C 2) CuCl2, -78 C DMF:THF 7:1

OTMS R'

R R'' R'' O

+ R'' CAN -TMS+

R'''

58%

Paquette, L.A. et al. JACS, 1978, 100, 1600 Ruzziconi, R. et al. TL 1993, 34, 721

28A-11.handouts 3/14/95 1:24 PM

Ketone Enolate Applications II

Application: Carboxylic Acid Dianions


Total Synthesis of Racemic Wikstromol

Coupling of 2-acetylthiophene
O

1) LDA, THF, -78 C


S O

S S O

BnO

85%
MeO CO2H

1) LDA (2 eq) 2) I2 (0.5 eq) 3)Ac2O

BnO MeO

OBn OMe

2) CuCl2, DMF, -40 C

HCl, Ac2O
S O S

87%

A H2S, HCl
S S S

1) MeOH 2) BH3-THF 3) H+

BnO MeO

OBn OMe

61% from acid

73%
HO

HO
Kagan, J. et al Heterocycles, 1983, 20, 1941

OH OMe

MeO

-Coupling of Ketone Enolates


O O O

1) KH, THF, 20 C 90% 2) Br2, CH2Cl2, 0 C

Belletire, J.L. et al, JOC, 1988, 53, 4724

Anhydride "obtained as a single diastereomer by NMR analysis."

Total Synthesis of Racemic Hinokinin

1) CH3Li 2) TsOH, pyr., reflux 90%

BnO

61%
MeO CO2H

O O O O O

Mixture of erythro and threo acids; anhydride exclusively threo.


Hart, H. et al. TL, 1977, 18, 2307 Belletire, J.L. et al, JOC, 1987, 52, 2549

28A-12.handouts 3/14/95 1:27 PM

Applications: Ketone Enolates and Silyl Enol Ethers


Synthesis of Racemic Hirsutene
O OLi O

1)LiC(SPh)3 2) s-BuLi -78 C +


CLi(SPh)2

-78 C

OLi

OLi

O H

FeCl3, DMF -78 to -40, 30 m -40 to r.t., 10 h


PhS SPh

PhS H SPh

64%

Isolated as a single diastereomer; proof by conversion to hirsutene.


Cohen, T. JOC, 1992, 57, 1968

Studies Toward the Synthesis of Brackenin


HO OH O HO OH

OH O HO OH

HO

OTMS
2

Ag2O
MeO OMe OMe

MeO

O MeO

38% 1:1 dl:meso


OMe

"Use of Li enolates [for coupling] proved to be unsatisfactory."


Drewes, S.E. JCS Perk. Trans 1, 1989, 1585

28A-13.handouts 3/14/95 1:28 PM

Stereoselective Synthesis of Succinamides

Oxidative Coupling of Enamines


Silyl Enol Ether Trapping

Oxazolidine Auxiliary Experiments


O Et O

OTBDMS

1) LDA, THF, -78 C


O N

Nc O Et

N Nc t-Bu

+2

CAN (2 eq)
Ph

2) [O]

MeCN, rt 30 min

t-Bu O

Ph

63%

1
O

Oxidant I2 CuCl2

Yield 40-50% 50-55%

1: others 92:8 98:2

OTBDMS N CO2Me

2 TBCAN EtCN, -78 C K2CO3


N

+2

Ph CO2Me

91%

Ph

No model for induction proposed


Porter, N.A. et al. TL, 1993, 34, 4457

Proposed Mechanism
O O OTBS Ph t-Bu O

Oxazolidinone Auxiliary Experiments


O O N Bn O t-Bu

CAN
N

-et-Bu

Ph

1) LDA, THF, -78 C 2) Cu(OTf)2, MeCN


Xp

O Xp O

OTBS

50% yield 5:1 dl mixture


O

CAN -e Also obtained ca. 30% yield of meso dimer


t-Bu

+ + Ph
OTBS

-TBS+ H 3O
+

O t-Bu O Ph

J. Ellman and M. Dart, unpublished results Narasaka, K. et al. Chem. Letters 1992, 2099

28A-14.handouts 3/14/95 1:30 PM

Chemistry 206 Advanced Organic Chemistry


Handout28A

Oxidative Coupling of Enolates & Enol Derivatives

Chuck Scales Evans Group Seminar, March, 1995

D. A. Evans
28A-00-Cover page 11/20/03 3:14 PM

Friday , November 21, 2003

Chemistry 206 Advanced Organic Chemistry


Handout28B

Asymmetric Organocatalysis Using Chiral Amines

O Me

Me O

3 mol% L-proline DMF, 20h, rt


O

Me O

OH

93% ee

An Evans Group Friday Seminar Jonathan Lawrence November 14th 2003

D. A. Evans
00-00-Cover page 11/20/03 5:29 PM

Friday , November 21, 2003

Asymmetric Organocatalysis Using Chiral Amines


Contents: Background Aldol reactions Mannich reactions Amination/Oxidation reactions Michael reactions Cycloaddition reactions Alkylation reactions
Me Me CO2H N H O N Me Ph N H Me Ph N H Me S N H O N Me Me Me Me CO2H N H NR2

An Evans Group Friday Seminar Jonathan Lawrence November 14th 2003


Revent Reviews: List, B. "Proline Catalyzed Asymmetric Reactions", Tet. 2002, 58, 5573-5590 Miller, S. "Amino Acids and Peptides as Asymmetric Organocatalysts", Tet. 2002, 58, 2481-2495 List, B. "Asymmetric Aminocatalysis", Synlett 2001, 11, 1675-1686 Dalko, P. "Enantioselective Organocatalysis", ACIEE 2001, 40, 3726-3748

Other Chiral Amines


Cinchona alkaloids:

The "nucleophilic" catalysts

H H X N

N R R

H OH N H N

NMe2 H N R1 R N N R HH R2 N R

R = OMe, X = OH [(-)-quinine] R = H, X = OH [(-)-cinchonidine]

R = OMe [(+)-quinidine] R = H [(-)-cinchonine]

Reviews: Pracejus, H. Fortschr. Chem. Forsch, 1967, 8, 493. Morrison, J., Mosher, H. Asymmetric Organic Reactions; Prentice-Hall: Englewood Cliffs, 1971. Wynberg, H. Top. Stereochem. 1986, 16, 87. Relevant Group Seminars: Karl Scheidt, Asymmetric Catalysis with Chiral Lewis Bases (Part I), March 2001 Hemaka Rajapakse, Nonmetal-Based Asymmertic Catalysis (Part II), March 2001 Essa Hu, Asymmetric Catalysis with Chiral Lewis Bases (Part III), March 2001 Jake Janey, Asymmetric Catalysis with Chiral Lewis Bases (Part IV), March 2001

01-02 11/20/03 5:26 PM

Preliminary Findings
Yamada, 1969:
Me Ph CHO H CONR2

N H

N O

Me Ph

preformed enamine
O Me

MeOH/benzene 1:9

Me Ph O

AcOH, H2O

Me Ph

CHO Me O

49% ee
Yamada, S. TL 1969, 10, 4237.

The Seminal Experiments


O Me O R O R O

3 mol% L-proline DMF, 20h, rt


O OH

R = Me R = Et

100%, 93% ee 71%, 99% ee

O Me O

Me

3 mol% L-proline DMF, 72h, rt 52%


O

O Me

OH

74% ee the use of protic solvents severly diminishes enantioselectivity other amino acids as catalysts lead to decreased chemical yield and enantioselectivity Eder, Sauer, and Weichert obtained the corresponding aldol condensation products in similar optical purity using 47 mol% L-proline and 1N HClO4
Hajos, J., Parrish, D. JOC 1974, 39, 1615. Eder, U., Sauer, G., Weichert, R. ACIEE 1971, 10, 496.

03-04 11/20/03 3:24 PM

Effect of the Catalyst


CO2H N Me DMF

Me O

Me

Me Me O H HO O "major product" Me O

racemic

Me O

Me

CO2Me N H DMF O

racemic
OH O

"major product"
Me O Me O N H DMF CO2H

no reaction

Hajos, J., Parrish, D. JOC 1974, 39, 1615.

Transition States
Agami, 1984-1986 Houk, 2001-2003
H N O Me CO2O O H Me O

H
-

O 2C H

N O

NH

general hydrogen bond energies -OH - - O 3.0-8.0 kcal -CH - - O 0.5-3.8 kcal

favorable (enamine) N-H---O hydrogen bond N-H anti to carboxylate electrostatically favored reaction is second-order in proline (non-linear effect observed) second proline acts as a proton shuttle, allowing enamine to be nucleophilic
Agami, C. TL 1986, 13, 1501. Houk, K. JACS 2001, 123, 12911. Houk, K., List, B. JACS 2003, 125, 16.

N-H---O hydrogen bond does not lower energy of transition state favorable O-H---O hydrogen bond additional NC-H---O hydrogen bond further stabilizes system reaction is first order in proline (supported by kinetic data) and no non-linear effect observed
for a discussion on R3N+-C-H---O=C bonds, see: Houk, K. JACS, 2002, 124, 7163.

05-06 11/20/03 3:26 PM

Direct Aldol Addition 1


The initial reaction:
O O Me Me 20 vol%

30 mol% L-proline
Me NO2

OH

DMSO, 4hr, rt 68%

76% ee

NO2

Catalyst screen: (selected examples) compound (L)-His, (L)-Val (L)-Tyr, (L)-Phe


CO2H

% yield < 10

% ee -HO

compound
S N H CO2H HO N H CO2H Me S N H Me CO2H N H CO2H

% yield 67

% ee 73

N H

55

40

85

78

CO2H NH

< 10

--

> 50

- 62

List, B., Barbas, C. JACS, 2000, 122, 2395-2396 *Barbas, C. JACS, 2001, 123, 5260-5267

66

86

Direct Aldol Addition 2


Substrate scope: variation of the aldehyde
Me O OH R N H 1 CO2H

Me Me S CO2H N H 2

product R =

cat. 1 2 1 2 1 2

% yield 68 60 62 60 74 65

% ee 76 86 60 89 65 67

product R =
Cl

cat. 1 2 1 2 1 2

% yield 94 71 54 60 97 61

% ee 69 74 77 88 96 94

NO2

Br

DMTC 2 is catalyst of choice for aromatic aldehydes, although chemical yield decreases due to slower rate of reaction unbranched aldehydes yield no appreciable amount of product with proline catalyst 1 due to enolization and self-aldolization under reaction conditions (DMSO/acetone = 4:1)
List, B. JACS, 2000, 122, 2395. Barbas, C. JACS, 2001, 123, 5260.

07-08 11/20/03 3:28 PM

Direct Aldol Addition 3


Substrate scope: variation of the ketone donor
CO2H N H 1 O Me Me S CO2H N H 2

+
R1 Me H R2

20 mol % catalyst DMSO, rt, 24-48hr


R1

OH R2

product R =
O Me NO2 O Me O OH OH Me Me OH

cat.

% yield

% ee

1 2 1 2

65 57 60 <5

77 74 80 ---

1
NO2

Barbas, C. JACS, 2001, 123, 5260.

syn anti syn anti

24 35 21 41

67 90 69 89

Substrate scope: use of -unbranched aldehydes


O Me Me

Direct Aldol Addition 4


O H R

20 mol% L-proline CHCl3, rt, 3-7 d


Me

OH R

+
Me

O R

20 mol% product R=
Me

1 % ee 70 67 * 73 * 72 product R=
Me Me Me Me Me

2 % yield 1 % yield 2 23 34 22 46 42 50 % ee 61 73 * 36

% yield 1 % yield 2 29 31 35 34 0 38 40 35

Me

* reaction performed neat in acetone

use of cyclic ketones (cyclopentanone, cyclohexanone) result in moderate yield and diastereoselectivity, and up to 95% ee enone products arise from a Mannich addition-elimination sequence
List, B. OL 2001, 3, 573.

09-10 11/20/03 3:29 PM

Direct Aldol Reaction Mechanism


R2 O HO Me

HN

H O

+H2O -H2O
R2

N Me O

H O

N Me HO

H O

R1CHO
H O O H Me R1 H N

R2 HN HO H O

R1 OH O

Me

+H2O -H2O

R2 R1 OH N Me O H O

H H

R2

previously proposed T.S.: metal-free Zimmerman-Traxler model


H R1 R2 H N O H Me O H O

Houk's calculated T.S. synclinal approach of aldehyde R1 in pseudo-eqitorial position C-H - - O distance ~ 2.4 DFT calculations in DMSO

List, B. JACS 2000, 122, 2395. List, B., Houk, K. JACS 2003, 125, 2475.

Synthesis of Anti-1,2-Diols
CO2H O Me OH H O R

20 mol% catalyst DMSO/acetone 4:1 24-72 hr, rt % ee >99 95 >99 -->99 --67 (32) 92 (78)
Me Me

OH R OH

N H 1

Me Me S CO2H N H 2

product R = cat. 1 2
Me Me

dr >20:1 >20:1 >20:1 -->20:1 --3:2 3:2

% yield 60 45 62 <5 51 <5 95 60

product R =

cat. 1 2 1 2

dr 1:1 1:1 3:1 --1.7:1 --2:1 ---

% yield 83 52 62 <5 38 <5 40 <5

% ee 80 (n.d.) 95 (50 ) 79 (33) -->97 (84 ) -->97 (97 ) ---

1 2 1 2 1 2

Me

Cl

2:1

Me Me O

1 2 1 2

Me Me

List, B. JACS, 2000, 122, 2395. Barbas, C. JACS, 2001, 123, 5260.

more substituted enamine formed due to: increased acidity of proton removed increased stability of enamine due to On.b. --> * C=C

11-12 11/20/03 3:30 PM

Use of Aldehydes as Donors in Direct Aldol


O R1 H 2 equiv. H O R2

10 mol% L-proline DMF, 11-26 hr, 4C


H

OH R OH

product R =
O H O H O H Me O H Me OH Me OH OH Me Me OH Me Me

dr 4:1

% yield 80

% ee 99

product R =
O H Me O OH Me Bu O Me Me Bn Me OH Me Me

dr 24:1

% yield 82

% ee >99

3:1

88

97

H OH

24:1

80

98

14:1

87

99

19:1

75

91

3:1

81

99

reaction requires lower catalyst loading, shorter times, and only 2 equivalents of aldehyde donor

MacMillan, D. JACS, 2002, 124, 6798.

Trimerization of Acetaldehyde
3
Me O H

0.5 mol% L-proline THF, 0C, 5hr 10%


H

OH Me

90% ee

THF at 0C was found to be the optimal conditions for yield and ee (DMSO @ rt = 13% y, 57% ee, CHCl3 @ rt = 2% y, 68% ee) Mechanism:
O CO2H H Me

HN

-H2O +H2O

Me H

N CO2

N CO2H O Me O H

HN CO2H
Barbas, C. JOC, 2002, 67, 301.

O H

OH Me

Me

Me OH

N CO2

Mannich condensation

13-14 11/20/03 3:31 PM

Propionaldehyde Trimerization
A method for carbohydrate assembly
O H OH OH Me Me Me Me OH 6% Me O H Me Me OH OH Me Me OH Me HO O Me HO O Me

3
H

O Me

10 mol% L-proline DMF, 4C, 3d

1:2 :, 11% ee

47%

reaction analygous to an aldolase enzyme that furnishes the minor product shown above propionaldehyde added slowly dropwise in order to obtain trimer over dimer products enantioselectivity erodes with longer reaction times (after 10 hr product ee = 47%) substituent at C-6 variable by using 1 eq. of corresponding aldehyde and 2 eq. of propionaldehyde
Barbas, C. TL, 2002, 43, 9591.

Mechanism of Propionaldehyde Trimerization


O H Me H O Me

L-proline
H

OH Me Me

L-proline
H

OH Me Me

1
O

2
O Me

L-proline

L-proline

Me

O H

OH OH Me Me Me H

OH OH Me Me Me

incubating isolated 1 with L-proline led to formation of 2 through epimerization (1:1 ratio of 1:2 after 96 hr)
Barbas, C. TL 2002, 43, 9591.

15-16 11/20/03 3:34 PM

Aldehyde Aldol Addition to Activated Carbonyl Compounds


O H R EtO2C O CO2Et

20 mol% L-proline CH2Cl2, 1-3 hr, rt


H

OH CO2Et CO2Et

product R = Me Et i-Pr

% yield 90 91 88 94

% ee
H H N O H H CO2Et O O

90 85 85 88 84 0

EtO2C R

n-Hex Ph
Jorgensen, K. Chem. Comm. 2002, 620.

91 97

protection of the aldehyde as the dioxolane prevents epimerization of the center during column chromatography

Other Chiral Amine Catalysts for the Direct Aldol Addition


Chiral diamines:
N H 1 O Me Me H O R N N H N

2 catalyst 30C, 2hr


Me O OH R Me O R

3 product R = cat. 1.5 mol% 1 2TfOH + 1.5 mol% 1 3 mol% 2 1.5 mol% 1 2TfOH + 1.5 mol% 1 3 mol% 2 % yield 3 60 72 37 13 % ee % yield 4 88 93 83 91 7 7 32 25

NO2

proposed mechanism similar to that of proline catalyzed reactions, with proton transfer from protonated tertiary N to O
Yamamoto, Tet. 2002, 58, 8167.

17-18 11/20/03 3:35 PM

Mannich Reaction: First Report


Required Conditions: enamime addition must be faster to the imine than to the corresponding aldehyde formation of the aldimine from a primary amine must be faster than the aldol addition NMR studies show that Keq(aldehyde imine) = 1
O O OH R O O

R'NH2
R

NR' H R

NHR' R

kAldol

Keq=1

kMannich

3-component reaction verifies hypotheses:


CHO O Me Me NO2 NH2 OMe OMe

35 mol% L-proline acetone/DMSO 1:4 12 hr 50%


Me

HN

(+ <20% aldol product)


NO2

20 vol%

1 equiv.

1.1 equiv.

94% ee

10 mol% proline and 1.3 eq ketone used without loss of efficiency


List, B. JACS, 2000, 122, 9336.

Mannich Reaction: Scope


Variation of the ketone donor:
CHO O Me R NO2

35 mol% L-proline 1.1 eq. p-anisidine acetone/DMSO 1:4 12 hr, rt


Me

HN

PMP

NO2

R=

product
O HN PMP

% yield

dr

% ee

Me

2.5
Ar PMP

95

>20:1

99

Me Me O Me O HN HN

1
Ar PMP Ar

---

>20:1

94

OMe

Me OMe O HN

93

>20:1

98

PMP Ar

OH
List, B. JACS, 2000, 122, 9336.

Me OMe

92

>20:1

>99

19-20 11/20/03 3:36 PM

Mannich Reaction: Transition States

Mannich
R

O H X

CO2H Me

Aldol

MeO N N H R H

H O Me X

H O

O O

non-bonding interactions govern which diastereoface of electrophile is favored


R

N O H H

Me X

ArHN R X
List, B. JACS 2002, 124, 827.

O Me R

OH

O Me

Mannich Reaction: Scope 2


Variation of the aldehyde: aliphatic aldehydes, including -unbranched are good substrates (60-90% yield, 73-93% ee) aromatic aldehydes are excellent substrates, (79-92% yield, 61-99% ee) Effect of electron donation from the aldehyde:
O Me OH R NHPMP

R= CN H Me OMe

% yield 88 83 85 88

dr 15:1 9:1 5:1 3:1

% ee 99 93 86 61

Variation of the catalyst: proline proves to be the best catalyst, with other catalysts affording reduced yield and optical purity. Reaction of acetone with isovaleraldehyde:

List, B. JACS 2002, 124, 827.

CO2H N H 60% y., 16% ee

N H

26% y., 0% ee

21-22 11/20/03 3:37 PM

-Imino Ethyl Glyoxylate as Mannich Acceptor 1


Addition of ketones:
O R1 R2 PMP H O N OEt

An entry to -amino acids


20 mol % L-proline DMSO, 2hr, rt
R1 R2 O NHPMP CO2Et

20 vol% product
O Me O Me R2

% yield 86 R2 = Me allyl OH 72 79 62 47

dr ---

% ee 99

NHPMP CO2Et NHPMP CO2Et

>19:1 >19:1 >19:1 >19:1

>99 >99 99 >99

O Me O

NHPMP CO2Et Me NHPMP CO2Et

81

>19:1

>99

Barbas, C. JACS 2002, 124, 1842

-Imino Ethyl Glyoxylate as Mannich Acceptor 1


Addition of aldehydes:
O H R PMP H O N OEt

An entry to -amino acids


5 mol % L-proline dioxane, 2-24hr, rt
H R O NHPMP CO2Et

20 vol% R= Me Et i-Pr n-Bu n-Pent % yield 72 57 81 81 89 71 dr 1.1:1 1.5:1 >10:1 3:1 >19:1 >19:1 % ee 99 99 93 99 >99 >99

aqueous workup or column chromatography may lead to decreased diastereoselectivities reaction has been performed in aqueous media (Barbas, TL 2003, 44, 1923)
Barbas, C. JACS 2002, 124, 1866.

23-24 11/20/03 3:38 PM

Anti-Selective Mannich Reaction


Addition of aldehydes:
O H R PMP H O N OEt

20 mol%

N H

OMe H

NHPMP CO2Et R

DMSO, 24-48h, rt

20 vol% proposed transition state R= Et i-Pr n-Bu t-Bu n-Pent n-Hex % yield 44 52 54 57 78 68 dr 1:1 10:1 10:1 >10:1 >10:1 >19:1 % ee 75 82 74 92 76 76
For a review of SMP use in asymmetric synthesis, see: Enders, D. Synthesis 1996, 1403. Barbas, C. TL 2002, 43, 7749. N H H CO2Et MeO N H MeO N N H CO2Et H OMe

bettter?
OMe H H

Direct Amination 1
Addition of aldehydes:
O H R Cbz N N Cbz

10 mol% L-proline CH3CN, 0C-rt, 3hr then NaBH4, EtOH


HO R

Cbz N N H Cbz

1.5 equiv.

1 equiv.

R= Me n-Pr n-Bu i-Pr Bn

% yield 97 93 94 99 95

% ee >95 >95 97 96 >95


X
BnO2C

H N H Me CO2Bn O

N
N

longer reaction time leads to epimerization, so aldehyde is reduced in situ


List, B. JACS 2002, 124, 5656.

25-26 11/20/03 3:39 PM

Direct Amination 2
Addition of ketones:
O R1 R2 EtO2C N N CO2Et O R1 R2 CO2Et N N H CO2Et R2 O CO2Et N N H CO2Et

10 mol% L-proline CH3CN, 1-4d, rt

1
product 1
O Me Me O Me Bn O Me O Et Jorgensen, JACS 2002, 124, 6254. Me N N CO2Et N N H CO2Et

2
% ee

ratio 1:2 % yield (1+2)

10:1

80

95 (93)

CO2Et N H CO2Et

4.5:1

92

98 (94)

CO2Et N H CO2Et

3:1

99

99 (99)

i-Pr CO2Et N N H CO2Et

---

79

94 (93)

Oxidation of Aldehydes with Nitrosobenzene 1


The choice of reaction conditions determine N or O selelctive addition:
O OH N Ph

uncatalyzed X=Li, SnBu3, SiMe3

OX Ph

O N

Lewis acid X=SiMe3

O O N H Ph

Yamamoto, H. OL, 2002, 4, 3579.

Larger basicity of nitrogen allows proline to catalyze O-nucleophilic addition:


O H R Ph O N

5 mol% L-proline CHCl3, 4C, 4 h

O O R N H Ph

H O N N
O

O H

a possible transition state

MacMillan, D. JACS 2003, 125, 10808.

27-28 11/20/03 3:40 PM

Oxidation of Aldehydes with Nitrosobenzene 2


Aldehyde scope:
O H R Ph O N

5 mol% L-proline CHCl3, 4C, 4 h


H

O O R N H Ph

3 equiv.

1 equiv.

R= Me n-Bu i-Pr CH2CH=CH2

% yield 88 79 85 99

% ee 97 98 99 96

R= Bn Ph (CH2)3OTIPS CH2-(3'-Nmethyl indole)

% yield 95 60
76

% ee 97 99
98

83

98

product most easily isolated as the primary alcohol (NaBH4 reduction)


MacMillan, D. JACS 2003, 125, 10808.

Asymmetric Organocatalysis of the Michael Reaction


Two mechanistic possibilities exist:
N

EWG

or

:Nu

enamine

imminium

Examples include: additions to: alkylidene malonates ,unsaturated nitroalkenes additions of: malonate esters nitroalkanes aromatics (Friedel-Crafts reactions) silyloxy furans Diels-Alder reaction Dipolar cycloaddition

29-30 11/20/03 3:41 PM

Michael Additions using Enamine Catalysis:


Moderate Success has been Achieved
O S O Me H O

100 mol% L-proline DMF, -15C, 7d 81%


O

28% ee
Kozikowski, A. JOC, 1989, 54, 2275. O

O BnN Me CO2Et

100 mol% L-proline DMF, rt, 7d 45%


BnN

Me CO2Et

34% ee
Momose, T. J.Chem.Soc., Perkin Trans., 1992, 509.

Enamine Catalysis: Examples 2


Recent examples: List:
O Me Me Ph NO2

15 mol% L-proline DMSO, 24 hr, rt 97%


Me

NO2 Ph

7% ee
List, B. OL 2001, 3, 2423.

Enders:
O Me Me Ph NO2

20 mol% L-proline MeOH, 24 hr, rt 74%


Et

NO2 Ph Me

73% ee, dr=7.3:1 use of MeOH as solvent increases ee

H
O N O Enders, Synlett 2002, 26.

O O

N H

Me Ph
H

31-32 11/20/03 3:42 PM

Enamine Catalysis: Examples 3


Recent examples: Barbas:
O Me Me EtO Ph O O OEt

20 mol%

N H

N Me

EtO2C O

CO2Et Ph

THF, rt 59%

47% ee
Barbas, C. TL, 2001, 42, 4441.

O H R Ph NO2

20 mol%

N H

N O Et R O

NO2 Ph

THF, rt 85% proposed transition state:


R2N O2N H H N

R = Me 56% ee, dr=9:1 R = i-Pr 72% ee, dr=11:1

Ph

R
Barbas, C. OL 2001, 3, 3737.

A Highly Enantioselective Michael Addition Using Enamines


A New Chiral Diamine Catalyst
syn/anti
O NO2 Ph OR

O Me OR Ph NO2

15 mol%

N H Me

R Me H

yield 75 79

dr 1:5 5:1

ee 69 98

Me

10 equiv.

CHCl3, rt, 7d

HH N Me H O Me N Ph H NO2 Me

Alexakis, A. OL, 2003, 5, 2559.

with variation of aromatic group on nitroolefin: ee = 96-98% dr = 3.5:1 - 19:1 selection of aromatic groups used: tolyl, p-methoxyphenyl, p-chlorophenyl, 2-thienyl

33-34 11/20/03 3:43 PM

Imminium Catalysis of Conjugate Additions 1


Proline has been used with only mild success:
O O i-PrO O Oi-Pr CO2Rb O

5 mol%

N H CO2i-Pr CO2i-Pr

CHCl3, rt 91% 58% ee

Proline rubidium salt gives lower ee in the Hajos-Parrish-Weichert reaction

CO2Rb O Me Me NO2

5 mol%

N H Me

Me NO2 Me Me

CHCl3, rt 74%

68% ee
Yamaguchi, JOC 1996, 61, 3520.

MacMillan Introduces A New Catalyst


Imminium ion formation lowers the LUMO of the system and allows catalysis to occur:
LUMO

versus

LUMO

E
HOMO HOMO

L.A.

Consensation of an aldehyde with the catalyst produces an imminium complex:


O N Me Ph N H Me O N Me N Ph O H Me H Me Me

00 0000 00 00

:Nu PM3 minimized structure

35-36 11/20/03 3:44 PM

Diels-Alder Cycloaddition 1
H R O

5 mol% 1
R CHO R

MeOH-H2O, rt 12-24 hr

CHO

endo

exo

Dienophile scope: R= Me n-Pr i-Pr Ph Furyl


MacMillan, D. JACS 2000, 122, 4243.

% yield 75 92 81 99 89

endo:exo 1:1 1:1 1:1 1.3:1 1:1

% ee(endo) % ee(exo) 86 86 84 93 91 90 90 93 93 93
Ph N H O N Me Me HCl Me

Diels-Alder Cycloaddition 2
R H R O X

20 mol% 1 MeOH-H2O, rt 12-24 hr

CHO X

endo adduct

Diene scope: diene


Ph O Ph

R Me

product
O Ph CHO Ph Me

% yield 75

exo:endo % ee 35:1 96

H
Me Me

82
CHO

1:14 ------1:5 1:11

94
O N Me Me Ph N H Me HCl

H H Me
Me Ph CHO R CHO Me Me

84 90 75 75
CHO OAc

89 83 90 90 85

Ph

Me

H H

OAc

72
CHO

MacMillan, D. JACS 2000, 122, 4243.

37-38 11/20/03 3:45 PM

Application to Complex Synthesis


Me O N CHO Me Ph N Ts Me N H Me HCl N Ts Me Me CHO

Me

40 mol% DMF/MeOH (1:1) 5% H2O, 36 h 35% yield, % de (endo) 70


Me Me SCN Me NTs

exo (92% ee)

Me NTs CHO

endo (93% ee)

Kerr, M. JACS 2003, ASAP

(+)-Hapalindole Q

Nitrone Cycloaddition
Z Z N O R R1 H O Z N O R CHO R1 R CHO N O R1

20 mol% catalyst MeNO2-H2O -20C

endo Z R R1 Me Me Me Me Me Me Me Me Me H H H H H H endo:exo 94:6 93:7 95:5 92:8 93:7 98:2 93:7 95:5 99:1 81:19 86:14 85:15 80:20 81:19 91:9 yield 98 73 66
78 76 93 82 98 70 72 80

exo

ee (endo) 94 98 99
95 94 91 97 93 99 90 92

O N

Me Me

Bn Ph allyl Ph Me Ph Bn C6H4Cl-4 Me C6H4Cl-4 Bn C6H4OMe-4 Me C6H4Me-4 Bn 2-napth Bn c-Hex Bn Ph Bn Ph Bn C6H4Cl-4 Bn C6H4Cl-4 Bn 2-napth Bn C6H4OMe-4

Ph

N H

Me HClO4

catalyst

80 80 82 83

90 91 90 90

HClO4 proved to be the best Bronsted acid cocatalyst to promote only enantioselective catalysis high endo selectivity attributed to to favorable placement of R group away from geminal dimethyl substituents on catalyst

MacMillan, D. JACS 2000, 122, 9874

38b-39 11/20/03 3:46 PM

Friedel-Crafts Alkylation 1: Pyrroles


H N Me R O

20 mol% catalyst THF-H2O, 3-5 d ee 91 90 91 93 91


87 90

N Me

O R H O N Me Ph N H Me TFA Me

temp(C) -60 -50 -50 -30 -30 -60 -50

yield 83 81 80 87 79
90 72

Me n-Pr i-Pr Ph PMP CH2OBn CO2Me

catalyst use of N-benzyl pyrrole and N-allyl pyrrole give similar results

Substitution on the pyrrole is also possible:


H Bu N Me Ph O

20 mol% catalyst THF-H2O, 3-5 d 20 mol% catalyst


O Bu N Me Pr O Ph H O Ph H

87% y, 90% ee

Pr Ph N Me MacMillan, D. JACS 2001, 123, 4371.

THF-H2O, 3-5 d

N Me

68% y, 97% ee

Alkylation of Indoles 1
The need for a new amine catalyst: 20 mol%
H N Me Me O Ph N H O N Me Me TFA N Me Me Me H O

CH2Cl2, -40C, 2d 83%

58% ee

Indole is less electron-rich than pyrrole, so is less nucleophilic toward conjugate addition Second generation catalyst: a more reactive variant lone pair exposed CH3 - lone pair interaction
O N Me Me O N Me Ph N H Me Me Me Me

0000 0 00 0 00 00 00 00
MacMillan, D. JACS 2002, 124, 1172.

Ph

N H

00000 00 0 0000 0

Kinetic studies indicate rate of reaction influenced by imminium formation as well as carbon-carbon bond forming event

40-41 11/20/03 3:47 PM

Alkylation of Indoles 2
R H N Me O N Ph N H TFA R O H O N Me

20 mol% catalyst THF-H2O, 3-5 d

Me Me Me Me

An increase in rate of reaction and enantioselectivity:

catalyst R temp(C) yield 82 80 74 84


84 89

ee 92 93 93 90
96 91

000 0 000 00 000

O N Ph N

Me Me Me Me

Me -83 n-Pr -60 i-Pr -50 Ph -55 CH2OBz -83 CO2Me -83

:Nu increased top-face coverage nucleophile-geminal dimethyl interation removed

MacMillan, D. JACS 2002, 124, 1172.

Alkylation of Indoles 3
H

H O

20 mol% catalyst
O

N R

R1

CH2Cl2-i-PrOH, 3-24 hr

N R

Indole Scope: R Me H allyl Bn H Me H Y H H H H H H Cl Z R1 temp(C) -87 -60 -72 -60 -60 -87 -60 % yield 82 72 70 80 94 90 73 % ee 92 91 92 89 94 96 97
Me MeO Me N CO2H Me N

H Me H Me H Me H Me Me -CH2OBz OMe -CH2OBz H -CH2OBz

Application to Simple Synthesis:


MeO

1. 20 mol% catalyst
H Me O

87% ee, 82% y. COX-2 inhibitor

2. AgNO3, NaOH
Br MacMillan, D. JACS 2002, 124, 1172. Br

42-43 11/20/03 3:48 PM

Alkylation of Benzenes
Z H R2N X O Z X H O R2N OMe OMe N Me2N N

10 mol% catalyst CH2Cl2, 2-4 d

1 aniline 1 2 1 1 2 1 2 3 3 1 2 X Me Me Et CH2OBz CH2OBz CO2Me CO2Me Ph p-ClPh p-NO2-Ph p-NO2-Ph temp(C) -40 -20 -50 -20 +20 -20 -20 -50 -10 -10 -20 % yield 86 70 68 73
90 89

2 % ee 89 87 88
92 92

O N Ph N H

Me Me Me Me

HCl

catalyst other substituted anilines used with similar results catalyst loading can be lowered to 1% without significant loss of yield and enantioselectivity

90 97 84 92

97 82 87

87 82

92 90

MacMillan, D. JACS 2002, 124, 7894.

Mukaiyama-Michael Reaction 1
Previous Michael additions with silyloxy furans:
O TMSO O Me N O O O

Chiral Lewis Acid (bisoxazoline or pyridyl bisoxazoline ligands)

O H N Me O O O

Katsuki, Tet. 1997, 53, 17015 Desimoni, G. Tet. 2001, 57, 10203

note that Lewis acids promote 1,2-addition products when possible, such as ,unsaturated enals Optimized reaction conditions:
TMSO O Me R H O O

20 mol% catalyst CH2Cl2, 2 equiv H2O % yield 81 87 80 77


86 84

O Me O R H O N Me Me Me N H Me DNBA

R Me n-Pr i-Pr Ph CH2OBz CO2Me

temp (C) -70 -50 -20 -40 -70 -60

syn:anti 22:1 31:1 7:1 1:6 20:1 11:1

% ee 92 84 98 99
90 99

Ph

catalyst
MacMillan, D. JACS 2003, 125, 1192.

44-45 11/20/03 3:49 PM

Mukaiyama-Michael Reaction 2
Variation of the silyloxy furan
O TMSO X O R Me H O

20 mol% catalyst CH2Cl2, 2 equiv H2O


X

O R O Me H

R H Me Et

X H H H

% yield 87 80 83
86 83 73

syn:anti 8:1 22:1 16:1


6:1 1:7 24:1

% ee 90 92 90
98 98 90

CO2Me H CO2Me H Me Me

TFA as cocatalyst TfOH as cocatalyst

MacMillan, D. JACS 2003, 125, 1192.

Another Chiral Amine Catalyst


Asymmetric Michael Additions
O R1 R2 BnO2C CO2Bn

10 mol%
Bn N H

NMe CO2H BnO2C R1 CO2Bn O R2

neat, rt

R1

R2

% yield
84 58 75 95

% ee
89 77 92 88

NMe N Me CO2H

p-NO2Ph Me p-NMe2Ph Me Me 2-furyl -pyridyl Me 2 n-Bu Me i-Pr Me Me MeO2C Ph Me Ph Et Ph i-Pr

61 33 59 86 66 2

91 84 59 99 95 94

:Nu

Nitroalkane additions to ,unsaturated ketones has also been performed in good to excellent selectivity (Jorgensen, K. JOC 2002, 67, 8331.)
Jorgensen, K. ACIEE 2003, 42, 661.

46-47 11/20/03 3:50 PM

A Listing of Other Asymmetric Organocatalytic Reactions


[4+3] cycloadditions:
OTMS CHO Me O Me O Me N N H TFA Me Me Me Me O CHO O Me

20 mol%
Ph

CH2Cl2, -78C, 4d 64% Michael Reactions:


Ph OH O R O O Me H N CO2H N H

endo only (89% ee)


Harmata, M. JACS, 2003, 125, 2058.

10 mol%
Ph

OH

O Me

CH2Cl2, rt

75 - 88% ee Tandem Knoevenegel-Diels-Alder Reactions:


O Ph O2N H O Me Me O O O Jorgensen, K. ACIEE 2003, 42, 4955 Me S N H Me Me CO2H p-NO2Ph O O O O Ph Me

O Me

MeOH, rt, 4 d 88%

86% ee
Barbas, C. ACIEE 2003, 42, 4233

Reactions are direct: Donors can be used without modification -- no need to deprotonate or silylate prior to reaction Electrophiles can be generated in situ (Mannich reaction) most of the time Catalysts are: inexpensive commerially available or easily prepared in both enantiomeric forms non-toxic recoverable Many reactions can be run at room temperature, under an aerobic atmosphere, with wet solvents Many types of reactions can be catalyzed; for some reactions, organocatalysis is the only highly efficient way known (Mannich and Mukaiyama-Michael additions) Reaction yield and enantioselectivity is highly dependent on solvent system so require "fine tuning" Only reactions that use ketones or aldehydes as donors (electrophiles for Michael additions) can be catalyzed Organocatalysis using small molecules is a field that has emerged only within the past decade. It is bound to receive increasing attention in the future; as a result, new catalysts will emerge which will allow for the catalysis of reactions previously unutilized in the realm of organocatalysis.

Summary

48-49 11/20/03 3:51 PM

D. A. Evans

Ambiphilic Functional Groups3: Sulfur-Based Activating Groups Relevant Background Reading

Chem 206

http://www.courses.fas.harvard.edu/~chem206/

General: Simpkins, N.S. Sulphones in Organic Synthesis, Pergamon Press, New York, 1993.

Chemistry 206 Advanced Organic Chemistry


Lecture Number 29

General: Magnus, P.D. Tetrahedron 1977, 33, 2019. Julia: Blakemore, J. Chem. Soc. Perkin Trans I. 2002, 2563. Electrophilic Properties: Trost, B.M. Bull.Chem. Soc. Jpn. 1988, 61, 107. SO2 Extrusion: Vogtle, F.; Rossa, L. ACIEE 1979, 18, 515. Ramberg-Bcklund Rxn: Paquette, L.A. Org. Reactions 1977, 25, 1. Triflones: Hendrickson, J.B. Org. Prep. Proc. Int. 1977, 175. Sulfoximides: Johnson, C.R. Tetrahedron 1984, 40, 1225

Ambiphilic Functional Groups3 Sulfur-Based Activating Groups


Sulfur-Ylides Sulfur-Stabilized Carbanions: Structure Sulfone-Based Transformations Pummerer Rearrangement

Cum Question, 1998: The stereoselective construction of trans olefins through carbanion-mediated condensation processes has still not been rendered general. One transformation that may be used in certain circumstances is the "one-step" Julia transformation illustrated below. PPProvide a mechanism for this transformation.

O S O
1

RS H

LiN(iPr)2 RACHO THF 78 25 C

RS
+

RA
+

S O N H

Reading Assignment for this Week: Carey & Sundberg: Part A; Chapter 7 Carbanions & Other Nucleophilic Carbon Species Carey & Sundberg: Part B; Chapter 2 Reactions of Carbon Nucleophiles with Carbonyl Compounds
"Chemical Chameleons: Organosulfones as Synthetic Building Blocks" B. M. Trost, Bull. chem. Soc. Japan, 1988, 61, 107-124 (handout) Monday, November 24, 2003

RS RA

SO2

The cruel mechanistic problems that you should be prepared for in Chem 206 O O MeO MeO N O CO2Me O S Et Ac2O/HOAc 70% MeO MeO Padwa et al. JOC 1996, 61, 4888 N CO2Me

D. A. Evans
29-00-Cover Page 11/24/03 8:32 AM

D. A. Evans

Sulfur-Based Functional Groups-1 Relevant Background Reading


General: Simpkins, N.S. Sulphones in Organic Synthesis, Pergamon Press, New York, 1993. Magnus, P.D. Tetrahedron 1977, 33, 2019. Blakemore, J. Chem. Soc. Perkin Trans I. 2002, 2563. Trost, B.M. Bull.Chem. Soc. Jpn. 1988, 61, 107. Vogtle, F.; Rossa, L. ACIEE 1979, 18, 515. Paquette, L.A. Org. Reactions 1977, 25, 1. Hendrickson, J.B. Org. Prep. Proc. Int. 1977, 175. Johnson, C.R. Tetrahedron 1984, 40, 1225 HO R O O P O O O NH2 N N N O Me S NH2 N S N2 HO NH2 O Me S + O

Chem 206

Reactions of Sulfonium Ylids


Synthesis:
R S R

Me

S N2

General: Julia: Electrophilic Properties: SO2 Extrusion: Ramberg-Bcklund Rxn: Triflones: Sulfoximides:

(+)

R + S CH3 I R Sulfonium Salt: pKa ~ 18 NH2 N N N N

R2S

()
OH OH R

OH OH S-Adenosylmethionine R S CH2 R O CH3 S CH2 CH3 H H pKa ~ 38

R2S
S CH3 R pKa ~ 18 O

Acidities of Sulfur-based Functional Groups


Bordwell, F. G.; Zhang, X.-M. Acc. Chem. Res. 1993, 2 6, 510-17.

Deprotonation:

Na H

pKa (DMSO) CH4 pKa (~56) NH3 pKa (~41) Sulfoxide ~35
CH3

S CH3 CH3

Na H

CH3

S CH3 O

Sulfide

(45)

Leaving Group Potential:


L

R2S

C(+)
L L S: L

CH3

S CH3 O

S CH3 + L (+)

Nu:

S N2

+ Me Nu

CH3 CH3

S CH3 O S CH3

Sulfone

~ 31

HOH pKa 31

Sulfonium Salt

18.2 Excellent LG

CH3 O CH3 S CH3 CH3

R S CH3 R

Nu:

S N2

R S R

Me Nu

Oxo-Sulfonium Salt ()
O

R2S

C
(+)

Reactivity Pattern Nonalternate

Good LG

S CH3 O

Nu:

S N2

O R S O

+ Me Nu

29-01 Sulfur chem-1 11/23/03 6:07 PM

D. A. Evans, K. Scheidt

X-ray Structures of Metallated Sulfones & Sulfoxides


O S Me CLi

Chem 206

Sulfone- & Sulfoxide Based Carbanions: Structure


Sulfone- and sulfoxide-stabilized carbanions are extremely useful carbon
nucleophiles in organic synthesis.
O Ar S O O Ar S CH3 LDA Ar CH3 LDA Ar O S O O S CH2Li El(+) CH2Li El(+) O Ar S O O Ar S CH2 El CH2 El + TMEDA

Boche, etal. Angew. Chem. Int. Ed. 1986, 25, 1101

Li

However, until recently little information was available on the solid state structures of these species:
"The Structure of Lithium Coumpounds of Sulfones, Sulfoximides, Sulfoxides, Thioethers, 1,3 Dithianes, Nitriles, Nitro Compounds, and Hydrazones." Boche, G. Angew. Chem., Int. Ed. Engl. 1 9 8 9, 2 8, 277.

Li

Here are several examples taken from the Boche review:


O S O CH2Li[TMEDA]

Li

Gais, etal. Angew. Chem. Int. Ed. 1985, 24, 859

The Li counterions are not associated with the charged carbon. The carbanions are largely trigonal.

29-02 Sulfone anion-Xrays 11/23/03 5:16 PM

D. A. Evans, K. Scheidt

X-ray Structures of Phenylsulfinyl Carbanions

Chem 206

O S

Me CLi

O S O CHLi

+ TMEDA

+ TMEDA

Li

Li

Li Li

Li

Boche, etal. Angew. Chem. Int. Ed. 1986, 25, 1101


29-02a Sulfone anions-Xrays-2 11/23/03 5:16 PM

Boche, etal. Angew. Chem. Int. Ed. 1985, 24, 573

D. A. Evans
Reactions with ketones: ()

Sulfur-Based Functional Groups-2


R2S
H S C Me H

Chem 206

Reactivity Pattern: Nonalternate Synthesis:


O CH2 Me Me

Reactions of Sulfones
PhS: Br Me H2O2 Me S Ph HIO4

Me

Me S

Me

Me Me S O S O Ph O H2O2 Ph

Corey & Chaycovski, JACS 1965, 87, 1353-1364. (Handout) O Me S C Me H O CH2 Me S Me Me

()

R2S

C
Me O Br :S Ph O

Me

Me Me O

O S Ph

O CH2 Me S Me

O CH2

(+)

R2S

Me

(Sulfinate anion)

Sulfinate ester not observed

"Twenty-five Years of Dimethylsulfoxonium Methylide (Corey's Reagent).", Gololobov, Y. G.; Lysenko, V. P.; Boldeskul, I. E. Tetrahedron 1987, 43, 2609. (electronic handout)

Good review article: Magnus, Tetrahedron 1977, 33, 2019-2045.

Reactions of OxoSulfonium Ylids: Conjugate Addition


Me Me Me O S CH2
(+)

Reactions of Sulfones
Me O S

Me

S Me

Me O Me Me O

B ()

(+)

pKa ~ 25 ()

more nucleophilic than:

R2S R2S C
Me O S Me Nonalternate reactivity pattern revealed in consecutive reactions O CH2 O

C
O

R2SO2

Ph O

poorer leaving group than:

R + S CH3 R

(+)

R2SO2
Me Me O Li S Ph O
(+)

O
(+)

??

1,2- vs 1,4-addition ?? Will function as LG ??

29-03 Sulfur chem-2 11/23/03 6:32 PM

Ph

Me BuLi

Li Ph

R + S CH2 R

D. A. Evans, P. Carter
Me Me O Li S Ph O O
(+)

Sulfur-Based Functional Groups-3


(+)

Chem 206

Me Me

SO2Ph O

The Sulfone group may also be readily removed reductively:


R SO2Ph BuLi El(+) R El SO2Ph Na(Hg) MeOH R El H

1,2-addition ()

R2SO2

C
Me Me

PhSO2 MEMO O

Fragment Coupling with Sulfonyl Carbanions


OH Heathcock, C.H.; et al. J.Org.Chem. 1988, 53, 1922.

Alkoxide not sufficiently nucleophilic to displace PhSO2 anion. However!!


Me Me O Li S Ph O Me Me O OEt

Not observed
Me Me SO2Ph Me Me OEt

Me
1. MsCl, TEA 2. PhSLi, THF, RT 3. mCPBA

Me

SO2Ph

MEMO Me Me X MEMO Me Me

2 eq. nBuLi THF/HMPA; then add iodide

OTBS I OTBS
65 - 85% yield

Me

Me

Industrial synthesis developed by M. Julia Me H

OLi CO2Et H Me

()

(+)

R2SO2

R2SO2

Me Me

Me X= SO2Ph X=H H Me S S

Me 50% yield

trans chrysanthemic acid Synthesis of Vitamin A:


Me Me Me

Li wire, Na2HPO4 THF/HMPA/tBuOH

Julia & Co-workers, Bull. Soc. Chim. Fr. 1985, 130


Me Me Me CO2R Me Me SO2Ph (+) H KOH/MeOH TBDPSO Me CO2R MeO PhO2S Smith, A.B. III; et al. Tet.Lett. 1989, 30, 6963. O

SO2Ph Li

TBDPSO MeO HO PMBO 77% yield S SO2Ph Me S

PMBO

Me

()

R2SO2

Br

TBDPSO Me Me Me Me CO2H Me MeO H O PMBO S Me S

TBDPSO
Al-Hg, aq. THF Reflux MeO

TFAA, DMSO; NEt3 CH2Cl2, -78 oC

(+)

SO2Ph Me O PMBO S S

R2SO2 C
29-04 Sulfur chem-3 11/23/03 6:03 PM

90% yield

D. A. Evans, T. Dunn
Functionalization of cyclic Ethers
BuLi PhO2S O O SO2Ph El(+)

Sulfur-Based Functional Groups-4


Total synthesis of Okadaic Acid
Me El O SO2Ph SO2Ph Lewis acid PhO2S O n-BuLi, DMPU THF, -78o C PhO2S H OH O

Chem 206

Me I

R2SO2

() C
ROH

Me H O OH H O

R2SO2

(+) C

El

O OR

El

O H Me H O

OH

1.) Addition of iodide, -78o C to RT 2.) CSA, MeOH Me HO Me

Me

O H COOH Me OH O

Me

O O 90% yield

Ley et al, Synlett, 1992, 395; Ley et. al, Tetrahedron, 1992, 48, 7899

Total Synthesis of Routiennocin (CP-61,405)


Me OBn n-BuLi, DMPU PhO2S O H THF, -78oC PhO2S O H R Me I O O C Ring 1.) Addition of iodide, -78o C RT 2.) H+, H2O O H O N HO HOOC HO Me 68% yield n-Pr Me H O O H OBn MeO2C H

Ley et al, J. Chem. Soc., Perkin Trans. 1, 1998, 3907.

Total synthesis of Bryostatin 2


OTBDMS Me Me O OTf H PhO2S O H OPMB O NiPh Li OTBDMS

87% yield

Me N H O Me

OTBDMS Me Me HO O O Me OH C Ring O Me Me O O OH H OH O Me CO2Me O OH H O H

Me

OPMB

O NHPh

O OH H

OTBDMS

Routiennocin

Evans et al, JACS. 1999, 121, 7540-7552.

Bryostatin 2

29-05 Sulfur chem-4 11/23/03 6:06 PM

D. A. Evans, P. Carter

Sulfur-Based Functional Groups-5: Julia Olefin Synthesis


Julia Olefination - Ionomycin
PhO2S Li OTBDPS Me OAc R R' SO2Ph Na(Hg) MeOH R' R R' R Me Me CHO O Me 1. add RCHO, -78 oC; add Ac2O, -78 oC R.T. 2. Na/Hg, EtOAc/MeOH, 30oC

Chem 206

First Generation Julia Trans Olefin Synthesis:


OH R SO2Ph BuLi R'CHO R R' SO2Ph Problem: Work out the mechanism of reduction step. Ac2O

Elimination is stepwise; therefore, not stereospecific

70% yield 86:14 olefin mixture


Me Me Me O H O Me Me O Me H O Me Me OTBS

major

H O

Me H

Me

OTBS

Good sulfone review: Trost, Bull Chem. Soc. Japan, 1988, 61, 107-124. Julia Review, Blakemore, J. Chem. Soc. Perkin Trans I. 2002, 2563. (electronic handout)

Me Me Evans, et al. JACS 1990, 112, 5290.

OTBDPS Me Me

The reduction step is not stereosecific

Cytovaricin Synthesis:
Me H H

JACS 1990, 112, 7001

R2 X R1 SO2Ar SO2R

+ e-

R2 X R1

+ eR1

R2
+

Me R2 R1 TBSO DEIPSO Me

O H O H

OCH2OCH2CCl3
21

TESO

H CHO

HO + O Me TESO PhSO2 6% Na(Hg) -40 C O

Me

Me t-Bu O Si t-Bu O H O Me OTES H MeO Me O OTES H H

OPMB 2 LiNEt2, THF Ac2O, pyr Me OAc Me DEIPSO Me H H H O H H O H OPMB

OAc

Na(Hg) SO2Ar

Reactions accomplished: C=C construction C21 OH deprotection


Free acid must be used to prevent loss of C4 OH in 2nd step

OH HO
21

OTES
3

Me TESO

Me Me O

OAc

Kochenski, J. Chem. Soc Perkin Trans I, 1978, 834

TBSO

29-06 Julia-1 11/23/03 6:14 PM

t-Bu Si O overall yield, 66% t-Bu Me H H MeO O O Me OTES OTES H H

D. A. Evans, P. Carter

Sulfur-Based Functional Groups-5: Julia Olefin Synthesis-2


HO O N

Chem 206

Phorboxazole B Synthesis
Br
46

The Mechanism:
13

O H
H

H H

19

O
4

S CH2 N O S A O S ONa O I R CH3 N S O R CH3 O I O N R

S O I S O CH3

MeO Me
H

H
Me

O H O OH
33

MeO Me H

disconnection

38

N O

HO

MeO

Evans, Smith, Fitch, Cee JACS 2000, 122, 10033-10046.

Olefin stereochemistry could be established in the formation of A.


NaO N S R

OMe Br Julia Construction CH3 I

I CH3 SO2

C39C46 Synthon

Br

OMeO O S S

OMe N NaHMDS O H I Br CH3 E/Z = >95:5 75% I Ph N N C

Recent Modifications of the Julia Process:


Kocienski, SynLett 2000, 3, 365-366. O2 S C 4H 9 KHMDS 60 Crt OHC R N N C O2 S Ph C4H9

N N

E/Z: 99:1 (75%)

NaHMDS RCHO

THF, -78C - rt CH3

KHMDS 60 Crt OHC C9H19

Ph

OMe ONa Br S N S O CH3 O I Br

OMe I CH3 SO2 NaO S N Ph N N C

N N

C9H19

R = Ph: E/Z: 29:71 (70%) R = tBu: E/Z: <1:99 (95%)

Mechanism??

O2 S R H

KHMDS O 60 Crt Me R' RO Me R R'

N N

E/Z: >10:1 (64%)

RO Metternich, JOC 1999, 54, 9632

29-07 Julia-2 11/23/03 6:15 PM

D. A. Evans, N. Finney

Sulfur-Based Functional Groups-5

Chem 206

Carbonyl Anions: A useful Reversed Polarity Equivalent


Consider the two possible polar disconnections of the CR2 bond of the ketone shown below:
T1 O R1 C R2 T2 R1 R1 O C O C + R 2: + R 2:

The overall set of reactions which establishes the equivalency of the hypothetical carbonyl anion 1 and its equivalent synthon 2 is shown below:

S S

Li R

Construction Step

S S

El R

El

1 Equivalent Synthons
O R1 C:

() CG
H 3O +

(+) CE
El

carbonyl anion

Carbonyl anions are not normally accessible via aldehyde deprotonation


O R1 C Cl
Mg0

O R1 C MgCl not feasible !! Why??

T2
El O C

Operational equivalents to the carbonyl anion are useful in synthesis

O R1 C:

Nitronate Anions are also useful Carbonyl Anions


pKa 18
O +N O R H R
() HO

O +N O

R
()

1,3-Dithianes as Carbonyl Anion Equivalents


SH
RCHO

nitronate anion highly stablized


Construction Step

H2O +H2O

S S

H R

RLi

S S

Li R O +N O H R El O2N C R

SH

pKa ~ 39 Reactivity Patterns:

El

() CG

(RS)2 C(+)

(RS)2 C()
Equivalent Synthons
O

Latest Innovations: A. B. Smith, JACS 2003, 125, 14435-14445 (Handout) O S R3Si S H R1 S R3Si LiO R1 S HMPA S Li R3SiO R1 S El(+) S El R3SiO R1 S

Nef Reaction

(+) CE
El O C R

C:

El

T2

tBuLi

Dithianes anions highly nucleophilic (indiscriminate): Nitonate anions higly discriminating

29-08 carbonyl anions 11/24/03 8:44 AM

D. A. Evans

Charge Affinity Inversion Operators Nature's Inversion Operators

Chem 30

Introduction. As you know, transform T1 conforms to the polar bias mapped on to the carbon skeleton by =O, while transform T2 does not. Although T1 is the more common transform, sometimes, because of the presence other functionality in either R1 or R2, the "reversed-polarity" transform is more suitable for the particular synthesis at hand. T1
O R1 C R2 R1 O C+ O C: + R 2: R 2: +

There is a clear need in nature to have both types of polar bond constructions exemplified by Transforms T1 and T2 (Eq 1-2). One such reaction is shown below. This reaction, which is enzyme-catalyzed, requires the cofactor thiamine which functions as the inversion operator in these biological processes.
O

Eq 1 Eq 2

T2
R1

-Ketol Transferases: + thiamine (Q:)

OH H R1

O R1 O O OH R2 O: OH H 2C R2 OH C R2

H 2C OH

Reversed Polarity Synthon

Ideally, one might visualize a catalytic agent (Q) which might react reversibly with an aldehyde in conjunction with inverting its charge affinity pattern. Nature has designed such reagents.

This unit is transferred H

H2C OH

General Scheme
Charge Affininty Inversion Step: The structural constraints on (Q) are that it must be nucleophilic, add reversibly to aldehydes, and stabilize an adjacent carbanionionic center.
O R1 C H O OH H R1 C OH

R2 O

Crucial bond construction:

H2C

C:

+ Q:

R1

OH Q Equivalent synthons

+
O H 2C R2 OH C

Q:

Lets call (Q) a charge affininty inversion operator since in operates on RCHO and reverses the intrinsic polar reactivity of the RCHO carbon from (+) to ().

Overall Process:
R1

O C H

OH

Related transform:

OH R2

H2C OH

C:

+ Q:

R1

The Inversion Operator


O R1 C

The Thiamine Coenzyme (Virtamin B1) & how it functions El(+)


H2N H N N Me O

acidic proton
R S O P O O O P O HOCH2 OH OH HOCH2 S R R N C O N

OH

El

+ Q:

S R R N

S R

R1

El
Me Me Me

thiamine (Q:)
O C R1 H O: C N O R1 S R OH

Benzoin Condensation:
O C

OH Ar C

H C OH H 2C OH C: HOCH2 R N

C O Ar

Q H :CN

:CN

OH Q

HOCH2 R

Cyanide ion is the best example of a reagent which functions as an inversion operator. The benzoin reaction is restricted to aromatic aldehydes. Why?

Equivalent synthons

S R

H R1

29-09 Inversion Operators 11/24/03 8:50 AM

Me

Me

Me

D. A. Evans
Basic Transformation:
O Ar S R H+ Ar OH S H R H2O Ar S H R

The Pummerer Rearrangement

Chem 206

Transformations Mediated by the Pummer Rearrangement


+H2O ArSH O H Ar R S O TsOH O Ar O Ar O S 86% S

The Pummerer Rearrangement facilitates the transformation of a sulfinyl aldehyde transformation. The rearrangement may be initiated by either a Bronsted acid or an anhydride such as trifluoroacetic anhydride (TFAA). With the latter reagent, the transformation occurs at room temperature. O Ar S R OTFA TFAA Ar S H TFAOH S R Ar H
R +TFAO

O Ph S N

O Ph TFAA H N R Et N R H S N

Ar

R OTFA

Et

Leading References
De Lucchi, Miotti, et al. (1991). The Pummerer reaction of sulfinyl compounds. Organic Reactions 1991, 40: 157. Grierson, and Husson (1991). Polonovski- and Pummerer-type Reactions and the Nef Reaction. Comprehensive Organic Synthesis. Trost and Fleming. Oxford, Pergamon Press. 6: 909. Padwa, A., D. E. Gunn, et al. Application of the Pummerer reaction toward the synthesis of complex carbocycles and heterocycles. Synthesis 1997 1353-1377. Carreno, Applications of sulfoxides to asymmetric synthesis of biologically active compounds. Chem. Reviews 1995 95, 1717-1760. Kita, Y. and N. Shibata (1996). Asymmetric pummerer-type reactions induced by O-silylated ketene acetals. Synlett(4): 289-296. RHN

81% N H N

P. Magnus et al. Accts Chem Res. 1984, 17, 35 PhS

Et

O S Ac2O/HOAc N O CO2Bn >80% O CO2Bn RHN N S

OAc

The Related Polonovski Reaction:


O R N R R H+ OH R N R H R H2O R R N H R +H2O R2NH O H R MeO MeO

The cruel mechanistic problems that you should be prepared for in Chem 206 O O N O S Et Ac2O/HOAc 70% MeO MeO Padwa et al. JOC 1996, 61, 4888 N O CO2Me

Regioselectivfity: Depends on the relative kinetic acidy of the protons

29-10-Pummerer 11/20/03 3:06 PM

CO2Me

D. A. Evans

The Pummerer Rearrangement


O O S CO2Me Et Ac2O/HOAc 70% O CO2Me MeO MeO Padwa et al. JOC 1996, 61, 4888 Me S O N O N Me O Et

Chem 206

Mechanism??
MeO MeO N O

Exam 3, 2000: Question 5 (11 points). An interesting rearrangement which also results in the construction of this same ring system (Question 4) has been reported by Langlois & coworkers (J.Org. Chem. 1985, 50, 961). This rearrangement is illustrated below. Provide a mechanism for this transformation. N N
TsOH, THF 78 %

H N Me SMe

Et O

Pummerer Ac O S Et H O N MeO OMe SEt O N MeO OMe O SEt O N COOCH3 MeO N OMe OMe MeO O O COOCH3 Product O CO2Me N
OAc OAc

S Pummerer N

Et

Pummerer

O Me O OMe CO2Me H S O

H N Me O Et

Pummerer S Me

H N Me O Et

CO2Me

MeO

N H SEt O Product N Me H H SMe Et O Me H N S Me O Et N

Endo [4+2]

Endo [4+2]
MeO OMe

CO2Me

29-11-Pummerer-2 11/20/03 3:06 PM

Chemistry 206 Advanced Organic Chemistry


Handout 29A

Overview of the JuliaKocienski Olefination

Evans Group Seminar by Scott Peterson, September 26, 2003

S R1 S

N O O

Base

S O S

N O Li O H R1 R2

+
H O R2

R1

H R2

D. A. Evans

Friday, November 21, 2003

H29A-00 11/20/03 2:51 PM

Examples of Direct Olefination from Carbonyl Compounds


X
+

H H R2 O R1

H R2 H

R1

X
R3P+ B.E Maryanoff, A.B. Reitz, Chem. Rev., 1989, 89, 863 L.F. van Staden, D Gravstock, D.J. Ager,Chem. Soc. Rev., 2002, 31, 195 P.R. Blakemore, J. Chem. Soc., Perkin Trans. 1, 2002, 2563

Reaction Wittig Horner-Wittig Horner-Wadsworth-Emmons Peterson

R2P(=O)
(RO)2P(=O) R3Si

ArS(=O)(=NMe) Johnson ArSO2


HetSO2 classical Julia modified Julia

Classical Julia Olefination


O SO2Ph n-BuLi R1
-78 oC THF

SO2Ph R1 Li

R2 R1

SO2Ph R2 OAc

single electron donor Na/Hg SmI2

H R1 H R2

2) Ac2O

C5H11

C5H11
E:Z = 80:20

C5H11

E:Z = 90:10

E:Z = >99:1

H29A-01 11/24/03 9:03 AM

Mechanism of Olefin Formation


Ph O S O R1 R2 OAc M Ph O S OM R2 R1 OAc

H R1

R2 H OAc

M
-Originally proposed mechanism for Na/Hg elimination, though Keck has shown this is not the case -Believed to be mechanism for SmI2 elimination

H R1 H R2 H R1

R2 H OAc

G.E. Keck et al., J. Org. Chem., 1995, 60, 3194

Mechanism of Olefin Formation


Na/Hg + MeOH

PhSO2 R1

H R2 OAc

OMe -NaOAc

Ph O S O R1 H R2

Na/Hg 1e-PhSO2Na

R1 H

R2

Na/Hg 1e-Using MeOD results in >90% Deuterium incorpration -After initial elimination, there is no equilibration, explaining why Na/Hg and SmI2 can give different results

H R1 H R2

Na
quench

R1 H

R2

G.E. Keck et al., J. Org. Chem., 1995, 60, 3194


H29A-02 11/24/03 9:05 AM

Synthesis of Bryostatin 2
OTBS TBSO O H O SO2Ph Me Me O H TBSO 1) n-BuLi, THF, -78 C 2) Ac2O, DMAP 3) Mg, 20mol% HgCl2 Single Olefin Product (E) 64% over 3 steps OTBS OPMB Me Me O H OTBS OPMB
o

OTBS

Me Me

D.A.Evans, P.H. Carter, et al., J. Am. Chem. Soc., 1999, 121, 7540

TBSO Me Me PhSO2 O OTf TBSO O TBSO O NHPh OPMB n-BuLi (2eq) THF, HMPA Me Me O

HO O

Me

Me

OPMB

NHPh O H OTBS O OTBS Me OPMB

Me Me

OTBS OPMB Me Me HO OH O O O

Me

MeO2C

D.A.Evans, P.H. Carter, et al., J. Am. Chem. Soc., 1999, 121, 7540

Me Me O

OH H OH O Me O CO2Me

O OH

n-Pr
H29A-03 11/24/03 9:08 AM

Modified Julia Olefination


N S SO2R N SO2R N N N N SO2R N N N N SO2R

BT

PYR

PT

TBT

Bt =

(benzotriazole)

R N N N

Modified Julia Olefination - Smiles Rearrangement

S R2CHO O2S

N OLi R2

S O2S R1

N O

Li

O S O R1 Li

R1

R2

N OH S
H29A-04 11/24/03 9:10 AM

SO2

R2 R1

O LiO S R1

R2 OBT

Diastereoselectivity of BT-Sulfones

OLi R1 R2 SO2BT anti

O Li S O O R1 H R2 H

slow R1 R2

S O N SO2 H Li R1 BTO

SO2Li R2

R2 R1 Li BT + S O2 R1 H O R2 R1 Z E R2

OLi R1 R2 SO2BT syn

N fast R1 H

S O N SO2 R2 Li R1 BTO H R2 SO2Li H

O Li O S O R1 R2 HH

J.B. Baudin, Bull. Soc. Chim. Fr., 1993, 130, 856

H29A-05 11/24/03 9:12 AM

Effects of Solvent and Counterion with BT-Sulfone


CHO BTSO2
(Me3Si)2N-M -78 oC to RT

M Li Na K

Toluene

Et2O 49:51 50:50 51:49

THF 66:34 62:38 54:46

DME

50:50 54:46 54:46

70:30 75:25 76:24

Solvent Screen in U-106305 Synthesis


Me O NH

solvent Me SO2-BT NaHMDS -78 C to RT OHC OTIPS


o

E:Z 9:91 9:91 11:89 52:48 71:29 78:22

toluene Me OTIPS CH2Cl2 Et2O THF DME

Me + OHC

SO2-BT

DMF

OTIPS

NaHMDS, THF-DMF -60 C to RT


o

92% 81:19

A.B. Charette, et al., J. Am. Chem. Soc., 1996, 118, 10327


H29A-06 11/24/03 9:15 AM

Retroaddition - Addition with BT-Sulfone


O BT S O
n-C8H17CHO LDA, THF -78 C to RT E:Z = 23:77
o

C8H17

OLi C8H17 Ph
slow

Ph

C8H17 SO2BT
anti

RCHO

Ph BTSO2

RCHO Ph

OLi C8H17 SO2BT


syn fast

C8H17 Ph

CHO OH Ph BTSO2 O2N


LDA, THF -78 oC to RT

NO2 Ph
40% E:Z = 98:2

Ph
60% E:Z = 92:8

P.R. Blakemore, Ph.D. Thesis, University of Glasgow, Glasgow, 1999

Aromatic Aldehydes with BT-Sulfones


S OLi R1 Ar SO2BT R1 Ar O H N SO2 H Li -BTOLi

-SO2

Ar C H H R1

R1 Ar
cis

OLi R1 Ar SO2BT R1 H O H

S N SO2 Ar Li -BTOLi O S O -SO2 R1


trans

Ar

H C Ar H R1

CHO R R BTO2S
LDA, THF -78 oC to RT J.B. Baudin, et al. Bull. Soc. Chim. Fr., 1993, 130, 856

yield

E:Z

OMe 95% 99:1 H 68% 94:6 Cl 51% 77:23

H29A-07 11/24/03 9:16 AM

Reversibility in Rapamycin Synthesis


P. Kocienski, et al., Synthesis, 1996, 285 Me S S O H OMe CHO Me MN(SiMe3)2, THF -78 oC to RT OMe OTBS Me Me Me M Li OMe CHO Me BTSO2 Me Me OTBS Na yield 68% 21% E:Z 95:5 78:22 M Li Na K yield 75% 79% -E:Z 29:71 43:57 18:82 BTSO2 Me Me OTBS

Me

S O H

Me

S O H

MN(SiMe3)2, THF -78 oC to RT

Possible Explanation for Diastereoselctivity

O S S H O O H R2 N

SO2BT H O O Li R1 OLi R2 R1 R2

or
Li

BT R1 H

R1

R2

Chelate (closed) Transition State favored for non-polar solvents, small counter-ions (Li) Non-chelate (opened) Transition State favored for polar solvents, large counter-ions (K)

LiO H R1

BT S

O R2 H

H Li O R2 R1

SO2BT R2 OLi R1 R2

BT R1 H

S O

S. Peterson, Meandering Thoughts, 2003


H29A-08 11/24/03 9:21 AM

Ipso Substitution with BT-Sulfones

N S

O S CH3 O

LDA THF, -78 C 3 hr 52%


o

S N O O S O N

S N S O S O N SO2Me

J.B.Baudin, et al., Bull. Soc. Chim. Fr., 1993, 130, 856

Synthesis of ent-Bengamide E
TBSO OHC TBSO OTBS O OCH3 OCH3 LiHMDS DME -78 oC to RT 62% TBSO OCH3 OCH3 TBSO OTBS O

N S

O S O

OH

OCH3

H N

O NH

OH K.J.McRae, PhD Thesis, Research School of Chemistry, Canberra, 2001 J.B.Baudin, et al., Bull. Soc. Chim. Fr., 1993, 130, 856

OH

O ent-Bengamide E

H29A-09 11/24/03 9:23 AM

Synthesis of LAF389
O S N + OHC O CH3 H3C O CH3 O OCH3 1) LiHMDS, -78 oC, THF CH3 2) TMSCl H3C 3) aldehyde 4) -78 oC: 1 hour 0 oC to RT: 1 hour 50 oC: 1 hour 45% single isomer, white crystalline solid CH3 H3C CH3 OH OCH3 O NH O L. Waykole, et al., Organic Process Research and Development, 2003, ASAP (Novartis Process Group) O LAF389 H3C O H3C O CH3 O O OCH3

O S O H3C H3C

H N

OH

OH

Synthesis of LAF389
S N + H O

O S O H3C H3C CH3

1) LiHMDS, -78 oC 2) TMSCl 3) aldehyde 4) -78 oC to 50 oC

TMS N S

O O S O Ph

t-Bu Ph

H3C H3C CH3 Ph

Ph TMS TBAF E:Z = 1:1 CH3 Ph Ph N S O O S O Ph L. Waykole, et al., Organic Process Research and Development, 2003, ASAP (Novartis Process Group) t-Bu Ph

MeSO3H E only

H3C H3C CH3 Ph

Ph

H3C H3C

H29A-10 11/24/03 9:26 AM

Synthesis of LAF389
O S N + OHC O CH3 H3C O CH3 O OCH3 1) LiHMDS, -78 oC, THF CH3 2) TMSCl H3C 3) aldehyde 4) -78 oC: 1 hour 0 oC to RT: 1 hour 50 oC: 1 hour 45% single isomer, white crystalline solid CH3 H3C CH3 OH OCH3 O NH O L. Waykole, et al., Organic Process Research and Development, 2003, ASAP (Novartis Process Group) O LAF389 H3C O H3C O CH3 O O OCH3

O S O H3C H3C

H N

OH

OH

Synthesis of LAF389
S N + H O

O S O H3C H3C CH3

1) LiHMDS, -78 oC 2) TMSCl 3) aldehyde 4) -78 oC to 50 oC

TMS N S

O O S O Ph

t-Bu Ph

H3C H3C CH3 Ph

Ph TMS TBAF E:Z = 1:1 CH3 Ph Ph N S O O S O Ph L. Waykole, et al., Organic Process Research and Development, 2003, ASAP (Novartis Process Group) t-Bu Ph

MeSO3H E only

H3C H3C CH3 Ph

Ph

H3C H3C

H29A-11 11/24/03 9:26 AM

Pyridinyl (PYR) Sulfones Examples


Me OTIPS KHMDS toluene E,Z + OTIPS

Me

CHO

PYR-SO2

Me E,E temp -78 oC 0 oC 25 oC yield 35% 53% 67% Ratio E,Z : E,E 84:16 90:10 91:9

OTIPS

Potassium metallate is stable at RT for 5 min!

A. B. Charette, et al., Tet Lett, 2001, 42, 5149

Pyridinyl (PYR) Sulfones Examples


Me

OMe

Curacin B N S Me

Me Me + PYR-SO2

Me CHO KHMDS, toluene 25 oC 64%, 88:12 Me OTIPS Me Me

OTIPS

Me Me

Me SO2-BT + CHO LiHMDS, CH2Cl2 -78 oC to RT 75%, 6.5:1 ( 87:13 ) Me Me Me

H29A-12 11/24/03 1:20 PM

A. B. Charette, et al., Tet Lett, 2001, 42, 5149

1-Phenyl-1H-tetrazol-5-yl Sulfones
N N N N Ph M Li Na K toluene 51:49 65:35 77:23 O O S O Me MN(SiMe3)2 H -78 C to RT
o

Me

Et2O 61:39 65:35 89:11

THF 69:31 73:27 97:3

DME 72:28 89:11 99:1

n-C5H11

n-C4H9

n-C5H11 Me

n-C5H11

n-C5H11 Me E:Z = 99:1

E:Z = 94:6

E:Z = 96:4 1.5eq aldehyde, KHMDS, DME -78 oC to RT

P.J. Kocienski, et al., Synlett, 1998, 26

Kinetically Controlled Diastereoselectivity - Irreversible


HO Me KHMDS, DME -60 oC to rt, 1.5hr PT

Me O2S

Me 91%, E:Z = 98:2

TESO

Me

TBAF, DME -60 oC to rt, 1.5hr

Me Me 91%, E:Z = 2:98

Me O2S

PT

CHO OH Ph PTSO2 LDA, THF -78 oC to RT O2N Ph 100% E:Z = 98:2 Ph 0% NO2

P.R. Blakemore, Ph.D. Thesis, University of Glasgow, Glasglow, 1999


H29A-13 11/24/03 1:23 PM

Synthesis of Herboxidine
O O N + CH3 CHO N N N SO2Et KHMDS DME -60 C, 45min 93%, 93:7 dr
o

O O CH3

1) AD-mix 2) TsOH, MeOH CH3 O CO2All CH3 H H H3CO O H 10 steps

CH3 CH3 OH

O P.J.Kocienski, et al., J. Chem. Soc., Perkin Trans. 1, 1999, 955

CH3 O CO2All CH3 H O S N SO2 OMe Me PCBO Me Me O Me H 1) LDA, THF -78 C to -20 C 81%, 91:9 dr 2) K2CO3
o o

Synthesis of Herboxidine
CH3 O CO2H CH3 H Me O Me

OMe Me OH Me

P.J.Kocienski, et al., J. Chem. Soc., Perkin Trans. 1, 1999, 955

H29A-14 11/24/03 1:25 PM

Synthesis of (+)-Ambruticin
OTBS OTBS TBDPSO O O H + PT O S O Me Me Me Me O Me

M-HMDS Na OTBS OTBS TBDPSO O Me Me Me Me K O Me Li Li Li Na K

solvent THF THF DMF

temp -78 oC -35 oC -60 oC

E:Z 1:8 1:6 1:1 1:3 3:1 >30:1 >30:1

DME/18-c-6-60 oC THF/HMPA -60 oC DMF/HMPA-35 oC DMF/HMPU-35 oC

E.N. Jacobsen, P. Liu, J. Am. Chem. Soc., 2001, 123, 10772

tert-Butyl-1H-tetrazol-5-yl Sulfones
SO2Het Me 1) KHMDS, DME -60 oC, 2 hr 2) H2O Het SO2Het Me BT PT TBT yield 0% 20% 91%

O N N N N S

1) KHMDS, DME -60 oC, 30min n-C9H19 2) n-C9H19CHO -60 oC to RT

n-C9H19

39% 67:33 O N N N N S O 1) KHMDS, DME -60 oC, 30min n-C9H19 2) n-C9H19CHO -60 oC to RT

n-C9H19 60% 4:96

H29A-15 11/24/03 1:29 PM

Diastereoselectivity of TBT-Sulfones
OLi R2 SO2TBT anti N N N N t-Bu O Li S O O H HR2 N t-Bu N slow R2 O N N SO2 H Li PYRO H SO2Li R2

R2 N N N t-Bu N S O2 Li + H O E R2 R2 N N N t-Bu N O Li S O O R2 HH Z

OLi R2 SO2TBT syn

N t-Bu N fast H O

N N Li PYRO R2 SO2Li H

SO2 R2

Sulfone Synthesis
RX, BX = halide, tosylate, triflate SH N N S R N N

N N

ROH R'O2CN=NCO2R' PPh3 N N SH S 2-mercaptobenzothiazole 100g = $18.00


H29A-16 11/24/03 1:33 PM

N SH

N SH

N N

N N

t-Bu N C S

NaN3

1-phenyl-1-H-tetrazole-5-thiol tert-butyl isothiacyanate; 25g = $57.80 Sodium azide; 25g = $51.90 25g = $22.60

Sulfone Synthesis
N N N N Ph MCPBA (NH4)6Mo7O24-4H2O / H2O2 Na2WO4-2H2O / H2O2 Oxone CH3CO3H KMnO4 P.R. Blakemore, J. Chem. Soc., Perkin Trans. 1, 2002, 2563 Mo(VI) W(VI) S R [O] N N O S O R N N Ph

Oxidation Problems - Allylic Sulfones


C(CF3)2OTES C(CF3)2OTES C(CF3)2OTES N S Mo(VI), H2O2 EtOH - H2O, r.t. N N O2 S HO

N N

N N

38% O PT S R [2,3] PT O S R

11%

H. Hilpert, B. Wirz, Tetrahedron, 2001, 57, 681 D.A. Evans, G.C. Andrews, Acc. Chem. Res., 1974, 7, 147
H29A-17 11/24/03 1:34 PM

Synthesis of the Proposed Structure of Amphidinolide-A


AcO Me H O OTBS O O Me S N S KHMDS THF -78 oC to RT 78% 4:1 dr inseparable AcO Me OTBS 1) HF-pyr (separate isomers) 2) t-BuOOH, Ti(OiPr)4 3) EDC, DMAP, CH2Cl2, (E)-iodobut-2-enoic acid OTBS + O PT S O2 O KHMDS DME -55 oC to RT Me O 92% 95:5 dr 40g scale I G.Pattenden, H.W.Lam, Angew. Chem. Int. Ed., 2002, 41, 508 M. Hirama, Tet. Lett., 1999, 40, 4897 Me O AcO Me O Me O Me O Me Me

Me

OTBS Me H O

Synthesis of Proposed Structure of Amphidinolide-A


OTES TESO TESO Me TESO SnBu3 I Me O O O n-Pr I Me O SnBu3 AcO AcO Me Me O O Me n-Pr

1) Pd2(dba)3, Ph3As 2) PPTS 3) Pd2(dba)3, Ph3As, LiCl OH HO HO HO Me O Me O Me O n-Pr HO HO HO Me O Me O OH Me O n-Pr

NOT Amphidinolide-A

NOT Amphidinolide-A

G.Pattenden, H.W.Lam, Angew. Chem. Int. Ed., 2002, 41, 508


H29A-18 11/24/03 1:38 PM

Synthesis of Okadaic Acid


Ph O O O Ph Ph O O O Ph "small quantity of the corresponding (Z)-isomer was also detected by 600 MHz 1H NMR" S.V. Ley, et al., J. Chem. Soc., Perkin Trans. 1, 1998, 3907 O O H H O O H H O O SO2-BT Me NaHMDS DMF-THF -60 oC to RT 66% O H OBn O Me H O H H Me H O O Me O H OTBDPS OHC H OBn H O H H Me H O O Me O H OTBDPS O

OTIPS

OTIPS

Synthesis of Vinylsilanes
O Ph SiMe3 + O O S PT SiMe3 Ph Ph Ph

O O S PT LiO R SiMe3 Brook O O S PT TMSO R

Ph

Smiles Ph

M-HMDS SiMe3 Ph Li Li Li

temp -78 oC -85 oC -95 oC

yield 93% 89% 84% 50% NA

E:Z 64:36 74:26 75:25 59:41 NA

Na -78 oC Ph Ph OTMS Ph K -78 oC

J. Wicha, et al., Org. Lett., 2003, 5, 2789

H29A-19 11/24/03 1:42 PM

Conclusions
BT and PT sulfones have become useful funtional groups for the synthesis of olefins from aldehydes inexpensive starting materials sulfones can be made in high yield from alcohols olefination reactions occur under mild conditiions, and are typically high yielding and selective Stereochemical outcome is kinetically controlled in most cases. Though reaction conditions can often influence selectivities Polar solvents with soft counter ions often favor E olefins Non-polar solvents with hard counter ions often favor Z olefins

BT sulfones are most useful for the synthesis of conjugated dienes through reaction with ,unsaturated aldehydes R SO2BT + OHC

R2

R2

PT sulfones are most useful for the synthesis of non-conjugated (E) olefins R + R

SO2PT

OHC

R2

R2

PYR and TBT sulfones both produce high levels of cis selectivity, though yields are typically lower than BT reactions

Definitely more to come in the future

H29A-20 9/25/03 2:45 PM

D. A. Evans

Pairwise Functional Group Relationships


Relevant Questions

Chem 206

http://www.courses.fas.harvard.edu/~chem206/

The pyridoxal co-factor (Vitamin B6) 1, facilitates the decarboxylation of -amino acids. Provide a mechanism by which 1 carries out this transformation.
H O O PhosphateO OH H2 N R OH -CO2 H2 N R O C O

Chemistry 206 Advanced Organic Chemistry


Lecture Number 30
1
N

Me

Ambiphilic Functional Groups4


s Construction of Consonant & Dissonant FG Relationships s Charge Affinity Inversion Operators

Cume Question, Fall 2001. The reaction illustrated below was recently reported by Murry and co-workers from the Merck Process Group (JACS 2001, 123, 9696-9697). Provide a mechanism for this transformation.
Tol + Ar1 H Ar2 N H Ar2 10 mol% 1 OCMe3 Et3 N, CH2 Cl2 35 C Ar1 O N H O OCMe3

SO2

Lecture 27 and handout 27A


Me N

S CH2 CH2 OH

Handout 30A: Homoenolates: Synthesis & Applications Mesembrine Syntheses: Keely, S. L.; Tahk, F. C. JACS. 1968, 90, 5584. Stevens, R. V.; Wentland, M. P. JACS 1968, 90, 5580 (handouts) Stetter, The catalyzed nucleophilic addition of aldehydes to electrophilic double bonds. Org. React. (N.Y.) 1991, 40, 407. Ahlbrecht, "Stereoselectivity of chiral homoenolate equivalents." Synthesis 1999, 365-390.

1 (used as catalyst)

The nucleophilic addition of aldehydes to electrophilic double bonds catalyzed by thiazolium salt 1 is referred to as the Stetter Reaction (Stetter, H.; Kuhlmann, H. Org. Reactions 1991, 40, 407). Provide a mechanism for this transformation.
O Me Me H O O 10 mol% 1 Et3 N, EtOH 60 C Me O Me

D. A. Evans
30-00-Cover Page 12/1/03 9:07 AM

Monday, December 1, 2003

D. A. Evans

Summary of Functional Group Classification Scheme


Classification of Functional Groups
G-Functions () (+) ()

Chem 206

Each substituent attached to carbon activates that carbon toward a polar reaction by either resonance or induction or both. Induction (+) (+) () ()

Those ideal FGs which create nucleophilic carbon at point of attachment. Exhibit strictly alternate charge affinity patterns.

F1
Resonance

C
(+)

F2

C
()

F3

C
(+)

F4

C
()

These are your metallic FGs such as Li, Mg, etc.


CH3 CH2
()

Li

CH2
()

CH

CH2
()

MgBr

Symbol

C (+)

C ()

C ()

Note that a 2-electron reduction (or oxidation) will transform an E-Class


FG to a G-Class FG. (+) +2 e ()

Charge Affinity Patterns


E C C C A A C
() (+) () (+) () () (+) ()

E
CH3 CH2
(+)

C
Br

G
CH3

C
CH2
()

C C A

C
(+) (+)

G C C

C
MgBr

() ()

Real functional groups are assigned to a class designation by inspection of the chemistry of that FG, along with that of its conjugate acid and conjugate base Charge affinities of real functional groups form a subset of the ideal FG classes. E-Functions
CH3

A-Functions

()

() ()

(+) (+)

All sites activated equally for electrophilic & nucleophilic reactivity. Those ideal FGs which exhibit nonalternate polar site reactivity are included.
One might visualize a process wherein A-functions are gradually polarized towards either E or G behavior in response to changes in inductive and resonance effects. () () () (+) () (+)

(+) () (+)

E
(+)

C
()

C
OR CH3
() (+)

CH2

OR

CH

()

CH3
()

(+) ()

CH2
(+)

CH

(+)

CH2

Br
()

CH3

(+)

CH2

()

Br

CH3

(+)

CH2

()

NR2

Note that the issue of oxidation state in not explicitly incorporated.


This issue is subordinate to that of defining site reactivity. For example,
OR CH3
() (+) ()

(+-) (+-) (+-)

C A C C
() () () () (+) ()

O is represented as:

() (+)

() (+)

E and not: C

C E2 E1

A-functions are some of the most useful FGs in organic synthesis because of the unique reactivity provided.

30-01 Pairwise FGs-1 11/30/03 6:09 PM

()

C A

D. A. Evans

Pairwise Functional Group Relationships A-Functions: Real Examples

Chem 206

Classification of Pairwise Difunctional Relationships


Consider the paired relationships of E-functions. There are two relationships.

A-Functions

()

(+) (+)

() ()

A-functions are composed of polyatomic arrangements of N & O.


NO2 NOH NNR2 N NR2 N(O)R N N N

A-functions are composed of second-row elements such S and P.


S R R P R O S R O P R R O S R O R P R R R S R

Functional groups derived from many of the transition elements Synthesis of Targets containing E-Functions

Transforms utilizing target E-function in synthesis plan given highest priority.

E
(+) () (+) ()

E
(+) () (+) ()

E
(+) () (+) ()

Given the resident E-function, the charge affinity pattern dictates the nature of the polar coupling process and thus functional groups to be employed in synthesis.

30-02 Pairwise FGs-2 11/30/03 6:11 PM

()

E#
(+) () (+) ()

C C

add E

E# C C

E C C
Charge affinity patterns are "matched." Pairwise relationship is "consonant".

(+) () (+) ()

E#
(+) () (+) ()

add E

E# C C C

E C
Charge affinity patterns are "unmatched."

(+) () (+) ()

E# E
(+) () (+) ()

Pairwise relationship is "dissonant".

Consonant & dissonant relationships may be established with E-E, E-G, or G-G pairings. Most target structures are composed of E-functions.

E
(+)

G
() (+) ()

C C

GFG lost in construction


Me

Representative difunctional relationships


O O OH O Me NR2 E E'

E
(+) ()

E' C
(+) ()

Classification: 1,3-C symbolic representation

E'FG lost in construction

E
(+) () (+)

G C C
()

Cl

E E'

GFG lost in construction

HO NH2 OH

symbolic representation
G Li E Cl

Classification: 1,2-D

symbolic representation

Classification: 1,1-D

D. A. Evans

Pairwise Functional Group Relationships2

Chem 206

Classification of Pairwise Difunctional Relationships A single FG residing either in or appended to a cycle may establish a FG
relationship with itself.
E E E E

Pairwise Relationships: Path-Path Interconversions via Sigmatropic Rearrangements


[3,3] Sigmatropic Rearrangements:
R R O

1,2D

1,4D
OMe R

Consonant cycles

Dissonant cycles 1,2C


HO

OMe R O

1,5C
OMe OMe

Consonant & Dissonant Relationships: Path-Cycle Interconversions


Linear molecules may be transformed into cycles & vice-versa:
O O
(+) (+) NH2

For these rearrangements, CC', DD' but CD not possible

Relationship: MeO 1,5-C

(+)

HN

[1,2] Sigmatropic Rearrangements: Consonant Cycles


O CH2N2

C cycle

D cycle

Relationship: 1,5-C

Cl
(+) (+) (+)

OH O O O Me Me O O Me

[2,3] Sigmatropic Rearrangements:


R O R

Me

Dissonant Cycles
O

Relationship: 1,4-D
Me

1,2D

1,3C
OMe OMe

Me O

Me OR

Me

General Rule For [m,n] Sigmatropic Rearrangements: When the sum of m+n is even, the FG relationship is maintained, e.g. CC' When the sum of integers is odd, the FG relationship is changed, e.g. CD

Path-cycle interconversions such as those illustrated permute, but do not eliminate the relationship. i.e. D-bond paths are transformed into D-cycles.

30-03 FG Relationships-1 11/30/03 6:12 PM

D. A. Evans

Pairwise Functional Group Relationships3


A Specific Case
Target structure:
RO O OH Me O

Chem 206

Pairwise Relationships in Inorganic Reagents E-functions in their most stable oxidation states (HO, NH3, Cl ) are
represented as E(). There exist an important family of reagents which have E-FGs directly coupled: EE Reagents

Step I: There are 4 bonds interconnecting E & E'. Hence generate the 4 transforms leading to monofunctional precursors:

E# C C

E C C
RO

OLi Me RO

O Me O OH

[H]

O RO

OH Me

(+) () (+) ()

Br HO

Br HO OH H2N

NH2 NH2

In each of these reagents there is a 0,0-D relationship

E# C C

E C C
RO

OLi H

O Me RO

These reagents are used to construct D-Relationships:


O R Me Br2 -HBr R O Br R R Br Br2 H 2O R R OH

(+) () (+) ()

Me O OH Me
[H]

E# C C

E C C
RO

O Me

HO

(+) () (+) ()

RO

Synthesis of Targets containing Consonant Pairwise Relationships


E# C C E C C E#
(+)

E
() (+) () Aldol, Claisen Mannich Rxns

E# C C

E C C

O Me

HO

O HO Me

[Ox]

O RO

O Me

(+) () (+) ()

(+) () (+) ()

E# C C

E C C

E#
(+) ()

Step II: Evaluate the efficiency of the 4 plausible routes to the target from available precursors. Given the oxidation state in the target, the second synthesis looks the best and the fourth looks the worst.

E C
(+) ()

(+) () (+) ()

The Constraint of Quaternary Centers

E C

E C C C

E C

(+) () (+) ()

(+) () (+) ()

E () Conjugate Addition

If a quaternary center occurs along the consonant bond path, one is limited to bond constructions on either side of that restriction O Me Me O RO O Me Me O Me RO OLi Me RO Me O Me Me Me O Me OLi Me

E C C C
E #()

E
(+) () (+) ()

(+) () (+) ()

Consonant difunctional relationships can be constructed from just the functions illustrated & polar bond constructions.

RO

30-04 FG Relationships-2 11/30/03 6:13 PM

D. A. Evans

Pairwise Functional Group Relationships4


Synthesis of Dissonant Pairwise Relationlships
The pairwise relationship is "unmatched"; hence, the illustrated E-functions cannot be used exclusively to construct the bond path. Let's consider the simplest case: a 1,2-D relationship.

Chem 206

Quaternary Centers & Bridgehead Restrictions


Me N H H O H
()

Lucidulene Synthesis: JACS 94, 4779 (1972)


Me H
(+)

E# C

E C

Me N H H H Me

E
()

(+) ()
Resident E-functions do not provide required charge affininty pattern for coupling

(+)

Focus on the shortest consonant bond path:

(+)

E# E
(+) ()

E#
(+) ()

E (+)
This transform defined a path-cycle permutation of the D-relationship

The two permitted bond constructions along illustrated bond path flank the bridgehead carbon
Me
(+) ()

E
(+) (+)

Me H H Me H2C N+ H H HO H Me

In the illustrated polar disconnections, one of the fragments may exploit the charge affinity pattern of the resident FG while the other may not. Hence dissonant pairwise relationships may not be constructed via just the functions present in the target.

O (+)

Mannich Transform
Me N H H O H Me X O H H H Me

Dissonant Pairwise Relationlships via A-Functions


E# E
(+) ()

E# A
(+) ()

E#
(+)

A
()

Me + N

In implementing this strategy you must know all important 1,1-AE FG transformations

C
Me N S

A
S R

E
O

Enamine Acylation
Me H N H H Me
(CH2O)n , isoamyl alcohol

R H Me H R

R
HO

Option Selected:
O

H H

Ph trifluoroacetic
anhydride

R O N O R
1) OH 2) H3O +

CF3CO2 SPh R R O R R

The Pummerer Rearrangement


"The Pummerer reaction of sulfinyl compounds.", De Lucchi, etal. Org. Reactions 1991, 40, 157.

Corrolary: -conjugation cannot be extended through bridgehead or quaternary centers


R

The Nef Reaction

30-05 Synth Consonant-1 11/30/03 6:15 PM

D. A. Evans

Pairwise Functional Group Relationships5


Bond path analysis of simple alkaloids
CH2OH
(+) (+) (+)

Chem 206

OMe OMe

E2
(+)

Mesembrine
Curphey, T. J.; Kim, H. L. Tetrahedron Lett. 1968, 1441. Keely, S. L.; Tahk, F. C. JACS. 1968, 90, 5584. Stevens, R. V.; Wentland, M. P. JACS 1968, 90, 5580

lupinine

N
(+)

E1
(+)

Every complex polyfunctional molecule may be analyzed structurally in terms of its individual consonant or dissonant construction paths or cycles. For example, in the alkaloid lupinine all possible construction paths interconnecting E1 and E2 are consonant. Consonant paths within the polyatomic framework define seams in the structure that may be constructed using aldol and related processes.

N Me

Shamma, M.; Rodrigues, H. R. Tetrahedron 1968, 24, 6583

Begin the disconnection process by focusing on the shortest consonant bond path. In this case, there are 4 bonds, hence 4 disconnections.
(+) (+) (+)

In the analysis of potential routes to structures like mesembrine, identify the shortest consonant bond path and then proceed to carry out all polar disconnections along that bond path. Since there four bonds interconnecting =O and N (E1 and E2), there will be four associated transforms which one may execute using the illustrated functional groups. Ar Ar CE2
equivalent to:

E2
(+)

(+) C

E2
(+) equivalent to:

CH2=O

T1

E1 (+) () (+) E2 () Ar

HN Me Ar

CC

(+)

(+)

E1
(+) (+) (+)

E1
(+) (+)

N Ar E2
(+) equivalent to:

CC T1 E1 (+) () (+) E2 () Ar CC T1 E1 (+) () (+) E2 () Ar CE1

equivalent to:

OH
()E1 (+) (+) E2 ()

HO Ar
equivalent to:

N Me

CC

(+)

(+)

E1
(+) (+)

Shortest consonant bond path

RO2C

N Me

(+)

E2
(+) equivalent to:

CHO

Ar
equivalent to:

CE1

(+)

(+)

E1
(+) (+)

N H

T1
() E1

(+)

(+) E2 () ()

HO

N Me

(+)

E2 ()
(+) equivalent to:

OH2 CH2

CE2 (+)
(+)

(+)

E1
(+)

Now consider further analysis of T1: Again, select the shortest E1-E2 bond path and disconnect next to quaternary center. Dissonant element is localized in 5-membered enamine Ar Ar (+) (+) Ar
() () (+) E2 () equivalent to:

Note that oxidation states of precursors is not yet considered.

E1 (+) () (+) E2 ()

E1 (+) ()

Me

N Me

30-06 Synth Consonant-2 11/30/03 6:16 PM

Handouts Stevens, R. V.; Wentland, M. P. JACS 1968, 90, 5580

Keely, S. L.; Tahk, F. C. JACS. 1968, 90, 5584.

D. A. Evans

Inversion Operators-1: The Electronic Characteristics of Cyanide Ion


The Benzoin Condensation
O O H OH Me O
X

Chem 206

Inaccessible Reactivity Modes in Carbonyl Deprotonation


Example:
O H O H Me :CN

:CN

Cyanide ion is such a "catalyst"


O O H Ph H CN

C O

Me

carbonyl anion inaccessible

base
X

Ph

CH2

homoenolate anion inaccessible OH

Can one design "catalysts" which will provide access to carbonyl anion equivalents in situ??

CN

Ph

CN

Equivalent to:
O Ph C

Let Q be such a catalyst, we will call it an "inversion operator"


Ph O R H R O Q H

Ph OH Ph

OH El CN Ph

O H

How do we classify the

N functional group?

OH

Hydrogen cyanide is a fairly good Bronsted acid (pKaHOH 9.5) Equivalent to:
O

H C N
Acetonitrile can be attacked be nucleophiles:

H+

() +

:C N

O R El R

OH Q El

Me C N

Nu:

N Me C Nu

(+)

Acetonitrile can be deprotonated by strong bases (pKa DMSO ~ 30)

Me C N
30-07 Inversion Operators-1 11/30/03 6:18 PM

H+

+ H 2C C N

() (+)

C C

D. A. Evans

Thiazolium Salts as Inversion Operators


Cyanide-based Carbonyl Anion Equivalents Thiazolium Salts: Nature's Inversion Operators

Chem 206

Extensions of the Benzoin condensation concept are possible in some instances: O Ar H CO2Et

Reactions equivalent to the benzoin are catalyzed by biological co-factors to make (and break) dissonant difunctional heteroatom-heteroaton relationships NH2 H N N Me S OH The pka of this proton has been the subject of considerable study. The current estimates are that the value falls in the range of 16-20 but this number is not firm. F. G. Bordwell JACS 113, 985, (1991)

NaCN DMF Ar

O OEt O

44% yield
Me

The CC Bond Construction


OH R C CN O OEt

Stetter, Org. Reactions 1991, 40, 407. OH Ar OEt CN O

The thiamine cofactor


H Me N S base Me N

Me

Me

Me O R C

Me O R C

The in situ use of cyanide ion as an inversion operator is limited. Greater generality may be achieved by multistep alternatives:

Carbonyl anions might be similarly stabilized

Aldehyde Derivatization Step


O R H Me3SiCN ZnI2 R OH :C N H R H CN OSiMe3 H CN Et

In the absence of electrophiles 1 & 2 dimerize as would be expected for carbene reactivity. Me Me N H S OEt CN O C

base

Me

N S

Me Me N S OLi

Me

O R

OEt R

O H

R Li R

O C Li R

OLi R R OLi

Substrate Deprotonation Step


OSiMe3 R H CN Et O R H CN OEt LiNR2 R O CN LiNR2 R OSiMe3 CN Et OEt Deprotonation possible for All R groups G. Stork JACS 93, 5286 (1971) Deprotonation possible only for R = Ar due to Si migration.

Reactions catalyzed by thiamine


O Me O Me O OH Me H Me O H O Me OH

O C O

30-08 Inversion Operators-2 11/30/03 6:19 PM

D. A. Evans

Thiazolium Salts as Inversion Operators2


Aldehyde dimerization by Thiazolium Salts
O Me H
Thiazolium ion catalysis base

Chem 206

Cataylzed Michael Reactions byThiazolium Salts


O O R' R H O CH2Ph N H S R"" R R' O R''

O Me Me OH

The Reaction

The Reaction 1,2D

1,4D

The Catalytic Cycle


O Me H R R N S O H Me

Me

The Catalyst:
HO

The Conditions: 0.1 equiv catalyst, Et3N or NaOAc, EtOH or DMF at 60-80 C

Examples:
O

R N S

R N S OH

Me O O Me Me O

Equivalent to:
Me C

"The catalyzed nucleophilic addition of aldehydes to electrophilic double bonds.", Stetter, H.; Kuhlmann, H. Org. Reactions 1991, 40, 407. O Me Me 61% yield Me O O O Me Me Me O Ph Me O O Me

H O

41% yield

R O Me Me OH S Me H Me N

Me R Me O

OH O

Me H O

Me Ph O O

Me

21% yield

1,2D

Ph n-Pr H O

Ph

Hence dissonant relationships may made from E-functions if "inversion operator" is employed () (+) (+) ()
inversion operator

n-Pr Ph O

Ph

70% yield

C E

1,4-D relationships may also be made from E-functions if "inversion operator" is employed.

C E

C C E

C E

1,2D (+)

The is a fundamental strategy for handling the formation and cleavage of D-relationships in nature.

C E

(+) () (+) + C C C

inversion operator

C E

C E

1,4D

There is no analogue to this reaction in nature.

30-09 Inversion Operators-3 11/30/03 6:21 PM

D. A. Evans

Thiazolium Salts as Inversion Operators3


Design Attributes of Inversion Operators

Chem 206

Decarboxylation Cataylzed by Thiazolium Salts


Background: Decarboxylation from consonant difunctional relationships is facile:

O R (+) () O (+) OH

Inversion operators are constructed from A-functions or molecules containing D-relationships. The pyridoxal Co-factor (Vitamin B6)
H PhosphateO N O OH Me O OH N Me H2N R

O O R

O
(+) C

() The reverse processe can be achieved under basic conditions: O R M O (+)C O R O M O O O R (+) () O (+) OH

The critical difunctional relationlship is that between =O & =N. This is a 1,4-D relationship

H+

O OH

()

The Reaction

-CO2

H2N R

O C O

Such consonant relationships may be readily made (and broken) via the resident functional groups. The analog reactions for dissonant relationships not possible. O For example: Me OH X Me O H O C O

1,4-D relationship The Mechanism


O

1,2-D relationship
O R (+) OH N (+) (+) N (+) H R N H

O Nature uses inversion operators to break such 1,2-D relationships R O Me O OH S Me O R R N S Me R O Me H S N OH S N O H O R N O C O

H 2N R

OH

- H2 O

O C O

1,6-C relationship

N H R

H 2N R

Tautomerization

+ H2O
N

Me

30-10 Inversion Operators-4 11/30/03 6:37 PM

Chemistry 206 Advanced Organic Chemistry


Handout 30A

Homoenolates: Synthesis & Applications

Evans Group Seminar by Jason Burch, March 24, 2000

O X

MLn

E X

D. A. Evans

Monday, December 1, 2003

30A-01 12/1/03 11:16 AM

Enolates and Homoenolates


The Tautomerism Problem

Enolates

O M

tautomerism is generally not a problem because oxyanionic tautomer still acts as carbon nucleophile

Homoenolates

M
tautomerism is a much larger problem because it is often irreversible and oxyanioic tautomer rarely acts as a carbon nucleophile Nakamura in Comp.Org.Synth., 1991, 2, 441

Homoenolate Equivalents
Definition: species containing an ionic carbon to a moeity which can be converted into a carbonyl group Examples:

O O

O O

in situ protection

Y X
X = OR, NR2, etc. Y = H, R, OR, NR2, etc.

O NR
Werstiuk in "Umpoled Synthons", Hase, Ed.; Wiley: New York, 1987, Chap. 6 Ahlbrecht, Synthesis, 1999, 365 (chiral examples)

30A-02 12/1/03 11:18 AM

The First "Homoenolate"


Me Me O Me Me O

KOtBu, HOtBu 250 C, 4h

(+)-camphenilone

()-camphenilone

no racemization occurred in >4 days at 250 C in the absence of base proposed to proceed via a "homoenolate anion"

Me Me Me O
OtBu

Me

Me Me O

Me Me O

Nickon, J.Am.Chem.Soc., 1962, 84, 4604

Cyclopropane Ring Opening


Synthesis of Titanium Homoenolates
O RO TiCl3 R = Et
Me i Pr

OTMS OR

TiCl4

if conducted in CDCl3 leads to a deep wine-red color; precipitates as purple needles in hexanes IR spectrum strongly supports coordinated carbonyl (C=O = 1603 for R = iPr in benzene) molecular weight by cryoscopy is 560-620 indicating dimeric structure later verified in solid state by x-ray crystal structure (Floriani)

Relevant bond lengths (A): Ti-C 2.081 Ti-O 2.072 C=O 1.235 Nakamura, J.Am.Chem.Soc. 1983, 105, 651 Floriani, Organometallics, 1993, 12, 2845

30A-03 12/1/03 11:19 AM

Cyclopropane Ring Opening


Regioselectivity of Ring Cleavage - Titanium
in general, cleavage occurs selectively at the least substituted cyclopropane bond

O RO OTMS TiCl 4 OR R' RO O R'

TiCl3
R R' A:B >95 : 5 60 : 40 78 : 22

A
TiCl3

Pr

Me Me Ph

Me Et

R'

A can be isolated, but B is too unstable; only detected by in situ quench with electrophiles (i.e. Br2, RCHO) if non-racemic starting material is used, quench with electrophiles indicates non-racemized A and totally racemic B i.e.

OTMS OMe
>99:1

1. TiCl4 2. E+

O MeO

E
+

O MeO

E Me

Me

1:1

> 99% ee

Me

60 : 40 racemic Nakamura, J.Am.Chem.Soc., 1986, 108, 3749

Cyclopropane Synthesis
most common method

Cl O Me Cl O

OEt

2 Na

OTMS OEt

TMSCl

- used to prepare substituted cyclopropanes

Me OEt
2 Na

OTMS OEt

TMSCl

use Simmons-Smith for ketone-derived substrates

OTMS

OTMS

Et2Zn CH2I2
Ruhlmann, Synthesis , 1971, 236. Salaun, Org. Synth., 1985, 63, 147 Murai, J.Org.Chem., 1973, 38, 4354
30A-04 12/1/03 11:20 AM

Alkoxide-Modified Homoenolates
Tuning Titanium Homoenolate Reactivity
Problem: trichlorotitanium homoenolates are not reactive enough for some applications can also lead to chlorinated byproducts Idea: replace 1 or more chlorides with alkoxides to increase nucleophilicity

O
i

TiCl3
+

1/2 equiv. Ti(O Pr)4


i

O PrO O
i

TiCl2OiPr

+ 1/2 TiCl2(OiPr)2

PrO O TiCl3
+

1/2 equiv. Ti(O Bu)4

TiCl2OtBu

+ 1/2 TiCl2(OtBu)2

PrO

PrO

Have not been characterized to the detail of the trichlorohomoenolates Appear to have "significant contribution from monomeric forms" (from molecular weight data) Are more reactive than trichloro homoenolates towards homoaldolisation Nakamura, J.Am.Chem.Soc., 1986, 108, 3745

Cyclopropane Ring Opening


Zinc Homoenolates
Zinc homoenolates can be prepared in a similar method to titanium

O OTMS OR

OR OR
vacuum

ZnCl2 Et2O
RO

+ 2 TMSCl

Zn

OEt2 RO

Zn

C=O = 1662 cm-1 1740 cm-1 4 inequivalent methylene groups in 1H-NMR

C=O = 1645 cm-1 2 inequivalent methylene groups in 1H-NMR Nakamura, Organometallics, 1985, 4, 641

30A-05 12/1/03 11:21 AM

Direct Oxidative Addition


Zinc and Lanthanide Homoenolates
O EtO O Et I I

ZnCu
Benzene / DMA 60 C, 3-4 h

O EtO O Et

ZnI
Quantitative

ZnCu
Benzene / HMPA 25 C, 1 h

ZnI
Poor yield (~50%)

O MeO

Br

Ln, "pinch" I2

LnBr Poor yields of


homoaldolate (50-60%) due to competition with McMurray coupling of aldehyde Yoshida, Tetrahedron Lett., 1985, 26, 5559 Yoshida, Angew.Chem.,Int.Ed.Engl., 1987, 26, 1157 Fukuzawa, Chem. Commun., 1986, 475

THF MeO RT, 5 min Ln = La, Ce, Nd and Sm

Enolate Homologation
Synthesis of Zinc Homoenolates

OTMS
1. MeLi, 25 C 2. ZnEt2, CH2I2

ZnX

less reactive than most zinc homoenolates can also be used to form aldehyde homoenolates

OTMS H
1. MeLi, 25 C 2. ZnEt2, CH2I2

O ZnX

Knochel, J.Org.Chem., 1993, 58, 2694

30A-06 12/1/03 11:23 AM

Direct Tin-Titanium Exchange


New Route to Titanium Homoenolates
Treatment of tri-n-butylstannyl esters with TiCl4 directly forms titanium homoenolate

O RO

SnBu3

TiCl 4

O RO

TiCl3
+ Bu3SnCl

R = Me, iPr

CH2Cl2

Isotope labelling studies showed rxn does not proceed via cyclopropane Substrates can easily be prepared by two methods:

O MeO O
i

Bu3SnH, 80 C, 4 h

O MeO O
i

SnBu3

1. LiHMDS, -70 C, 15 min 2. ICH2SnBu3

SnBu3

PrO

PrO

Goswami, J.Org.Chem., 1985, 50, 5907 van der Kirk, J. Appl. Chem., 1957, 7, 356 Still, J.Am.Chem.Soc., 1978, 100, 1481

The First Homoaldol Reaction


O RO TiCl3

Synthesis of -hydroxyesters and -lactones O O O R'CHO + + O R' CH2Cl2 RO RO


OH R'
Workup Product

R' Cl
Yield

Aldehyde

Temp (C) Time (h)

O O H O H Me O O Ph
i

Et
n-Oct

Acidic

O n-Oct O Me

81

Pr

2.5

Neutral

67
i

PrO OH

Ph (85:15)

Et
H Ph

1.5

Neutral

EtO Cl

Ph

90

Nakamura, J.Am.Chem.Soc., 1977, 99, 7360

30A-07 12/1/03 11:24 AM

Homoaldol Reactions with Alkoxide-modified Homoenolates


Homoaldol Reactions with Aromatic Aldehydes and Ketones O
O
i

O
+

TiCl3
+ 1/2 Ti(OR)4

2 i

O PrO

PrO

CH2Cl2
1 hour

R1

O R2 R1

OH

A
Electrophile Benzaldehyde Crotonaldehyde Acetophenone R
i i

B
Yield (A or B) 90(A) 88(A) 66(A), 12(B) 93(B) 62(B) 91(B) 91(B) dr = 88 : 12 equatorial attack

Temp(C) 0 0 20 20 20 20 20

Pr Pr

i t

Pr Bu

Cyclohexanone

i Pr t

Bu Bu

Me

Nakamura, J.Am.Chem.Soc., 1986, 108, 3745

Homoaldol Reactions of Zinc Homoenolates


First Catalytic Homoaldol Reactions
1.2 eq

OTMS OEt

+ RCHO

cat. ZnX2

O R OTMS

CH2Cl2, RT EtO

TMSCl generated is essential (i.e. no reaction if removed in vacuo for stoichiometric case) Aldehyde Catalyst, yield ZnCl2 (30-50 mol%) ZnI2 (0.1-1 mol%) 84 94 89 84

PhCHO
Ph O O O2N n-Pent CHO CHO OBn
30A-08 12/1/03 11:27 AM

CHO CHO

91

95

--

84

79 93:7 syn : anti chelation product

--

Nakamura, J.Am.Chem.Soc., 1987, 109, 8056

Homoaldol Reactions of Zinc Homoenolates


Proposed Catalytic Cycle
EtO OTMS O ZnX
+ Me3SiX

ZnX2
X O EtO R OTMS EtO O ZnX
+

EtO

O R

SiMe3 H

RCHO

with ZnI2, the homoenolate is reactive enough to add to ketones:

O
+ 1.2 eq

OTMS Me
1 mol% ZnI2 OTMS CH2Cl2, RT 77%

OEt

Ph

Ph Me

OEt O

- no reaction even with stoichiometric ZnCl2 Nakamura, J.Am.Chem.Soc., 1987, 109, 8056

Gleason's Homoaldol reaction


First Catalytic Titanium Homoaldol Reaction
1. 0.1 equiv. TMSOTf 0.1 equiv. (R)-Binol-Ti(OiPr)2 3:1 CD3CN / CDCl3

EtO OTMS
1.5 eq +

O R1 R2
R2

R2 R1

2. pTsOH

R1

Conditions 0 C, 24 h 0 C, 36 h 0 C, 36 h 0 C, 80 h 45-50 C, 54 ha 45-50 C, 60 ha

Yield 99 76 82 84 52 78

Ph Ph TMS Cl
t

H H H H

Bu

H Me

Ph
a

2 equiv. of cyclopropane used Gleason, Org. Lett., 1999, 1, 1643

30A-09 12/1/03 11:28 AM

Gleason's Homoaldol reaction


Proposed Catalyst and Catalytic Cycle
Catalyst:

O O

Ti

OiPr OiPr

TMSOTf

O O

Ti

OiPr OTf

+ TMSOiPr observed in 1 H-NMR

catalyst appears as mixture of interconverting species in NMR at RT Catalytic cycle:


i

EtO OTMS O O O Ti O Pr OTf O

OiPr Ti
+ TMSOTf

O OEt OiPr Ti
+

O EtO Ph OTMS

O O

OTf
O R OEt

TMS H

RCHO

Gleason, Org. Lett., 1999, 1, 1643

Diastereoselective Homoaldol Reactions of Amide-homoenolates


Synthesis of syn- or anti--methyl--hydroxyamides
1. ZnCu, benzene/DMA 2. (iPrO)3TiCl, THF, 0C

O
i

Me R
syn +

O
i

I Me

3. RCHO, THF, 0 C RT Method A 1. Activated Zn, 0.1 equiv. TMSCl CH2Cl2, RT, 1 hour 2. RCHO, 2 equiv. TMSCl CH2Cl2, RT Method B

Pr2N O Me

OH

Pr2N

Pr2N
anti

R OH

Method

Time (h) 3 2.5 3 0.6 0.5 0.6

Yield 79 82 87 95 61 60

syn:anti 94 : 6 13 : 87 85 : 15 25 : 75 96 : 4 38 : 62

Ph
o-MeOPh 2-furyl

A B A B A B

Asaoka, J.Chem.Soc. Perkin Trans. 1, 1995, 285

30A-10 12/1/03 11:29 AM

Tandem Asymmetric Enolate Homologation - Homoaldol Reaction


Asymmetric Synthesis of -alkyl, -hydroxy Carbonyl Compounds

idea:

O R Xc

1. nBuLi 2. Zn(CH2I)2

ICH2Zn

O Xc R

1. Cl3TiOiPr 2. R'CHO

O R' OH R Xc

2 new stereocenters generated

O
Xc = Me

Me

initial results:

O Xc

Bn

1. nBuLi 2. Zn(CH2I)2 3. H +

O Me Xc Bn 30-40%
McWilliams, J.Am.Chem.Soc., 1996, 118, 11970 But ReactIR showed disappearance of enolate and appearence of new species

Tandem Asymmetric Enolate Homologation - Homoaldol Reaction


Asymmetric Synthesis of -alkyl, -hydroxy Carbonyl Compounds
proposal: zinc enolate unreactive towards homolagation; higher order zincate (zincate + extra enolate) active species in migration

Bn

OLi Bn Xc

OLi Bn Xc Xc O Xc (ICH2)2Zn Bn OR' O Xc

Zn(CH2I)2 (ICH2)2Zn
Bn O

2 eq. enolate Me needed so max yield is 50%

H
Xc Bn

ICH2Zn

O Xc Bn

involvement of higher order zincates in 1,2migrations is known (Harada)

idea: add an equivalent of alkoxide to take the place of the enolate in the higher order zincate

O Bn Xc

1. nBuLi 2. ROLi 3. Zn(CH2I)2 4. H +

O Me Bn Xc

ROLi

Conversion 35 74 82 (78% isolated) 31

EtOLi nPrOLi BnOLi LiO(CH2)2OLi

McWilliams, J.Am.Chem.Soc., 1996, 118, 11970 Harada, J.Org.Chem., 1993, 113, 2958

30A-11 12/1/03 11:30 AM

Tandem Asymmetric Enolate Homologation - Homoaldol Reaction


Asymmetric Synthesis of -alkyl, -hydroxy Carbonyl Compounds
How about the homoaldol reaction?

BnO ICH2Zn O Xc R
R T (C) -20 -20 -40 -40 -50 -20

O R Xc

1. nBuLi, <-65 C 2. Zn(CH2I)2, 3 equiv. BnOLi -70 C Aldehyde (R')

1. 2 equiv. Cl3TiOiPr -80 C T C 2. R'CHO, T C

O R' OH R Xc

de (%) 99 82 82 80 76 64

Yield 59 58 50 44 53 53

Bn BocHN

Bn Me Bn Me Bn Bn

phenyl phenyl
iso-propyl n-butyl

McWilliams, J.Am.Chem.Soc., 1996, 118, 11970

Reactive Homoenolates
First Synthesis of a Metal-free Homoenolate
TMS O OMe

TBAT

Bu4N

O OMe

O OMe

reactive enough to add to imines, as well as aldehydes and ketones e.g.:

TMS

O OMe

1. TBAT 2.

Ph Ph Ph

NHPh O

Ph
+

Ph N O

Ph

OMe
single diast. 22%

Ph

1:1 dr 60%

substrate made easily using Nishiguchi method

O
Mg anode, 2 e

TMS
-

O OMe

OMe

TMSCl, DMF

Fry, Tetrahedron Lett., 1999, 40, 7945 Nishiguchi, Tetrahedron Lett., 1992, 33, 5515

30A-12 12/1/03 11:31 AM

Conjugate Addition of Zinc Homoenolates


Synthesis of ,-(silylenolether)-esters
O
Zn(CH2CH2CO2Et)2 (1.2 equiv.) + + TMSCl (2.4 equiv.) Important!!

OTMS R
2

CuBrDMS (2 mol%) Et2O / HMPA

R2 R3 R4 O OEt

R3

R4

Enone

Product

Yield

Enone

Product

Yield

OTMS
93

O H Me O
78

OTMS Me H Me Me Me
75

(CH2)2CO2Et O OTMS
72:28

91:9

(CH2)2CO2Et OTMS
73

(CH2)2CO2Et Me Me

Me Et Me

(CH2)2CO2Et

Nakamura, J.Am.Chem.Soc., 1987, 109, 8056 Nakamura, Org. Synth., 1987, 66, 43

Acylation of Zinc Homoenolates


Synthesis of -ketoesters - Yoshida
O ZnI
+

1.5 equiv.

O R Cl

O
4 mol% Pd(PPh3)4

EtO

Benzene / DMA
RT, 30 min

EtO O

Yield 100

Ph

90

Me Ph Cl
92

100

Yoshida, Tetrahedron Lett., 1985, 26, 5559

30A-13 12/1/03 11:33 AM

Acylation of Zinc Homoenolates


Synthesis of -ketoesters - Nakamura
OR1
0.5 equiv.

O O R2 Cl
5 mol% PdCl2(PPh3)2

Zn

Et2O, RT

R 1O O

R2

R 1O

note: only 0.5 equiv. of Zn species needed so both homoenolates are transferred R1 R2 Yield 93 when carried out in CDCl3 got quantitative O-acylation (with or without Pd) i.e.

Et
i

Ph
Me

Pr

81

Me Ph
t

OR1 OCOR2

Et
i

83

Pr

Bu

50

Nakamura, J.Org.Chem., 1987, 26, 8056

Application of Homoenolate Acylation


Synthesis of ,-disubstituted -butyrolactones by Diastereoselective Reduction
Me R O O
DIBAL

Me R OH

O NiPr2
MsCl / Et3N CH2Cl2 0 C, 1 h

NiPr2 THF, -78 C, 10 min


Prepared by Yoshida method

dr 100:0 for all substrates

Me

MsO
Silica Gel 1-3 days

Me R OMs

O NiPr2

NiPr2

Me

Yields Cyclization Reduction 87 93 90 62 82 95 92 83 Asaoka, Heterocycles, 2000, 52, 227

Ph
R O O
p-ClPh p-MeOPh

Bn
30A-14 12/1/03 11:34 AM

Arylation and Vinylation of Zinc Homoenolates


Synthesis of -vinyl and -aryl esters - Yoshida
O EtO
X = I, OTf Coupling Partner
t

ZnI

X
+

O
1-4 mol% PdCl2(P(o-Tol)3)2 Benzene-DMA 60 C 0.5-1 h

EtO

Yield 74 Only vinylation example given

Bu

OTf

MeO

96

Br

67

O2N

80

Yoshida, Tetrahedron Lett., 1986, 27, 955

Arylation and Vinylation of Zinc Homoenolates


Synthesis of -vinyl and -aryl esters - Nakamura
OR
1.5 equiv. O

X
+

O
1-2 mol% PdCl2(P(o-Tol)3)2

Zn

THF, 0-20 C
X = I, Br, OTf

RO

RO

R = Et, iPr

Coupling Partner

Yield 90

Bu

I SiMe3

Me

87

Br X
X = OTf Br I 0 67 79

Me Br O
Nakamura, J.Org.Chem., 1987, 26, 8056 49

30A-15 12/1/03 11:35 AM

Arylation of Palladium Homoenolates


Catalytic Formation of aryl Ketones
O R2 R1 Ar

R1
1.5 equiv.

R2 OTMS+ ArOTf

[PdCl(C3H5)]2 (5 mol%) Ph3P (20 mol%) HMPA, 100 C, 12 h

proposed to proceed via

O R2 R1

PdArLn

R1

R2

Ar
1-napthyl p-NO2Ph

Yield 84 68 65 58

-CH2CH2CH2CH2-CH2CH2CH2CH2H
n-Heptyl p-OMePh

Phenyl
1-napthyl

Nakamura, J.Am.Chem.Soc., 1988, 110, 3296

Allylation of Zinc Homoenolates


Synthesis of ,-unsaturated Esters - Yoshida
O O EtO ZnI
+

R1 R2

EtO
20 mol% CuCN THF / DMA RT, 24 h

SN2' +

1 R2

O EtO

R1 R
2

SN2

R1

R2

X OTs
OTs Br Cl

Yield 89 80 93 99 80

SN2' : SN2 -87 : 13 88 : 12 87 : 13 100 : 0

H Ph CO2Me

H H H

Br

Yoshida, J.Org.Chem., 1987, 52, 4418

30A-16 12/1/03 11:35 AM

Allylation of Zinc Homoenolates


Synthesis of ,-unsaturated Esters - Nakamura
OiPr
i

O Cl R
2

O
i

Zn

R1

PrO O
+

SN2'

5 mol% CuBrMe2S THF / DMF RT, 24 h


i

1 R2

PrO

R = Et, iPr

PrO R
2

R1

SN2

Allyl chloride

Yield

SN2' : SN2 96 : 4 1 : 99 Catalyst changed to 5 mol% NiCl2dppe

Ph Me Me Me AcO

Cl Me Cl Me

97 65

81

88 : 12

72

100 : 0

Cl
Nakamura, J.Am.Chem.Soc., 1987, 109, 8056

Carbonylative Symmetrical Coupling of Palladium Homoenolates


Catalytic Synthesis of 4-keto Pimelates
CO (1 atm) PdCl2(PPh3) (2 x 2.5 mol%) CDCl3, 60 C, 32 h

OTMS OR
R = Et, Pr, n-Hex
i

O RO O

O OR
Yields: 65-80%

proposed to proceed via

O RO

PdArLn

evidence

OTMS OR

PdCl2(PPh3) (2 x 2.5 mol%) CDCl3, 60 C, 32 h

O RO

O RO

Nakamura, Tetrahedron Lett., 1988, 29, 1541

30A-17 12/1/03 11:36 AM

Crimmins' Cyclopentenone Synthesis


Introduction and Generality

O X R EtO

O
2

1.2 equiv.

O X R

X
OR N

Yields 50-60% 80-90%

Zn

TMSCl, 2.4 equiv. CuBrMe2S, 0.15 equiv. HMPA, 2.4 equiv.

O X EtO R OH

O
2

2.4 equiv.

O X N R HO

OR

50-80% 80-90%

Zn

TMSCl, 4.8 equiv. CuBrMe2S, 0.3 equiv. HMPA, 4.8 equiv.

Functionality supported in R: ethers, epoxides, furans, , unsaturated esters

Crimmins, J.Org.Chem., 1993, 58, 1038

Crimmins' Cyclopentenone Synthesis


Cl Si O EtO R M O OEt
M = Cu or Zn

Mechanistic Considerations
TMSO OEt

R EtO O ZnX

H+
EtO2C

CO2Et R

main side product

problem: two steps have opposite electronic requirements appears amide and ester have right balance; ketone too electron poor to cyclize i.e.

O Me R EtO

O
2

TMSO Zn EtO O

Me

O OEt R

TMSCl CuBrDMS HMPA 80%

Crimmins, J.Org.Chem., 1993, 58, 1038

30A-18 12/1/03 11:37 AM

Leahy Cyclopentannulation
Synthesis of 3,3-disubstituted cyclopentanones
O R OHC
1

1.2 equiv.
2

EtO2C R2 R1 TMSO
H2SO5, EtOH 67% avg (from enal)

EtO

Zn

R2

TMSCl, 2.4 equiv. CuBrDMS, 0.5 equiv. HMPA, 2.4 equiv.

O R R1
Substrates studied: R1 = R2 = n-Bu R1 = R2= Ph R1 = Ph, R2 = Me R1 = Ph, R2 = H
2

1. NaH, PhH, EtOH (cat) 2. HCl 77% avg

EtO2C R2 EtO2C R1

Leahy, J.Org.Chem., 1994, 59, 5496

Tandem Aldol / Aldol / Homoaldol Reaction


One-pot synthesis of a 6,7,6-tricycle

OH OEt

1. CH3MgBr, THF, 0C 2.

OH H H OH

OLi
67% -78 25 C

Relative stereochemistry verified by x-ray crystallography

A possible mechanism:

O OH CH MgBr 3 OEt OMgBt OEt O MgBr O O O OH OH O O Mg O O MgBr

Helquist, J.Am.Chem.Soc., 1986, 108, 8313

30A-19 12/1/03 11:45 AM

Synthetic Examples
Depresosterol
OH Me Me Me H AcO H CHO H PrO
60%, > 6:1 dr
i

TiCl2OiPr Me H AcO

Me Me H O

OiPr

trichlorotitanium homoenolate is not reactive enough to add to hindered aldehyde Nu

H H

Me O

Via Nakamura, J.Am.Chem.Soc., 1985, 107, 2138

Synthetic Examples
Depresosterol - completion of the formal synthesis
O CO2iPr R
R = steroid ring system 1. MsCl, Et3N 2. KOH, MeOH, 3. aqueous HCl 4. TBSCl, Et3N (reprotects steroid OH) 84%

OH Me

O Me R
1. LDA, -78 -10 C 2. CH2O (g) 3. aqueous HCl 70%, ~10:1 dr

OAc Me Me H Me H AcO
Triacetyldepresosterol Kashman Depresosterol

OAc Me OH Me
1. MeLi, THF, 45 C 2. Aqueous AcOH, rt

O O Me R OH

3. Ac2O, pyr., rt 46%

Nakamura, J.Am.Chem.Soc., 1985, 107, 2138 Kashman, Tetrahedron, 1981, 37, 2397

30A-20 12/1/03 11:46 AM

Synthetic Examples
Depresosterol - Maximizing Homoenolate Functionality

O O Me Me H O

Me
O Me

OH Me

OH Me

Me

OH Me

Nakamura, J.Am.Chem.Soc., 1985, 107, 2138

Synthetic Examples
Pumiliotoxin 251D
1. 2Ti Cp

OMe N Cbz O

Me Al Me Cl

Me N Cbz Me OH H O

6.2 eq

TiCl3

EtO
49%

2. 1 N HCl 87%

Me OH H
3 steps

H2, Pd/C

O N Cbz H Me OH OEt

N Me
Pumiliotoxin 251D

Gallagher

N O

100%

Me

"single diastereomer"

Nu via:

N H CBz

Ti Ln

O
Nu

Barrett, J.Org.Chem., 1999, 64, 1410 Gallagher, J.Am.Chem.Soc., 1991, 113, 2652

30A-21 12/1/03 12:44 PM

Synthetic Examples
EtO O

()-Cortisone

6 steps

O
() -

Me

Me
i PrO

O
2

Zn

2.6 equiv.

O
()

Me

Me OH

OTMS H

CuBrDMS, 0.15 equiv. O HMPA, 2.6 equiv. BF3OEt2, 4.8 equiv. i PrO >95:5 dr, >80%y

reaction in absence of lewis acid was unselective

OH Me Me H O
()-cortisone Nakamura, J.Org.Chem., 1986, 51, 4324

O OH
7 steps

Synthetic Examples
()-Ginkgolide B
O O EtO2C
()

EtO
1.2 equiv.

2 Zn

CO2Et

OTES Bu

TMSCl, 2.4 equiv. CuBrDMS, 0.25 equiv. HMPA, 2.4 equiv. THF, 23 C, 4 h 82%

OTES tBu
h (366 nm) hexanes 100%, 98:2 dr

HO OH

O O

CO2Et

Me

HO O O

Bu

TESO

Bu

()-Ginkgolide B Crimmins, J.Am.Chem.Soc., 1999, 121, 10249

30A-22 12/1/03 12:45 PM

D. A. Evans

Carbocations: Stability & Structure

Chem 206

http://www.courses.fas.harvard.edu/~chem206/

Other Relevant Background Reading


March, Advanced Organic Chemistry, 4th Ed. Chapter 5, pp165-174. Lowery & Richardson, Mech. & Theory in Org, Chem., 3rd Ed. pp 383-412. Arnett, Hoeflich, Schriver in Reactive Intermediates Vol 3, Wiley, 1985, Chapter 5, p 189. Saunders, M. and H. A. Jimenez-Vazquez (1991). Recent studies of carbocations. Chem. Rev. 91: 375. Stang, P. J. (1978). Vinyl Triflate Chemistry: Unsaturated Cations and Carbenes. Acc. Chem. Res. 11: 107. Olah, G. A. and G. Rasul (1997). Chemistry in superacids .26. From Kekule's tetravalent methane to five-, six- and seven-coordinate protonated methanes. Acc. Chem. Res. 30(6): 245-250. Olah, G. A. (1995). My search for carbocations and their role in chemistry (Nobel lecture). Angew. Chem., Int. Ed. Engl. 34, 1393-1405
Qumulative Exam Question Fall, 2001. The reaction illustrated below was recently reported by Snider and co-workers (Org. Lett. 2001, 123, 569-572). Provide a mechanism for this transformation. Where stereochemical issues are present, provide clear three dimensional drawings to support your answer. O Me Me Me Me R EtAlCl2 CH2Cl2, 0 C Me Me O R

Chemistry 206 Advanced Organic Chemistry


Lecture Number 30

Introduction to Carbonium Ions


Carbocation Stabilization Carbocation Structures by X-ray Crystallography Vinyl & Allyl Carbonium Ions

Reading Assignment for this Lecture:


Carey & Sundberg, Advanced Organic Chemistry, 4th Ed. Part A Chapter 5, "Nucleophilic Substitution", 263-350 .
Birladeanu, L. (2000). "The Story of the Wagner-Meerwein Rearrangement. J. Chem. Ed. 2000, 77, 858. (handout) Olah, G. A. (2001). 100 Years of Carbocations and their Significance in Chemistry. J. Org. Chem. 2001, 66, 5944-5957. (handout) Walling, C. (1983). An Innocent Bystander Looks at the 2-Norbornyl Cation. Acc. Chem. Res. 1983, 16, 448. (handout) Laube (1995). X-Ray Crystal Structures of Carbocations Stabilized by Bridging or Hyperconjugation. Acc. Chem. Res.1995, 28,: 399 (handout)

Carey & Sundberg-A, p 337: Provide mechanisms for the following reactions.
OH NH2 CHO

NaNO2 HOAc/H2O

OH NH2

NaNO2 HOAc/H2O

D. A. Evans
31-00-Cover Page 12/3/03 8:35 AM

Wednesday, December 3, 2003

CMe3

CMe3

D. A. Evans. B. Breit
Carbocation Subclasses
Carbon-substituted R1 R3 R1 R3 Heteroatomstabilized

Carbocations: Stability
Hydride ion affinities (HI) Me CH2 27 Me2 CH 249 CH2 276 37 +21 H 2C CH 287 20 +81 18 Me3 C 231 HC C 386

Chem 206

O R2

R R3

R
N R2

276 H 3C

R2

RR3 = alkyl or aryl

RR3 = alkyl or aryl

RR3 = alkyl or aryl

The following discussion will focus on carbocations unsubstitutred with heteroatoms

Classical vs nonclassical carbonium ions


C C

C C

Ph
C

C C

CH2

Me

CH2 276

C C symmetrical bridging

H 2C

CH 256

CH2

239

open trivalent

hyperconjugation no bridging

unsymmetrical bridging

The effect of beta substituents: Rationalize Me CH2 7 MeCH2 CH2 270

increasing nonclassical character classical nonclassical

276

Stability: Stabilization via alkyl substituents (hyperconjugation) Order of carbocation stability: 3>2>1
R R C R H > R C R H > H C R H > H C H
Hydride ion affinities CH3+ CH3CH2+ (CH3)2CH (CH3)3C
+ +

Hydride ion affinities versus Rates of Solvolysis PhCH2Br rel rate HI -HI 100 239 0 CH=CHCH2Br Me2CHBr 52 256 +17 0.7 249 +10

Due to increasing number of substituents capable of hyperconjugation

The relative stabilities of various carbocations can be measured in the gas phase by their affinity for hydride ion. R + H RH + HI

314 276 249 231 287 386 239

Relative Solvolysis rates in 80% EtOH, 80 C


A. Streitwieser, Solvolytic Displacement Reactions, p75

Hydride Affinity = G HI increases C(+) stability decreases Note: As S-character increases, cation stability decreases due to more electronegative carbon.
J. Beauchamp, J. Am. Chem. Soc. 1984, 106, 3917.

H2C=CH+

C+

PhCH2+

Conclusion: Gas phase stabilities do not always correlate with rates of solvolysis

Carey & SundbergA, pp 276-

31-01 Carbocations-1 12/3/03 8:08 AM

M. Shair, D. Evans

Carbocation Generation & Stability

Chem 206

Carbocation Stability: The pKR+ value


Definition: R + + H 2O KR+ = ROH + H+

Removal of an energy-poor anion from a neutral precursor via Lewis Acids

R 3C X aROH aH+ aR+ aH2O a = activity

LA

R 3C

LAX

pKR+ = log KR+

Carey & Sundberg, A, p 277

LA: Ag , AlCl3, SnCl4, SbCl5, SbF5, BF3, FeCl3, ZnCl2, PCl3, PCl5, POCl3 ... X: F, Cl, Br, I, OR Acidic dehydratization of secondary and tertiary alcohols

Table: pKR+ values of some selected carbenium salts


(4-MeO-C6H4)3C 0.82 CH2 Fe 0.40 H7C3 + H7C3 7.2 C 3H 7 4.77 most stable Fe 0.75 Ph3C 6.63 CHPh (3-Cl-C6H4)3C 11.0 Ph2CH 13.3 least stable

R3C OH + HX

- H2 O

R 3C

CHPh R Cr(CO)3 10.4

CPh2 Co2(CO)6 7.4

R: Aryl + other charge stabilizing substituents X: SO42-, ClO4-, FSO3-, CF3SO3From neutral precursors via heterolytic dissociation (solvolysis) - First step in SN1 or E1 reactions R 3C X
solvent

R 3C

+ X

Carey & Sundberg, A, pp 276-

Carbocation Generation
Hydride abstraction from neutral precursors

Ability of X to function as a leaving group: -N2+ > -OSO2R' > -OPO(OR')2 > -I -Br > Cl > OH2+ ... Addition of electrophiles to -systems
R R R R H H H R R R R H Provide a Mechanism of this transformation H2SO4
Br

chemistry
R

R 3C H

Lewis-Acid

R 3C
R RS RS R2N R 2N

H R 3C H = H

chemistry

H H

H H

H H

etc.

Lewis-Acid:

Ph3C BF4, BF3, PCl5

HC C CH2OH

Me O

J.C.S.,CC 1971, 556

31-02-carbocation-2 12/3/03 8:14 AM

D. A. Evans, B. Breit

Carbocations: Structure

Chem 206

Carbocation Stabilization Through Hyperconjugation


R H H C

Physical Evidence for Hyperconjugation: The Adamantyl Cation Bonds participating in the hyperconjugative interaction, e.g CR, will be lengthened while the C(+)C bond will be shortened. First X-ray Structure of an Aliphatic Carbocation 1.431

+
C

R+ H H H C C H H

FMO Description Take linear combination of CR (filled) and C pz-orbital (empty):


E

+
100.6 1.608
Me

[F5SbFSbF5]
C

+
Me

CR

CR

Me

+
C

H H

+
C

H H

CR

CR

T. Laube, Angew. Chem. Int. Ed. 1986, 25, 349

Syn-planar orientation between interacting orbitals

CH versus CC Hyperconjugation: The t-Pentyl Cation

Calculated carbocation agrees with solution structure

The Adamantane Reference (MM-2) 1.528


H

Me H H C

+
C

Me Me

1.582

1.092

110
Me Me Me

1.530

+
1.107

R. P von Schleyer in Stable Carbocation Chemistry, 1997, p 46-47

31-03 Carbocations-3 12/3/03 8:14 AM

D. A. Evans, K. Scheidt

Carbocations: Structure

Chem 206

Me
[F5SbFSbF5]
C

Me Me

SbF5

Me Me

+
C Me

F5Sb F

SbF5

Me Me

2 SbF5
Me

Me Me

Me

1.466

1.421

+
1.622

1.439

CC1: 1.439 CC2: 1.446 CC3: 1.442

1.621

98.2 1.608

F5Sb F

SbF5

1.551

T. Laube, Angew. Chem. Int. Ed. 1986, 25, 349

1.508 reference structure: CSP3Csp2 bond length 1.342

T. Laube, JACS 1993, 115, 7240

31-04 Carbocations-4 12/3/03 8:15 AM

D. A. Evans, K. Scheidt

Carbocations: Structure

Chem 206

Cl Cl

Cl SbF5

Cl

Cl + SbClF5

Me Me Ph F

SbF5

Me Me

+
C Ph +

SbF6

122 1.668 Cl 1.432 1.408

+
1.422 Cl 1.725 1.371

+
1.432 1.491

1.442 121.0

117.8 1.446

+
1.439

1.505

121.2
1.508

T. Laube, JACS 1993, 115, 7240


reference structure: CSP2Csp2 bond length 1.342 reference structure: CSP3Csp2 bond length

31-05 Carbocations-5 12/3/03 8:15 AM

D. A. Evans, K. Scheidt

Carbonium Ion X-ray Structures: Bridged Carbocations


Ph Me
[F5SbFSbF5]
Me H Me Me

Chem 206
Ph + C AgSbF6

Cl

Me

Me

Me Me F5Sb F SbF5

+
Me H Me Me

Me

2 SbF5

T. Laube, Angew. Chem. Int. Ed. 1987, 26, 560

2.092

1.467 1.739 **

1.495

+
1.467

+
1.442

1.855

**One of the longest documented CC bond lengths.


C

1.503

C C

C C

hyperconjugation no bridging

unsymmetrical bridging

T. Laube, JACS 1989, 111, 9224

31-06 Bridged cations 12/3/03 8:15 AM

D. A. Evans, K. Scheidt

Carbonium Ion X-ray Structures: A Summary

Chem 206

(ref 1.513 )PhC(Me)=CH2 1.442 1.446 1.408 + 1.432

+
1.439

2.092

1.739

1.371

1.491

1.467

+
1.442

1.508 1.668 1.342 Cl 1.432

1.422 Cl 1.466 1.421 1.725 1.467 1.495

+
1.855 1.621 98.2 1.608 1.622

Nomenclature: classical vs nonclassical 1.551


C C

1.503
C

C C

C C

open trivalent

hyperconjugation no bridging

unsymmetrical bridging

symmetrical bridging

increasing nonclassical character classical nonclassical

31-07 C(+) Summary 12/3/03 8:16 AM

D. A. Evans, B. Breit

Vinyl & Allyl Carbocations


Allyl & Benzyl Carbocations Carbocation Stabilization via -delocalization

Chem 206

Vinyl & Phenyl Cations: Highly Unstable


Evidence suggests that vinyl cations are linear.
OTf
HOSolv

OSolv R R R R

As ring size decreases, the rate of hydrolysis also diminishes. Implying that the formation of the linear vinyl cation is disfavored due to increasing ring strain.
R D R OTf
H+

allyl cation

R D

C C

Hyperconjugation

Stabilization by Phenyl-groups The Benzyl cation is as stable as a t-Butylcation. This is shown in the subsequent isodesmic equations: H0r [kcal/mol] (CH3)3C + PhCH3 (CH3)3C + PhCH2Cl (CH3)3CH + PhCH2 (CH3)3CCl + PhCH2 3.8 - 0.8

A secondary kinetic isotope effect was measured to be KH/KD = 1.5 (quite large) indicating strong hyperconjugation and an orientation of the vacant p orbital as shown above.
P. J. Stang J. Am. Chem Soc. 1971, 93, 1513; P. J. Stang J.C.S. PT II 1977, 1486.

Hydride ion affinities (HI) +21 +81

H 3C

CH2 276

H 2C

CH 287

HC

C 386 Hydride ion affinities (HI) Ph CH2 8 Me3 C 231

239 H 2C CH 287 298 The ring geometry opposes rehybridization (top) so the vacant orbital retains sp2 character. Additionally, the empty orbital lies in the nodal plane of the ring, effectively prohibiting conjugative stabilization. PhCH2Br 100 HI -HI 239 0 Me2CHBr 0.7 249 +10 CH=CHCH2Br 52 256 +17 +11

Phenyl Cations

Hydride ion affinities versus Rates of Solvolysis

Relative Solvolysis rates in 80% EtOH, 80 C


A. Streitwieser, Solvolytic Displacement Reactions, p75

31-08 Vinyl/allyl (+) 12/3/03 8:17 AM

D. A. Evans, B. Breit

Cyclopropyl-carbinyl & Bridgehead Carbocations

Chem 206

Carbocation Stabilization via Cyclopropylgroups

Solvolysis rates represent the extend of that cyclopropyl orbital overlap contributing to the stabiliziation of the carbenium ion which is involved as a reactive intermediate:
OTs Me Me OTs Cl

See Lecture 5, slide 5-05 for discussion of Walsh orbitals Me

krel = 1
OTs

krel = 1
OTs

krel = 1
Why??

A rotational barrier of about 13.7 kcal/mol is observed in following example:


H

NMR in super acids Me (CH3) = 2.6 and 3.2 ppm O

Cl 1.517 1.478 1.222

X-ray Structures support this orientation 1.302

krel = 106

krel = 108

krel = 10-3
CareyA, p 286

R
1.474 Me

Bridgehead Carbocations

1.464 1.541 1.409

Me Me

OTs TsO TsO TsO

why so reactive?

1.444 1.534

10-7

10-13

104

Bridgehead carbocations are highly disfavored due to a strain increase in achieving planarity. Systems with the greatest strain increase upon passing from ground state to transition state react slowest. 24 why so reactive?

TsO

TsO

R. F. Childs, JACS 1986, 108, 1692

31-09 Cyclopropyl-carbinyl C+ 12/3/03 9:11 AM

D. A. Evans, J.Tedrow

A Stable Hypervalent Carbon Compound ?

Chem 206

"The Synthesis and Isolation of Stable Hypervalent Carbon Compound (10-C-5) Bearing a 1,8-Dimethoxyanthrecene Ligand" Akibe, et al. JACS 1999, 121, 10644-10645 +
Me

OMe

CO2Me OMe

Me

MeO O C

OMe O

Me3O+BF4

B2F7

"The relevant CO distances are longer than a covalent CO bond (1.43 ) but shorter than the sum of the van der Waals radii (3.25 )."

2.428

+
2.428 2.452 1.483

+
2.452

For a recent monograph on hypervalent Compounds see: "Chemistry of Hypervalent Compounds", K. Akiba, Wiley-VGH, 1999

31-10-5 coord (C+) 12/3/03 8:17 AM

D. A. Evans, J.Tedrow

Transition Metal-Stablized Carbocations-1

Chem 206
Ph

Carbocation Stabilization - d( ) stabilization via Transition metal Fragments


Transition metal not bound directly to carbenium ion: Ferrocenes Fe
C CH CF CO H 3 2 C CH 2 stable in CF3COOH 1+ 1.376 1.391 an important resonance contribution 1.509 1.385

+
Ph

Fe

Fe

C Fe

R R Fe

R R

Cyclopropene MM 2
R. L. Sime, etal. JACS 1974, 96, 892 1.389 1.428 1.297

+
1.412

Transition metal not bound directly to carbenium ion: Colbalt-Acetylenes


R OH R R' R''
1) Co2(CO)8 2) HX

R C (OC)3Co C R

Co(CO)3

isolable, basis of the Nicholas reaction


J. Lukasser, etal. Organometallics 1995, 14, 5566-5578

See Houk etal. JACS 1999, 121, 3596-3606

31-11-transition-M C(+) 12/3/03 8:18 AM

D. A. Evans, J. Tedrow

Transition Metal-Stablized Carbocations-Pd Catalysis


AcO
Nu, CH2Cl2, -20 C

Chem 206
Nu = BnNH

Carbocation Stabilization - d( ) stabilization via Transition metal Fragments


Transition metals bound to carbenium ions: Allyl Pd(II) Complexes
R L4Pd(0) inversion OAc R OAc OAc Ph Ph Pd L Nu Ph 98% ee 95% ee Ph Ar2P O S R
1a Ar = Ph

Nu

Nu = CH(CO2Me)2

94% ee
Nu

91% ee
O S CMe3 1b, Ar = -naphthyl Me CHMe2

1+ Nu inversion L

OAc

1b
Nu, CH2Cl2, -20 C

94% ee
Nu

91% ee

Ar2P

Nu OAc Me CHMe2
Nu, CH2Cl2, -20 C

2 mol% [C3H5-PdCl]2, 1 Nu, CH2Cl2, 20 C

96% ee
Nu

97% ee

OCO2Et

1b
Nu, CH2Cl2, -20 C

Nu = CH(CO2Me)2 Nu = BnNH

94% ee

94% ee
conditions: 2 mol% [C3H5-PdCl]2, 2b

BocN Me O P S Pd H CMe3 CHMe2 Ph Ph C1 O P Pd S Me CHMe2 CMe3

BocN

Ph

P S

Ph

Ph C1

C2 Ph

C2

Selected Bond Lengths: Pd-C1, 2.16; Pd-C2, 2.28 Pd-P, 2.29 ; Pd-S, 2.38

Selected Bond Lengths: Pd-C1, 2.17; Pd-C2, 2.26 Pd-P, 2.25 ; Pd-S, 2.36

C1

C2

Pd-C1, 2.16

Nu C1
Pd-C2, 2.28
Evans, Campos,Tedrow, Michael, Gagn, JACS 2000, 122, 7905

C2

Pd-C1, 2.17

Nu Pd-C2, 2.26

31-12-transition-M C(+)-2 12/3/03 8:18 AM

D. A. Evans

Carbocations: Stability, Structure, & Rearrangements

Chem 206

http://www.courses.fas.harvard.edu/~chem206/

Here is a typical carbonium ion question that you should be able to handle by the end of the course. Write out a mechanism for the following transformation. Me Me
BF3OEt2

Me H O

Chemistry 206 Advanced Organic Chemistry


Lecture Number 32
Me Me Me O CH2

Me

CH2Cl2

60% yield

OBF4 Me

Me

Introduction to Carbonium Ions2


Allyl- & Vinylsilanes: The -Silicon Effect Carbonium Ion Rearrangements
Me Me

Me OBF4 A. Srikrishna, Chem Commun 1994, 2259 Question 13. Final Exam, 1999. During Corey's synthesis of Aspidophytine (JACS, 1999, 121, 6771), the pivotal intermediate 3 was assembled by the union of 1 and 2 under the specified conditions. Provide a mechanism for this single-pot transformation. CHO OHC CO2R temp, 5 min Me3Si
2 1) mix at room 2) 2 equiv.

Reading Assignment for this Lecture:


Carey & Sundberg, Advanced Organic Chemistry, 4th Ed. Part A Chapter 5, "Nucleophilic Substitution", 263-350 .
Walling, C. (1983). An Innocent Bystander Looks at the 2-Norbornyl Cation. Acc. Chem. Res. 16: 448. (handout) Birladeanu (2000). The Story of the Wagner-Meerwein Rearrangement. J. Chem. Ed. 77: 858. (handout) Lambert, (1999). The effect of silicon and related manifestations of conjugation. Acc. Chem. Res. 32, 183-190. (handout)

Other Relevant Background Reading


Saunders, M. and H. A. Jimenez-Vazquez (1991). Recent studies of carbocations. Chem. Rev. 91: 375. MeO

NH2 TFAA, 0 C

Ra N Rc

Rb Rd

3) excess NaBH3CN

N CO2R MeO MeO

D. A. Evans
32-00-Cover Page 12/5/03 8:52 AM

Friday, December 5 , 2003

N OMe Me N Me H
3

M. Shair, D. Evans

Carbocation Rearrangements-1

Carbocation [1,2] Sigmatropic Rearrangements


1,2 Sigmatropic shifts are the most commonly encountered cationic rearrangements. When either an alkyl substituent or a hydride is involved, the term Wagner-Meerwein shift is employed to identify this class of rearrangments. Birladeanu (2000). The Story of the Wagner-Meerwein Rearrangement. J. Chem. Ed. 77: 858. (handout)

Pinacol rearrangement (vicinal diol): Driving force is the gen. of C=O


OH
H
+

OH CH2 NH2
Deamination " NO+ "

OH CH2

Stereoelectronic requirement for migration.... retention of stereochemistry

Demjanov-rearrangement (Driving force: relief of ring strain) A B C D A C D B

bridging T.S.

Wagner-Meerwein Rearrangements: Application in Total Synthesis


Me H Me
H2SO4

Me H Me Me Me

A B

C D

Me Me OH H A Me H Me
-caryophyllene alcohol

2-electron Huckel transition state


If migration accompanies ionization, the migration terminus will be inverted. Overlap between the C-C (migration origin) and the * C-X (migration terminus) will be maximized in an antiperiplanar arrangement. H OH RO H RO H OH H H O Me HO

H Me Me H

equiv to

OH Me H Me Me Me

Preparation of A: MsO H TBSO H OH H OH


Et2 AlCl CH2 Cl2, -78C

Me H Me Me h [2+2] O H H Me Me
MeMgBr

32-01-carbocation Rearrang-1 12/5/03 11:13 AM

S. L. Schreiber et al Tetrahedron Lett. 1989, 30, 3765.

M. Shair, D. Evans

Carbocation Rearrangements-2

Chem 206

Synthesis of ()-Isocomene: Pirrung, JACS 1979, 7130; 1981, 82.


Me H+ Me Me Me Me Me Me Me Me ()-Isocumene OTs Me Me Me Me Me Me OTs HOAc OTs
non-classical carbonium ion

krel = 95

R. G. Lawton, JACS 1961, 2399

krel = 1 Cram, JACS 1949, 71, 3865


H AcO H Me H Ph Me OAc H H 98% chemical fidelity Ph TsO Me AcO Me H H Ph Me OAc Me H 98% chemical fidelity Ph Me H L-Erythro H L-Erythro Me Me D-Threo Me L-Threo

Carbocations: Neighboring Group Participation s Groups with accessable electron density (heteroatoms, arenes) and the correct stereoelectronic orientation (anti-periplanar) can "assist" in the ionization of a leaving group.
X: R R X R R Y R R
Nuc:

The Cram Phenonium Ion Experiments:


TsO OTs H Me Me Me

R R
Y-

X: R R

R R Nuc

H H Ph

L-Threo

OTs
+3

H Me

krel = 10

OTs O O Me

HOAc

O Me O

Me Me Ph

L-Erythro

Physical Evidence for Neighboring Group Participation


krel = 1
OTs O O Me HOAc OAc O O Me Cl HF-SbF5 SO2 ClF tetrahedral carbon in C13 NMR (68.8 ppm) Phenonium ion G. Olah JACS 1976, 98, 6784.

xx-02-carbocation Rearrang-2 12/5/03 9:19 AM

See Lowry & Richardson, pp 434-439 for discussion of this controversy

M. Shair, D. Evans

Allyl- & Vinylsilanes: The -Silicon Effect


Lambert Acc. Chem. Res. 1999, 32, 183-190
Lambert, JACS 1990, 112, 8120; 1996, 118, 7867. Fleming, Organic Reactions 1989, 37, 54. Fleming, Chem. Rev., 1997, 2063.

Chem 206

References:

Magnitude of the -Silicon Effect


SiMe3 Me3C H 1 H Me3C H 3 H SiMe3 Solvolysis (CF3CH2OH) k3 k4 Me3C = 4 x 10+4 H 4 H H H Solvolysis (CF3CH2OH) k1 k2 = 2.4 x 10+12 Me3C H 2 Me H H

Allyl & Vinylsilanes react with electrophiles


R3Si SiMe3 E E

(trapped by solution Nu) E "R3Si+" "R3Si "


+

OCOCF3

OCOCF3

H Me

Mechanism - the simple picture: -Silicon stabilizes carbocation


R3Si E Nu R3Si E Nu SiMe3 E Fleming, Organic Reactions 1989, 37, 54. E

OCOCF3

OCOCF3

"These figures established the -effect as one of the kinetically strongest in organic chemistry": J. Lambert Data provide no distinction between open and bridged intermediates

SiMe3 E

H 2C

-Silicon Effect: the origin of regioselectivity

Proof for a stepwise mechanism provided the following protodesilylation experiment:

Si

occ pz

SiC pz empty
SiC

pz

Me3Si

SiMe2Ph

H
Me3Si SiMe2Ph Me3Si

SiMe2Ph

Me3Si H3Si H 3C H CH2 H

SiMe2Ph

CH2 H

versus

both silanes

Calculation: A more stable than B by 38 kcal/mol.


Jorgensen JACS 1985, 107, 1496.

yield the same product mixture. Hence, the reaction proceeds most likely via a common intermediate, a carbeniumion

32-03-beta Si-Effect 12/5/03 8:20 AM

M. Shair, D. Evans

Allyl- & Vinylsilanes: The -Silicon Effect-2


trans-alkene: anti-addition of E+ with respect to SiR3 Examples:
Ph t-BuCl, TiCl4 H Me CH2Cl2 Ph t-Bu H Me

Chem 206

General: Allylsilanes are more nucleophilic than alkenes HOMO is higher in energy due to negative hyperconjugation

The stereochemical consequences for the major product are:

( *)

CSi

Me3Si

Houk, JACS 1982, 104, 7162.

SiC SiC

But
Ph H HF R Protodesilylation

JACS 1982, 104, 4962. Ph

Me3Si

E:Z 88:12! Eschenmoser, Helv. Chim. Acta 1979, 62, 1922.

Electrophile Addition - Stereoelectronics


R3 R
3

Carbonyl Addition of Allylsilanes: Open Transition States


R2

Si

H
1

Me3Si is not sufficiently Lewis acidic to activate C=O through pre-association; however (RO)2MeSi is Lewis acidic enough to activate C=O through pre-association. These allylsilanes add to RCHO througl closed transition states X nM R1 O R2 R R1
OR

CSi
R

R Si

R H

CSi

R1

R R2

A1,3-strain minimized

Antiperiplanar TS
H

Synclinal TS
O X nM H

anti addition observed


R2 R3

E E R R2 R1 R
+ Nu - NuSiMe3

SiMe3

SiMe3

Si

Calculations by Houk et al. show that the relative energy differences between the antiperiplanar and and synclinal transition states are negligible. Both the antiperiplanar and synclinal models predict a syn selectivity for the newly formed stereogenic centers. O H + Me3Si Me TiCl4 CH2Cl2 R Me O R H + Me3Si Me Hayashi, TL 1983, 2865. TiCl4 CH2Cl2 R OH R Me Me OH OH > 95:5 syn

R3

CSi
R H
+ Nu

R E

H Si

- NuSiMe3

R3 R H E

R21 R

R3 H R

E R2 R1

syn

major (trans)

minor (cis)

ca. 65

35

32-04-beta Si-Effect-2 12/5/03 8:26 AM

Catalytic Enantioselective Addition of Allylic Organometallic Reagents to Aldehydes and Ketones, Denmark and Jiping Fu, Chem. Rev. 2003, 103, 2763-2793 (handout)

B. Breit, D. Evans

Allylsilanes:

Reactions with Electrophiles


Reactions Proceedilng through Silicon-Migration

Chem 206

Allylsilanes add to aldehydes and acetals under Lewis acid promotion


O Me3Si Ph + H O Me3Si Ph + H Me Me TiCl4 Ph OH TiCl4 Ph n-C3H7 O OH Me n-C3H7 (Pri)3Si

Si migration may be promoted by using hindered Si substituents OTiCl4 TiCl4 CH2Cl2 Si(iPr)3 Si (iPr)3 Me ?? O Me Me OTiCl4

regioselectivity: Allyl inversion

Felkin Selectivity also holds with this class of nucleophiles


A. I. Meyers, J. Org. Chem. 1998, 63, 5517

Acetals can be used as well


Me3Si Me Me Me3Si Me Me + OCH3 + H3CO n-C4H9 OCH3 H3CO n-C4H9 OCH3 TiCl4
(80%)

diastereoselection: 97:3 (Pri)3Si 85% yield n-C4H9 Me Me OCH3 Me Me O Me diastereoselection: 96:4 68-70% yield 75% SiMe3 Me3Si Me O n-C4H9 MeO2C Ar (Pri)3Si Ph(Pri)2Si MeO2C MeO2C Ar SiR3 CO2Me ZrCl4 CH2Cl2 Ar SiR3 MeO

OZrCl4 CO2Me MeO

OZrCl4 CO2Me Ar SiR3

TiCl4
(83%)

The Sakurai Reaction (Enone Conjugate Addition)


OTiCl4 O Me Me3Si TiCl4 CH2Cl2 Me

Can you work out the mechanism??


Me PhMe2Si O CO2Me Me H BF3OEt2 rt 8 h PhMe2Si Me CO2Me Me CHO diastereoselection: >30:1 93% yield Panek, J. Org. Chem. 1993, 58, 2345

Fleming, Org. Reactions 1989, 37, 127-133

17% Me 78% O

Me

SiMe3

EtAlCl2 CH2Cl2

Majetich, Tetrahedron 1987, 43, 5621

32-05-allyllsilanes 12/5/03 8:27 AM

B. Breit, D. Evans

Vinylsilanes: Reactions with Electrophiles


Summary Statements

Chem 206

Stereochemistry of Electrophile Addition to Vinylsilanes


Vinyl/Allylsilanes in Organic Synthesis - Selected Examples
Fleming, Org. Reactions 1989, 37, 54. O N O I H Me R1 R2 SiMe3 H I
-

1. Me3C+ is more stable than Me3Si+ in spite of the fact that Si is less electronegative than C.
H Si Me C H Me Me C C H Me H Me C C H Me Si Me H H C H H CSi bond length: 1.87

R1 R2 RETENTION

El El
R1 H Me3Si R2

Rotation in direction a favored (avoidance of eclipsing interactions, Principle of least motion


El El H R1 R2 R1 H SiMe3 R2 SiMe3

+ Nu - NuSiMe3

CC bond length: 1.54 H H H CSi hyperconjugation is less pronounced than the anaologous CC hyperconjugation do to the impact of the longer CSi bond lengths.

H R1

El R2 SiMe3

2. Carbonium ions to Si are less stabilized than carbonium ions to Si.


Me Me C H H Me Si Me Me3Si C H C H H H H C C H H C H H

( C(+) to silicon

Me Si Me Me3Si

Et Et SiMe3 + ClCH(OMe)2 TiCl4


- 78 C (73%)

Et Et OMe OMe

( C(+) to silicon

R TiCl4 R Me H Me OH SiMe3 Me O Me

CSi hyperconjugation is less pronounced than the anaologous CC hyperconjugation do to the impact of the longer CSi bond lengths.

Fleming p 289

3. According to Lambert, silicon has a propensity to stabilize carbonium ion via hyperconjugation (vertical stabilization) rather than bridging (nonvertical stabilization.
Me3Si Me3Si C H C H H Me3Si H H C C H H H H C C H H

( C(+) to silicon
N H Fleming p 148 Me Me NH SiMe3 (CH2O)n TsOH N H N

hyperconjugation more important than bridging

32-06-allyl/vinylsilanes 12/5/03 8:31 AM

4. Silicon has a lower propensity to undergo WagnerMeerwein like rearrangements than carbon.

M. Shair, D. Evans
R1 R3 N R2 R4

Stabilized Cations: Iminium-Ions 1


Ph

Chem 206

Iminium Ions

X-

TFA

Me3Si
(Z)

N rel rates: 7000/1 Ph

Ph

Common Methods of Generation:


R1 R2 OR2 N R1 R3 R4
H+, -H2O or Lewis Acid

N H H

H N

R1 N R2

R3 R4

(E)

TFA

N Me3Si Ph

Overman et al. TL 1984, 25, 5739.

H+, -ROH

Ph H N H SiMe3 N H H H

N
or Lewis Acid

R1 H

Oxidation of Amines
N HgX2 Me X H

Hg

X rds

Hg(0) Me N H HX

SiMe3

(Z) vinylsilane)

(E) vinylsilane)

N H Me

Only in the case of the (Z) vinylsilane is the emerging p orbital coplanar with C-Si bond. Full stabilization of the empty orbital cannot occur with the (E) vinylsilane.....hence the rate difference.

Stereoelectronic Effects on Nu Addition to Iminium Ions


N N H H MeO2C R N H H H H H CO2Me OH Nu (favored) OH N H H H MeO2C one diastereomer Stork et al. JACS 1972, 94, 5109. OH N
71 %

N
Hg(OAc)2/EDTA NaBH4

O Me

TMS Me R

Me
PPTS, MeOH 80C

TMS N H MeOH

H N H

Me

R steps

pumiliotoxin A

Me

OH

one double bond isomer "Least Motion Argument"

C=N Stereoelectronic Effects: Lecture 20

Overman et al. JOC 1989, 54, 2591.

32-07-iminium ions 12/5/03 8:34 AM

D. A. Evans, M. Calter
Review:

The Aza-Cope Rearrangement The 3-aza-Cope Rearrangement


First Neutral Case: Hill TL 1967, 1421.

Chem 206

Heimgartner, H. In "Iminium Salts in Organic Chemistry"; Bohme, H., Viehe, H., Eds.; Wiley: New York, 1979; Part 2, pp 655-732. The 3-aza-Cope Rearrangement: Neutral Variant:
R
3 2
1

Me R
1

Me N Me

250oC, 1 hr

Me

Me N "Practically quantitative", no real Me yields given.

[3,3]

Exothermic as written by ~7-10kcal/mole. First Cationic Case: Elkik Compt. Rend. 1968, 267, 623.

Ammonium Variant: p. p.
R R N
[3,3]

R R

Even more exothermic than the neutral version, since enamine lacks resonance and iminium salt has stronger p-Bond than imine does.

Me + N Me

Me Me

80 oC, 2-3 hr

Me Me

Me Me
No yields given.

H2O

2-aza-Cope Rearrangement:
2

OHC

Me Me

R N
1

[3,3]

R N

In the simplest case, degenerate. Steric effects, conjugation, or selective trapping of a particular isomer, will drive equilibrium. As with the 3-aza-Cope, the cationic version proceeds under much milder conditions.

Good way to allylate aldehydes: Opitz Angew. Chem. 1960, 72, 169.
R'' N R'' H
+

OHC R

R'

-H2O

R''

N R'' R

R'
+

1-aza-Cope Rearrangement:
1

R'''

[3,3]

2
1

The 3-aza-Cope rearrangement can be driven in reverse by judicious choice of substrates(i.e., incorporating the imine into a strained ring or by making R an acyl group).

H O

R'
H 2O

R'' R''

R' N R R'''
[3,3]

R'' R''

R' N R R'''

R R'''

32-08 Aza Cope-1 12/5/03 8:41 AM

D. A. Evans, M. Calter

The Aza-Cope Rearrangement

Chem 206

The 2-aza-Cope Rearrangement


First Reported Case: Horowitz JACS 1950, 72, 1518.
H Ph NH2
HCHO

Mechanism for Yohimbane Analog Formation:


NH H2N
HCHO, H+, -H2O

H
2-aza-Cope

Ph

Ph

H 2O +

N H

HCOOH 100oC, 2hr.

N H 2-aza-Cope, driven by conjugation N N

PhCHO

Equilibrium between A and B driven towards B by conjugation of iminium double bond to the aromatic ring in B. Application to Yohimbine Analog Synthesis: Winterfeldt Chem. ber. 1968, 101, 2938. N H H OH CO2Me H OMe N H

p. p.

MeOH

..

N H

N H

N-Acyliminium Ion Rearrangements: Hart JOC 1985, 50, 235.


Hart observed an unusual product while trapping the intermediates of N-acyliminium olefi cyclizations. OH
TFA

Yohimbane

Yohimbine

NH
POCl3

NH
NaBH4

N C 3H 7 O

N C 3H7 O CF3CO2 N Et3SiH C 3H 7 N O

O N H

N H
NaBH4

HCHO, MeOH, Cat. H+, 85%

NH

C 3H7

O 2-Aza Cope rearrangements add to complexity of cyclization process

N H

OMe N H N C 3H 7 O

15-Methoxy-isoyohimbane

N 40:60 ratio C3H7 O

32-09 Aza Cope-2 12/5/03 8:43 AM

M. Shair, D. Evans

Stabilized Cations: AcylIminium-Ions


OH N AcO NH2
+

Chem 206

N-Acyliminium Ion Rearrangements


Synthesis of (-)-hastanecine: Hart JOC 1985, 50, 235.
O

SiMe3 TFA Me O N Me ??? H N

SiMe3

H 67% O N Me

CH2

Me BnO

Me

O
81%

The origin of the modest diastereoselection has not been attributed to 2-aza-Cope process.

[3,3]

SiMe3

H N O Me

CH2

BnO OBn Me Me O [3,3] BnO OBn Me Me O N H OAc Me Me N O OAc


HCO2H HCO2H

BnO OH N O
HCO2H

29% Me

OAc
NaBH4, MeOH, 83%

O Me Me N

OAc Gelas-Mailhe, Tet. Lett, 1992, 33, 73 O

N OAc

Me Me

Competing 2-Aza-Cope and Pinacol Rearrangements: Which Dominates??


OH Ph Ph cyclization N Me Me N Me Pinacol Ph Ph OH Mannich N Me Ph Me N Me O
:OH

Ph

BnO Me HCO2 Me N

OAc
CSA, 60oC, 1.5 hr, 79%

Ph

O O

Ph

Ph

Me

[3,3]
Me

HO

OH HO

BnO Me

OH

N Me

Homo-chiral

Ph

(-)-hastancine

Me

N O Me

racemic product
Conclusion: 2-aza-Cope rearrangements afford a low-barrier to competing processes

32-10-iminium ions 12/5/03 8:46 AM

M. Shair, D. Evans

Stabilized Cations: Iminium-Ions 2 Another aza-Cope-Mannich sequence:


OR O O CH2O/HCl NHBn NR2 O N Bn O O Axial Attack 98 %!! Mannich Rxn H2/Pd-C CH2O/HCl O H HO [3,3] Mannich 97% N BF3 Ar OH

Chem 206

2-Aza-Cope-Mannich sequence:
OR OR

N H HO

(CH2O)n, Na2SO4 MeCN, 80C HO NR2

[3,3] HO NR2

HO

Ar

N Bn [3,3]

Ar

ROH2C O N O

N H Bn

Bn

O NR2 steps

OR O O NR2 N H

O H 67% Pictet-Spengler cyclization

O H O N steps O HO O N Pancracine

equivalent to

N H N O H HO
strychnine Overman et al. JACS 1995, 117, 5776. Overman et al. JOC 1991, 56, 5005

HO

32-11-iminium ions-2 12/3/03 5:15 PM

D. Evans, E. Shaughnessy

The Prins-Pinacol Reaction References


Evidence for Prins-Pinacol Mechanism
O Me O O Me Me SnCl4, CH2Cl2 Me Me O Ph Me Me

Chem 206

Prins reaction: Adams, D.R.; Bhaynagar, S. D. Synthesis 1977, 661 Prins & carbonyl ene reactions: Snider, Comprehensive Organic Synthesis, 1991, Vol. 2

Ph

The Prins Process:


O R2 R1 H HX R1 X O R1 H R2 H R1CHO R2 O R1 O R2 R1 OH R2
-

Me X R2 Ph
-

>95% ee

OH

Me Me O

Ph

Me O Me

Cl4SnO Me Me

Prins

OH R1 - H+ R2

Cl4SnO Me Me

[3,3]

pinacol
O Ph

enantiopure

Ph H Cl4SnO Me Me O

Me Me

Aldol (fast) racemic

Me Me O

Me Me

The Prins-Pinacol Variant:


Ph Me Me O O Me Me SnCl4 Ph H Me Me O Me Me Lewis Acid SnCl4 O Me Me O Ph Me Me Me

>95% ee

>95% ee

If a [3,3] rearrangement were intervening, the product would be racemic. Overman, JACS 2000, 122, 8672 Overman, Org Lett 2001, 3, 1225

Examples of Stereoselective THF Formation


O O O Me Me SnCl4, CH2Cl2 -70 -23 C 82% BF3OEt2 (E)-CH=CHPhCHO CH2Cl2, -55 C 97% Me Me O Ph Me Me O Me Me OH O

Pinacol Prins
Ph H Cl4SnO Me Me Me HO

Me

Cl4SnO

O Me Me

syn
Me

32-12-Prins-1 12/3/03 5:15 PM

7:1 anti:syn

D. Evans, E. Shaughnessy
Prins-Pinacol Mechanism
Ph SnCl4 CH2Cl2 Me Me Me O O Me
-

The Prins-Pinacol Reaction


Overman: Magellanine Synthesis
Me Me Prins O Me Me
-

Chem 206
Me N O (-)-Magellanine Me H OH

Ph Cl4SnO Me Me O

Me Me

JACS, 1993, 115, 2992

Cl4SnO

Ph

[3,3]
Ph H O MeMe Me Me Aldol

pinacol O Me Me O Ph

enantiopure

The pivotal transformation O TESO SnCl4 CH(OMe)2 57% 1. OsO4, HIO4 2. Ph2CHNH3Cl NaBH3CN (-)-Magellanine Me N O H CHPh2 N O OH Me Steps H OMe H OMe

Homo-chrial
Cl4SnO

Me Me

>95% ee

Prins cyclization faster than [3,3] rearrngement

2-aza-Cope vs. Pinacol:


CSA, 60oC, 1.5 hr, 79%

Ph

OH

Ph Ph cyclization N Me Me

:OH

Ph

Ph O N Me

Ph

Me

N Me Pinacol TESO CH(OMe)2 Ph SnCl4 TESO Me

[3,3]
Me Ph OH Me N Me Me Ph Mannich Ph O

Homo-chiral

N Me

O H OMe

O H OMe

racemic product

[3,3] rearrngement faster than Mannich cyclization


32-13-Prins-2 12/3/03 5:15 PM

mixture of diastereomers

D. Evans, E. Shaughnessy

The Prins Reaction-3

Chem 206

Overman Synthesis of a Eunicellin Diterpene


Overman & MacMillan JACS, 1995, 117, 10391 Me Me Me Me H HO AcO O

Overman: Synthesis of trans-Kumausyne


AcO

(-)-7-Deacetoxy-alcyoninacetate

JACS, 1991, 113, 5378


O Br Et

trans-Kumausyne
Me I t-BuLi, THF, -78 C Me Me OHC Me O Me OMe Me TMS Me OHC Me OH [3,3] O Me R Me TMS Me R Me O TMS 1. TMS Et BF3OEt2 -78 C rt 73% 2. TBSCl HO H O O OTBS Et O O 1. H2, Pd-C, 88% 2. Swern, 100% OTIPS Me OH H Me BF3OEt2 (3 equiv.) CH2Cl2, -55 -20 C 79% O H OBn O O m-CPBA 72% 4:1 regioselectivity O H OBn O H PPTS, MeOH 64% OH TMS OH Felkin Control (Lecture 20) Me OH BnOCH2CHO RSO3H, rt OH 69% OH [3,3] O OBn O OBn OH

Felkin Control (Lecture 20) O H Et OTBS DIBAL -78 C

Me single stereoisomer ds = 9:1

CHO O TMS

H CHO O H

O H

Me2HC Me OTIPS 6 steps, 39% yield from (S)-carvone

97%

32-14-Prins-3 12/3/03 5:16 PM

D. A. Evans

The Prins Reaction-4 Mukaiyama AldolPrins Cascade


Rychnovsky JACS, 2001, 123, 8420

Chem 206

Application to Leucasandrolide
Me OH O OMe O O O OMe O OH HO O O Me Me Me Me OH

The Basic Process


SiMe3 El R O R O El Me

The seco acid


5.5:1 ratio Me

El SiMe3

TMSX Me SiMe3 H O R O El O OBn TBSO O

The Pivotal Step:


SiMe3 BF3OEt2 base, 78%

Prins
R O El

OH

OBn

OTBS

Control of hydroxyl center: see Lecture 20 Let El(+) = Lewis acid activated RCHO
SiMe3 RCHO R O F 3B BF3OEt2 base O R SiMe3 O O R O BF3 R Me I BnO OTBS Myers, JACS 1997, 119, 6496 base = Me3C N CMe3 R O No (little) control over this () stereocenter HO R R BnO O TMS H O R BF3OEt2 R BnO OH O R

anti selection: ~58:1

Evans et al., JACS 1996, 116, 4322

Aldehyde Synthesis Chiral enolate alkylation: see Lecture 23


Me Me N OH Ph BnO OTBS O Me LDA Me Me N OH Ph H+ 77% O OBn Me H O H CH2 OBn SiMe3 BF3OEt2 O OBn OAc Me O DIBALH Ac2O

aldol
H SiMe3

Prins
R O

OX R

diastereoselection >20:1 Me

32-15-Prins-4 12/3/03 5:16 PM

cyclic oxo-carbenium ion addition: see Lecture 19

Donald J.Cram (19192001)

Phenonium Ion Experiments:

Cram, JACS 1949, 71, 3865


TsO H AcO H Me H Me Ph Me OAc H H Me Me

OTs H H Ph Me Me

H Me

98% chemical fidelity

Cram: "Just remember Dave, old deadwood is better than young deadwood."

Ph

"A View from the Far Side. Memorable Characters and Interesting Places." Evans, D. A. Tetrahedron 1999, 55, 8589-8608.
Quotes-7 12/5/03 9:31 AM

D. A. Evans

Iminium Ions and Their Transformations


CHO OHC
1 1) mix at room temp, 5 min

Chem 206
OR N RO N OMe Me TFA O OR TMS OR

http://www.courses.fas.harvard.edu/~chem206/

Chemistry 206 Advanced Organic Chemistry


Lecture Number 33
Me3Si
2

CO2R

MeO

Introduction to Carbonium Ions3


Stabilized Carbocations: Iminium Ions (C=NR2(+)) Stabilized Carbocations: OxoCarbenium Ions (C=OR(+)) Stabilized Carbocations: Addition & Rearrangements
MeO MeO A H N N Me

reversible?

O OR TMS

OR MeO TMS N OMe Me

Reading Assignment for this Lecture:


Carey & Sundberg, Advanced Organic Chemistry, 4th Ed. Part A Chapter 5, "Nucleophilic Substitution", 263-350 .
Question 13. Final Exam, 1999. During Corey's recent synthesis of Aspidophytine (JACS, 1999, 121, 6771), the pivotal intermediate 3 was assembled by the union of 1 and 2 under the specified conditions. Provide a mechanism for this single-pot transformation.

N CO2R CO2R MeO MeO

MeO MeO

N Me

N Me

CHO 1) mix at room


temp, 5 min

OHC CO2R Me3Si


2

2) 2 equiv. TFAA, 0 C

Ra +N Rc

Rb Rd

3) excess NaBH3CN

The product-determining step could be step A.

NaBH3CN

N N CO2R MeO N MeO


3

NH2

CO2R

MeO

N OMe Me

MeO MeO

N Me

Me

D. A. Evans
33-00-Cover Page 12/7/03 8:55 PM

Monday, December 8 , 2003

D. A. Evans, M. Calter
Review:

The Aza-Cope Rearrangement The 3-aza-Cope Rearrangement


First Neutral Case: Hill TL 1967, 1421.

Chem 206

Heimgartner, H. In "Iminium Salts in Organic Chemistry"; Bohme, H., Viehe, H., Eds.; Wiley: New York, 1979; Part 2, pp 655-732. The 3-aza-Cope Rearrangement: Neutral Variant:
R
3 2
1

Me R
1

Me N Me

250oC, 1 hr

Me

Me N "Practically quantitative", no real Me yields given.

[3,3]

Exothermic as written by ~7-10kcal/mole. First Cationic Case: Elkik Compt. Rend. 1968, 267, 623.

Ammonium Variant: p. p.
R R N
[3,3]

R R

Even more exothermic than the neutral version, since enamine lacks resonance and iminium salt has stronger p-Bond than imine does.

Me + N Me

Me Me

80 oC, 2-3 hr

Me Me

Me Me
No yields given.

H2O

2-aza-Cope Rearrangement:
2

OHC

Me Me

R N
1

[3,3]

R N

In the simplest case, degenerate. Steric effects, conjugation, or selective trapping of a particular isomer, will drive equilibrium. As with the 3-aza-Cope, the cationic version proceeds under much milder conditions.

Good way to allylate aldehydes: Opitz Angew. Chem. 1960, 72, 169.
R'' N R'' H
+

OHC R

R'

-H2O

R''

N R'' R

R'
+

1-aza-Cope Rearrangement:
1

R'''

[3,3]

2
1

The 3-aza-Cope rearrangement can be driven in reverse by judicious choice of substrates(i.e., incorporating the imine into a strained ring or by making R an acyl group).

H O

R'
H 2O

R'' R''

R' N R R'''
[3,3]

R'' R''

R' N R R'''

R R'''

33-01 Aza Cope-1 12/4/01 8:19 PM

M. Shair, D. Evans

Stabilized Cations: AcylIminium-Ions


OH N AcO NH2
+

Chem 206

N-Acyliminium Ion Rearrangements


Synthesis of (-)-hastanecine: Hart JOC 1985, 50, 235.
O

SiMe3 TFA Me O N Me ??? H N

SiMe3

H 67% O N Me

CH2

Me BnO

Me

O
81%

The origin of the modest diastereoselection has not been attributed to 2-aza-Cope process.

[3,3]

SiMe3

H N O Me

CH2

29% Me

BnO OBn Me Me O [3,3] BnO OBn Me Me O N H OAc Me Me N O OAc


HCO2H HCO2H

BnO OH N O OAc
NaBH4, MeOH, 83%

O Me Me N

OAc Gelas-Mailhe, Tet. Lett, 1992, 33, 73

N OAc

Me Me

Competing 2-Aza-Cope and Pinacol Rearrangements: Which Dominates??


OH Ph Ph cyclization N Me Me N Me Pinacol Ph Ph OH Mannich N Me Ph Me N Me O
:OH

Ph

BnO Me HCO2 Me N

OAc
CSA, 60oC, 1.5 hr, 79%

Ph

O O

Ph

Ph

Me

[3,3]
Me

HO

OH HO

BnO Me

OH

N Me

Homo-chiral

Ph

(-)-hastancine

Me

N O Me

racemic product
Conclusion: 2-aza-Cope rearrangements afford a low-barrier to competing processes

33-02-iminium ions 12/7/03 8:57 PM

M. Shair, D. Evans

Stabilized Cations: Iminium-Ions 2 Another aza-Cope-Mannich sequence:


OR

Chem 206

2-Aza-Cope-Mannich sequence:
OR OR

O N H HO NR2 (CH2O)n, Na2SO4 MeCN, 80C HO NR2 N [3,3] HO NR2 N HO O CH2O/HCl NHBn

Ar OH O N Bn O BF3 N Bn [3,3] O HO [3,3] Mannich 97% N Bn Ar

Axial Attack 98 %!! Mannich Rxn O H2/Pd-C CH2O/HCl ROH2C N O NR2 steps OR O O O H

Ar

N H Bn

equivalent to
NR2 O H 67% N H N H N O H HO
strychnine Overman et al. JACS 1995, 117, 5776. Overman et al. JOC 1991, 56, 5005

O H O N steps O HO HO N O Pancracine

Pictet-Spengler cyclization

33-03-iminium ions-2 12/7/03 8:58 PM

D. Evans, E. Shaughnessy

The Prins-Pinacol Reaction References


Evidence for Prins-Pinacol Mechanism
O Me O O Me Me SnCl4, CH2Cl2 Me Me O Ph Me Me

Chem 206

Prins reaction: Adams, D.R.; Bhaynagar, S. D. Synthesis 1977, 661 Prins & carbonyl ene reactions: Snider, Comprehensive Organic Synthesis, 1991, Vol. 2

Ph

The Prins Process:


O R2 R1 H HX R1 X O R1 H R2 H R1CHO R2 O R1 O R2 R1 OH R2
-

Me X R2 Ph
-

>95% ee

OH

Me Me O

Ph

Me O Me

Cl4SnO Me Me

Prins

OH R1 - H+ R2

Cl4SnO Me Me

[3,3]

pinacol
O Ph

enantiopure

Ph H Cl4SnO Me Me O

Me Me

Aldol (fast) racemic

Me Me O

Me Me

The Prins-Pinacol Variant:


Ph Me Me O O Me Me SnCl4 Ph H Me Me O Me Me Lewis Acid SnCl4 O Me Me O Ph Me Me Me

>95% ee

>95% ee

If a [3,3] rearrangement were intervening, the product would be racemic. Overman, JACS 2000, 122, 8672 Overman, Org Lett 2001, 3, 1225

Examples of Stereoselective THF Formation


O O O Me Me SnCl4, CH2Cl2 -70 -23 C 82% BF3OEt2 (E)-CH=CHPhCHO CH2Cl2, -55 C 97% Me Me O Ph Me Me O Me Me OH O

Pinacol Prins
Ph H Cl4SnO Me Me Me HO

Me

Cl4SnO

O Me Me

syn
Me

33-04-Prins-1 12/7/03 9:00 PM

7:1 anti:syn

D. Evans, E. Shaughnessy
Prins-Pinacol Mechanism
Ph SnCl4 CH2Cl2 Me Me Me O O Me
-

The Prins-Pinacol Reaction


Overman: Magellanine Synthesis
Me Me Prins O Me Me
-

Chem 206
Me N O (-)-Magellanine Me H OH

Ph Cl4SnO Me Me O

Me Me

JACS, 1993, 115, 2992

Cl4SnO

Ph

[3,3]
Ph H O MeMe Me Me Aldol

pinacol O Me Me O Ph

enantiopure

The pivotal transformation O TESO SnCl4 CH(OMe)2 57% 1. OsO4, HIO4 2. Ph2CHNH3Cl NaBH3CN (-)-Magellanine Me N O H CHPh2 N O OH Me Steps H OMe H OMe

Homo-chrial
Cl4SnO

Me Me

>95% ee

Prins cyclization faster than [3,3] rearrngement

2-aza-Cope vs. Pinacol:


CSA, 60oC, 1.5 hr, 79%

Ph

OH

Ph Ph cyclization N Me Me

:OH

Ph

Ph O N Me

Ph

Me

N Me Pinacol TESO CH(OMe)2 Ph SnCl4 TESO Me

[3,3]
Me Ph OH Me N Me Me Ph Mannich Ph O

Homo-chiral

N Me

O H OMe

O H OMe

racemic product

[3,3] rearrngement faster than Mannich cyclization


33-05-Prins-2 12/7/03 9:00 PM

mixture of diastereomers

D. Evans, E. Shaughnessy

The Prins Reaction-3

Chem 206

Overman Synthesis of a Eunicellin Diterpene


Overman & MacMillan JACS, 1995, 117, 10391 Me Me Me Me H HO AcO O

Overman: Synthesis of trans-Kumausyne


AcO

(-)-7-Deacetoxy-alcyoninacetate

JACS, 1991, 113, 5378


O Br Et

trans-Kumausyne
Me I t-BuLi, THF, -78 C Me Me OHC Me O Me OMe Me TMS Me OHC Me OH [3,3] O Me R Me TMS Me R Me O TMS 1. TMS Et BF3OEt2 -78 C rt 73% 2. TBSCl HO H O O OTBS Et O O 1. H2, Pd-C, 88% 2. Swern, 100% OTIPS Me OH H Me BF3OEt2 (3 equiv.) CH2Cl2, -55 -20 C 79% O H OBn O O m-CPBA 72% 4:1 regioselectivity O H OBn O H PPTS, MeOH 64% OH TMS OH Felkin Control (Lecture 20-21) Me OH BnOCH2CHO RSO3H, rt OH 69% OH [3,3] O OBn O OBn OH

Felkin Control (Lecture 20) O H Et OTBS DIBAL -78 C

Me single stereoisomer ds = 9:1

CHO O TMS

H CHO O H

O H

Me2HC Me OTIPS 6 steps, 39% yield from (S)-carvone

97%

33-06-Prins-3 12/7/03 9:02 PM

D. A. Evans

The Prins Reaction-4 Mukaiyama AldolPrins Cascade


Rychnovsky JACS, 2001, 123, 8420
Me OH O OMe O O El O R O El Me Me Me Me O OMe O OH HO O

Chem 206

Application to Leucasandrolide
Me OH

The Basic Process


SiMe3

The seco acid


5.5:1 ratio Me

El SiMe3

TMSX Me SiMe3 H O R O El O OBn TBSO O

The Pivotal Step:


SiMe3 BF3OEt2 base, 78%

Prins
R O El

OH

OBn

OTBS

Control of hydroxyl center: see Lecture 20-21 Let El(+) = Lewis acid activated RCHO
SiMe3 RCHO R O F 3B BF3OEt2 base O R SiMe3 O O R O BF3 R Me I BnO OTBS Myers, JACS 1997, 119, 6496 base = Me3C N CMe3 R O No (little) control over this () stereocenter HO R R BnO O TMS H O R BF3OEt2 R BnO OH O R

anti selection: ~58:1

Evans et al., JACS 1996, 116, 4322

Aldehyde Synthesis Chiral enolate alkylation: see Lecture 24


Me Me N OH Ph BnO OTBS O Me LDA Me Me N OH Ph H+ 77% O OBn Me H O H CH2 OBn SiMe3 BF3OEt2 O OBn OAc Me O DIBALH Ac2O

aldol
H SiMe3

Prins
R O

OX R

diastereoselection >20:1 Me

see Lecture 24

33-07-Prins-4 12/8/03 8:49 AM

cyclic oxo-carbenium ion addition: see Lecture 20

N. Finney, D. A. Evans
Squalene Biosynthesis
farnesyl pyrophosphate (C15)

Squalene Biosynthesis; Squalene Oxide Cyclization


Me Me Me O P P

Chem 206

Squalene Oxide Cyclase


Prestwilch, et. al. Chem. Rev. 1993, 93, 2189-2206 Cornforth Proposal: ACIEE, 1968, 903.

Me Me Me
Enz-X

Me

Me O P P Me

Me

Me

Me Me

O squalene oxide Me Me Me Enz-X O P P Me Me Me Me Me Ha Me Me Me Me O P P HO Me Me Me Me Me H Me Me Me Me Me NAD H Me Me Me Me Me Me Me HO Me Me Me H Me Me Me Me Me Me HO Me H Me H Me H Me H Me Me Me H Me Me O Me H Me

Me

Me Me Me

Me

Me Me

Me

Me Hb Ha Me

Me Me

Me

presqualene pyrophosphate

+
H Me

Me

H Me Me Me Me H Me

Me

squalene (C30)
33-08- Squalene, Lanosterol 12/7/03 9:03 PM

Me

Me

Me

Me

Me

lanosterol

N. Finney, D. A. Evans

The Corey Addendum to the Cyclization Process

Chem 206

Squalene Oxide Cyclase - Cornforth Proposal: ACIEE, 1968, 903.


Me HO Me H Me Me H Me Me

CoreyVirgil Revision: JACS 1991, 113, 4025-4026; 8171-8172


Me Me Me Me

+
H Me

Me Me H Me Me Me Me O Me Me
in vivo

O H

Me

Me HO Me H

Me

Me

X-Enz Me

squalene oxide

H O

Me Me Me Me
in vivo

Me

Me

Me stereocenter in question

Me HO Me H

Me

Me Me Me Me Me Me Me H

O Me H

Me

Me Me Me

Me

X-Enz

Me

HO Me Me Me

lanosterol

Me HO Me Me
NOT

Me HO Me H

Me

Me

O H Me H

Me

Me Me Me HO

Me

Me Me

Me

Me

+
H H Me

81 kcal/mol (MM2)
17 kcal/mol

H anti-migration Me Me Me H Me

Me

Me

anti-migration

64 kcal/mol
Me HO Me H Me

Sir John Cornforth: "That outpost of empire Australia, If you are anxious for over-exposure, to prepublication disclosure, produces some curious mammalia, to good food and good drink, the kangaroo rat, without leisure to think, the blood-sucking bat, try IUPAC symposia." and Aurthur J. Birch, inter alia."

Me

lanosterol

33-09-Revised SqOC 12/7/03 8:45 PM

N. Finney, D. A. Evans

Cationic Cyclizations in the Laboratory


Me

Chem 206

Biomimetic Polyene Cyclizations


Me H2 C Me R Me TiCl4 78 C O O O O Me Me Me R Me Me Me Me SiPhMe2
MeAlCl2 78 C, 57%

Me Me

Me

Me Me Me Me HO Me Me Me Me

OH

Johnson, et al. JACS 1987, 109, 2517. SiEt3 RO Me R' Me CO2iPr

R = H ca. 30% R = CH=CH2 77% PrO2iC AcO Me R' Me CH2

E. J. Corey, et al. Tetrahedron Lett. 1994, 35, 9149.

HO Me Johnson, et al. JACS 1987, 109, 5852.

Cyclization to Form Simpler Bicyclics


Me R = OH, R' = H 20%, 24 hr. R = OAc, R' = CH=CH2 80%, 1 min. Me
TiCl4 78 C

CH2 SiEt3 Me Me

CH2

Post cyclization transformation: Johnson


Me B Me Me C D O3 O O Me Me O Me O3 Base C D Me Base B O Me Me A B Me

O Me HO

+ Me HO O

Me

Me

96 : 4

Me

Johnson, et al. JACS 1984, 106, 1138.

Introduction of chiral auxiliaries for C=O groups

33-10-Synth Appl 12/8/03 8:37 AM

Andrew Ratz

Proposed Biosynthesis of the Daphniphyllum Alkaloids


CO2H

Evans Group Seminar, June 1993

R Me Me HN
i

CO2Me O Me N
i

Me O O
i

CO2H

Pr HN

Me

Pr Pr HN

Pr N

Me

Methyl Homodaphniphyllate

Me

HO
pyridoxal

CHO OP* Me N
H+

OP*

HO
Secodaphniphylline +

OHC Me Me Me Me CHO
Squalene Dialdehyde

Me Me

OHC R

Me CO2H
i

CO2H

Pr N

Me

pyridoxamine

R HO Me

N OP* N Me HO

Pr N

Me

HO OP* Me N H

OP* N

CO2Me OHC formal [4+2] R OHC

O Ar NH NH R R

CO2H

Pr N

Ar NH R N Ar

Pr HN

Methyl Homodaphniphyllate

Me O

N Me Me Me

formal Me [4+2] Me N

R Me Me

R Me HN

CH2O

Pr N

O Me

Daphnilactone A

33-11-Daphniphyllum Biosysyn 12/8/03 8:35 AM

Pure & Appl. Chem. 1989, 61, 289 J. Am. Chem. Soc. 1986, 108, 8274

D. A. Evans

Polyene Cyclizations
Several Comp[ex Mechanisms

Chem 206

http://www.courses.fas.harvard.edu/~chem206/

Chemistry 206 Advanced Organic Chemistry


Handout 33A
Me Me OHC

OBn OBn

1. MeNH2

Me Me Me HN

OHC
2. HOAc, NH4OAc 75%

Introduction to Carbonium Ions4


Stabilized Carbocations: OxoCarbenium Ions (C=OR(+)) Introduction to Terpene Biosynthesis Squalene, Lanosterol, & Cholesterol Biosynthesis Polyene Cyclizations Chiral Acetals as C=O Auxiliaries

Me

Angew. Chem. Int. Ed. Engl. 1978, 17, 476. J. Org. Chem. 1990, 57, 2544.

Br OMe TFA, reflux NMe Bu3SnH O O O Me N O

TIPSO

Reading Assignment for this Lecture:


Carey & Sundberg, Advanced Organic Chemistry, 4th Ed. Part A Chapter 5, "Nucleophilic Substitution", 263-350 .
Heathcock, C. H. (1992). The enchanting alkaloids of Yuzuriha. Angew. Chem., Int. Ed. Engl. 31: 665. (handout)

A. P. Johnson et al., JCS Chem. Commun. 1994, 6, 765

Other Excellent References


Bartlett, P. A. (1984). "Olefin Cyclisation Processes That Form Carbon-Carbon Bonds". Asymmetric Synthesis. Stereodifferentiating Reactions, Part B. J. D. Morrison. New York, AP. vol 3: 341. Thebtaranonth, C. and Y. Thebtaranonth (1994). "Cyclization Reactions". Boca Raton, FL, CRC Press. Corey & Liu, enzyme "Mechanism for Polycyclic Triterpene Formation," Angew. Chem. Int Ed. 2000, 39, 2812-2833

"That outpost of empire Australia, produces some curious mammalia, the kangaroo rat, the blood-sucking bat, and Aurthur J. Birch, inter alia." If you are anxious for over-exposure, to prepublication disclosure, to good food and good drink, without leisure to think, try IUPAC symposia."

Sir John Cornforth:

D. A. Evans
33A-00-Cover Page 12/7/03 9:07 PM

Monday, December 8 , 2003

D. A. Evans
Suggested Reading

Terpene Biosynthesis
Representative Terpenes
H Me Me Me Me Me HO Me H Me H Me H HO H H

Chem 206

"An Experimental Demonstration of the Stereochemistry of Enzymic Cyclization of 2,3-Oxidosqualene...." E. J. Corey, S. C. Virgil JACS, 1991, 113, 4025. "New Mechanistic and Stereochemical Insights on the Biosynthesis of Sterols from 2,3-Oxidosqualene Cyclase" E. J. Corey, et al. JACS 1991, 113, 8171. "Enzymatic Cyclization of Squalene and Oxidosqualene to Sterols and Terpenoids" Abe, et al. Chem. Rev. 1993, 93, 2189. "Isoprenoid Biosynthesis. Stereochemistry of the Cyclization of Allylic Pyrophosphates." D. E. Cane Acc. Chem. Res. 1985, 18, 220.

Me Me Me Me Me

lanosterol

cholesterol

OAc

"Biomimetic Polyene Cyclizations" W. S. Johnson Angew. Chem. Int. Ed. Engl, 1976, 15, 9. taxol
RO

Me Me Me HO BzO AcO

O Me OH

Interesting Reading Science, 1997, v277: "Structure and Function of the Squalene Cyclase", G. Shultz, p. 1811. "Structural Basis for Cyclic Terpene Biosynthesis by Tobacco 5-epi-Aristolochene", J. P. Noel, p. 1815. "Crystal Structure of Pentalene Synthase: Mechanistic Insights on Terpenoid Cyclization Reactions in Biology", D. W. Christianson, p. 1821. "The Stereochemistry of Allylic Pyrophosphate Metabolism." D. E. Cane Chem. Rev. 1980, 36, 1109. "Biosynthesis of Natural Products" P. Manitto; Wiley&Sons, NY: 1981.
HO Me Me Me

O O O O

CH2

Me O H 2C O OC OH Me CO2H Me

OH

caryophyllene

picrotoxinin
O

gibberellic acid
33A-01-terpene refs 12/8/00 9:00 AM

D. A. Evans

Terpenes: Occurance in Nature

Chem 206

Definition: Natural products whose carbon skeletons are built up largely from isoprene subunits:
Me Me OH isoprene Me Me menthol Me (S)-carvone caraway Me (R)-carvone spearmint H Me O H Me O

Role in insects: hormones, pheromones (communication chemicals)


O O O Me H2C Me H
Me

periplanone sex attractant pheromone of the American cockroach

Me

O OMe

Cecropia junenile hormone blocks development at larval growth stage


note that the starred methyls are not derived from isoprene Me

Occurance: Plants, insects, higher organisms Role in plants: hormones, defense etc. example: pyrethrin insecticides Me
Me Me H Me Me H OH chrysanthemic acid O Me H Me O O Me

O Me

alarm pheromone for aphids H O Me

Role in mammals: hormones, pheromones ??


H Me Me Me Me Me Me H Me H Me cholesterol steroid hormones

CHO Me OH CHO

example: antifeedants such as warburganal


H Me Me HO O Me Me citronellal lemon oil H nepetalactone oil of catnip active at nanomolar-femptomolar concentrations is this related to a feline pheromone? Me H O O H

example: flavor constituents


Me Me Me nootkatone grapefruit flavor O Me

Me

33A-02-terpene-1 1/25/00 2:49 PM

D. A. Evans
Me
head tail

Isoprene : Building Block of Nature (2)


Classification of terpenes
monoterpenes : 10 C-atoms (2 isoprene units) sesquiterpenes : 15 C-atoms (3 isoprene units) diterpenes triterpenes : 20 C-atoms (4 isoprene units) : 30 C-atoms (6 isoprene units)

Chem 206

Terpene Biosynthesis
There are two isoprene units which are used to build up terpenes: Me OX enzyme Me Me OX

2-methyl-1,3-butadiene isoprene

t h t

OH

OH OH O

isopentenyl pyrophosphate (IPP) O O

,-dimethylallyl pyrophosphate (DMAP) O

R O P O P OH citronellol menthol camphor O O


O

ROX

R O S O

CH3

ROTs

geraniol

two components in rose oil

O O Me Me H

pyrophosphate: nature's leaving group

tosylate: chemist's leaving group Me

The basic process: alkene addition to electrophiles: Me Me OX DMAP Me Me H H Me OX -HB Me Me Me OX geranyl pyrophosphate Me OH H2O/OH-OX
-

Me Me CH2

H 2C

OX

sex attractant pheromone of the American cockroach

cinnamolide antifungicide

B-carotene

natural rubber Practice: Recognize the isoprene units in the above structures.

n
Me Me

geraniol

33A-03-terpenes-2 12/17/99 8:00 AM

D. A. Evans

Isoprene : Building Block of Nature (3)

Chem 206

From isoprene to pinene and bornene


Me Me Me OX OX Me OX isomerization R3 Me DMAP Me C R4 R1 C
+

The Basic Process: 1-2 rearrangement

-OX-

R2 CH3

R4 R1

R3 R2 CH3

geranyl pyrophosphate Me Me

Me

-OXOX Me Me Me Me

-H+
CH3 CH3

Me limonene

The precursor of bornene could also lead to camphene. Show the mechanism! camphene

Me

Me Me Me

-H+

Me

Tetrahydrocannabinol (THC)
Me OH

H+
aromatic substitution CH3

Me OH

pinene 1,2 shift Me Me OH chrysanthenol Me Me Me Me Me Me Me bornene Me O C5H11 OH HO olivetol Me HO C5H11

-H+

-H+
Me Me O H

OH

C5H11

33A-04-terpenes-3 12/17/99 8:03 AM

N. Finney, D. A. Evans
Cyclic Monoterpenes
Me Me OH Me

Cyclic and "Irregular" Monoterpenes


Biosynthetic Study of Monoterpenes
Me Me Me H
14

Chem 206

CO2H OH HO Me CO2H

C label

O O HO2C

HO Me OH
() MVA

-terpineol
Me Me Me H Me Me

Me

HO Me () MVA lactone

borneol
Me
+

Me Me

hydroxy-methyl glutarate cholesterol-lowering drugs operate here

Me OH O P P

Me Me -pinene Me

Me

GPP

Me Me OH

H Me Me
+H

Me

Me Me Me O

carene
Me Me camphor

O -thujone

Me Me

Me

Iridoids and Secoiridoids


Me HO OR O OHC CO2CH3
loganin R = -D-glucose

terpinolene Me
+H

CH2 OR O

Me OH Me

Me

Me

CO2CH3
secologanin

terpinenol Me
Me H
+

Irregular Monoterpenoids
Me Me Me Me Me Me Me Me 33A-05-Monoterpenes 12/8/00 8:52 AM Me Me CO2H H Me
chrysanthemic acid nezukone

Me Me O

-thujene
Me

N. Finney
Me

SesquiterpenesC15 Terpenoids
Me

Chem 206

Me Me Me

O P P (Z)-FPP Me Me

Me Me

(E)-FPP O P P

Me

Me

Me Me Me Me Me

CH3 Me

Me
+

Me

Me

Me

Me

Me

Me

Me

Me Me Me Me Me

Me Me Me

Me Me Me

CH3 Me Me

CH3 Me Me

Me

[Ox]

O O

Me Me

Me Me OH

Me

Me

Me

Me O H 2C Me

OH O

Me Me

Me Me Me

Me Me

CH2 H 3C Me Me Me

Me

cuparene picrotoxinin

widdrol

thujopsene

caryophyllene

humulene

33A-06-Sesquiterpenes 12/8/00 8:54 AM

N. Finney
Squalene Biosynthesis
farnesyl pyrophosphate (C15)

Squalene Biosynthesis; Squalene Oxide Cyclization


Me Me Me O P P

Chem 206

Squalene Oxide Cyclase


Prestwilch, et. al. Chem. Rev. 1993, 93, 2189-2206 Cornforth Proposal: ACIEE, 1968, 903.

Me Me Me
Enz-X

Me

Me O P P Me

Me

Me

Me Me

O squalene oxide Me Me Me Enz-X O P P Me Me Me Me Me Ha Me Me Me Me O P P HO Me Me Me Me Me H Me Me Me Me Me NAD H Me Me Me Me Me Me Me HO Me Me Me H Me Me Me Me Me Me HO Me H Me H Me H Me H Me Me Me H Me Me O Me H Me

Me

Me Me Me

Me

Me Me

Me

Me Hb Ha Me

Me Me

Me

presqualene pyrophosphate

+
H Me

Me

H Me Me Me Me H Me

Me

squalene (C30)

Me

Me

Me

Me

Me

lanosterol

33A-07- Squalene, Lanosterol 12/7/03 8:45 PM

N. Finney, D. A. Evans

The Corey Addendum to the Cyclization Process

Chem 206

Squalene Oxide Cyclase - Cornforth Proposal: ACIEE, 1968, 903.


Me HO Me H Me Me H Me Me

CoreyVirgil Revision: JACS 1991, 113, 4025-4026; 8171-8172


Me Me Me Me

+
H Me

Me Me H Me Me Me Me O Me Me
in vivo

O H

Me

Me HO Me H

Me

Me

X-Enz Me

squalene oxide

H O

Me Me Me Me
in vivo

Me

Me

Me stereocenter in question

Me HO Me H

Me

Me Me Me Me Me Me Me H

O Me H

Me

Me Me Me

Me

X-Enz

Me

HO Me Me

lanosterol

Me HO Me Me

Me HO Me H

Me

Me

Me

Me Me

Me
NOT

Me

O H Me H

81 kcal/mol (MM2)
17 kcal/mol

H anti-migration Me Me Me H Me HO Me H Me

Me

Me Me

Me

+
H H Me

64 kcal/mol
Me HO Me H Me

Me

anti-migration

Me

lanosterol

33A-08-Revised SqOC 12/8/03 8:06 AM

N. Finney

Cationic Cyclizations in the Laboratory


Biomimetic Polyene Cyclizations
Me Me H2 C Me TiCl4 78 C Me R Me Me Me Me O O Me Me SiPhMe2
MeAlCl2 78 C, 57%

Chem 206

Me Me

Me

Me

Me Me Me Me HO Me Me Me Me

OH

Johnson, et al. JACS 1987, 109, 2517. SiEt3 RO Me R' Me CO2iPr

R = H ca. 30% R = CH=CH2 77% PrO2iC AcO Me R' Me CH2

E. J. Corey, et al. Tetrahedron Lett. 1994, 35, 9149.

HO Me Johnson, et al. JACS 1987, 109, 5852.

Cyclization to Form Simpler Bicyclics


Me R = OH, R' = H 20%, 24 hr. R = OAc, R' = CH=CH2 80%, 1 min. Me
TiCl4 78 C

CH2 SiEt3 Me Me

CH2

Post cyclization transformation: Johnson


Me B Me Me C D O3 O O Me Me O Me O3 Base C D Me Base B O Me Me A B Me

O Me HO

+ Me HO O

Me

Me

96 : 4

Me

Johnson, et al. JACS 1984, 106, 1138.

Introduction of chiral auxiliaries for C=O groups

33A-09-Synth Appl 12/8/00 8:48 AM

D. A. Evans
Me Me Me O Me O Me SnCl4 C6H6, R.T. RO Ratio = 72 : 28 H Me

Chiral Acetal Auxiliaries-1


R H Me Me O Me O Me TMS
TiCl4 CH2Cl2, -78oC 91% yield

Chem 206
R O Me OH Me R = C8H17 O Me R O Me

H RO de = 84%

W. S. Johnson & Co-workers, J. Am. Chem. Soc., 1976, 98, 6188.

P. A. Bartlett & Co-workers, J. Am. Chem. Soc., 1983, 105, 2088 Ph


HBr CCl4

Ratio = 88 : 12 Br Ph

Ph R H O Me O Me
LiAlD4 AlCl3

CH3 O CO2Me

CH3 OH CO2Me
Br2

CH3 O CO2Me

R D O Me OH Me R D HO Me ratio 01:99 22:78 14:86

R O Me

O MeO2C

O MeO2C

O MeO2C

G. Castaldi & Co-workers, Angew. Chem. I. E., 1986, 25, 259. ( CH2)6

94 % yield ( CH2)6

de = 86 %

W. J. Richter, J. Org. Chem., 1981, 46, 5119.

Et t-Bu Ph

R Me O Me O Me
LiAlH4 AlCl3

R O Me Me OH Me R Et t-Bu Ph O R OTMS CH2 C Et


SnCl4 CH3CN, -20oC

Zn-Cu, CH2I2

R Me HO Me ratio 73:24 85:15 51:46 O Et OH Me R Ph n-C9H19 Et O Me ratio 16:1 03:1 R O Me P. Mangeney & Co-workers, Tetrahedron Lett., 1987, 28, 2363. O Me H O
PhCu

O O MOMO Me

O OMOM

Et2O,

O MOMO

O OMOM

K. A. Nelson & E. A. Mash, J. Org. Chem., 1986, 51, 2721.

de = >95 % R
R

W. J. Richter, J. Org. Chem. ,1981, 46, 5119.

(1) CuBr, PBu3 (2) LiBr, BF3 Et2O, -78oC

R H O Me O Me

O Me

OH Me
% de 95 (R) 85 (R) 69 (S)

O Me

Me

Cu

Acetal Alkylating Product Isomer Agent (R) E/Z E E Z (1) (2) (1) 100/0 95/5 100/0

% Yield 75 70 75

J. M. McNamara, Y. Kishi, J. Am. Chem. Soc., 1982, 104, 7371.

(error in R/S nomenclature in paper)

33A-10-chiral acetals-1 12/17/99 8:57 AM

D. A. Evans
C8H17 H O O TMS TiCl4 CH2Cl2, -78oC 98% yield
P. A. Bartlett & Co-workers, J. Am. Chem. Soc., 1983, 105, 2088.

Chiral Acetal Auxiliaries-2


C8H17 O OH OH C8H17 O O O Me Ratio = 87 : 13
W. S. Johnson & Co-workers, Tetrahedron Lett., 1984, 25, 591.

Chem 206

PhO H TMS C N TiCl4 CH2Cl2, 0 C 92% yield


o

CN Ar HO O Me

CN Ar O HO Me

Ratio = 97 : 3, 88% de

C8H17 H O Me O Me Me
W. S. Johnson & Co-workers, J. Am. Chem. Soc., 1983, 105, 2904.

C8H17 TMS TiCl4 CH2Cl2, -78oC 96% yield Me O Me OH Me + diastereomer Me

H O

C6H13 O O R RLi, CuBr Me2S Et2O Me

R O

H C6H13 COOH

Ratio = 93 : 7
S. L. Schreiber & J. Reagan Tetrahedron Lett., 1986, 27, 2945.

Temp.(oC) 0 -30

% Yield 58 90

% de 94 97

C 6H 5 H O Me O Me
W. S. Johnson & Co-workers, J. Org. Chem., 1983, 48, 2294.

C8H17 TMS C N TiCl4 CH2Cl2, -20oC 97% yield O Me C N OH Me + diastereomer Me O Ratio = 96.5 : 3.5 Me Me C8H17 OTBS O OtBu

Me Ph

COOH 1) TiCl4, CH2Cl2, -780C 2) CF3COOH, 22oC 98 % yield C8H17 HO Me O Me

c-C6H11 CH3 O Me O Me
H. Yamamoto & Co-workers, Tetrahedron Lett., 1983, 24, 4581.

c-C6H11 Br2AlH Et2O, -20 C 99% yield Me


o

CH3 OH Me

J. D. Elliott & Co-workers, Tetrahedron Lett., 1985, 26, 2535.

Ratio = 98 : 2

+ diastereomer

H 3C Ratio = 23 : 1 O O CONMe2 1) Me3Al 2) Ac20 - Py OXc

Et O Me O Me

Bu DIBAL CH2Cl2, 0oC 90 % yield

Et OH Me O Me

Bu + diastereomer

Me2NCO

H. Yamamoto & Co-workers, Tetrahedron Lett., 1984, 25, 5911.

97% yield de = 77%

H. Yamamoto & Co-workers, Tetrahedron Lett., 1986, 27, 983.

Ratio = 97 : 3 diastereoselection >99%

33A-11-chiral acetals-2 12/17/99 8:59 AM

Andrew Ratz

Daphniphyllum Alkaloids
Me O

Evans Group Seminar, June 1993

Daphniphyllum Alkaloids
O CO2Me

"Classical" Total Synthesis of Methyl Homodaphniphyllate


CO2Me CO2Me

O Me AcO
i

Me R
*

Pr N
Daphniphylline

Pr N
i

Pr N

Pr

* *

N H Me Me
*

Methyl Homodaphniphyllate

Me O O Me O CO2Me O Me
i

Me R N H Me

BnO Me O O R O N

O O

Pr HN

Me

Pr HN

Me

Secodaphniphylline

Methyl Homosecodaphniphyllate

O N O O BnO O N

O O

O O
i

O Me

Pr N

O Me

N
Bukittinggine

Daphnilactone A

Isolated from bark and leaves of Yuzuriha tree Used to cure asthma and vermicide in the early 20th century Compounds have been known since 1909. First structure solved in 1965. 38 members in this alkaloid family
J. Org. Chem. 1992, 57, 2531-2594 Kyoto Igaku Zasshi 1909, 6, 208 Tet. Lett. 1965, 965 O Me O N
H

O CO2H + O O

O Me

O NH2

J. Org. Chem. 1992, 57, 2531 J. Am. Chem. Soc. 1975, 97, 6116 Angew. Chem. Int. Ed. Eng. 1992, 31, 665

33B-01 12/11/00 11:53 AM

Andrew Ratz

Daphniphyllum Alkaloids

Evans Group Seminar, June 1993

Synthesis of Methyl Homodaphniphyllate


HO2C O O H 2N O Me
Et3N ClCO2Et 89%

Construction of the Methyl Homodaphniphyllate Skeleton


O OBn O OBn O Me Me
1. Et3OBF4

O Me

O N
H

O O

pTsOH, tol. 83%

N O

O N
1 isomer

LDA, THF

N Br S Me
4:1

BnO
73%

2. Et3N 80%

BnO Me N O
i

O O

BnO

1. Me3OBF4 2. NaBH4 3. HOAc, HCl 87%

O Me O O
8:1

LDA, THF

O
i

Pr

Me O O
74%

OBn N O O Me N
Lawesson's Reagent

OBn O O
1. LTMP, PhSeCl 2. MCPBA 60%

Pr

O Me N H O O

H O

BnO BnO Me O R S
R=iPr 10:1 46% R=Me 83:17

BnO

H2SO4, Acetone

O N O

1. H3O+ 2. HO(CH2)2OH

S
3. LDA, THF

OBn S N O
14%

N O

OBn O Me N H O H HO
NaOMe MeOH 70% 2 steps

Me O

J. Am. Chem. Soc. 1986, 108, 5650 J. Org. Chem. 1992, 57, 2531 J. Am. Chem. Soc. 1968, 90, 6177 Nouv. J. Chem. 1980, 4, 47

N O

33B-02 12/8/00 9:51 AM

Andrew Ratz

Daphniphyllum Alkaloids

Evans Group Seminar, June 1993

OBn

Synthesis of Methyl Homodaphniphyllate cont'd


O

Proposed Biosynthesis of the Daphniphyllum Alkaloids

Me N H O

O
LDA, THF

Me R O H Me O N O

H30+

Me R O H Me N O OBn Me Me Me CHO
Squalene Dialdehyde

OHC Me

Me Me

OHC R

MeCHO 100% HO

pyridoxamine

OBn
1. Me2CuLi (EtO)2POCl 83%

OBn Me

R HO Me

N OP* N

Pr N

H N O
68% + Me

(EtO)2PO2

O2P(OEt)2
Li/EtNH2 64%

2. LDA, THF, HMPA O (EtO)2POCl 73%

N O HO
13%

OHC

OH

OH

OH O H
i

OHC

Pr H H
??

120 atm H2 Pd(OH)2

Pr H H H Pr N
+

Ar H H
1. Jones 2. MeOH, H+

NH NH

Ar NH R N Ar

EtOH, 120C 81%

1:1 ratio

R R Me N Me Me Me N Me Me Me HN R

Me Me H H N

R H Tet. Lett. 1969, 2145.


i

CO2Me

Pr H H N

Me

Pure & Appl. Chem. 1989, 61, 289 J. Am. Chem. Soc. 1986, 108, 8274

33B-03 12/11/00 11:27 AM

Andrew Ratz

Daphniphyllum Alkaloids Proposed Biosynthesis cont'd


CO2H CHO

Evans Group Seminar, June 1993

Retrosynthesis of Methyl Homosecodaphniphyllate

15

Me

Me14 Me 10

HO Me HN
+

Pr

OP* CO2Me

OR

OR

Me CO2H

Squalene

Pr HN

Me HN

Me H

+N

Me

Pr
+N

Me

HO OP* CO2H Me N
H+
i

CO2H OR Pr
+ N

Me

OR

Pr
+ N

Me

HO OP* Me N H Me CO2Me Me

+ HN

HO OP* Me N

H Me OR

+N

Me

Pr N OHC Me Me OR R O
+

CO2H

Methyl Homodaphniphyllate
i

OHC Me

Pr HN

Me
i

O Pr N
Daphnilactone A

O Me

Me R O
+

I Me Me

33B-04 12/11/00 11:31 AM

Andrew Ratz

Daphniphyllum Alkaloids Synthesis of Methyl Homosecodaphnipyllate


OBn
+

Evans Group Seminar, June 1993

The Vollhardt "Ammonia" Incident


I OBn

Me MeO2C
+

N O

Me
LDA, THF -78C to 25C

Me OHC Me Me OHC Me

OBn
1. MeNH2 2. HOAc, NH4OAc 75%

Me Me Me HN

OBn N O MeO2C Me
4%

OBn N O MeO2C N O MeO2C Me Me


87%

OBn

Me

Me Me

Me

Me

Me
8%

OBn

OBn

1. DIBAL 83% 2. KOH, H2O, 95C 3. HCl,H2O 100%

Me Me

CH3 N H Me

Me

OBn OHC Me Me OHC Me


1. NH3 2. AcOH, NH4OAc 70C 82-95% 1. LAH 96% 2. Swern

R O O Me Me Me OBn OBn

OR

CO2Me
1. H2, Pd/C

CH3
i

+N

Me

Me H N

Me HN

2. Jones 3. MeOH, H+

Pr HN

Me
Angew. Chem. Int. Ed. Engl. 1978, 17, 476. J. Org. Chem. 1990, 57, 2544.

J. Org Chem. 1989, 54, 1215. Tet. Lett. 1986, 27, 959. Top. Stereochem. 1990, 20, 87.

33B-05 12/11/00 11:36 AM

Andrew Ratz

Daphniphyllum Alkaloids
Stepwise vs Concerted Cyclization
R
+

Evans Group Seminar, June 1993

One-Pot Pentacyclization Reaction: Formation of Protodaphniphylline

R
Stepwise

H R

R H N+

Me Me

Me

Me I

LDA, THF CH3CO2tBu 83%

Me Me

Me OtBu O
1. LDA, Br(CH2)3CH(OMe)2 2. H3O+ 83%

Me

R R H
+N

R
Concerted

Me Me Me Me CO2tBu OH Me
70%

Me

Me

R H

+N

Me

Me tBuO2C Me Me CHO

BnO OHC OHC Me Me Me R Me HN


NH3 44%

CO2tBu R
AcOH

Me Me

R Me N
AcOH 70C 95%

OR

1. MsCl, Et3N 2. DBU 3. DIBAL 80% 1. Swern 2. NH3 3. AcOH, 80C 15%

25C 90%

Me

Me Me

Me

Me

OH

Me Me OH Me Me

Me HN

R R OHC OHC
Stepwise

R Me Me H N R Me Me HN Me Me

R H N
+

Me

1. Swern 2. MeNH2 3. HOAc, 80C 65%

Me Me

Me Me

Me Me
Concerted

Me

Other amines: Glycine (S)-(+)-Alanine (S)-(+)-Valine (+)--Phenylethyl amine 38%y 32%y 1-2%ee 13%y 20-25%ee no reaction

Me

Me Me Me HN

79% isolated yield

33B-06 12/11/00 11:43 AM

Andrew Ratz

Daphniphyllum Alkaloids
Synthesis of (-)-Secodaphniphylline
Me O O CO2Me O Me COX O O Me
+
i

Evans Group Seminar, June 1993

Rationale for Stereoselectivity in Michael Additions


1. LDA, THF

OBn N O MeO2C Me Me

Me

OBn Pr HN N Me O

2.

CO2Me

3.

Pr HN

Me Me Me

Me

Me

O HO2C Me

Me Me 1. Weinreb O
2.

O Me O Me

Me Me LAH O
Darvon Alc. 93%

BnO O Me Me OH
92% ee KAPA 1,3-diaminoprop. 87%

OBn

Me Me O Li H O O N OMe N O LiO OMe H

Me

Li

Me HOCH2 O
1 1. RuCl3, NaIO4 2. CH2N2

CH2OH O + Me
:

Me

O
5

O Me

HgSO4 H2SO4 95%

Me Me O Me Me O Li O OMe N Me H H OBn Me Me Me Me N O MeO2C OBn

Me OH

Me MeO2C O O Me
+

CO2Me Me O

1. separate

COCl Me O O Me
i

MeO

OLi

O 2. KOH EtOH Me

3. (COCl)2, CH2Cl2 31%

Pr HN
90% ee

92% ee

Me Me Me MeO Me N O Li O
Interference with Li cluster ??

Me O O Me
J. Org. Chem. 1980, 45, 582 Tet. Lett. 1981, 22, 4171 J. Org. Chem. 1981, 46, 3936 iPr Tet Lett. 1967, 8, 215

Me O O
NaCN, DMSO 150C

OBn H

N Me Me O MeO2C Me

O Me MeO2C
i

O BnO Me

HN

Me

43% overall

Pr HN

Tet Lett 1986, 27, 959 J. Org. Chem. 1990, 55, 132 Topics in Stereochemistry 1989, 227

Me

99.6% ee

33B-07 12/11/00 11:47 AM

Andrew Ratz

Daphniphyllum Alkaloids Retrosynthesis of Daphnilactone A


CO2H O OH

Evans Group Seminar, June 1993

Synthesis of Daphnilactone A
Br
LDA, homogeranyliodide 85%

EtO
1. LAH

Br O Br Me COBr Me Me

O O

EtO2C

2.

Pr N

O Me

Pr HN

Me

Me Me HN OEt Me CO2Et Me

Br
3. H3O+ 95%

Pr

Zn, THF, HMPA 89%

Br O O O Me Me Me Me Me Me
LAH 90%

O O

H O O Me

OZnBr

O N

Pr HN

O Me Me Me Me

Me

Me

HO Br Br O Me Me Me Me Me Me O O EtO EtO2C OEt CO2Et Me Me Me HO

1. Swern 2. NH3 3. HOAc, 87%

O HN Me

H2, Pt 100%

Pr HN

O Me

DIBAL tol. 87%

Org. React. 1975, 22, 423 Ang. Chem. Int. Ed. Eng. 1967, 6, 1 Ang. Chem. Int. Ed. Eng. 1969, 8, 535 J. Org. Chem. 1992, 57, 2585 Angew. Chem. Int. Ed. Engl. 1992, 31, 665
i

OH

HO

Pr HN

Me

Pr HN

Me

33B-08 12/11/00 11:51 AM

4.5

Andrew Ratz

Daphniphyllum Alkaloids

Evans Group Seminar, June 1993

Total Synthesis of Daphnilactone and Conversion to the Daphniphylline Skeleton


OH O
i

Pr HN

Me

1. CrO3, H2SO4 2. CH2O, pH7 50%

Pr N

O Me

CO2H Oi

Pr HN

Me

O Me
+N

Pr

OH

CO2Me

Pr HN

Me

1. CrO3, H2SO4 2. MeOH, H+ 60%

Pr HN

Me

PhNCO

CO2Me
HCO2H
i i

CO2Me

Pr N

Pr O HN Ph N

50%

Me

33B-09 12/11/00 11:52 AM

CarfbeneOlefin Cycloaddition: The FMO Analysis

HOMO

LUMO

C H

H C R

C R

Carbene demo 12/10/03 9:12 AM

D. A. Evans

Introduction to Carbenes & Carbenoids-1


Useful References to the Carbene Literature

Chem 206

http://www.courses.fas.harvard.edu/~chem206/

Chemistry 206 Advanced Organic Chemistry


Lecture Number 34

Books: Modern Catalytic methods for Organic Synthesis with Diazo Compounds; M. P. Doyle, Wiley, 1998. Carbene Chemistry, 2nd ed. Academic Press, Kirmse, W., 1971.

Introduction to Carbenes & Carbenoids-1


Carbene Structure & Electronics Methods for Generating Carbenes Simmons-Smith Reaction Carbene-Olefin Insertions Carbene Rearrangements

Provide a mechanism for the following transformations.

EtO2C O N CO2Et CH3 N2 MeO2C Rh2(OAc)4 CO2Me O H 3C N O CO2Me

Reading Assignment for this Lecture:


Carey & Sundberg, Advanced Organic Chemistry, 4th Ed. Part B Chapter 10, "Reactions Involving Highly Reactive Electron-Deficient Intermediates", 595680. Handout 09A Simmons-Smith Reaction: Enantioselective Variants Handout 26B Synthetic Applications of -Diazocarbonyl Compounds
Chiral DirhodiumCarboxamidates: Catalysts for Highly Enantioselective Syntheses of Lactones and Lactams, Aldrichchimica Acta. 1996, 29, 3 (handout) Doyle, Catalytic Methods for Metal Carbene Transformations, Chem. Rev. 1986, 86, 919-939 (electronic handout) McKervey, Organic Synthesis with -Diazocarbonyl Compounds, Chem. Rev. 1994, 94, 1091-1160 (electronic handout) Muller, Catalytic Enantioselective Aziridinations & Asymmetric Nitrene Insertions, Chem. Rev. 2003, 103, 2905-2919 (electronic handout) O O N

CO2Me

O CO2Et CH3 N2 1. Rh2(OAc)4


2. MeO2C

O EtO2C CO2Me N CH3 MeO2C CO2Me

JACS 1990 2037

H O N2

150C (71%)

H O

D. A. Evans

Monday, December 10 , 2003

34-00-Cover Page 12/10/03 8:12 AM

D. A. Evans
Suggested Reading: Doyle, Chem Rev. 1988, 86, 919. Kodadek, Science, 1992, 256, 1544.

Carbenes: An Introduction
Carbene Configuration: Triplet vs. Singlet
p

Chem 206

Energy

Recent Review Article: Chemistry of Diazocarbonyls: McKervey et al. Chem Rev. 1994, 94, 1091. Books: Modern Catalytic methods for Organic Synthesis with Diazo Compounds; M. P. Doyle, Wiley, 1998. Carbenes and Nitrenes in "Reactive Molecules: The Neutral Reactive Intermediates in Organic Chemistry", Wentrup, C. W. 1984, Wiley, p. 162. Rearrangements of Carbenes and Nitrenes in Rearrangements in Ground & Excited States, Academic Press, DeMayo ed., Jones, W. M. 1980, p. 95. Carbene Chemistry, 2nd ed. Academic Press, Kirmse, W., 1971.

S1 810 kcal/mol T1
p triplet singlet

Due to electron repulsion, there is an energy cost in pairing both electrons in the orbital. If a small energy difference between the and p orbitals exists, the electrons will remain unpaired (triplet). If a large gap exists between the and p orbitals the electrons will pair in the orbital (singlet).

the History of the Singlet-Triplet Gap


Year 1932 Method Qual. Thermochem Qual. QM Ab initio Kinetics SCF MINDO Expt An Initio Expt Expt Author Muliken Walsh Gallup Harrison Hase Pople Dewar Lineberger Schaeffer Zare Haydon HCH Angle Grnd State 90-100 180 160 138 132 134 138 singlet triplet triplet triplet triplet triplet triplet triplet triplet triplet triplet ST Splitting kcal/mol small 30 >33 89 19 8.7 19.5 19.7 8.1 8.5
(Wentrup)

Carbenes: Electronic Structure

Carbene Configuration: Triplet vs. Singlet


1.078

1947 1957

H
133.8

H H

H
Triplet (two unpaired e-) Often has radical-like character
empty filled

1969 1971 1971 1974 1976 1976


filled

Singlet (all e- paired) Often has electrophilic or nucleophilic character: A-type (Ambiphilic)
empty

Nitrene
R N

H R N

filled

1978 1982

Singlet (all e- paired)

Nitrenium ion
34-01-carbenes intro 12/10/03 7:39 AM

M. Shair, D. A. Evans

Carbenes: Structure and Generation


Bamford-Stevens Reaction: See Lecture 28 on Hydrazones
Shapiro Org. Rxns. 1976, 23, 405.
Me Me Me Me Me

Chem 206

Heteroatom-Substituted Carbenes: Singlets


The p orbital of carbenes substituted with p-donor atoms (N, O, halogen) is raised high enough in energy to make the pairing of the electrons in the orbital energetically favorable. As a result, these carbenes are often in the singlet state.

Me

OR

p Energy triplet carbene


Cl

donor p orbital

Me

H N N Ts Me

Ts
Me

Me

N Me

N Ts Me

Me

N N

N2

Heteroatomsubstituted carbene
H

-donor heteroatom

Me H R1 N N R2 diazirines R1 N N R2 R1

Me

diazo compounds
C Singlet

N N

h or heat

Examples:
Cl

Singlet C6 H5

R1 C + N2 R2

h or heat Chem. Soc. Rev. 1982, 11, 127.

Methods of Synthesis
Alkyl Halides:
Cl Cl R R Cl C H Cl C H

R2

metal-catalyzed decomposition Doyle Chem Rev. 1986, 86, 919 (handout)


OH
Cl C Cl R C R R2 O Rh O Rh R1 N N Me O Rh O Rh Rh2(OAc)4 (ligands omitted for clarity) R2 R1 N N Me O Rh Me Me O Rh

OH

N2

ketenes
R C R C O

carbenoid
R1 R2

heat or h

R 2C

CO

R1 R2

O Rh

O Rh

34-02-carbenes intro 12/10/03 8:49 AM

M. Shair, D. A. Evans

Carbenes: Structure and Generation


Cyclopropanation The Skell Rule:
1 CH 2
NaH, THF

Chem 206

"Stable Carbenes"
"Stable CarbenesIllusion or reality"? Regitz, M. Angew. Chem. Int. Ed. Engl. 1991, 30, 674 (handout) Cl

R + R

N N

N N

cat. tBuOK (89%)

Singlet carbenes add to olefins stereospecifically;


R 3 CH 2 + R H2C R R ISC H 2C R R

Arduengo et al. J. Am. Chem. Soc. 1991, 113, 361; 1992, 114, 5530. Arduengo et al. J. Am. Chem. Soc. 1994, 116, 6812, Neutron diffraction study:

N N

N N

N N R R R H 2C R ISC R H 2C R R

Arduengo argues that these resonance structures are not players based on electron distribution from neutron diffraction. These are nucleophilic carbenes which display high stability. For reviews on the subject, see: Regitz, M. Angew. Chem. Int. Ed. Engl. 1996, 35, 725. Regitz, M. Angew. Chem. Int. Ed. Engl. 1991, 30, 674.

Triplet carbenes add non-stereospecifically


Skell and Woodworth JACS, 1956, 78, 4496.

Synthetic Applications Simmons-Smith Cyclopropanation (See Tedrow hanndout 10B)

Me N S

Me F Au F F F F Xray Structure

Simmons, H.; Smith, R. J. Am. Chem. Soc., 1958, 80, 5323. OH CH2I2 Zn(Cu) OH

>99:1 diastereoselectivity

The intermediate organometallic reagent: ICH2ZnI Zn(Cu) H. G. Raubenheimer Chem. Comm. 1990, 1722. OH CH2I2 150:1 cis : trans OH 75% yield

Winstein & Sonnenberg, JACS 1961, 91, 3235

34-03-carbenes intro 12/10/03 8:20 AM

M. Shair, D. A. Evans
The Furakawa Simmons-Smith Variant

The Simmons-Smith Reaction


Catalytic Asymmetric Cyclopropanation:
R3 R2 R1 2 ICH2ZnEt OH
(0.12 eq.) ZnEt2, CH2I2

Chem 206

For a recent general review of the Simmons-Smith reaction see: Charette & Beauchemin, Organic Reactions, 58, 1-415 (2001) Et2Zn, CH2I2 Solvent EtZnEt + ICH2I

R3 R2 R1
66-82 % ee

OH

NHSO2R NHSO2R

Kobayashi, et al. Tetrahedron Lett. 1994, 35, 7045. For a Lewis Acid catalyzed process in which the rate of the catalyzed process is faster than the uncatalyzed, see: Charette, A. B. JACS 1995, 117, 11367.

Furukawa, J.; Kawabata, N.; Nishimura, J. Tetrahedron, 1968, 24, 53 Furukawa, J.; Kawabata, N.; Fujita, T. Tetrahedron, 1970, 26, 243 Et2Zn, PhCHI2 ether, rt 69% syn : anti 94 : 6

Applications in Synthesis
Ph O Me O MeO O
>99:1 diastereoselectivity

Hydroxyl directivity is a powerful atribute of the SS Rxn


Me OH Me OH Me OMe
CH2I2 Zn(Cu) For an review of the directed Simmons-Smith, see: Evans, D. A.; Hoveyda, A.; Fu, G. Chem. Rev. 1993, 93, 1307. CH2I2 Zn(Cu) CH2I2 Zn(Cu)

NH O O Me Calipeltoside A

New SS variant: see Shi, Tet. Lett 1998, 39, 8621 O Cl


Et2Zn CF3COOH CH2I2

Me

OH Me OH

Me Me H MeO Me O

Me

Me OMe

no Me diastereoselectivity

OH O O O

50:1 diastereoselectivity

Cl Evans, Burch Org. Lett. 2001, 3, 503 R HN O O

>99:1 diastereoselectivity

Cl Me

Me Me O

Me
U-106305

O HO

Me OBn OBn Me
Et2Zn CH2I2

FR-900848 H R=

N O OH OH

Me
R=

NH O

N O

Me

O HO

Me OBn OBn
Charrette, A. B.; J. Am. Chem. Soc. 1996, 118, 10327.

Charette, A. B. JACS 1991, 113, 8166.

>50 : 1 diastereoselection

Falck J. Am. Chem. Soc. 1996, 118, 6096. Barrett, JOC, 1996, 61, 3280.

34-04-carbenes intro 12/9/03 8:47 PM

D. A. Evans

Simmons-Smith Reaction the Shi Variant

Chem 206

34-04a-Shi/SS summary 12/9/03 9:24 PM

M. Shair, D. A. Evans
Synthetic Applications
H O TBSO Me O
Cu(I)

Carbenoids: Cyclopropanation
Buchner Reaction

Chem 206

N2

H TBSO Me O H O N2

CO2Me

cat.

CO2Me CO2Me

Me
1. Br2 2. DBU

Me O
Rh2(OAc)4 (84%)

H TBSO Me O H O

Et2AlCl

H TBSO Me H O H O OH Me H H HO Me O O O CO2Me AcO Me

1,3-shift

N2

AcO Me O Me H O

Corey & Myers JACS 1985, 107, 5574.

Me OTBS O
McKervey et al. JCS PTI, 1991, 2565.

Me

Antheridic Acid confertin

O O

CO2Me O N2
Cu Powder 130C, Xylenes

O CO2Me

Wolff Rearrangement
THPO OMe OMe O O2N Me N2
AgOBz H 2O

THPO O O CuLi
2 Et2O, -12C

OMe OMe O2N Me OMe

retention CO2H

CO2Me
prostaglandins

OMe
Evans et al. J. Org. Chem. 1993, 58, 471.

(+) Macbecin

THPO

Corey and Fuchs JACS 1972, 94, 4014.

34-05-carbenes 12/9/03 8:49 PM

D. A. Evans

Carbenes: Enantioselective Cyclopropanation


Mechanism

Chem 206

Characterization of metal carbenoid intermediates: not much data!


CO2Et Ph N2 Rh2(OAc)4 cat. CO2Et Ph

There is no definitive evidence for metal-catalyzed cyclopropanation and the possibility that metallacyclobutane intermediates are involved cannot be ruled out. Me3C R Cu Me3C CO2Me Ph R
Reductive Elimination

+ O N N Me Me O

For a detailed mechanistic study which provides supporting evidence for the intermediacy of a Rh carbene, see: Kodakek, Science, 1992, 256, 1544. SiMe3 N Cu N SiMe3 SiMe3 N Cu N SiMe3 spectroscopically observed "Copper(I) Carbenes: The Synthesis of Active Intermediates inCu-Catalyzed Cyclopropanation" P. Hoffmann et al, Angew. Chem. Int. Ed. 2001, 40, 1288-1290 H R

N2CHCO2R R -N2

Me3C CO2R N Cu H N H Me3C

+ O Me Me O

OTf

Me3C Me3C

CH2 CH2 N2

CO2Me Ph

Me3C Me3C

OTf

Me3C CO2R H H R CO2R N Cu H N H Me3C O Me Me O

Catalytic Asymmetric Variants: Chiral Cu(I) Complexes

Me L* = O N

Me O N CMe3

OTf

Catalytic Asymmetric Variants:Chiral Rh(II) Complexes


O Ph
(5 mol %)

Me3C CO2BHT Ph N2

L* CuOTf Ph

CO2BHT 99% ee 94:6 trans/cis

N O H N CO2Me O H nPr H H O
95% ee

nPr

Rh Rh O N2 O
Doyle et al. Tetrahedron Lett. 1995, 36, 7579.

R
+

EtO2C

CuOTf

N2

R R

ent-6b

CO2Et a, R = Ph, >99% ee b, R = Me, >99% ee

CH2Cl2

Evans, et al. J. Am. Chem. Soc. 1991, 113, 726.

34-06-carbenes 12/10/03 8:38 AM

How do these complexes really work??

D. A. Evans

Carbenes: Enantioselective Cyclopropanation


The Carbene Complex
Doyle, JACS 1993, 115, 9968
(5 mol %)

Chem 206

Catalytic Asymmetric Variants: Chiral Rh(II) Complexes


O Ph nPr O N H N CO2Me O N2 O
Doyle et al. Tetrahedron Lett. 1995, 36, 7579. CH2Cl2

Molecular mechanics: favored by 3 kcal/mol H nPr H H O


95% ee

Rh Rh O

variable ligand

H N
MeO2C

O Rh O N

CO2Me

CO2Me H N O H Rh N CO2Me O

27A
H N
MeO2C

O Rh O N

CO2Me N H O H Rh N C H CO2Me O CO2Me CO2Me

H Note NO trans influence

styrene N 2 O 1 Rh O 3 4 N Numbers designate increasing steric hindrance in each quadrant Doyle, JACS 1993, 115, 9968 H Ph 48%ee Ph CO2Me H CO2Me

27B

34-06a-carbenes-Doyle 12/10/03 8:43 AM

H 86%ee H

M. Shair, D. A. Evans
Carbene-Carbene Rearrangements
H

Carbenes: Rearrangements
Other Rearrangements
H O N2
150C (71%)

Chem 206

H O

H O
Wu, Tetrahedron Lett. 1973, 3903.

H O

H H N2 O
200C (92%) Schecter, J. Am. Chem. Soc. 1971, 93, 5940.

O O O O

Skattebol Rearrangement

Sammes, Chem. Comm. 1975, 328. C13

C Br

Br

Vinylidenes
BuLi

Corey-Fuchs:

Danishefsky et al. J. Am. Chem. Soc. 1996, 118, 9509.

TIPS

TIPS

[1,2]

C13

Teoc

O O Br

Ph

Teoc

O O
(81%)

Ph Ph

Tetrahedron Lett. 1973, 2283.

Ph 2 eq. BuLi
-78C

OTBS

Br

OTBS

34-07-carbenes 12/9/03 8:52 PM

M. Shair, D. A. Evans
Carbene Rearrangements

Carbenes: Rearrangements
O O
OTf NaSO2Ar CH2Cl2, 20C

Chem 206

O H N2 P(OEt)2

BuLi 78 C RCHO R

OLi

O P(OEt)2 N R C N N H N2 R O C H vinylidene carbene


-PhI

SO2Ar I+Ph

I+Ph

N2

O SO2Ar N
(63%)

R C C H

O Me N2

O P(OEt)2
+

OMOM
Stang et al. J. Am. Chem. Soc. 1994, 116, 93.

MeO2C

CHO OMOM

K2CO3 MeOH 25C

MeO2C
(83%)

carbene intermediates are accessible at high temperatures, more later!

Bestmann, et al. Synlett 1996, 521.

Me Me O O H O Me Me H N2 O P(OEt)2 KO-t-Bu 78 C 68% Me Me O O Me

Me O H
620C

Me

Me H

Me

Me H H

H Me Me CH insertion Me O H
620C

H Me
capnellene

Gilbert, JOC 1983, 48, 5251

Me

Me H

Me O H

H Me

H Me

34-08-carbenes 12/10/03 8:45 AM

M. Shair, D. A. Evans
CH Insertions continued...
N2 H Me O H
Rh2(OAc)4

Carbenes: Rearrangements
NH Insertions are also possible...
O CO2Me Me
(83%)

Chem 206

HO Me

H H NH O

N2 CO2PNB
Rh2(OAc)4

HO Me

H H N O CO2PNB

CO2Me

Electrophilic carbenes are very sensitive to electronic effects Stork Tetrahedron Lett. 1988, 29, 2283.

HO Me

H H N S CO2
thienamycin

NH3

CO2Me N2 N O O
CuOTf

HO Me

CO2Me H N
(75%)

O O O

H H N O CO2PNB

Salzmann, JACS, 1980, 102, 6163.

Insertions (X-H): Stereochemical outcome


Sulikowski, J. Org. Chem. 1995, 60, 2326.

Enantioselective C-H Insertion

R2

R1

R3

R2 A B

R1

+ +
H A B

R2

R1

H A B

O Ph O O O N2
CH2Cl2 (5 mol %)

R3

R3

N H N CO2Me H O O H
99%, 97% ee

Retention
O

Rh Rh MeO2C N2

O H Me Ph Me
Rh2(OAc)4

CO2Me Ph

Doyle, JACS 1994, 116, 4507

Chiral DirhodiumCarboxamidates: Catalysts for Highly Enantioselective Syntheses of Lactones and Lactams, Aldrichchimica Acta. 1996, 29, 3 (handout)

Taber JACS, 1996, 107, 196.

34-09-carbenes 12/9/03 8:54 PM

M. Shair, D. A. Evans
Ring Opening
O H H N2 O
h EtOH

Carbenes: Rearrangements

Chem 206

Problem 332: Wolff Rearrangement Stoltz ACS 2003, 125, 13624


R O H H C H O MeO O N2 H
HOEt

MeO AgOBz, Et3N Me THF, 45

O Me
95% yield

H R

OH CO2Et
(88% Tetrahedron Lett. 1990, 31, 6589.

AgOBz, Et3N N2 O H Me Wolff Rearr H

O C Me H H H

Ring Contraction
N2 H S O O N CO2Me
h iPr2NH

H
carbene intermediate

O N

H H S N

O
(81%)

CO2Me

Vinylolgous Wolff Rearrangement Doyle pp520-521


Me Me N2 O Me Me Cu(I) MeOH Me CO2Me Me

Moore et al. J. Org. Chem. 1983, 48, 3365.

Wolff-[2+2]
N2 Me O
h

Me H

C +

Me Me Me
J. Org. Chem. 1980, 45, 2708. (74%)

Me

Me O

Me

Me O

C Me Me H

O Me

34-10-carbenes 12/9/03 9:16 PM

D. A. Evans

Introduction to Carbenes & Carbenoids-2


Useful References to the Carbene Literature

Chem 206

http://www.courses.fas.harvard.edu/~chem206/

Chemistry 206 Advanced Organic Chemistry


Lecture Number 35

Recent Review Article: Chemistry of Diazocarbonyls: McKervey et al. Chem Rev. 1994, 94, 1091. Books: Modern Catalytic methods for Organic Synthesis with Diazo Compounds; M. P. Doyle, Wiley, 1998. Carbenes and Nitrenes in "Reactive Molecules: The Neutral Reactive Intermediates in Organic Chemistry", Wentrup, C. W. 1984, Wiley, p. 162. Rearrangements of Carbenes and Nitrenes in Rearrangements in Ground & Excited States, Academic Press, DeMayo ed., Jones, W. M. 1980, p. 95. Carbene Chemistry, 2nd ed. Academic Press, Kirmse, W., 1971.
R R R C X R R R O R R

Introduction to Carbenes & Carbenoids-2


Thermally Induced Carbene Rearrangements Carbonyl Ylides and their Reactions
R C R R R R C R X R O R R O R R

Oxonium & Sulfonium Ylides and their Reactions


R

The Automerization of Naphthalene (The Cume Question from Hell!)

Rationalize

Reading Assignment for this Lecture:


Carey & Sundberg, Advanced Organic Chemistry, 4th Ed. Part B Chapter 10, "Reactiona Involving Highly Reactive Electron-Deficient Intermediates", 263-350 . Handout 10B Simmons-Smith Reaction: Enantioselective Variants Handout 27B Synthetic Applications of -Diazocarbonyl Compounds Handout 35A The Use of Fischer Carbenes in Organic Synthesis

13C-labeled C10H8 is isomerized into 13C-labeled C10H8 at 1035 C L. T. Scott, JACS 1991, 113, 9692.

Provide a Mechanism for this Transformation

1000 C 10-4 Torr

Corannulene 10%

D. A. Evans

Friday, December 12 , 2003 Scott, L.T., et. al., JACS 113 7082 (1991)

35-00-Cover Page 12/12/03 8:16 AM

D. A. Evans

Carbenes: Enantioselective Cyclopropanation


The Carbene Complex
Doyle, JACS 1993, 115, 9968
(5 mol %)

Chem 206

Catalytic Asymmetric Variants: Chiral Rh(II) Complexes


O Ph nPr O N H N CO2Me O N2 O
Doyle et al. Tetrahedron Lett. 1995, 36, 7579. CH2Cl2

Molecular mechanics: favored by 3 kcal/mol H nPr H H O


95% ee

Rh Rh O

variable ligand

H N
MeO2C

O Rh O N

CO2Me

CO2Me H N O H Rh N CO2Me O

27A
H N
MeO2C

O Rh O N

CO2Me N H O H Rh N C H CO2Me O CO2Me CO2Me

H Note NO trans influence

styrene N 2 O 1 Rh O 3 4 N Numbers designate increasing steric hindrance in each quadrant Doyle, JACS 1993, 115, 9968 H Ph 48%ee Ph CO2Me H CO2Me

27B

34-06a-carbenes-Doyle 12/10/03 8:43 AM

H 86%ee H

M. Shair, D. A. Evans
Carbene-Carbene Rearrangements
H

Carbenes: Rearrangements
Other Rearrangements
H O N2
150C (71%)

Chem 206

H O

H O
Wu, Tetrahedron Lett. 1973, 3903.

H O

H H N2 O
200C (92%) Schecter, J. Am. Chem. Soc. 1971, 93, 5940.

O O O O

Skattebol Rearrangement

Sammes, Chem. Comm. 1975, 328. C13

C Br

Br

Vinylidenes
BuLi

Corey-Fuchs:

Danishefsky et al. J. Am. Chem. Soc. 1996, 118, 9509.

TIPS

TIPS

[1,2]

C13

Teoc

O O Br

Ph

Teoc

O O
(81%)

Ph Ph

Tetrahedron Lett. 1973, 2283.

Ph 2 eq. BuLi
-78C

OTBS

Br

OTBS

34-07-carbenes 12/9/03 8:52 PM

M. Shair, D. A. Evans
Carbene Rearrangements

Carbenes: Rearrangements
O O
OTf NaSO2Ar CH2Cl2, 20C

Chem 206

O H N2 P(OEt)2

BuLi 78 C RCHO R

OLi

O P(OEt)2 N R C N N H N2 R O C H vinylidene carbene


-PhI

SO2Ar I+Ph

I+Ph

N2

O SO2Ar N
(63%)

R C C H

O Me N2

O P(OEt)2
+

OMOM
Stang et al. J. Am. Chem. Soc. 1994, 116, 93.

MeO2C

CHO OMOM

K2CO3 MeOH 25C

MeO2C
(83%)

carbene intermediates are accessible at high temperatures, more later!

Bestmann, et al. Synlett 1996, 521.

Me Me O O H O Me Me H N2 O P(OEt)2 KO-t-Bu 78 C 68% Me Me O O Me

Me O H
620C

Me

Me H

Me

Me H H

H Me Me CH insertion Me O H
620C

H Me
capnellene

Gilbert, JOC 1983, 48, 5251

Me

Me H

Me O H

H Me

H Me

34-08-carbenes 12/10/03 8:45 AM

M. Shair, D. A. Evans
CH Insertions continued...
N2 H Me O H
Rh2(OAc)4

Carbenes: Rearrangements
NH Insertions are also possible...
O CO2Me Me
(83%)

Chem 206

HO Me

H H NH O

N2 CO2PNB
Rh2(OAc)4

HO Me

H H N O CO2PNB

CO2Me

Electrophilic carbenes are very sensitive to electronic effects Stork Tetrahedron Lett. 1988, 29, 2283.

HO Me

H H N S CO2
thienamycin

NH3

CO2Me N2 N O O
CuOTf

HO Me

CO2Me H N
(75%)

O O O

H H N O CO2PNB

Salzmann, JACS, 1980, 102, 6163.

Insertions (X-H): Stereochemical outcome


Sulikowski, J. Org. Chem. 1995, 60, 2326.

Enantioselective C-H Insertion

R2

R1

R3

R2 A B

R1

+ +
H A B

R2

R1

H A B

O Ph O O O N2
CH2Cl2 (5 mol %)

R3

R3

N H N CO2Me H O O H
99%, 97% ee

Retention
O

Rh Rh MeO2C N2

O H Me Ph Me
Rh2(OAc)4

CO2Me Ph

Doyle, JACS 1994, 116, 4507

Chiral DirhodiumCarboxamidates: Catalysts for Highly Enantioselective Syntheses of Lactones and Lactams, Aldrichchimica Acta. 1996, 29, 3 (handout)

Taber JACS, 1996, 107, 196.

34-09-carbenes 12/9/03 8:54 PM

M. Shair, D. A. Evans
Ring Opening
O H H N2 O
h EtOH

Carbenes: Rearrangements

Chem 206

Problem 332: Wolff Rearrangement Stoltz ACS 2003, 125, 13624


R O H H C H O MeO O N2 H
HOEt

MeO AgOBz, Et3N Me THF, 45

O Me
95% yield

H R

OH CO2Et
(88% Tetrahedron Lett. 1990, 31, 6589.

AgOBz, Et3N N2 O H Me Wolff Rearr H

O C Me H H H

Ring Contraction
N2 H S O O N CO2Me
h iPr2NH

H
carbene intermediate

O N

H H S N

O
(81%)

CO2Me

Vinylolgous Wolff Rearrangement Doyle pp520-521


Me Me N2 O Me Me Cu(I) MeOH Me CO2Me Me

Moore et al. J. Org. Chem. 1983, 48, 3365.

Wolff-[2+2]
N2 Me O
h

Me H

C +

Me Me Me
J. Org. Chem. 1980, 45, 2708. (74%)

Me

Me O

Me

Me O

C Me Me H

O Me

34-10-carbenes 12/9/03 9:16 PM

D. A. Evans, D. Guterman

Thermal Generation of Carbene Intermediates

Chem 206

Carbenes are Accessible via Sigmatropic Rearrangement

The Automerization of Naphthalene (The Cume Question from Hell!)

[1,2] Shifts: Alpha-Alkynone Cyclizations


O O
[1,2]

Rationalize

H
H

O
C-H insertion

H
H

13C-labeled C10H8 is isomerized into 13C-labeled C10H8 at 1035 C

Conditions: 620 C, 12-16 Torr, Quartz filled Quartz Tube

Mechanism-1: L. T. Scott, JACS 1977, 99, 4506;


2s + 2a 2s + 2a

S.M.
O

3
O

2
O

1
O

Recovery 80%

_
O O

60 87 _
O

22

18
O

2s + 2a

2s + 2a

13 _

78%
For the azulenenaphthalene Isomerization: G = 30.7 kcal/mol (298K) The Activation energy for the isomerization: G = +86 kcal/mol

54
O O

46

89%

Mechanism-2,3: L. T. Scott, JACS 1991, 113, 9692.


H H X H H X H H

92

8
O

90%
Option2

Karpf, M., Dreiding, A., Helv. Chim. Acta. 65 13 (1982)


620 C

H H

BF

900 C

B + A 21% + 79% H H

Me 14 Torr
N2 , 1 Hr

Me O

BF H Me Me Me H H

H H

80% (+ 19% isomers)

Clovene Option3
B (HBHA) = 3.4 kcal/mol (MNDO) A

Karpf, M., Dreiding, A.S., Helv. Chim. Acta. 67 1963 (1984)

35-01-carbenes/heat 12/11/03 9:03 PM

D. A. Evans, D. Guterman

Thermal Generation of Carbene Intermediates


Carbenes: Reaction with Heteroatoms
Suggested Reading Corannulene 10% Houk and Wu J. Org. Chem. 1991, 56, 5657. Padwa and Hornbuckle Chem. Rev. 1991, 91, 263.

Chem 206

Provide a Mechanism for this Transformation

1000 C 10-4 Torr

Scott, L.T., et. al., JACS 113 7082 (1991)

Review Articles Padwa and Krumpe Tetrahedron 1992, 48, 5385. Hoffman, R. W. Angew. Chem. Int. Ed. Engl. 1979, 18, 563.

35-02-carbenes/heat-2 12/11/03 9:06 PM

0000 00000 00 00 00 00 00 00 000 00 00000000 00 00 00 00 00 00 0000000 00 00 00 00000000 00 00000000 00 00 00 0000000 00 00000 00 0000 00
H H H 1000 C 10-4 Torr C H H

McKervey et al. Chem. Rev. 1994, 94, 1091. Ylide Formation by the Interaction of Carbeneoids with Carbonyl Lone Pairs R Internal Ring: 6 e External Ring: 14 e
H C

R R R

R R

O R

R R

O R

Generally, the carbene precursor of choice is a diazoalkane or, more frequently, an -diazocarbonyl reagent. These can be decomposed via thermolysis or photolysis. However, the most common method involves catalytic amounts of transition metals, such as copper or rhodium. Dipolar Cycloaddition (See Lecture 18)

CH Insertion

R R

O R

R R X

O Y

R R

D. A. Evans

Thermal Generation of Carbene Intermediates

Chem 206

Web Problem 88. Please provide a mechanism for the following high temperature reaction that was reported by Yranzo and co-workers (J. Org. Chem. 1998, 63, 8188).

Web Problem 172. Scott has recently reported the transformation illustrated below (Tetrahedron Lett. 1997, 38, 1877) which is implemented by flash vacuum pyrolysis (FVP) at the indicated temperature.

Ph

H N N Ph

Ph flash pyrolysis
+ 820 C N2

Cl

CH2
FVP, 900 C 2 HCl

Draw a plausible mechanism for the transformation to the major product isomer. Do not invoke any radical intermediates in your answer.

Cl

CH2

Ph

H N N Ph

1,5-H shift

Ph

H N N Ph N2

Ph

thermal isomerization

Provide a concise mechanism for this reaction in the space below.

H Ph
C-H insertion 700 C

Ph

Web Problem 198. Provide a mechanism for the thermal conversion of triquinacene to azulene (JACS, 1973, 2724).

Ph
1,5-H shift

Ph
1,5-H shift

Ph H

Web Problem 135. Anderson has reported the transformation illustrated below (Aust J. Chem. 1990, 43, 1137) which is implemented by flash vacuum pyrolysis (FVP). As indicated, this reaction proceeds through intermediate A.

O O O Ph
CO2 CO FVP, 900 C CO2 CO 95% yield

Web Problem 282. Hoffmann has reported the mechanistically interesting thermally induced transformation illustrated below (Chem. Ber. 1985, 634.).

NC

N
400 C

NC

Provide a mechanism for this reaction and identify intermediate A in your answer.

Provide a plausible mechanism for this reaction.

35-03-Carbenes/heat-3 12/12/03 8:37 AM

D. A. Evans, D. Barnes

Carbonyl Ylids: Dipolar Cycloaddition


Reactions of Diazoimides: [3+2] addition
H O O N CO2Et O Me N Bn N2 O O Me Rh2(OAc)4 PhCH3, 110 C Bn N2 74% O O CO2CH3 O CO2CH3 O O O O O R H H CH3 O Me N Bn Et3SiH / BF3Et2O CH2Cl2 H N O H Me O OH Bn N O Me COMe N O

Chem 206

Tandem Intramolecular CyclizationIntermolecular Cycloaddition


R O CHN2 O O NPh O R O O H H O R NPh O O R

R O N C CO2Et

Rh2(OAc)4

Me

COMe O

RCHO DMAD
R

Me

68%

Dipolar-Dipolarophile Cycloadditions: HOMOLUMO Energies


Carbonyl Ylides have very small HOMO-LUMO gaps

Bn Maier, M. E.; Evertz, K. Tetrahedron Lett. 1988, 29, 1677-1680.

O O Energy N N2 O O Me H Rh2(OAc)4 PhH, reflux 88% H O N O Me O O Me N2 HOMO N H H "high yield" Padwa et. al. Tetrahedron Lett. 1992, 33, 4731-4734. N H N O N O O O Me

ylide

dipolarophile

Therefore, either raising the dipolarophile HOMO (electron-donating substituents) or lowering the LUMO (electron-withdrawing) will accelerate the reaction.
O LUMO

35-04-carbonyl ylids-2 12/11/03 9:06 PM

D. A. Evans, K. Beaver

Carbonyl Ylids: Dipolar Cycloaddition2

Chem 206

Dipolar Cycloadditions: Carbonyl Ylides


Me Me OAc H H O N2 H
Rh2(OAc)4

Reactions of Diazoimides: [3+2] addition [4+2] retroaddition


Me Me H O O N
2 R1 R2 R Me

AcO H O

Me Me H H
(86%)

AcO

O Z N2 Rh2(OAc)4 PhCH3, 110 C R2

R1

R2

CO2Et O H CO2Et O H

MeN=C=O O CO2Et R1 +O Z R2 N Me O R2 R2 N Me O Z O R1

Dauben, JOC 1993 7635

Tigilane Skeleton R
2

Me N

O O OMe
Rh2(pfb)4

OM MeO2C O N Me
(93%)

O MeO2C O H H N Bz Lysergic Acid Skeleton O N O N Me

O N2 N Bz

R1 = H, OBn R2 = H, Me Z = COMe, CO2Me

Yields = 44-63%

Maier, M. E.; Schffling, B. Chem. Ber. 1989, 122, 1081-1087.

Padwa, JOC 1995 2704

N Bz

O Me N2

Rh2(OAc)4, PhH, 80 C CO2Me

MeOH MeO

O N H MeO2C O COCH3 CO2Me

85% MeO2C

O Me N N

O CO2Me Et N 2 O N

O N O N O Et O CO2Me
(95%)

O Et O O N O Me MeO2C

CO2Me O N

O MeO2C Me MeOH O

CO2Me OH Me O N CO2Me

Rh2(OAc)4

Me

CO2Me

Padwa, JOC 1995 6258

N Me

Vindoline Skeleton Padwa, A.; Hertzog, D. L.; Chinn, R. L. Tetrahedron Lett. 1989, 30, 4077-4080.

35-05-carbonyl ylids-2 12/11/03 9:07 PM

D. A. Evans, D. Barnes

Carbonyl Ylids: Dipolar Cycloaddition (and More!)3


O Me Me EtO

Chem 206
CO2Et O Me Me

The Carbonyl Ylide - Azomethine "Dipole Cascade"


O O O CHN2 Rh2(OAc)4 HC C CO2CH3 O H CO2Me Me

The Synthesis of Furans


CHOCH3 O CHN2 CuSO4 160 C Me O Me hydrolysis decarboxylation Me

N Me

N Me

CO2CH3

CO2CH3 ~30%

H N Me O O

not observed Me

N O

rearrange

HC C CO2CH3

MeO2C O O OH

Methyl vinhaticoate
Me CO2CH3

N Me O

O H Me

N O CO2Me Me

Spencer, T. A., et. al. JACS 1967, 89, 5497.

CO2Me

Can you propose a rational mechanism for this transformation?


O O O O CH3O N2 CO2CH3 CH3O HO CO2CH3 Cu(acac)2 89% O

The 1,3-proton shift is catalyzed by trace amounts of water. Azomethine ylide formation requires a proton at the tertiary center.
Padwa, A.; Dean, D. C.; Zhi, L. J. Am. Chem. Soc. 1989, 111, 6451-6452. Padwa, A.; Dean, D. C.; Zhi, L. J. Am. Chem. Soc. 1992, 114, 593-601.

The Synthesis of Furans


Intermolecular addition to ,-unsaturated carbonyls
OMe O Et O CHN2 O CuSO4 160 C OMe

O O EtO2C O O

O O O CH3O CO2CH3

2-methoxymethylenecholestanone-3

CH3O CO2CH3 EtO2C

Spencer Tetrahedron Lett. 1967, 1865-1867.

O 29%

Hildebrandt, Tetrahedron Lett. 1988, 29, 2045-2046.

35-06-carbonyl ylids-3 12/11/03 9:07 PM

D. A. Evans, D. Barnes

Carbene Heteroatom Transformations: Ylide Formation


Tandem O-H Insertion/Claisen Rearrangement
O MLn EtO2C 1 Me N2 98% ee O O OMe
+

Chem 206

Ylid Formation versus Cyclopropanation


O EtO O catalyst CHN2 EtO O O

Me OH Me
Rh2(OAc)4

HO PhH, , 20 min Me (66%) 95% ee O

O OCH3

EtO O

EtO 2 O

H+ O O OEt O X O Me CO2Me Z-Enol Transition State slow O Me Me O O Me OMe PhH, , 18 hrs (75%) N O EtO Me CH2t-Bu + insertion products OH H H NH N2 O OH H H N O CO2 S HO O Me CO2Me E-Enol Transition State [3,3] fast

X = H, Rh(II) ??
X O Me CO2Me Wood, JACS 1997 9641 Wood, JACS 1999, 121, 1748 Me The opposite enantiomer is observed! O OCH3 O 47% ee

catalyst Pd2Cl2(C3H5)2 Cu(PhCOCHCOMe)2 (MeO)3PCuI Rh2(OAc)4 None

Temp (C) RT 80 RT RT 80

%1 53 50 3 1 15

%2 3 13 35 58 54

Me

O Me

it is evident that these are reactions of metal carbenoids


Bien, S.; Gillon, A.; Kohen, S. J. Chem. Soc. Perkin Trans. I. 1976, 489-492 O O Me N2 O N t-Bu CO2Et
pfb = perfluorobutyrate

HO Me

Rh2(pfb)4 PhH, reflux

Me

Merck Thienamycin Process


O O NO2 OH H H Rh2(oct)4 Me PhH, 80 C 100% NH3 Thienamycin Salzmann, JACS 1980 6161 N O OH CO2p-NB

O Me L4Rh2

O N CH2t-Bu H transfer Me

OH

O N CH2t-Bu

- O+
EtO H

O EtO

Doyle, M. P., et al. J. Org. Chem. 1991, 56, 820-829. Doyle, M. P.; Taunton, J.; Pho, H. Q. Tetrahedron Lett. 1989, 30, 5397.

35-07-carbonyl ylids-4 12/11/03 9:09 PM

M. Shair, K. Beaver

Carbene Heteroatom Transformations: Sulfonium & Oxonium Ylids


Ring expansion reactions have been investigated

Chem 206

Ylide Formation
O R N2 Reviews: R 2X OEt catalyst R X R R O OEt

Methods based on sulfur ylides: (review) Vedejs, Accts. Chem. Res. 1984, 17, 358 CO2Et S N2 CO2Et CO2Et S 50%

Cu(I)

Padwa, Chem. Rev. 1991 263 Padwa, Chem. Rev. 1996 223 Barnes, Evening Seminar, March 16, 1993
2 3

KOt-Bu

TfO S

CO2Et
DBU

S Me O 72%

X is generally S, O or N and can be sp or sp hybridized Ylides often undergo sigmatropic rearrangements or cycloadditions [2,3]-Sigmatropic rearrangement:
Ph SPh O O OMe N2 Kido and Kato, JCS Perkins 1 1992 229 SPh E MeO O Cl Me O N2
Rh2(OAc)4

S EtO2C

72%

HO O Me

S E

Me O Et O

OH Methynolide has been synthesized by Vedejs using this ring-expansion methodology Me Vedejs, JACS 1989, 111, 8430

OMe O CO2Me
Rh2(OAc)4

OMe

O CO2Me

MeO Me Cl

O Me

Me

Acorenone B

Pirrung et al JACS, 1991, 113, 8561.

OMe

O OH O

Stevens Rearrangement ([1,2] alkyl shift):


MeO O R2 N R1 O N2 Rh2(OAc)4 N R1 R2 N R2 O AcO R1 O H N2 Clark et al. Tetrahedron Lett. 1996, 37, 5605. Me
Cu(I) or Rh(II)

AcO Me

O O

Cl

O Me
griseofulvin

H AcO Me O O

West, JACS 1993 1177

35-08-ylide formation 12/11/03 9:10 PM

Chemistry 206 Advanced Organic Chemistry


Handout35A

The Use of Fischer Carbenes in Organic Synthesis


"...every synthetic chemist is well advised to follow this fascinating field with appropriate attention." - Schmalz, H.-G., ACIEE, 1994, 303.

Brian Connell Evans Group Seminar, February, 1999

D. A. Evans
00-Cover page 12/12/03 8:12 AM

Friday , December 12, 2003

B. Connell

Fischer Carbenes
The Use of Fischer Carbenes in Organic Synthesis
"...every synthetic chemist is well advised to follow this fascinating field with appropriate attention." - Schmalz, H.-G., ACIEE, 1994, 303.

Chem 206

Brian Connell Evans Group Seminar 2/12/99 Outline Introduction and Fundamentals Reactions Cyclopropanation Diels-Alder Cycloaddition Other Cycloadditions Dotz Reaction and Analogs Photochemistry Conjugate Additions Other Reactions
General References: Wulff,Organometallics, 1998, 3116. Wulff,Comprehensive Organic Synthesis, Vol. 5, Chap 9.2: Metal Carbene Cycloadditions, Pergamon Press, 1991. Wulff, Comprehensive Organometallic Chemistry II, Vol. 12, Chap 5.3: Transition Metal Carbene Complexes: Alkyne and Vinyl Ketene Chemistry, Pergamon Press, 1994. Hegedus, Comprehensive Organometallic Chemistry II, Vol. 12, Chap 5.4: Transition Metal Carbene Complexes: Photochemical Reactions of Carbene Complexes, Pergamon Press, 1994.

Introduction

Definition: electrophilic, heteroatom stabilized complexes having formal metal-to-carbon double bonds Group 6 metals (Cr, Mo, W) are the most common metals used. First prepared by Fischer (ACIEE, 1964, 580):

Cr(CO)6

RLi

Li
(CO)5Cr

O R

(CH3)4NBr
(CO)5Cr

N(CH3)4 can be prepared on large scale and stored R for long periods

"hard" alkylating reagents (CH3)3OBF4, CH3COBr excellent yields for all steps cheap starting materials (20-50/mmol)
(CO)5Cr R OCH3

air, silica stable crystalline, easy to handle colored (yellow to red)

01/02 12/20/99 3:49 PM

B. Connell

Fischer Carbenes
Nitrogen Analogs

Chem 206

O O (CO)5Cr R

CH3 Br (CO)5Cr R O O

N(CH3)4

H3C

NHR2

NR2 (CO)5Cr R

O R1 NR2

OTMS

K2Cr(CO)5

TMSCl
R
1

NR2 R1 (CO)5Cr NR2 (CO)5Cr R1

(CO)5Cr

NR2

Dotz, Synlett, 1991, 381.

Selected Physical Data

OCH3 (CO)5Cr CH3 (CO)5Cr

H3C N CH3 O CH3

H3C N CH3

CH3

Bond Lengths Ccarbene - O = 1.33 Ccarbene - Cr = 2.04 Cr - COcis = 1.86 - 1.91 Cr - COtrans = 1.87 IR Frequencies vCO ~2070, 1992, 1953 cm-1 H NMR Ccarbene - CH3 = ~5
13 1

Bond Lengths Ccarbene - N = 1.31 Ccarbene - Cr = 2.16 Cr - COcis = 1.90 Cr - COtrans = 1.85 IR Frequencies vCO = ~2060, 1970, 1940 cm-1 H NMR Ccarbene - CH3 = ~3.2
13 1

Bond Lengths Ccarbonyl - N = 1.29 Ccarbonyl - O = 1.23

IR Frequencies vCO = ~1650 cm-1


1

H NMR

Ccarbonyl - CH3 = ~2.1


13

C NMR Ccarbene 320-360 ppm

C NMR Ccarbene = 250-290 ppm

C NMR

Ccarbonyl = ~165 ppm

03/04 12/20/99 4:21 PM

Major Contributors
Ernst Otto Fischer
I was born in Solln, near Munich, on 10 November 1918 as the third child of the Professor of Physics at the Technical College of Munich, Dr. Karl T. Fischer (died 1953), and his wife, Valentine, ne Danzer (died 1935). After completing four years at elementary school I went on to grammar school in 1929, from which I graduated in 1937 with my Abitur. Following a subsequent period of "work service" and shortly before the end of my two years' compulsory military service, the Second World War broke out. I served in Poland, France and Russia. In the winter of 1941/2 I began to study Chemistry at the Technical College in Munich during a period of study leave. I was released by the Americans in the autumn of 1945, and resumed my study of Chemistry in Munich after the reopening of the Technical College in 1946. I graduated in 1949. I took up a position as scientific assistant to Professor Walter Hieber in the Inorganic Chemistry Department, and under his guidance I dedicated myself to working on my doctoral thesis, "The Mechanisms of Carbon Monoxide Reactions of Nickel II Salts in the Presence of Dithionites and Sulfoxylates". After receiving my doctorate in 1952, I was invited by Professor Hieber to continue my activities at the college and consequently chose to specialise in the study of transition metal and organo-metallic chemistry. I wrote my university teaching thesis on "The Metal Complexes of Cyclopentadienes and Indenes". I was appointed a lecturer at the Technical College in 1955 and in 1956 I completed a scientific sojourn of many months in the United States. In 1957 I was appointed Professor at the University of Munich. After turning down an offer of the Chair of Inorganic Chemistry at the University of Jena I was appointed Senior Professor at the University of Munich in 1959 . In 1957 I was awarded the Chemistry Prize by the Gttingen Academy of Sciences. The Society of German Chemists awarded me the Alfred Stock Memorial Prize in 1959. In 1960 I refused an appointment as Senior Professor in the Department of Inorganic Chemistry at the University of Marburg. In 1964 I took the Chair of Inorganic Chemistry at the Technical College of Munich, which had been vacated by Professor Hieber. In the same year I was elected a member of the Mathematics/Natural Science section of the Bavarian Academy of Sciences; in 1969 I was appointed a member of the German Academy of Scientists Leopoldina. In 1972 I was given an honorary doctorate by the Faculty of Chemistry and Pharmacy of the University of Munich. Lectures on my fields, particularly those on metallic complexes of cyclopentadienes and indenes, metal-pie-complex s of six-ringed aromatics, mono-, di- and oligo-olefins and most recently metalcarbonyl carbene and carbyne complexes, led me on lecture tours of the United States, Australia, Venezuela, Brazil, Israel and Lebanon, as well as numerous European countries, including the former Soviet Union. In 1969 I was Firestone Lecturer at the University of Wisconsin, Madison,Wisconsin, USA; in 1971 Visiting Professor at the University of Florida, Gainesville, USA, as well as the first Inorganic Chemistry Pacific West Coast Lecturer. In the spring of 1973 I held lectures as the Arthur D. Little Visiting Professor at the Massachusetts Institute of Technology, Cambridge, Massachusetts, USA; and that was followed by a period when I was Visiting Distinguished Lecturer at the University of Rochester, Rochester, New York, USA.

Nobel lecture: On the way to carbene and carbyne complexes. Angew. Chem. (1974), 86(18), 651-63.

Karl Heinz Dotz


Kekule-Institut fur Organische Chemie und Biochemie der Universitat Bonn Born1943 Ph.D. Technical University of Munich (E.O. Fischer) 1971. Habilitation Technical University of Munich 1980. Professor of Organometallic Chemistry University of Marburg 1986-1992, Dean of the Faculty 1990-1991. Professor of Organic Chemistry University of Bonn since 1992. Karl's research is focused on the following areas: Synthetic Organometallic Chemistry (metal carbenes and planar-chiral arene complexes) Physical Organic and Organometallic Chemistry (distorted fused arenes and cyclophanes: synthesis and structure-chiroptics correlation,transition metal NMR spectroscopy, Ab initio-calculations on organometallic complexes and intermediates) Metal-Mediated Organic Synthesis (stereoselective C-C formation via metal carbenes, diastereoselective benzannulation and cyclopentannulation) Organometallic Catalysis (chromium-catalyzed cyclopropanation, axial-chiral and redox-active biaryl ligands) Transition Metal Modified Sugars (metal glycosylcarbenes and glycosylidenes: synthesis and application in C-glycosidation, disaccharide mimetics,conformation of acyclic metal modified sugars) 01a contributors1 2/13/99 8:42 AM

Major Contributors

Claude F. Bernasconi
University of California at Santa Cruz Claude was born in Zurich, Switzerland. He received his undergraduate and Ph.D. degrees from the Swiss Federal Institute of Technology (ETH) with Heinrich Zollinger. Following a postdoctoral year with Manfred Eigen at the Max Planck Institute for Biophysical Chemistry in Gottingen, he joined the chemistry faculty at the University of California at Santa Cruz in 1967, where he has been a professor of chemistry since 1977. His main research interests are in physical organic chemistry and center on problems of mechanism, structure-reactivity relationships, intrinsic barriers of reactions, and catalysis in organic and organometallic reactions, particularly proton transfer reactions, nucleophilic addition to electrophilic alkenes, nucleophilic vinylic substitution and reactions of Fischer carbene complexes.

Charles P. Casey
University of Wisconsin-Madison Born 1942, St. Louis, MO B.S. 1963, St. Louis University Ph.D. 1968, MIT Chuck received a Ph.D. in organic chemistry from MIT in 1968, where he studied organocopper chemistry under the direction of Professor George M. Whitesides. After spending 6 months at Harvard as an NSF postdoc with Paul D. Bartlett he joined the faculty at Wisconsin. Chuck is interested in studying the mechanisms of organometallic reactions and in developing an understanding of homogeneous catalysis. In addition, he is trying to design new organometallic reagents for synthesis and new heterobimetallic catalysts.

William D. Wulff
University of Chicago Born Eau Claire, Wisconsin, 1949 B. S. 1971, University of Wisconsin-Eau Claire Ph.D. 1979, Iowa State University Professor Wulff received his Ph.D. degree from Iowa State University in 1979 with Professor Thomas Barton. After NIH postdoctoral work with Martin Semmelhack at Princeton University, he accepted a position at the University of Chicago in 1980. Professor Wulff's research interests are in the applications of organometallics in organic synthesis as both reagents and catalysts.

Jose Barluenga
University of Oviedo Jose Barluenga obtained his Ph.D. degree (solvomercuration of dienes) at the University of Zaragoza in 1966 under the direction of Professor V. Gomez-Aranda. He spent 3.5 years as a postdoctoral fellow at Max Planck Institut Fur Kohlenforschung, Mulheim, in the group of Professor Hoberg studying aluminum chemistry. In 1970 he took a position as a research associate at the University of Zaragoza, where he was promoted to Associate Professor in 1972. In 1975 he moved to the University of Oviedo as Professor of Organic Chemistry in the Department of Organometallic Chemistry. His major research interest is focused on the development of new synthetic methods in the area of heterocyclic chemistry and functionalized systems.

Louis S. Hegedus
Colorado State University Born Cleveland, Ohio, 1943 B.S, 1965, Penn State University M.A. 1966, Penn State University Ph.D., 1970, Harvard Lou was born in 1943 in Cleveland, Ohio, but grew up in rural Ohio, away from big city temptations. He did his undergraduate studies at Pennsylvania State University, where studied aqueous chromium redox chemistry with Professor Albert Haim. After Ph.D. studies at Harvard on nickel carbonyl chemistry with E. J. Corey (1970), and a NIH postdoctoral year at Stanford with J. P. Collman studying polymer-supported homogeneous catalysis, he moved to Colorado State University, where he remains today as a professor of chemistry. His research interests center on the use of transition metals in organic synthesis.

01b contributors2 2/13/99 8:40 AM

B. Connell

Fischer Carbenes
Recurring Themes Resonance
OCH3 CO OC OC CO Cr R OCH3 (CO)5Cr R (CO)5Cr R OCH3

Chem 206

The Wall of CO
CO

Rotation about heteroatom carbene bond is restricted by 14-25 kcal/mol 53Cr NMR is consistent with strong resonance contribution Hegedus and Dotz JACS, 1988, 8413. Kinetic Electrophilicity
OCH3

(CO)5Cr is sterically very large

HOCH3

OCH3 (CO)5Cr OCH3 CH3 H

(CO)5Cr CH3

Formation of tetrahedral intermediate is 109 faster than CH3O addition to BnO2CCH3.

Bernasconi Chem. Soc. Rev., 1997, 299. JACS, 1998, 8632.

pKa Data Thermodynamic Acidity


H3C N (CO)5Cr CH3 CH3 O CH3 H3C N CH3

OCH3 (CO)5Cr CH3

pKa(THF) = 8 pKa(H2O) = 12.3 equivalent to p-cyanophenol

pKa = 20.4 (DMSO)

pKa = 35 (DMSO)

Casey, JACS, 1974, 1230.

05/06 12/20/99 4:22 PM

B. Connell

Fischer Carbenes
Sensitivity to Acid

Chem 206

OCH3 (CO)5Cr CH3

pyridine

H3CO

(CO)5Crpyr

pyridine
H (CO)5Cr CH2

reductive elimination

OCH3 (CO)5Cr

pyridinium

OCH3 CH2

See Hegedus, JACS, 1990, 6255.

Metal Removal and Functionalization


O

OCH3 (CO)5Cr Ar

CH3Li, ClCH2I
Ar CH3

Barluenga, TL, 1994, 9471. 80-90%

OCH3 (CO)5Cr R

CH2N2
R

OCH3

Casey, TL, 1973, 1421.

OCH3 (CO)5Cr Ar

Ph3PCH2

OCH3 H2C Ar

Casey, JACS, 1972, 6543.

Via similar intermediates:

OCH3 (CO)5Cr Ar (CO)5Cr

OCH3 R (CO)5Cr Ph3P

OCH3 Ar

Cl

N2

07/08 12/20/99 4:22 PM

B. Connell

Fischer Carbenes
Selected Reactions of Unsaturated Fischer Carbene Complexes
OCH3 R1 R2 OCH3 R3 (CO)5Cr R1 R2 Nu R3 R1 R4 R R
3 5

Chem 206

OH R
3

(CO)5Cr

Nu
4

OCH3

R2 = H
R2 R HN R4 R
5

O R2 R

OCH3 (CO)5M R1 R2 R3
2 3

R ,R H

R1 OCH3

H3CO

R4 R R4

R2 = NR2
R OCH3

Bu3SnH
R3 R2 R2 R1 R3 R1 OCH3 R4

OCH3 Bu3Sn O R R
3

Selected Reactions of Saturated Fischer Carbene Complexes


OCH3 CH3 XR6 (CO)5M OH OCH3 CH3 O H3C R2 R3 R
1

R1

OCHR2R3 OCHR2R3

O O R

RCHO
(CO)5M XR6 R O (CO)5M CH3 R OCH3 R1 N XR (CO)5M R R3Sn XR
6 6

XR6 O

CH3

R O

XR6 = OCH3, N(R5)2 RX R3SnCl

R2 R2 N XR6 R1 CH3 O

09/10 12/20/99 3:51 PM

B. Connell

Fischer Carbenes
Cyclopropanation
CO2CH3 OCH3 (CO)5Mo n-Bu n-Bu

Chem 206

THF, 65 C 78%

OCH3

CO2CH3

E:Z 1.9:1

Harvey, TL, 1990, 2529.

OCH3 (CO)5Mo

OTBS CH3

25 C, 49%
CH3

TBSO

OCH3

>95% cis Wulff Pure Appl. Chem., 1988, 137. JACS, 1988, 2653.

Cyclopropanation Reaction with Alkynes


R2 OCH3 (CO)5Cr R2 R1 R R1
2

OCH3 M(CO)4 R1

OCH3 M(CO)4

OCH3 (CO)5Cr n-Bu O O

n-Bu

OCH3

PhH, 100 C 81%

Harvey, JACS, 1992, 8424.

PhH
OCH3 (CO)5Cr

reflux 97%

OCH3

Hoye, JACS, 1988, 2676.

11/12 12/20/99 3:54 PM

B. Connell

Fischer Carbenes
Selective Cyclopropanation

Chem 206

OCH3 Ph N H t-Bu (CO)5Cr Ph Ph Ph H H OCH3 H N t-Bu

Ph

OCH3 H O H

SiO2

Ph H

THF, 80 C, 3 h

OCH3 (CO)5Cr n-Bu n-Bu H OCH3

O3

n-Bu H

OCH3 O H

Fe

DMF, 152 C, 4% BHT 30 min 88%, 97% de

Fe

Barluenga Chem. Commun., 1995, 665. JACS, 1997, 7591. Cyclopropanation Reaction with Alkynes
O

O CH3 O Mo(CO)5 CO2Et

PhH
65 C, 55%
O CH3 H CO2Et

H O CH3 CO2Et

Harvey, JOC, 1992, 5559.

13/14 12/20/99 3:55 PM

B. Connell

Fischer Carbenes
[4 + 3] Annulation
Cr(CO)5 Ph NHt-Bu Et N H Ph OCH3 NHt-Bu

Chem 206

-78 C to -40 C 91%


H3CO N Et

[2 + 1]

Ph

Ph

NHt-Bu

[3,3]
HN OCH3 Et NHt-Bu H3CO N H

Et

Barluenga Chem. Commun., 1994, 321. JACS, 1995, 9419. JACS, 1996, 695. Chem. Eur. J., 1996, 88.

Diels-Alder Cycloaddition

H3CO OCH3 O CH3 O

25 C, 7 mo 54%
H3C

70:30 regioselectivity

H3CO OCH3

10 times faster than methyl acrylate

CH3

(CO)5Cr

(CO)5Cr

25 C, 3 h 70%
H3C

92:8 regioselectivity

Wulff, JACS, 1990, 4550.

15/16 12/20/99 3:56 PM

B. Connell

Fischer Carbenes
Diels-Alder Cycloadditions
(OC)5Cr CH3 H3CO

Chem 206

Me

25 C, 2 h 85%
H3CO

Cr(CO)5

O CH3 H3CO

Me O H3CO

170 C, 1 h 22%

O Si(CH3)2 O H H3CO Si(CH3)2 H3CO

Wulff JACS 1983, 6726. JACS 1990, 3642.


W(CO)5 Si(CH3)2 O H

120 C, 48 h
H3CO O
addition of LA causes decomp

25 C, 2 h 100%
H3CO W(CO)5

Diels-Alder Cycloadditions

CH3 Me Cr(CO)5

+ 25 C, 2 h
H3CO Cr(CO)5

87%

Me OCH3 H3CO Cr(CO)5

9:1

H3C H3CO (CO)5Cr

+ 25 C, 2 h 88%
H3CO Me Cr(CO)5

Me Cr(CO)5 OCH3

8.5:1.5

Wulff, JACS, 1990, 3642.

17/18 12/20/99 3:57 PM

B. Connell

Fischer Carbenes
Asymmetric Exo-Selective Diels-Alder Reaction
CH3 N O N Ph OCH3 CH3 TBSO O N (CO)4Cr H3CO OTBS O H3C OCH3 O HC M C N O Ph H3C NCH3 O C C Ph CH3 CH3 N CH3

Chem 206

(CO)4Cr CH3 TBSO

25 C, 12 h, 80%

exo:endo >96:4

non-oxygenated dienes give ~85:15 exo:endo


O

Wulff, JACS, 1997, 6438.

Proposed Transition State

[2 + 2] Cycloaddition

O H3C H3CO W(CO)5 O W(CO)5 H3C

DMSO
10 min 95%

25 C, 6 h 97%
CH3 CH3

CH3

corresponding ester does not react

O H3CO O O

H3C O O CO2CH3

+ 180 C, 16 h
O OCH3 H3CO O CO2CH3

36%

24%

Wulff, JACS, 1988, 8727.

19/20 12/20/99 3:58 PM

B. Connell

Fischer Carbenes
[3 + 2] Cycloaddition

Chem 206

OCH3 H3CO M(CO)5

OCH3 CH3

TMSCHN2
25 C, 2 h 76 - 87%
CH3

(OC)5M HN N

CAN
97% 30 min

O HN N

CH3

>300:1 regioselection

H3CO

OCH3

H3CO CH3 H3C O

TMSCHN2
70 C, 5 d
CH3

O HN N

+
HN N

25%

48% Wulff, JACS, 1986, 6726. Barluenga TL, 1998, 4887. Barluenga JCS Perkin I, 1997, 2267.

Dotz Reaction Thermal Reaction of Unsaturated Carbene Complexes "...one of the most utilized reactions in natural product synthesis involving an organometallic process."

OH OCH3 (CO)5Cr RL RS

RL

CO
(CO)3Cr OCH3

RS

Observed Connectivity:
O RL

RS H3CO (CO)5Cr

Dotz ACIEE, 1975, 644. New J. Chem., 1990, 433. ACIEE, 1984, 587.

21/22 12/20/99 3:59 PM

B. Connell

Fischer Carbenes
Dotz Proposed Reaction Mechanism
OCH3 OCH3

Chem 206

CO
(CO)4Cr

OCH3

RL

RS

(CO)4Cr RL RS

(CO)5Cr

S = +6.2 e.u. H = +26 kcal/mol rate-limiting CO dissociation

OCH3 (CO)3Cr RS (CO)4Cr RL RS OCH3 (CO)4Cr

OCH3

RL

RL

RS

O OCH3 RL RS Cr(CO)3 HO OCH3 RL RS Cr(CO)3

800 psi CO

HO

OCH3 RL RS

Cr(CO)6

Barluenga, JACS, 1994, 11191. Dotz Alternative Workup Procedures air


HO OCH3 RL RS

or FeCl3

HO

OCH3 RL RS Cr(CO)3

(NH4)2Ce(NO3)6 H2O

RL

RS

(NH4)2Ce(NO3)6
O

OCH3 OCH3 RL RS

CH3OH

23/24 12/20/99 4:00 PM

See Wulff, JOC, 1984, 2293.

B. Connell

Fischer Carbenes
Dotz: Nitrogen Analog

Chem 206

N N (CO)5Cr CO2Et H Ph

THF, 60 C
then SiO2 95%
Ph OH CO2Et

Wulff, JOC, 1995, 4566. Barluenga, JOC, 1998, 7588.

Dotz: Large Scale Applicability

OCH3 (CO)5Cr Ph

1.5 equiv.
n-Bu

OAc n-Bu

THF, reflux, 45-60 min 1.1 equiv. Ac2O 1.1 equiv. NEt3 0.16 equiv. DMAP 400 g scale 68% Biaryl Synthesis
OCH3

U-66,858 lipoxygenase inhibitor

Timko TL, 1988, 2513. Org. Synth., 1992, 72.

OCH3

2 equiv.

(CO)5Cr R2

H R3

R1

R1

OCH3 R1 R1 HO R
2 3OH

R3

R2 OCH3 R

The concept works, in moderate to low yield, but the reactions must be run stepwise. Occasionally CO insertion is suppressed and five-membered rings are formed.

25/26 12/20/99 4:00 PM

Wulff, JACS, 1996, 2166.

B. Connell

Fischer Carbenes
Intercepted Intermediates

Chem 206

(CO)6Cr H3CO

CH3 Et Et Et

O H3C

37%
H3C OH Et

CH3 OCH3

via

HO O H3C Et OCH3 CH3 Et Et Cr(CO)3 Et OCH3

CH3 O CH3

HO

OCH3 Et Et Cr(CO)3

Wulff, Chem. Commun., 1996, 1863.

Synthetic Uses

OCH3 TMS OCH3 OCH3 Cr(CO)5 H3CO2C

OCH3 CH3 TMS

OCH3 OH OCH3 CH3

CH3CN, 45 C, 24 h
66%
OCH3 OCH3

CO2CH3

O OCH3 CH3

OH

OCH3

menogaril antitumor antibiotic

Wulff, JOC, 1998, 840.

27/28 12/20/99 4:05 PM

B. Connell

Fischer Carbenes
Synthetic Uses 1)
Ot-Bu O OCH3 AcO

Chem 206

AcO

OTf

CH2Cl2, 45 C, 24 h
H3CO Cr(CO)4

O CH3O OCH3 Ot-Bu

2) Tf2O, pyr 67%

H O disaccaride R OH OH O

OCH3 OH OH O CH3

trisaccaride

R = CH3 Chromomycin R = H, Olivomycin antitumor antibiotics Application to Synthesis


TMS

Wulff, Synthesis, 1999, 80.

8 equiv. OTMS

TMS (CO)5Cr

OTMS

(CO)5Cr H3CO

1.5 equiv.

O H3CO O O O O TMS OTMS

50 C, THF, 3 d ~25%
OCH3

OH

O CH3 OH

O O OTMS OTMS

OCH3 O

OH

OH OCH3

Daunomycinone

29/30 12/20/99 4:05 PM

Wulff, JACS, 1984, 434 & 7565.

B. Connell

Fischer Carbenes
Dihydrofluorene Synthesis
OCH3 H3CO H3C W(CO)5 H3C

Chem 206

-20 C to rt +
N O Ph

12 h >95%
O

OCH3 H3C H3C W(CO)5 N O

OCH3 OCH3 W(CO)5 H3C

N O

N O

Barluenga, Chem Commun., 1995, 1973.

Alkyne-Alkene Reactions 1)
CH3 OCH3 (OC)5Cr CH3 H3C H3C O O

CH3CN, 70 C, 3 h 83% 2) H+

CH3

H+

O H3C (OC)5Cr H3C CH3 OCH3 H 3C

H3C OCH3 H3C Cr(CO)3 CH3

O H3C OCH3 CH3

Wulff, JOC, 1993, 5571.

31/32 12/20/99 4:06 PM

B. Connell

Fischer Carbenes
Ketene Cyclizations

Chem 206

OCH3 Cr(CO)5

OCH3

h 60 - 90%

OH

Via:
CH3O Cr(CO)4 O

Merlic, JACS, 1992, 5602.

"Asymmetric" Benzopentaannulation

CH3 H3C O (CO)5W Ph CH3 Ph Li H3C

R *O

then MeOTf quench

90%, 2:1 diastereoselectivity

Ph

R *O

H3C W(CO)5 Ph

OR* (CO)5W Ph

R *O

Ph

R *O

[1,3]

MeOTf
Ph

H3C W(CO)5

(CO)5W Ph

Ph

Barluenga, JACS, 1998, 12129.

33/34 12/20/99 4:07 PM

B. Connell

Fischer Carbenes
Furan Synthesis

Chem 206

R3

1)
O R1 R2 R4 (CO)5Cr

OCH3 CH3

CH3 O R4

66 - 84%
R1 R2

2) H3O

H3O+

R4

R2 R1

R4

R2 R1

R2 R1 R4 O R3

O (CO)4Cr H3C

O R3 (CO)4Cr H3C R3

OCH3

H3C OCH3

OCH3

Herndon, JOC, 1998, 4564.

"A Versatile [4 + 2 + 1 - 2] Cycloaddition"


O OCH3 (CO)5Cr RL RS RL

99:1 dioxane:H2O 8 h,

40 - 90%
RS OCH3

O (CO)4Cr RL OCH3 RS RL RL RS (CO)3Cr O OCH3 RS (CO)3Cr RL O OCH3 RS CH3O Cr (CO)3

loss of H2C CH2

O OCH3 (CO)5W RL

same conditions 65%


O RL

OCH3 RS OCH3

Cr0, H2O

RL

RS

OCH3

RS

Herndon JACS, 1988, 3334. TL, 1989, 295. JOC, 1990, 786. JACS, 1991, 7808. JACS, 1992, 8394.

35/36 12/20/99 4:08 PM

B. Connell

Fischer Carbenes
Photochemistry

Chem 206

Electronic absorption consists of three low-lying bands: ~500 nm: spin-forbidden M carbene * charge transfer transition 360-450 nm: spin allowed M carbene * charge transfer transition (visible) 300-350 nm: ligand field transition In addition, all carbene complexes absorb strongly below 300 nm.

Exposure to light leads to a reversible CO insertion:

OCH3 (CO)4Cr OC CH3

OCH3 (CO)4Cr CH3 O

H3CO (CO)4Cr O

CH3

Geoffroy, JACS, 1983, 3064.

Molecular Orbital Diagram


OCH3 (CO)5Cr (CO)5Cr CH3 CH3 OCH3

LUMO is carbene carbon p-orbital centered

HOMO is metal-d-orbital centered

Photolysis results in a formal, reversible, one electron oxidation of the metal.

Geoffroy, JACS, 1983, 3064.

37/38 12/20/99 4:09 PM

B. Connell

Fischer Carbenes
Ketene [2 + 2]
S OCH3 (CO)5Cr CH3 N H3C H3CO O H S N

Chem 206

h 81%

Hegedus Tetrahedron, 1985, 5833. JOC, 1997, 3586.

Ph (CO)5Cr N O O

OCH3 C8H17 C8H17 H3CO O

Ph

N O

MCPBA
92%

Ph C8H17 H3CO N O O O O

h 84%, 97% de

Hegedus JOC, 1995, 3787. JOC, 1996, 6121. Organometallics, 1997, 2313. JOC, 1998, 4691 & 8012.
H2N

TBAF 90%
H3CO C8H17

1) NaOCl 30%
C8H17 O

2) NH3 100%
O

(+)-Cerulenin Ene Carbamate Synthesis


Ph

OH HN (CO)5Cr CH3 PhO OPh Ph

2 equiv. NaH
O N O O

88%

O O N (CO)4Cr CH3 Ph

PhONa

O N (CO)4Cr CH2 Ph

PhOH

O N (CO)4Cr H CH2 Ph

39/40 12/20/99 4:10 PM

Hegedus, JACS, 1990, 6255.

B. Connell

Fischer Carbenes
Amino Acid Synthesis
H3C H3C (CO)5Cr CH2R O

Chem 206

h, t-BuOH
N Ph

R Ph N

Ot-Bu CH3 CH3

60-80% 97% de

Mutiply-labeled amino acids are readily prepared from (13CO)6CO and CH3OD:
O
13

R Ph

13

C Ot-Bu

C D CH3 CH3

Hegedus JACS, 1990, 2264. JACS, 1992, 5602. JACS, 1993, 87. Acc. Chem. Res., 1995, 299. JOC 1995, 5831. JACS, 1995, 3697. JOC 1997, 7704.

Peptide Synthesis

H3C

CH3 N H CH3 CH3 CO2t-Bu

h
H3C (CO)5Cr CH3 N Ph H3C NH2 CO2t-Bu Ph

H3C N

68 - 88% yield 80-96% de

Solid Support Merrifield Resin: Acc. Chem. Res., 1995, 299. PEG: JOC 1995, 5831; JOC 1997, 7704.

Reactions have been performed and do work, but are not as practical because the failure to achieve 100% yields and high diastereoselectivity limit this method. hard to work with because the polymer "sticks to everything, making quantitative transfer difficult". Acc. Chem. Res., 1995, 299.

41/42 12/20/99 4:10 PM

B. Connell

Fischer Carbenes
Aryl Glycines
Ph Ph HN Ar H N Ph

Chem 206

h
OH (CO)5Cr HO Ph O

Ar

H N

Ph

(CO)5Cr Ar

Ph

H2, PdCl2

Aryl glycines cannot be made via the previous route due to instability of the required carbenes:
H3C H3C (CO)5Cr Ar

H3N Ar

COO

N Ph

40 - 95% yield 56 - 98% ee

Hegedus, JOC, 1992, 6914.

Tertiary Amino Acids


O O N N Ph H H3C

H3C H3C (CO)5Cr

O N

Ph O N O

H3C

1) 0.2 M HCl 2) phosgene

O N O CH3O2C

Ph R O H

O N

Ph R O N O O O Ph N O N O

HCl, CH3OH

1) KHMDS 2) RX, DMF

tertiary amino acids Hegedus, JOC, 1993, 5918.

43/44 12/20/99 4:11 PM

B. Connell

Fischer Carbenes
Zwitterionic Aza Cope Rearrangement
OCH3 (CO)5Cr CH3 N Bn Bn N O OCH3 CH3

Chem 206

h 71%

91:8

81:19

H3CO (CO)4Cr O Bn

CH3

H3CO

CH3

[3,3]
N O Bn N

Hegedus, JOC, 1996, 2871.

Michael Reaction
(CO)4 Cr N N NCH3 O NCH3

(CO)4Cr

O OLi

-40 C R Ph CH3 Yield 81% 92% syn:anti 99.5:0.5 98.3:1.7 Wulff, JACS, 1993, 4602.

O N(CH3)2 (CO)5Cr CH3

a) n-BuLi b)
O (CO)5Cr (CH3)2N

DMSO
71%
(CH3)2N

-78 C, 20 min 85% Heathcock, JOC, 1986, 279.

45/46 12/20/99 4:12 PM

B. Connell

Fischer Carbenes
Asymmetric Michael Addition
OLi Ph H3C O CH3
(CO)4 Cr

Chem 206

O N NCH3 CH3

(CO)4Cr
N H3C

O NCH3

-20 C
Ph CH3

90% 3:1diastereoselection
O R1 R2 R2
(CO)4 Cr

Ph

(CO)4Cr
H3C N

O NCH3

a) n-BuLi b) O
R1

O N NCH3

Ph

CH3

Ph

CH3

80% >96:4 diastereoselection


(CO)4 Cr

O R1

CH3

O O

1) CAN
N NCH3

CH3

O OCH3

2) NaOCH3
Ph CH3

R1

Wulff, Chem. Commun., 1996, 2601.

Asymmetric Michael Reaction


OLi OCH3 CH3 (CO)5Cr CH3 OCH3 OR*

Ph

74%, 84% de
Ph H3C O (CO)5Cr Ph CH3 CH3

OLi H3C Ph

OR* (CO)5Cr

Ph

O Ph CH3

89%, 99% de Barluenga, Chem. Eur. J., 1995, 236.

47/48 12/20/99 4:12 PM

B. Connell

Fischer Carbenes
Michael Reactions: Selectivity

Chem 206

CH3 OC CO CO CO NCH3 OC OC CH3 CO R Cr H3C O CH3

R1 O

O OC Cr H CO C R H
2

Ph

Open Transition States are Postulated

Pyridine Synthesis
H EtO W(CO)5 H3C HN O N CH3 CH3 O

then HBF4
Ph

CH3 Ph

75%

HBF4

CH3 HN (CO)5W CH3 Ph O

W(OC)5 (CO)5W EtO EtO H H N H CH3 O Ph H CH3 H3C O NH

EtO

W(OC)5 HN CH3

EtO

W(OC)5 HN CH3

CH3

H O H3C H3C

Aumann, Synlett, 1993, 669.

49/50 12/20/99 4:13 PM

B. Connell

Fischer Carbenes
Alkylation
OCH3 (CO)5Cr CH3

Chem 206

a) n-BuLi b) MeI 22%


(CO)5Cr

OCH3

Hegedus Org. Synth., 1987, 140.


CH2 CH3

OCH3 (CO)4Cr Bu3P CH3

a) n-BuLi b) EtI 65%


(CO)4Cr Bu3P

OCH3

OCH3

+
CH3

(CO)4Cr Bu3P CH3 CH3

7:3

Bu3P substitution often leads to different reactivity: TL, 1995, 8159.

OCH3 (CO)5Cr CH3

a) n-BuLi b)
OTf (CO)5Cr

OCH3

Wulff, JOC, 1987, 3263.

80%

N (CO)5Cr CH3

a) n-BuLi b) EtI 87%


(CO)5Cr

N CH3

Wulff, TL, 1989, 4061.

"Aldol" Reaction
OCH3 (CO)5Cr CH3

a) n-BuLi
(CO)5Cr

OCH3 OH

Wulff, JACS, 1985, 503.

b) 10 equiv. PhCHO/BF3OEt2 81%

Ph

a) n-BuLi
N (CO)5Cr CH3 N

b) 1.1 equiv. PhCHO/BF3OEt2 96%

(CO)5Cr

OH

Wulff, JACS, 1989, 5485.

Ph
Ketones work as well.

(CO)4Cr
H3C N

O NCH3

a)n-BuLi b) Bu3B c) i-PrCHO

OH H3C CH3

O
N

O NCH3

Wulff, JOC, 1994, 6882.


Ph CH3
oxazolidinone complexes were unstable: Hegedus, JACS, 1990, 6255.

Ph

CH3

then CAN 74%

ds: 98.6: 1.4

51/52 12/20/99 4:13 PM

in absense of Bu3B, ds: 91:9

B. Connell

Fischer Carbenes
Ene Reaction

Chem 206

OCH3 H3CO W(CO)5 OR (CO)5W CH3 (CO)5W RO RO

OCH3 CH3

+
CH3

25 C, 4 h R = CH3 90% R = TBS 96% + R = CH3 23: 77 R = TBS 90:10

Wulff, JACS, 1990, 6419.

Reaction of Ketene Acetals


H3C CH3

1)

OCH3 OCH3 H3C

O O

80 C, 3 d
O (CO)5Cr CH3

24:1 trans:cis
CH3 H3C

2) aq. HCl 50%

()-eldanolide sex pheromone

H3C CH3 H3C CH3 O (CO)5Cr CH3 OCH3 OCH3 (CO)5Cr CH3 OCH3 H O OCH3 H3C

CH3

O H3C OCH3 OCH3

Wulff, JACS, 1992, 10665.

53/54 12/20/99 4:14 PM

B. Connell

Fischer Carbenes
Vinylsilane Synthesis

Chem 206

RCH2Li or RCH2MgX

CeCl3
-78 C, cold quench
OCH3 (CO)5Mo SiPh2CH3 H3CPh2Si

R = i-Pr, 94%, 96:4 E:Z R = Bu, 86%, 96:4 E:Z R = i-Bu, 85%, 97:3 E:Z

RCH2Li or RCH2MgX

CeCl3, HMPA
-78 C to rt

H3CPh2Si R

R = i-Pr, 76%, 92:8 Z:E R = Bu, 79%, 97:3 Z:E R = i-Bu, 61%, 97:3 Z:E

Iwasawa, Chem. Lett., 1994, 231.

Vinylsilane Synthesis Proposed Mechanism

OCH3 (CO)5Mo SiPh2CH3

n-BuLi

Li (CO)5Mo

OCH3

H2O
-78 C

H (CO)5Mo

OCH3

n-Bu
SiPh2CH3

n-Bu
SiPh2CH3

warm LiOCH3

HOCH3

n-Bu
Li (CO)5Mo SiPh2CH3

CH3OLi

n-Bu (CO)5Mo SiPh2CH3

(CO)5Mo SiPh2CH3

hydrogen migration

H SiPh2CH3 SiPh2CH3 (CO)5Mo SiPh2CH3

55/56 12/20/99 4:14 PM

B. Connell

Fischer Carbenes
Allyl Stannane Synthesis

Chem 206

W(CO)5

SnBu3

Bu3SnH, pyr
H3C OCH3

73%

H3C

OCH3

Ph H3CO

Cr(CO)5 OCH3

Bu3SnH, pyr
69%
H3CO

Ph

SnBu3 OCH3

1,1 addition is usually observed, however 1,3 addition is possible:

W(CO)5 OCH3 TMS

Bu3SnH, pyr
88%

Bu3Sn TMS

OCH3

E:Z 1:1

W(CO)5

Bu3SnH, pyr
MOM Me

Bu3Sn Me

OMOM

66%

Merlic, TL, 1995, 1007.

Halban-White Cyclizations
O Ph Ph O OPh O Ph OPhO Ph PhO Ph Ph O O O O Ph

Ph

PhCO

COPh

PhCO

COPh

Ph

Andres, A. Dissertation, Strasbourg, 1911. van Halban, Helv. Chim. Acta, 1948, 1899.

O OCH3 (CO)5M R4 R6 R5 R1 R2 O R1 O

R2

R6 R5 R4 R3

yields 30-70% Dotz byproduct

O O

6 R2 R

R5

R4 R H3CO
1

57/58 12/20/99 4:15 PM

Wulff JACS, 1990, 1645. JACS, 1991, 5459.

D. A. Evans

Introduction to Organosilicon Chemistry


Problems to Contemplate
Me Explain what drives this rearrangement. TBS = Si Me TBSO Bu3Sn OH
KHMDS

Chem 206

http://www.courses.fas.harvard.edu/~chem206/

Chemistry 206 Advanced Organic Chemistry


Lecture Number 36

CMe3

OH Bu3Sn OMe
THF, -78 C 94%

OTBS

Introduction to Organosilicon Chemistry


Silicon Bonding Considerations The SiliconProton Analogy C=O Addition of Organosilanes Sigmatropic Rearrangements of Organosilanes Anionic (Brook) Rearrangements Peterson Olefination Reaction Survey of Silicon (and related) Protecting Groups

OMe Me

Calter, M. A. Ph. D. Thesis, Harvard University, 1993.

The C=O addition illustrated in eq 1 proceeds while the carbon analogue (eq 2) does not. Explain TMS O RO P OR O R H Me O RO P OR fails! R R O SiR3 O P RO OR Me O P RO OR R OTMS O (1) P R RO OR

Reading Assignment for this Lecture:


Carey & Sundberg, Advanced Organic Chemistry, 4th Ed. Part B Chapter 9, " CC Bond Forming Rxns of Boron, Silicon & Tin", 595680. Fleming, I.; Barbero, A.; Walter, D. "Stereochemical control in organic synthesis using silicon-containing compounds." Chem. Rev. 1997, 97, 2063-2192. (Web) Moser, W. H. "The Brook Rearrangement in Tandem Bond Formation Strategies," Tetrahedron 2001, 57, 2065-2084 (handout) Masse, C. E.; Panek, J. S. "Diastereoselective reactions of chiral allyl- and allenylsilanes with activated C-X pi-bonds." Chem. Rev. 1995, 95, 1293-1316. Ager, D. J. "The Peterson olefination reaction." Org. Reactions 1990, 38, 1-224 Colvin, E. "Silicon in Organic Synthesis," Butterworths, 1981 Bois, et al. "SiliconTethered Reactions" Chem. Rev. 1995, 95, 1253-1277. (Handout) Me

OMe P O (2)

RO OR

Provide a mechanism for the indicated transformation OLi H O X Takeda, Org. Lett, 2000, 2, 903-1905 Me H X O OSiMe3

Me3Si

36-00-Cover Page 12/14/03 8:46 PM

D. A. Evans

Monday, December 15, 2003

D. A. Evans

Bonding Considerations: Carbon vs Silicon

Chem 206

Bonding Considerations: Carbon vs Silicon


Average Bond dissociation eneregies (Kcal/mol)

Hypervalent 5-Coordinate Silicon Compounds


Akiba, "Chemistry of hypervalent Compounds" Wiley-VCH, Chapters 4-5, 1999

CC 83

CSi 76

SiSi 53

CF 116

SiF 135

CO 86 CH 83

SiO 108 SiH 76

Penta-coordinate silicates are commonly observed


MeSiF4 NEt4

Average Bond Lengths ()

Ph3SiF2 NR4

CC 1.54
C C

CSi 1.87

CO 1.43

SiO 1.66

Nucleophilic substitution at Silicon


ROSiMe3 F RO
+

FSiMe3

C CC

better than

Si

Si

CSi

OSiMe3 KOCMe3
Si-SP3

O THF 20 2h

K Me3SiOCMe3

C-SP3 CC

C-SP3

C-SP3 CSi

Duhamel et al. J. Org. Chem. 1996, 61, 2232

H3CCH3 BDE = 83 kcal/mol Bond length = 1.534

H3CSiH3 BDE ~ 76 kcal/mol Bond length = 1.87

OSiMe3 MeLi THF

Li Me3SiMe

This trend is even more dramatic with pi-bonds: CC = 65 kcal/mol CSi = 36 kcal/mol
+

SiSi = 23 kcal/mol

Stork et al. JACS. 1968, 90, 4462, 4464

Thermal Rearrangements One may readily access divalent intermediates C Si


Me Me Si thermolysis Me Si CH2 Me Me Me Si H H Me thermolysis Si Me Me Si Me H2C CH2 Colvin, pp 7-9

Group IV Electronegativities (Pauling)


Carbon 2.55 Silicon 1.90 Germanium 2.01 Tin 1.96 Lead 2.33

+2 Oxidation state becones increasingly more stable

36-01-Si intro 12/14/03 8:46 PM

K. Scheidt, D. A. Evans

Hypervalent Silicon Ate-Complexes


F3C O Ph 1.689 Ph Si F CF3

Chem 206

1.647 CF3 F Ph F Si F Ph

Inorg. Chem. 1 9 8 4, 23, 1378 J. Am. Chem.Soc. 1987, 109, 476


S(NMe2)3

Cl 2.198 1.668 Me O 1.604 1.597 F F Ph Si F F 2.104

J. Organomet.Chem. 1981, 221, 137.


36-01a-5 coord Si(-) 12/14/03 8:48 PM

Acta Crystallogr. Sect. C 1 9 8 4, 4 0, 476

D. A. Evans

Bonding Considerations: Carbon vs Silicon Carbonyl addition Reactions The prospect of catalysis was investigated
Me3Si CN
+

Chem 206

1970 DAE Objective: Develop a reagent that will transform aldehydes into protected cyanohydrins in one step
O
+

ZnI2
1-5 min

OSiMe3 C5H11 CN H

O H

CN

reaction was instaneous and quantiltative

SiR3 G R

OSiR3 G H

LiNR2

OSiR3 R Li G

Me

1-5 min

G = carbanion-stabilizing FG

Carbonyl Anion Equivalent

Principle established that normally inaccessible cyanohydrin derivatives may now be accessed
Me Me Me OTMS Me Me
92% yield only 1,2-addition (ZnI2 catalysis)

TMSO CN Me

O
>95% yield only 1,2-addition (CN catalysis)

R3Si R3Si

G CN

Candidates

Carbonyl Adducts OSiR3 R H OSiR3 CN

CN
>95% yield (ZnI2 catalysis)

TMSO

CN

with Truesdale, Carroll, Chem Commun. 1973, 55; J. Org. Chem.. 1974, 39, 914 Tetrahedron Lett 1973, 4929 (first discussion of Nu catalysis)

R3Si

OSO2Ar

R H

SO2Ar OSiR3

"The Silicon Advantage" From the preceding case, it is clear that HSi is more exothermic than HH
O R + R X CN R R OX CN

R3Si

OPR2

R H

POR2

HSi > HH

Thermal C=O addition of TMSCN is not a clean reaction


Nucleophilic Catalysis
Me3Si CN
+

O R R R C N

O R CN Me3Si CN

O H

50 C 2-5 hr

OSiMe3 C5H11 CN ratio: 65:35 H


+

OSiMe3 C4H9 H

Me

OTMS C N + R CN R

36-02-Si-C=O addition 12/14/03 8:50 PM

D. A. Evans

Carbonyl Additon Reactions-2


Explain the following observations "The ProtonSilicon Correlation"

Chem 206

O + O O + O TMSCN benzene C N THF/H2O

OH

1-4 addition
CN OH O

Organosilanes undergo carbonyl addition processes in direct analogy with their proton counterparts but with an attendant greater exothermicity. Organosilanes undergo a range of thermal rearrangements processes in direct analogy with their proton counterparts.
X X H A. J. Ashe III, JACS 1970. 92, 1233 rt H

1-2 addition
C N TMSO CN

k(Si) =10+6 K(H)

SR R1 SR R2

O ZnI2 R1 Me3Si
+

RS R2 SR

OSiMe3 R1 SR R2 Me

Me3Si O O O MeMe Me O R N SiR3

heat Me Me O R

SiMe3 O Me Me C O Colvin, pp 37-8

with Truesdale, Grimm, Nesbitt, JACS 1975, 97, 3229 JACS 1977, 99, 5009 O R H O Me3Si N2 OEt CN or F
1-5 min

SiR3 N SiR3

OTMSO R N2 with Truesdale, Grimm JOC 1976, 41, 3335 OEt

G* 15-22kcal/mol
Si transfer is intramolecular

SiR3

Yoder et al., JACS 1974. 96, 4283 C N SiMe3

Non-catalyzed processes may also occur if a proper geometry for atom transfer can be achieved
TMS O H
+

Me3Si C N OTMS O P R RO OR

O R

O R

SiR3 O P

Organosilicon hydrides undergo transition metal catalyzed hydrosilylation processes in direct analogy with normal hydrogenation reactions
Me HSiR3 R Rh(I) catalysis Rh(I) catalysis Me HH R Me H

RO P OR O R

RO OR R RO TMSO O P OR

OTMS RO RO P O R

SiR3

with Hurst, Takacs JACS 1978, 100, 3467

"Hydrosilylation of CC Bonds". T. Hayashi In Comprehensive Asymmetric Catalysis, Jacobsen, E. N.; Pfaltz, A.; and Yamamoto, H. Editors; Springer Verlag: Heidelberg, 1999; Vol I, 319-332.

36-03-Si-C=O addition-2 12/15/03 8:09 AM

D. A. Evans

Carbonyl [1,2] & [1,3] Sigmatropic Rearrangements [1,3]-Sigmatropic Rearrangements


R3Si Y C R X Y R3Si C R X

Chem 206

"The Brook Rearrangement(s)"


A. G. Brook Accts. Chem. Research 1974, 7, 77-84 R3Si Ph C Ph OH Ph Ph C H SiR3 O

Et2NH DMSO

Y = C; X = O
Me Np Si Ph Ph O Me Ph Si Np Me O Ph 110 C Np Si Ph Ph O

Et2NH

Ea ~ 8-11 kcal/mol
SiR3

Complete retention of Si stereochemistry was noted. Ea = 28 kcal/mol

R3Si Ph C Ph O H N Et Et Et H Ph Ph

C H N

O H Et

A. G. Brook Accts. Chem. Research 1974, 7, 77-84

Brook speculates that a hypervalent Si intermediate might be involved in the rearrangement.

Brook has documented that retention at Silicon & inversion at Carbon occur.

Transformations Involving the Brook Rearrangement


Me CH2 H 500 C Ph Si H Moser, W. H. "The Brook Rearrangement in Tandem Bond Formation Strategies," Tetrahedron 2001, 57, 2065-2084

Y = C; X = C
Me Np Si Ph

Inversion of Si stereochemistry was noted. Ea = 48 kcal/mol

Acylsilanes
O R Cl Li SiR3 Cu(I) R O R2BH SiR3 [Ox] R SiR3

Np H 2C

H. Kwart et al., JACS 1973. 95, 8678 O R3Si R O El R

Theoretical calculations lead to the conclusion that the concerted [1,3] sigmatropic rearrangement with retention of Si-configuration should represent the lower energy pathway. Yamabe, JACS 1997, 119, 808 At the present time these rearrangements are not well studied,

SiR3

Li

El(+)

Li

O R

[1,2] Si R3Si R

Li R El(+)

R R3Si

36-04-Si/thermal Rearrangement 12/14/03 8:52 PM

D. A. Evans

Transformations Involving the Brook Rearrangements


Intramolecular alkylations may be carried out:
O R SiR3 (CH2)4I R OTMS C R R Li H3O+ O

Chem 206

Transformations Involving the Brook Rearrangement


Moser, W. H. "The Brook Rearrangement in Tandem Bond Formation Strategies," Tetrahedron 2001, 57, 2065-2084 O R SiR3 Li R El(+) R3Si R O El

Reich JACS 1980, 102, 1423

Li

O R

[1,2] Si R3Si R

Li R El(+) PhS O El R

O OLi SiR3 R PhS

OSiMe3

R R3Si

OH R

R3Si

s-BuLi R3Si

Li O R

El(+) R3Si

LiO

SiR3 [1,2] Si

OSiMe3

PhS Li O SiR3 O R O R

PhS

Takeda JACS 1993, 115, 9351; Synlett 1994, 178; SynLett 1997, 255

These reagents are useful homoenolate anion equivalents

CH2()

O SiR3 PhS

OLi Me3Si R R

OSiMe3

"Metalated Allylic Ethers as Homoenolate Anion Equivalents". Evans, D. A.; Andrews, G. C.; Buckwalter, B. JACS 1974, 96, 5560. SiVariant: Still & MacDonald JACS 1974, 96, 5561

OLi

Brook Equilibrium
Me3Si Ph Ph Me3Si Ph H C C

Reich JACS 1980, 102, 1423 (see footnote 8) THF OLi Ph Ph C Li THF OLi Ph H C Li SiMe3 O R SiMe3 O PhS O LiO SiR3 [1,2] Si PhS R O PhS OSiMe3

[3,3] OSiMe3

OLi

R Tetrahedron 2001, 57, 2065-2084, footnote 16

36-05-Brook Rearrangments-2 12/14/03 8:54 PM

D. A. Evans
The natural product target:
RO H Me CHO

Transformations Involving the Brook Rearrangements


The key reaction
OLi Me3Si Me X Me O Me3Si CH SMe Li Ph O Ph Me3Si CH2 SMe nBuLi TMEDA Me3Si CH SMe Li MeS Ph C H Ph

Chem 206

carbanion-stabiizing groups facilitate elimination H Me O OSiMe3 H X

Me

Elimination could also be effected with dilute acid


Me3Si OH R R

Takeda, Org. Lett, 2000, 2, 903-1905

The -Effect (lecture 32)


10% H2SO4 rt R R Me3Si OH

The Peterson Olefination Reaction


Ager, D. J. "The Peterson olefination reaction." Org. Reactions 1990, 38, 1-224

C Si

N P

analogy provided by Whitmore et al. JACS 1947, 69, 1551

The key paper: Peterson, J. Org. Chem. 1968, 33, 780-784


It was Peterson's intent to find a silicon analog to the Wittig rxn. The reaction concept is outlined below: O Me3Si CH2M R R Me3Si R OM R

Mechanistic aspects of Beta-OH Elimination


Me3Si H Pr H H+ H Pr H

Pr

Anti Elimination
Pr

OH KH

Me3Si

OM R R

M Me3Si O R R Me3Si H Pr

The -Effect (lecture 32)


OK H Pr Pr H H

Syn Elimination
Pr Colvin chapter 12, pp 141

Magnesium alkoxides: Stable O Me3Si CH2MgCl R R Me3Si R R R Me3Si OK (Na) OMgCl R these adducts are quite stable H Me3Si

Hudrlik et al. JACS 1975, 97, 1464

Na & K alkoxides: Eliminate Me3Si R OH R KH rt

O H Pr

R3Si R2CuLi H R

OH H Pr

KH

H R

H Pr

reaction is stereospecific note site of nu attack. Why?

36-06-Brook Rearrangments-3 12/15/03 8:18 AM

D. A. Evans

The Peterson Olefination Reaction


Me CH2 >90% O Me O H OH Me O O O H OH Me O Me OH

Chem 206

Simple Examples: Taken from Organic Rxns review


O Me3Si OH CH2MgBr SiMe3
NaH orTsOH

Miyakolide (+)-1
Evans et al. JACS 1999, 121, 6816-6826.

This reagent is better that H2C=PPh3 for hindered ketones Me Me

Miyakolide presents an interesting olefin geometry challenge

O Me

H2C Me

Boeckman, Tet. Lett 1973, 3437

OH H Me O Xp
1

OMe O O OMe Xp Me
1

Me OMe

O Me3Si OEt

O
LiN(TMS)2

TMS

OMe

OEt O

>95% Nozaki, JACS 1974, 96, 1620


9

O O OPMB

Conditions 78 C
9

OPMB 19-E

CO2Me

Me3Si

t-BuLi MgBr2

OH SiMe3 SOCl2 >90%

entry 1 2 3 4

base LDA NaHMDS LDA LDA

solvent THF THF Et2O PhMe

E:Z 73 : 27 18 : 82 66 : 33 66 : 33

Chan, Tet. Lett 1978, 2383

O Me O Me3Si B O Me Me RCHO Me R SiMe3


H+

OH H OLi CMe3 Me3Si n-Hexyl

O CMe3
BF3OEt2

OH Me3Si n-Hexyl

OH CMe3
KH

OH

KH

Me3Si n-Hexyl

Chan, Chem. Commun 1982, 969

R
Hudrlik, JACS 1981, 103, 6251

O n-Hexyl CMe3 n-Hexyl

OH CMe3

36-07-Peterson Olefination 12/14/03 8:57 PM

D. A. Evans
OTMS Me Me OTMS O TMSO O Me KN(TMS)2 OTMS Me Me OTMS O TMSO O Me N O Me3Si N

The Peterson Olefination Reaction


O O

Chem 206

Bunnelle-Peterson Allylsilane Synthesis


N Me R O O R R OH THPO Me 90% SiMe3 Ph 93% Me SiMe3 SiMe3 SiMe3 silica gel R SiMe3

(E):(Z) = 13:1 O O N Me (E):(Z) = 1:7

2 Me3Si

CH2M

M = Li M = CeCl2

t-BuLi OTMS Me Me OTMS O TMSO O Me O (t-Bu)Me Si 2

Application to Leucasandrolide: Rychnovsky JACS, 2001, 123, 8420


Me Me OH Me O OMe O O O Me Me Me Me O OMe O OH HO O OH O N

O N

Bell et al. Tetrahedron 1994, 50, 6643

Reaction may be altered significantly with an attendant change in stereoselection

Ireland Enolate Claisen Coupled to Peterson Olefination


R O O (See Lecture 15) SiMe3 OH OH R CO2Me SiMe3 KH BF3OEt2 CO2Me CO2Me R TMSO Sato et al. Chem. Lett 1986, 1553 TBSO O OEt TMSCH2MgCl CeCl3 silica gel TMSO TBSO syn:anti = 96:4 O O OBn TBSO

The seco acid


5.5:1 ratio Me

LiN(TMS)2 TMSCl CH2N2

Me H O

The Pivotal Step:


SiMe3 BF3OEt2 base, 78%

OH

OBn

OTBS

SiMe3 87%

36-08-Peterson Olefination 12/15/03 8:20 AM

J. Leighton, D. A. Evans

ROH Protecting Groups

Chem 206

Silyl Ethers: Relative stabilities:


Me Me Si OR Me
trimethylsilyl (TMS)

Et Et Si OR Et
triethylsilyl (TES)

Me t-Bu Si OR Me
tert-butyldimethylsilyl (TBS or TBDMS) TES ~102 times more stable to acidic hydrolysis than TMS TBS ~104 times more stable to acidic hydrolysis than TMS

t-Bu Ph t-Bu Si OR Ph
tert-butyldiphenylsilyl (TBDPS)

t-Bu Si O

i-Pr i-Pr Si OR i-Pr


triisopropylsilyl (TIPS)

Me

OSiR3

1% HCl in EtOH 22.5 C SiR3 Half-life < 1 min 18 min 244 min

Me

OH

O R

TBS TIPS TBDPS

di-tert-butyldimethylsilylene (DTBS)

Me

OSiR3

5% NaOH in EtOH 90 C SiR3 Half-life 1h 14 h <4h

Me

OH

Formation:
By far the two most common methods:

TBS TIPS TBDPS

R OH

R3Si-Cl, imidazole DMF, R.T.

R OSiR3 OSiR3
2 equiv TBAF THF, 22.5 C

2 equiv of imidazole are required relative to R3SiCl. Corey, E. J.; Venkateswarlu, A. J. Am. Chem. Soc. 1972, 94, 6190.

OH

R OH

R3Si-OTf, 2,6-lutidine CH2Cl2, 0 C

R OSiR3

SiR3 TBS TIPS

Half-life 76 min 137 min

Corey, E. J. et al., Tetrahedron Lett. 1981, 22, 3455.

Cunico, R. F.; Bedell, L. J. Org. Chem. 1980, 45, 4797-4798.

36-09-Si Protecting Grps-1 12/14/03 8:58 PM

J. Leighton, D. A. Evans
Selective Protection:

ROH Protecting Groups-2


Me OH
TBDPSCl, imidazole DMF, RT 95%

Chem 206

OH Me Me Me

OH Me Me

OTBDPS
TBSOTf, 2,6-lutidine

CO2Me Me OTBS Me Me OH HO OTBS Me OTBS HO


CH2Cl2, -78 C

OTBS Me

Me

Me

Me

CO2Me OTBS Me

71%

O MeO

OH

OH Me OTIPS
TESCl, DMAP CH2Cl2, 0 C 77%

O MeO

OTESOH Me OTIPS

Me

White, J. D. et al., J. Am. Chem. Soc. 1989, 111, 790.

Evans et al.JACS 1999, 121, 7540-7552.

HO CO2Me Me HO Me OH HO CO2Me OH H
Yankee, E. W.; Bundy, G. L. J. Am. Chem. Soc. 1972, 94, 3651. TMS-NEt2 acetone, -45 C

Me OH H
TBSOTf, 2,6-lutidine CH2Cl2, -78 C 91%

Me

Me TMSO Me OH

OH OTMS

OTBS OTMS

TMS-NEt2 has been reported to selectively protect equatorial alcohols in the presence of axial alcohols: Weisz, I. et al. Acta. Chim. Acad. Sci. Hung. 1968, 58, 189.

Askin, D.; Angst, D.; Danishefsky, S. J. Org. Chem. 1987, 52, 622.

OH

OH

O N

O
TBSOTf, 2,6-lutidine

HO SO2Ph
TBSCl, imid. DMF, RT 80%

HO SO2Ph Me Me Me

CH2Cl2, 0 C

92%

Bn TBSO
Evans, Ng JACS 1993, 115, 11446

OTBSOH

O N

O O

HO

Donaldson, R. E.; Fuchs, P. L. J. Am. Chem. Soc. 1981, 103, 2108.

Me

Me

Me

36-10-Si Protecting Grps-2 12/14/03 9:00 PM

J. Leighton, D. A. Evans
Selective Protection:
OH Me Me Me Me Bn
Evans, Ng JACS 1993, 115, 11446

ROH Protecting Groups-2


Selective Deprotection:
O
TBSOTf, 2,6-lutidine

Chem 206

OH

O N

Me
> 80%

Me

OMe O
Calter, M. A. Ph. D. Thesis, Harvard University, 1993

CH2Cl2, 0 C

TMSO O Me O O

OTBSOH Me Me Me Me

O N Bn

OTMS Me OMe Me Me
K2CO3

Me

OMe O

OH Me Me Me

OH

O N

O
TBSOTf, 2,6-lutidine

TMSO Me O

CH2Cl2, -10 C

MeOH 100% 83%

OH Me OMe Me

Me Bn OH Me Me Me

OTBSO N Me Bn

O O OTBS O (NCCH2CH2O)2P O Me Me O O PivO OMe OH O Me OTBS JACS 1992, 114, 9434


Evans, Gage, Leighton

Evans, D. A.; Dart, M. J. Unpublished

Me O

OH Me Me Me TESCl, imidazole
DMAP

Me H

Me H Me

H O HO MeO OTES Me Me

CH2Cl2, -78 C

Me H

Me H

Me H O HO MeO O
98%

(NCCH2CH2O)2P
HFpyr, pyridine THF 94%

Me Me O O

Me OTBS

Evans, Ratz JACS 1995, 117, 3448

PivO

OMe

36-11-Si Protecting Grps-3 12/14/03 9:01 PM

J. Leighton, D. A. Evans

ROH Protecting Groups-2


1,2-Migration:
OTBDPS
K2CO3 MeOH 100%

Chem 206

Selective Deprotection:

OH Me OTBDPS

Me

Me O

Me

Me

Me O

Me

Me CO2Me

Me

OH

Mulzer, J.; Schollhorn, B. Angew. Chem., Int. Ed. Eng. 1990, 29, 431-432.

TBSO Me

O Me

N3 Me

O Me

OTBS
1,3-Migration:

OTBSOH
TBAF, THF RT 90%

OH
KHMDS THF, -78 C 94%

OTBS

Bu3Sn OMe Me

Bu3Sn OMe Me

Calter, M. A. Ph. D. Thesis, Harvard University, 1993.

Me

Me O

Me

Me

Me O

Me

Me CO2Me

HO Me

O Me

N3 Me

O Me

OTBS OTBS H O
66%

OH OPiv
?

OTBS

Nakaba, T.; Fukui, M.; Oishi, T. Tetrahedron Lett. 1988, 29, 2219, 2223.

Bu3Sn OMe Me

Me Bu3Sn OTBS OPiv MgBr Me

KHMDS 94% THF, -78 C

TESO

TESO CO2H
4 AcOH:THF:H2O (8:8:1) 4 h, 20 C 76%

CO2H
4

OTBS OH

Bu3Sn OH Me
4:1

C5H11 TESO H OTBS

C5H11 HO H OTBS

1. MeOTf, t-Bu N t-Bu Bu3Sn 2. DIBAl-H, CH2Cl2, -78 C 80%

OMe Me

Hart, T. W.; Metcalfe, D. A.; Scheinmann, F. J. Chem. Soc., Chem. ommun. 1979, 156.

Calter, M. A. Ph. D. Thesis, Harvard University, 1993.

36-12-Si Protecting Grps-4 12/14/03 9:02 PM

J. Leighton, D. A. Evans
Principle Methods for Benzylation of Alcohols:
1.
R OH

ROH Protecting Groups


Priciple Methods for Deprotection:
R R OBn PMB

Chem 115

1. NaH, DMF, 0 C 2. BnBr/PMBBr


NH Ar

1. Hydrogenation
R OBn

10% Pd on C, H2 EtOH/EtOAc/AcOH

OH

2.
R OH O

CCl3

R R

OBn PMB

See Greene, p. 49.

cat. TfOH Ar = Ph: CH2Cl2 Ar = 4-MeO-Ph: Et2O

Ar = Ph:

Iversen, T.; Bundle, K. R. J. Chem. Soc., Chem. Commun. 1981, 1240.

2. Transfer Hydrogenation
R OBn

Ar = 4-MeO-Ph: Yonemitsu, O. et al., Tetrahedron Lett. 1988, 29, 4139. 3.


R OH

10% Pd on C Hydrogen Source Ref.

OH

Hydrogen Source 1. (Bu3Sn)2O, PhMe, 2. BnBr, N-methylimidazole


R OBn

Cyclohexene Cyclohexadiene HCO2H i-PrOH

Synthesis 1981, 396. J. Org. Chem. 1978, 43, 4194. J. Org. Chem. 1979, 44, 3442. Tetrahedron Lett. 1986, 27, 2497

Cruzado, C.; Bernabe, M.; Martin-Lomas, M. J. Org. Chem. 1989, 54, 465. Review: David, S.; Hanessian, S. Tetrahedron, 1985, 41, 643-663. 4.
R OH

Ag2O, BnBr DMF

OBn

3. Lewis Acids
R OBn R OH

Van Hijfte, L.; Little, R. D. J. Org. Chem. 1985, 50, 3940.

Via Benzylidene Acetal:


Ar O R1 H O

Reagents DIBAl-H CH2Cl2, 0 C


OH R1 O Ar R2

Ref. Tetrahedron Lett. 1989, 30, 5713. J. Am. Chem. Soc. 1989, 111, 1923. Chem. Pharm. Bull. 1987, 35, 3880.

BF3OEt2, EtSH 1. BCl3, -78 C to 0 C. 2. MeOH, -78 C. TMSBr, C6H5SMe

R2

Takano, S. et al., Synthesis 1986, 811-817.

36A-13 Benzylic 12/7/01 8:58 AM

J. Leighton, D. A. Evans
PMB Deprotection:
OMe O

ROH Protecting Groups


DDQ is incompatible with:
DDQ 20:1 CH2Cl2:H2O
R N C Me

Chem 115

ROH + ArCHO
Me Me

-1 e

styryl
OMe OMe O OH OPMB

extended conjugated polyenes

O H

+
O Cl CN CN O OMe R

DDQ =

-H+

Cl

H 2O
OMe O

DDQ 20:1 CH2Cl2:H2O

H O OMe

Selective Benzylation:
-1 eBr Me Br Me

H Ar

OH R

OPMB

DDQ, CH2Cl2 3 mol sieves


R

H O OH OH

NaH, BnBr DMF, -70 C 97%


OH

H OBn

Fukuzawa, A. et al. Tetrahedron Lett. 1987, 28, 4303. n

Yonemitsu, O. et al., Tetrahedron 1986, 42, 3021.


O O

Other Oxidants: NBS, Br2, CAN ((NH4)2Ce(NO3)6). Acta Chem. Scand. Ser. B, 1984, B38, 419. J. Chem. Soc., Perkin Trans. I, 1984, 2371.
HO

OH O

1. (Bu3Sn)2O, PhMe,
OH

2. BnBr, N-methylimidazole 92%

BnO

OH O

OBn

36A-14 Benzyl Protect 12/7/01 8:18 AM

Cruzado, C.; Bernabe, M.; Martin-Lomas, M. J. Org. Chem. 1989, 54, 465.

J. Leighton, D. A. Evans
Me O Me 33 OH
H H H H

ROH Protecting Groups


Me
5
1

Chem 115
OH
17

OH O
8

O
11

OH
8

OH

CO2H
1

O
H

O O 25 Me

I Me Me OH

Me Me O

Me

Me

Me

Me

20

11

Me
17 14

OH X Me Me

OR
10

O H Me
8

OH
4

CO2R'
1

Et

HO

Me Me

Rutamycin B (Streptomyces aureofaciens)


Ng, H. P. Ph. D. Thesis, Harvard University, 1993

Me

Me

Me

17

C9-C17 Subunit

C1-C8 Subunit

O Me Me 33 OH
H H H H

Ph O
10

14

H OH O
10

O
H

Me OH O 25 Me O N Me Me Bn O O O Xq Me Me Me

Spiroketal Fragment

(c-hex)2BCl, EtNMe2 Et2O, 0 C 84%

14

Ph

(Xq)

Et 20

B(OH)2

NaBH(OAc)3, AcOH 81%

OH
17

O
11

OH
8

OH

CO2H
1 14

OH Ph

OH
10

O Xq

I Me Me

Me

Me

Me

Me

Me

Me

Me

Polypropionate Fragment

36A-15 Rutamycin 12/7/01 8:18 AM

J. Leighton, D. A. Evans
OH
14

ROH Protecting Groups


HO O
1

Chem 115

OH
10

O Xq Me

Ph Me Me Me

Me 33

H H

O
H H H

Me OTES O 25 Me Me

OTBS O
8

TBSO PMBO

O
10

?
Ph

14

HFpyr pyridine, THF


H

TBSO

99%

Me O
11

OTBS

Me

Me

Me HO
1

Me

Selective silylation was unsuccessful.


Me

20

Et

TBSO

Me Me

17 5

Me 33 OH
14

H H

OH
10

O Xq

OH

OH
10

OH TBSO

O
H H

Me OH O 25
H

OTBS O
8

Ph Me Me Me

LiBH4, H2O Et2O, 0 C 94%

14

Me Me O

Ph Me OMe OMe Me Me

Me

OTBS

CSA, DMF 91%


PMP

20

11

Me

Et

TBSO

Me Me

PMP TBSO
14

MeO OH O

17

O
10

O
10

TBSOTf, 2,6-lut. CH2Cl2, 0 C 100%


Ph

14

Ph Me Me Me

1. Cl3C6H2COCl DMAP, Et3N, benzene 2. (aq.) HF, CH3CN, H2O


Me
H

Me

Me

Me

Me O O
H

Me OH O O Me O OH

DIBAl-H 100% CH2Cl2, 0 C 84%


H Me Me Me

OH

H H

Me Me O HO

TBSO PMBO
14 10

OH

TBSO PMBO

O
10

Ph Me Me Me

Swern Ox. 97%

Me Me Me

14

Ph

Et

Rutamycin B

36A-16 Rutamycin 12/7/01 8:18 AM

J. Leighton, D. A. Evans
OH
37

ROH Protecting Groups


O
33

Chem 115

Me O N H
30

MeO Me2N

TBSO N
25 26

O OMe

OH

MeO Me2N TBSO Me TESO


21 OH

No Reaction

(+) Calyculin A
O N C1 Me
8

Me

(HO)2 P O 17 Me
13

Me Me O O

(aq.) HF, CH3CN, H2O 24 h


OMe MeO Me2N

OH

O OMe OH

MeO Me2N TESO

Me

Me

OH

OH

OMe

Evans, D. A.; Gage, J. R.; Leighton, J. L. J. Am. Chem. Soc. 1992, 114, 9434-9453.

After protonation of the amine, coulombic repulsion insulates against formation of another cationic site in the vicinity.

(aq.) HF, CH3CN, H2O 92 h

70%

OTBS

N C1 Me
8

Me

Me

PMBO 17
13

Me Me O O

Me

21 OTBS

TESO
37

O
33

Me O N H
30

Me

Me

OTBS OTBS OMe

MeO Me2N

OTES

N
25

26

DDQ 20:1 CH2Cl2:H2O


OTBS

O N C1 Me
8

(PMBO)2P Me Me
13

O 17

Me Me O O

Me Me Me O O Me

21 OTBS

Me OTBS OTBS OMe PivO

Me
13

RO 17

Me

Me

21 OTBS

R = PMB R=H

DDQ 20:1 CH2Cl2:H2O

94%

OTBS OTBS OMe

36A-17 Calyculin 12/7/01 8:18 AM

J. Leighton, D. A. Evans
Me H Me OH H Me H H H O H OH

ROH Protecting Groups


Me O O O H O O O Me Me HO OH Me H Me OMe O H O OH H Me O OH H H O H MeO Me OH Me N OH

Chem 115

Rapamycin
OH

H Me O OH Me O OMe Me Me OH OMe

Cytovaricin

Evans, D. A.; Kaldor, S. W.; Jones, T. K.; Clardy, J.; Stout, T. J. J. Am. Chem. Soc. 1990, 112, 7001-7031.

1. Pd(Ph3P)4, HCO2NH4, THF HFpyr, pyridine 74% THF, RT


Et

Romo, D.; Meyer, S. D.; Johnson, D. D.; Schreiber, S. L. J. Am. Chem. Soc. 1993, 115, 7906-7907. 30% overall
O

2. Dess-Martin Periodinane
i-Pr

DEIPS =

Si Et

3. HFpyr, pyridine, THF Aloc =

O Me Me OTBS H Me DEIPSO Me TBSO O Me TESO H H O H H H H O H O O O Me Me OTES Me O Si O t-Bu Me OAloc ODEIPS Me OTES Me OMe Me Me Me OAloc OTIPS O H OMe Me OMe O H O N OAloc

TESO

t-Bu H H MeO

36A-18 Cytovaricin 12/7/01 8:18 AM

J. Leighton, D. A. Evans

ROH Protecting Groups


O O H 2N OH O Me OH OH OH OH OH OH O OH HO OH OH

Chem 115

1. DDQ, t-BuOH-CH2Cl2; Ac2O, DMAP, pyr. 2. aq. HClO4, THF, 8 days 3. LiOH, H2O/MeOH/THF, RT, 20 h 4. TBAF, THF/DMF, RT, 90 h 5. AcOH, H2O, RT, 36 h 35%

OH

HO

OH OH

OH O HO OH Me Me O OH Me OH Me HO HO OH OH OH HO O Me HO OH O OH OH

~97.5% per protecting group

O HO

OH O Me OH O OH

OH OH

O O H N O Me3Si(CH2)2O TBSO OTBS OTBS O Me OTBS OAc OTBS OTBS OPMB O MeO OAc Me Me O OAc Me Me PMBO OTBS
PMBO

HO O O O TBSO Me Me TBSO TBSO OTBS OTBS

HO OH

OH

OH

OH

Palytoxin Carboxylic Acid


Kishi, Y. et al., J. Am. Chem. Soc. 1989, 111, 7525, 7530.

OTBS

OPMB OPMB OPMB

OTBS OTBS

O PMBO

TBSO O Me TBSO OTBS

OPMB O Me OPMB O BzO BzO OMe

OTBS OTBS

OAc OBz OBz

OAc

36A-19 Palytoxin 12/7/01 8:19 AM

You might also like